Sei sulla pagina 1di 489

McGraw-Hill

SPECIALTY BOARD REVIEW

Neurology
Second Edition

Nizar Souayah, MD
Assistant Professor of Neurology
Program Director
University of Medicine and Dentistry of New Jersey
Director, EMG Laboratory and Peripheral Neuropathy Center
Newark, New Jersey

New York Chicago San Francisco Lisbon London Madrid Mexico City Milan
New Delhi San Juan Seoul Singapore Sydney Toronto
Copyright 2010 by The McGraw-Hill Companies, Inc. All rights reserved. Except as permitted under the United States Copyright Act of 1976, no part of
this publication may be reproduced or distributed in any form or by any means, or stored in a database or retrieval system, without the prior written permission
of the publisher.

ISBN: 978-0-07-154966-0

MHID: 0-07-154966-8

The material in this eBook also appears in the print version of this title: ISBN: 978-0-07-154965-3, MHID: 0-07-154965-X.

All trademarks are trademarks of their respective owners. Rather than put a trademark symbol after every occurrence of a trademarked name, we use names in
an editorial fashion only, and to the benefit of the trademark owner, with no intention of infringement of the trademark. Where such designations appear in this
book, they have been printed with initial caps.

McGraw-Hill eBooks are available at special quantity discounts to use as premiums and sales promotions, or for use in corporate training programs. To contact
a representative please e-mail us at bulksales@mcgraw-hill.com.

Medicine is an ever-changing science. As new research and clinical experience broaden our knowledge, changes in treatment and drug therapy are required.
The authors and the publisher of this work have checked with sources believed to be reliable in their efforts to provide information that is complete and
generally in accord with the standards accepted at the time of publication. However, in view of the possibility of human error or changes in medical sciences,
neither the authors nor the publisher nor any other party who has been involved in the preparation or publication of this work warrants that the information
contained herein is in every respect accurate or complete, and they disclaim all responsibility for any errors or omissions or for the results obtained from use of
the information contained in this work. Readers are encouraged to confirm the information contained herein with other sources. For example and in particular,
readers are advised to check the product information sheet included in the package of each drug they plan to administer to be certain that the information
contained in this work is accurate and that changes have not been made in the recommended dose or in the contraindications for administration. This
recommendation is of particular importance in connection with new or infrequently used drugs.

TERMS OF USE

This is a copyrighted work and The McGraw-Hill Companies, Inc. (McGrawHill) and its licensors reserve all rights in and to the work. Use of this work is
subject to these terms. Except as permitted under the Copyright Act of 1976 and the right to store and retrieve one copy of the work, you may not decompile,
disassemble, reverse engineer, reproduce, modify, create derivative works based upon, transmit, distribute, disseminate, sell, publish or sublicense the work or
any part of it without McGraw-Hills prior consent. You may use the work for your own noncommercial and personal use; any other use of the work is strictly
prohibited. Your right to use the work may be terminated if you fail to comply with these terms.

THE WORK IS PROVIDED AS IS. McGRAW-HILL AND ITS LICENSORS MAKE NO GUARANTEES OR WARRANTIES AS TO THE ACCURACY,
ADEQUACY OR COMPLETENESS OF OR RESULTS TO BE OBTAINED FROM USING THE WORK, INCLUDING ANY INFORMATION THAT
CAN BE ACCESSED THROUGH THE WORK VIA HYPERLINK OR OTHERWISE, AND EXPRESSLY DISCLAIM ANY WARRANTY, EXPRESS OR
IMPLIED, INCLUDING BUT NOT LIMITED TO IMPLIED WARRANTIES OF MERCHANTABILITY OR FITNESS FOR A PARTICULAR PURPOSE.
McGraw-Hill and its licensors do not warrant or guarantee that the functions contained in the work will meet your requirements or that its operation will be
uninterrupted or error free. Neither McGraw-Hill nor its licensors shall be liable to you or anyone else for any inaccuracy, error or omission, regardless of cause,
in the work or for any damages resulting therefrom. McGraw-Hill has no responsibility for the content of any information accessed through the work. Under no
circumstances shall McGraw-Hill and/or its licensors be liable for any indirect, incidental, special, punitive, consequential or similar damages that result from
the use of or inability to use the work, even if any of them has been advised of the possibility of such damages. This limitation of liability shall apply to any
claim or cause whatsoever whether such claim or cause arises in contract, tort or otherwise.
Contents

Preface .......................................................................................................................................................................v

1. Anatomy and Physiology of the Central and Peripheral Nervous System..........................................1


Questions ............................................................................................................................................................1
Answers and Explanations ............................................................................................................................22

2. Localization Signs in Neurology.................................................................................................................45


Questions ..........................................................................................................................................................45
Answers and Explanations ............................................................................................................................67

3. Pediatrics ..........................................................................................................................................................99
Questions ..........................................................................................................................................................99
Answers and Explanations ..........................................................................................................................115

4. Neurophysiology, Epilepsy, Evoked Potentials, and Sleep Disorders ..............................................151


Questions ........................................................................................................................................................151
Answers and Explanations ..........................................................................................................................177

5. Neuromuscular Diseases ............................................................................................................................199


Questions ........................................................................................................................................................199
Answers and Explanations ..........................................................................................................................215

6. Behavioral Neurology..................................................................................................................................243
Questions ........................................................................................................................................................243
Answers and Explanations ..........................................................................................................................250

7. Cerebrovascular Diseases ...........................................................................................................................265


Questions ........................................................................................................................................................265
Answers and Explanations ..........................................................................................................................271

8. Infections of the Nervous System.............................................................................................................283


Questions ........................................................................................................................................................283
Answers and Explanations ..........................................................................................................................290

9. Neuroimmunology.......................................................................................................................................301
Questions ........................................................................................................................................................301
Answers and Explanations ..........................................................................................................................308

iii
iv Contents

10. Neuropharmacology and Neurochemistry..............................................................................................323


Questions ........................................................................................................................................................323
Answers and Explanations ..........................................................................................................................331

11. Neurogenetics ..............................................................................................................................................341


Questions ........................................................................................................................................................341
Answers and Explanations ..........................................................................................................................348

12. Neuroophthalmology ..................................................................................................................................361


Questions ........................................................................................................................................................361
Answers and Explanations ..........................................................................................................................366

13. Neurooncology..............................................................................................................................................373
Questions ........................................................................................................................................................373
Answers and Explanations ..........................................................................................................................377

14. Movement Disorders ...................................................................................................................................385


Questions ........................................................................................................................................................385
Answers and Explanations ..........................................................................................................................391

15. Neuropathology ............................................................................................................................................401


Questions ........................................................................................................................................................401
Answers and Explanations ..........................................................................................................................417

16. Neuroradiology.............................................................................................................................................429
Questions ........................................................................................................................................................429
Answers and Explanations ..........................................................................................................................442

17. Psychiatry.......................................................................................................................................................447
Questions ........................................................................................................................................................447
Answers and Explanations ..........................................................................................................................456

Index ......................................................................................................................................................................469
Preface

My experience in teaching medical students, neurology The book is divided into 17 chapters, including
residents, and fellows has been that they love multiple- one devoted to psychiatry and another to localization
choice questions combined with didactic teaching. For signs, thus providing comprehensive coverage of the
that reason I developed several board-style questions at neurology board topics. The content represents
the end of most of my recent lectures, encouraging my almost a total rewrite of the previous edition, with
listeners to memorize the provided information. new questions in the new neurology board format as
The idea of this review initially arose when I was well as numerous illustrations, including color illus-
studying for the neurology board myself. I was sur- trations, and updated answers using the current
prised by the lack of review books in a question-and- literature for references to reflect the most recent
answer format to help me assess my progress and advances.
identify areas of weakness. The few neurology reviews The best way to go through this book is to follow
that did have multiple-choice questions were intended these steps:
for medical students preparing for the USMLE, but
First read the question without reading the answer
they lacked the deep, broad coverage needed for the
choices (A to E) and guess the answer based on the
neurology board exam.
information offered by the question.
Additional impetus for this book came from my
Then read the answer choices (A to E); these may
years of teaching. Many of my students, residents, fel-
confirm or refute the answers you have already
lows, and even colleagues asked me whether there was
developed. If your first answer proves wrong,
a book in question-and-answer format that could be
choose another from the remaining choices.
used as a tool in preparing for neurology board certifi-
Go to the answer section of the chapter, check if your
cation/recertification. The format of the present work
answer is correct, and read the answer.
simulates the board exam and serves as an excellent
tool for identifying areas of strength and weakness and The answers offer a comprehensive review of the
sharpening knowledge that has already been acquired. different question choices. Whether or not you
In preparing a second edition of this book, I was answered the question correctly, I strongly advise you
driven not only by recent changes in the question for- to go in depth through the incorrect choices to find out
mat of the neurology board exam and the steady why they are incorrect by consulting the answer refer-
advances in nearly all aspects of neurology over the past ences or other references. They will enlarge the knowl-
five years but also by the response my work had edge provided by the question and help to identify any
received from its readers. I am grateful for their positive hidden weakness in your knowledge base.
comments. As I reviewed the first edition in preparation I am indebted to my family, who allowed me the
for the second, I recognized clearly all the very signifi- luxury of time to produce this second edition and who
cant and exciting changes had recently occurred in the carried most of my personal daily workload with
neurology and neurosciences fields. understanding.
This book is not designed to substitute for the I am also grateful for the assistance of the editorial
didactic lectures, seminars, and conferences offered and publishing staff of McGraw-Hill throughout the
during residency or fellowship training but rather to preparation of this second edition.
augment residents experimental learning, reinforce I hope you have fun studying for the neurology
their self-assessment and growth, and better prepare board via the multiple-choice question-and-answer
them for the certification examination of the American study approach.
Board of Psychiatry and Neurology. Nizar Souayah, MD

v
To my son, Sami: let your dreams be your guide through this life.
To my daughters, Leila and Nora: keep your dreams alive.
Achievement requires faith, hard work, determination, and persistence.
To my wife, Sonia: without your help and sacrifice this would not have been possible.
To my parents: for your love, great affection, inspiration, and patience.
CHAPTER 1

Anatomy and Physiology of the Central


and Peripheral Nervous System
Questions

1. Which of the following sensory pathways proj- (C) bitemporal hemianopia


ects outside the thalamus? (D) superior quadrantic hemianopia
(A) The visual pathway (E) inferior quadrantic hemianopia
(B) The auditory pathway
3. All fibers in the posterior funiculus are
(C) The vibration sense pathway
(D) The olfactory pathway (A) primary afferent fibers
(E) The temperature sense pathway (B) fibers that establish synapses in lamina
III of the cervical enlargement
2. Damage to the area indicated by the arrow in (C) unimodal afferents
Figure 1-1 causes (D) ascending fibers
(A) ipsilateral blindness (E) activated by nociceptors
(B) homonymous hemianopia

FIG. 1-1. (Reproduced with permission from Afifi AK, Bergman RA. Functional Neuroanatomy: Text and
Atlas. 2nd ed. New York: McGraw-Hill; 2005.)

1
2 1: Anatomy and Physiology of the Central and Peripheral Nervous System

4. Which of the following pathways is used for 9. The rhomboid muscles are innervated by the
propioception of the hindlimb but not for
(A) long thoracic nerve
vibration of the hindlimb?
(B) suprascapular nerve
(A) The dorsal spinocerebellar tract (C) axillary nerve
(B) The spinocervical tract (D) dorsal scapular nerve
(C) The ventral spinocerebellar tract (E) subclavian nerve
(D) The lateral spinothalamic tract
(E) The rubrospinal tract 10. To differentiate between a lesion of the upper
brachial plexus and one of the lateral cord, the
5. The long thoracic nerve innervates the most useful muscle to test is the
(A) serratus anterior muscle (A) teres minor
(B) rhomboid muscle (B) biceps
(C) levator scapulae (C) pronator teres
(D) supraspinatus muscle (D) flexor carpi radialis
(E) infraspinatus muscle (E) abductor pollicis brevis

6. The axillary nerve 11. To differentiate between a lesion of the lower


trunk and one of the medial cord, the most
(A) arises from C7C8
useful muscle to test is the
(B) innervates the deltoid muscle by its infe-
rior branch (A) flexor pollicis longus
(C) is a pure motor nerve (B) abductor pollicis brevis
(D) is one whose injury may lead to weak- (C) abductor digiti minimi
ness of arm adduction (D) extensor indicis proprius
(E) may be affected in isolation in 10% of (E) first dorsal interosseous
cases in neuralgic amyotrophy
12. To differentiate between lesions of the middle
7. A musculocutaneous nerve lesion affects trunk and those of the posterior cord, the most
useful muscle to test is the
(A) hand sensation
(B) supination with the forearm extended (A) triceps
(C) supination with the elbow in flexion (B) pronator teres
(D) wrist extension (C) anconeus
(E) upper arm abduction (D) extensor digitorum communis
(E) extensor indicis proprius
8. Which of the following is a compression site of
the radial nerve? 13. The lateral division of the sciatic nerve inner-
vates the
(A) The suprascapular notch
(B) The carpal tunnel (A) semimembranosus
(C) The spinoglenoid notch (B) long head of the biceps femoris
(D) The elbow posterior to the medial epi- (C) semitendinosus
condyle (D) short head of the biceps femoris
(E) The spiral groove in the posterior aspect (E) adductor magnus muscle
of the humerus
Questions: 421 3

14. In carpal tunnel syndrome, the median nerve is (D) The interstitial nucleus of the medial
entrapped longitudinal fasciculus
(A) beneath the flexor retinaculum ligament (E) The fastigial nucleus
(B) above the flexor retinaculum ligament
18. Information regarding eye position reaches the
(C) at the hamate bone abducens nucleus from neuronal integrator
(D) in Guyons canal neurons coming from the
(E) on the radial side of the wrist at the level
(A) rostral interstitial nucleus of the medial
of the styloid process
longitudinal fasciculus
15. The sartorius muscle is innervated by the (B) interstitial nucleus of Cajal
(C) parapontine reticular formation
(A) obturator nerve
(D) red nucleus
(B) femoral nerve
(E) nucleus prepositus hypoglossi
(C) genitofemoral nerve
(D) superior gluteal nerve 19. The gaze-holding neural integrator for verti-
(E) inferior gluteal nerve cal gaze is located in the
(A) interstitial nucleus of Cajal
16. The structure indicated by the broken arrow
in Figure 1-2 is the (B) medial vestibular nucleus
(C) nucleus prepositus hypoglossi
(A) hypoglossal nucleus
(D) red nucleus
(B) nucleus ambiguus
(E) rostral interstitial nucleus of the medial
(C) medial longitudinal fasciculus longitudinal fasciculus
(D) facial nucleus
(E) nucleus solitarius 20. Which of the following cranial nerves exits the
brainstem from its dorsal aspect?
(A) The oculomotor nerve
(B) The facial nerve
(C) The trigeminal nerve
(D) The glossopharyngeal nerve
(E) The trochlear nerve

21. The perihypoglossal nuclei are involved in


(A) extraocular movement
(B) proprioceptive innervations of the
tongue
(C) motor innervation of the tongue
FIG. 1-2. (Reproduced with permission from Afifi AK, Bergman RA. Functional
(D) pain sensation of the posterior third of
Neuroanatomy: Text and Atlas. 2nd ed. New York: McGraw-Hill; 2005.) the tongue
(E) pharyngeal gag reflex
17. Which of the following structures is involved in
vertical saccades?
(A) The parietal cortex
(B) The cerebellar vermis
(C) The caudate nuclei
4 1: Anatomy and Physiology of the Central and Peripheral Nervous System

TEGMENTUM

BASIS PONTIS

FIG. 1-3. (Reproduced with permission from Afifi AK, Bergman RA. Functional
Neuroanatomy: Text and Atlas. 2nd ed. New York: McGraw-Hill; 2005.)

22. The nucleus ambiguus 25. Damage to the structure indicated by the
broken arrow in Figure 1-3 causes
(A) is the dorsal motor nucleus of the vagus
(B) receives general somatic afferent fibers (A) ipsilateral hemiplegia
from the external ear (B) anosognosia
(C) has neurons that convey special visceral (C) internuclear ophthalmoplegia
efferent impulses to the pharynx (D) ipsilateral facial weakness
(D) is involved in epiglottic taste sensation (E) impaired salivary secretion
(E) receives fibers that convey thermal sen-
sation from the posterior third of the 26. The semilunar ganglion conveys
tongue
(A) proprioceptive fibers of the trigeminal
nerve
23. Cardiorespiratory control is partially conveyed
by the (B) pain sensation from the face
(C) the jaw reflex
(A) nucleus ambiguus
(D) the corneal reflex
(B) nucleus solitarius
(E) taste in the anterior two thirds of the
(C) pontine reticular nuclei tongue
(D) restiform body
(E) spinal trigeminal nucleus 27. The structure indicated by the arrow in Fig-
ure 1-4 is
24. Taste sensation is conveyed by
(A) the trigeminal nerve
(A) the facial nerve, the cochleovestibular (B) the facial nerve
nerve, and the glossopharyngeal nerve (C) the abducens nerve
(B) the cochleovestibular nerve, the glos- (D) the trochlear nerve
sopharyngeal nerve, and the vagus
(E) the oculomotor nerve
nerve
(C) the facial nerve, the glossopharyngeal 28. Proprioceptive impulses from the muscles of
nerve, and the vagus nerve mastication and the periodontal memebrane
(D) the glossopharyngeal nerve, the vagus are conveyed by the
nerve, and the spinal accessory nerve
(A) mesencephalic nucleus of the trigeminal
(E) the facial nerve, the spinal accessory
nerve
nerve, and the hypoglossal nerve
Questions: 2232 5

30. Noradrenergic innervations to most of the


nervous system is provided by the
(A) substantia nigra
(B) locus ceruleus
(C) dorsal raphe nucleus
(D) red nucleus
(E) ventral tegmental nucleus

31. The structure indicated by the arrow in Figure


1-6 is the
(A) interstitial nucleus of Cajal
(B) nucleus of Darkschewitsch
FIG. 1-4. (Reproduced with permission from Afifi AK, Bergman RA. Functional
Neuroanatomy: Text and Atlas. 2nd ed. New York: McGraw-Hill; 2005.) (C) nucleus of the posterior commissure
(D) oculomotor nucleus
(E) red nucleus
(B) interpeduncular nucleus
(C) dorsal tegmental ncleus
(D) rubrospinal tract
(E) medial lemniscus

29. The structure indicated by the arrow in Fig-


ure 1-5
(A) conveys fibers from the red nucleus to
the spinal cord
(B) conveys auditory fibers
FIG. 1-6. (Reproduced with permission from Afifi AK, Bergman RA. Functional
(C) conveys kinesthetic fibers Neuroanatomy: Text and Atlas. 2nd ed. New York: McGraw-Hill; 2005.)
(D) conveys fibers of pain and temperature
sensation
32. The Marcus Gunn phenomenon
(E) conveys fibers supplying the superior
oblique muscle (A) is caused by a lesion of the oculomotor
nerve
(B) is characterized by prolonged pupillary
contraction in response to light
(C) is characterized by a paradoxical dilata-
tion of the affected pupil that occurs
when light is shone in the symptomatic
eye after having been shone in the
normal eye
(D) preserves the consensual light reflex
(E) is associated with damage to the ciliary
ganglion

FIG. 1-5. (Reproduced with permission from Afifi AK, Bergman RA. Functional
Neuroanatomy: Text and Atlas. 2nd ed. New York: McGraw-Hill; 2005.)
6 1: Anatomy and Physiology of the Central and Peripheral Nervous System

33. A waddling gait is seen with 38. Which of the following neurological structures
travels outside the cavernous sinus?
(A) a cerebellar lesion
(B) an acute vestibular lesion (A) The sympathetic carotid plexus
(C) Guillain-Barr syndrome (B) The oculomotor nerve
(D) hip muscles weakness (C) The mandibular branch of the trigeminal
(E) a corticospinal tract lesion nerve
(D) The trochlear nerve
34. The anterior interosseous nerve innervates the (E) The abducens nerve
(A) pronator teres and pronator quadratus
39. Which of the following is TRUE about the
(B) flexor digitorum profundus I and II and trigeminal nerve?
flexor pollicis longus
(C) flexor pollicis longus and abductor (A) The spinal nucleus of the trigeminal
pollicis brevis nerve subserves light touch on the ipsi-
lateral side of the face.
(D) first dorsal interosseous and abductor
digiti minimi (B) The motor nucleus of the trigeminal
nerve lies in the pons medial to the sen-
(E) flexor carpi radialis and pronator
sory nucleus and sends axons to the max-
quadratus
illary division of the trigeminal nerve.
35. The posterior interosseous nerve innervates the (C) The three divisions of the trigeminal
nerve converge at the Gasserian ganglion.
(A) supinator muscle (D) The mesencephalic nucleus of the trigem-
(B) triceps muscles inal nerve subserves pain and tempera-
(C) anconeus muscle ture on the ipsilateral side of the face.
(D) extensor carpi radialis longus (E) The mandibular division of the trigemi-
(E) brachioradialis nal nerve subserves sensation to the ipsi-
lateral angle of the mandible.
36. The tensor fasciae latae is innervated by the
40. Which of the following pairs of cranial nerves
(A) superior gluteal nerve travel through the internal auditory canal?
(B) inferior gluteal nerve
(A) Vestibulocochlear and trigeminal
(C) sciatic nerve
(B) Facial and trigeminal
(D) obturator nerve
(C) Facial and optic
(E) femoral nerve
(D) Facial and vestibulocochlear
37. The deep peroneal nerve supplies sensory (E) Vestibulocochlear and vagus
innervation to
41. The taste fibers of the anterior two thirds of
(A) the lateral distal lower leg the tongue have their neurons of origin in the
(B) the dorsum of the foot
(A) sensory nucleus of the trigeminal nerve
(C) the skin in the web between the first and
second toes (B) motor facial nucleus
(D) the lateral heel (C) superior salivary nucleus
(E) no part of the body because it is a pure (D) inferior salivary nucleus
motor nerve (E) geniculate ganglion
Questions: 3348 7

42. A 40-year-old man developed chronic pain in (D) cerebral cortex S striatum S external
the right forearm that lasted for hours each day. globus pallidus S substantia nigra pars
Neurological examination demonstrated compacta S dorsal thalamus S cerebral
normal sensation, mild weakness on right fore- cortex
arm pronation, and weak flexion of the termi- (E) cerebral cortex S striatum S internal
nal phalanges of the right thumb as well as the globus pallidus S subthalamic nucleus S
index and middle fingers. An attempt to make internal globus pallidus S dorsal
a full circle by applying the end phalanx of the thalamus S cerebral cortex
thumb to that of the index finger with firm
pressure showed consistent weakness. Which 46. The indirect circuit loop between the basal gan-
of the following structures is affected? glia and the cerebral cortex is
(A) The right anterior interosseous nerve (A) cortical fibers S striatum S external
(B) The right median nerve at the upper axilla globus pallidus S subthalamic
(C) The right ulnar nerve nucleus S internal globus pallidus S
(D) The right radial nerve dorsal thalamus S cerebral cortex
(E) The right musculocutaneous nerve (B) cortical fibers S striatum S external
globus pallidus S internal globus pallidus
43. The substantia gelatinosa of the spinal cord is S dorsal thalamus S cerebral cortex
located in (C) cortical fibers S striatum S external
globus pallidus S substantia nigra pars
(A) lamina I reticulata S dorsal thalamus S cerebral
(B) lamina II cortex
(C) lamina IV (D) cortical fibers S striatum S external
(D) lamina VII globus pallidus S subthalamic nucleus
(E) lamina IX S substantia nigra pars compacta S
dorsal thalamus S cerebral cortex
44. The neostriatum is formed by the (E) cortical fibers S striatum S external
globus pallidus S subthalamic nucleus S
(A) caudate nucleus and globus pallidus
internal globus pallidus S substantia
(B) putamen and globus pallidus
nigra pars compacta S dorsal
(C) substantia nigra and olfactory tubercles thalamus S cerebral cortex
(D) caudate nucleus and putamen
(E) caudate nucleus and subthalamic 47. The superior cerebellar peduncle contains one
nucleus cerebellar afferent, which is the
(A) ventral spinocerebellar tract
45. The direct circuit loop between the basal gan-
glia and the cortex is (B) pontocerebellar tract
(C) dorsal spinocerebellar tract
(A) cerebral cortex S striatum S internal
(D) olivocerebellar fibers
globus pallidus S substantia nigra pars
(E) reticulocerebellar fibers
reticulata S dorsal thalamus S cerebral
cortex
48. The dentate nucleus sends efferent projections
(B) cerebral cortex S striatum S external
to the
globus pallidus S substantia nigra pars
reticulata S dorsal thalamus S cerebral (A) red nucleus
cortex (B) ventrolateral nucleus of the thalamus
(C) cerebral cortex S striatum S internal (C) vestibular nucleus
globus pallidus S dorsal thalamus S (D) pontine reticular nucleus
cerebral cortex (E) oculomotor nucleus
8 1: Anatomy and Physiology of the Central and Peripheral Nervous System

49. The cerebellar cortex contains (C) fornix


(A) pyramidal cells (D) paraventricular nucleus
(B) Purkinje cells (E) supraoptic nucleus
(C) fusiform cells
55. The most likely neurotransmitter for cerebellar
(D) horizontal cells of Cajal climbing fibers is
(E) hair cells
(A) acetylcholine
50. The source of noradrenergic projection to the (B) glutamate
cerebellum is the (C) aspartate
(A) dorsomedial nucleus of the hypothalamus (D) dopamine
(B) locus ceruleus (E) glycine
(C) raphe nucleus
56. The only efferent fibers from the cerebellar
(D) thalamus cortex come from the
(E) inferior olivary nucleus
(A) axons of Purkinje cells
51. The emboliform nucleus sends efferent projec- (B) mossy fiber projections
tions to the (C) parallel fibers
(A) nucleus ambiguus (D) climbing fibers
(B) nucleus solitarius (E) axons of Golgi cells
(C) vestibular nucleus
57. Hyperphagia is caused by a lesion in the
(D) pontine reticular nucleus
(E) red nucleus (A) ventromedial nucleus
(B) supraoptic nucleus
52. The learning of complex motor tasks and motor (C) anterior nucleus
plasticity are two functions that mainly involve (D) arcuate nucleus
the (E) mammillary nucleus
(A) olivocerebellar climbing fibers
(B) mossy fibers 58. Which of the following extraocular muscles is
innervated by a nucleus located on the con-
(C) emboliform nuclei
tralateral side?
(D) motor cortical area
(E) parallel fibers from granular cell axons (A) Superior rectus
(B) Inferior rectus
53. The fastigial nucleus sends efferent projections (C) Medial rectus
to the (D) Lateral rectus
(A) nucleus ambiguus (E) Inferior oblique
(B) nucleus solitarius
59. Which of the following cranial nerve nuclei are
(C) vestibular nucleus
sources of special visceral efferents?
(D) superior olive
(E) locus ceruleus (A) Oculomotor nuclei
(B) Trochlear nuclei
54. The tuberoinfundibular hypothalamic tract (C) Abducens nuclei
arises from the (D) Motor nuclei of the facial nerve
(A) arcuate nuclei (E) Hypoglossal nuclei
(B) mammillary nuclei
Questions: 4969 9

60. The parasympathetic innervation of the parotid (D) IV


gland is provided by the (E) V
(A) facial nerve
65. The extrathalamic cortical modulatory system,
(B) vestibulocochlear nerve
using acetylcholine as a neurotransmitter,
(C) glossopharyngeal nerve arises from the
(D) vagus nerve
(A) midbrain raphe
(E) spinal accessory nerve
(B) locus ceruleus
61. Which of the following is true about the trigem- (C) ventral midbrain
inal nerve nuclei? (D) nucleus basalis of Meynert of the basal
forebrain
(A) The trigeminal nerve has two sensory
nuclei. (E) hypothalamic nuclei
(B) Pain and temperature are carried pre-
66. Prosopagnosia is associated with
dominantly by the spinal nucleus of the
trigeminal nerve. (A) a lesion of the posterior parietal associa-
(C) Most small fibers afferent of the spinal tion cortex
tract of the trigeminal nerve end in the (B) a lesion of the temporal association cortex
main sensory nucleus of that nerve. (C) a lesion of the prefrontal association cortex
(D) The motor nucleus of the trigeminal (D) labile emotion
nerve innervates the muscles of mastica- (E) a problem-solving deficit
tion via its maxillary division.
(E) The motor nucleus of the trigeminal 67. Which of the following arteries supplies the
nerve contains only alpha motor neurons. medial part of the lateral geniculate body?

62. The sole output neurons of the cerebellar cortex (A) The ophthalmic artery
are (B) The anterior communicating artery
(C) The anterior choroidal artery
(A) pyramidal cells
(D) The posterior choroidal artery
(B) granular cells
(E) The middle cerebral artery
(C) Purkinje cells
(D) horizontal cells 68. Which of the following cranial nerves is
(E) fusiform cells responsible for eye closure?

63. The principal efferent neuron layer of the cere- (A) The oculomotor nerve
bral neocortex is (B) The trochlear nerve
(C) The abducens nerve
(A) layer II
(D) The facial nerve
(B) layer III
(E) The spinal accessory nerve
(C) layer IV
(D) layer V 69. Which of the following structures receives
(E) layer VI afferents responsible for taste sensation in the
anterior two thirds of the tongue?
64. Thalamocortical afferents have their main ter-
minals in the cerebral cortex layer number (A) The submaxillary ganglion
(B) The pterygopalatine ganglion
(A) I
(C) The superior salivary nucleus
(B) II
(D) The geniculate ganglion
(C) III
(E) The submandibular ganglion
10 1: Anatomy and Physiology of the Central and Peripheral Nervous System

70. Which of the following arteries supplies the trical signal generation in the central
intracranial part of the facial nerve? nervous system.
(A) The middle meningeal artery (B) Sodium channel clustering is located
within the juxtaparanodal axonal region.
(B) The superior cerebellar artery
(C) Sodium channel clustering is initiated by
(C) The posteroinferior cerebellar artery
Schwann cells in the peripheral nervous
(D) The anteroinferior cerebellar artery system.
(E) The posterior auricular artery (D) Shaker-type potassium channels are clus-
tered in the node of Ranvier and may
71. The third-order neurons of the auditory path- serve to inhibit sodium channel clustering.
way terminate at the
(E) Axonal sodium channel expression
(A) inferior colliculus decreases in multiple sclerosis.
(B) auditory radiation
(C) medial geniculate body 76. Which of the following proteins is found in
noncompact myelin of the peripheral nervous
(D) lateral geniculate body
system?
(E) dorsal portion of the cochlear nucleus
(A) Connexin 32
72. The glossopharyngeal nerve crosses the jugular (B) Myelin basic protein
foramen with the (C) Proteolipid protein (PLP)
(A) facial and vestibulocochlear nerves (D) Peripheral myelin protein 22 (PMP22)
(B)vestibulocochlear and vagus nerves (E) Myelin-oligodendrocyte-specific protein
(C) vagus and spinal accessory nerves
77. Schwann cells
(D) vestibulocochlear and spinal accessory
nerves (A) ensheathe multiple axons
(E) facial and spinal accessory nerves (B) make multiple myelin sheaths for differ-
ent axons
73. Glomus jugulare tumors may cause (C) express myelin-oligodendrocyte glyco-
(A) ipsilateral trapezoid weakness protein
(B) vertigo (D) do not have a basal lamina
(C) diplopia (E) are responsible for myelin synthesis in
the central nervous system
(D) ipsilateral tongue deviation
(E) blepharospasm 78. Which of the following characteristics is
common to astrocytes and oligodendrocytes?
74. Which of the following arteries supplies the
midbrain? (A) Class I major histocompatibility complex
(MHC) expression
(A) The vertebral artery
(B) Class II MCH expression
(B) The superior cerebellar artery
(C) Expression of costimulatory molecules
(C) The anteroinferior cerebellar artery
(D) Expression of complement components
(D) The posteroinferior cerebellar artery
(E) Expression of myelin basic protein
(E) The recurrent artery of Heubner
79. The resting membrane potential in a typical
75. Which of the following is true about ion channel neuron is
sequestration in central nervous system axons?
(A) 50 mV
(A) Sodium and potassium channel cluster-
(B) 70 mV
ing causes an inhibition of proper elec-
Questions: 7089 11

(C) 80 mV 85. Which of the following brain structures derives


(D) 30 mV from the telencephalon?
(E) 40 Mv (A) The internal capsule
(B) The cerebral aqueduct of Sylvius
80. Which of the following types of axons has the (C) The thalamus
fastest conduction velocity?
(D) The fourth ventricle
(A) Ia (E) The third ventricle
(B) Ib
(C) II 86. The diencephalon develops into the
(D) III (A) cerebral cortex
(E) IV (B) fourth ventricle
(C) cerebral aqueduct of Sylvius
81. Neurulation does not occur when the embryo
(D) olfactory bulb
is exposed to colchicine because
(E ) thalamic nuclei
(A) it inhibits induction
(B)it inhibits anterior neuropore closure 87. Defects in the closure of the posterior neuro-
(C) it inhibits posterior neuropore closure pore cause
(D) it induces microfilament-based contrac- (A) spina bifida
tion (B) anencephaly
(E) it induces the depolymerization of (C) tethered cord syndrome
microtubules
(D) holoprosencephaly
82. After fertilization, the anterior neuropore closes (E) lissencephaly
at
88. Which of the following cells derives from the
(A) 14 days neural crest?
(B) 20 days
(A) Radial glial cells
(C) 18 days
(B) Neuroblasts
(D) 24 days
(C) Schwann cells
(E) 28 days
(D) Purkinje cells
83. Failure of the anterior neuropore to close causes (E) Astrocytes

(A) anencephaly 89. Which of the following structures derives from


(B) spina bifida the basal plate?
(C) meningocele (A) General somatic efferent
(D) meningomyelocele (B) General visceral afferent
(E) tethered cord syndrome (C) General somatic afferent
(D) Special visceral afferent
84. Failure of the forebrain to undergo cleavage
results in (E) Special somatic afferent

(A) anencephaly
(B) holoprosencephaly
(C) myelodysplasia
(D) meningoencephalocele
(E) spina bifida
12 1: Anatomy and Physiology of the Central and Peripheral Nervous System

90. Which of the following structures sends affer- 96. Heterotopia is caused by
ents to the mammillary body?
(A) failure of secondary neurulation
(A) The medial temporal cortex (B) disrupted migration of immature neu-
(B) The retinal pregeniculate nucleus rons
(C) The nucleus of locus coeruleus (C) abnormal migration of neural crest cells
(D) The arcuate nucleus (D) failure of the posterior neuropore to
(E) The supraoptic nucleus close
(E) failure of the anterior neuropore to close
91. The anterior thalamic nuclei receive afferent
connections from the 97. Which of the following structures pass through
the jugular foramina?
(A) lateral geniculate body
(B) basal ganglia (A) The facial nerve
(C) spinal cord (B) The cochleovestibular nerve
(D) mammillary body (C) The hypoglossal nerve
(E) cerebellum (D) The vagus nerve
(E) The trigeminal nerve
92. The dorsomedial nucleus of the thalamus
receives afferent connections from the 98. Which of the following arteries supply the thal-
amus?
(A) frontal lobe
(B) occipital lobe (A) The middle cerebral artery
(C) fornix (B) The anterior choroidal arteries
(D) mammillary body (C) The posterior choroidal artery
(E) spinal cord (D) The anterior cerebral artery
(E) The superior cerebellar artery
93. The medial geniculate body sends efferents to the
(A) calcarine cortex 99. Which of the following structures is a part of
the Papez circuit?
(B) temporal gyrus of Heschl
(C) cerebellum (A) The medial lemniscus
(D) mammillary body (B) The nucleus ambiguous
(E) spinal cord (C) The hypothalamus
(D) The hippocampus
94. Anencephaly results from (E) The lateral geniculate body
(A) a defect of prosencephalization
100. The fibers of the medial lemniscus terminate in
(B) failure of the anterior neuropore to close
a somatotropic manner within the
(C) failure of the posterior neuropore to close
(D) failure of secondary neurulation (A) ventral posterolateral nucleus
(E) defective development of the neural crest (B) ventral posteromedian nucleus
(C) lateral dorsal nucleus
95. Secondary neurulation defects cause (D) midline nuclei
(A) tethered cord syndrome (E) reticular nucleus
(B) anencephaly
101. Which of the following hypothalamic nuclei is
(C) prosencephaly
responsible for body temperature control?
(D) congenital hydrocephalus
(E) DandyWalker malformation
Questions: 90110 13

(A) The supraoptic nucleus (C) Rexed laminae I and II


(B) The paraventricular nucleus (D) Rexed laminae I and V
(C) The suprachiasmatic nucleus (E) Rexed laminae IV and V
(D) The preoptic nucleus
(E) The anterior nucleus 107. Which of the following is true about the
ascending pathway for sympathetic afferents?
102. Brodmann area 6 of the cerebral cortex corre- (A) The cell bodies of origin of sympathetic
sponds to the fibers are located in the posterior root
(A) frontal eye field ganglia at about levels S2 to S4.
(B) Broca convolution (B) The sympathetic afferent fibers termi-
nate in spinal cord laminae I and V.
(C) premotor somatosensory cortex
(C) The sympathetic afferent fibers enter the
(D) angular gyrus
spinal cord via the anterior root.
(E) supramarginal gyrus
(D) The sympathetic afferent fibers are
formed by large myelinated fibers.
103. In the medulla, the vascular supply of the medial
lemniscus is provided by (E) The input originating from physiological
receptors travels primarily in sympa-
(A) penetrating branches of the basilar thetic afferent fibers.
artery
(B) the posterior cerebral artery 108. The restiform body is supplied by the
(C) the posterior spinal artery (A) anteroinferior cerebellar artery
(D) the anterior spinal artery (B) posteroinferior cerebellar artery
(E) the superior cerebellar artery (C) superior cerebellar artery
(D) posterior cerebral artery
104. The thalamocortical cells and fibers arising
from the ventral posterolateral nucleus (E) basilar artery

(A) conveys inhibitory input to the cortex 109. Mossy fibers


(B) are gluteminergic neurons
(A) are the only afferent fibers of the cerebel-
(C) are GABAergic neurons lar cortex
(D) terminate in the auditory cortex (B) originate from the inferior olivary
(E) receive ascending input from the nucleus
trigeminothalamic tract (C) utilize glutamate as a neurotransmitter
(D) are inhibitory to granule cerebellar cells
105. The cell bodies of the trigeminal primary affer-
ent neurons are located in the (E) synapse exclusively with the Purkinje
cerebellar cells
(A) semilunar ganglion
(B) nucleus ambiguus 110. The climbing cerebellar fibers
(C) red nucleus (A) are cerebellar efferent fibers
(D) nucleus solitarius (B) excite Purkinje cells
(E) cuneate nucleus (C) originate from the locus ceruleus
(D) terminate exclusively withn the granule
106. The A nociceptive primary afferent fibers
layer of the cerebellum
target
(E) use norepinephrine as a neurotransmitter
(A) Rexed laminae I, II, and IV
(B) Rexed laminae II and V
14 1: Anatomy and Physiology of the Central and Peripheral Nervous System

111. The parasympathetic postganglionic cell bodies (C) lateral posterior nucleus of the thalamus
of the oculomotor nerve are located in the (D) lateral geniculate body
(A) pterygopalatine ganglion (E) cerebellum
(B) submandibular ganglion
115. The medial geniculate nucleus
(C) otic ganglion
(D) ciliary ganglion (A) is a relay thalamic nucleus in the audi-
(E) inferior salivary nucleus tory system
(B) receives fibers from the optic tract
112. The parasympathetic preganglionic cell bodies (C) receives input from the somatosensory
of the facial nerve are located in the cortex
(A) Edinger-Westphal nucleus (D) is involved in motor corrdination
(B) inferior salivatory nucleus (E) is a part of the limbic system
(C) otic ganglion
116. The only cerebellar neuron that sends its axons
(D) nucleus ambiguus outside the cerebellum is the
(E) superior salivatory nucleus
(A) basket cell
113. The anterior group of the thalamus has recip- (B) stellate cell
rocal connection with (C) golgi cell
(A) mammillary bodies (D) granule cell
(B) amygdaloid nucleus (E) Purkinje cell
(C) auditory cortex
117. The structure indicated by the arrow in Figure
(D) striatum 1-7 is the
(E) visual cortex
(A) optic chiasm
114. The prefrontal cortex has reciprocal connection (B) hypothalamic sulcus
via the anterior thalamic peduncle with the (C) pituitary gland
(A) anterior nuclear group of the thalamus (D) lamina terminalis
(B) dorsomedial nucleus of the thalamus (E) mammillary body

FIG. 1-7. (Reproduced with permission from Afifi AK, Bergman RA. Functional Neuroanatomy: Text and Atlas. 2nd ed.
New York: McGraw-Hill; 2005.)
Questions: 111119 15

FIG. 1-8. (Reproduced with permission from Afifi AK, Bergman RA. Functional Neuroanatomy: Text and Atlas. 2nd ed.
New York: McGraw-Hill; 2005.)

118. The structure indicated by the arrow in Figure 119. The structure indicated by the arrow in Figure
1-8 is the 1-9 is the
(A) optic nerve (A) cingulate gyrus
(B) olfactory bulb (B) corpus callosum
(C) optic chiasm (C) cingulate sulcus
(D) optic tract (D) parahippocampal gyrus
(E) oculomotor nerve (E) subcallosal gyrus

FIG. 1-9. (Reproduced with permission from Afifi AK, Bergman RA. Functional Neuroanatomy: Text and Atlas. 2nd ed.
New York: McGraw-Hill; 2005.)
16 1: Anatomy and Physiology of the Central and Peripheral Nervous System

120. The first intracranial branch of the internal 123. Each of the lateral ventricles is connected to
carotid artery is the the third ventricle by the
(A) anterior cerebral artery (A) cerebral aqueduct
(B) anterior choroidal artery (B) foramen of Magendie
(C) recurrent artery of Heubner (C) foramen of Luschka
(D) superior cerebellar artery (D) quadrigeminal cistern
(E) ophthalmic artery (E) foramen of Monro

121. The anterior limb and genu of the internal cap- 124. The structure indicated by the arrow in Figure
sule are supplied by the 1-11 is the
(A) middle cerebral artery (A) vertebral artery
(B) recurrent artery of Heubner (B) basilar artery
(C) superior cerebellar artery (C) anteroinferior cerebellar artery
(D) vertebral artery (D) anterior spinal artery
(E) anteroinferior cerebellar artery (E) superior cerebellar artery

122. The structure indicated by the arrow in Figure


1-10 is the
(A) postcentral gyrus
(B) intraparietal sulcus
(C) central sulcus
(D) cingulate sulcus
(E) precentral gyrus

B1

B2

B3

B4

FIG. 1-10. (Reproduced with permission from Martin JH. Neuroanatomy. Text FIG. 1-11. (Reproduced with permission from Martin JH. Neuroanatomy. Text
and Atlas. 3rd Ed. New York: McGraw-Hill; 2003.) and Atlas. 3rd ed. New York: McGraw-Hill; 2003.)
Questions: 120129 17

FIG. 1-12. (Reproduced with permission from Martin JH. Neuroanatomy. Text and Atlas. 3rd ed. New York: McGraw-Hill; 2003.)

125. The structure indicated by the arrow in Figure (D) medial geniculate nucleus
1-12 is the (E) red nucleus
(A) insular cortex
128. The preganglionic neurons that innervate the
(B) midbrain
parotid gland originate from the
(C) corpus callosum
(D) lateral geniculate body (A) inferior salivary nucleus
(E) globus pallidus (B) superior salivary nucleus
(C) EdingerWestphal nucleus
126. Although the cerebrospinal fluid is mainly (D) dorsal motor nucleus of the vagus
secreted by the choroid plexus, it is also (E) solitary nucleus
secreted by the
(A) inferior sagittal sinus 129. Clarkes nucleus
(B) the capillaryastrocyte complex (A) is located in the caudal medulla
(C) dura mater (B) relays somatic sensory information from
(D) pineal gland the upper limbs to the cerebellum
(E) pia mater (C) gives rise to the dorsal spinocerebellar
tract
127. The first central nervous system relay for taste (D) receives afferent fibers that course in the
is located in the cuneate fascicle
(A) nucleus ambiguus (E) gives rise to crossed fibers that enter the
cerebellum via the superior cerebellar
(B) solitary nucleus
peduncle
(C) lateral geniculate nucleus
18 1: Anatomy and Physiology of the Central and Peripheral Nervous System

FIG. 1-13. (Reproduced with permission from Martin JH. Neuroanatomy.


Text and Atlas. 3rd ed. New York: McGraw-Hill; 2003.)

130. The inferior olivary nucleus hosts neurons of the


(A) cerebellar climbing fibers
(B) cerebellar mossy fibers
(C) cerebellar parallel fibers
(D) reticulospinal fibers
(E) trigeminal cerebellar fibers

131. The structure indicated by the arrow in Figure


1-13 is the
(A) lateral sulcus
(B) angular gyrus
(C) central sulcus
(D) precentral sulcus
(E) middle frontal gyrus

132. The structure indicated by the arrow in Figure


1-14 is the
(A) red nucleus
(B) oculomotor nerve
(C) mammillary body
(D) pineal gland
FIG. 1-14. (Reproduced with permission from Martin JH. Neuroanatomy.
(E) substantia nigra Text and Atlas. 3rd ed. New York: McGraw-Hill; 2003.)
Questions: 130134 19

FIG. 1-15. (Reproduced with permission from Martin JH. Neuroanatomy. Text and Atlas.
3rd ed. New York: McGraw-Hill; 2003.)

133. The structure indicated by the arrow in Figure


1-15 is the
(A) trigeminal nerve
(B) oculomotor nerve
(C) facial nerve
(D) abducens nerve
(E) trochlear nerve

134. The structure indicated by the arrow in Figure


1-16 is the
(A) oculomotor nerve
(B) trigeminal nerve
(C) abducens nerve
(D) hypoglossal nerve
(E) vagus nerve

FIG. 1-16. (Reproduced with permission from Martin JH. Neuroanatomy.


Text and Atlas. 3rd ed. New York: McGraw-Hill; 2003.)
20 1: Anatomy and Physiology of the Central and Peripheral Nervous System

135. The structure indicated by the arrow in Figure 136. The structure indicated by the arrow in Figure
1-17 is the 1-18 is the
(A) trigeminal nerve (A) pineal gland
(B) abducens nerve (B) mammillary body
(C) facial nerve (C) superior colliculus
(D) oculomotor nerve (D) trochlear nerve
(E) trochlear nerve (E) third ventricle

FIG. 1-18. (Reproduced with permission from Martin JH. Neuroanatomy.


Text and Atlas. 3rd ed. New York: McGraw-Hill; 2003.)

FIG. 1-17. (Reproduced with permission from Martin JH. Neuroanatomy.


Text and Atlas. 3rd ed. New York: McGraw-Hill; 2003.)
Questions: 135137 21

137. The structure indicated by the arrow in Figure (C) inferior cerebellar peduncle
1-19 is the (D) medial lemniscus
(A) pyramid (E) solitary nucleus
(B) reticular formation

FIG. 1-19. (Reproduced with permission from Martin JH. Neuroanatomy. Text and Atlas. 3rd ed. New York: McGraw-Hill; 2003.)
Answers and Explanations

1. (D) The olfactory pathway is the only sensory cochlear nuclei in the pons. The second-order
pathway that does not project to the thalamus. neurons of the auditory pathway are formed by
The olfactory nerve penetrates the cribriform fibers of cochlear nuclei crossing to the con-
plate of the ethmoid bone and enters the olfac- tralateral inferior colliculus. The latter contains
tory bulb to synapse with the second-order neu- the third-order neurons and serves as the cen-
rons: mitral and tufted cells. The axons of the tral relay nucleus in the auditory pathway. The
second-order neurons course posteriorly as the projections from the inferior colliculus termi-
olfactory tract in the orbital surfaces of the nate in the medial geniculate body, a thalamic
frontal lobe and project to the primary olfac- auditory relay nucleus. The fourth-order neu-
tory cortex in the temporal lobe. In the visual rons are formed by the geniculotemporal fibers
pathway, axons of the ganglion cells in the that project to the primary auditory cortex.
retina gather together at the optic disk to form Vibration sensation is mediated by Merkel disk
the optic nerve. The two optic nerves come receptors and Meissners corpuscules. Fibers
together at the optic chiasma, where a partial mediating vibration terminate in the deeper
crossing of optic nerve fibers takes place. The layers of the dorsal horn. Second-order neu-
crossed and uncrossed fibers from the optic rons from the dorsal horn ascend through the
nerves join caudal to the optic chiasma to form ipsilateral dorsolateral funiculus, terminating
the optic tracts, which extend from the dorso- on neurons in the nuclei of the posterior dorsal
lateral corners of the chiasma to the lateral column (nucleus gracilis and nucleus cunea-
geniculate bodies. These constitute a thalamic tus) in the medulla oblongata. Axons of these
nucleus that provides a relay station for all the nuclei cross the midline to form the medial
axons of the retinal ganglion cells subserving lemniscus, which ascends to the thalamus (ven-
vision. The lateral geniculate nucleus is lami- tral posterolateral nucleus) and from there to
nated into six layers. Not all parts of the retina the primary sensory cortex. Temperature sen-
are represented equally in it. The central area of sation is mediated by the lateral spinothalamic
the retina has larger representation than does tract, which projects into the ventral postero-
the periphery of the retina. Axons of neurons of lateral nucleus of the thalamus. (Parent, 748754;
the lateral geniculate nucleus project to the Afifi, 53, 314, 319; Brazis, 133138, 307309)
visual cortex in the occipital lobe via the genicu-
localcarine tract. Geniculocalcarine fibers proj- 2. (E) The lateral geniculocalcarine tract connects
ect to the visual cortex. the lateral geniculate nucleus to the visual
The auditory pathway is described as a cortex. The geniculocalcarine fibers from the
four-tiered neuronal network. The auditory upper halves of both retinas course directly
cochlear nerve, extending from the organ of backward around the lateral ventricle in the
Corti to the cochlear nucleus, generates action inferior part of the parietal cortex to reach the
potentials that travel in the afferent nerve fibers visual cortex. Geniculocalcarine fibers from the
via the central components (axons) of bipolar lower halves of both retinas course forward
neurons in the spiral ganglion to reach the toward the tip of the temporal horn of the

22
Answers: 17 23

lateral ventricle and then loop backward in the 5. (A) The long thoracic nerve arises from the
temporal lobe to reach the visual cortex. motor roots of C5, C6, and C7. It courses down-
Because of the spread of geniculocalcarine ward through and in front of the medial
fibers in the parietal and temporal lobes, a scalenus muscle and further descends dorsal to
lesion involving the upper fibers located in the the brachial plexus along the medial axillary
temporal lobe (as shown in Figure 1-1) pro- wall to innervate the serratus anterior muscle.
duces a contralateral inferior quadrantic visual The suprascapular nerve innervates the sup-
field defect. A lesion involving the lower fibers raspinatus and infraspinatus. The dorsal scapu-
located in the parietal lobe produces a con- lar nerve innervates the rhomboid and levator
tralateral superior quadrantic visual field scapulae. (Parent, 276; Staal, 19)
defect. (Afifi, 314)
6. (E) The axillary nerve originates from the pos-
3. (D) All the fibers forming the posterior funicu- terior fascicle of the brachial plexus and carries
lus are ascending fibers throughout the spinal fibers from C5 and C6. It is divided into supe-
cord and synapse on the posterior column rior and inferior branches. The superior branch
nuclei (nucleus gracilis and nucleus cuneatus) travels with the circumflex artery around the
in the medulla oblongata. Approximately 85% humeral head neck to innervate the deltoid
of ascending fibers in the posterior funiculus muscle. The inferior branch supplies the teres
are primary afferents. These have cell bodies in minor muscle. The axillary nerve sends a sen-
the dorsal root ganglia and are activated by sory branch, the lateral brachial cutaneous
stimulation of mechanoreceptors but not noci- nerve, to the skin of the upper outer surface of
ceptors. These cells are called unimodal affer- the arm, mainly in the deltoid region. An axil-
ents. Approximately 15% of fibers in the lary nerve lesion results in weakness of arm
posterior funiculus are nonprimary afferents. abduction against resistance in the horizontal
These have cell bodies in the dorsal root gan- position. The first 30 degrees of abduction of the
glia, establish synapses in laminae III to V in upper arm from the trunk is performed by the
the posterior horns of the cervical and lumbar supraspinatus muscle, which is innervated by
enlargement, and are activated by stimulation the suprascapular nerve, not by the axillary
of both mechanoreceptors and nociceptors. nerve. There is also weakness in retracting the
They are called polymodal afferents. (Afifi, 53) horizontal upper arm against resistance, with
sensory loss in the skin area overlying the del-
4. (A) In lesions of the posterior column, discrep- toid muscle. The axillary nerve is often involved
ancies in loss of vibration and position sense in neuralgic amyotrophy; in about 10% of cases,
have been observed. A possible explanation of it is affected in isolation. (Parent, 275277; Staal,
this is that different pathways are used for the 2729; Brazis, 3233)
transmission of the two modalities. In experi-
mental animals, it has been shown that vibration 7. (C) The musculocutaneous nerve arises from
sensation in forelimbs and hindlimbs transmits the lateral cord of the brachial plexus and car-
its impulses via the dorsal columns and the spin- ries fibers from the roots of C5, C6, and C7.
ocervical thalamic tract. In contrast, propiocep- The nerve proceeds obliquely downward
tive sensations from the forelimbs utilize the between the axillary artery and the median
dorsal column, while those from the hindlimbs nerve. The nerve pierces the coracobrachialis
travel with the gracile tract to the level of the muscle while giving off branches to it, and it
dorsal nucleus of Clarke. From there, these sen- descends further between the biceps and
sations leave the gracile tract, synapse in the brachialis muscles to supply them both. The
nucleus dorsalis of Clarke, and travel with the lateral cutaneous nerve of the forearm is the
dorsal spinocerebellar fibers to terminate on the sensory continuation of the musculocutaneous
nucleus of Z, a small collection of cells in the nerve. It innervates the skin from the elbow to
most rostral part of the nucleus gracilis of the the wrist and covers the entire forearm
medulla. (Afifi, 54) from the dorsal to the ventral midline. The
24 1: Anatomy and Physiology of the Central and Peripheral Nervous System

coracobrachialis muscle is a forward elevator of spinatus muscles, and the subclavian nerve
the arm. The biceps is a forearm supinator, innervating the subclavian muscle. Prior to
especially if the elbow is flexed at 90 degrees. forming the brachial plexus trunks, the long
Isolated lesions of the musculocutaneous nerve thoracic nerve arises from the C5C7 anterior
are rare. Such lesions would cause weakness of primary rami to innervate the serratus ante-
elbow flexion against resistance in a fully rior muscle, and the dorsal scapular nerve
supinated hand, possible weakness in arm ele- arises from the C4C5 anterior primary rami to
vation, arm pain, and radial forearm paresthe- innervate the rhomboid and levator scapulae
sia. (Brazis, 3435; Staal, 3133) muscles. The teres minor muscles are inner-
vated by the axillary nerve, which originates
8. (E) The radial nerve arises from the posterior from the posterior cord and does not carry any
cord of the brachial plexus and comprises fibers contribution from the lateral cord. The inner-
from spinal levels C5 to C8. After descending vation of the four other muscles listed in the
posterior to the axillary artery, the nerve question is carried by both the upper trunk and
courses posterior to the humerus in the spiral lateral cord. The pronator teres, flexor carpi
groove. It is at this site that the nerve is most radialis, and abductor pollicis brevis are inner-
often damaged by compression. (Staal, 35) vated by the median nerve, which is formed in
the axilla where the lateral cord joins with the
9. (D) The dorsal scapular nerve is a pure motor medial cord of the brachial plexus. The biceps
nerve. It carries fibers from the C4 and C5 muscle is innervated by the musculocutaneous
spinal nerves and, after piercing the medial nerve, which arises from the lateral cord of the
scalenus muscle, courses downward behind brachial plexus. (Brazis, 7576)
the brachial plexus to innervate the levator
scapulae. From there it courses along the 11. (D) The extensor indicis proprius as well the
medial border of the scapula to the rhomboid other muscles innervated by the C8 radial
muscles. These elevate and adduct the medial nerve receive their innervation via the lower
border of the scapula, antagonizing the serratus plexus, posterior division of the lower trunk,
anterior. Along with the levator scapulae, the and posterior cord without any contribution
rhomboid muscles rotate the scapula so that from the medial cord. (Brazis, 7576)
the inferior angle moves medially. These mus-
cles are tested by having the patient press his or 12. (B) The pronator teres is innervated by the
her elbow backward against resistance while C6C7 median nerve. The lateral cord (derived
the hand is on the hip. (Staal, 17; Brazis, 29) from the upper and middle trunk of the
brachial plexus) provides innervations to the
10. (A) The upper trunk of the brachial plexus is pronator teres without any contribution from
formed by the anterior primary rami of the fifth the posterior cord. (Brazis, 7576)
and sixth cervical roots. They course down-
ward between the scalenus medius and ante- 13. (D) The sciatic nerve is a mixed nerve that car-
rior muscles and unite to form the upper trunk. ries fibers from L4 to S3 and leaves the pelvis
The latter traverses the supraclavicular fossa, through the sciatic foramen below the piriform
accompanied by the middle and lower trunk. muscle. The nerve then curves laterally and
Lateral to the first rib and behind the axillary downward beneath the gluteus maximus
artery, the three trunks split into three anterior muscle and runs on the dorsal side of the
and three posterior divisions. The anterior divi- femoral bone to terminate at the proximal part
sions of the upper and middle trunks unite to of the popliteal fossa, where it divides into the
form the lateral cord. The posterior divisions of tibial nerve medially and the peroneal nerve
the three trunks unite to form the posterior laterally. Within the sciatic nerve, as proximal
cord. Two branches arise from the proximal as the gluteal region, the fibers of the tibial and
aspect of the upper trunk, the suprascapular peroneal nerves are arranged into two sepa-
nerve innervating the suprasinatus and infra- rate divisions: the medial and the lateral trunks.
Answers: 820 25

The medial part of the nerve innervates the medulla between the pyramid and inferior
adductor magnus and the hamstring muscles olive. (Afifi, 8788)
except for the short head of the biceps femoris
(it is the only thigh muscle supplied by the lat- 17. (D) Vertical saccades are controlled by cortical
eral peroneal division). The hamstring muscles pathways descending to the rostral interstitial
are flexors of the knee joint and include the nucleus of the medial longitudinal fasciculus at
semimembranosus muscle, the semitendinosus the junction between the midbrain and the thal-
muscle, and the short and long heads of the amus. (Kline and Bajandas, 50)
biceps femoris. (Staal, 117118)
18. (E) The abducens nucleus is the site of horizon-
14. (A) The point of entrapment of the median tal versional control. The nucleus of the abducens
nerve in carpal tunnel syndrome lies under the nerve contains two types of neurons: those that
flexor retinaculum, which forms the roof of the innervate the ipsilateral lateral rectus and those
carpal tunnel, whereas the carpal bones and that project via the contralateral medial longi-
their connective tissue components form the tudinal fasciculus to the contralateral oculo-
floor of the carpal tunnel. In Guyons canal, motor nucleus. The parapontine reticular
the hamate and pisiform bones are sites of com- formation contains cells that project to the
pression of the ulnar nerve at the wrist. Rarely, abducens nucleus and activate it. The para-
radial nerve compression occurs at the level of pontine reticular formation contains excitatory
the styloid process, just proximal to the wrist. burst neurons that discharge just prior to a hor-
(Staal, 5666; Brazis, 38) izontal saccade to stimulate cells in the
abducens nucleus. Once the eye reaches a new
15. (B) The femoral nerve supplies the sartorius eccentric position at the end of the saccade,
muscle (a flexor and everter of the thigh) and the stimulation of the abducens nucleus by the
quadriceps (an extensor of the leg). The obturator parapontine reticular formation burst neurons
nerve supplies the adductor muscles of the thigh. is substituted by a tonic gaze-holding mecha-
The genitofemoral nerve is predominantly a nism to maintain the eccentric position. This
sensory nerve. It divides near the inguinal lig- requires a neuronal network that integrates a
ament into the external genital branch (respon- velocity-coded signal into a position-coded
sible for the innervation of the cremaster signal. This is referred to as the neural integra-
muscle) and the medial femoral branch (respon- tor, which includes the horizontal gaze center,
sible for the innervation of the skin of the upper the medial vestibular nucleus, and the nucleus
thigh over the femoral triangle). The superior prepositus hypoglossi. (Kline and Bajandas, 55)
gluteal nerve innervates the gluteus medius
and gluteus minimus. The inferior gluteal nerve 19. (A) The rostral interstitial nucleus of the
innervates the gluteus maximus. (Brazis, 5156) medial longitudinal fasciculus contains excita-
tory burst neurons for vertical and torsional
16. (A) This is a brainstem section at the level of saccade. It projects bilaterally to the oculomo-
the medulla. The structure indicated by the tor nuclei in the case of upward gaze and
arrow is the hypoglossal nucleus. The section mainly ipsilaterally in the case of downward
shows a schematic diagram of the origin and gaze. The gaze-holding neural integrator for
intramedullary course of rootlets of the vertical gaze is located in the interstitial nucleus
hypoglossal nerve. Except for its most rostral of Cajal. (Kline and Bajandas, 52)
and caudal levels, the nucleus of the hypoglos-
sal nerve extends throughout the medulla 20. (E) The trochlear nerve is purely a motor nerve
oblongata. It is divided into cell groups that and is the only cranial nerve to exit the brain
correspond to the tongue muscles they supply. dorsally. The trochlear nerve supplies one
The root fibers of the nerve course in the muscle: the superior oblique. The cell bodies
medulla oblongata lateral to the medial lemi- that originate in the trochlear nerve are located
niscus and emerge on the ventral surface of the in the ventral part of the brainstem in the
26 1: Anatomy and Physiology of the Central and Peripheral Nervous System

trochlear nucleus. The trochlear nucleus gives They receive direct projections from the senso-
rise to fibers that cross to the other side of the rimotor cortex. The cortical input to these
brainstem just prior to exiting the pons. Thus nuclei provides the basis for cortical influences
each superior oblique muscle is supplied by on the baroreceptor reflex and sympathetic
nerve fibers from the trochlear nucleus of the vasomotor mechanisms for the control of blood
opposite side. The nerve travels in the lateral pressure. (Afifi and Bergman, 92)
wall of the cavernous sinus and then enters the
orbit via the superior orbital fissure. It passes 24. (C) The rostral and lateral zone of the nucleus
medially and diagonally across the levator solitarius is concerned with taste sensation. It
palpebral superioris and superior rectus muscles receives gustatory sensations via three cranial
to innervate the superior oblique. (Parent, 531) nerves: the facial nerve conveys taste sensation
from the anterior two thirds of the tongue; the
21. (A) The perihypoglossal nuclei are nuclear glossopharyngeal nerve conveys taste sensa-
masses in close proximity to the hypoglossal tion from the posterior third of the tongue; and
nerve. They receive input from the cerebral the vagus nerve conveys taste sensation from
cortex, vestibular nuclei, accessory oculomotor the epiglottis. (Afifi and Bergman, 92)
nuclei, and paramedian pontine reticular for-
mation. The output of these nuclei terminates in 25. (B) The structure indicated by the arrow in
the cranial nuclei involved in extraocular move- Figure 1-3 is the pontine reticular nucleus.
ments: the oculomotor nerve, trochlear nerve, Damage to the pontine reticular nuclei in the
and abducens nerve. The output of the perihy- tegmentum and corticospinal fibers in the basis
poglossal nuclei also terminates in the thalamus pontis is associated with anosognosia for hemi-
and the cerebellum. (Afifi and Bergman, 88) plegia, in which patients are unware of their
motor deficit. A similar syndrome occurs in
22. (C) The nucleus ambiguus is the ventral motor damage to the nondominant parietal lobe. (Afifi
nucleus of the vagus. Axons of neurons in this and Bergman, 105106)
nucleus convey special visceral efferent
impulses to the branchiomeric muscles of the 26. (B) The afferent roots of the trigeminal nerve
pharynx and larynx. It also contributes efferent contains general somatic sensory fibers that
fibers to the glossopharyngeal and accessory convey pain, temperature, and touch sensation
nerves. The dorsal motor nucleus of the vagus from the face and anterior aspect of the head.
is located dorsolateral or lateral to the The neurons of origin of these fibers are situated
hypoglossal nucleus. It receives afferent fibers in the semilunar ganglion (gasserian ganglion).
from the vestibular nuclei and conveys efferent The peripheral processes of neurons in the gan-
preganglionic parasympathetic fibers respon- glion are distributed in the three divisions of
sible for general efferent impulses to the vis- the trigeminal nerve. The proprioceptive fibers
cera in the thorax and abdomen. The nucleus of from the deep structures of the face are periph-
the spinal tract of the trigeminal nerve receives eral processes of unipolar neurons in the mes-
general somatic afferent fibers from the external encephalic nucleus of the trigeminal nerve that
ear. The nucleus solitarius receives special vis- travel via afferent and efferent roots of the
ceral afferent fibers that convey taste sensation trigeminal nerve. The proprioceptive fibers in
from the region of the epiglottis. It also receives the mesencephalic nucleus convey pressure and
general visceral afferent fibers that convey pain and kinesthesia from the teeth, periodontium,
sensation from the mucosa of the posterior third hard palate, and joint capsules as well as
of the tongue. (Afifi and Bergman, 9091) impulses from stretch receptors in the muscles of
mastication. The mesencephalic nucleus of the
23. (B) The nucleus solitarius, the dorsal motor trigeminal nerve is involved in control of the
nucleus of the vagus, and the caudal and ros- bite force. The facial nerve conveys taste sensa-
tral ventrolateral medulla comprise the brain- tion from the anterior two third of the tongue.
stem nuclei involved in cardiovascular control. The motor nucleus of the trigeminal nerve is
Answers: 2132 27

involved in eliciting the jaw reflex. The facial is a synaptic station in neural systems con-
motor nuclei are involved in eliciting the corneal cerned with movement, linking the cerebral
reflex. Collaterals from the secondary ascending cortex, cerebellum, and spinal cord. The ventral
trigeminal tracts establish synapses with the tegmental nucleus is a part of a circuit con-
facial motor nuclei on both sides, resulting in the cerned with emotion and behavior. (Afifi and
bilateral blink reflex and the corneal reflex in Bergman, 133135, 138)
response to unilateral corneal stimulation. (Afifi
and Bergman, 115118) 31. (E) Figure 1-6 is a schematic diagram of the
midbrain at the level of the superior colliculus.
27. (D) Figure 1-4 is a schematic diagram of the The structure indicated by the arrow in Figure
ventral surface of the midbrain and pons. 1-6 is the red nucleus. It is a prominent part of
The structure indicated by the arrow is the the tegmentum at the level of the superior col-
trochlear nerve. The nerve fascicles course pos- liculus, comprising a caudal magnocellular
teroinferiorly around the aqueduct to decus- and a rostral parvocellular part. It is involved
sate in the dorsal midbrain in the anterior in motor coordination. (Afifi and Bergman, 139)
medullary velum. After traveling on the under-
surface of the tentorial edge, it pierces the dura 32. (C) Pupillary size is under the dual control of
and travels to the cavernous sinus to reach the sympathetic and parasympathetic systems that
superior orbital fissure and innervate the supe- innervate rings of radially arranged dilator and
rior oblique muscle. (Afifi and Bergman, 130; constrictor fibers, respectively. The resting size
Brazis, Masdeu, and Biller, 185) of the pupil depends on the intensity of light
falling on the retina and the integrity of the
28. (A) The mesencephalic nucleus of the trigem- parasympathetic nerves. A light stimulus is
inal nerve is homologous in structure to the conveyed from the retina to the optic nerve,
dorsal root ganglion but is uniquely placed optic chiasm, and lateral geniculate body. Ten
within the central nervous system. It contains percent of the afferent fibers subserve the light
unipolar neurons with axons that convey pro- reflex and are related, in the periaqueductal
prioceptive impulses from the muscles of mas- gray, to both EdingerWestphal nuclei (which
tication and the periodontal membranes. (Afifi induce papillary constriction) and the consen-
and Bergman, 132) sual light reflex. The parasympathetic fibers
are then carried by the third cranial nerve to the
29. (A) Figure 1-5 is a schematic diagram of the ciliary ganglion and to the pupillary constrictor
midbrain at the level of the inferior colliculus. fibers. The sympathetic system starts from the
The structure indicated by the arrow is the hypothalamus; its fibers pass to the cervi-
rubrospinal tract. It conveys fibers from the red cothoracic spinal cord at levels C8 and T1. The
nucleus to the spinal cord and inferior olive. second-order neurons pass from the spinal cord
(Afifi and Bergman, 132) to the superior cervical ganglion. The third-
order neurons supply the pupillodilator fibers
30. (B) The locus ceruleus is located in the rostral and the blood vessels of the eye, passing over
pons and caudal mesencephalon. The neurons the carotid artery. Any lesion affecting those
of the locus ceruleus provide noradrenergic afferent pathways that include the retina, optic
innervations to most central nervous system chiasm, optic tract, and particularly the optic
regions. The substentia nigra has a neuronal nerve will cause a Marcus Gunn pupil. When a
population consisting of pigmented and light is shined in the normal eye, both pupils
nonpigmented neurons. The neurotransmitter constrict (direct and consensual light reflexes).
in pigmented neurons is dopamine, whereas When the light is then swung to the sympto-
nonpigmented neurons are either cholinergic matic eye, less light reaches the oculomotor
or GABAergic. The dorsal raphe nucleus sends nucleus because of optic nerve damage. The
serotonergic fibers to the substantia nigra, cau- oculomotor nucleus senses the less intense light
date, putamen, and neocortex. The red nucleus and shuts off the parasympathetic response,
28 1: Anatomy and Physiology of the Central and Peripheral Nervous System

resulting in paradoxical pupillary dilatation. cis longus, extensors pollicis longus and brevis,
Adies pupil is characterized by widely dilated and extensor indicis. (Brazis, Masdeu, and Biller,
pupil and a sluggish, prolonged pupillary con- 47; Staal, 3536)
traction in reaction to light. It results from
pathology of the ciliary ganglion within the 36. (A) The tensor fascia lata is innervated by the
orbit. (Patten, 79; Afifi and Bergman, 143144) superior gluteal nerve, which also innervates the
gluteus medius and gluteus minimis. (Staal, 113)
33. (D) A waddling gait is seen with weak hip
muscles, particularly the gluteus medius. This 37. (C) Although the deep peroneal nerve is
results in an excessive drop of the hip and mainly a motor nerve, it provides sensory inner-
trunk, tilting the pelvis to the side opposite to vation to the skin of the first interosseous space
the foot placement. The hips oscillate up and and the adjacent skin of the sides of the first
down with every step, making the patient seem and second toes. (Brazis, Masdeu, and Biller, 59)
to waddle. Lesion to the anterior lobe of the
cerebellum may cause discrete gait impairment. 38. (C) The medial wall of the cavernous sinus
Severe loss of sensation in large muscle fibers contains the abducens nerve, the internal
may result in a steppage gait. This is character- carotid artery, and the sympathetic fibers of the
ized by excessive flexion of the hips and knees carotid plexus. The lateral wall contains the
with every step. With severe sensory loss in the oculomotor and trochlear nerves and the oph-
lower extremities, the heel tends to strike the thalmic and maxillary divisions of the trigem-
ground heavily. Greater foot clearance is then inal nerve. (Haines, 136)
used to avoid tripping on the toes or on irregu-
larities of the floor or ground that are poorly 39. (C) The trigeminal nerve is a mixed nerve. It
felt. A steppage gait may be seen in Guillain subserves the sensory innervation of the ipsi-
Barr syndrome and other demyelinating lateral side of the face and the ipsilateral mus-
polyneuropathies. Acute vestibular lesions cles of mastication (masseter, temporalis, and
cause vestibular ataxia. This is characterized by pterygoids). The sensory nucleus of the trigem-
instability and the patients tendency to veer or inal nerve extends from the midbrain to the
even fall to the side of the lesion. Corticospinal upper cervical cord: (a) The mesencephalic
tract lesions cause a spastic gait. (Brazis, Masdeu, nucleus subserves proprioception and deep
and Biller, 2021) sensation from the tendons and muscles of
mastication. (b) The main sensory nucleus
34. (B) After it passes between the two heads of (located in the pons) subserves light touch. (c)
the pronator teres, the median nerve gives off The spinal nucleus (which extends from the
a purely motor anterior interosseous nerve that pons to the upper cervical cord and is divided
innervates the flexor pollicis longus, flexor dig- into segments that correspond to concentric
itorum profundus I and II, and pronator quad- dermatomes around the mouth) subserves pain
ratus. (Brazis, Masdeu, and Biller, 36) and temperature. The trigeminal nerve sup-
plies sensation to the ipsilateral side of the face
35. (A) The posterior interosseous nerve is a pure via three branches: the ophthalmic division
motor branch of the radial nerve. At some (which innervates the frontal, lacrimal, and
point, varying between 3 cm above and a sim- nasociliary areas), the maxillary division
ilar distance below the humeroradial joint, the (which innervates the cheek and lower eyelid),
radial nerve divides into a deep motor branch, and the mandibular division (which innervates
which is the posterior interosseous nerve, and the lower lip, tongue, and mandible except for
a superficial sensory branch. The posterior the angle of the mandible). The motor nucleus
interosseous nerve innervates the following lies medially to the main sensory nucleus and
muscles: the supinator, extensor carpi radialis sends axons to the mandibular division of the
brevis, extensor digitorum, extensor digiti trigeminal nerve. All divisions of the trigeminal
minimi, extensor carpi ulnaris, abductor polli- nerve converge at the gasserian ganglion,
Answers: 3345 29

which lies in Meckels cave of the temporal jections from all areas of the cerebral cortex and
bone. (Afifi and Bergman, 117118) from certain thalamic nuclei, the substantia
nigra, and other brainstem nuclei. The term
40. (D) The facial nerve leaves the pons and trav- neostriatum refers to the caudate nucleus and
els with the vestibulocochlear nerve through putamen. The caudate nucleus, an elongated
the internal auditory canal. (Parent, 154168) gray mass whose pear-shaped head is continu-
ous with the putamen, lies adjacent to the infe-
41. (E) The nervus intermedius is the sensory and rior border of the anterior horn of the lateral
parasympathetic division of the facial nerve. ventricle. The slender end curves backward and
Its preganglionic parasympathetic fibers arise downward as the tail; it enters the roof of the
from the superior salivary nucleus and synapse temporal horn of the lateral ventricle and tapers
in the pterygopalatine and submandibular off at the level of the amygdala. The putamen is
nerves, which then send postganglionic fibers located lateral to the globus pallidus and medial
to submandibular, sublingual, lacrimal, palatal, to the external capsule. It is separated from the
and nasal glands. The sensory fibers of the caudate nucleus by the internal capsule except
nervus intermedius arrive at the nervus inter- rostrally, where the head of the caudate and the
medius via the geniculate ganglion. They pro- putamen are continuous around the anterior
vide taste sensation to the anterior two thirds of limb of the internal capsule. (Afifi and Bergman,
the tongue. (Afifi and Bergman, 112) 180181)

42. (A) The patient described in this vignette has 45. (C) In the direct loop, cortical fibers project to
a pure motor deficit. The right pronator quad- the striatum, and striatal efferent neurons proj-
ratus is weak because of paresis of forearm ect to the internal globus pallidus and the sub-
pronation. Also, there is paresis of the flexor stantia nigra pars reticulata. Efferents from the
digitorum profundus I and II and the flexor internal globus pallidus and the substantia
pollicis longus because of loss of flexion of the nigra pars reticulata project to the dorsal thal-
terminal phalanges of the second and third fin- amus, and the thalamic neurons project to spe-
gers and thumb, respectively. All of these mus- cific areas of the cerebral cortex. Both the
cles are innervated by an anterior interosseous glutaminergic corticostriate projections and the
nerve. The characteristic feature of a lesion of thalamocortical projections are excitatory.
this nerve is the inability to make a circle with However, the efferents from the striatum to the
the thumb and index finger. (Staal, 5556) internal globus pallidus and the substantia
nigra pars reticulata, as well as their projec-
43. (B) The gray matter of the spinal cord is divided tions to the thalamus, are all GABAergic
into the 10 laminae of Rexed, which form a inhibitory. The glutaminergic corticostriate
cytoarchitectonic map of the spinal cord that cor- fibers excite a select population of striatal effer-
relates well with synaptic connections and neu- ent neurons that project to the internal globus
rophysiological data. Laminae I, II, III, and IV pallidus and the substantia nigra pars reticu-
encompass most of the dorsal horn, which lata. These striatal efferents, using GABA and
receives primary sensory fibers. Lamina I corre- substance P as a neurotransmitter, inhibit the
sponds to the nucleus postmarginalis, lamina II spontaneous firing of the internal globus pal-
corresponds to the substantia gelatinosa, and lidus and substantia nigra pars reticulata effer-
laminae III and IV correspond to the nucleus pro- ents to the thalamus. These latter projections
prius dorsalis. All these nuclei integrate and mod- inhibit cortical relay neurons in the dorsal thal-
ulate sensory information. They relay sensory amus. Inhibition of these inhibitory neurons in
information to higher centers like the cerebellum, the internal globus pallidus and substantia
thalamus, and brainstem. (Afifi and Bergman, 4950) nigra pars reticulata leads to a disinhibition of
the thalamocortical projections and an increase
44. (B) The striatum is the main receiving station in cortical activity. (Afifi and Bergman, 185188;
for the basal ganglia. It receives massive pro- Brazis, Masdeu, and Biller, 421424)
30 1: Anatomy and Physiology of the Central and Peripheral Nervous System

46. (A) In the indirect loop, linking the cerebral peduncle contains one cerebellar afferent path-
cortex to the basal ganglia, cortical fibers proj- way, the ventral spinocerebellar tract, which
ect to the striatum, and striatal efferent neu- carries proprioceptive information to the cere-
rons project to the external globus pallidus. bellum from the lower extremities and trunk.
Efferents from the external part of the globus (Burt, 352)
pallidus project to the subthalamic nucleus.
Neurons in the subthalamic nucleus project to 48. (B) Cerebellar nuclei are the principal source of
both the internal globus pallidus and the sub- efferent fibers from the cerebellum projecting to
stantia nigra pars reticulata. The internal the dorsal thalamus, vestibular nuclei, red
globus pallidus and the substantia nigra pars nucleus, and other brainstem nuclei. The den-
reticulata project to the dorsal thalamus. The tate nucleus receives projections from Purkinje
dorsal thalamus projects to the cerebral cortex. cells in the cerebrocerebellum and collaterals
The glutaminergic corticostriate fibers excite a from some of the pontocerebellar fibers. Fibers
specific population of striatal efferent neurons from the dentate nucleus enter the brachium
that project to the external globus pallidus. conjunctivum in the superior cerebellar pedun-
These striatal efferent neurons are GABAergic, cle, cross at the level of the inferior colliculus,
with enkephalin as a cotransmitter. They and terminate in the contralateral ventral
inhibit neurons on the external globus pallidus. nucleus of the thalamus. (Burt, 352354)
The external globus palliduss projection to the
subthalamic nucleus has a high rate of sponta- 49. (B) The cerebellar cortex contains three lami-
neous firing and is inhibitory. Neurons of the nated cellular layers: the outermost molecular
subthalamic nucleus are excitatory and gluta- cell layer, a sheet of single large neurons, the
minergic; they project to both the internal Purkinje cell layer, and a deeper granular cell
globus pallidus and the substantia nigra pars layer. These layers contain six types of neu-
reticulata. Because of the high spontaneous rons: basket, satellite, Purkinje, Golgi, granule
firing rate of the inhibitory neurons in the exter- cells, and the relatively rare Legato cells.
nal globus pallidus, the excitatory effects of the Pyramidal cells are the most abundant cells of
subthalamic nucleus on neurons in the internal the cerebral cortex neuron types, are not found
globus pallidus and substantia nigra pars retic- in the cerebellum, and are the most character-
ulata are normally minimal. However, when istic of the cerebral cortex. (Afifi and Bergman,
the activity of the indirect loop increases, there 203; Burt, 354)
is a disinhibition of the subthalamic nucleus.
The increased rate of firing of the subthalamic 50. (B) The monoaminergic projections to the cere-
nucleuss neurons in the internal globus pal- bellum originate from the pontine raphe nuclei,
lidus and substantia nigra pars reticulata the locus ceruleus, and the hypothalamus. The
results in inhibition of the thalamic relay neu- raphe nuclei are the source of serotoninergic
rons. A corresponding decrease in the level of projections to both the granular and molecular
cortical activity occurs. (Afifi and Bergman, layers. The locus ceruleus is the source of nora-
185188; Brazis, Masdeu, and Biller, 421424) drenergic projection to the three layers of the
cerebellar cortex. The dorsomedial, dorsal, and
47. (A) The cerebellum communicates with the lateral areas of the hypothalamus are the
brainstem through three pairs of massed pro- sources of histaminergic projections to all three
jection fibers called cerebellar peduncles: the layers of the cerebellar cortex. (Afifi and Bergman,
superior cerebellar peduncle, the middle cere- 214215)
bellar peduncle, and the inferior cerebellar
peduncle. The superior cerebellar peduncle 51. (E) The emboliform nucleus receives projec-
contains most of the cerebellar efferent fibers tions from the Purkinje cells in the spinocere-
and all those arising from the dentate nucleus, bellum and collaterals from the fibers entering
the emboliform nucleus, and the globose the restiform body and ventral spinocerebellar
nucleus. In addition, the superior cerebellar tract. Fibers from the emboliform nucleus enter
Answers: 4658 31

the brachium conjunctivum, decussate to the a single cell layer. Their axons project primarily
contralateral side, and terminate in both the to the cerebellar nuclei, although a few exit the
contralateral ventral nucleus of the thalamus cerebellum and terminate directly in the
and the red nucleus. (Burt, 352354) vestibular nuclei. The Purkinje cell axon is the
primary route for information leaving the cere-
52. (A) The learning of complex motor tasks bellar cortex. Each Purkinje cell axon courses
requires modifying motor responses or through the granule cell layer and deep white
sequences in order to adapt the responses to a matter to project onto deep cerebellar nuclei.
new situation or changes in the surrounding However, some Purkinje cell axons from the
conditions. A major component of this learning vermis bypass the deep cerebellar nuclei to
ability resides in the cerebellum and in the reach the lateral vestibular nucleus. Mossy
olivocerebellar climbing fiber system. Selective fibers, climbing fibers, parallel fibers, and
damage to this system results in loss of the abil- monoaminergic fibers are afferent projections
ity to modify a motor response and the ability to to the cerebellum. Golgi cells are inhibitory
maintain or store a modified response. (Burt, interneurons in which the cell axons branch
363) profusely in the granular layer and synapse
with dendrites of a large number of granule
53. (C) The fastigial nucleus receives axons of the cells, forming a negative feedback loop. (Burt,
Purkinje cells in the vestibulocerebellum. It 450451)
projects primarily to the lateral and inferior
vestibular nuclei and to the pontine and 57. (A) The hypothalamus plays a major role in
medullary reticular formation. (Burt, 352354) regulating eating behavior. The ventromedial
nucleus is located in the tuberal region of the
54. (A) The tuberoinfundibular tract is an efferent hypothalamus. Animal studies have demon-
hypothalamic pathway. It arises from the strated that bilateral lesions of the ventromedial
arcuate and periventricular nuclei. Axons from nuclei of the hypothalamus cause hyperphagia,
these neurons extend into the infundibular obesity, and savage behavior, whereas lesions
stalk of the neurohypophysis, where they end. of the lateral hypothalamus produce loss of
(Afifi and Bergman, 271) appetite. The supraoptic nucleus belongs to the
suprachiasmatic region and is located above
55. (C) Climbing fibers are axons of neurons orig- the optic tract. With the paraventricular
inating from the contralateral inferior olivary nucleus, the supraoptic nucleus is responsible
nucleus that project to all areas of the cerebel- for the secretion of vasopressin and oxytocin.
lar cortex. Climbing fibers are excitatory. Lesions of the paraventricular nucleus or the
Aspartate is the most likely transmitter for supraoptic nucleus cause diabetes insipidus.
these fibers. Each single climbing fiber estab- The anterior nucleus is located in the suprachi-
lishes 1,000 to 2,000 synaptic contacts with its asmatic region. Stimulation of this nucleus may
Purkinje cell. When the climbing fibers fire, cause excessive water intake. The arcuate
there is a massive synchronous depolarization nucleus is located in the tuberal region. This
of Purkinje cells, which activates Ca2 chan- nucleus contains dopamine, which is responsi-
nels in the dendritic membrane. The major ble for the control of prolactin and the secretion
source of climbing fibers in the cerebellum is of growth hormone. The mammillary nucleus
the inferior olive. Degeneration of the inferior plays a role in memory. (Afifi and Bergman, 274)
olive (as seen in olivocerebellar atrophy)
induces a drop in aspartate level in the cere- 58. (A) General somatic efferent fibers of the ocu-
brospinal fluid. (Afifi and Bergman, 211; Burt lomotor nerve arise from the oculomotor
359) nucleus, situated near the midline of the mid-
brain at the level of the superior colliculus. This
56. (A) Purkinje cells are the largest cells in the nucleus is formed by subnuclei for each of the
central nervous system. Their cell bodies form extraocular muscles. The superior rectus
32 1: Anatomy and Physiology of the Central and Peripheral Nervous System

muscle receives innervation from neurons in mation. The motor nucleus of the trigeminal
the contralateral subnucleus. The levator palpe- nerve provides somatic visceral efferents that
bral superioris muscle receives innervation innervate the muscles of mastication via the
from a medial subnucleus. The inferior rectus, mandibular division and contains and motor
medial rectus, and inferior oblique muscles neurons. (Afifi and Bergman, 117118)
receive innervation from ipsilateral subnuclei.
(Burt, 403406) 62. (C) Purkinje cells are found in the cerebellum.
They constitute the sole output neurons of the
59. (D) General somatic efferents provide the cerebellar cortex. Their cell bodies are arranged
motor innervation of somatic structures devel- in a single sheet at the border zone between the
oped from the embryonic ectoderm and molecular and the granule cell layers. Their
somatic mesoderm. The oculomotor nucleus axons project primarily to cerebellar nuclei,
provides general somatic efferent innervation although some axons from the vermis bypass
to all extraocular muscles except the lateral rectus the deep cerebellar nuclei to reach the vestibu-
(which is innervated by the abducens nerve) and lar nuclei directly. (Afifi and Bergman, 203204)
the superior oblique (which is innervated by the
trochlear nerve). The hypoglossal nucleus pro- 63. (D) The cerebral neocortex has a laminar pat-
vides general somatic efferents to the tongue tern of organization because of the distribution
musculature. The facial motor nucleus provides and size of neuronal cells and the horizontal
special visceral efferents to the muscles of facial pattern of incoming afferents. It is divided into
expression and the stapedius muscle. (Burt, 404) six layers: Layer I, primarily a synaptic area, is
the molecular layer. It is the most superficial
60. (C) The glossopharyngeal nerve provides paras- layer of the cerebral cortex; its most characteris-
ympathetic innervation to the parotid gland via tic cells are horizontal cells. Layer II, the external
the otic ganglion. (Afifi and Bergman, 9192) granular layer, is characterized by an abundance
of small, densely packed neurons and a paucity
61. (B) The trigeminal nerve has three sensory of myelinated fibers. The dendrites of neurons in
nuclei: the spinal nucleus, main sensory nucleus, this layer project to layer I, while their axons
and mesencephalic nucleus. The spinal nucleus project to deeper layers. Layer III, the external
of the trigeminal nerve is a long column of neu- pyramidal layer, contains medium-to-large
rons extending from the point of entry of the pyramidal cells and granule cells. Axons of most
trigeminal nerve to the upper cervical spinal pyramidal cells descend through the cortex,
cord. It is divided into three parts: the oral part, forming cortical association fibers, both callosal
responsible for tactile sensation from the oral and intrahemispheric. Layer IV, the internal
mucosa; the interpolar part, receiving afferents granular layer, is the principal receiving station
for dental pain; and the caudal part, receiving of the cerebral cortex. Layer V, the internal
pain and temperature sensations from the face. pyramidal layer, is the principal efferent layer of
Most of the small afferent fibers of the spinal the cortex. This layer contains pyramidal cells
tract of the trigeminal nerve terminate in the that send their axons through the cortical white
spinal nucleus. Most of the afferent large fibers matter to the internal capsule and all subcortical
that originate from the trigeminal ganglion end sites except the thalamus, which receives fibers
in the main sensory nucleus and are responsible from layer VI. Layer VI, the fusiform layer, con-
for the transmission of discriminative touch. The tains fusiform and pyramidal cells, which are the
mesencephalic nucleus is located at the rostral principal source of corticothalamic fibers and con-
pons. It receives afferent fibers conveying kines- tribute to the intrahemispheric cortical association
thesia and pressure from the teeth, periodon- fibers. (Afifi and Bergman, 230232; Burt, 451452)
tium, hard palate, joint capsules, and stretch
receptors from the muscles of mastication. It 64. (D) Layer IV of the cerebral cortex, the inter-
sends efferent fibers to the cerebellum, thalamus, nal granular layer, is the principal receiving
motor nuclei of the brainstem, and reticular for- station of the cortex. The input from the
Answers: 5971 33

modality-specific thalamic nuclei projects the patient to make an informed decision. The
mainly onto neurons in lamina IV, with some emotional deficit is characterized by bizarre and
projections on laminae III and IV. The nonspe- socially unacceptable behavior. The patient has a
cific thalamocortical input originating from labile and unpredictable emotional status. (Burt,
nonspecific thalamic nuclei projects diffusely 466468)
on all laminae and establishes mostly axoden-
dritic types of synapses. (Afifi and Bergman, 67. (D) The lateral geniculate body receives a dual
230232; Burt, 451452) arterial supply: from the anterior choroidal
artery laterally and the lateral posterior
65. (D) There are at least six neurochemically dis- choroidal artery medially. (Brazis, Masdeu, and
tinct extrathalamic projection systems that Biller, 140)
reach the cerebrum monosynaptically, without
a relay in the thalamus: three arise from the 68. (D) The orbicularis oculi controls eye closure
brainstem reticular formation, two from the and is innervated by the facial nerve. Eye open-
hypothalamus, and one from the basal fore- ing is controlled by the levator of the lid, which
brain. The first system arises from the locus is innervated by the oculomotor nerve. (Brazis,
ceruleus of the pontine reticular formation, Masdeu, and Biller, 287290)
using norepinephrine. The second system
arises from the midbrain raphe nuclei, using 69. (D) The nervus intermedius is the sensory and
serotonin. The third system arises from the ven- parasympathetic division of the facial nerve. It
tral midbrain, using dopamine. The fourth and carries preganglionic parasympathetic fibers
fifth systems arise from two hypothalamic to the submaxillary ganglion and to the ptery-
nuclei, using histamine and GABA, respec- gopalatine ganglion. It receives sensory fibers
tively, as neurotransmitters. From the nucleus from the geniculate ganglion. This ganglion
basalis, located in the basal forebrain, arises a receives fibers that carry taste sensation from
cholinergic extrathalamic modulatory system. the anterior two thirds of the tongue and affer-
The basal forebrain contains four populations ents from the mucosae of the pharynx, nose,
of cholinergic neurons with projection to the and palate. (Afifi and Bergman, 112)
cerebral cortex, with the nucleus basalis of
Meynert as the principal source of cholinergic 70. (D) The intracranial portion of the facial nerve
neurons. (Burt, 459463) is supplied by the anteroinferior cerebellar
artery. The intrapetrosal portion is supplied by
66. (B) Damage to the right posterior parietal lobe the superficial branch of the middle meningeal
causes neglect of the left side of the body. This artery and the stylomastoid branch of the pos-
neglect may have several dimensions: sensory, terior auricular artery. The extracranial part of
motor, cognitive, and attentional. In addition to the facial nerve is supplied by the stylomas-
the primary auditory function, the whole tem- toid, posterior auricular, superficial temporal,
poral cortex is involved in associative function. and transverse facial arteries. (Brazis, Masdeu,
Temporal lobe damage may lead to difficulties in and Biller, 290)
the performance of visually cued tasks requiring
a high degree of visual discrimination. Temporal 71. (C) The first-order neurons of the auditory
lobe lesions may also cause prosopagnosia, an pathway have their cell bodies in the spinal
inability to recognize familiar faces. This disorder ganglion of the cochlear nerve and enter the
is caused by impairment of some of the path- brainstem at the level of the ventral cochlear
ways responsible for visual processing. The nuclei as the cochlear nerve. The second-order
patient is still able to recognize family and friends neurons arise from the ventral and dorsal
from the sound of their voices. Lesions of the cochlear nuclei and send several projections to
prefrontal association cortex may cause prob- the contralateral brainstem; these ascend as the
lem-solving and emotional deficits. A problem- lateral lemniscus. Fibers in the lateral lemniscus
solving deficit is characterized by the inability of project on the nucleus of the lateral lemniscus
34 1: Anatomy and Physiology of the Central and Peripheral Nervous System

and then to the inferior colliculus. The inferior peripheral nervous system just after the latter
colliculus contains the third-order neurons of become committed to myelination. In the periph-
the auditory pathway. These neurons project to eral nervous system, conduction is invariably
the medial geniculate body. Geniculotemporal blocked when the myelin is stripped from the
fibers, the fourth-order neurons of the audi- entire internode, but it can be restored by only
tory pathway, project to the primary auditory minimal glial ensheathment. This restoration
cortex. (Brazis, Masdeu, and Biller, 307310) of conduction is likely related to the early
sodium channel clustering that accompanies
72. (C) The glossopharyngeal nerves travel initial steps in remyelination. As myelination
through the jugular foramen with the vagus proceeds, Na channels are initially found in
nerve and the bulbar fibers of the spinal acces- broad zones within gaps between neighboring
sory nerve. (Burt, 420423) oligodendroglial processes and then are con-
densed into focal clusters. This process appears
73. (A) Glomus jugulare tumors or basal skull to depend on the formation of axoglial junc-
fractures may cause jugular foramen syn- tions. It has been suggested that juxtaparan-
drome. The glossopharyngeal, vagus, and odal potassium channels may serve to inhibit
spinal accessory nerves may be injured in this repetitive activation of nodal sodium chan-
syndrome. Clinical signs include ipsilateral nels. Sodium channel expression is increased
trapezius and sternocleidomastoid weakness, in demyelinated lesions in multiple sclerosis.
dysphonia, dysphagia, depressed gag reflex, (Rasband and Shrager, 6373)
ipsilateral loss of taste in the ipsilateral poste-
rior third of the tongue, ipsilateral vocal cord 76. (A) The myelin sheaths of the central and
paresis, and anesthesia of the posterior third of peripheral nervous system contain distinct sets
the tongue. There is no tongue deviation, since of proteins. In the peripheral nervous system,
the twelfth cranial nerve is not affected. (Brazis, the noncompact myelin contains E-cadherin,
Masdeu, and Biller, 327) myelin-associated glycoprotein (MAG), and
connexin 32 (C32). The compact myelin in the
74. (B) Paramedian and circumferential vessels peripheral nervous system contains protein 0
supply the midbrain. The paramedian vessels (P0), peripheral myelin protein 22 (PMP22), and
arise from the posterior cerebral arteries and myelin basic protein (MBP). In the central nerv-
include the thalamoperforating and peduncu- ous system, the compact myelin contains prote-
lar arteries, which supply the medial peduncles olipid protein (PLP), oligodendrocyte-specific
and midbrain tegmentum. The circumferential protein (OSP), myelin-oligodendrocyte basic
arteries include the quadrigeminal arteries protein, and myelin basic protein. (Arroyo and
(which supply the superior and inferior colli- Scherer, 118)
culi), the superior cerebellar arteries (which
supply the cerebral peduncles and brachium 77. (A) Myelination in the central nervous system
conjunctivum), and the posterior and anterior differs from that in the peripheral nervous
choroidal arteries. (Brazis, Masdeu, and Biller, 361) system in several ways. Oligodendrocytes are
responsible for myelination in the central nerv-
75. (C) Na and K channel localization and clus- ous system, whereas Schwann cells are respon-
tering are essential for proper electrical signal sible for myelination in the peripheral nervous
generation and transmission in central nerv- system. Both oligodendrocytes and Schwann
ous systems myelinated nerve fibers. In par- cells ensheathe multiple axons; however, each
ticular, Na channels are clustered at high Schwann cell is responsible for the myelina-
density at nodes of Ranvier, and Shaker-type K tion of only one axon. Each oligodendrocyte
channels are sequestered in juxtaparanodal makes multiple myelin sheaths. The number
zones, just beyond the paranodal axoglial junc- varies from tract to tract and appears to relate
tions. There is strong evidence that Schwann to the caliber of the axons. Oligodendrocytes
cells initiate sodium channel clustering in the make fewer sheaths in tracts containing large
Answers: 7283 35

myelinated fibers; this is the result of axooligo- groups I, II, III, and IV. The thicker the diame-
dendrocyte interactions rather than an intrin- ter of the axon and its myelin, the faster the
sic trait of the oligodendrocytes themselves. conduction velocity. Class I axons are large and
Oligodendrocytes do not have a basal lamina heavily myelinated. Classes II and III are pro-
or microvilli, and their incisures do not have gressively smaller and less myelinated. Class
distinguishing molecular markers such as con- IV axons are smallest and unmyelinated. Class I
nexin 32 (C32), myelin-associated glycopro- is divided into subclasses Ia and Ib; the faster Ia
tein (MAG), or E-cadherin. The molecular fibers supply muscle spindles and the slower Ib
components of the central nervous system fibers supply Golgi tendon organs. (Dumitru, 17)
myelin sheaths partially overlap with those of
the peripheral nervous system. Both contain 81. (E) Neurulation is brought about by morpho-
high amounts of lipids, especially cholesterol logical changes in the neuroblasts, the imma-
and sphingolipids, including galactocerebro- ture and dividing future neurons.
side and sulfatide. Similarly, in both the central Microfilaments in each cell form a circular
nervous system and the peripheral nervous bundle parallel to the future laminar surface,
system, compact myelin contains myelin basic whereas microtubules extend along the length
protein (MBP), and the adaxonal surface con- of the cell. Colchicine may stop neurulation by
tains MAG. Like myelinating Schwann cells, inhibiting microfilament-based contraction or
oligodendrocytes also express C32, but by depolymerizing microtubules. (Haines, 72)
mainly on their cell bodies and proximal
processes. Oligodendrocytes express two pro- 82. (D) Secondary neurulation induces the for-
teins that are not expressed by Schwann cells: mation of the neural canal, which is open to the
myelin-oligodendrocyte glycoprotein (MOG) amniotic cavity both rostrally and caudally. In
on their outer cell membrane and myelin- the rostral opening, the anterior neuropore
oligodendrocyte basic protein (MOBP) in the closes at about 24 days; in the caudal opening,
major dense line of compact myelin. (Arroyo the posterior neuropore closes about 2 days
and Scherer, 118) later. (Haines, 7374)

78. (A) All glial cells express class I MHC, but only 83. (A) Congenital malformations associated with
astrocytes and microglia express class II MHC. defective neurulation are called dysraphic
All glial cells express adhesion molecules and defects. There is an intimate relationship between
synthesize cytokines. Both astrocytes and the neural tissue and the surrounding bone,
microglial cells express costimulatory molecules meninges, muscle, and skin. They are interde-
(B7) and complement components. They may pendent via inductive factors, so failure of neu-
act as antigen-presenting cells. (Antel, Birnbaum, rulation also impairs the formation of these
and Hartung, 2931) surrounding structures. Most dysraphic disor-
ders occur at either the anterior or posterior
79. (B) The plasma membrane of a nerve controls neuropores. Failure of the anterior neuropore to
ion transport, so that sodium and chloride are close causes anencephaly. In this disorder, the
more concentrated outside the cell than inside it, brain is not formed, the surrounding meninges
whereas potassium and organic anions are rela- and skull may be absent, and there may be
tively more concentrated inside the cell. The inte- associated facial abnormalities. Failure in the
rior of the cell ends up with a relative excess of closure of the posterior neuropore causes a
negative charges, so a voltage difference exists range of malformations known collectively as
across the cell membrane. This voltage difference myeloschisis. These defects always involve a
is called the resting potential; in a typical neuron failure of the vertebral arches at the affected
it has a value of about 70 mV. (Dumitru, 8) levels to form completely and fuse to cover the
spinal cord. This defect is called spina bifida.
80. (A) Sensory axons are grouped on the basis of Spina bifida may be accompanied by a saccu-
their diameters and myelin thickness into lar structure that contains only meninges and
36 1: Anatomy and Physiology of the Central and Peripheral Nervous System

cerebrospinal fluid. This defect is called a 88. (C) Neuroblasts arise from the ventricular sur-
meningocele. If the saccular structure contains face of the developing brain, which is the lumi-
meninges, cerebrospinal fluid, and spinal nal surface of the neural tube. (Haines, 81)
neural tissue, it is called a meningomyelocele.
Myelodysplasia is a malformation of the neural 89. (A) In the brainstem, the dorsal portion of the
tube during secondary neurulation, such as a neural tube rotates dorsolaterally as the devel-
tethered cord syndrome, in which the conus oping cerebellum invades it. The central canal
medullaris and filum terminale are abnormally of the mesencephalon invaginates into the
fixed to a defective vertebral column. (Haines, 72) fourth ventricle. This results in a lateral-to-
medial orientation of the sensory area, which
84. (B) Prosencephalization is the process whereby is the alar plate, versus the motor area of the
the forebrain vesicle differentiates into dien- developing brainstem, which is the basal plate.
cephalon and telencephalon. Failure of this The basal plates in the brainstem give rise to
process results in a holoprosencephaly. (Haines, cranial nerve motor nuclei, whereas the alar
74) plates give rise to cranial nerve sensory nuclei.
(Haines, 8182)
85. (A) The process of forebrain development into
diencephalon and telencephalon is referred to 90. (A) The mammillary body is located in the
as central induction and occurs mainly in the posterolateral part of the hypothalamus. The
second month of gestation. The adult telen- mammillary body receives afferents from the
cephalon derivatives include the cerebral mediotemporal lobe through the fornix, from
cortex, the subcortical white matter (including the midbrain tegmental nuclei through the
the internal capsule), the olfactory bulb and mammillary peduncle, and from the septal
tract, the basal ganglia, the amygdala, and the nuclei through the medial forebrain bundle.
hippocampus. (Haines, 74) (Burt, 384386)

86. (E) The diencephalon develops into the thala- 91. (D) The thalamus is the largest component of
mic nuclei and associated structures. The ven- the diencephalon. It is divided by a band of
tricular system is an elaboration of the lumen of myelinated fibers, the internal medullary
the cephalic portions of the neural tube. The lamina, into the rostrocaudal and the medio-
cavities of the telencephalon become the lat- lateral group of nuclei. The anterior nucleus of
eral ventricles, the diencephalic cavity becomes the thalamus consists of a large principal
the third ventricle, the rhombencephalic cavity nucleus and two smaller nuclei. The cells of
becomes the fourth ventricle, and the mesen- this nucleus receive dense limbic-related pro-
cephalic cavity becomes the narrow cerebral jections from the mammillary nuclei via the
aqueduct of Sylvius. The cerebral cortex and mammillothalamic tract and the medial tem-
the olfactory bulb are derivatives of the telen- poral lobe via the fornix. The output of this
cephalon. (Haines, 74) nucleus is primary directed to the cingulated
gyrus through the anterior limb of the internal
87. (A) Defects in the closure of the posterior neu- capsule. (Haines, 233; Martin, 40)
ropore cause a range of malformations known
collectively as myeloschisis. The defect always 92. (A) The dorsomedial nucleus of the thalamus
involves a failure of the vertebral arch at the is formed by a dorsomedial magnocellular divi-
affected levels to form completely and fuse to sion, a dorsolateral parvicellular division, and
cover the spinal cord. If the skin covering the a paralaminar division. It has a reciprocal con-
defect is closed, the malformation is called nection with the prefrontal cortex via the ante-
spina bifida occulta. If the skin over the verte- rior thalamic peduncle and with the frontal eye
bral defect is not closed, leaving a a patent fields. It also receives inputs from the temporal
aperture, the malformation is called spina neocortex, amygdaloid nucleus, substantia
bifida aperta. (Haines, 7273) nigra pars reticulare, and adjacent thalamic
Answers: 8499 37

nuclei, particularly the lateral and intralami- sia. Anencephaly is caused by failure of the
nar groups. The dorsomedial nucleus plays a anterior neuropore to close. Prosencephaly results
role in control of eye movement, affective from failure of the prosencephalon to undergo
behavior, decision making and judgment, cleavage and differentiate into the diencephalic
memory, and the integration of somatic and and telencephalic vesicles. (Haines, 7476)
visceral activity. (Haines, 233; Afifi and Bergman,
158159) 96. (B) Heterotopia is a defect in migration of
immature neurons from the ventricular surface
93. (B) The medial geniculate body carries audi- to the cerebral cortex. This defect causes mature
tory information from the brachium of the infe- neurons to appear in the intermediate zones.
rior colliculus and sends efferents to the The degree of disruption varies from mild
transverse temporal gyrus of Heschl. The lat- microscopic clusters of neurons in the white
eral geniculate body is a relay station for the matter and deeper cortical layers to large
visual pathway, which receives afferents from macroscopic clusters of neurons that can be
the retinal ganglion cells; these are the axons seen grossly and on neuroimaging. (Haines, 84)
that form the optic tract and from which the
axons that project to the calcarine cortex 97. (D) In addition to containing the glossopha-
through the optic radiation originate. (Brazis, ryngeal nerve, the vagus nerve, and the spinal
Masdeu, and Biller, 399401) accessory nerve, the jugular foramen also
serves as a conduit for other important struc-
94. (B) The process of infolding of the neural tube tures. The jugular foramen may be subdivided
from the neural plate is called primary neuru- into three compartments, each with its own
lation. Secondary neurulation occurs in the cau- contents: The anterior compartment transmits
dalmost portion of the neural tube, which will the inferior petrosal sinus. The intermediate
give rise to sacral and coccygeal levels of the compartment transmits the glossopharyngeal,
cord. Congenital malformations associated vagus, and accessory nerves. The posterior
with defective neurulation are called dysraphic compartment transmits the sigmoid sinus and
defects. Most of them occur at the location of some meningeal branches from the occipital and
the anterior or posterior neuropore. Failure of ascending pharyngeal arteries. (Haines, 217218)
the anterior neuropore to close results in anen-
cephaly. In this defect, the brain is not formed 98. (C) The thalamic arteries arise from the poste-
and the surrounding meninges and skull may rior communicating arteries and the perimes-
be absent. (Haines, 72) encephalic segment of the posterior cerebral
arteries. The polar arteries arise from the pos-
95. (A) Myelodysplasia refers to malformations of terior communicating arteries and supply the
the part of the neural tube formed by second- reticular, ventral, and medial anterior nuclei.
ary neurulation. A common malformation of The posteromedian and posterolateral choroidal
sacral and coccygeal levels of the cord is teth- arteries as well as the thalamogeniculate and thal-
ered cord syndrome, in which the conus amomesencephalic arteries originate from the
medullaris and filum terminale are abnormally posterior cerebral artery and are responsible for
fixed to the defective vertebral column. The the vascular supply of the thalamus. (Afifi, 166)
sustained traction damages the cord, with sub-
sequent loss of sensations from the legs and 99. (D) The Papez circuit is a closed circuit starting
feet and problems with bladder control. and ending in the hippocampus. It is thought to
Congenital hydrocephalus is caused by a pre- play a role in emotional reactions. The circuit
natal obstruction of the cerebral aqueduct. consists of outflow of impulses from the hip-
Patients with the DandyWalker malformation pocampus, fornix, mammillary body, mam-
have enlarged ventricles, including cystic millothalamic tract, anterior and dorsomedial
dilatation of the fourth ventricle accompanied thalamic nuclei, cingulate gyrus, and cingu-
by a variable degree of cerebellar vermis apla- lum. (Haines, 505506)
38 1: Anatomy and Physiology of the Central and Peripheral Nervous System

100. (A) The ventral posterolateral and the ventral lamina. The ventral posterolateral nucleus
posteromedian nuclei of the thalamus receive receives ascending input from the medial lem-
somatosensory input from the contralateral niscus, whereas the ventral posteromedial
side of the body. The medial lemniscus and the nucleus receives input from the trigeminothal-
spinothalamic fibers terminate in a somatosen- amic tract. The ventral posterolateral nucleus
sory manner (cervical fibers medial, sacral has two populations of identified neurons. The
fibers lateral) within the ventral posterolateral first population consists of large-diameter mul-
nucleus of the thalamus, whereas the trige- tipolar cells that give rise to axons traversing
minothalamic fibers from the spinal trigemi- the posterior limb of internal capsule and ter-
nal nucleus and the principal trigeminal minate mainly in the primary and secondary
sensory nucleus terminate in the ventral pos- somatosensory cortices. These thalamocortical
teromedian nucleus of the thalamus. Both the cells and fibers send excitatory glutaminergic
ventral posteromedian and the ventral pos- input to the cortex. The second population con-
terolateral nuclei of the thalamus project to the sists of inhibitory GABAergic local circuit
somatosensory cortex of the parietal lobe. interneurons. (Haines, 270)
(Haines, 234235)
105. (A) Most of the somatosensory information
101. (D) The preoptic nucleus of the thalamus is from craniofacial structures, including the oral
involved in the control of body temperature and nasal cavities, is transmitted to the brain-
and the heat loss mechanism. Nuclei of the chi- stem over the trigeminal nerve. The cell bodies
asmatic region are generally involved in regu- of the trigeminal primary afferent neurons are
lating hormone release (preoptic, supraoptic, located in the trigeminal semilunar (gasserian)
and periventricular nuclei), cardiovascular ganglion and in the mesencephalic trigeminal
function (anterior), and circadian rhythms nucleus. The central processes of trigeminal
(suprachiasmatic). The ventromedial nucleus ganglion cells form the large sensory root of
of the thalamus is regarded as the satiety center the trigeminal nerve as they enter the lateral
governing food intake. (Haines, 238239) aspect of the pons. Within the brainstem, cen-
tral processes of most trigeminal ganglion cells
102. (A) Brodmann area 6 is part of the frontal bifurcate into ascending and descending
cortex in the human brain. The current best evi- branches before terminating on second-order
dence in humans supports the view that the neurons in the brainstem trigeminal sensory
frontal eye field is primarily in Brodmann area nuclei. (Haines, 270)
6. Situated just anterior to the primary motor
cortex, it is composed of the premotor cortex 106. (D) The central target of nocieptive primary
and, medially the supplementary motor area. afferent fibers includes laminae I, II, and V of
Brodmann area 6 projects nuclei in the mid- the posterior horn. Rexed lamina I, the pos-
brain and pons that, in turn, project into the teromarginal nucleus, receives mainly input
oculomotor, trochlear, and abducens nuclei con- from A fibers. Neurons in this nucleus project
trolling eye movement. (Haines, 248) to other spinal cord laminae, the brainstem
reticular formation, and the thalamus. Lamina
103. (D) The anterior spinal artery supplies the V neurons receive both noxiouis and nonnox-
medial lemniscus in the medulla; penetrating ious input and project to the medullary and
branches of the basilar artery supply in the mesencephalic reticular formation, the thala-
pons. (Haines, 269) mus, and the hypothalamus. (Haines, 286)

104. (B) The ventral posterior nucleus is composed 107. (B) Information originating from nociceptors is
of the laterally located ventral posterolateral conducted almost exclusively by sympathetic
nucleus and the medially located ventral pos- nerves. In contrast, input originating from
teromedial nucleus. These two nuclei are sep- physiological receptors travels primarily in
arated from each other by fibers of the arcuate parasympathetic nerves. Most sympathetic
Answers: 100115 39

afferent fibers are either unmyelinated or thinly innervate the sphincter muscle of the iris and
myelinated and therefore are slowly conduct- the ciliary muscle. (Haines, 480)
ing fibers. Cell bodies of origin of sympathetic
fibers are located in the posterior root ganglia 112. (E) The preganglionic parasympathetic fibers
at about levels T1 to L2 The central processes of of the facial nerve originate in the superior sali-
these fibers enter the spinal cord via the lateral vary nucleus. They exit the brainstem in
division of the posterior root. They may ascend the intermediate nerve. Some of these fibers
or descend one or two spinal levels in the pos- course via the greater petrosal nerve to termi-
terolateral fasciculus before termining in lam- nate in the pterygopalatine ganglion, which
inae I and V and/or laminae VII and VIII. supplies the lacrimal gland and nasal and
(Haines, 304) palatal mucous glands. Other preganglionic
fibers travel via the chorda tympani to the sub-
108. (B) The posteroinferior cerebellar artery orig- mandibular ganglion, which innervates the
inates from the vertebral artery and supplies submandibular and sublingual salivary glands.
the posterolateral medulla, including the resti- (Haines, 480)
form body, the choroid plexus of the fourth
ventricle, and the caudomedial regions of the 113. (A) The anterior nuclear group of the thala-
inferior cerebellar surface, including the mus consists of two nuclei: principal anterior
vermis. (Haines, 435) and anterodorsal. The anterior nuclear group of
the thalamus has reciprocal connections with
109. (C) The afferent fibers to the cerebellar cortex the mammillary body and the cingulate gyrus.
are grouped into three types: the climbing It also receives a significant input from the hip-
fibers, the mossy fibers, and multilayered pocampal formation of the cerebral cortex via
fibers. The cerebellar afferent axons that end as the fornix. (Afifi and Bergman, 158)
mossy fibers originate from cell bodies in the
cerebellar nuclei and from other nuclei of the 114. (B) The dorsomedial nucleus of the thalamus
spinal cord and the brainstem. In the cerebellar develops in parallel with the prefrontal cortex
granular layer, mossy fibers synapse with the and is reciprocally connected with it via the
granule cells to form the mossy fibers rosettes. anterior thalamic peduncle and the frontal eye
Mossy fibers utilize gluconate as their neuro- fields. It also receives input from the temporal
transmitter and are excitatory to the granule neocortex, amygdaloid nucleus and substan-
layer cells. (Haines, 439440) tia nigra pars reticulate, and adjacent thalamic
nuclei particularly the lateral and intra-laminar
110. (B) The inferior olivary nuclei are the only groups. (Afifi and Bergman, 158)
source of cerebellar afferent axons that end as
climbing fibers in the cerebellar cortex. 115. (A) The medial geniculate nucleus is part of
Climbing fibers are afferent cerebellar fibers the auditory thalamus and represents the
that terminate in the molecular layer to synapse thalamic relay between the inferior collicu-
with the dendritic trees of Purkinje cells. Each lus and the auditory cortex. It is made up of
Purkinje cell is innervated by a single climbing a number of subnuclei that are distinguished
fiber, but olivocerebellar axons may branch to by their neuronal morphology and density,
serve several Purkinje cells. Climbing fibers their afferent and efferent connections, and
use aspartate as a neurotransmitter and excite the coding properties of their neurons. The
Purkinje cells and cerebellar nuclear neurons. auditory fibers reach the medial geniculate
(Haines, 440) nucleus via the brachium of the inferior col-
liculus. The medial geniculate nucleus receives
111. (D) The visceral motor component of the ocu- feedback from the primary auditory cortex in
lomotor nerve arises from the EdingerWestphal the temporal lobe. The efferent outflow from
nucleus. These preganglionic fibers terminate the medial geniculate nucleus forms the audi-
in the ciliary ganglion. Postganglionic axons tory radiation of the internal capsule to the
40 1: Anatomy and Physiology of the Central and Peripheral Nervous System

primary auditory cortex in the temporal lobe. via the cingulum and functions as an integral
(Afifi and Bergman, 163) part of the limbic system, which is involved
with the formation and processing of emotion,
116. (E) Five types of cells are distributed in the learning, and memory. (Afifi and Bergman,
three cortical layers of the cerebellum. Basket 283284)
and stellate cells are located in the molecular
layer, Purkinje cells are located in the Purkinje 120. (E) The internal carotid arteries arise at the
cell layer, and granule and Golgi cells are bifurcation of the common carotid artery in the
located in the granule cell layer. Of these five neck, ascend in front of the transverse
types, the Purkinje cell constitutes the principal processes of the upper three cervical vertebrae,
neuron of the cerebellum, since it is the only and enter the base of the skull through the
cerebellar neuron that sends its axon outside carotid canal. Within the cranium, the internal
the cerebellum. All the other cells are intrinsic carotid artery lies in the cavernous sinus. It
neurons and establish connections within the then pierces the dura to begin its subarachnoid
cerebellum. Purkinje cells send inhibitory pro- course. The internal carotid artery give rise to
jections to the deep cerebellar nuclei and con- the ophtalmic, anterior choroidal, anterior
stitute the sole output of all motor coordination cerebral, middle cerebral, and posterior com-
in the cerebellar cortex. (Afifi and Bergman, municating branches. The ophthalmic artery is
203204) the first intracranial branch of the internal
carotid as it courses through the cavernous
117. (E) This is a schematic diagram showing the sinus. The ophthalmic artery supplies the optic
four regions of the medial hypothalamus. The nerve and the central artery of the retina. (Afifi
structure indicated by the arrow is the mam- and Bergman, 349)
millary body. The mammillary bodies are two
spherical masses protruding from the ventral 121. (B) The recurrent artery of Heubner arises from
surface of the hypothalamus caudal to the the anterior cerebral artery either proximal or
tuber cinereum and rostral to the interpedun- distal to the anterior communicating artery. It
cular fossa and the anterior perforated sub- supplies the anterior limb and genu of the inter-
stance. The mammillary bodies consist of two nal capsule and parts of the head of the caudate,
groups of nuclei, medial and lateral. The rostral putamen, and globus pallidus. (Afifi and
medial nucleus is the main target of the fornix Bergman, 350)
and the source of the mammillothalamic tract.
(Afifi and Bergman, 269271) 122. (C) Figure 1-10 shows the lateral surface of
cerebral hemisphere and brainstem and a por-
118. (B) Figure 1-8 is a ventral view of the brain tion of the spinal cord. The structure indicated
showing components of the rhinencephalon. by the arrow is the central sulcus. It is a promi-
The structure indicated by the arrow is the nent landmark of the brain, separating the pari-
olfactory bulb, the main relay station in the etal lobe from the frontal lobe and the primary
olfactory pathways. (Afifi and Bergman, 281) motor cortex from the primary somatosensory
cortex. (Martin, 14)
119. (A) Figure 1-9 is a midsagittal view of the brain
showing components of the limbic lobe. The 123. (E) The foramina of Monro are channels that
structure indicated by the arrow is the cingu- connect the paired lateral ventricles with the
late gyrus. A gyrus in the medial part of the third ventricle at the midline of the brain. As
brain, it partially wraps around the corpus cal- channels, they allow cerebrospinal fluid pro-
losum and is limited above by the cingulate duced in the lateral ventricles to reach the third
sulcus. It receives inputs from the anterior ventricle and then the rest of the brains ven-
nucleus of the thalamus and the neocortex as tricular system. The crescent-shaped interven-
well as from somatosensory areas of the cere- tricular foramina are located on the medial and
bral cortex. It projects to the entorhinal cortex inferior aspect of the lateral ventricles. Each
Answers: 116129 41

foramen is bounded by the fornix and thala- CSF in the adult is about 140 mL. The volume of
mus. (Martin, 19) the ventricles is about 25 mL. CSF is produced at
a rate of 0.2 to 0.7 mL per minute or 600 to 700 mL
124. (D) The structure indicated by the arrow in per day. The CSF is absorbed across the arachnoid
Figure 1-11 is the anterior spinal artery. As the villi into the venous circulation. Although the
blood vessel that supplies the anterior portion of chorid plexus is the main source of CSF, approx-
the spinal cord, it arises from branches of the imately one third of the CSF is generated by
vertebral arteries and, descending in front of the extrachoroidal sources. The capillaryastrocyte
medulla oblongata, unites with its fellow of the complex in the bloodbrain barrier (BBB) has
opposite side at the level of the foramen been implicated as an active producer of brain
magnum. The single trunk thus formed descends interstitial fluid. CSF from this extrachoroidal
on the front of the medulla spinalis and is rein- source enters the ventricular system through the
forced by a succession of small branches that ependymal cells, the ciliated cuboidal epithelial
enter the vertebral canal through the interverte- cells that line the ventricles. Another likely source
bral foramina. These branches are derived from of CSF is the ependyma lining the ventricles.
the vertebral artery and the ascending cervical (Martin, 9899; Johanson, c2008)
artery of the inferior thyroid artery in the neck;
from the intercostal arteries in the thorax; and 127. (B) Gustatory fibers innervating the taste buds
from the lumbar artery, iliolumbar artery, and enter the brainstem and collect in the solitary
lateral sacral arteries in the abdomen and pelvis. tract located in the dorsal medulla. The axon
They unite, by means of ascending and descend- terminals leave the tract and synapse on neu-
ing branches, to form a single anterior median rons in the surrounding solitary nucleus, which
artery, which extend as far as the lower part of is the first central nervous system relay for taste.
the medulla spinalis and continues as a slender (Martin, 211)
twig on the filum terminale. (Martin, 8586; Afifi
and Bergman, 67) 128. (A) The inferior salivary nucleus is a cluster of
neurons controlling the parasympathetic input
125. (E) The structure indicated by the arrow in to the parotid gland. It is one of the compo-
Figure 1-12 is the globus pallidus. It is a wedge- nents of the glossopharyngeal nerve. It con-
shaped nuclear mass located between the puta- tains parasympathetic preganglionic neurons
men and the internal capsule. It can be divided whose axons course in the glossopharyngeal
into two parts: the globus pallidus externa nerve and synapse on postganglionic neurons
(GPe) and the globus pallidus interna (GPi). in the otic ganglion. Parasympathetic postgan-
Both receive input from the caudate and puta- glionic neurons in the otic ganglion innervate
men and both are in communication with the the parotid gland, which secretes saliva.
subthalamic nucleus. It is the GPi, however, (Martin, 269)
that sends the major inhibitory output from
the basal ganglia back to the thalamus. The GPi 129. (C) Clarkes nucleus and the accessory cuneate
also sends a few projections to an area of the nucleus are the principal nuclei relaying
midbrain, presumably to assist in postural con- somatosensory information to the spinocere-
trol. (Martin, 91; Afifi and Bergman, 185) bellum. Clarkes nucleus is a small section of
gray matter located in lamina VII of the inter-
126. (B) The cerebrospinal fluid (CSF) is produced mediate zone of the spinal cord, which is found
from arterial blood by the choroid plexuses of ventral to the gracile and cuneate columns and
the lateral and fourth ventricles by a combined is involved in unconscious proprioception. It is
process of diffusion, pinocytosis, and active found at the level of the eighth cervical
transfer. The choroid plexus consists of tufts of segment to approximately the second lumbar
capillaries with thin fenestrated endothelial cells. segment on the spinal cord and relays
These are covered by modified ependymal cells somatosensory information from the lower
with bulbous microvilli. The total volume of limbs and trunk. Clarkes nucleus is the origin
42 1: Anatomy and Physiology of the Central and Peripheral Nervous System

of the dorsal spinocerebellar tract, which orbit through the superior orbital fissure,
ascends in the outermost portion of the ipsilat- between the two heads of the lateral rectus. The
eral lateral column to reach the cerebellum via superior branch of the oculomotor nerve sup-
the inferior cerebellar peduncle. (Martin, plies the superior rectus and levator palpebrae
315317) superioris. The inferior branch of the oculomotor
nerve supplies the medial rectus muscle, the infe-
130. (A) The climbing fibers originate entirely from rior rectus muscle, and inferior oblique. (Martin,
the inferior olivary nuclear complex. These 416)
fibers pass through the pons and enter the cere-
bellum, where they form synapses with the 134. (C) The structure indicated by the arrow in
deep cerebellar nuclei and Purkinje cells. Figure 1-16 is the abducens nerve. It leaves the
(Martin, 312313) brainstem at the junction of the pons and the
medulla, then entering the subarachnoid space
131. (A) The lateral sulcus (also called the Sylvian when it emerges from the brainstem. It runs
fissure) is one of the most prominent structures upward between the pons and the clivus and
of the human brain. It divides the frontal and then pierces the dura mater to run between the
parietal lobes above from the temporal lobe dura and the skull. At the tip of the petrous
below. It occurs in both hemispheres of the temporal bone, it makes a sharp turn forward
brain but is longer in the left hemisphere. The to enter the cavernous sinus. In the cavernous
lateral sulcus is one the earliest-developing sinus, it runs alongside the internal carotid
sulci of the human brain. It first appears around artery. It then enters the orbit through the supe-
the 14th gestational week. (Martin, 411; Chi, rior orbital fissure and innervates the lateral
8693) rectus muscle of the eye. (Martin, 418)

132. (C) The structure indicated by the arrow in 135. (E) The structure indicated by the arrow in
Figure 1-14 is the left mammillary body. It is a Figure 1-17 is the trochlear nerve. It emerges
small, round, paired cell group that protrudes from the dorsal aspect of the brainstem at the
into the interpeduncular fossa from the inferior level of the caudal mesencephalon, just below
aspect of the hypothalamus. It receives a major the inferior colliculus. It passes between the
bundle of hippocampal fibers from the fornix posterior cerebral artery and the superior cere-
and projects fibers to the anterior thalamic nuclei bellar artery and then pierces the dura just
and into the tegmentum of the brainstem. under the free margin of the tentorium cere-
(Martin, 414) belli, close to the crossing of the attached margin
of the tentorium and within millimeters of the
133. (B) The structure indicated by the arrow in posterior clinoid process. It enters the cavernous
Figure 1-15 is the oculomotor nerve. It arises sinus, where it is joined by the other two
from the anterior aspect of mesencephalon. On extraocular nerves (oculomotor and trochlear
emerging from the brain, the nerve is invested nerves), the internal carotid artery, and portions
with a sheath of pia mater. It passes between the of the trigeminal nerve. Finally, it enters the
superior and posterior cerebral arteries and then orbit through the superior orbital fissure and
pierces the dura mater anterior and lateral to the innervates the superior oblique muscle. (Martin,
posterior clinoid process, passing between the 420; Bisaria, 2935)
free and attached borders of the tentorium cere-
belli. It runs along the lateral wall of the cav- 136. (A) The structure indicated by the arrow in
ernous sinus and above the other orbital nerves, Figure 1-18 is the pineal gland. It is reddish-gray
receiving in its course one or two filaments from in color, located just rostrodorsal to the superior
the cavernous plexus of the sympathetic nervous colliculus and behind and beneath the stria
system and a communicating branch from the medullaris, between the laterally positioned thal-
ophthalmic division of the trigeminal nerve. It amic bodies. It is part of the epithalamus. (Martin,
then divides into two branches, which enter the 422)
Answers: 130137 43

137. (C) The structure indicated by the arrow in Chi JG, Dooling EC, Gilles FH. Gyral development of the
Figure 1-19 is the inferior cerebellar peduncle. human brain. Ann Neurol. 1997;1(1):86-93.
It carries many types of input and output fibers Dumitru D, Amato AA, Zwarts MJ. Electrodiagnostic
that are mainly concerned with integrating pro- Medicine. 2nd ed. Philadelphia: Hanley & Belfus; 2002.
Johanson CE, Duncan JA III, Klinge PM, Brinker T, Stopa
prioceptive sensory input with motor vestibu-
EG, Silverberg GD. Multiplicity of cerebrospinal fluid
lar functions, such as the maintenance of
functions: new challenges in health and disease.
balance and posture. (Martin, 442) Cerebrospinal Fluid Res. 2008;5:10.
Kline LB, Bajandas, FJ, eds. Neuroophthalmology Review
Manual. 6th ed. Thorofare, NJ: Slack; 2001.
REFERENCES Martin JH. Neuroanatomy. Text and Atlas. 3rd ed. New York:
McGraw-Hill; 2003.
Afifi AK, Bergman RA, eds. Functional Neuroanatomy: Text Parent A, ed. Carpenters Human Neuroanatomy. 9th ed.
and Atlas. 2nd ed. New York: McGraw-Hill; 2005. Media, PA: Williams & Wilkins; 1996.
Antel JP, Birnbaum G, Hartung HP, eds. Clinical Rasband MN, Shrager P. Ion channel sequestration in cen-
Neuroimmunology. Oxford, UK: Blackwell Science; 1998. tral nervous system axons. J Physiol. 2000;525(1):63-73.
Arroyo EJ, Scherer SS. On the molecular architecture of Staal A, Van Gijn J, Spaams F, eds. Mononeuropathies.
myelinated fibers. Histochem Cell Biol. 2000;113(1):1-18. Examination, Diagnosis and Treatment. London: Saunders;
Bisaria KK. Cavernous portion of the trochlear nerve with 1999.
special reference to its site of entrance. J Anat. 1988;159: Van der Werf YD, Witter MP, Groenewegen HJ. The
29-35. intralaminar and midline nuclei of the thalamus.
Brazis PW, Masdeu JC, Biller J, eds. Localization in Clinical Anatomical and functional evidence for participation in
Neurology. 5rd ed. London: Little, Brown; 2007. processes of arousal and awareness. Brain Res Rev.
Burt AM, ed. Textbook of Neuroanatomy. Philadelphia: 2002;39:107-140.
Saunders; 1993.
This page intentionally left blank
CHAPTER 2

Localization Signs in Neurology


Questions

1. Small irregular pupils fixed to light but reactive 5. A 45-year-old right-handed woman with a his-
to accommodation are suggestive of tory of mitral valve stenosis developed a pat-
tern of acute pressured speech and the inability
(A) HolmesAdie pupil
to follow simple commands. Her speech was
(B) ArgyllRobertson pupil profuse and filled with paraphasic errors. She
(C) epidemic encephalitis was unable to repeat sentences or name objects
(D) Parinaud syndrome correctly. The most likely location of a brain
(E) Marcus Gunn pupils lesion associated with her condition is the
(A) left frontal lobe
2. Postganglionic Horner syndrome may accom-
(B) left superior temporal gyrus
pany
(C) left inferior frontal gyrus
(A) ipsilateral internal artery dissection (D) bilateral medial occipital gyrus
(B) hypothalamic infarction (E) left cerebellar gyrus
(C) midbrain infaction
(D) breast cancer 6. A pure lesion of the dominant angular gyrus
(E) lung cancer causes
(A) Broca aphasia
3. A unilateral large, poorly reactive pupil is
(B) Wernicke aphasia
caused by
(C) alexia without agraphia
(A) botulism (D) alexia with agraphia
(B) traumatic carotid dissection (E) anosognosia
(C) third nerve palsy
(D) an acute pontine lesion 7. Alexia without agraphia is caused by damage
(E) delirium to the
(A) combined medial dominant occipital
4. Opsoclonus usually indicates a lesion of the region and contralateral splenium of the
(A) cerebellum corpus callosum
(B) visual cortex (B) dominant angular gyrus
(C) retina (C) nondominant angular gyrus
(D) optic nerve (D) bilateral lateral geniculate body
(E) lateral geniculate body (E) bilateral occipital cortex

45
46 2: Localization Signs in Neurology

8. Repetition is preserved in (C) alexia without agraphia


(A) conduction aphasia (D) tameness
(B) Broca aphasia (E) anosognosia
(C) Wernicke aphasia
14. The alien hand sign is seen in lesions of the
(D) global aphasia
(E) transcortical sensory aphasia (A) occipital cortex
(B) medial temporal lobe
9. Gerstmann syndrome is characterized by (C) hippocampus
(A) damage to the nondominant occipital (D) cingulate gyrus
cortex (E) parietal cortex
(B) dysarthria
15. Blunted affect associated with impotence and
(C) dressing apraxia
the inability to plan and execute multistepped
(D) visual anosognosia processes results from damage to the
(E) finger agnosia
(A) orbitofrontal area
10. Optic ataxia is (B) cingulate gyrus
(C) hippocampus
(A) characterized by failure to shift gaze on
command (D) parietal cortex
(B) defined by a disturbance of reaching a (E) corpus callosum
target under visual control
16. Lack of kinesthetic transfer associated with
(C) caused by cerebellar damage
double hemianopia results from a damage to
(D) usually accompanied by an optic nerve the
lesion
(E) a part of Gerstmann syndrome (A) orbitofrontal area
(B) cingulate gyrus
11. Anton syndrome (denial of blindness) results (C) hippocampus
from a (D) precentral gyrus
(A) bilateral lateral occipital lesion (E) corpus callosum
(B) bilateral mesial occipital lesion
17. Hemiballismus is caused by a lesion located in
(C) hippocampal lesion
the
(D) lesion of the cingulate gyrus
(E) callosal lesion (A) ipsilateral cerebellum
(B) contralateral caudate nucleus
12. Impaired ipsilateral scanning may result from (C) contralateral subthalamic nucleus
a (D) ipsilateral globus pallidus
(A) lateral occipital lesion (E) ipsilateral substantia nigra
(B) mesial occipital lesion
18. A unilateral lesion of the anteroventral portion
(C) lesion of the mesial frontal lobe
of the caudate nucleus causes contralateral
(D) lesion of the cingulate gyrus
(E) nondominant parietal lesion (A) tremor
(B) dystonia
13. Bilateral medial temporal damage may cause (C) parkinsonism
(A) sensory aphasia (D) choreoathetosis
(B) alexia with agraphia (E) hemiballismus
Questions: 826 47

19. A sudden bilateral paramedian thalamic lesion (D) tectal dysfunction


may cause (E) pontine dysfunction
(A) visual hallucinations
25. Damage to shaded area (indicated by the
(B) akinetic mutism
broken arrow) in Figure 2-1 results in
(C) hyperphagia
(D) hypersexual behavior (A) loss of pain and temperature in the con-
tralateral face
(E) amnesia
(B) contralateral ataxia
20. Prolonged latency of visually evoked saccadic (C) vertigo
eye movements results from (D) homolateral half of the tongue paralysis
(A) a lesion of the pulvenar (E) diplopia response
(B) a lesion of the lateral geniculate body
(C) optic radiation
(D) a lesion of the hypothalamus
(E) a lesion of the anterior nucleus of the
thalamus

21. CheyneStokes respiration implies


(A) medullary dysfunction
(B) abnormal reduction of ventilatory
response to CO2 FIG. 2-1. (Reproduced with permission from Afifi AK, Bergman RA. Functional
(C) a lower pontine lesion Neuroanatomy: Text and Atlas. 2nd ed. New York: McGraw-Hill; 2005.)

(D) midbrain lesion


(E) forebrain damage
26. Damage to the shaded area indicated by the
22. Apneustic breathing is most likely related to
broken arrow in Figure 2-2 can cause
(A) lower pontine tegmental dysfunction
(A) homolateral paralysis of half of the tongue
(B) damage to the basis pontis
(B) vertigo
(C) medullary dysfunction
(C) hiccups
(D) hypothalamic damage
(D) ipsilateral ataxia
(E) forebrain damage
(E) ipsilateral Horner syndrome
23. Medullary dysfunction may cause
(A) cluster breathing
(B) ataxic breathing
(C) apraxia for deep breathing
(D) apneustic breathing
(E) CheyneStokes respiration

24. Bilateral small (pinpoint) pupils in a comatose


patient are suggestive of
Xllth nerve
(A) midbrain dysfunction
(B) third nerve dysfunction
FIG. 2-2. (Reproduced with permission from Afifi AK, Bergman RA. Functional
(C) diencephalic dysfunction Neuroanatomy: Text and Atlas. 2nd ed. New York: McGraw-Hill; 2005.)
48 2: Localization Signs in Neurology

28. What is the earliest sign of damage to the area


of the brain indicated by arrow A in Figure
2-5?
(A) Deep coma
(B) Moderate anisocoria
(C) External ophthalmoplegia
(D) Fully dilated pupil nonreactive to light
(E) Decorticate posturing

C
FIG. 2-3. (Reproduced with permission from Afifi AK, Bergman RA. Functional
Neuroanatomy: Text and Atlas. 2nd ed. New York: McGraw-Hill; 2005.)

27. Which of the following clinical manifestations A


is newly developed by a 56-year-old man
patient when his central nervous system
damage extends from the dashed area in Figure
2-3 to the dashed area in Figure 2-4?
D
(A) Contralateral limb paralysis
(B) Ipsilateral paralysis of facial muscles
(C) Ipsilateral paralysis of ocular abduction
(D) Vertigo
(E) Dysphagia FIG. 2-5. (Reproduced with permission from Hauser SL, Josephson SA,
English JD, Engstrom JW (eds). Harrisons Neurology in Clinical Medicine.
New York: McGraw-Hill; 2006.)

29. What is the earliest sign of damage to the area


of the brain indicated by arrow B in Figure 2-5?
(A) Eupneic breathing with deep sighs and
yawns
(B) Stupor
(C) Decorticate posturing
(D) Hypothermia
(E) CheyneStokes respiration

30. In a 72-year-old comatose man with a large


frontal lobe hemorrhage, the occurrence of
ataxic breathing with a drop in blood pressure
and irregular heart rate indicate that the patient
has moved to the
(A) early diencephalic stage
FIG. 2-4. (Reproduced with permission from Afifi AK, Bergman RA. Functional
Neuroanatomy: Text and Atlas. 2nd ed. New York: McGraw-Hill; 2005.) (B) late diencephalic stage
Questions: 2734 49

(C) midbrain stage (C) dysarthria


(D) lower pontine stage (D) ipsilateral weakness of mastication
(E) medullary stage (E) ipsilateral limb weakness

31. Pupillary examination of 37-year-old comatose 34. A lesion affecting the dashed area in Figure 2-7
woman reveals pupils in midposition, unre- causes
sponsive to light with hippus. These findings
(A) ipsilateral loss of facial sensation
are suggestive of central nervous system
damage located in the (B) dysphagia
(C) dysarthria
(A) midbrain tectum
(D) ipsilateral weakness of mastication
(B) diencephalon
(E) ipsilateral facial muscle weakness
(C) midbrain tegmentum
(D) pons
(E) medulla

32. The occurrence of a long inspiratory pause,


after which the air is retained for several sec-
onds and then released in a comatose patient,
indicates damage located in the
(A) medulla
(B) pons
(C) cerebellum
(D) midbrain
(E) diencephalon

33. A lesion affecting the dashed area in Figure 2-6


causes
FIG. 2-7. (Reproduced with permission from Afifi AK, Bergman RA. Functional
Neuroanatomy: Text and Atlas. 2nd ed. New York: McGraw-Hill; 2005.)
(A) diplopia
(B) dysphagia

Corticospinal tract
FIG. 2-6. (Reproduced with permission from Afifi AK, Bergman RA. Functional
Neuroanatomy: Text and Atlas. 2nd ed. New York: McGraw-Hill; 2005.)
50 2: Localization Signs in Neurology

35. A lesion affecting the dashed area indicated by 37. A lesion affecting the dashed area indicated by
arrow A in Figure 2-8 causes arrow C in Figure 2-10 causes
(A) ipsilateral loss of facial sensation (A) ipsilateral loss of facial sensation
(B) dysphagia (B) contralateral tremor
(C) dysarthria (C) dysarthria
(D) ipsilateral dilated nonresponsive pupil (D) ipsilateral weakness of mastication
(E) ipsilateral tremor response (E) ipsilateral tremor

FIG. 2-8. (Reproduced with permission from Afifi AK, Bergman RA. Functional FIG. 2-10. (Reproduced with permission from Afifi AK, Bergman RA. Functional
Neuroanatomy: Text and Atlas. 2nd ed. New York: McGraw-Hill; 2005.) Neuroanatomy: Text and Atlas. 2nd ed. New York: McGraw-Hill; 2005.)

36. A lesion affecting the dashed area indicated by 38. A pretectal lesion causes
arrow B in Figure 2-9 causes
(A) upward gaze paralysis
(A) ipsilateral loss of facial sensation (B) ipsilateral tremor
(B) contralateral limb weakness (C) ipsilateral weakness of mastication
(C) dysarthria (D) ipsilateral loss of facial sensation
(D) contralateral dilated nonresponsive (E) dysarthria
pupil
(E) ipsilateral tremor 39. Infarction of the anteromedial branches of the
posterior cerebral artery causes
(A) ipsilateral ataxia
(B) ipsilateral oculomotor nerve palsy
(C) ipsilateral tremor
(D) hallucinations
(E) contralateral lid retraction

40. A 75-year-old woman developed hallucina-


tions of animals and people sharing her room.
B This patient may have brain damage located
in the
(A) midbrain
(B) medulla
FIG. 2-9. (Reproduced with permission from Afifi AK, Bergman RA. Functional
Neuroanatomy: Text and Atlas. 2nd ed. New York: McGraw-Hill; 2005.) (C) frontal cortex
Questions: 3549 51

(D) parietal cortex side of the upper arm down to the olecranon is
(E) hypothalamus suggestive of a
(A) first thoracic root lesion (T1)
41. Pain radiating into the arm and shoulder with
(B) fifth cervical root lesion (C5)
weakness of shoulder abduction is suggestive
of a (C) sixth cervical root lesion (C6)
(D) seventh cervical root lesion (C7)
(A) first thoracic root lesion (T1)
(E) eighth cervical root lesion (C8)
(B) fifth cervical root lesion (C5)
(C) sixth cervical root lesion (C6) 46. Proximal forearm pain exacerbated by elbow
(D) seventh cervical root lesion (C7) extension with muscle wasting in the ventral
(E) eighth cervical root lesion (C8) arm, weakness of elbow flexion, and loss of the
biceps reflex is suggestive of
42. Weakness of elbow flexion in the fully supine
(A) a radial nerve lesion at the spiral groove
and half-pronated positions associated with
(B) ulnar nerve entrapment at the wrist
deep aching pain spreading down the lateral
forearm to the thumb and index finger, affect- (C) a musculocutaneous nerve lesion
ing both the palmar and dorsal aspects of the (D) a median nerve lesion in the upper arm
hand, is suggestive of a (E) an axillary nerve lesion
(A) first thoracic root lesion (T1)
47. Saturday night palsy is suggestive of
(B) fifth cervical root lesion (C5)
(C) sixth cervical root lesion (C6) (A) a radial nerve lesion at the spiral groove
(D) seventh cervical root lesion (C7) (B) ulnar nerve entrapment at the wrist
(E) eighth cervical root lesion (C8) (C) a musculocutaneous nerve lesion
(D) a median nerve lesion in the upper arm
43. Weakness of shoulder adduction, elbow exten- (E) an axillary nerve lesion
sion, and flexion and extension of the wrist with
pain in the forearm, radiating into the middle, 48. Pain in the little finger and medial half of the
index, and ring fingers, is suggestive of a ring finger with weakness of all intrinsic hand
muscles except for thumb abduction is sug-
(A) first thoracic root lesion (T1)
gestive of
(B) fifth cervical root lesion (C5)
(C) sixth cervical root lesion (C6) (A) a radial nerve lesion at the spiral groove
(D) seventh cervical root lesion (C7) (B) ulnar nerve entrapment at the wrist
(E) eighth cervical root lesion (C8) (C) a musculocutaneous nerve lesion
(D) a median nerve lesion in the upper arm
44. Weakness of the long extensor and flexor mus- (E) an axillary nerve lesion
cles of the hand with pain in the olecranon,
radiating into the little and ring fingers, is sug- 49. Pain from the thumb to the middle finger and
gestive of a forearm with weakness of wrist flexion, thumb
abduction, and the inability to form an O with
(A) first thoracic root lesion (T1)
the thumb and index fingers (pinch sign) is
(B) fifth cervical root lesion (C5) suggestive of
(C) sixth cervical root lesion (C6)
(A) a radial nerve lesion at the spiral groove
(D) seventh cervical root lesion (C7)
(B) ulnar nerve entrapment at the wrist
(E) eighth cervical root lesion (C8)
(C) a musculocutaneous nerve lesion
45. Weakness of all intrinsic hand muscles with (D) a median nerve lesion in the upper arm
pain in the shoulder joint, axilla, and medial (E) an axillary nerve lesion
52 2: Localization Signs in Neurology

50. A 65-year-old woman developed a pain located sensations from her left lateral thigh spreading
diagonally across the thigh, with weakness on to the popliteal fossa. The most likely anatomic
hip flexion, knee extension, and thigh adduc- location of the damage is in the
tion. The most likely anatomic location of the
(A) lateral femoral cutaneous nerve
damage is in the
(B) sacral plexus
(A) lumbar plexus (C) obturator nerve
(B) sacral plexus (D) sciatic nerve
(C) obturator nerve (E) tibial nerve
(D) femoral nerve
(E) tibial nerve 55. A 66-year-old woman with a past medical his-
tory of breast cancer developed loss of sensa-
51. A 23-year-old pregnant woman developed pain tion in her left posterior thigh, lateral calf, and
in the medial right thigh and weakness of thigh dorsum of the foot, including all the toes.
adduction. The most likely anatomic location of Motor examination demonstrated weakness of
the damage is in the left hip extension, left thigh abduction, and all
left foot movements. The most likely anatomic
(A) lumbar plexus
location of the damage is in the
(B) sacral plexus
(C) obturator nerve (A) lateral femoral cutaneous nerve
(D) femoral nerve (B) sacral plexus
(E) tibial nerve (C) obturator nerve
(D) sciatic nerve
52. A 50-year-old man developed a progressive loss (E) lumbar plexus
of sensation to all modalities in the sole and lat-
eral border of the foot with weakness on plan- 56. A 71-year-old man on warfarin for severe car-
tarflexion and inversion of the foot. The most diomyopathy developed, after a fall on his
likely anatomic location of the damage is the back, severe pain in his anterior left thigh and
medial leg down to his ankle. Neurological
(A) lumbar plexus
assessment was significant for the absence of
(B) sacral plexus left knee jerk and weakness of knee extension.
(C) obturator nerve The most likely anatomic location of the
(D) femoral nerve damage is in the
(E) tibial nerve (A) femoral nerve
53. After an intramuscular injection in his left but- (B) sacral plexus
tock, a 56-year-old man developed a left flail (C) obturator nerve
foot, weakness of left knee flexion, decreased (D) sciatic nerve
left Achilles reflex, and loss of sensation in the (E) lumbar plexus
lateral left leg and dorsum of the foot. The most
likely anatomic location of the damage is in the 57. A 78-year-old woman developed loss of sensa-
tion in the dorsum of her foot and a dull ache
(A) femoral nerve
in the anterolateral leg after a left knee arthro-
(B) sacral plexus
plasty. Neurological examination demonstrated
(C) obturator nerve weakness on dorsiflexion and eversion of her
(D) sciatic nerve left foot. The most likely anatomic location of
(E) tibial nerve the damage is in the
(A) femoral nerve
54. A 22-year-old female began, late in her preg-
nancy, to complain of tingling and burning (B) peroneal nerve
Questions: 5066 53

(C) obturator nerve (D) optic nerve compression at the junction


(D) sciatic nerve with the optic chiasm
(E) tibial nerve (E) compression of the optic chiasm

58. A 22-year-old woman developed pain in her 63. Bitemporal hemianopia may be associated with
perineum and clitoris as well as fecal inconti- (A) cecocentral scotoma
nence. The most likely anatomic location of the
(B) binasal hemianopia
damage is in the
(C) optic neuritis
(A) femoral nerve (D) optic nerve compression at the junction
(B) sacral plexus with the optic chiasm
(C) obturator nerve (E) compression of the optic chiasm
(D) sciatic nerve
(E) pudendal nerve 64. Cortical blindness is associated with
(A) Anton syndrome
59. B12 deficiency is associated with
(B) binasal hemianopia
(A) cecocentral scotoma (C) optic neuritis
(B) binasal hemianopia (D) infarction of the anterior visual cortex
(C) optic neuritis (E) compression of the optic chiasm
(D) optic nerve compression at the junction
with the optic chiasm 65. Hemianopia with macular sparing is associ-
(E) compression of the optic chiasm ated with
(A) Anton syndrome
60. Hydrocephalus is associated with
(B) binasal hemianopia
(A) cecocentral scotoma (C) optic neuritis
(B) binasal hemianopia (D) infarction of the anterior visual cortex
(C) optic neuritis (E) compression of the optic chiasm
(D) optic nerve compression at the junction
with the optic chiasm 66. Which of the following is true about Bells phe-
(E) compression of the optic chiasm nomenon in Bells palsy?
(A) Eyeball deviation occurs up and slightly
61. A 23-year-old male treated with ethambutol
outward on the affected side when the
for tuberculosis may develop
patient attempts to close both eyes.
(A) cecocentral scotoma (B) Eyeball deviation occurs down and
(B) binasal hemianopia slightly inward on the affected side
(C) optic neuropathy when the patient attempts to close both
(D) optic nerve compression at the junction eyes.
with the optic chiasm (C) Eyeball deviation occurs down and
(E) compression of the optic chiasm slightly inward on the normal side when
the patient attempts to close both eyes.
62. Ipsilateral central scotoma with contralateral (D) Eyeball deviation occurs up and slightly
temporal visual field defect may be associated inward on the normal side when the
with patient attempts to close both eyes.
(E) Eyeball deviation is not a physiologic
(A) cecocentral scotoma
Bells phenomenon and is seen only in
(B) binasal hemianopia
peripheral facial nerve disease.
(C) optic neuritis
54 2: Localization Signs in Neurology

67. TolosaHunt syndrome and metacarpophalangeal extension. Right arm


extension was normal. Elbow flexion was
(A) is characterized by recurrent unilateral
weak, with the thumb pointing to the ceiling.
facial pain
The lesion is most likely located in the
(B) is characterized by transient facial nerve
palsy (A) radial nerve at the upper arm
(C) is caused by herpesvirus infection (B) radial nerve at the axilla
(D) is characterized by a sensitivity to corti- (C) musculocutaneous nerve
costeroids (D) axillary nerve
(E) has a female predominance (E) long thoracic nerve

68. A 28-year-old right-handed volleyball player 71. A 60-year-old man described progressive diffi-
has noticed, for the previous 3 weeks, mild right culty in extending his right little finger. This symp-
arm weakness on elevation, especially when tom progressed over several weeks to total
shaving or combing his hair. The weakness is inability to extend the fingers and thumb.
accompanied by a dull ache in the shoulder. On Neurological examination demonstrated dropped
neurological examination, there was winging fingers without wrist drop. The lesion is most
of the right scapula when the patient was asked likely located in the
to push against the wall with both arms. The
(A) radial nerve at the upper arm
lesion is most likely located in the
(B) radial nerve at the axilla
(A) C7-C8 cervical root (C) radial nerve at the forearm
(B) long thoracic nerve (D) radial nerve at the wrist
(C) suprascapular nerve (E) axillary nerve
(D) dorsal scapular nerve
(E) trapezius muscle 72. After he had been handcuffed by a police offi-
cer, a 30-year-old man developed a shooting
69. A 45-year-old right-handed porter used to keep pain on the radial side of his right wrist and
his arms outstretched and forearms in maxi- paresthesias radiating into the thumb and
mum supination when carrying heavy bags on index finger. The lesion is most likely located in
his back. He consulted a neurologist because of the
weakness on extension of his right elbow, wrist,
(A) radial nerve at the upper arm
and fingers with slightly decreased sensation to
all modalities in his lateral arm and posterior (B) radial nerve at the axilla
forearm and the web between the index finger (C) radial nerve at the forearm
and thumb. The lesion is most likely located (D) radial nerve at the wrist
in the (E) axillary nerve
(A) radial nerve at the upper arm
73. A 40-year-old woman, recently diagnosed with
(B) radial nerve at the axilla
non-Hodgkins lymphoma, noticed difficulty
(C) musculocutaneous nerve in holding a glass with her right hand and
(D) axillary nerve numbness of the palmar side of the thumb,
(E) long thoracic nerve index, and middle fingers of her right hand.
Neurological examination of the right hand
70. A 56-year-old alcoholic man awoke in the demonstrated weak pronation and abduction
morning after a heavy alcoholic binge com- of the wrist against resistance, weak flexion of
plaining of inability to extend his right wrist the proximal and distal interphalangeal joints
and fingers, including the thumb. Neurological against resistance of the second and third fin-
examination demonstrated weakness of wrist gers, and inability to form an O with the thumb
Questions: 6778 55

and index finger. The lesion is most likely fingers. After a few weeks, she noticed atro-
located in the phy of her nails in the two last fingers as her
hand became claw-like. Neurological exami-
(A) median nerve at the upper arm
nation demonstrated weakness on right wrist
(B) median nerve at the elbow abduction and adduction, decreased pinprick
(C) median nerve at the wrist sensation in the palmar surface of the right
(D) ulnar nerve at the elbow hand, and diminished dorsal sensation of the
(E) ulnar nerve at the wrist little finger and medial aspect of the ring
finger. There was weakness on flexion of the
74. A 60-year-old African-American man, a few little finger. The lesion is most likely located
days after reduction of a dislocated left elbow, in the
developed dull pain in his left forearm over (A) median nerve at the wrist
several days. The pain spread to his index
(B) anterior interosseous nerve
finger and thumb. Neurological examination
demonstrated normal sensation and wrist flex- (C) posterior interosseous nerve
ion. There was weakness in pronation of the (D) ulnar nerve at the elbow
forearm when the elbow was flexed but not (E) ulnar nerve at the wrist
when it was extended. The lesion is most likely
located in the 77. A 37-year-old man developed a history of right
hand weakness, mostly in the ring and little fin-
(A) median nerve at the upper arm
gers, over the preceding 6 weeks. Neurological
(B) anterior interosseous nerve examination demonstrated weakness on thumb
(C) posterior interosseous nerve adduction, little finger abduction, and flexion.
(D) ulnar nerve at the elbow There was mild hypothenar muscle atrophy. The
(E) ulnar nerve at the wrist right palmaris brevis was spared on clinical
examination. Sensory examination of the right
75. For the previous 4 months, a 45-year-old hand was normal. The lesion is most likely
woman with a history of rheumatoid arthritis located in the
had noticed an intermittent sensation of pins (A) median nerve at the wrist
and needles in both hands. The pain worsens
(B) anterior interosseous nerve
upon awakening from sleep. She has also
noticed clumsiness with fine finger move- (C) posterior interosseous nerve
ments. Neurological examination demon- (D) ulnar nerve at the elbow
strated abnormal pinprick sensation on the (E) ulnar nerve at the wrist
palmar surface of both hands, with hypesthe-
sia in the distal aspect of the first three digits 78. A 45-year-old woman had recently been started
bilaterally. There was mild thenar atrophy, on warfarin for atrial fibrillation. She devel-
weakness of thumb abduction, and weakness oped severe pain in her anterior right thigh
on opposing the thumb against the little finger. and difficulty walking and rising from a chair.
The lesion is most likely located in the Neurological examination demonstrated weak-
ness on right thigh and hip flexion as well as
(A) median nerve at the wrist
weakness on extension of the leg against resist-
(B) anterior interosseous nerve ance. The most likely diagnosis is
(C) posterior interosseous nerve
(A) right lower extremity embolism
(D) ulnar nerve at the elbow
(B) obturator nerve neuropathy
(E) ulnar nerve at the wrist
(C) femoral nerve neuropathy
76. A 40-year-old woman noticed a pins-and- (D) sciatic nerve neuropathy
needles sensation in her right ring and little (E) tibial nerve neuropathy
56 2: Localization Signs in Neurology

79. Lesions of the papillomacular bundle cause ness in closing the left eye. Neurological exam-
which of the following visual field defects? ination demonstrated a left abducens and facial
nerve palsy associated with left sensorineural
(A) Paracentral scotoma
deafness. This is suggestive of
(B) Wedge-shaped temporal scotoma
(C) Comma-shaped extension of the blind (A) MillardGubler syndrome (lesion of the
spot ventrocaudal pons)
(D) Bjerrum arcuate scotoma (B) Foville syndrome (lesion of the dorsal
pons)
(E) Nasal step of Ronne
(C) RaymondCestan syndrome (lesion of
80. Optic tract lesions result in the ventromedial pons)
(D) subarachnoid syndrome of the abducens
(A) congruous hemianopia cranial nerve
(B) unilateral atrophy of the retinal nerve (E) Gradenigo syndrome (lesion of the apex
fiber of the temporal bone)
(C) normal pupillary reflex
(D) Wernicke pupil 84. A 70-year-old woman with a history of hyper-
(E) decreased visual acuity tension experienced a sudden onset of left-
sided weakness with facial involvement.
81. A left homonymous superior quadranopsia Neurological examination demonstrated right
(pie in the sky) visual field defect is caused abducens nerve paresis, right facial paresis,
by a lesion of the and left hemiplegia. This is suggestive of
(A) left geniculate body (A) MillardGubler syndrome (lesion of the
(B) left optic tract ventrocaudal pons)
(C) right parietal lobe (B) Foville syndrome (lesion of the dorsal
pons)
(D) left parietal lobe
(C) RaymondCestan syndrome (lesion of
(E) right anterior temporal lobe
the ventromedial pons)
82. A 50-year-old woman with a history of pituitary (D) subarachnoid syndrome of the abducens
adenoma developed pain and paresthesias in cranial nerve
the right periorbital area. The neurological (E) Gradenigo syndrome (lesion of the apex
examination demonstrated right ophthalmo- of the temporal bone)
plegia with loss of sensation in the distribution
of the ophthalmic branch of the trigeminal 85. A 20-year-old woman with a history of pseudo-
nerve. This is suggestive of tumor cerebri developed an acute headache
and blurred vision. Neurological examination
(A) MillardGubler syndrome (lesion of the demonstrated bilateral papilledema and
ventrocaudal pons) abducens nerve palsy. This is suggestive of
(B) Foville syndrome (lesion of the dorsal
pons) (A) MillardGubler syndrome (lesion of the
ventrocaudal pons)
(C) RaymondCestan syndrome (lesion of
the ventromedial pons) (B) Foville syndrome (lesion of the dorsal
pons)
(D) subarachnoid syndrome of the abducens
cranial nerve (C) RaymondCestan syndrome (lesion of
the ventromedial pons)
(E) cavernous sinus syndrome
(D) subarachnoid syndrome of the abducens
83. Following an otitis media, a 36-year-old man cranial nerve
developed diplopia, left facial pain, and weak- (E) Gradenigo syndrome (lesion of the apex
of the temporal bone)
Questions: 7992 57

86. A 75-year-old woman with a history of hyper- (A) Ipsilateral superior rectus palsy
tension consulted a neurologist because of (B) Bilateral ptosis
acute left-sided weakness. Neurological exam- (C) Contralateral inferior oblique paresis
ination demonstrated horizontal conjugate
(D) Contralateral ptosis
gaze palsy with right trigeminal, facial, and
cochleovestibular nerve palsies and right (E) Contralateral medial rectus palsy
Horner syndrome. This is suggestive of
90. A 52-year-old woman with a history of
(A) MillardGubler syndrome (lesion of the migraine and hypertension developed a left
ventrocaudal pons) ptosis. Neurological examination demon-
(B) Foville syndrome (lesion of the dorsal strated left oculomotor nerve palsy without
pons) pupillary abnormality. The symptoms improved
(C) RaymondCestan syndrome (lesion of over the next 2 months. A follow-up visit in
the ventromedial pons) the third month showed an ArgyllRobertson
(D) subarachnoid syndrome of the abducens pupil on the left side during convergence. The
cranial nerve most likely cause of the pupillary abnormal-
ity is
(E) Gradenigo syndrome (lesion of the apex
of the temporal bone) (A) recurrence of left oculomotor palsy
(B) ischemic mononeuropathy of the oculo-
87. A 26-year-old male was brought to the motor nerve
Emergency Room because of a car accident (C) primary aberrant regeneration of the
with head trauma. Physical examination oculomotor nerve
demonstrated mastoid ecchymosis, otorrhea,
(D) secondary aberrant regeneration of the
hemotympanum and left trigeminal, abducens,
oculomotor nerve
and facial nerve palsy. This is suggestive of
(E) migraine
(A) Millard-Gubler syndrome (lesion of the
ventrocaudal pons) 91. A 40-year-old man developed a new onset of
(B) Cavernous sinus syndrome diplopia. Neurological examination demon-
(C) Raymond-Cestan syndrome (lesion of strated eye misalignment on vertical gaze (the
the ventro medial pons) right eye higher than the left eye). This wors-
(D) Petrous bone fracture ened on left gaze deviation and when the head
was tilted to the right. Which of the following
(E) Gradenigo syndrome (lesion of the apex
ocular muscles was affected?
of the temporal bone)
(A) Right superior oblique
88. A 70-year-old man with a history of diabetes (B) Left superior oblique
developed a left hemiparesis with right abducens (C) Right inferior rectus
nerve palsy. This is suggestive of
(D) Left inferior rectus
(A) MillardGubler syndrome (lesion of the (E) Right inferior oblique
ventrocaudal pons)
(B) cavernous sinus syndrome 92. A lesion at which of the following spinal cord
(C) RaymondCestan syndrome (lesion of segments causes inversion of the brachioradi-
the ventromedial pons) alis reflex?
(D) petrous bone fracture (A) C8
(E) Gradenigo syndrome (lesion of the apex (B) C4
of the temporal bone) (C) C7
(D) C6
89. Which of the following is most suggestive of a
lesion in the nucleus of the oculomotor nerve? (E) T1
58 2: Localization Signs in Neurology

93. A 35-year-old man developed progressive (D) amyotrophic lateral sclerosis


lower extremity weakness and gait ataxia over (E) tabes dorsalis
6 months. Neurological examination demon-
strated bilateral lower extremity spasticity, 96. A 45-year-old woman developed thermoanes-
increased deep tendon reflexes throughout, and thesia in a cape-like distribution involving both
bilateral Babinski signs. Sensory examination upper extremities, with preservation of light
showed no sensory level but loss of proprio- touch sensation and proprioception. These
ception and vibratory sensation in both legs findings are suggestive of
with preservation of temperature and pinprick
sensations. These findings are suggestive of (A) syringomyelia
(B) B12 deficiency
(A) BrownSquard syndrome
(C) occlusion of the anterior spinal artery
(B) syringomyelia
(D) amyotrophic lateral sclerosis
(C) B12 deficiency
(E) tabes dorsalis
(D) occlusion of the anterior spinal artery
(E) amyotrophic lateral sclerosis 97. A 40-year-old man developed a sudden onset
of back pain, followed by flaccid areflexic para-
94. A 50-year-old man had chronic lancinating leg plegia with urinary incontinence. Neurological
pain, urinary incontinence, and gait ataxia pro- examination showed loss of sensation to pain
gressing over 3 months. Neurological exami- and temperature at the T4 level with preserva-
nation demonstrated impaired vibratory and tion of vibration and proprioception. These
joint position sense in the lower extremities, findings are suggestive of
decreased tactile localization, and presence of
the Romberg sign. Examination of the feet (A) syringomyelia
showed chronic trophic changes. These findings (B) B12 deficiency
are suggestive of (C) occlusion of the anterior spinal artery
(A) syringomyelia (D) amyotrophic lateral sclerosis
(B) B12 deficiency (E) tabes dorsalis
(C) occlusion of the anterior spinal artery
98. Following back trauma, a 60-year-old woman
(D) amyotrophic lateral sclerosis developed weakness of the right lower extremity
(E) tabes dorsalis and urinary incontinence. Neurological exam-
ination demonstrated spastic monoplegia of
95. A 30-year-old man consulted the neurologist the right lower extremity, loss of vibration
because of generalized weakness and muscle sense, proprioception in the right side below
atrophy in the right hand and foot. These the T6 level, and loss of pain and temperature
symptoms had been progressing over the pre- sensation on the left side below the T6 level.
vious 2 years and were associated with painful These findings are suggestive of
cramps. Neurological examination demonstrated
explosive dysarthria, generalized spasticity, (A) BrownSquard syndrome
increased deep tendon reflexes throughout, and (B) syringomyelia
bilateral Babinski signs. There was prominent (C) B12 deficiency
muscle atrophy in the right hand and both feet, (D) occlusion of the anterior spinal artery
with fasciculations. Sensory examination was (E) amyotrophic lateral sclerosis
normal. Bladder and rectal sphincters were not
affected. These findings are suggestive of 99. Funnel vision is seen in
(A) syringomyelia (A) glaucoma
(B) B12 deficiency (B) hysteria
(C) occlusion of the anterior spinal artery (C) malingering
Questions: 93106 59

(D) pituitary tumor (E) a lesion of the facial nerve at the stylo-
(E) a lesion of the lateral geniculate body mastoid foramen

100. A retrochiasmatic lesion of which of the fol- 104. Ipsilateral facial palsy with loss of taste in the
lowing may cause a strictly unilateral visual anterior two thirds of the tongue and normal
field defect? hearing is associated with

(A) The anteriormost aspect of the calcarine (A) a fascicular lesion of the facial nerve
cortex (B) a lesion of the facial nerve distal to the
(B) The lateral geniculate body meatal canal and proximal to the nerve
(C) The optic radiation to the stapedius
(D) The medial occiput (C) a lesion of the facial nerve in the meatal
canal
(E) The optic tract
(D) a lesion of the facial nerve between the
101. Ipsilateral facial nerve palsy with normal audi- departure of the nerve to the stapedius
tory and taste function is caused by and the departure of the chorda tympani
(E) a lesion of the facial nerve at the stylo-
(A) a fascicular lesion of the facial nerve mastoid foramen
(B) a lesion of the facial nerve in the meatal
canal 105. Ipsilateral facial palsy, loss of taste sensation in
(C) a lesion of the geniculate ganglion the anterior two thirds of the tongue, and
(D) a lesion of the facial nerve between the hyperacusis are associated with
departure of the nerve to the stapedius (A) a fascicular lesion of the facial nerve
and the departure of the chorda tympani
(B) a lesion of the facial nerve distal to the
(E) a lesion of the facial nerve at the stylo- meatal canal and proximal to the nerve
mastoid foramen to the stapedius
(C) a lesion of the facial nerve esion in the
102. Foville syndrome (lesion of the dorsal pons) is
meatal canal
associated with
(D) a lesion of the facial nerve between the
(A) a fascicular lesion of the facial nerve departure of the nerve to the stapedius
(B) a lesion of the facial nerve in the meatal and the departure of the chorda tympani
canal (E) a lesion of the facial nerve at the stylo-
(C) a lesion of the geniculate ganglion mastoid foramen
(D) a lesion of the facial nerve between the
departure of the nerve to the stapedius 106. Ipsilateral facial palsy with deafness and loss of
and the departure of the chorda tympani taste in the anterior two thirds of the tongue are
(E) a lesion of the facial nerve at the stylo- associated with
mastoid foramen (A) a fascicular lesion of the facial nerve
(B) a lesion of the facial nerve distal to the
103. Ramsay Hunt syndrome is associated with meatal canal and proximal to the nerve
(A) a fascicular lesion of the facial nerve to the stapedius
(B) a lesion of the facial nerve in the meatal (C) a lesion of the facial nerve in the meatal
canal canal
(C) a lesion of the geniculate ganglion (D) a lesion of the facial nerve between the
(D) a lesion of the facial nerve between the departure of the nerve to the stapedius
departure of the nerve to the stapedius and the departure of the chorda tympani
and the departure of the chorda tympani (E) a lesion of the facial nerve at the stylo-
mastoid foramen
60 2: Localization Signs in Neurology

107. MillardGubler syndrome (lesion of the ven- (D) Foville syndrome (lesion of the dorsal
trocaudal pons) is associated with pontine tegmentum)
(A) a fascicular lesion of the facial nerve (E) MillardGubler syndrome (lesion of the
ventrocaudal pons)
(B) a lesion of the facial nerve distal to the
meatal canal and proximal to the nerve
110. The neurological assessment of a 65-year-old
to the stapedius
comatose patient demonstrated right facial
(C) a lesion of the facial nerve in the meatal weakness, left gaze deviation, left hemiplegia,
canal and left Babinski sign. These findings are asso-
(D) a lesion of the facial nerve between the ciated with
departure of the nerve to the stapedius
and the departure of the chorda tympani (A) medial medullary syndrome
(E) a lesion of the facial nerve at the stylo- (B) Wallenberg syndrome (lateral medullary
mastoid foramen syndrome)
(C) locked-in syndrome
108. A 70-year-old man with a history of atrial fib- (D) Foville syndrome (lesion of the dorsal
rillation developed a sudden onset of vertigo, pontine tegmentum)
nausea, vomiting, diplopia, and dysarthria. He (E) MillardGubler syndrome (lesion of the
also had pain in the right face and left arm and ventrocaudal pons)
leg. Neurological examination demonstrated
right Horner syndrome and decreased tem- 111. A 65-year-old woman with a history of diabetes
perature sensation in the painful areas. The and hypertension developed severe headache,
right palate and vocal cord were paralyzed. followed by a dysarthria that progressed to total
The right arm and leg were ataxic. These find- aphonia and generalized weakness. Neurological
ings are suggestive of examination found an awake and alert patient
(A) medial medullary syndrome with quadriparesis and ophthalmoplegia bilat-
erally with sparing of vertical eye movement and
(B) Wallenberg syndrome (lateral medullary
blinking. These findings are suggestive of
syndrome)
(C) locked-in syndrome (A) medial medullary syndrome
(D) Foville syndrome (lesion of the dorsal (B) Wallenberg syndrome (lateral medullary
pontine tegmentum) syndrome)
(E) MillardGubler syndrome (lesion of the (C) locked-in syndrome
ventrocaudal pons) (D) Foville syndrome (lesion of the dorsal
pontine tegmentum)
109. A 50-year-old woman with a history of dia- (E) MillardGubler syndrome (lesion of the
betes developed left-sided weakness and ventrocaudal pons)
dysarthria. Neurological examination demon-
strated tongue deviation to the right side, left 112. A 30-year-old man with a history of cocaine
hemiplegia, and loss of vibratory and position abuse developed a sudden onset of ataxia and
sensation in the left arm and leg with preser- left-sided weakness. Neurological examination
vation of temperature and pain sensation. demonstrated right arm ataxia, left hemiparesis,
These findings are suggestive of and loss of temperature and pain sensation in
(A) medial medullary syndrome the left arm. These findings are associated with
(B) Wallenberg syndrome (lateral medullary (A) MarieFoix syndrome (lateral pontine
syndrome) lesion)
(C) locked-in syndrome (B) Weber syndrome (lesion of the medial
cerebral peduncle)
Questions: 107117 61

(C) Benedict syndrome (lesion of the mesen- (A) MarieFoix syndrome (lateral pontine
cephalic tegmentum) lesion)
(D) Sylvian aqueduct syndrome (B) Weber syndrome (lesion of the medial
(E) MillardGubler syndrome (lesion of the cerebral peduncle)
ventrocaudal pons) (C) Benedict syndrome (lesion of the mesen-
cephalic tegmentum)
113. A 40-year-old woman with a history of diabetes (D) Sylvian aqueduct syndrome
developed a sudden onset of diplopia and left- (E) MillardGubler syndrome (lesion of the
sided tremor. Neurological examination demon- ventrocaudal pons)
strated right ophthalmoplegia and left intention
tremor. These findings are associated with 116. A 5-year-old boy was brought to the neurology
(A) MarieFoix syndrome (lateral pontine clinic because of the insidious onset of a
lesion) staggering gait. Neurological examination
(B) Weber syndrome (lesion of the medial demonstrated axial ataxia without limb ataxia
cerebral peduncle) and spontaneous nystagmus. MRI of the head
showed a cerebellar mass suggesting medul-
(C) Benedict syndrome (lesion of the mesen-
loblastoma. These findings are suggestive of
cephalic tegmentum)
(D) Sylvian aqueduct syndrome (A) rostral vermis syndrome
(E) MillardGubler syndrome (lesion of the (B) caudal vermis syndrome
ventrocaudal pons) (C) posteroinferior cerebellar artery occlu-
sion syndrome
114. An 80-year-old man with a history of hyper- (D) anteroinferior cerebellar artery occlusion
tension consulted the neurologist because of a syndrome
new onset of double vision and left-sided (E) superior cerebellar artery occlusion syn-
weakness. Examination demonstrated right drome
oculomotor paresis with a dilated pupil and
left hemiplegia. These findings are associated 117. A 70-year-old diabetic woman developed a
with sudden onset of dizziness, nausea, vomiting,
(A) MarieFoix syndrome (lateral pontine right-sided ataxia, and right hearing loss.
lesion) Cranial nerve examination revealed sen-
(B) Weber syndrome (lesion of the medial sorineural deafness on the right, peripheral
cerebral peduncle) facial palsy, and loss of facial pain and tem-
perature sensation. The rest of the neurological
(C) Benedict syndrome (lesion of the mesen-
examination demonstrated sensory loss to pain
cephalic tegmentum)
and temperature in the left trunk, arm, and leg
(D) Sylvian aqueduct syndrome and right Horner syndrome. These findings are
(E) MillardGubler syndrome (lesion of the suggestive of
ventrocaudal pons)
(A) rostral vermis syndrome
115. A 20-year-old man consulted the neurologist (B) caudal vermis syndrome
because of his chronic headache and blurred (C) posteroinferior cerebellar artery occlu-
vision. Neurological examination demon- sion syndrome
strated paralysis of upward gaze and conver- (D) anteroinferior cerebellar artery occlusion
gence retraction nystagmus on upward gaze. syndrome
Magnetic resonance imaging (MRI) of the head (E) superior cerebellar artery occlusion syn-
showed a pineal tumor with hydrocephalus. drome
These findings are associated with
62 2: Localization Signs in Neurology

118. A 50-year-old man with a history of ethanol (D) anteroinferior cerebellar artery occlusion
abuse consulted the neurologist because of pro- syndrome
gressive exacerbation of gait ataxia and slurred (E) superior cerebellar artery occlusion syn-
speech. Neurological examination demonstrated drome
mild dysarthria, axial ataxia with minimal ataxia
on the heel-to-shin maneuver, and normal arm 121. The most common location of neurogenic gas-
coordination. These findings are suggestive of trointestinal ulceration after an acute hypo-
thalamic lesion is the
(A) rostral vermis syndrome
(B) caudal vermis syndrome (A) upper esophagus
(C) posteroinferior cerebellar artery occlu- (B) lower esophagus
sion syndrome (C) fundus of the stomach
(D) anteroinferior cerebellar artery occlusion (D) ileum
syndrome (E) colon
(E) superior cerebellar artery occlusion syn-
drome 122. The lesion most consistently associated
with memory disturbance in patients with
119. A 60-year-old man with a history of hyperten- WernickeKorsakoff syndrome has been found
sion developed a sudden onset of vertigo and in the
gait disturbance. Neurological examination
(A) mammillary bodies
demonstrated right Horner syndrome with
horizontal nystagmus, left sensorineural deaf- (B) pulvinar
ness, right limb ataxia, and intention tremor. (C) medial dorsal nucleus of the thalamus
Sensory examination was significant for left- (D) fornix
sided pain and temperature loss. These find- (E) ventromedial region of the hypothalamus
ings are suggestive of
123. Hemiballismus occurs with damage to the
(A) rostral vermis syndrome
(B) caudal vermis syndrome (A) internal capsule
(C) posteroinferior cerebellar artery occlu- (B) subthalamic nucleus
sion syndrome (C) pituitary gland
(D) anteroinferior cerebellar artery occlusion (D) substantia nigra
syndrome (E) hypothalamus
(E) superior cerebellar artery occlusion syn-
drome 124. Sensory inattention occurs most commonly
with a lesion of the
120. A 75-year-old woman with a history of hyper-
(A) inferior parietal lobe
tension and atrial fibrillation developed acute
vertigo, headache, dysarthria, and gait distur- (B) thalamus
bance. Neurological examination was signifi- (C) mesencephalic reticular formation
cant for left limb ataxia, facial loss of temperature (D) dorsolateral frontal lobe
sensation, vocal cord palsy, Horner syndrome, (E) cingulate gyrus
and temperature and pain loss in the right
trunk, arm, and leg. These findings are sug- 125. Lesion of the septal region may induce
gestive of
(A) sensory aprosodia
(A) rostral vermis syndrome (B) indifference to pain
(B) caudal vermis syndrome (C) blunt affect
(C) posteroinferior cerebellar artery occlu- (D) rage reaction
sion syndrome (E) depression
Questions: 118136 63

126. An orbitofrontal lesion may cause 132. Anger and hostility may occur with
(A) sensory aprosodia (A) right orbitofrontal lesions
(B) indifference to pain (B) right parietotemporal lesions
(C) blunt affect (C) bilateral anterior temporal lesions
(D) rage reaction (D) bilateral anterior cingulate lesions
(E) impulsive behavior (E) left dorsofrontal lesions

127. Apathy with indifference and psychomotor 133. Paranoid behavior may occur with
retardation is associated with lesions of the
(A) bilateral anterior temporal lesions
(A) septal region (B) bilateral anterior cingulate lesions
(B) orbitofrontal lesion (C) left dorsofrontal lesions
(C) frontal convexity (D) left temporal lobe lesions
(D) medial frontal cortex (E) right temporal lesions
(E) cingulate lesion
134. Which of the following hallucination types is
128. Mutism with gait disturbance is associated associated with damage to the neocortex of the
with lesions located in the temporal lobe?

(A) cingulate (A) Nocturnal bright-colored figures with a


(B) medial frontal lobe cartoon-like appearance
(C) lateral frontal lobe (B) Predominantly black-and-whitecolored
linear zigzags
(D) parietal lobe
(C) Predominantly multicolored patterns
(E) temporal lobe
(D) Pleasant dream-like visual hallucinations
129. Sensory aprosodia is associated with (E) Dj vcu illusions (illusions of previous
experiences)
(A) right orbitofrontal lesions
(B) right parietotemporal lesions 135. Which of the following hallucination types is
(C) bilateral anterior temporal lesions associated with occipital seizures?
(D) bilateral anterior cingulate lesions (A) Nocturnal bright-colored figures with a
(E) left dorsofrontal lesions cartoon-like appearance
(B) Predominantly black-and-whitecolored
130. Bland affect is associated with linear zigzags
(A) right orbitofrontal lesions (C) Predominantly multicolored patterns
(B) right parietotemporal lesions (D) Pleasant dream-like visual hallucinations
(C) bilateral anterior temporal lesions (E) Dj vcu illusions
(D) bilateral anterior cingulate lesions
136. Which of the following hallucination types is
(E) left dorsofrontal lesions
associated with midbrain lesions?
131. Indifference to pain may occur with (A) Nocturnal bright-colored figures with a
cartoon-like appearance
(A) right orbitofrontal lesions
(B) Predominantly black-and-whitecolored
(B) right parietotemporal lesions
linear zigzags
(C) bilateral anterior temporal lesions
(C) Predominantly multicolored patterns
(D) bilateral anterior cingulate lesions
(D) Pleasant dream-like visual hallucinations
(E) left dorsofrontal lesions
(E) Dj vcu illusions
64 2: Localization Signs in Neurology

137. Which of the following hallucination types is 141. Which of the following is characteristic of the
associated with the Charles Bonnet syndrome? frontal alien limb syndrome of the hand?
(A) Nocturnal bright-colored figures with a (A) Occurrence exclusively in the nondomi-
cartoon-like appearance nant hand
(B) Predominantly black-and-whitecolored (B) Caused by hemispheric disconnection
linear zigzags (C) Involving the compulsive manipulation
(C) Predominantly multicolored patterns of tools
(D) Pleasant dream-like visual hallucinations (D) Intermanual conflict
(E) Dj vcu illusions (E) Apraxia

138. Which of the following hallucination types is 142. Limb-kinetic apraxia is caused by a lesion in
associated with migraine? the
(A) Nocturnal bright-colored figures with a (A) perirolandic cortex
cartoon-like appearance (B) mesial frontal cortex
(B) Predominantly black-and-whitecolored (C) supplementary motor cortex
linear zigzags (D) parietal cortex
(C) Predominantly multicolored patterns (E) corpus callosum
(D) Pleasant dream-like visual hallucinations
(E) Dj vcu illusions 143. Somatosensory disturbance is associated with
damage located in the
139. Which of the following is characteristic of
(A) mesial occipital lobe
Balint syndrome?
(B) lateral occipital lobe
(A) Gaze apraxia (C) bilateral anterior tip of the temporal lobe
(B) Denial of blindness (D) lateroinferior aspect of the nondominant
(C) Agraphia temporal lobe
(D) Color agnosia (E) parietal postcentral gyrus
(E) Finger agnosia
144. Impaired saccade with pure agraphia is asso-
140. A 67-year-old man with a history of diabetes ciated with
and hypertension suddenly developed a severe
(A) a lesion of the parietal postcentral gyrus
headache and blurred vision on the left. Neuro-
logical examination demonstrated a left homo- (B) a lesion of the mesial parietal lobe
nymous hemianopia, normal response to (C) a lateral parietal lesion in the dominant
threat, normal optokinetic nystagmus, and hemisphere
normal drawing and copying. Imaging studies (D) a mesiofrontal lesion
showed an acute ischemic stroke. This is most (E) alateral frontal premotor lesion
likely a
(A) right occipitoparietal lesion 145. Lack of kinesthetic transfer is associated with
(B) right temporoparietal lesion (A) a lesion of the parietal postcentral gyrus
(C) bilateral occipital lesion (B) a lesion of the mesial parietal lobe
(D) bilateral lesion of the inferior banks of (C) a lateral parietal lesion in the dominant
the calcarine fissure hemisphere
(E) bilateral lesion of the superior banks of (D) a callosal frontal lesion
the calcarine fissure (E) a lateral frontal premotor lesion
Questions: 137154 65

146. Visual field defect with visual field agnosia and (D) lateroinferior aspect of the nondominant
hallucination is associated with a lesion located temporal lobe
in the (E) parietal postcentral gyrus
(A) mesial occipital lobe
151. Transcortical sensory aphasia is associated with
(B)lateral occipital lobe
(C) bilateral anterior tip of the temporal lobe (A) a lesion of the parietal postcentral gyrus
(D) lateroinferior aspect of the nondominant (B) a lesion of the mesial parietal lobe
temporal lobe (C) lateral parietal lesion in the dominant
(E) parietal postcentral gyrus hemisphere
(D) a callosal frontal lesion
147. Amnesia with storage impairment of geometric (E) a lateral frontal premotor lesion
pattern is associated with a lesion located in the
(A) mesial occipital lobe 152. Akinesia with perseveration and alien hand
syndrome is associated with
(B)lateral occipital lobe
(C) bilateral anterior tip of the temporal lobe (A) a lesion of the parietal postcentral gyrus
(D) lateroinferior aspect of the nondominant (B) a lesion of the mesial parietal lobe
temporal lobe (C) a lateral parietal lesion in the dominant
(E) parietal postcentral gyrus hemisphere
(D) a callosal frontal lesion
148. Blunt affect with impaired association of social (E) a mesiofrontal lesion
nuance is associated with a
(A) lateral parietal lesion in the dominant 153. Alexia with agraphia, finger agnosia, and acal-
hemisphere culia is associated with
(B) mesiofrontal lesion (A) a lesion of the parietal postcentral gyrus
(C) lateral frontal premotor lesion lesion
(D) orbitofrontal lesion (B) a lesion of the mesial parietal lobe
(E) callosal frontal lesion (C) a lateral parietal lesion in the dominant
hemisphere
149. Alexia with agraphia, impaired ipsilateral scan- (D) a callosal frontal lesion
ning, and nystagmus are associated with a (E) a mesiofrontal lesion
lesion located in the
(A) mesial occipital lobe 154. A 74-year-old man developed loss of sensation
to all modalities in the right mandible and
(B)lateral occipital lobe
lower external ear after a right endarterectomy.
(C) bilateral anterior tip of the temporal lobe Motor examination showed weakness of right
(D) lateroinferior aspect of the nondominant lateral and anterior head flexion and rotation as
temporal lobe well as weakness on external rotation of the
(E) parietal postcentral gyrus scapula. These findings are associated with
(A) ErbDuchenne palsy
150. KlverBucy syndrome is associated with a
lesion located in the (B) DejerineKlumpke palsy
(C) a lesion of the cervical plexus
(A) mesial occipital lobe
(D) thoracic outlet syndrome
(B) lateral occipital lobe
(E) lumbar plexopathy
(C) bilateral anterior tip of the temporal lobe
66 2: Localization Signs in Neurology

155. A 33-year-old woman with a history of cervical the left limb. The forearm was held in extension
cancer developed an insidious onset of lower and pronation because of elbow flexion weak-
back pain and right proximal thigh and buttock ness. These findings are suggestive of
pain. Sensory examination demonstrated loss
(A) ErbDuchenne palsy
of sensation in the lateral right leg and dorsum
of the foot. Motor examination demonstrated a (B) DejerineKlumpke palsy
flail right foot; weakness of right knee flexion (C) a lesion of the cervical plexus
as well as abduction and internal rotation of the (D) thoracic outlet syndrome
right thigh; and paresis of hip extension. These (E) lumbar plexopathy
findings are suggestive of
(A) DejerineKlumpke palsy 158. A 52-year-old man with a history of colorectal
cancer consulted the neurologist because of a
(B) a lesion of the cervical plexus
new onset of back and right-lower-extremity
(C) thoracic outlet syndrome pain. Neurological assessment demonstrated
(D) lumbar plexopathy sensory loss to pinprick over the lateral and
(E) sacral plexopathy medial right thigh and weakness of hip flexion,
leg extension, and thigh adduction. These find-
156. A 40-year-old woman consulted the neurologist ings are suggestive of
because of recurrent coldness and cyanosis of
(A) ErbDuchenne palsy
the left upper extremity with pain at the ulnar
border of the right hand. Examination of the (B) DejerineKlumpke palsy
right hand demonstrated thenar wasting. These (C) a lesion of the cervical plexus
findings are suggestive of (D) thoracic outlet syndrome
(A) ErbDuchenne palsy (E) lumbar plexopathy
(B) DejerineKlumpke palsy
159. A 65-year-old man with a history of thoraco-
(C) a lesion of the cervical plexus tomy for a left lung cancer consulted the neu-
(D) thoracic outlet syndrome rologist because he developed paresthesias of
(E) lumbar plexopathy the left medial arm and forearm and a claw
deformity of the left hand. These findings are
157. A 22-year-old football player consulted the neu- suggestive of
rologist because of an acute transient episode of
(A) ErbDuchenne palsy
intense burning and weakness in his left upper
extremity following a sudden depression of his (B) DejerineKlumpke palsy
left shoulder during a football game. A few (C) a lesion of the cervical plexus
weeks later, neurological examination showed (D) thoracic outlet syndrome
weakness of internal rotation and adduction of (E) lumbar plexopathy
Answers and Explanations

1. (B) ArgyllRobertson pupil is classically seen aqueductal area. The syndrome is character-
in patients with neurosyphilis. The lesion is ized by dilated, fixed pupils to light; loss of
thought to be in the rostral midbrain, injuring upward gaze; defective convergence; skew devi-
the supranuclear inhibitory fibers that affect ation; light near dissociation (reaction to accom-
the visceral oculomotor nuclei. The pupils are modation but not to light); and lid retraction.
usually affected bilaterally and are irregularly Any lesion affecting the afferent path-
miotic with variable iris atrophy. There is a waysthe retina, optic chiasm, optic tract, and
decrease or absence of the pupillary light reac- particularly the optic nervewill cause Marcus
tion with conservation of the near response in Gunn pupil. When the abnormal eye is stimu-
the presence of normal visual acuity. lated by light, it will slowly and briefly con-
HolmesAdie or tonic pupil syndrome is a strict and may start to dilate while it is still
condition of unknown cause related to degen- illuminated. This is also illustrated in the
eration of nerve cells in the ciliary ganglion. The swinging flashlight test: the abnormal eye
degeneration of short ciliary nerves, with sub- dilates instead of constricting when the light is
sequent collateral sprouting, results in a pre- rapidly alternated from one eye to another,
dominance of accommodation elements in the whereas the normal eye constricts and stays
innervation of the iris. It is more frequent in small. (Brazis, Masdeu, and Biller, 197205; Thompson
females and can be unilateral at first. Typically, and Miller, 9611040)
the patient presents with blurred near vision,
loss of knee and ankle jerks, and impaired 2. (A) Pupillary size is under the dual control of
sweating. The pupil is round and widely dilated; sympathetic and parasympathetic systems that
it reacts poorly to light but better to accommo- innervate rings of radially arranged dilator and
dation. The minimal reaction to accommoda- constrictor fibers, respectively. The sympathetic
tion or to light is probably related to partial system starts from the hypothalamus; its fibers
reinnervation of parasympathetic fibers and descend in the brainstem and lateral column of
slow inhibition of sympathetic fibers. The diag- the spinal cord to exit at the cervical (C8) and
nosis is confirmed by pupillary reaction to pilo- thoracic (T1-T2) levels as a second-order neu-
carpine drops. Pilocarpine is rapidly hydrolyzed rons. These neurons pass from the spinal cord to
by acetylcholine esterase and has no effect on the the superior cervical ganglion and exit as third-
normal pupil. In HolmesAdie syndrome, the order neurons, which supply the pupillodilator
denervated pupil with enzyme depletion allows fibers and blood vessels of the eye, passing
the piloloconstrictor effect to occur. over the carotid artery. Horner syndrome may
Epidemic encephalitis lethargica causes loss result from a lesion anywhere along the three
of convergence with parkinsonism. The patients neural pathways of the sympathetic nervous
pupils react to light but not to accommodation. sytem. It is characterized by the triad of miosis,
Parinaud syndrome results from a lesion in ptosis, and anhidrosis of the the forehead.
the dorsal rostral midbrain that interferes with Patients with central or first-order Horner syn-
the decussating light reflex fibers in the peri- drome can usually be identified by the presence

67
68 2: Localization Signs in Neurology

of associated hypothalamic, brainstem, or naming, and repetition abilities were affected.


spinal cord signs or symptoms. The syndrome The most likely diagnosis of this patient is
occurs more commonly with vascular damage Wernicke aphasia. In most right-handed
to the brainstem, as with midbrain infarction patients and more than two thirds of left han-
and spinal artery thrombosis. It may also be ders, the left superior temporal gyrus and the
seen with hypothalamic infarction, hemor- neighboring inferior parietal lobule play the
rhage, or tumor. The second-order neuron greatest role in processing language-related
Horner syndrome may include neck or arm auditory stimuli. Lesion centered in the poste-
pain, anhidrosis involving the face and neck, rior two thirds of the superior temporal gyrus
brachial plexopathy, vocal cord paralysis, or affecting the auditory association cortex (area
phrenic nerve palsy. Neoplasms located in the 22 of Brodmann, or Wernicke area) tend to
head, neck, brachial plexus, lung, or breast may cause the greatest impairment in the compre-
also cause a second-order Horner syndrome. In hension of auditory language. Patients with
postganglionic (third-order) Horner syndrome, Wernicke aphasia are unable to repeat sen-
the patient may have ipsilateral pain and other tences, assemble phonemes, and name things
symptoms suggestive of cluster migraine correctly. However, their speech is fluent
headaches. Postganglionic Horner syndrome (effortless, melodic, well woven, and produced
may be caused by ipsilateral cartotid dissection, at a normal or even faster rate). (Brazis, Masdeu,
Cavernous sinus lesions, neoplasms, otitis and Biller, 484485)
media, and inflammatory or infectious lesions.
(Brazis, Masdeu, and Biller, 199201) 6. (D) Alexia with agraphia is usually associated
with a pure lesion of the dominant angular
3. (C) A unilateral large, poorly reactive pupil gyrus if the patient does not have Wernicke
may occur in third-nerve palsy, contusion of aphasia. In addition to the reading and writing
the eye, focal seizure, and during accidental disturbance, affected patients usually have
exposure to aerosolized anticholenergic drugs. acalculia, finger agnosia, right-left disorienta-
Botulism as well as delirium may cause bilat- tion, and difficulty with spelling words and
eral mydriasis with normal reaction to light. A understanding spelled-out words. (Brazis,
unilateral small reactive pupil may occur as a Masdeu, and Biller, 482)
part of Horner syndrome in traumatic carotid
dissection. An acute pontine lesion may cause 7. (A) In alexia without agraphia, the affected
reactive, bilateral pinpoint pupils. (Brazis, patient is able to write on dictation but not able
Masdeu, and Biller, 204) to read. This difficulty results from damage to
the pathways conveying visual input from both
4. (A) Opsoclonus consists of rapid, involuntary, hemispheres to the dominant angular gyrus,
multivectorial, unpredictable, conjugate fast which itself remains intact but disconnected
eye movements without intersaccadic inter- from visual regions. This phenomenon usually
vals. It indicates brainstem (especially mesen- occurs with combined lesions of the dominant
cephalic) or cerebellar disease. It persists medial occipital region and the fibers that reach
during eye closure and during sleep and is the angular gyrus from the nondominant occip-
thought to be due to dysfunction of omnipause itotemporal cortex. These fibers are often dam-
neurons that normally exert tonic inhibition on aged in the splenium of the corpus callosum.
burst neurons. (Brazis, Masdeu, and Biller, 218) (Brazis, Masdeu, and Biller, 482)

5. (B) The patient described in the vignette devel- 8. (E) Transcortical sensory aphasia is seen in
oped an acute onset of speech dysfunction. An patients who have poor comprehension but
acute embolic stroke could be the cause of her retain fluent grammatical speech. They can
condition because of her past medical history of repeat words well but are unable to understand
mitral stenosis. Although her speech fluency their meaning, whether the words are spoken
was relatively preserved, her comprehension, or written. Except for spared repetition,
Answers: 317 69

transcortical sensory aphasia is an analogue of termed placidity or tameness; (b) abnor-


Wernicke aphasia. It is most often due to mal dietary changes, as with eating inappro-
lesions in the posterior middle temporal gyrus, priate things and/or overeating (e.g., bulimia);
angular gyrus, white matter of the temporal (c) hyperorality; (d) hypersexuality; (e) hyper-
isthmus, or posterior periventricular area. metamorphosis; and (f) visual agnosia. (Brazis,
(Brazis, Masdeu, and Biller, 487488) Masdeu, and Biller, 506)

9. (E) Gerstmann syndrome is characterized by the 14. (D) The alien limb sign includes failure to rec-
association of finger agnosia, rightleft disorien- ognize ownership of ones limb when visual
tation, agraphia, and acalculia. It may be seen in cues are removed, a feeling that a body part is
damage to the angular and supramarginal gyri foreign, personification of the affected body
of the dominant hemisphere. Such damage can part, and autonomous activity of the limb that
also involve the subangular white matter, affect- is perceived as being beyond voluntary con-
ing the forceps of the splenium of the corpus cal- trol. This sign is seen with damage to the
losum. (Brazis, Masdeu, and Biller, 492) corpus callosum, mesial frontal lobe, or cingu-
late gyrus or with combined infarction of the
10. (B) Optic ataxia is a part of Balint syndrome, posterior corpus callosum and thalamus.
which follows a bilateral parietooccipital lesion (Brazis, Masdeu, and Biller, 507)
in the convexity of the hemispheres and is char-
acterized by (a) failure to shift gaze on com- 15. (A) The orbitofrontal cortex is interlinked with
mand and difficulty in redirecting attention limbic and reticular areas and is activated with
voluntarily; (b) optic ataxia, a disturbance of the emotions of anger or fear; some of its neurons
reaching a target under visual control, mani- respond selectively to aversive stimuli. Damage
fested by a clumsiness of object-bound move- to the orbitofrontal area may cause blunted
ments of the hand performed under visual affect, impaired appreciation of social nuances,
guidance; and (c) reduced visual attention, impaired goal-directed behavior, impotence,
affecting mainly the peripheral visual fields facetiousness, and environmental dependency
and resulting in constriction of the fields to syndrome. (Brazis, Masdeu, and Biller, 503508)
tunnel vision. (Brazis, Masdeu, and Biller, 481)
16. (E) The corpus callosum is a structure of the
11. (B) Patients with acute bilateral and extensive mammalian brain in the longitudinal fissure that
medial occipital lesions that render them blind connects the left and right cerebral hemispheres.
may deny any difficulty with seeing and may It also facilitates communication between the
confabulate about what they see. (Brazis, two hemispheres. Much of the interhemispheric
Masdeu, and Biller, 482) communication in the brain is conducted across
the corpus callosum. Damage to the corpus cal-
12. (A) Damage to the lateral occipital cortex may losum may specifically affect tasks that require
result in alexia with agraphia, visual allesthesia, coordination between the left and right hemi-
palinopsia, impaired ipsilateral scanning, and spheres. Such damage also causes lack of kines-
impaired optokinetic nystagmus. (Brazis, thetic transfer with inability to mimic position of
Masdeu, and Biller, 506) the contralateral hand, left-hand apraxia, left-
hand agraphia, right-hand constructional
13. (D) Bilateral medial temporal lobe damage apraxia, and intermanual conflict. Other signs of
may cause KlverBucy syndrome. The amyg- damage to the corpus callosum include per-
dala has been particularly implicated in the plexity, double hemianopia, and left hemipara-
pathogenesis of this syndrome. Affected lexia. (Brazis, Masdeu, and Biller, 508)
patients may develop (a) abnormal docility,
characterized by the exhibition of diminished 17. (C) Hemiballismus results from damage to the
fear responses or unusually low levels of contralateral subthalamic nucleus. (Brazis,
aggressiona phenomenon that has been Masdeu, and Biller, 424)
70 2: Localization Signs in Neurology

18. (D) Choreoathetosis results from damage to apnea, CO2 reaccumulates until it exceeds the
the contralateral anteroventral portion of the respiratory threshold, and the cycle repeats
caudate nucleus. (Brazis, Masdeu, and Biller, 508) itself. (Brazis, Masdeu, and Biller, 560)

19. (B) Sudden bilateral paramedian thalamic 22. (A) Apneustic respiration is an abnormal pat-
lesions may cause a decreased level of alertness tern of breathing characterized by deep, gasp-
ranging from somnolence to coma. Akinetic ing inspiration with a pause at full inspiration
mutism may follow bilateral paramedian followed by a brief, insufficient release. It is
lesions. It is characterized by absolute mutism seen in dysfunction of the lower pontine
and complete immobility except for the eyes, tegmentum. (Plum, 35)
which are kept open and move in all directions.
The patient appears awake and maintains a 23. (B) Ataxic breathing has a completely irregu-
sleepwake cycle without the ability to com- lar pattern in which both deep and shallow
municate. Akinetic mutism has also been breaths occur randomly. Irregular pauses
described with injury of the mesencephalic appear haphazardly, and there is no predicting
reticular formation. (Brazis, Masdeu, and Biller, 407; the future respiratory rhythm from the pattern
Afifi, 153) of past breaths. Physiologically, ataxic breath-
ing represents primary functional disruption
20. (A) Lesion of the pulvenar may result in a of the medullary neuronal populations that
decrease in the critical flicker frequency and normally generate the respiratory rhythm.
neglect of visual objects at the periphery of the (Plum, 38)
contralateral visual field, prolonged latency of
visual evoked saccadic eye movements, and 24. (E) Pontine lesions in the tegmentum interrupt
paucity of spontaneous eye movements directed descending sympathetic pathways and pro-
toward the contralateral hemifield. (Brazis, duce bilaterally small pupils. Pinpoint pupils
Masdeu, and Biller, 412) generally mean pontine hemorrhage and are
believed to result from parasympathetic irrita-
21. (E) This is a pattern of periodic breathing in tion in combination with sympathetic inter-
which phases of hyperpnea regularly alternate ruption. (Plum, 4445)
with apnea. The breathing waxes from breath to
breath in a smooth crescendo and then, once a 25. (C) Figure 2-1 is a schematic diagram of the struc-
peak is reached, wanes in an equally smooth tures involved in lateral medullary syndrome:
decrescendo. The hyperpneic phase usually lasts spinal nucleus of the trigeminal nerve and its
longer than the apneic phase. CheyneStokes tract, adjacent spinothalamic tract, nucleus
respiration implies bilateral dysfunction of ambiguus or its axons, restiform body, vestibuilar
neurological structures usually lying deep in the nuclei, descending sympathetic fibers from the
cerebral hemispheres or diencephalon. Cheyne hypothalamus, and olivocerebellar fibers. The
Stokes respiration is the outcome of a combina- neurological signs and symptoms resulting from
tion of an abnormally increased ventilatory the shaded area in Figure 2-1 include the follow-
response to CO2 stimulation, causing hyperp- ing: loss of pain and temperature sensation in
nea, and an abnormally decreased forebrain the ipsilateral face and contralateral half of the
ventilatory stimulus, permitting posthyperven- body, ataxia, vertigo, loss of gag reflex and diffi-
tilation apnea. Patients with bilateral hemi- culty with swallowing, ipsilateral Horner syn-
spheric lesions overbreathe when stimulated by drome, vomiting, nausea, nystagmus, hiccups,
CO2, a phenomenon reminiscent of other facili- and ocular lateropulsion. (Afifi, 99)
tated neurological responses to stimulation. As
a result of the overbreathing, the bloods CO2 26. (A) Figure 2-2 is a schematic diagram of the
content drops below the level where it stimu- structures involved in medial medullary
lates the respiratory centers and, in the presence syndrome: medial lemniscus, rootlets of the
of brain dysfunction, breathing stops. During hypoglossal nerve or its nucleus within the
Answers: 1832 71

medulla and pyramid. The neurological signs deviation of the eye to a down and out posi-
and symptoms resulting from the shaded area in tion due to loss of innervation to all ocular
Figure 2-2 include paralysis of the homolateral motility muscles except for the lateral rectus
half of the tongue, contralateral upper motor (innervated by the abducens cranial nerve) and
neuron syndrome, and contralateral loss of the superior oblique (innervated by the
kinesthesia and discrimination touch. (Afifi, 99) trochlear cranial nerve). (Plum, 109)

27. (C) Figure 2-3 is a schematic diagram of the 29. (A) Unlike temporal masses, frontal, parietal,
structures involved in the caudal pontine syn- or occipital masses first compress the dien-
drome (MillardGubler): the corticospinal tract cephalon, which, as the supratentorial pressure
and facial nerve. The manifestations of this syn- increases, shifts downward and buckles over
drome include ipsilateral facial paralysis of the the midbrain. Subsequently, flattening of the
peripheral type and contralateral hemiplegia of midbrain and pons in the rostrocaudal direc-
the upper motor neuron type. Frequently, the tion causes elongation and rupture of the para-
lesion may extend medially and rostrally to median perforating arteries feeding these
include the rootlets of the sixth nerve, as illus- structures, resulting in infarction and hemor-
trated in Figure 2-4. In addition to the mani- rhage in the tegmentum of the midbrain first
festations of the MillardGubler syndrome, the and pons afterward. The first evidence that a
affected patient may develop ipsilateral paral- supratentorial mass is beginning to impair the
ysis of ocular abduction. (Afifi, 124) diencephalon is usually a change in alertness or
behavior. Initially, subjects with such lesions
28. (B) In uncal herniation, the innermost part of find it difficult to concentrate and tend to lose
the temporal lobe, the uncus, can be squeezed the orderly details of recent events. Respiration
so much that it goes by the tentorium and puts in the early diencephalic stage of the central
pressure on the brainstem, most notably the syndrome is commonly interrupted by deep
midbrain. As illustrated in Figure 2-5, an sighs, yawns, and occasional pauses. As such
expanding lesion (which could be located in patients sink into deeper somnolence, many
the lateral middle fossa or temporal lobe) com- have periodic breathing of the CheyneStokes
monly pushes the medial edge of the uncus type. (Plum, 103104)
and hippocampal gyrus toward the midline
and over the free lateral edge of the tentorium. 30. (E) In the medullary stage of central herniation,
Because the diencephalon may be not the first ataxic breathing occurs and soon gives way to
structure encroached upon, impaired con- apnea. The blood pressure drops and the pulse
sciousness is not consistently an early sign of becomes irregular. (Brazis, Masdeu, and Biller, 574)
impending uncal herniation, and the state of
alertness in different subjects may vary from 31. (A) Pretectal or tectal midbrain lesions affect-
near wakefulness through stupor to coma. The ing the posterior commissure abolish the light
uncus can squeeze the third cranial nerve, reflex, but the pupils, which are midsized or
which controls parasympathetic input to the large, may show spontaneous oscillation in size
eye on the side of the affected nerve. This inter- (hippus) and become larger when the neck is
rupts parasympathetic neural transmission, pinched (ciliospinal reflex). Tegmental lesions
causing the pupil of the affected eye to dilate that involve the third-nerve nucleus may cause
and fail to constrict as it should in response to irregular constriction of the sphincter of the iris,
light. Thus, the earliest consistent sign of uncal with a resultant pear-shaped pupil or displace-
herniation is the unilaterally dilated pupil. ment of the pupil to one side (corectopia). (Brazis,
Moderate anisocoria with a sluggish light reac- Masdeu, and Biller, 563)
tion of the dilated pupil can somtimes last for
several hours before other signs appear. 32. (B) Apneustic breathing is characterized by a
Pupillary dilation often precedes a later finding long inspiratory pause, after which the air is
of oculomotor nerve compression, which is retained for several seconds and then released.
72 2: Localization Signs in Neurology

This abnormality appears with lesions of the sive pupil, drooping of the eyelid, and devia-
lateral tegmentum of the lower half of the pons. tion of eye downward and outward) and con-
(Brazis, Masdeu, and Biller, 561) tralateral tremor. (Afifi, 151)

33. (D) Figure 2-6 is a schematic diagram showing 38. (A) A pretectal lesion causes Parinaud syn-
structures involved in the rostral basal pontine drome. Patients with this syndrome present
syndrome: the trigeminal nerve and the corti- with upward gaze paralysis, pupillary abnor-
cospinal tract. A basal pontine lesion at the level malities (large pupil, lightnear dissociation),
of the trigeminal nerve results in ipsilateral lid retraction, and convergence retraction nys-
paralysis of the masticatory muscles (masseter, tagmus on upward gaze. (Afifi, 152)
temporalis, medial and lateral pterygoid mus-
cles), ipsilateral loss of facial sensation, and 39. (B) Infarction in the anteriomedial branches of
contralateral limb paralysis. (Afifi, 125) the posterior cerebral artery is the cause of
Claude syndrome in the majority of patients. It
34. (E) Figure 2-7 is a schematic diagram show- is characterized by ipsilateral oculomotor nerve
ing structures involved in the medial tegmen- palsy, contralateral tremor, and ataxia. (Afifi, 152)
tal syndrome: the nucleus and the rootlets of
the abducens nerve, the genu of the facial 40. (A) The patient described in this vignette suf-
nerve, and the medial lemniscus. The manifes- fers from peduncular hallucinosis syndrome. It
tations of the lesion therefore include ipsilateral is characterized by nonthreatening hallucina-
abducens nerve paralysis with lateral gaze tions, often formed nonstereotypically, that are
palsy, ipsilateral facial nerve paralysis of colored and vivid; they usually occur in som-
peripheral type, and contralateral loss of kines- nolent patients with presumed lesions of the
thesia and discriminative touch. (Afifi, 125) tegmentum and cerebral peduncle. Peduncular
hallucinosis has been reported in vascular and
35. (D) The dashed area indicated by arrow A in infective lesions of the thalamus, pars reticulata
Figure 2-8 includes rootlets of the oculomotor of substantia nigra, midbrain, pons, and basal
nerve. Damage to the oculomotor nerve causes diencephalon as well as by compression of the
a dilated unresponsive pupil, drooping of the midbrain. Transient peduncular hallucinations
eyelid, and deviation of eye downward and due to extrinsic compression of the midbrain
outward. (Afifi, 152) by cystic craniopharyngioma have also been
reported. Such hallucinations have also been
36. (B) The dashed area indicated by arrow B in reported following the excision of a posterior
Figure 2-8 includes rootlets of the oculomotor fossa medulloblastoma. (Afifi, 153; Kumar,
nerve and the underlying cerebral peduncle. 183185; Kumar, 500503)
Damage to the dashed area causes Weber syn-
drome, which is characterized by ipsilateral 41. (B) A C5 root lesion induces neck, shoulder,
oculomotor paralysis (dilated unresponsive and anterior upper arm pain. Muscle weakness
pupil, drooping of the eyelid, and deviation of occurs predominantly but variably in the fol-
eye downward and outward) and contralat- lowing muscles: the supraspinatus and deltoid,
eral upper motor neuron paralysis that resulting in weakness of shoulder abduction,
includes the lower face. (Afifi, 151152) and the rhomboid, serratus anterior, infra-
spinatus, biceps, and brachioradialis. Biceps
37. (B) The dashed area indicated by arrow C in and brachioradialis reflexes may be depressed.
Figure 2-8 includes rootlets of the oculomotor (Brazis, Masdeu, and Biller, 93; Patten, 288291)
nerve within the tegmentum of the mesen-
cephalon and the underlying red nucleus. 42. (C) A C6 root lesion is often caused by com-
Lesions of the dashed area cause the syndrome pression from disk herniation at the C5-C6 ver-
of Benedikt. It is characterized by ipsilateral tebral level. It results in pain in the lateral arm
oculomotor nerve paralysis (dilated unrespon- and dorsal forearm. Paresthesia and hypesthesia
Answers: 3348 73

occur in the lateral forearm, lateral hand, and the biceps brachii and the brachialis, to the lat-
first and second digits. Muscle weakness eral side of the arm; a little above the elbow, it
occurs in the biceps, pronator teres, and bra- pierces the deep fascia lateral to the tendon of
chioradialis, inducing weakness of elbow flex- the biceps brachii and is continued into the
ion in both the fully supine position and forearm as the lateral antebrachial cutaneous
half-pronated positions. The extensor carpi nerve. In its course through the arm it inner-
radialis longus, flexor carpi radialis brevis, vates the coracobrachialis, biceps brachii, and
supinator, and serratus anterior are also the greater part of the brachialis. Musculo-
affected by C6 root damage. The biceps and cutaneous dysfunction causes atrophy of the
brachioradialis reflexes may be depressed. biceps and brachialis, resulting in wasting of
(Brazis, Masdeu, and Biller, 93; Patten, 288291) the ventral aspect of the upper arm and
absence of the biceps reflex. (Brazis, Masdeu, and
43. (D) In monoradiculopathy, C7 is the most com- Biller, 34; Patten, 292296)
monly affected level of the cervical roots, fol-
lowed by the C6 nerve root. In C7 47. (A) The radial nerve originates from the pos-
radiculopathy, pain is located in the dorsal fore- terior cord of the brachial plexus with roots
arm and middle and ring fingers. Paresis from C5, C6, C7, C8, and T1. The radial nerve
occurs variably in the pectoralis major and and its branches supply the dorsal muscles,
latissimus dorsi (inducing weakness of shoul- such as the triceps brachii, the extrinsic exten-
der adduction), the triceps (inducing weakness sors of the wrist and hands, and cutaneous
of elbow extension), and the flexor carpi radi- nerve supply to most of the back of the hand.
alis, extensor carpi radialis longus, extensor It divides into a deep branch (which becomes
carpi radialis brevis, and extensor digitorum the posterior interosseous nerve) and continues
(inducing weakness of wrist extension). The as the superficial branch, which goes on to
triceps reflex may be affected. (Brazis, Masdeu, innervate the dorsum (back) of the hand.
and Biller, 93; Patten, 288291) Saturday night palsy corresponds to com-
pression of the radial nerve within the spiral
44. (E) A C8 nerve root lesion causes pain in the groove of the humerus. Clinical signs may
medial arm, the forearm, and the fifth digit. include paralysis of extension of the wrist and
Paresis occurs predominantly in the long fore- elbow flexion and weakness of supination.
arm extensors and flexors of the fingers. (Brazis, Elbow extension is preserved because the
Masdeu, and Biller, 94; Patten, 288291) radial nerve branches to the triceps muscle
originate proximal to the spiral groove. (Brazis,
45. (A) A T1 nerve root lesion causes a deep Masdeu, and Biller, 4548; Patten, 292296)
aching sensation in the shoulder joint, axilla,
and medial side of the upper arm down to the 48. (B) The ulnar nerve comes from the medial
olecranon. There is a loss of intrinsic hand mus- cord of the brachial plexus and runs inferior on
cles, including the abductor pollicis brevis the posterior and medial (posteromedial)
muscle, which differentiates T1 nerve root aspects of the humerus down the arm, going
lesions from ulnar nerve lesions (in ulnar nerve behind the medial epicondyle and through the
lesions, all intrinsic muscles of the hand are cubital tunnel at the elbow. At the forearm, it
affected except the abductor pollicis brevis). enters the anterior compartment of the fore-
(Brazis, Masdeu, and Biller, 94; Patten, 288291) arm through the two heads of flexor carpi
ulnaris and runs alongside the ulna. There it
46. (C) The musculocutaneous nerve arises from supplies 112 muscles (the flexor carpi ulnaris
the lateral cord of the brachial plexus, opposite and medial half of flexor digitorum profun-
the lower border of the pectoralis minor, its dus). After it travels down the ulna, the ulnar
fibers being derived from the fifth, sixth, and nerve enters the palm of the hand and passes
seventh cervical nerves. It penetrates the coraco- superficial to the flexor retinaculum via the
brachialis muscle and passes obliquely between ulnar canal to give off the superficial and deep
74 2: Localization Signs in Neurology

branches of the ulnar nerve. The deep branch of divisions of the second and third nerves divide
the ulnar nerve supplies the hypothenar mus- into two branches, a smaller branch from each
cles (opponens digiti minimi, abductor digiti uniting to form the lateral femoral cutaneous
minimi, flexor digiti minimi brevis), adductor nerve and a larger branch from each joining
pollicis, the third and fourth lumbrical mus- with the dorsal division of the fourth nerve to
cles, and the dorsal and palmar interossei. The form the femoral nerve. The accessory obtura-
superficial branch of the ulnar nerve supplies tor, when it exists, is formed by the union of
the palmaris brevis. An ulnar nerve lesion at two small branches given off from the third
the wrist may cause paralysis of all intrinsic and fourth nerves. A lumbar plexus lesion
hand muscles except the abductor pollicis causes pain across the thigh. Sensation may
brevis, which is innervated by the median be lost in the inguinal region and over the
nerve. Since the ulnar nerve lesion is proximal genitalia innervated by the iliohypogastric,
to the origin of the superficial terminal cuta- ilioinguinal, and genitofemoral nerves. The
neous branch of the ulnar nerve, there is sen- sensation of the anterior and medial parts of
sory loss on the distal part and the palmar the thigh may be affected. Motor signs include
surfaces of the fifth and medial half of the paresis and atrophy of muscles innervated by
fourth fingers. (Brazis, Masdeu, and Biller, 4044; the femoral and obturator nerves. Thus, there
Patten, 292296) is weakness of thigh flexion because of paresis
of the iliopsoas, leg extension because of pare-
49. (D) A median nerve lesion at the upper arm sis of the quadriceps, thigh eversion because of
may cause pain in the thumb, index, and paresis of the sartorius, and thigh adduction
middle fingers that spreads up from the fore- because of paresis of the adductor muscles. The
arm to the elbow. Motor signs may include patellar and cremasteric reflexes may be
paresis of forearm pronation, radial wrist flex- decreased or absent. (Brazis, Masdeu, and Biller,
ion, distal flexion of the thumb, palmar abduc- 8184; Patten, 299314)
tion, opposition of the thumb, and flexion of
the second and, to a lesser extent, third fingers. 51. (C) The obturator nerve arises from the ventral
Weakness of the pinch sign results from pare- divisions of the second, third, and fourth
sis of the flexor digitorum profundus of the lumbar nerves; the branch from the third is the
index finger and of the flexor pollicis longus. largest, while that from the second is often very
(Brazis, Masdeu, and Biller, 3540; Patten, 292296) small. It descends through the fibers of the
psoas major and emerges from its medial
50. (A) The lumbar plexus is formed by the loops border near the brim of the pelvis; it then
of communication between the anterior divi- passes behind the common iliac vessels, on the
sions of the first three and the greater part of lateral side of the hypogastric vessels and
the fourth lumbar nerves; the first lumbar often ureter, and runs along the lateral wall of the
receives a branch from the last thoracic nerve. lesser pelvis, above and in front of the obtura-
The first lumbar nerve, frequently supple- tor vessels, to the upper part of the obturator
mented by a twig from the last thoracic, splits foramen. Here it enters the thigh through the
into upper and lower branches; the upper and obturator canal and divides into anterior and
larger branch divides into the iliohypogastric posterior branches, which are separated at first
and ilioinguinal nerves; the lower and smaller by some of the fibers of the obturator externus
branch unites with a branch of the second and lower down by the adductor brevis. The
lumbar to form the genitofemoral nerve. The obturator nerve is responsible for the sensory
remainder of the second lumbar nerve and the innervation of the skin of the medial aspect of
third and fourth lumbar nerves split into ven- the thigh. It is also responsible for the motor
tral and dorsal divisions. The ventral division innervation of the adductor muscles of the
of the second lumbar nerve unites with the lower extremity (external obturator, adductor
ventral divisions of the third and fourth lumbar longus, adductor brevis, adductor magnus,
nerves to form the obturator nerve. The dorsal gracilis). The patient in this vignette developed
Answers: 4954 75

sensory and motor disturbance in the territory pelvis through the greater sciatic foramen, below
of the obturator nerve: disturbance of sensa- the Piriformis muscle. It descends midway in the
tion in the medial aspect of the thigh and weak- greater trochanter of the femur and the tuberos-
ness of the adductor muscles. Pregnancy may ity of the ischium, and along the back of the
cause compression of the obturator nerve in thigh to about its lower third, where it divides
the obturator canal. (Brazis, Masdeu, and Biller, into two large branches, the tibial and common
5355; Patten, 299314) peroneal nerves. The nerve gives off articular
and muscular branches. The articular branches
52. (E) The tibial nerve is a branch of the sciatic (rami articulares) arise from the upper part of
nerve. It passes through the popliteal fossa and the nerve and supply the hip-joint, perforating
below the arch of soleus. In the popliteal fossa, the posterior part of its capsule. The muscular
the nerve gives off branches to the gastrocne- branches (rami musculares) are distributed to
mius, popliteus, soleus, and plantaris muscles, the following muscles of the lower limb: Biceps
an articular branch to the knee joint, and a cuta- femoris, Semitendinosus, Semimembranosus,
neous branch that becomes the sural nerve. The and Adductor magnus. The nerve to the short
sural nerve is joined by fibers from the head of the Biceps femoris comes from the
common peroneal nerve and runs down the common peroneal part of the sciatic, while the
calf to supply the lateral side of the foot. Below other muscular branches arise from the tibial
the soleus muscle, the nerve lies close to the portion, as may be seen in those cases where
tibia and supplies the tibialis posterior, the there is a high division of the sciatic nerve. The
flexor digitorum longus, and the flexor hallucis muscular branch eventually gives off the tibial
longus. The nerve passes into the foot running nerve and common peroneal nerve, which
posterior to the medial malleolus. Here it is innervates the muscles of the (lower) leg. The
bound down by the flexor retinaculum in com- tibial nerve goes on to innervate all muscles of
pany with the posterior tibial artery. In the foot, the foot except the extensor digitorum brevis
the nerve divides into medial and lateral plan- (peroneal nerve). The patient described in this
tar branches. The medial plantar nerve sup- vigentte has a flail foot because of paralysis of
plies the abductor hallucis, the flexor digitorum flexors and extensors of the left foot, and knee
brevis, the flexor hallucis brevis, and the first flexion weakness due to paresis of the ham-
lumbrical. Cutaneous distribution of the medial string muscles. A single lesion in the sciatic
plantar nerve is to the medial sole and medial nerve would result in these and in loss of sen-
312 toes, including the nail beds on the dorsum sation in the lateral leg. Decreased Achilles
(like the median nerve in the hand). The lateral reflex can occur in sciatic lesions because the
plantar nerve supplies the quadratus plantae, tibial nerve subserves this reflex. Sciatic nerve
flexor digiti minimi, adductor hallucis, interos- injury may be caused by fracture dislocation of
sei, three lumbricals, and abductor digiti the hip, pelvic cancer surgery, infection, and
minimi. Cutaneous innervation is to the lateral buttock intramuscular injection, as illustrated in
sole and lateral 112 toes. The patient described this case. (Brazis, Masdeu, and Biller, 5764; Patten,
in the vignette has sensory loss in the territory 299314)
of the tibial nerve. Weakness of plantarflexion
and inversion of the foot is caused by weakness 54. (A) The lateral cutaneous nerve of the thigh is
of the gastrocnemius and the tibialis posterior a nerve of the lumbar plexus. It arises from the
muscles (both are innervated by the tibial nerve). dorsal divisions of the second and third lumbar
(Brazis, Masdeu, and Biller, 57; Patten, 299314) nerves. It emerges from the lateral border of the
psoas major at about its middle and crosses the
53. (D) The sciatic nerve is the longest and widest iliacus muscle obliquely, toward the anterosu-
single nerve in the body. It supplies nearly the perior iliac spine. It then passes under the
whole of the skin of the leg, the muscles of the inguinal ligament and over the sartorius muscle
back of the thigh, and those of the leg and foot. into the thigh, where it divides into anterior
The nerve enters the lower limb by exiting the and posterior branches. The anterior branch
76 2: Localization Signs in Neurology

becomes superficial about 10 cm below the soleus), toe dorsiflexion and plantarflexion,
inguinal ligament and divides into branches foot eversion and inversion (due to weakness
that are distributed to the skin of the anterior of the peronei and tibialis anterior and the pos-
and lateral parts of the thigh as far as the knee. terior calf muscles, respectively). All of these
The posterior branch pierces the fascia lata and muscles are in the sciatic distribution. Paresis of
subdivides into filaments that pass backward hip extension results from weakness of the glu-
across the lateral and posterior surfaces of the teus maximus, innervated by the inferior
thigh, supplying the skin from the level of the gluteal nerve. Paresis of abduction and internal
greater trochanter to the middle of the thigh. rotation of the thigh results from weakness of
The patient described in this vignette has a sen- the gluteus medius and minimus, innervated
sory disturbance located in the region of the by the superior gluteal nerve. Thus the patient
lateral femoral cutaneous nerve. Entrapment of described in this vignette has symptoms and
this nerve is also known as meralgia paresthet- signs in the distribution of the sciatic nerve,
ica. Entrapment usually occurs at the inguinal superior gluteal nerve, and inferior gluteal
ligament. The peak incidence for this condition nerve. Compression of the left sacral plexus by
is in middle age. The entrapment may be from metastasis from the breast may explain the
intrapelvic, extrapelvic, or mechanical causes. patients clinical picture. (Brazis, Masdeu, and
Intrapelvic causes would include pregnancy, Biller, 8183; Patten, 299314)
abdominal tumors, uterine fibroids, divertic-
ulitis, and appendicitis. Examples of extrapelvic 56. (A) The femoral nerve, the largest branch of
causes include trauma to the region of the the lumbar plexus, arises from the dorsal divi-
anterosuperior iliac spine (e.g., a seatbelt from sions of the second, third, and fourth lumbar
a motor vehicle accident), tight garments, belts, nerves. It descends through the fibers of the
girdles, or stretch from obesity and ascites. psoas major, emerging from the muscle at the
Mechanical factors include prolonged sitting or lower part of its lateral border, and passes
standing and pelvic tilt from leg-length dis- down between it and the iliacus, behind the
crepancy. Symptoms include anterior and lat- iliac fascia; it then runs beneath the inguinal lig-
eral thigh burning, tingling, and/or numbness ament into the thigh, and splits into anterior
that increase with standing, walking, or hip and posterior divisions. Under the inguinal lig-
extension. Symptoms may also increase with ament, it is separated from the femoral artery
lying prone. Symptoms are usually unilateral by a portion of the psoas major. Within the
but may be bilateral in rare cases. The symp- abdomen, the femoral nerve gives off small
toms usually improve with sitting unless com- branches to the iliacus and a branch that is dis-
pressive forces, such as tight belts or garments, tributed upon the upper part of the femoral
remain. Physical examination findings may be artery; the latter branch may arise in the thigh.
completely normal. Findings may include In the thigh, the anterior division of the femoral
hyperesthesia over the lateral thigh (usually in a nerve gives off anterior cutaneous and muscu-
smaller area than the symptoms). Pain can be lar branches. The muscular branches supply
produced by pressure medial to the anterosupe- the pectineus and sartorius muscles. The pos-
rior iliac spine . A positive Tinel sign may be pres- terior division of the femoral nerve gives off the
ent over the anterosuperior iliac spine or inguinal saphenous nerve and muscular and articular
ligament. (Brazis, Masdeu, and Biller, 5556; Patten, branches.The muscular branches supply the
299314) four parts of the quadriceps femoris. The
patient reported in this vignette developed
55. (B) The patient described in this vignette weakness of left knee extension, presumably by
developed loss of sensation in the territory of quadriceps weakness, and pain in the anterior
the left posterior femoral cutaneous and sci- thigh and medial leg caused by injury to the
atic nerves. There is weakness of all movement medial cutaneous nerve of the thigh and the
of the foot. This includes foot plantarflexion saphenous nerves (both are branches of the
(due to weakness of the gastrocnemius and femoral nerve). These symptoms suggest left
Answers: 5561 77

femoral nerve dysfunction. As the patient in burning pain, pins and needles, numbness, or
this case has a history of falling and of warfarin hypersensitivity. Usually, the symptoms are
use, a retroperitoneal hematoma with femoral made worse by sitting and better by either
nerve compression is the most likely etiology. standing or lying down. (Brazis, Masdeu, and
However, retroperitoneal hematoma is more Biller, 5657; Patten, 299314)
commonly associated with diffuse weakness
because multiple portions of the lumbosacral 59. (A) B12 deficiency and ethanol abuse may cause
plexus are involved. (Brazis, Masdeu, and Biller, toxic optic neuropathy. Peripheral vision is usu-
5153; Patten, 299314) ally spared, since the pattern of loss typically
involves a central or cecocentral scotoma, a
57. (B) The common peroneal nerve is derived visual field defect at or surrounding the point of
from the dorsal branches of the fourth and fifth fixation. B12 deficiency may affect mitochondrial
lumbar and the first and second sacral nerves. It oxidative phosphorylation, causing an acquired
descends obliquely along the lateral side of the mitochondrial optic neuropathy. Vision loss in
popliteal fossa to the head of the fibula, close to B12 deficiency is bilateral, symmetric, painless,
the medial margin of the biceps femoris muscle. gradual, and progressive. Dyschromatopsia, a
It lies between the tendon of the biceps femoris change in color vision, is often the first symp-
and lateral head of the gastrocnemius muscle, tom. Some patients notice that certain colors,
winds around the neck of the fibula between particularly red, are less bright or vivid; others
the peroneus longus and the bone, and divides have a general loss of color perception. Loss of
beneath the muscle into the superficial and deep visual acuity may start with a blur or haze at the
peroneal nerves. Previous to its division it gives point of fixation followed by a progressive
off articular and lateral sural cutaneous nerves. decline. The degree of vision loss can extend to
The deep fibular nerve supplies muscular total blindness, but a loss beyond 20/400 is rare.
branches to the tibialis anterior, extensor digito- (Kline, 159)
rum longus, fibularis (peroneus) tertius, and
extensor hallucis longus (propius). The patient 60. (B) Bilateral compression of the lateral optic
described in the vignette developed weakness of chiasm is rare. It may be caused by the intra-
foot dorsiflexion and eversion associated to a sen- cavernous part of an arteriosclerotic carotid
sory loss suggestive of left peroneal neuropathy. artery pushing the chiasm against the oppo-
(Brazis, Masdeu, and Biller, 5762; Patten, 299314) site carotid artery. It may also be caused by
dilatation of the third ventricle secondary to
58. (E) The pudendal nerve originates in the sacral chronic aqueductal stenosis. The chiasm is
plexus; it derives its fibers from the ventral splayed laterally by the dilated third ventricle
rami of the second, third, and fourth sacral and damaged by the pulsatile carotid arteries
nerves (S2, S3, and S4). It passes between the pressing against its lateral edge. Bilateral com-
piriformis and coccygeus muscles and leaves pression of the lateral chiasm may cause
the pelvis through the lower part of the greater binasal hemianopia. However, binasal hemi-
sciatic foramen. It crosses the spine of the anopia is most often caused by retinal or optic
ischium and reenters the pelvis through the nerve disease. Intracranial causes may include
lesser sciatic foramen. The pudendal nerve skull fracture, neurosyphilis, chiasmal arach-
gives off the inferior rectal nerves. It soon noiditis, and neoplasms. (Kline, 227)
divides into two terminal branches: the per-
ineal nerve, and the dorsal nerve of the penis 61. (C) The drug ethambutol is commonly associ-
(in males) or the dorsal nerve of the clitoris (in ated with toxic optic neuropathy. The optic
females). The patient described in the vignette neuropathy that occurs is dose-dependent and
has clinical manifestations suggesting puden- duration-related. Loss of vision does not tend
dal nerve damage. Pudendal neuropathy can to occur until the patient has been on the drug
cause rectal, perineal, or genital pain. The for at least 2 months, but there are rare reports
symptoms may include stabbing, twisting, or of early onset of severe bilateral visual loss
78 2: Localization Signs in Neurology

even with appropriate dosing. Symptoms gen- 66. (A) With eyelid closure, reflex innervation of
erally appear between 4 months to a year after the extraocular muscles results in an upward
the initiation of treatment. This onset may be and slightly outward rotation of the globe. This
sooner if the patient has concurrent renal dis- reflex eye movement is Bells phenomenon, a
ease, because this will result in reduced excre- physiological mechanism that protects the
tion of the drug and therefore elevated serum cornea from exposure and ulceration. In
levels. The clinical presentation may include a patients who have reduced or absent Bells
central visual deficit. Some patients may com- phenomenon, a tarsorrhaphy or placement of a
plain of such a deficit and often claim that if gold weight in the upper eyelid is sometimes
they could see around it, their vision would be needed to protect the affected eye. (Brazis,
normal. The clinical presentation may also Masdeu, and Biller, 245)
include dyschromatopsia. Therefore, appropri-
ate color vision testing is of particular importance 67. (D) The TolosaHunt syndrome, a painful
in screening patients on this drug. (Kline, 157) ophthalmoplegia, is characterized by steady,
unremitting retro- and supraorbital pain in the
62. (D) At the chiasm, fibers from the inferior part trigeminal nerves ophthalmic distribution in
of the nasal retina are ventral in the chiasm and association with paresis of the oculomotor,
loop into the proximal part of the contralateral trochlear, and abducens nerves as well as a
optic nerve before turning back to join uncrossed diminished corneal reflex. Sensory loss and
inferotemporal fibers in the optic tract. pain in the mandibular trigeminal distribution
Compression of the junction between the optic may also occur. Less frequently, the optic nerve
nerve and optic chiasm may cause an ipsilat- and oculosympathetic fibers may be affected.
eral central scotoma with a contralateral supe- Symptoms may persist for weeks to months.
rior temporal visual defect. (Brazis, Masdeu, and Both sexes are equally affected. The sedimen-
Biller, 162163) tation rate may be elevated. Pathologically, a
low-grade, noninfectious granulomatous
63. (E) Bitemporal hemianopsia is a type of partial process adjacent to the cavernous sinus or
blindness where vision is missing in the outer within the superior orbital fissure has been
half of both the right and left visual fields. It is identified. The granulomas consist of lympho-
usually associated with lesions of the optic cytes and plasma cells. The TolosaHunt syn-
chiasm. Bitemporal hemianopia most com- drome typically responds dramatically to
monly occurs as a result of tumors located at systemic corticosteroids, although symptoms
the midoptic chiasm, such as pituitary adeno- may recur months to years later. Spontaneous
mas and craniopharyngiomas. (Brazis, Masdeu, remissions have also been reported. (Goetz and
and Biller, 162163) Pappert, 167)

64. (A) Antons blindness is a rare symptom of 68. (B) The patient described in the vignette has
brain damage occurring in the occipital lobe. weakness on right arm elevation associated
The affected patients tend to dismiss their fail- with winging of the scapula. These signs point
ure to see through confabulation. This condi- toward weakness of the serratus anterior
tion is mostly seen following an acute extensive muscle. This muscle fixes and stabilizes the
and medial occipital stroke. (Brazis, Masdeu, and scapula against the chest wall. It is tested by
Biller, 482483) observing for scapular winging (the vertebral
border of the scapula stands away from the
65. (D) The anterior visual cortex is supplied by thorax, forming a wing, while the patient
the posterior cerebral artery. Infarction of this pushes the extended arm against a fixed
area causes a macula-sparing hemianopia object). The serratus anterior muscle is inner-
because the macular cortex has a dual vascular vated by the long thoracic nerve, which may
supply from the middle and posterior cerebral be affected by a variety of athletic activities,
arteries. (Brazis, Masdeu, and Biller, 162163) like volleyball, for example. A C7 cervical root
Answers: 6272 79

lesion may cause weakness of the serratus sor of the hand), the extensor digitorum (an
anterior muscle, but in combination with extensor of the metacarpophalangeal joint
weakness of the extensors of the arm, wrist, or of the second through the fifth fingers), the
fingers. Volleyball players are also prone to extensor digiti minimi (an extensor of the
suprascapular nerve injuries. However, a metacarpophalangeal joint of the fifth finger),
lesion of the suprascapular nerve results in the extensor carpi ulnaris (an ulnar extensor
weakness of arm abduction and external rota- of the hand), the abductor pollicis longus (an
tion without scapular winging. A dorsal scapu- abductor of the metacarpal of the thumb), the
lar nerve lesion causes weakness of the extensor pollicis longus and brevis (extensors
rhomboid and levator scapulae muscles, result- of the thumb), and the extensor indicis (an
ing in weakness of elevation and adduction of extensor of the second finger). A lesion of the
the medial border of the shoulder blade. radial nerve at the axilla causes weakness of
Weakness of the trapezius muscle may cause elbow extension, loss of the triceps reflex (tri-
winging of the scapula on abduction of the arm; ceps muscle), wrist drop, finger drop, and
the shoulder is lower on the affected side sensory loss on the entire extensor surface of
because there is weakness on elevation and the arm, the forearm, the web between the
retraction. (Brazis, Masdeu, and Biller, 29; Staal, 1921) index finger and the thumb, and the radial
side of the dorsum of the hand. There is also
69. (B) The radial nerve derives from the poste- weakness of forearm flexion and a depressed
rior cord of the brachial plexus and com- radial reflex (brachioradialis muscle). (Brazis,
prises fibers from spinal levels C5 to C8. In Masdeu, and Biller, 4549; Staal, 3548)
the axilla, the nerve gives rise to the poste-
rior cutaneous nerve of the arm, which sup- 70. (A) An extensive description of the anatomy
plies the skin over the posterior aspect of and the course of the radial nerve was
the arm as far down as the olecranon. A sec- reported in the answer to question 69. A lesion
ondary sensory branch, the posterior cuta- of the radial nerve in the upper arm causes the
neous nerve of the forearm, innervates the same symptoms as those seen in radial nerve
skin on the distal extensor aspect of the arm lesions at the axilla with sparing of the triceps
and the extensor aspect of the forearm up to and the posterior cutaneous nerve of the skin
the wrist. Within or proximal to the spiral of the arm. A radial nerve lesion in the upper
groove, the radial nerve supplies the triceps arm may by seen in alcohol-induced sleep,
and the anconeus; both are forearm exten- where acute retrohumeral nerve compression
sors. At the level of the lateral condyle of occurs. The tingling and pain that normally
the humerus, the radial nerve gives off wake normal individuals do not occur in the
branches to the brachialis muscle (an elbow inebriated. (Brazis, Masdeu, and Biller, 4549; Staal,
flexor that is also innervated by the muscu- 3548)
locutaneous nerve), the brachioradialis
muscle (a forearm flexor midway between 71. (C) A lesion of the radial nerve at the forearm
pronation and supination), and the extensor will spare the triceps, brachioradialis, and
carpi radialis longus (radial extensor of the extensor carpi radialis muscles. Typically, the
hand). The radial nerve then bifurcates into patient has finger but not wrist drop. There is
superficial and deep branches. The superfi- a radial deviation of the extended hand when
cial branch emerges in the distal forearm and the patient is asked to make a fist, illustrating
supplies the skin of the medial aspect of the the weakness of the extensor carpi ulnaris
back of the hand and the dorsum of the first compared with the extensor carpi radialis
four fingers. The deep branch is a purely muscles. (Brazis, Masdeu, and Biller, 4549; Staal,
motor nerve and is referred to as the poste- 3548)
rior interosseous nerve. It supplies the
supinator muscle (a forearm supinator), the 72. (D) The patient described in this question has
extensor carpi radialis brevis (a radial exten- purely sensory symptoms that correspond to a
80 2: Localization Signs in Neurology

wrist compression of the dorsal digital nerve. 74. (B) Dislocation of the elbow may expose the
(Brazis, Masdeu, and Biller, 2126; Staal, 3548) patient to injury of the median nerve in its
anterior interosseous branch. The purely
73. (A) The median nerve carries fibers from C5 to motor anterior interosseous nerve innervates
T1 roots. It is a mixed nerve formed in the axilla the flexor pollicis longus (a flexor of the ter-
by the joining of the lateral cord of the brachial minal phalanx of the thumb), the flexor digi-
plexus with the medial cord. The nerve descends torum profundus I and II (a flexor of the
on the medial side of the arm and enters the terminal phalanges of the second and third
forearm between the two heads of the pronator digit), and the pronator quadratus (a forearm
teres to supply the pronator teres (C6-C7), the pronator). Neurological signs in this case are
flexor carpi radialis (C6-C7) (a radial flexor of the purely motor. They include the inability of the
hand), the palmaris longus (C7-T1) (a flexor of patient to form a small circle by pinching the
the wrist), and the flexor digitorum superficialis end of the phalanx of the thumb and index
(C7-T1) (a flexor of the middle phalange of the finger together, resulting from a weakness of
second to the fifth fingers). After it passes the flexor digitorum profundus of the index
between the two heads of the pronator teres, it finger and the flexor pollicis longus, and weak-
gives off the anterior interosseous nerve. It then ness of forearm pronation on flexion because of
courses deep to the flexor retinaculum at the weakness of the pronator quadratus muscle.
wrist to reach the hand. The palmar cutaneous Elbow pronation on extension is conserved
branch takes off to the flexor retinaculum either because of the integrity of the pronator teres.
subcutaneously or through the superficial liga- (Brazis, Masdeu, and Biller, 3637; Staal, 5260)
ment fibers to supply the skin over the thenar
eminence and the proximal palm on the radial 75. (A) The patient described in this question has
aspect of the hand. The purely motor anterior the clinical features of carpal tunnel syndrome.
interosseous nerve innervates the flexor pollicis There is sensory loss in the palmar aspect of the
longus (a flexor of the terminal phalanx of the hand, weakness, and atrophy of the abductor
thumb), the flexor digitorum profundus I and II pollicis brevis and opponens pollicis. Carpal
(a flexor of the terminal phalanges of the second tunnel syndrome occurs when the median
and third digits), and the pronator quadratus (a nerve, which runs from the forearm into the
forearm pronator). At the distal end of the carpal hand, becomes pressed or squeezed at the
tunnel, the median nerve divides into its termi- wrist. The carpal tunnela narrow, rigid pas-
nal branches. The motor branches innervate the sageway of ligament and bones at the base of
first and second lumbricals (which are flexors of the handhouses the median nerve and ten-
the proximal phalanges and extensors of the dons. Sometimes thickening from irritated ten-
two distal phalanges of the second and third dons or other swelling narrows the tunnel and
fingers), the thenar muscles (which include the causes the median nerve to be compressed. The
abductor pollicis brevis, an abductor of the result may be pain, weakness, or numbness in
thumb, the opponens pollicis, and the superficial the hand and wrist, radiating up the arm.
head of the flexor pollicis brevis). Soft tissue Symptoms usually start gradually, with fre-
tumors, such as lymphomas, may cause a com- quent burning, tingling, or itching numbness in
pression of the median nerve in the upper arm. the palm of the hand and the fingers, especially
Signs of a lesion at this level include sensory the thumb and the index and middle fingers.
loss in the territory of the palmar cutaneous and The symptoms often first appear in one or both
palmar digital branches, atrophy of the thenar hands during the night, since many patients
eminence muscles, paresis of forearm pronation, sleep with flexed wrists. A patient with carpal
radial wrist flexion, distal flexion of the thumb, tunnel syndrome may wake up feeling the
palmar abduction, opposition of the thumb, and need to shake out the hand or wrist. As
flexion of the second and, to a lesser extent, third symptoms worsen, the patient might feel tin-
fingers. (Brazis, Masdeu, and Biller, 3637; Staal, gling during the day. Decreased grip strength
5260) may make it difficult to form a fist, grasp small
Answers: 7379 81

objects, or perform other manual tasks. (Brazis, 77. (E) The patient presented in this vignette has a
Masdeu, and Biller, 3637; Staal, 5260) pure motor deficit with weakness of right hand
muscles innervated by the ulnar nerve except
76. (D) The ulnar nerve carries fibers from C7 to T1 the palmaris brevis muscle. Entrapment of the
roots. Immediately distal to the elbow joint, the ulnar nerve may occur distal to Guyons canal,
ulnar nerve innervates the flexor carpi ulnaris, between the distal border of the pisiform bone
an ulnar flexor of the wrist, and the flexor digi- and the point where the superficially running
torum profundus, a flexor of the terminal pha- ulnar nerve rounds the hook of the hamate
langes of the fourth and fifth fingers. In the bone before traversing the palm to innervate
middle of the forearm, the ulnar nerve gives off the interosseous, lumbrical, and adductor pol-
the palmar cutaneous branch, which supplies licus muscles. The superficial terminal branch
the skin over the hypothenar eminence. It then goes off within the canal; accordingly, the pal-
gives off a dorsal cutaneous branch, which sup- maris brevis muscle functions normally and
plies the dorsal aspects of the hand and of the there is no sensory loss. Thus, if the pamaris
fifth and fourth fingers. In the hand, it gives off brevis muscle is preserved whereas all other
the superficial terminal branch, a sensory branch hand muscles innervated by the ulnar nerve
to the skin of the distal part of the ulnar aspect are affected and sensation is normal, the site of
of the palm and the palmar aspect of the fifth compression lies distal to the fibers going to the
finger and half of the fourth finger. It then passes palmaris brevis muscle and just proximal to
between the pisiform and hamate bones to give where the deep terminal fibers divide into
off superficial terminal branches, which are branches to the interossei and the hypothenar
mainly sensory and deep motor terminal muscles. (Brazis, Masdeu, and Biller, 4045; Staal, 80)
branches. The superficial terminal branch inner-
vates the palmaris brevis muscle. The deep 78. (C) The patient described in this vignette
branch innervates the abductor digiti minimi shows weakness of the right quadriceps and
(an abductor of the fifth finger), the opponens psoas muscles, both of which are innervated by
digiti minimi, the flexor digiti minimi (a flexor the femoral nerve. In the context of recent anti-
of the fifth finger), and the lumbricals III and IV coagulation, compression of the femoral nerve
(flexors of the metacarpophalangeal joints and by a hematoma of the psoas muscle is the most
extensors of the proximal interphalangeal joints). likely diagnosis. (Staal, 104107)
The deep muscle branch also innervates the
interosseous muscles (flexors of the metacar- 79. (A) Retinal nerve fibers enter the optic disk
pophalangeal joints and extensors of the via the disks temporal aspect, the temporal
proximal interphalangeal joints; the dorsal aspect of its superior and inferior poles, and its
interosseous muscles are finger abductors, nasal aspect. The papillomacular bundle is
whereas the palmar interosseous muscles are formed by macular fibers that enter the disks
finger adductors) and the adductor pollicis. The temporal aspect. A lesion in these fibers may
patient described in this question has a claw-like cause a central scotoma (a defect covering cen-
hand, which is characteristic of ulnar nerve tral fixation), a centrocecal scotoma (a central
injury. It is caused by the unopposed action of scotoma connected to the blind spot), or a para-
long finger extensors from the paralyzed interos- central scotoma. The arcuate bundle is formed
sei and ulnar lumbrical muscles. Weakness of by fibers from the retina temporal to the disk
hand adduction and flexion associated with that enter its superior or inferior poles. A lesion
weakness of flexion of the little finger in the of the arcuate fibers may cause a Bjerrum arcu-
distal interphalangeal joint suggests involve- ate scotoma, Seidel scotoma (a defect in the
ment of the flexor carpi ulnaris and the flexor proximal portion of the nerve fibers that causes
digitorum III and IV, respectively. These findings a comma-shaped extension of the blind spot),
localize the lesion of the ulnar nerve at the elbow or nasal step of Ronne scotoma respecting the
in the cubital tunnel. (Brazis, Masdeu, and Biller, horizontal meridian. A defect in the nasal
4045; Staal, 7980) nerves fiber bundle results in a wedge-shaped
82 2: Localization Signs in Neurology

temporal scotoma arising from the blind spot. 83. (E) Gradenigo syndrome may be caused by a
(Kline and Bajandas, 67) localized inflammation of the petrous apex fol-
lowing complicated otitis media. Contact with
80. (D) An optic tract lesion causes a contralat- the tip of the petrous pyramid makes the por-
eral hemianopia. The defect is incongruous tion of the abducens nerve within Dorellos
(not identical in shape, location, and size in canal susceptible to injury when the petrous
both eyes), since the lesion is located anterior bone is inflamed. Clinical findings include
to the occipital lobe, and nerve fibers of cor- abducens nerve palsy, ipsilateral decreased hear-
responding points do not lie adjacent to one ing, facial pain, and ipsilateral facial palsy. (Brazis,
another. An optic tract syndrome also Masdeu, and Biller, 191196; Kline and Bajandas, 8593)
includes bilateral nerve fiber atrophy and rel-
ative afferent defect on the side opposite the 84. (A) MillardGubler syndrome is caused by a
lesion. Wernicke pupil consists of absence of lesion in the ventral pons that destroys the fas-
pupillary reaction to light stimulation of a cicles of the abducens and facial nerves and
blind retina, while stimulation of an intact the corticospinal tract. It is characterized by
retina causes a normal pupillary response. ipsilateral abducens nerve palsy, ipsilateral
(Kline and Bajandas, 1013) peripheral-type facial paralysis, and contralat-
eral hemiplegia. A facial palsy can occur inde-
81. (E) Ipsilateral inferotemporal fibers and con- pendently of an abducens nerve palsy when
tralateral inferonasal fibers course anteriorly the lesion is more lateral (based in the low
from the lateral geniculate body into the tem- pons). The ipsilateral facial palsy is of the lower
poral lobe, forming Meyers loop. A right ante- motor neuron type, involving the forehead,
rior temporal lobe lesion tends to produce left eyelids, and eyebrows. (Brazis, Masdeu, and Biller,
midperipheral and peripheral pie in the sky 191196; Kline and Bajandas, 8593)
homonymous superior quadrantanopia. (Kline
and Bajandas, 1314) 85. (D) Elevation of intracranial pressure may
result in downward displacement of the brain-
82. (E) The abducens nucleus is located in the dor- stem with stretching of the abducens nerve,
socaudal portion of the pons, separated from which is tethered as it exits from the pons and
the fourth ventricle by the genu of the facial in Dorellos canal. Pseudotumor cerebri may
nerve. The abducens nuclear complex coordi- cause papilledema; in 30% of cases it also
nates the action of both eyes to produce causes bilateral abducens nerve palsy. (Brazis,
horizontal gaze by sending axons to the medial Masdeu, and Biller, 191196; Kline and Bajandas, 8593)
longitudinal fasciculus; these end in the con-
tralateral nucleus of the third nerve. Axons of 86. (B) Foville syndrome is caused by a lesion
the abducens motor neurons ascend along the located in the pontine tegmentum that destroys
base of the pons in the prepontine cistern and the fascicles of the facial nerve, the paramedian
enter Dorellos canal beneath the petroclinoid pontine reticular formation, and the corti-
ligament. In the lateral wall of the cavernous cospinal tract. It is characterized by horizontal
sinus, the abducens nerve lies between the oph- gaze palsy, dysfunction of the facial and
thalmic artery medially and the ophthalmic vestibulocochlear nerves, ipsilateral Horner
branch of the trigeminal nerve laterally. After syndrome, and contralateral hemiplegia. (Brazis,
passing through the orbital fissure, the abducens Masdeu, and Biller, 191196; Kline and Bajandas, 8593)
nerve innervates the lateral rectus muscle.
A pituitary tumor may compress the cav- 87. (D) Petrous bone fracture may follow head
ernous sinus, which can cause total ophthal- trauma. The trigeminal, abducens, facial, and
moplegia, Horner syndrome, and pain in the cochleovestibular nerves may be affected.
area innervated by the ophthalmic division of Associated signs may include otorrhea, hemo-
the trigeminal nerve. (Brazis, Masdeu, and Biller, tympanum, and mastoid ecchymosis. (Brazis,
191196; Kline and Bajandas,119124) Masdeu, and Biller, 191196; Kline and Bajandas, 8593)
Answers: 8091 83

88. (C) In RaymondCestan syndrome, the lesion ParksBielschowsky test. The median and lat-
is located in the basal pons but is less extensive eral rectus muscles do not have vertical action.
than the lesion in MillardGubler syndrome: Therefore, vertical misalignment of paretic eti-
there is only ipsilateral abducens nerve palsy ology is caused by weakness of one or more of
with contralateral hemiplegia. (Brazis, Masdeu, the following eight vertically acting muscles:
and Biller, 191196; Kline and Bajandas, 8593) right inferior oblique, left inferior oblique, right
superior oblique, left superior oblique, right
89. (B) Oculomotor nerve palsy secondary to a inferior rectus, left inferior rectus, right supe-
nuclear lesion is extremely rare. Each superior rior rectus, and left superior rectus.
rectus is innervated by the contralateral oculo- The first step is to find which is the higher
motor nerve nucleus; therefore a nuclear ocu- eye. In the vignette, the right eye is higher than
lomotor nerve lesion results in contralateral the left eye. The weak muscle is then a depres-
superior rectus palsy. Both levator muscles are sor of the right eye (right inferior rectus or right
innervated by one subnuclear structure, the superior oblique) or an elevator of the left eye
central caudal nucleus. Hence, a nuclear ocu- (left superior rectus or left inferior oblique).
lomotor nerve lesion results in bilateral ptosis. The second step is to find if the misalign-
(Kline and Bajandas, 9596) ment is worse on right or left gaze. The vertical
rectus muscles have their greatest vertical
90. (B) The pupillomotor fibers of the oculomotor action and least torsional action when the eye
nerve travel in the outer layers of the nerve and is abducted. The oblique muscles have their
are therefore closer to the nutrient blood supply. greatest vertical and least torsional action when
This may explain their susceptibility to com- the eye is adducted. In the vignette, the
pressive lesions of the oculomotor nerve rather patients hypertropia is getting worse on left
than to ischemic causes. The patient described abduction of the left eye and adduction of the
in the vignette developed a pupil-sparing iso- right eye (left gaze deviation). Therefore, the
lated oculomotor nerve palsy. This may sug- possible causes of right hypertropia are nar-
gest an ischemic oculomotor mononeuropathy. rowed from four muscles to two: worsening of
A follow-up examination revealed an Argyll the right hypertropia on right eye adduction
Robertson pupil in the affected eye, which is and left eye abduction is caused by right supe-
most likely a false ArgyllRobertson sign caused rior oblique weakness or left superior rectus
by aberrant regeneration of the oculomotor weakness, respectively.
nerve. Some of the medial rectus fibers may end The third step is to find out if the hyper-
up innervating the pupillary sphincter muscle, tropia is worse on left or right head tilt (by
so that there is more pupillary constriction moving the ear near the ipsilateral shoulder).
during convergence than in response to light. The superior muscles intort the eyes (superior
Secondary aberrant regeneration does not occur rectus and superior oblique) and the inferior
after ischemic oculomotor nerve palsy. The diag- muscles extort the eyes (inferior rectus and
nosis of ischemic oculomotor nerve palsy is inferior oblique). When the head is tilted down-
unlikely in this case. Other diagnosessuch as ward to the right shoulder, the eyes undergo
neoplasm, aneurysm, and traumashould be corrective torsion: the right eye is intorted and
considered. Primary aberrant regeneration of the left eye is extorted. Therefore, when the
the oculomotor nerve is not preceded by an head is tilted to the right, the right eye is
acute oculomotor palsy. It has an insidious intorted by contraction of the right superior
development accompanying signs of misdirec- rectus or right superior oblique. These two
tion. (Kline and Bajandas, 102103) muscles work together in affecting the intor-
sion and neutralize each others vertical action.
91. (A) If a patient has vertical misalignment due If one of these muscles is weak and thus
to recently acquired weakness of a single ver- responsible for the misalignment, then the ver-
tically acting muscle, then determination of the tical action is not neutralized and the hyper-
weak muscle follows the three steps of the tropia will become worse, as in this case. From
84 2: Localization Signs in Neurology

FIG. 2-11

steps one and two, weakness of only two mus- without flexion and supination of the forearm.
cles is left: the right superior oblique or left (Brazis, Masdeu, and Biller, 112113)
superior rectus. So, the muscle responsible for
the misalignment in this case is the right supe- 93. (C) The patient in this vignette developed
rior oblique. (Kline and Bajandas, 10113) signs of dorsal column dysfunction: loss of pro-
prioception and vibration sense in both legs as
92. (D) In a C6 radiculopathy, the biceps and bra- well as sensory ataxia. He also has bilateral
chioradialis reflexes are absent or diminished, corticospinal tract dysfunction resulting in
whereas the triceps reflex, mediated by the C7 spasticity, hyperreflexia, and bilateral Babinski
nerve root and spinal cord segments and the signs. The spinothalamic tract seems intact
finger flexor reflex, mediated by the C8 nerve because temperature and pain are conserved.
root and spinal cord segments, are exaggerated This selective damage of the posterior and lat-
as a result of injury of the corticospinal tract eral columns may occur in subacute combined
below the C6 spinal cord level. Thus, C5-C6 degeneration of the spinal cord due to vitamin
segmental lesions result in an inversion of the B12 deficiency (Figures 2-11 and 2-12). Early in
brachioradialis reflex. Tapping of the radius the course of B12 deficiency, the patient first
elicits exaggerated finger and hand flexion notices mild general weakness and paresthesias

FIG. 2-12
Answers: 9297 85

consisting of tingling, pins and needles feel- patients. The cardinal signs of tabes are loss of
ings, or other vaguely described sensations. reflexes in the legs; impaired position and vibra-
The paresthesias involve the hands and feet, tory sense; Rombergs sign; and, in almost all
more often the former, and tend to be constant cases, bilateral ArgyllRobertson pupils, which
and steadily progressive; they are the source of fail to constrict to light but accommodate. (Afifi,
much distress. As the illness progresses, the 7077; Brazis, Masdeu, and Biller, 103108; Hauser, chap-
gait becomes unsteady and stiffness and weak- ter 372; Ropper, chapter 32)
ness of the limbs, especially of the legs,
develop. If the disease remains untreated, an 95. (D) The patient described in this vignette has
ataxic paraplegia evolves, with variable pure chronic motor syndrome, including signs
degrees of spasticity. Sensory examination dis- of upper motor neuron dysfunction (paresis,
closes a disorder of the posterior and lateral spasticity, increased deep tendon reflexes, and
columns of the spinal cord, predominantly of the Babinski sign) and lower motor neuron
the former. Loss of vibration sense is by far the dysfunction (progressive muscular atrophy
most consistent sign; it is more pronounced in and fasciculations). The most likely diagnosis is
the feet and legs than in the hands and arms amyotrophic lateral sclerosis. The disease is
and frequently extends over the trunk. characterized by degenerative changes in the
Nervous system involvement in subacute com- anterior horn cells of the spinal cord, the motor
bined degeneration is roughly symmetric, and nuclei of the brainstem, and the corticospinal
sensory disturbances precede the motor ones. tract. Clinically, the onset of the disease is usu-
Mental and visual impairment may occur. ally focal or appears in one limb. Sensation is
(Afifi, 7077; Brazis, Masdeu, and Biller, 103108; usually preserved. Bulbar or pseudobulbar
Ropper, chapter 41) impairment is often superimposed, resulting
in explosive dysarthria, dysphagia, emotional
94. (E) The patient described in this vignette shows incontinence, tongue spasticity, and atrophy.
pure posterior column dysfunction with loss of Virtually, any striated muscle can be affected.
proprioception and vibratory sensation and pres- However, the urinary and rectal sphincters are
ence of the Romberg sign. Tabes dorsalis affects unaffected early in the illness because of the
the posterior columns selectively (see again sparing of Onufs nucleus, located in the ven-
Figure 2-11). Tabes dorsalis is a form of neu- tral margin of the anterior sacral horns (Figure
rosyphilis. All forms of neurosyphilis begin as a 2-13). These, as well as the extraocular mus-
meningitis, and a more or less active meningeal cles, are affected late in the illness. (Afifi, 7077;
inflammation is the invariable accompaniment of Brazis, Masdeu, and Biller, 103108)
all forms of neurosyphilis. The early clinical syn-
dromes are meningitis and meningovascular 96. (A) The patient in this vigentte shows a central
syphilis; the late ones are vascular syphilis (1 to 12 spinal cord syndrome with dissociation of sen-
years), followed still later by general paresis, tabes sory loss that is best exemplified by
dorsalis, optic atrophy, and meningomyelitis. The syringomyelia. Cord damage starts centrally
classic syndromes of tabes dorsalis as well as and spreads centrifugally to involve other
meningovascular syphilis of the spinal cord are spinal cord structures. Characteristically, the
now less frequent than in the past but must be decussating fibers of the spinothalamic tract,
considered in the differential diagnosis of spinal conveying pain and temperature sensation, are
cord disorders. The characteristic symptoms of affected first. This results in thermoanesthesia
tabes are fleeting and repetitive lancinating pains, and analgesia with suspended bilateral distri-
primarily in the legs or less often in the back, bution and preservation of sensation to light
thorax, abdomen, arms, and face. Ataxia of the touch as well as proprioception (Figure 2-14).
legs and gait due to loss of position sense occurs (Afifi, 7077; Brazis, Masdeu, and Biller, 103108)
in half of these patients. Paresthesias, bladder
disturbances, and acute abdominal pain with 97. (C) The patient described in this vignette
vomiting (visceral crisis) occur in 15% to 30% of developed a neurological deficit involving the
86 2: Localization Signs in Neurology

FIG. 2-13

territory of the anterior spinal artery. Spinal of pain and temperature. This is highly sug-
cord infarction is rare. The syndrome is char- gestive of right hemisection of the spinal cord
acterized by the abrupt onset of leg weakness at the T6 level. The weakness is caused by a
and urinary incontinence associated with loss lesion in the ipsilateral corticospinal tract; the
of thermoanesthesia and analgesia below the loss of proprioception is related to interrup-
level of the lesion. Position sense, vibration, tion of the ipsilateral ascending fibers of the
and light touch remain intact owing to preser- posterior columns. The loss of pain and tem-
vation of the dorsal columns (Figure 2-15). perature sensation in the contralateral side is
(Afifi, 7077; Brazis, Masdeu, and Biller, 103108) related to a lesion of the decussating spinothal-
amic tract (Figure 2-16) (Afifi, 7077; Brazis,
98. (A) The patient described in this vignette Masdeu, and Biller, 103108)
developed weakness of the right lower extrem-
ity and loss of proprioceptive function below 99. (B) This patient has funnel vision only when
the T6 spinal cord level, with contralateral loss the central vision is intact. Funnel vision should

FIG. 2-14
Answers: 98100 87

FIG. 2-15

not be confused with tunnel vision, a field thy, postpapilledema optic atrophy, and bilat-
defect characteristic of hysteria or malinger- eral occipital infarct with macular sparing.
ing. The latter field can easily be mapped onto (Brazis, Masdeu, and Biller, 144)
a tangent screen by plotting the fields with the
patient seated 1 and 2 meters from the screen. 100. (A) Disease of the choroid, retinal pigment
In the case of organic defect, the field projected epithelium, retina, optic disk, or optic nerve
at 2 meters is larger than the field plotted at 1 almost always causes monocular visual defects.
meter. A constricted visual field with retained The early stage of a chiasmatic lesion may
acuity may be seen in the case of glaucoma, cause monocular loss of vision in the temporal
retinitis pigmentosa, cancer-associated retinopa- field of the ipsilateral eye when the defect is

FIG. 2-16
88 2: Localization Signs in Neurology

located most anterior in the chiasm, affecting external ear are peripheral processes of neurons
the nasal retinal fibers crossing from the con- in the geniculate ganglion. Central processes
tralateral eye. A lesion located in the most ante- project on neurons in the spinal trigemnal
rior aspect of the calcarine cortex causes a nucleus. The neurons of the taste fibers origi-
crescent-shaped defect restricted to the tempo- nate in the geniculate ganglion. Peripheral
ral field of the contralateral eye. This is the only processes of these neurons reach the taste buds
retrochiasmatic lesion that may cause a unilat- in the anterior two thirds of the tongue; central
eral visual defect. (Brazis, Masdeu, and Biller, processes enter the brainstem with the nevus
144146) intermedius and project on neurons in the gus-
tatory part of the nucleus solitarius, along with
101. (E) The facial nerve innervates the muscles of fibers carried by the glossopharyngeal (from
facial expression and all other muscles derived the posterior third of the tongue) and vagus
from the second branchial arch and carries the (from the epiglottic region) nerves. The sen-
sensation of taste from the front of the tongue. sory and the gustatory fibers, along with the
It is both a sensory and a motor nerve. In addi- visceral motor component, form a separate lat-
tion, it has a parasympathetic role as described eral root of the facial nerve, the nervus inter-
below. Its fibers originate at the pontomedullary medius. The motor part of the facial nerve
junction, leave the posterior cranial fossa carries two types of motor fibers: somatic and
through the internal acoustic meatus, and enter secretomotor. The somatic motor fibers supply
the facial canal in the petrous part of the tem- the muscles of facial expression and the
poral bone. It has a motor root, and another root, stapedius, the stylohyoid and the posterior
the nervus intermedius, that is responsible for belly of the digastric. These fibers arise from the
carrying the sensation of taste and for parasym- facial motor nucleus in the pontine tegmen-
pathetic innervation (Figure 2-17). tum. They course dorsomedially and then ros-
The sensory component of the facial nerve trally in the tegmentum before bending
carries two types of sensory afferents: exterio- laterally over the abducens nucleus and turn-
ceptive fibers from the external ear and taste ing ventrolaterally to emerge at the lateral
fibers from the anterior two thirds of the border of the pons. After emerging from the
tongue. The exterioceptive fibers from the ventrolateral pons, the motor division enters

SENSORY COMPONENTS MOTOR COMPONENTS

Spinal Superior Facial


trigeminal Nucleus Abducens salivatory motor
nucleus solitarius nucleus nucleus nucleus

Geniculate ganglion

Chorda tympani and


lingual nerves

Submandibular
gland
Tongue Chorda tympani

Great superficial
petrosal nerve Sublingual
gland
Pterygopalatine Lacrimal Submandibular
ganglion gland ganglion
FIG. 2-17
Answers: 101108 89

the internal auditory meatus of the temporal 103. (C) Ramsay Hunt syndrome results from a
bone. Four portions of the facial nerve can be herpes zoster infection of the geniculate gan-
distinguished within the temporal bone: the glion. Clinical features may include hyperacu-
meatal segment, the labyrinthine segment (from sis, loss of taste in the anterior two thirds of the
which the greater superficial petrosal nerve tongue, geniculate neuralgia, and herpetic vesi-
arises), the horizontal segment, and the mas- cles of the external auditory meatus (Figure 2-
toid segment (from which the nerve to the 17). (Afifi, 112114; Brazis, Masdeu, and Biller,
stapedius muscle and the chorda tympani 287296)
arises). After giving off the chorda tympani, the
facial nerve exits the facial canal through the 104. (D) A lesion affecting the facial nerve, between
stylomastoid foramen. Near its exit, it gives rise the departure of the nerve to the stapedius and
to the posterior auricular nerve (to the occipi- the departure of the chorda tympani, results
talis, posterior auricular, and transverse and in ipsilateral facial palsy and loss of taste sen-
oblique auricular muscles), the digastric branch sation in the anterior two thirds of the tongue.
(to the posterior belly of the digastric muscle), Hearing is spared (Figure 2-17). (Afifi, 112114;
and the stylohyoid branch (to the stylohyoid Brazis, Masdeu, and Biller, 287296)
muscles). The facial nerve then pierces the
parotid gland, where it divides into the tempo- 105. (B) A lesion of the facial nerve within the facial
rofacial and cervicofacial branches, which fur- canal, distal to the meatal segment but proxi-
ther divide into several branches to supply all mal to the departure of the nerve to the
the facial mimetic muscles and the platysma stapedius muscle, results in ipsilateral facial
muscle. The secretomotor fibers arise from the motor paralysis, loss of taste over the anterior
superior salivary nucleus in the tegmentum of two thirds of the tongue, and hyperacusis.
the pons. They are preganglionic fibers that Lacrimation is preserved if the lesion is distal to
leave the brainstem with the nevus intermedius the greater superficial petrosal nerve (Figure
and travel in the greater superficial nerve and 2-17). (Afifi, 112114; Brazis, Masdeu, and Biller,
the nerve of the pterygoid canal before synaps- 287296)
ing in the pterygopalatine ganglion, from which
postganglionic parasympathetic fibers travel in 106. (C) A lesion of the facial nerve in the meatal
the maxillary, zygomatic, zygomaticotemporal, canal affects the facial and cochleovestibular
and lacrimal nerves to reach the lacrimal gland. nerves: there is ipsilateral facial nerve palsy,
Fibers destined for the submandibular and impaired taste sensation in the anterior two
sublingual glands join the chorda tympani and thirds of the tongue, impaired lacrimation, and
the lingual nerves and synapse in the sub- deafness (Figure 2-17). (Afifi, 112114; Brazis,
mandibular ganglion, from which postgan- Masdeu, and Biller, 287296)
glionic parasympathetic fibers arise. A lesion
of the facial nerve at the stylomastoid foramen 107. (A) A fascicular lesion of the facial nerve
produces isolated ipsilateral motor palsy with- results in a peripheral type of facial nerve palsy
out loss of hearing or taste (Figure 2-17). (Afifi, (Figure 2-17). A lesion located in the ventral
112114; Brazis, Masdeu, and Biller, 287296) pons and the corticospinal tract, destroying the
fascicles of the facial and abducens nerves,
102. (A) A lesion located in the pontine tegmentum causes the MillardGubler syndrome. This syn-
that destroys the fascicle of the facial nerve, drome is characterized by an ipsilateral periph-
the paramedian pontine reticular formation, eral facial nerve palsy, ipsilateral lateral rectus
and the corticospinal tract causes Foville syn- palsy, and contralateral hemiplegia. (Afifi, 112
drome. This is characterized by an ipsilateral 114; Brazis, Masdeu, and Biller, 287296)
peripheral-type facial palsy, paralysis of the
conjugate gaze to the side of the lesion, and 108. (B) The patient described in this vignette has
contralateral hemiplegia (Figure 2-17). (Afifi, decreased pain and temperature sensation in
112114; Brazis, Masdeu, and Biller, 287296) the right face and left side of the trunk and
90 2: Localization Signs in Neurology

extremities. This results from a lesion affecting paresis, aphonia, and impairment of horizontal
the right trigeminal spinal nucleus/tractus and eye movement with preservation of vertical
the right spinothalamic tract, respectively. A right gaze and maintenance of consciousness. These
nucleus ambiguus lesion causes weakness of the signs are highly suggestive of the locked-in
right palate and vocal cord paralysis, resulting in syndrome. It results from a ventral pontine
hoarseness. Ataxia of the right limb may be lesion, which may be caused by a vertebral
explained by a right cerebellar lesion. All these artery thrombosis, ventral pontine tumor, hem-
clinical findings are highly suggestive of a lateral orrhage, trauma, or central pontine myelinoly-
medullary lesion, also known as Wallenberg syn- sis from rapid correction of hyponatremia. The
drome. It is most often related to obstruction of quadriplegia is caused by bilateral corticospinal
the intracranial vertebral artery or the pos- tract lesions, the aphonia is caused by a lesion
teroinferior cerebellar artery. The syndrome has in the corticobulbar fibers that innervate the
been reported with cocaine abuse, medullary lower cranial nerves, and the ophthalmople-
metastasis, trauma, abscess, and demyelination. gia with impairment of horizontal eye move-
The clinical features of this syndrome may ment results from a lesion of the abducens
include Horner syndrome (due to injury to the nerve fascicles. The patient is fully awake
descending sympathetic fibers), vertigo, and because the reticular formation is spared. The
vomiting from involvement of the vestibular supranuclear oculomotor pathways as well as
nuclei. A lateral lesion of the rostral medulla is the blinking pathways are also spared because
associated with more severe dysphagia and dys- they lie more dorsally. Thus, the patient is only
phonia, whereas a caudal lesion is associated able to blink and to look up and down. (Brazis,
with more marked vertigo, nystagmus, and gait Masdeu, and Biller, 358359)
ataxia. (Brazis, Masdeu, and Biller, 352356)
112. (A) The patient described in this vignette has
109. (A) The patient described in this vignette has a right cerebellar syndrome and impairment
symptoms consistent with a lesion located in of the right spinothalamic and corticospinal
the right pyramidal system, right medial lem- tracts. A lesion of the lateral pons would
niscus, and right hypoglossal nerve, which explain these signs, which correspond to
explains the protrusion of the tongue away MarieFoix syndrome. (Brazis, Masdeu, and Biller,
from the hemiplegia toward the side of the 359360)
lesion. These symptoms are consistent with the
diagnosis of right medial medullary syndrome. 113. (C) The 40-year-old diabetic woman described
This may be caused by occlusion of the right in this question has signs highly suggestive of
anterior spinal artery or its parent, the vertebral a lesion affecting the mesencephalic tegmen-
artery. (Brazis, Masdeu, and Biller, 351352) tum (Benedict syndrome). Ophthalmoplegia is
caused by fascicular damage to the third cranial
110. (D) The neurological examination of the patient nerve on the ipsilateral side of the lesion, which
in this vignette demonstrates right facial palsy is on the right side in this case. The left sided
and contralateral hemiplegia. These findings tremor is caused by destruction of the right red
point to a lesion affecting the right pons. The nucleus, causing a rubral tremor. (Brazis,
presence of gaze deviation to the left may suggest Masdeu, and Biller, 362)
a lesion of the paramedian pontine reticular for-
mation or a right abducens lesion. The association 114. (B) In this question, the patient developed
of these findings puts the lesion in the right dorsal right oculomotor paresis and left hemiplegia.
pontine tegmentum in the caudal third of the This is compatible with a lesion in the right
pons. This is consistent with a diagnosis of Foville cerebral peduncle, which affects the right ocu-
syndrome. (Brazis, Masdeu, and Biller, 359) lomotor nerve fascicles and right corticospinal
tract, causing left hemiplegia. These signs are
111. (C) In this vignette,, the patients neurological consistent with the diagnosis of Weber syn-
assessment shows the association of quadri- drome. (Brazis, Masdeu, and Biller, 361362)
Answers: 109125 91

115. (D) The patient described in this question has peduncle. The left sided pain and temperature
signs highly suggestive of Sylvian aqueduct loss is caused by involvement of the spinothal-
syndrome. This results from a dorsal rostral amic tract. (Brazis, Masdeu, and Biller, 367378)
mesencephalic lesion. It is most often seen with
pineal gland tumors that cause hydrocephalus. 120. (C) The case described in this vignette results
The syndrome may include a paralysis of con- most likely from an embolic obstruction of the
jugate upward gaze and convergence retrac- posteroinferior cerebellar artery with infarction
tion nystagmus on upward gaze. (Brazis, of the inferior cerebellum and lateral medulla.
Masdeu, and Biller, 362) The loss of pain and temperature sensation is
caused by damage to the trigeminal spinal
116. (B) The clinical case described in this vignette nucleus and tract, the left limb ataxia is caused
shows a 5-year-old boy complaining of axial by damage of the inferior cerebellar peduncle,
ataxia without limb ataxia and spontaneous nys- the dysarthria and left vocal cord palsy are
tagmus. These symptoms are consistent with caused by damage to the left nucleus ambiguus,
damage to the floculonodular lobe in the caudal and the loss of pain and temperature sensation
part of the vermis, caused in this case by a on the right is caused by damage to the
medulloblastoma. (Brazis, Masdeu, and Biller, spinothalamic tract on the left. (Brazis, Masdeu,
367378) and Biller, 367378)

117. (D) The neurological abnormalities reported 121. (B) An acute hypothalamic lesion can cause
in the case described in this vignette suggest gastrointestinal erosions (called neurogenic
occlusion of the right anteroinferior cerebellar ulcers) that are most often located in the lower
artery. The vertigo is caused by ischemia of the esophagus. (Brazis, Masdeu, and Biller, 389)
vestibular nuclei. The right facial palsy and the
loss of sensation are caused by ipsilateral 122. (C) The medial dorsal nucleus of the thalamus
ischemia of the lateralpontomedullary is found to be most consistently associated with
tegmentum and the trigeminal nuclei and tract, memory loss in WernickeKorsakoff syndrome.
respectively. The right Horner syndrome is (Brazis, Masdeu, and Biller, 393)
caused by a compromise of the descending
oculosympathetic fibers. (Brazis, Masdeu, and 123. (B) Hemiballismus usually occurs with injury
Biller, 367378) to the contralateral subthalamic nucleus or any
lesion that disrupts its afferent or efferent
118. (A) The patient described in this vignette has a fibers. Hemiballismus may also be caused by
predominant axial ataxia. With the history of lesions affecting the caudate, putamen, globus
ethanol abuse, the most likely diagnosis is cere- pallidus, precentral gyrus, and thalamic nuclei.
bellar degeneration from chronic ethanol abuse. (Brazis, Masdeu, and Biller, 428429)
Ethanol results in atrophy of the anterior and
superior vermis. (Brazis, Masdeu, and Biller, 367- 124. (A) Sensory inattention occurs most com-
378) monly with a lesion of the contralateral inferior
parietal lobe. Less commonly, it may occur with
119. (E) The patient described in this vigentte has lesions of the temporoparietooccipital junction,
symptoms consistent with occlusion of the dorsolateral frontal lobe, cingulate gyrus, thal-
superior cerebellar artery. The vertigo and nys- amus, and mesencephalic reticular formation.
tagmus are caused by ischemia of the vestibu- (Brazis, Masdeu, and Biller, 466469)
lar nuclei. The Horner syndrome is caused by
a compromise of the descending oculosympa- 125. (D) The septal area has two divisions: the
thetic fibers; the left deafness results from a septum pellucidum and septum verum. The
lesion in the crossed right lateral lemniscus; septum pellucidum is a thin leaf that separates
and the right tremor is caused by a lesion of the the lateral vetricles. The septum verum is ven-
dentate nucleus and the superior cerebellar tral to the septum pellucidum; it is located
92 2: Localization Signs in Neurology

between the subcallosal gyrus rostrally and the sation, and incontinence. (Brazis, Masdeu, and
anterior commissure and the anterior hypo- Biller, 503; Trimble, 89104)
thalamus caudally. The septal area has recip-
rocal connection with the following areas: 129. (B) Emotions and their expression depend on
hippocampus, amygdala, hypothalamus, mid- the individuals state of arousal (mediated by
brain, habenular nucleus, cingulated gyrus, the reticular activating system), vegetative func-
and thalamus. Lesion of the septal area in tion (mediated in part by the hypothalamus),
animal species such as rats and mice produce retrieval system for previous experience (medi-
rage reactions and hyperemotionality. These ated by the hippocampus and other portions
behavioral alterations are usually transitory of the limbic system), ability to perceive, ability
and disappear 2 to 4 weeks after the lesion to evaluate stimuli that carry an affective com-
appears. Animals with septal damage tend to ponent, and ability to express emotion. A defect
consume increased amounts of water, to in the ability to perceive stimuli that carry affec-
demonstrate a high intitial state of activity in tive components may be seen in right pari-
response to novel stimulation, to learn tasks etotemporal damage. The patient understands
quickly, andonce they have been learnedto the semantic meaning of a verbal threat, but his
perform them effectively. (Afifi, 293294) or her perception of the emotional overtones
that accompany the utterance is impaired. This
126. (E) The orbitofrontal cortex is the part of the condition is known as sensory aprosodia.
frontal cortex resting above the orbits of the (Brazis, Masdeu, and Biller, 472474; Trimble, 89104)
eyes. It is defined as the part of the prefrontal
cortex that receives projections from the mag- 130. (C) Patients with bilateral anterior temporal
nocellular, medial, or mediodorsal thalamus. lesions have a bland affect. Other manifesta-
Because the orbitofrontal cortex is interlinked tions of bilateral temporal damage were
with limbic and reticular areas, lesions of this obtained from survivors of herpes simplex
area lead to disinhibition and changes of affect. encephalitis. They include memory disturbance,
Behavior is thus impulsive (pseudopsycho- hypermetamorphosis, irritability, eating and
pathic). Other characteristics include an inap- drinking problems, agnosia, inappropriate
propriate jocular affect, euphoria, emotional sexual display, and easy distractibility. (Brazis,
liability, poor judgment and insight, and dis- Masdeu, and Biller, 472474; Trimble, 89104)
tractibility. (Brazis, Masdeu, and Biller, 503; Trimble,
89104) 131. (D) The ability to evaluate the importance of
stimuli may be impaired in the case of bilateral
127. (C) The lateral frontal cortex is closely linked to lesions of the anterior cingulate; the patient
motor structures; therefore lesions of this area may be unconcerned in the presence of painful
lead to disturbances of movement and action stimuli. (Brazis, Masdeu, and Biller, 472474; Trimble,
with preservation of inertia. Patients are apa- 89104)
thetic, with occasional bursts of angry or
aggressive behavior. Other characterestics 132. (E) A lesion of the left dorsofrontal lobe may
include indifference, psychomotor retardation, cause anger and hostility, whereas a lesion of the
motor preservation and impersistence, dis- right orbitofrontal area may cause depression.
crepant motor and verbal behavior, and poor (Brazis, Masdeu, and Biller, 472474; Trimble, 89104)
word listing and visuospatial analysis. (Brazis,
Masdeu, and Biller, 503; Trimble, 89104) 133. (E) An epileptogenic right temporal lesion may
cause paranoid behavior, whereas an epilep-
128. (B) Medial frontal lobe syndrome is associated togenic lesion of the left temporal lobe may
with mutism, gait disturbance, and inconti- cause denial, sadness, and/or elation.
nence. Patients demonstrate a paucity of spon-
taneous movement and gesture, sparse verbal 134. (E) Focal seizures arising from the neocortex of
output, lower extremity weakness, loss of sen- the temporal lobe may give rise to experiential
Answers: 126141 93

illusions such as dj vcu (previously experi- attention affecting mainly the peripheral visual
enced) or jamais vcu (never previously expe- field. Altitudinal neglect may also be seen.
rienced). They may also give rise to visual (Brazis, Masdeu, and Biller, 481)
illusions such as dj vu (previously seen) or
jamais vu (never previously seen). (Brazis, 140. (A) The presence of left homonymous hemi-
Masdeu, and Biller, 477481) anopia without neglect and preservation of
response to threat as well as preservation of
135. (C) In occipital epileptic seizures, hallucina- drawing and copying ability is highly sugges-
tions are predominantly multicolored with a tive of damage to the right occipitoparietal
circular or spherical pattern. They are also more region in area 17 of the occipital cortex. It is
stereotyped than hallucinations from impaired usually caused by obstruction of the calcarine
visual acuity and are associated with other branch of the posterior cerebral artery. Right
seizure manifestations. (Brazis, Masdeu, and Biller, parietal and temporoparietal lesions involving
477481) areas 18 and 19 cause lack of awareness of
visual loss, contralateral neglect and abnormal
136. (D) A lesion of the upper midbrain bilaterally optokinetic nystagmus, lack of response to
may cause a complex visual hallucination that visual threat, and abnormal drawing and copy-
has a dream-like quality. Hallucinations are ing. Bilateral occipital lesions involving the
often hypnagogic, known to be unreal, and visual cortex in areas 17, 18, and 19 cause blind-
may be pleasant to the patient. (Brazis, Masdeu, ness, agitation, and amnesia. A lesion of the
and Biller, 477481) inferior calcarine fissure bilaterally causes
prosopagnosia, bilateral upper quadranopsia,
137. (A) Charles Bonnet syndrome may occur in and achromatopsia. A bilateral lesion of the
elderly patients with poor vision. Probably a superior calcarine fissure causes inferior quad-
release phenomenon, it is characterized by hal- ranopsia and Balint syndrome. (Brazis, Masdeu,
lucinations in the evening of small, brightly and Biller, 477482)
colored people or objects with a cartoon-like
appearance. The patient is usually aware of the 141. (C) Alien limb signs includes failure to recog-
unreality of these hallucinations. (Brazis, nize ownership of ones limb when visual cues
Masdeu, and Biller, 477481) are removed, feeling that ones body part is
foreign, personification of the affected body
138. (B) Simple visual hallucinations consisting of part, and autonomous activity of the limb that
flashes of light or lines of different colors, but is perceived as being outside of the patients
predominantly adopting zigzag or fortification voluntary control. Although the hand is most
patterns, are suggestive of migraine headache. frequently affected, any limb or combination of
(Brazis, Masdeu, and Biller, 477481) limbs may fulfill the alien limb criteria. The
diagnosis of alien hand sign should be reserved
139. (A) Balint syndrome may result from bilateral for cases where the hand feels foreign together
parietooccipital lesions in the convexities of with observable involuntary motor activity.
the hemispheres. It is characterized by the fol- Etiologies of this syndrome include multiple
lowing symptoms: (1) simultagnosia, or the infarcts and corticobasal degeneration. Two
inability to appreciate the meaning of the distinct alien hand syndromes have been
whole, though the elemental parts are well rec- described: (1) The frontal hand alien limb syn-
ognized; (2) gaze apraxia, or failure to shift drome occurs in the dominant hand and is
gaze on command and difficulty in the volun- associated with reflexive grasping, groping,
tary redirection of attention; (3) optic ataxia, or and compulsive manipulation of tools. It
disturbance reaching a target under visual con- results from damage to the supplementary
trol, manifested by clumsiness of object-bound motor area, anterior cingulate gyrus, medial
movements of the hand performed under prefrontal cortex of the dominant hemisphere,
visual guidance; and (4) decreased visual and anterior corpus callosum. (2) The callosal
94 2: Localization Signs in Neurology

form is characterized primarily by intermanual 146. (A) A mesial occipital lesion may cause a
conflict and requires only an anterior callosal visual field defect with visual hallucinations,
lesion. The occurrence of frontal alien hand visual agnosia, alexia without agraphia, and
syndrome in the dominant limb can be cortical blindness. (Brazis, Masdeu, and Biller,
explained by an increased tendency for domi- 506507)
nant limb exploratory reflexes coupled with
release from an asymmetrically distributed, 147. (D) A lesion of the lateroinferior aspect of the
predominant nondominant-hemisphere inhi- nondominant temporal lobe causes impaired
bition. Callosal alien hand syndrome is best recognition of facial emotional expression and
explained by hemispheric disconnection man- storage of nonverbal patterned materials such
ifested during behaviors requiring dominant- as geometric or tonal patterns. (Brazis, Masdeu,
hemisphere control. (Doody and Jankovic, 806810; and Biller, 506507)
Feinberg, Schindler, Flanagan, and Haber, 1924)
148. (D) A lesion of the orbitofrontal area may
142. (A) Lesions in the perirolandic cortex cause cause blunted affect and impaired apprecia-
impairment of the fine distal movements of the tion of social nuances, impaired goal-directed
contralateral hand. Picking up small objects by behavior, impotence, apraxia of speech, envi-
opposing the index finger and the thumb or ronmental dependency syndrome, and face-
handling a small coin may become impossible. tiousness. (Brazis, Masdeu, and Biller, 506507)
This type of apraxia has been termed limb-
kinetic apraxia. Because separate fine move- 149. (B) A lateral occipital lesion may cause alexia
ments of each finger are difficult, these patients with agraphia, impaired optokinetic nystag-
pick up a pen or a coin by pressing it against mus, palinopsia, visual allesthesia, and
the palm with the proximal portion of the impaired ipsilateral scanning. (Brazis, Masdeu,
thumb, much as infants do before they develop and Biller, 506507)
a pincer grip. (Brazis, Masdeu, and Biller, 497)
150. (C) A bilateral lesion of the anterior tip of the
143. (E) A parietal postcentral gyrus lesion causes temporal lobe causes KluverBucy syndrome
somatosensory disturbance with contralateral with visual agnosia, oral exploratory behavior,
proprioception, pain, and temperature loss hypersexuality, hypomobility, and a marked
accompanied by tactile extinction, paresthe- tendency to take notice of and attend to every
sias, and pain. (Brazis, Masdeu, and Biller, 506507) visual stimulus. (Brazis, Masdeu, and Biller,
506507)
144. (E) A lesion of the lateral frontal premotor area
may cause impaired contralateral saccades, 151. (B) A lesion of the parietal medial lobe may
pure agraphia (dominant hemisphere), con- cause transcortical sensory aphasia when it
tralateral weakness of the shoulder (mainly in affects the dominant lobe. (Brazis, Masdeu, and
abduction and elevation of arm), and weakness Biller, 506507)
of the hip muscles associated to limb-kinetic
apraxia. (Brazis, Masdeu, and Biller, 506507) 152. (E) A lesion of the medial frontal lobe, the cin-
gulate gyrus, causes akinesia, perseveration,
145. (D) A callosal lesion may cause lack of kinetic bilateral ideomotor apraxia, difficulties with
transfer with left-hand apraxia and agraphia, intiating contralateral arm movement, and the
right-hand constructional apraxia, and inability alien hand sign. (Brazis, Masdeu, and Biller,
to mimic the position of the contralateral hand. 506507)
Other neurological findings include interman-
ual conflict (alien left hand) as well as perplex- 153. (C) A lesion of the lateral parietal lobe results
ity and confabulation in trying to explain in alexia with agraphia (when the lesion affects
left-hand activity and double hemianopia. the angular gyrus), finger agnosia, acalculia,
(Brazis, Masdeu, and Biller, 506507) and rightleft disorientation. When the lesion
Answers: 142155 95

affects the dominant side, the patient may the scalenus anterior muscle. It enters the
develop parietal apraxia, finger agnosia, acal- thorax by passing between the subclavian
culia, rightleft disorientation, literal alexia, artery and vein. It lies anterior to the hilum of
and conduction aphasia. When the lesion affects the lung, between the pleura and the peri-
the nondominant side, the patient may develop cardium. The right and left phrenic nerves
anosognosia, autotopagnosia, spatial disorienta- pierce the diaphragm to supply it. The phrenic
tion, hemispatial neglect, constructional apraxia, nerve supplies the mediastinal pleura with sen-
dressing apraxia, loss of topographic memory, sory fibers. Endarterectomy may cause injury
allesthesia, hemisomatognosia, and asymbolia to to the cervical plexus. A lesion of the greater
pain. (Brazis, Masdeu, and Biller, 506507) auricular nerve causes loss of sensation of the
mandible and lower external ear. A lesion of the
154. (C) The cervical plexus is a plexus of the ven- muscular branches of the cervical nerves causes
tral rami of the first four cervical spinal nerves, weakness of the infrahyoid and scalene mus-
which are located in the C1 to C4 cervical seg- cles, resulting in inability to flex the head and
ment of the neck. They are located laterally to rotate it laterally. (Brazis, Masdeu, and Biller, 7374)
the transverse processes between prevertebral
muscles from the medial side and the verte- 155. (E) The sacral plexus is formed by the lum-
bral muscles (scalenus, levator scapulae, and bosacral trunk, the anterior division of the first,
splenius cervicis) from the lateral side. Nerves and portions of the anterior divisions of the
formed from the cervical plexus innervate the second and third sacral nerves. The lum-
back of the head as well as some neck muscles. bosacral trunk comprises the whole of the ante-
The cervical plexus has two types of branches: rior division of the fifth and a part of the fourth
cutaneous and muscular. The cutaneous lumbar nerve; it appears at the medial margin
branches are the lesser occipital nerve (which of the psoas major and runs downward over
innervates the lateral part of occipital region), the pelvic brim to join the first sacral nerve.
the greater auricular nerve (which innervates The anterior division of the third sacral nerve
skin near the concha acustica and external divides into an upper and a lower branch, the
acoustic meatus), the transverse cervical nerve former entering the sacral and the latter the
(which innervates the anterior region of neck), pudendal plexus. The nerves forming the sacral
the supraclavicular nerves (which innervate plexus converge toward the lower part of the
the region of the suprascapularis, shoulder, and greater sciatic foramen and unite to form a flat-
upper thoracic region). The deep branches of tened band, from the anterior and posterior
the cervical plexus innervate various muscles. surfaces of which several branches arise. The
The fibers from C1 innervate the deep muscles band itself is continued as the sciatic nerve.
of the neck. Some fibers from C1 run to join the which splits on the back of the thigh into the
hypoglossal nerve, from which they innervate tibial and common peroneal nerves. The sacral
the thyrohyoid and geniohyoid muscles. Fibers plexus lies on the back of the pelvis between
from C1 and C2 join to form the superior root the piriformis and the pelvic fascia; in front of
of the ansa cervicalis. Fibers of C2 and C3 join it are the hypogastric vessels, the ureter, and the
to form the inferior root of the ansa cervicalis. sigmoid colon. The superior gluteal vessels run
The inferior loop of the ansa cervicalis lies ante- between the lumbosacral trunk and the first
rior to the internal jugular vein and passes sacral nerve, and the inferior gluteal vessels
upward to join the superior root. The ansa cer- between the second and third sacral nerves.
vicalis supplies the strap muscles. Fibers from All the nerves entering the plexus with the
C2, C3, and C4 supply the sternocleidomas- exception of the third sacral split into ventral
toid, trapezius, and levator scapulae. The spinal and dorsal divisions. The nerves arising from
accessory nerve also supplies the sternocleido- these are the pudendal nerve (S2, S3, and S4),
mastoid and trapezius. The phrenic nerve is nerves to the piriformis and obturator inter-
formed by fibers from C3, C4, and C5. The nus muscles, pelvic splanchnic nerves (S2, S3,
nerve passes downward from its origin to lie on and S4), the sciatic nerve (L4, L5, S1, S2, and
96 2: Localization Signs in Neurology

S3), the superior gluteal nerve (gluteus medius plexus pass between the anterior and middle
and minimus), the inferior gluteal nerve (glu- scalene muscles. The superior trunk lies closest
teus maximus), the nerve to the quadratus to the surface and is formed by the C5 and C6
femoris, the posterior cutaneous nerve of the roots. The suprascapular nerve and the nerve to
thigh (skin of buttock and back of thigh) and the subclavius arise from the superior trunk.
the perforating cutaneous nerve (medial part of The suprascapular nerve contributes sensory
the buttock). A lesion of the sacral plexus fibers to the shoulder joint and provides motor
results in motor disturbances in the field of dis- innervation to the supraspinatus and infra-
tribution of the superior gluteal, inferior gluteal, spinatus muscles. The C7 root continues as the
and sciatic nerves. Weakness of dorsiflexion middle trunk and the C8 and T1 roots join to
and plantarflexion results in a flail foot. There is form the inferior trunk. The trunks divide into
weakness of knee flexion, abduction, and inter- anterior and posterior divisions, which sepa-
nal rotation of the thigh as well as weakness of rate the innervation of the ventral and dorsal
hip extension. The Achilles reflex is decreased halves of the upper limb. The cords are called
or absent. (Brazis, Masdeu, and Biller, 8184) the lateral, posterior, and medial cord, accord-
ing to their relationship to the axillary artery.
156. (D) The anterior rami of the spinal nerves of C5, The cords pass over the first rib close to the
C6, C7, C8, and T1 form the roots of the brachial dome of the lung and continue under the clav-
plexus. These emerge from the transverse icle immediately posterior to the subclavian
processes of the cervical vertebrae immediately artery. The lateral cord receives fibers from the
posterior to the vertebral artery. The spinal nerves anterior divisions of the superior and middle
that form the brachial plexus run in an inferior trunk and is the origin of the lateral pectoral
and anterior direction within the sulci formed by nerve (C5, C6, and C7). The posterior divisions
these structures. The trunks of the brachial plexus of the superior, middle, and inferior trunk com-
pass between the anterior and middle scalene bine to form the posterior cord. The upper and
muscles. The superior trunk lies closest to the lower subscapular nerves (C7, C8, C5, and C6)
surface and is formed by the C5 and C6 roots. The leave the posterior cord and descend behind
C7 root continues as the middle trunk and the C8 the axillary artery to supply the subscapularis
and T1 roots join to form the inferior trunk. The and teres major muscles, respectively. The thor-
trunks divide into anterior and posterior divi- codorsal nerve to the latissimus dorsi, also
sions, which separate the innervation of the ven- known as the middle subscapular nerve (C6,
tral and dorsal halves of the upper limb. The C8 C7, and C8), also arises from the posterior cord.
and T1 root and the lower trunk of the brachial The inferior trunk continues as the medial cord
plexus can be compressed, resulting in the rare and gives off the median pectoral nerve (C8,
entity called thoracic outlet syndrome, which T1), the medial brachial cutaneous nerve (T1),
may have vascular or neurological signs. Vascular and the medial antebrachial cutaneous nerve
signs are recurrent cyanosis, coldness, and pallor (C8, T1). The lateral cord divides into the lateral
of the hand. Adsons test (turn head to the side of root of the median nerve and the musculocuta-
symptoms, extend head, take a deep breath, and neous nerve. The musculocutaneous nerve
pull down on the limb) usually reduces or elim- leaves the brachial plexus sheath high in the
inates the radial pulse. Neuropathic signs usually axilla at the level of the lower border of the teres
involve the lower trunk of the brachial plexus major muscle and passes into the substance of
with pain in the ulnar border of the hand and the the coracobrachialis muscle. The posterior cord
medial forearm and arm and prominent paresis gives off the axillary nerve at the lower border
and wasting of thenar muscles. (Brazis, Masdeu, of the subscapularis muscle and continues
and Biller, 7580) along the inferior and posterior surface of the
axillary artery as the radial nerve. The axillary
157. (A) The brachial plexus is formed from the nerve supplies the shoulder joint, the surgical
anterior primary rami of the segments C4, C5, neck of the humerus, the deltoid, and the teres
C6, C7, C8, and T1. The trunks of the brachial minor muscles before ending as the superior
Answers: 156159 97

lateral brachial cutaneous nerve. The radial


REFERENCES
nerve continues along the posterior and inferior
surface of the axillary artery. The medial cord
Afifi AK, Bergman RA. Functional Neuroanatomy: Text and
contributes the medial root of the median nerve
Atlas. 2nd ed. New York: McGraw-Hill; 2005.
and continues as the ulnar nerve along the Brazis PW, Masdeu JC, Biller J. Localization in Clinical
medial and anterior surface of the axillary Neurology. 3rd ed. London: Little, Brown; 1996.
artery. The medial and lateral roots join to form Doody RS, Jankovic J. The alien hand and related signs. J
the median nerve which continues along the Neurol Neurosurg Psychiatry. 1992;55(9):806-810.
posterior and lateral surface of the axillary Feinberg TE, Schindler RJ, Flanagan NG, Haber LD. Two
artery. ErbDuchenne palsy results from alien hand syndromes. Neurology.1992;42(1):19-24.
damage to the fifth and sixth cervical roots, Goetz CG, Pappert EJ. Textbook of Clinical Neurology.
which may be caused by sudden forceful Philadelphia: Saunders; 1999.
depression of the shoulder during contact Hauser SL, Ropper AH. Diseases of the spinal cord In: Fauci
sports. Muscles supplied by these roots are AS, Braunwald E, Kasper DL, Hauser SL, Longo DL,
Jameson JL, Loscalzo J, eds. Harrisons Principles of Internal
weak and atrophic. These include the deltoid,
Medicine. 17th ed. Available at http://www.accessmedi-
biceps, brachioradialis, brachialis, supraspinatus,
cine.com/content.aspx?aID=2904373
infraspinatus, and subscapularis. The limb has a Kline LB, Bajandas, FJ, eds. Neuroophthalmology Review
characteristic position: it is internally rotated and Manual. 6th ed. Thorofare, NJ: Slack; 2001.
adducted because of deltoid, supraspinatus, and Kumar R, Behari S, Wahi J, Banerji D, Sharma K.
infraspinatus (shoulder abduction and arm exter- Peduncular hallucinosis: an unusual sequel to surgical
nal rotation) weakness. Therefore, the weakness intervention in the suprasellar region. Br J Neurosurg.
is of forearm flexion and of abduction and exter- 1999;13:500-503.
nal rotation of the arm. The biceps and brachio- Kumar R, Kaur A. Peduncular hallucinosis: an unusual
radialis reflexes may be depressed or absent. sequelae of medulloblastoma surgery. Neurol India.
(Brazis, Masdeu, and Biller, 7580) 2000;48:183-185.
Kline LB, Bajandas, FJ, eds. Neuroophthalmology Review
Manual. 6th ed. Thorofare, NJ: Slack; 2001.
158. (E) A lesion of the lumbar plexus can be caused
Patten J. Neurological Differential Diagnosis. 2nd ed. London
by a tumor of the pelvis. Its clinical signs
and New York: Springer; 1996.
include weakness of hip flexion (iliopsoas), leg Ropper AH, Brown RH. Infections of the nervous system
extension (quadriceps), thigh eversion (sarto- (bacterial, fungal, spirochetal, parasitic) and sarcoidosis.
rius), and thigh adduction. Sensory loss may In: Ropper AH, Brown RH, eds. Adams and Victors
involve the inguinal region; the lateral, anterior, Principles of Neurology. 8th ed. Available at http://
and medial thigh; and the medial aspect of the www.accessmedicine.com/content.aspx?aID=973096
lower leg. The patellar and cremasteric reflexes Ropper AH, Brown RH. Diseases of the nervous system
from the femoral nerve and the genitofemoral due to nutritional deficiency. In: Ropper AH, Brown RH,
nerve, respectively, may be decreased or absent. eds. Adams and Victors Principles of Neurology. 8th ed.
(Brazis, Masdeu, and Biller, 8184) Available at http://www.accessmedicine.com/con-
tent.aspx?aID=977600
Staal A. Mononeuropathies: Examination, Diagnosis and
159. (B) DejerineKlumpke palsy results from
Treatment. Philadelphia: Saunders; 1999.
injury to the eighth cervical and first thoracic
Thompson HS, Miller NR. Disorders of papillary function,
roots or the lower trunk of the plexus. Mass accommodation, and lacrimation. In: Miller NR, NJ
compression from a lung tumor may damage Newman NJ, eds. Walsh and Hoyts Clinical
the lower brachial plexus (Pancoast syndrome). Neuroophthalmology. 5th ed. Baltimore: Williams &
The patient may present with weakness of Wilkins; 1998:1:961-1040.
finger and wrist flexion as well as weakness of Trimble MR. Behavior and personality disturbance. In:
the intrinsic hand muscles, causing a claw hand Bradley WG et al, eds. Neurology in Clinical Practice.
deformity. (Brazis, Masdeu, and Biller, 8184) Boston: Butterworth-Heinemann; 2000:89-104.
This page intentionally left blank
CHAPTER 3

Pediatrics
Questions

1. A 6-year-old fully immunized girl developed a (C) Prolonged epileptiform discharges are
fever of 39C followed by weakness in her often associated with clinical symptoms
lower extremities and right upper extremity of seizure.
with numbness up to the midthoracic level. (D) Jitteriness is never seen in the newborn.
Cerebrospinal fluid (CSF) examination showed (E) Isolated apneic discharge is always a
mild protein and cell elevation with no bacte- seizure manifestation.
ria on Grams stain. The most likely diagnosis
is 4. Which of the following is the least common
(A) transverse myelitis among children with cerebral palsy?
(B) tick-borne paralysis (A) Spasticity
(C) poliomyelitis (B) Seizure
(D) botulism (C) Mental retardation
(E) vascular malformation (D) Hearing impairment
(E) Ataxia
2. Which of the following distinguish newborn
benign nocturnal myoclonus from seizure? 5. Which of the following is the least affected in
(A) It occurs solely during sleep. kernicterus, a syndrome associated with cere-
(B) It is activated by a stimulus. bral palsy?
(C) It may be accompanied by an abnormal (A) Hippocampus
electroencephalogram (EEG). (B) Substantia nigra
(D) Anticonvulsant drugs are the first-line (C) Basal ganglia
treatment. (D) Cortex
(E) It is usually seen in premature new- (E) Brainstem nuclei
borns.

3. Which of the following is true about neonatal


seizures?
(A) They may arise from the hemisphere or
brainstem.
(B) The lack of myelinated pathways facili-
tates the propagation of seizures from
one hemisphere to the contiguous
cortex.

99
100 3: Pediatrics

6. A 5-year-old girl was brought to the emergency 10. The first seizure usually occurs within the first
room because of recurrence of nocturnal right 24 hours after birth in case of
facial twitching and slurred speech over the pre-
(A) kernicterus
vious 3 weeks. Her parents did not report any
change in her mental status during these (B) urea cycle disturbances
episodes. Her older brother has absence (C) drug withdrawal
seizures. Which of the following is true about the (D) subarachnoid bleed
patients condition? (E) tuberous sclerosis
(A) The patient has absence seizures.
11. In pyridoxine dependency, the first seizures
(B) The patient has complex partial seizures.
usually occur within
(C) The patient has an autosomal recessive
disorder. (A) the fourth to eighth weeks after birth
(D) The patients electroencephalogram (B) the first 24 hours after birth
(EEG) may show central temporal spikes (C) the first 24 to 72 hours after birth
without overt seizure activity. (D) the first 72 hours to 1 week after birth
(E) The patient will need lifetime antiepilep- (E) the first to fourth weeks after birth
tic treatment.
12. Which of the following disorders may follow a
7. In a full-term infant, the most frequent cause of streptococcal infection?
neonatal seizure within the first 12 hours is
(A) Restless legs syndrome
(A) hypoxic encephalopathy (B) Infantile masturbation
(B) sepsis (C) Sydenham chorea
(C) subarachnoid hemorrhage (D) Hepatolenticular degeneration
(D) trauma (E) Essential tremor
(E) intraventricular bleed
13. An elevated blood lactate level is seen in
8. Which of the following is the best predictor of
developing cerebral palsy? (A) fructose 1,6 diphosphatase deficiency
(B) maple syrup urine disease
(A) Low birth weight
(C) citrullinemia
(B) Prematurity
(D) propionic acidemia
(C) The presence of echodense cystic lesions
(E) hypoparathyroidism
in the periventricular white matter with
a diameter greater than 3 mm
14. Which of the following is true about glycine
(D) An Apgar score less than 3 after 10 min- encephalopathy?
utes
(E) Hyperbilirubinemia (A) It is inherited as an autosomal dominant
disorder.
9. In neonatal adrenoleukodystrophy, seizures (B) Hiccuping is an early and continuous
usually appear during feature of affected newborns.
(C) The EEG is usually normal during the
(A) the first 12 hours after birth
acute phase.
(B) the first 24 hours after birth
(D) Hyperglycinemia is usually associated
(C) the first 24 to 72 hours after birth with hyperammonemia.
(D) the first 72 hours to 1 week after birth (E) Organic acidemia may be life-threaten-
(E) the first to fourth week after birth ing.
Questions: 624 101

15. A plasma ammonia concentration of 200 ml/l 20. Subcortical band heterotopias can be attributed
with a normal anion gap as well as a normal to loss of function of the
serum glucose concentration in a 6-day-old
(A) doublecortin gene
lethargic newborn is suggestive of
(B) ASPM gene (abnormal spindle protein-
(A) maple syrup urine disease like, microcephaly-associated gene)
(B) glycogen storage disease type 1 (C) filamin A gene
(C) urea cycle disorder (D) Fukutin gene
(D) fructose 1,6 diphosphatase deficiency (E) ADP-ribosylation factor guanine
(E) methylmalonic acidemia exchange factor gene

16. Seizure correlates with newborn serum ammo- 21. The association of hydrocephaly, agyria, and
nia concentrations greater than retinal dysplasia is suggestive of
(A) 120 mol/L (A) subcortical band heterotopia
(B) 180 mol/L (B) DandyWalker malformation
(C) 300 mol/L (C) type I lissencephaly
(D) 400 mol/L (D) cobblestone lissencephaly
(E) 500 mol/L (E) Joubert syndrome

17. Neonatal hypoglycemia in premature new- 22. By the age of 15 years, the prevalence of recur-
borns is defined as a whole-blood glucose con- rent headaches reaches
centration less than
(A) 2.5%
(A) 80 mg/dL (B) 5.1%
(B) 60 mg/dL (C) 15%
(C) 40 mg/dL (D) 23%
(D) 20 mg/dL (E) 75%
(E) 10 mg/dL
23. The most common form of migraine in ele-
18. Impaired development of the vermis associ- mentary school boys is
ated with increased CSF spaces causes
(A) migraine without aura
(A) microcephaly (B) migraine with aura
(B) DandyWalker malformation (C) basilar-type migraine
(C) type I lissencephaly (D) familial hemiplegic migraine
(D) periventricular heterotopia (E) abdominal migraine
(E) Joubert syndrome
24. A 7-year-old boy was brought to the emergency
19. Mutations in the genes coding for an actin- room because of recurrence of migraine
binding phosphoprotein that can promote actin headaches. Which of the following clinical his-
branching causes tory findings is suggestive of an alternative
diagnosis?
(A) subcortical band heterotopia
(B) DandyWalker malformation (A) Irritability
(C) type I lissencephaly (B) Pounding headache
(D) periventricular heterotopia (C) Photophonia
(E) Joubert syndrome (D) Photophobia
(E) Morning headache
102 3: Pediatrics

25. Missense mutation in the calcium channel gene 30. Sleepwalking


(CACNA1A) causes
(A) is a nonrapid-eye-movement (NREM)
(A) retinal migraine sleep parasomnia
(B) benign paroxysmal vertigo of childhood (B) typically starts at the age of 2 years
(C) basilar-type migraine (C) is caused by a mutation in a gene regu-
(D) familial hemiplegic migraine lating the expression of nicotinic acetyl-
(E) abdominal migraine choline receptors
(D) is a nocturnal frontal lobe seizure
26. The most frequent of migraine variants is (E) responds to treatment with phenytoin,
which is used as a first-line treatment
(A) retinal migraine
(B) benign paroxysmal vertigo of childhood 31. Which of the following brain structures is
(C) basilar-type migraine smaller in children with attention deficithyper-
(D) familial hemiplegic migraine activity disorder than in age-matched controls?
(E) abdominal migraine (A) Caudate nucleus
(B) Thalamus
27. The most frequent manifestation of vertebro-
basilar migraine is (C) Gray matter
(D) Midbrain
(A) vertigo
(E) Pons
(B) weakness
(C) nausea and vomiting 32. In children with attention deficithyperactivity
(D) ataxia disorder, functional magnetic resonance imag-
(E) diplopia ing (fMRI) may demonstrate hypoactivity of
the
28. The first-line pharmacological treatment for a (A) temporal cortex
12-year-old boy who developed an acute migraine
(B) occipital cortex
headache is
(C) cerebellum
(A) ibuprofen (D) thalamus
(B) triptans (E) midbrain
(C) dihydroergotamine
(D) opiates 33. Apneic spells in an otherwise normal-appear-
(E) intravenous fluids ing newborn are suggestive of
(A) brainstem immaturity
29. In response to a common insult such as
(B) seizure
hypoxia, the preterm brain exhibits more sus-
ceptibility than the term brain of the (C) subarachnoid hemorrhage
(D) bacterial meningitis
(A) gray matter
(E) hypoxic ischemic encephalopathy
(B) white matter
(C) brain stem
(D) cerebellum
(E) basal ganglia
Questions: 2542 103

34. The usual EEG pattern during the early stage of 39. Defects in voltage-gated calcium channels have
infantile spasm been associated with
(A) is normal (A) nocturnal frontal lobe epilepsy
(B) indicates hypsarrhythmia (B) benign epilepsy with centrotemporal
(C) shows a burst-suppression pattern spikes
(D) has polyspikes and waves (C) juvenile absence epilepsy
(E) shows triphasic waves (D) benign neonatal familial convulsions
(E) childhood absence epilepsy
35. The most frequent central nervous system
tumor observed in neurofibromatosis type 1 is 40. Juvenile myoclonic epilepsy is associated with
a defect in the
(A) ependymoma
(B) oligodendroglioma (A) GABAA-gated chloride channel
(C) teratoma (B) acetylcholine receptor
(D) astrocytoma (C) voltage-gated potassium channel
(E) pineoblastoma (D) voltage-gated sodium channel
(E) voltage-gated calcium channel
36. Narcolepsy is associated with loss of
41. Which of the following gene defects is a single-
(A) hypocretin
gene disorder maternally derived and associ-
(B) dopamine ated with autism and gross motor delay?
(C) serotonin
(A) Fragile-X syndrome
(D) glutamate
(B) The 15q duplication
(E) acetylcholine
(C) Tuberous sclerosis complex
37. Which of the following drugs is the first-line (D) Rett syndrome
pharmacological treatment for narcolepsy with (E) Joubert syndrome
excessive daytime sleepiness in a 12-year-old
boy? 42. Malignant migrating partial seizures in infancy
are characterized by
(A) Ritanserin
(B) Clomipramine (A) onset after the age of 3 years
(C) Sodium oxybate (B) occurrence of progressive microcephaly
(D) Modafinil (C) frequent myoclonus
(E) Zaleplon (D) mutation in sodium ion channels
(E) hypsarrhythmia
38. Which of the following drugs is the most effec-
tive in treating a 17-year-old boy who has nar-
colepsy with excessive daytime sleepiness and
cataplexy?
(A) Modafinil
(B) Amphetamine
(C) Sodium oxybate
(D) Armodafinil
(E) Methylphenidate
104 3: Pediatrics

43. A 3-year-old boy was referred because daily (C) cerebral venous thrombosis
spells consisting of atypical absences and clonic (D) intracerebral hemorrhage
seizures, lateralized either on the left or on the (E) extradural hemorrhage
right. The patient demonstrated normal psy-
chomotor development until the age of 10 46. Tear of the superficial veins by shearing forces
months, when he developed a first unilateral during prolonged delivery may be complicated
febrile seizure on the left side for about 10 min- by neonatal seizures and
utes. EEG was normal, and the patient was
treated with phenobarbital. Other convulsive (A) subdural hemorrhage
seizures, febrile and afebrile, occurred in the (B) subarachnoid hemorrhage
following months, either generalized or later- (C) cerebral venous thrombosis
alized on the left. At 13 months, myoclonic (D) intracerebral hemorrhage
attacks and brief atypical absences with pro-
(E) extradural hemorrhage
gressive and jerky head fall or complete fall
appeared several times a day. Psychomotor
47. Sepsis may be complicated by neonatal
development and hyperkinetic behavior then
seizures and
began to slow. Computed tomography (CT)
and MRI scans were then performed; biological (A) subdural hemorrhage
investigations were normal. The EEGs showed (B) subarachnoid hemorrhage
numerous generalized spike and polyspike (C) cerebral venous thrombosis
waves. Different antiepileptic drugs were pre- (D) intracerebral hemorrhage
scribed, but without success. At his neurologi-
(E) extradural hemorrhage
cal examination at the age of 3 years, the patient
presented with a slight ataxia. These findings
48. A newborn developed generalized seizures in
are suggestive of
the second week of life. He was doing well at
(A) myoclonic astatic epilepsy birth and then progressively became mildly
(B) West syndrome lethargic, with feeding difficulties and pro-
(C) cryptogenic late-onset epileptic spasm gressive hypotonia. Serologic studies showed
an increased concentration of branched-chain
(D) Dravet syndrome
amino acids. The addition of 2,4 dinitro-
(E) LennoxGastaut syndrome
phenylhydrazine reagent colored his urine
yellow. The most likely diagnosis is
44. Epileptic encephalopathy with suppression
bursts is characterized by (A) carbamyl phosphate synthetase defi-
ciency
(A) paroxysmal choreoathetosis
(B) glycine encephalopathy
(B) paroxysmal polyspikes lasting several
(C) bilirubin encephalopathy
seconds alternating with episodes of
low-amplitude tracing (D) maple syrup urine disease
(C) a spike followed by a bell-shaped slow (E) isovaleric acidemia
wave
49. Which of the following is true about DiGeorge
(D) a favorable prognosis
syndrome?
(E) mutation in voltage-gated sodium channels
(A) It results from a duplication of chromo-
45. Tear of the tentorium near its junction with the some 22q11.
falx may be complicated by neonatal seizures (B) There is hypoplasia of the second pha-
and ryngeal pouch.
(A) subdural hemorrhage (C) Hypercalcemia and stroke are among
the main features of the syndrome.
(B) subarachnoid hemorrhage
Questions: 4354 105

(D) Cardiac defect may be a cause of death. (A) His symptoms are caused by increased
(E) Apnea, jitteriness, and high-pitched cry latency from sleep onset to REM sleep of
may complicate the course of the disease. greater than 90 minutes.
(B) Sleep paralysis rarely complicates this
50. A 12-month-old boy was brought to the emer- condition.
gency room because of multiple episodes of (C) Vivid and usually pleasant visual and
stiffening, upward eye deviation, pupillary auditory perceptions occur at the transi-
dilatation, and alteration of respiratory pattern. tion between sleep and wakefulness.
Most of these episodes occurred during sleep (D) During the loss of tone, partial paralysis
and were complicated by enuresis. The mother, affecting just the face and hands is more
1 month earlier, noted a brief episode of trem- common than total paralysis.
bling of the eyelids and mouth with loss of
(E) Clonidine is the treatment of choice.
facial tone. An EEG showed a generalized burst
of 2.5 spike-wave complexes per second. Which
53. Lafora disease is characterized by
of the following is true about the patients con-
dition? (A) mental retardation complicating
myoclonic seizures
(A) Mental retardation is an unusual compli-
cation. (B) ataxia and spasticity occurring early in
the course of the disease
(B) Twenty percent of patients with this con-
dition may have a history of febrile (C) age of onset between 3 and 7 years
seizure. (D) easily controlled seizures
(C) Seizures in this condition are difficult to (E) inclusion bodies present in all stages of
control. the disease
(D) An underlying cause is rarely found.
54. A 4-year-old boy with normal language and
(E) Phenytoin may improve seizure control.
cognitive abilities developed progressive loss of
his cognitive skills. He had difficulty following
51. Which of the following statements is true about
commands and in language comprehension.
infantile spasms?
This was followed by a reduction in the volume
(A) Onset always occurs after the age of and content of speech. Three months later the
1 year. patient presented with multiple episodes of
(B) Pertussis immunization is a cause of staring, followed by confusion. Physical exam-
infantile spasms. ination and brain MRI were normal. The most
(C) It may be associated with agenesis of the likely diagnosis is
corpus callosum. (A) progressive myoclonic epilepsy
(D) Hypsarrhythmia is the usual EEG pat- (B) temporal lobe tumor
tern recorded during the late stages of (C) benign rolandic epilepsy
infantile spasm.
(D) LandauKleffner syndrome
(E) High-dose pyridoxine is the first line
(E) Rasmussen syndrome
treatment of seizure in infantile spasms.

52. A 17-year-old male developed daily multiple


sleep attacks over the previous 2 months. He
recently lost his job as a waiter because of mul-
tiple falls caused by a sudden loss of tone.
Which of the following is true about his condi-
tion?
106 3: Pediatrics

55. A 7-year-old boy living in Philadelphia devel- (E) Adolescent-onset myoclonic epilepsy in
oped the following symptoms over a period of remission for 3 years
4 days in January: progressive weakness, clum-
siness, and loss of facial expression. Neuro- 58. A newborn developed an asymmetric Moro
logical evaluation showed that the patient was reflex after a complicated delivery. The poorly
ataxic when walking or reaching, with responsive arm hung in adduction, rotated
decreased deep tendon reflexes and bilateral internally at the shoulder, extended and
facial weakness. Which of the following is true? pronated at the elbow, and flexed at the wrist.
The most likely affected roots are
(A) The condition is caused by a block of the
neuromuscular junction due to a neuro- (A) C7-C8
toxin produced by a tick. (B) T1-T2
(B) Respiratory failure and autonomic dys- (C) C3-C4
function may complicate the course of (D) T3-T4
the disease.
(E) C5-C6
(C) Positive response to edrophonium will
be a definitive diagnostic test in this 59. A 10-year-old boy was brought for a neurology
patient. consultation by his mother because he had been
(D) Impairment in swallowing, in pupillary disturbing his classmates with sniffing and
reflex response, and in extraocular grunting sounds for the previous 2 months.
muscle motility are classic findings in Neurological examination was normal. The
this patients disease. most likely diagnosis is
(E) An electromyogram (EMG) may show a
(A) attention deficithyperactivity disorder
decremental response after repetitive
(ADHD)
stimulation in this patient.
(B) anxiety
56. The most common cause of chorea in a school- (C) tics
aged child is (D) autism
(A) Sydenham chorea (E) Sydenham chorea
(B) Tourette syndrome
60. Which of the following is the least reliable sign
(C) Huntington disease for a ventriculoperitoneal shunt obstruction in
(D) Wilson disease a 4-month-old boy?
(E) lupus erythematosus
(A) Grade I bruit
57. In which of the following cases would discon- (B) Strabismus
tinuation of prophylactic antiseizure medica- (C) Splitting of the sutures
tions be strongly considered? (D) A setting-sun sign
(A) Seizure caused by a surgically treated (E) A bulging fontanelle
arteriovenous malformation (AVM) in
remission for 6 months 61. Which of the following is true about Tourette
syndrome?
(B) Absence seizures in remission for 8
months with normal EEG and MRI stud- (A) Onset after the age of 15 years
ies (B) Preservation of school performance
(C) Generalized tonicclonic seizure in (C) Motor tics as the most common initial
remission for 2 years with normal EEG symptoms
and MRI studies (D) Inability to temporarily suppress symp-
(D) Cerebral palsy with seizures in remis- toms
sion for 3 years (E) Greater incidence in girls than in boys
Questions: 5567 107

62. You are consulted about a 14-year-old boy with 65. A 10-year-old girl was reported by her parents
shoulder weakness and changes in facial expres- to have been spacing out at home and in
sion. Physical examination showed a smooth, school over the past month. Sometimes these
unlined face, protuberant lips, horizontal smile, spells were associated with lip smacking.
and muscle wasting in both shoulders with Which of the following is the most suggestive
sparing of the forearms. Lab workup showed a of absence seizure rather than complex partial
creatine kinase level of 600 UI/L. An EMG seizure?
showed a myopathic pattern. Which of the fol-
(A) Urinary incontinence during spells
lowing is true about the patients condition?
(B) Centrotemporal spikes on EEG
(A) This is an autosomal recessive myopa- (C) Spells lasting 1 to 2 minutes
thy.
(D) Presence of automatism
(B) The abnormal gene is located in Xp 21.2.
(E) Prompt recovery after spells
(C) The abnormal gene is responsible for the
reduced amount of dystrophin in the 66. The most frequent cause of athetoid cerebral
muscles. palsy is
(D) Retinal vascular abnormalities may com-
plicate the course of the disease. (A) prematurity
(E) Cardiac arrhythmia is a frequent symp- (B) perinatal asphyxia
tom. (C) intraventricular bleed
(D) bilirubin encephalopathy
63. A 16-month-old boy with a history of Downs (E) low birth weight
syndrome developed his first generalized
tonicclonic seizure, which lasted for 20 min- 67. A preterm newborn male was evaluated in the
utes. On physical examination, his tempera- nursery. He was delivered 8 weeks before the
ture was 39.5C and there was no evidence of expected date, with an Apgar score of 3 at 5
neurological abnormality. His older sister had minutes. Neurological examination demon-
a history of febrile seizure. His risk of devel- strated generalized hypotonia and increased
oping epilepsy in later life is closest to deep tendon reflexes. Which of the following is
(A) 4% the correct answer to the mothers concern
about her baby developing cerebral palsy?
(B) 10%
(C) 30% (A) His low Apgar score indicates a high
(D) 50% risk of cerebral palsy.
(E) 90% (B) It is very difficult to make the diagnosis
of cerebral palsy before the age of 6
64. A 10-year-old girl developed restless behavior, months.
deterioration in school performance, uncoor- (C) The presence of hypotonia on the initial
dinated movements, and angry outbursts. neurological examination of the new-
Physical examination showed slow writhing born is highly suggestive of cerebral
of limbs interrupted by high-amplitude, violent palsy.
flinging of her upper extremities with an (D) The presence of progressive neurological
inward compulsion to move. The most likely deficit is suggestive of cerebral palsy.
cause of her condition is (E) Seizure is the most frequent complica-
(A) Wilson disease tion of cerebral palsy.
(B) Sydenham chorea
(C) Tourette syndrome
(D) ADHD
(E) vascular accident
108 3: Pediatrics

68. A newborn with the diagnosis of myelomeningo- 72. A 15-month-old boy was diagnosed with bac-
cele should be evaluated for terial meningitis and started on antibiotics. On
the third day of his hospitalization, he devel-
(A) genitourinary dysfunction
oped an episode of left-sided twitching of his
(B) gastrointestinal dysfunction arm, face, and leg followed by left-sided weak-
(C) pulmonary complications ness that did not improve over the next 3 days
(D) ArnoldChiari malformation type I despite the improvement of his general status
(E) seizure and appetite and the disappearance of fever.
The most likely diagnosis is
69. You are asked to see a 10-year-old boy because (A) right hemispheric ischemic stroke
of facial weakness and increased hand clumsi-
(B) subdural empyema
ness progressing over a period of 6 months.
On physical examination, the patient has an (C) focal seizure with Todd paralysis
inverted V-shaped upper lip, thin cheeks, and (D) brain abscess
a concave temporalis muscle. He is unable to (E) increased intracranial pressure
close his eyelids tightly. Hand examination
shows mild intrinsic muscle wasting and use of 73. A premature neonate was diagnosed with
the wrist flexor to release grasp. The most likely gram-negative bacterial meningitis and started
diagnosis is on antibiotics. The follow-up examination
showed an improvement of his clinical status
(A) Duchenne muscular dystrophy
and CSF examination. On the fourth day, the
(B) WerdnigHoffman disease patient presented with episodes of bradycardia
(C) myotonic dystrophy and apnea. Head CT showed fluid of different
(D) myasthenia gravis contrast densities in the ventricles with an
(E) chronic demyelinating polyradiculopa- enhancing ependymal rim. A repeat lumbar
thy CSF examination was unchanged. The most
likely diagnosis is
70. Which of the following drugs may cause (A) brain abscess
pseudotumor cerebri in a 5-year-old boy?
(B) subdural empyema
(A) Ampicillin (C) ventriculitis
(B) Phenobarbital (D) idiopathic seizure
(C) Vitamin A (E) encephalitis
(D) Acetazolamide
(E) Furosemide 74. A 17-year-old girl experienced 15 pressure-type
headaches per month for the previous 8
71. Which of the following is the latest sign or months. Each headache lasted from 2 hours to
symptom of an acute rise in intracranial pres- 2 days. There has been no disturbance of her
sure in a 12-year-old boy? school performance or attendance. Her neuro-
logical examination is normal. The best treat-
(A) Decreased consciousness ment for her headache is
(B) Headache
(A) low-dose amitriptyline at bedtime
(C) Vomiting
(B) acetaminophen and codeine
(D) Irritability
(C) valproic acid
(E) Symptomatic papilledema
(D) propranolol
(E) clorazepate
Questions: 6881 109

75. Familial hemiplegic migraine ness. An EEG demonstrated periodic lateraliz-


ing epileptiform discharge. The most likely
(A) is characterized by a sudden onset of
diagnosis is
hemiparesis or hemisensory loss that is
usually followed by a contralateral (A) measles encephalitis
headache (B) Reye syndrome
(B) occurs more frequently in adults than in (C) herpes simplex encephalitis
children (D) postinfectious encephalitis
(C) improves rapidly, since the neurological (E) St. Louis encephalitis
impairment lasts no more than a few
hours 80. Which of the following may exacerbate brain
(D) always affects the same side ischemia during the management of increased
(E) is transmitted by an autosomal recessive intracranial pressure?
inheritance
(A) Head elevation
76. Which of the following is a migraine variant? (B) Hypothermia
(C) Osmotic diuretics
(A) Recurrent abdominal pain
(D) Hyperventilation to lower the arterial
(B) Recurrent chest pain carbon dioxide pressure from 40 to
(C) Recurrent urinary incontinence. 20 mmHg
(D) Sleepwalking (E) Pentobarbital coma
(E) Choreoathetotic movements
81. A 15-year-old boy is brought for a neurological
77. A 2-year-old boy developed a 5-minute febrile consultation because of exacerbation of headache
seizure. Which of the following conditions does and difficulties of accommodation. Neurological
not increase his risk of developing subsequent examination demonstrates loss of pupillary light
epilepsy? reflex, palsy of upward gaze with preservation of
downward gaze, retraction convergence nystag-
(A) Preexisting cerebral palsy
mus when upward gaze is attempted, and loss of
(B) Family history of epilepsy accommodation. Head MRI shows an enlarged,
(C) History of complex febrile seizure dense, noncalcified pineal gland area with irreg-
(D) Two febrile seizures in 1 year ular margins. Which of the following is true
(E) Preexisting development delay about this condition?
(A) Pineal germinoma is the most likely
78. Which of the following is a major form of diagnosis.
encephalitis in Asia?
(B) It is more common in females than
(A) California La Crosse encephalitis males.
(B) Eastern equine encephalitis (C) It is highly radioresistant.
(C) Japanese B encephalitis (D) The 5-year survival rate is less than 20%.
(D) St. Louis encephalitis (E) Bitemporal hemianopia is one of the ear-
(E) Western equine encephalitis liest signs of this condition.

79. A 10-year-old boy developed an episode of


fever, headache, nausea and vomiting, and irri-
tability. Three days later, he was brought to the
emergency room because of a brief episode of
twitching of his right face and arm with
decreased consciousness. Neurological exami-
nation showed mild right arm and leg weak-
110 3: Pediatrics

82. A 5-year-old boy with autistic behavior, long (D) congenital syphilis
face, enlarged ears, and macroorchidism may (E) toxoplasmosis
have which of the following gene defects?
(A) 5 p monosomy 87. Hexosaminidase deficiency causes
(B) Trisomy 21 (A) GM1 gangliosidosis
(C) Fragile-X syndrome (B) TaySachs disease
(D) Trisomy 18 (C) Krabbe disease
(E) Trisomy 13 (D) metachromatic leukodystrophy
(E) NiemannPick disease
83. A 3-year-old boy with hypotonia and a round
flat face as well as flat nape of neck may have 88. Beta-galactosidase deficit causes
which of the following gene defects?
(A) GM1 gangliosidosis
(A) 5 p monosomy (B) TaySachs disease
(B) Trisomy 21 (C) Krabbe disease
(C) Fragile-X syndrome (D) metachromatic leukodystrophy
(D) Trisomy 18 (E) NiemannPick disease
(E) Trisomy 13
89. Sphingomyelinase deficit causes
84. A 4-week-old newborn girl with pointed ears
micrognathia, occipital protuberance, narrow (A) GM1 gangliosidosis
pelvis, and rocker bottom feet may have which (B) TaySachs disease
of the following gene defects? (C) Krabbe disease
(A) 5 p monosomy (D) metachromatic leukodystrophy
(B) Trisomy 21 (E) NiemannPick disease
(C) Fragile-X syndrome
90. Arylsulfatase deficit causes
(D) Trisomy 18
(E) Trisomy 13 (A) GM1 gangliosidosis
(B) TaySachs disease
85. A 3-year-old boy with cri du chat syndrome (C) Krabbe disease
may have which of the following gene defects? (D) metachromatic leukodystrophy
(A) 5 p monosomy (E) NiemannPick disease
(B) Trisomy 21
91. Galactosylceramidase deficit causes
(C) Fragile-X syndrome
(D) Trisomy 18 (A) GM1 gangliosidosis
(E) Trisomy 13 (B) TaySachs disease
(C) Krabbe disease
86. A 3-year-old boy was evaluated for mental (D) metachromatic leukodystrophy
retardation. He was found to have bilateral (E) NiemannPick disease
hearing loss, interstitial keratitis, and peg-
shaped upper incisors. The most likely cause of 92. Which of the following is true of Leigh dis-
his mental retardation is ease?
(A) trisomy 21 (A) It is a disorder caused by multiple sulfa-
(B) cytomegalovirus infection tase deficiency.
(C) amino acidabnormal metabolism (B) The Hurler phenotype is prominent in
affected patients.
Questions: 82101 111

(C) Hypotonia, ocular motility, and respira- (D) mitochondrial abnormalities


tory abnormalities are typical features. (E) lack of regulation of fusion of primary
(D) Glucose load may improve symptoms. lymphocytes
(E) The brainstem and basal ganglia are typ-
ically spared. 98. A 4-year-old mentally retarded girl was brought
by her mother to the neurology clinic because
93. Subacute necrotizing encephalomyelopathy she developed a new onset of seizure. Skin
results from examination demonstrated leaf-shaped hypo-
chromic nevi on her left buttock and an angiok-
(A) defect in intestinal transport of copper eratoma on her face. The most likely diagnosis
(B) sphingomyelinase deficit is
(C) glucocerebrosidase deficit
(A) cerebral palsy
(D) mitochondrial abnormalities
(B) tuberous sclerosis
(E) lack of regulation of fusion of primary
(C) neurofibromatosis type I
lymphocytes
(D) Downs syndrome
94. Gaucher disease results from (E) Gaucher disease

(A) defect in intestinal transport of copper 99. Which of the following is a criterion for the
(B) sphingomyelinase deficit diagnosis of neurofibromatosis type II?
(C) glucocerebrosidase deficit
(A) Six caf au lait spots greater than 15 mm
(D) mitochondrial abnormalities in the postpubertal individual
(E) lack of regulation of fusion of primary (B) Optic glioma
lymphocytes
(C) Iris hamartoma
95. Menkes syndrome is caused by (D) Acoustic neurinomas
(E) Sphenoid dyplasia
(A) defect in intestinal transport of copper
(B) sphingomyelinase deficit 100. Which of the following is a necessary criterion
(C) glucocerebrosidase deficit for the diagnosis of Rett syndrome?
(D) mitochondrial abnormalities (A) Retinopathy
(E) lack of regulation of fusion of primary (B) Severe progressive dementia
lymphocytes
(C) Microcephaly at birth
96. ChediakHigashi syndrome is caused by (D) Optic atrophy
(E) Evidence of acquired neurological dis-
(A) defect in intestinal transport of copper ease
(B) sphingomyelinase deficit
(C) glucocerebrosidase deficit 101. The most likely diagnosis of a newborn with
(D) mitochondrial abnormalities vomiting, hepatomegaly, and the presence of
(E) lack of regulation of fusion of primary reducing substances in the urine is
lymphocytes (A) Mucopolysaccharide enzyme deficit
(B) Krabbe disease
97. NiemannPick syndrome is caused by
(C) Gaucher disease
(A) defect in intestinal transport of copper (D) TaySachs disease
(B) sphingomyelinase deficit (E) Galactosemia
(C) glucocerebrosidase deficit
112 3: Pediatrics

102. Zellweger syndrome is caused by 106. Which of the following is true about Duchenne
muscle dystrophy?
(A) a mitochondrial defect
(B) hypoxic ischemic encephalopathy (A) The gene defect is located at chromo-
(C) acid maltase deficiency some X p21.
(D) a neuromuscular transmission defect (B) The dystrophin content in Duchenne
muscular dystrophy is less reduced than
(E) a peroxisomal disorder
in Becker dystrophy.
103. Which of the following is true about spinal (C) The disease has an autosomal dominant
muscular atrophy type I? transmission.
(D) Motor function declines sharply before
(A) The age of onset is between 12 and the age of 3 years.
24 months after birth.
(E) Arthrogryposis is usually present.
(B) Arthrogryposis is usually present.
(C) Facial expression and extraocular move- 107. The gene defect of myotonic dystrophy is
ments are affected early and severely. located in
(D) There is a hypertrophy of type II fibers.
(A) 19q 13.3
(E) DNA analysis of chorionic villi may be
(B) 1q 31-32
used for prenatal diagnosis.
(C) Xp21.2
104. A male newborn with normal development (D) 17q23-25
started to have poor feeding, constipation, and (E) 17p11
failure to thrive after the age of 4 weeks.
Neurological examination demonstrated gen- 108. The gene defect of Duchenne muscular dys-
eralized hypotonia with areflexia, weak cry, trophy is located in
ptosis, and dilated pupils that reacted poorly to
(A) 17q25.3-q25.3
light. An EMG showed an incremental
response to repetitive stimulation between 20 (B) 1q 31-32
and 50 Hz. The most likely diagnosis is (C) Xp21.2
(D) 17q23-25
(A) GuillainBarr syndrome
(E) 17p11
(B) myasthenia gravis
(C) infantile spinal muscle atrophy 109. The gene defect of acid maltase deficiency is
(D) botulism located in
(E) Lowe syndrome
(A) 17q25.2-q25.3
105. Congenital myotonic dystrophy in newborns is (B) 1q 31-32
characterized by (C) Xp21.2
(D) 17q23-25
(A) an autosomal recessive disorder
(E) 17p11
(B) less than 50 DNA triplet repeats on chro-
mosome 19
110. The gene defect of familial hyperkalemic peri-
(C) myotonia usually elicited by muscle per- odic paralysis is located in
cussion
(D) a high incidence of polyhydramnios (A) 17q25.3-q25.2
during pregnancy (B) 1q31-32
(E) a change in repeat DNA triplet size that (C) Xp21.2
is greater from father to child than from (D) 17q23-25
mother to child (E) 17p11
Questions: 102118 113

111. The gene defect of familial hypokalemic peri- 115. A 12-year-old boy is brought for a neurological
odic paralysis is located in consultation because of muscle stiffness and
difficulty in moving. Neurological examina-
(A) 17q23-35
tion demonstrates a painless myotonia at rest
(B) 1q 31-32 that improves with activity as well as general-
(C) Xp21.2 ized muscle hypertrophy. EMG shows myotonic
(D) 17q23-25 discharges. The creatine kinase level is normal.
(E) 17p11 The most likely diagnosis is
(A) myotonia congenita
112. The gene defect of DejerineSottas syndrome is
(B) stiff-man syndrome
located in
(C) hypothyroidism
(A) 17q23 (D) SchwartzJampel syndrome
(B) 17p11 (E) neuromyotonia
(C) 19q 13.3
(D) Xq28 116. A 7-year-old boy is evaluated for pain, muscle
(E) 1p35 weakness with exercise, and recurrent rhab-
domyolisis. Neurological examination and
113. The gene defect of EmeryDreifuss muscular EMG are normal. The creatine kinase level is
dystrophy type 1 is located in 600 UI/L. An ischemic exercise test shows
abnormal ammonia and lactate levels. The most
(A) 17q23 likely diagnosis is
(B) 17p11
(A) KearnsSayre syndrome
(C) 19q 13.3
(B)Menkes syndrome
(D) Xq28
(C) myoadenylate deaminase deficiency
(E) 1p35
(D) carnitine palmitoyl transferase defi-
114. A 12-year-old boy is brought for a neurological ciency
consultation because of recurrent numbness (E) Brody disease
and tingling in his lower extremities with
unsteady gait over several months. His parents 117. Child ataxia with a pellagra-like skin rash after
report that, as of 3 months earlier, the patient exposure to sunlight may be seen in
had decreased nocturnal visual acuity. (A) Hartnup disease
Neurological examination showed cerebellar
(B) basilar migraine
ataxia, nystagmus, and decreased deep tendon
reflexes. CSF examination shows a protein level (C) maple syrup urine disease
of 300 mg/dL. Nerve conduction studies show (D) Miller Fisher syndrome
a marked decrease in conduction velocity (E) myoclonic encephalopathy
throughout. EMG results are consistent with
chronic denervation. The most likely diagnosis 118. Which of the following causes of progressive
is ataxia in a child is associated with an increased
blood level of very long chain fatty acids?
(A) Refsum disease
(B) DejerineSottas disease (A) Ataxia telangiectasia
(C) CharcotMarieTooth neuropathy (B) Sulfatide lipidosis
(D) metachromatic leukodystrophy (C) Hypobetalipoproteinemia
(E) EmeryDreifuss muscular dystrophy (D) Ramsay Hunt syndrome
(E) Adrenoleukodystrophy
114 3: Pediatrics

119. A 15-year-old boy developed night blindness, 122. The association between encephalopathy and
mental retardation, and dystonic dysarthria. progressive calcification of the basal ganglia is
Head MRI shows an eye-of-tiger appearance of suggestive of
the pallidum. Acanthocytes are seen in washed
(A) tardive dyskinesia
erythrocytes. The most likely diagnosis is
(B) Fahr disease
(A) Fahr disease (C) HallervordenSpatz disease
(B) HallervordenSpatz disease (D) neuroacanthocytosis
(C) neuroacanthocytosis (E) Sydenham chorea
(D) Sydenham chorea
(E) Harp syndrome 123. A 13-year-old boy with a history of asthma
treated by theophylline developed abnormal
120. A 10-year-old girl developed progressive dys- movements of the tongue and face, including
tonic rigidity and choreoathetotic movements. tongue protrusion and lip smacking. The most
On T2-weighted MRI of the head, low-intensity likely diagnosis is
images from the globus pallidus show a central
(A) tardive dyskinesia
area of increased intensity. The most likely
diagnosis is (B) Fahr disease
(C) HallervordenSpatz disease
(A) Fahr disease
(D) neuroacanthocytosis
(B) HallervordenSpatz disease
(E) Sydenham chorea
(C) neuroacanthocytosis
(D) Sydenham chorea 124. A 7-year-old boy developed dystonia of the
(E) Harp syndrome face and limbs associated with self-mutilation
of the lips. Lab workup shows abnormal ery-
121. The most common cause of acquired chorea in throcytes with thorny projection from the cell
children is surface but no lipoprotein abnormalities. The
most likely diagnosis is
(A) tardive dyskinesia
(B) Fahr disease (A) tardive dyskinesia
(C) HallervordenSpatz disease (B) Fahr disease
(D) neuroacanthocytosis (C) HallervordenSpatz disease
(E) Sydenham chorea (D) neuroacanthocytosis
(E) Sydenham chorea
Answers and Explanations

1. (A) The acute onset of symptoms in this child spinal cord. Vital capacity should be measured
is most likely related to acute transverse multiple times per day. Pulmonary emboli are
myelitis. This diagnosis is supported by the a rare complication during acute paralysis.
occurrence of a rapid asymmetric neurological Hypotension and sphincter incontinence, like
deficit and a sensory level suggesting cross-sec- other autonomic instabilities, are usually not
tional involvement of the spinal cord. Magnetic life-threatening. However, cardiac rhythm and
resonance imaging (MRI) of the spinal cord blood pressure should be monitored closely.
would be the procedure of choice to confirm (Fenichel, 264265)
the diagnosis. It may show evidence of cord
swelling at the level of demyelination and rule 2. (A) Nocturnal myoclonus can be distinguished
out any acute cord compression in the epidural from seizures and jitteriness because it occurs
space. Transverse myelitis is an acute demyeli- solely during sleep, it is not activated by a stim-
nation of the spinal cord that may progress over ulus, and the EEG is normal. (Fenichel, 3)
hours or days. It may be associated with optic
neuritis in Devic disease and uncommonly with 3. (A) Newborns with hydranencephaly or ate-
multiple sclerosis in childhood. The mean age of lencephaly are able to generate seizures, sup-
onset is 9 years. The level of demyelination is porting the fact that the brainstem may generate
usually thoracic. The motor deficit is commonly seizure activity. These seizures are confined to
asymmetric, and maximum weakness is the brainstem because of the absence of myeli-
reached within 48 hours. Recovery begins after nated fibers. Propagation of seizure activity to
6 days. Fifty percent of patients make a full a contiguous cortical area is enhanced by the
recovery, 10% have no recovery, and 40% maturity of myelinated pathways. Fifty percent
recover incompletely. Relapsing myelitis may of prolonged epileptiform discharges are not
occur. Corticosteroids remain the most common associated with visible clinical seizures.
treatment despite the absence of controlled Jitteriness is an excessive response to stimula-
studies. Poliomyelitis may cause asymmetric tion by low-frequency, high-amplitude shaking
weakness, but the presence of a sensory level of limbs and jaw. It may occur in a newborn
excludes this diagnosis. Tick-bite paralysis is with perinatal asphyxia and may be confused
unlikely to be the diagnosis because of the pres- with myoclonic seizures when it occurs without
ence of fever and the abnormal CSF. The apparent stimulation. The absence of eye
absence of ophthalmoplegia, the presence of movement or alteration in respiration pattern
fever, and abnormal CSF results are against the and the normal EEG can distinguish the two
diagnosis of botulism. Finally, a spinal cord vas- entities. However, the most practical way to
cular malformation is unlikely to be the diag- distinguish between seizures and jitteriness is in
nosis because of the presence of fever and the the following maneuver: grasping the jittering
absence of red blood cells in the CSF. In this limb will stop normal jitteriness but not seizure
case, respiratory failure is the major concern activity. Apneic spells of 10 to 15 seconds may
because of the involvement of the thoracic be seen in the premature newborn. They should

115
116 3: Pediatrics

be considered as a sign of brainstem immaturity 6. (D) The patient in this case developed a motor
rather than a pathologic condition. Apnea is seizure with preservation of consciousness,
seldom a seizure manifestation unless it is asso- most likely involving the left cortical areas con-
ciated with tonic deviation of the eyes or body trolling speech and right face movements.
stiffness. (Fenichel, 14) Benign rolandic epilepsy is the most likely diag-
nosis. It is an autosomal dominant condition
4. (E) Cerebral palsy comprises a spectrum of with incomplete penetrance occurring typically
static lesions of the central nervous system that between the age of 3 and 12 years. The area of
produce chronic problems with motor strength the brain around the sensorimotor fissure is
and/or control; it is not the result of a recog- involved in the genesis of seizure. In around
nized malformation. While the lesion is static 40% of cases, a family history of febrile seizure
by definition, it appears in a central nervous or epilepsy is found. Seventy percent of children
system that is undergoing developmental change have seizures only while asleep, and 15% only
during a period of rapid growth. As a result, its when awake. Interictal EEG may show unilat-
manifestations are not stable. Motor disability eral or bilateral spike discharges in the central
is the most frequent symptom, and it is the first or centrotemporal region. Seizures resolve
disability to be identified in affected children. spontaneously by the age of 14 in most cases.
Other central nervous system impairments Treatment of benign rolandic seizure is indi-
may be associated with the motor deficit. cated only in the case of frequent seizures or
Mental retardation is the most common asso- the occurrence of major motor seizures.
ciated disability, estimated to occur in 50% to Carbamazepine is a popular choice because of
66% of patients with cerebral palsy. Hearing its effectiveness in treating partial complex
deficits may be seen in up to 30% of affected seizures and its lack of cognitive side effects.
children, with higher prevalence in those who Absence seizure is unlikely to be the diagnosis.
have the athetoid form of cerebral palsy. It generally occurs during the daytime with
Seizures are seen in 30% to 50% of children brief stares with or without eyes blinking. The
with cerebral palsy, especially in hemiplegic preservation of consciousness rules out a com-
patients. Ataxic cerebral palsy is the rarest form plex partial seizure in this patient. Other
of cerebral palsy, most likely denoting dys- patients with benign rolandic epilepsy may
function of the cerebellum or its pathways. have complex partial seizures. (Fenichel, 3132)
Truncal and gait ataxias are more striking than
limb ataxia. (Fenichel, 269270; Nelson, 7380; 7. (A) Seizures may complicate any brain disor-
Rosenbloom, 350354) der in full-term infants. The time of onset of the
first seizure is helpful in establishing the cause.
5. (D) Kernicterus is a syndrome associated with Hypoxic-ischemic encephalopathy is the most
cerebral palsy. Unbound bilirubin can cross the frequent cause of seizure in the first 12 hours, fol-
bloodbrain barrier in the neonatal period, lowed by sepsis, meningitis, and subarachnoid
enter the nervous system, and produce neu- hemorrhage. Trauma and intrauterine infection
ronal damage. Neuronal necrosis and biliru- may cause a seizure in the first 24 hours.
bin staining in specific neuronal regions are the Pyridoxine dependency, intraventricular bleed
specific pathological features of kernicterus. in a term infant, and direct drug effects are rare
The most commonly affected regions of the causes of seizure in the first 24 hours. (Fenichel, 4)
brain are the basal ganglia, hippocampus, sub-
stantia nigra, cranial nerve, brainstem nuclei, 8. (C) Despite taking a comprehensive genetic his-
cerebellar nuclei, and anterior horn cells of the tory, doing a complete physical examination,
spinal cord. Cortical neurons are the least and performing extensive metabolic, chromo-
affected in kernicterus. This reflects the pre- somal, and neuroimaging studies, 50% of
dominance of extrapyramidal, gaze, and audi- patients with cerebral palsy have no evident
tory abnormalities as clinical symptoms of cause of their brain damage. Although prema-
kernicterus. (Ahdab-Barmada and Moossy, 4556) turity is the most common antecedent of cerebral
Answers: 413 117

palsy, the majority of infants who develop cere- enzyme systems responsible for urea synthesis.
bral palsy are full term. Lower birth weight Symptoms are caused by ammonia intoxication.
increases the risk of developing cerebral palsy; Seizures may complicate the initial hypotonia
however, even for very low birth weights (less and lethargy after the first week of birth. The
than 1500 g), the risk of developing cerebral most common drug withdrawal in the newborn
palsy is only 15% to 20%. Preterm infants who is narcotic withdrawal. Tremor, irritability, hyper-
develop intraventricular hemorrhage with activity, and autonomic instability are the earlier
extension to the white matter are at the greatest symptoms, which may be complicated by
risk for developing cerebral palsy. It seems that seizures in up to 5% of cases. First seizures typ-
in premature infants the best predictor of cere- ically occur from 24 to 72 hours after birth.
bral palsy is the presence of echodense cystic Subarachnoid hemorrhage may cause unex-
lesions in the periventricular white matter with pected seizures during the first day. Seizures
a diameter greater than 3 mm, which increases may occur after the first 24 hours in tuberous
the risk of developing diplegic spastic cerebral sclerosis. (Fenichel, 45)
palsy to 90%. The Apgar score has been a poor
indicator of babies at risk of cerebral palsy. Only 11. (B) Pyridoxine dependency is a rare autoso-
10% to 15% of newborns have an Apgar score of mal recessive disorder that may result from an
3 or less at 10 to 15 minutes, and the majority impairment of glutamic acid decarboxylase
of full-term newborns who develop cerebral activity. Seizures may begin immediately after
palsy have normal Apgar scores. Bilirubin birth or at any time thereafter. (Fenichel, 45)
encephalopathy may cause athetoid cerebral
palsy. Bilirubin is especially toxic to the basal 12. (C) Sydenham chorea is the neurological
ganglia and auditory nuclei in the brainstem. It sequela of rheumatic fever and the prototypical
makes the infant with athetoid cerebral palsy at immune disorder. Based on immunohisto-
high risk of developing neurosensory hearing chemical studies using concentrated IgG from
loss. It is not the best predictor of cerebral palsy. patients with Sydenham chorea, specific cross-
(Pidcock et al., 417422; Taft, 411418) reactivity of IgG to neuronal cytoplasmic anti-
gens in subthalamic and caudate nuclei were
9. (E) The time of onset of seizures is helpful in identified in 47% of acutely ill patients. Further,
determining their causes during the neonatal the streptococcal-induced aspect in Syndenham
period. Neonatal adrenoleukodystrophy is an chorea was proven by the absence of neuronal
autosomal recessive inheritance characterized staining following preabsorption of antibodies
by poor adrenal function with accumulation of with streptococcal membranes. Recent studies
very long chain fatty acids in the plasma. Initial focusing on the specific target site of the
symptoms are hypotonia and failure to thrive. autoantibodies in rheumatic fever have sug-
Other clinical manifestations include, facial gested lysogangliosides and [beta]-tubulin;
abnormalities, seizures, retinal degeneration, these compounds are a molecular mimic of the
poor muscle tone, enlarged liver, and adrenal group A streptococal carbohydrate epitope N-
dysfunction. Seizures generally appear between acetyl-[beta]-D-glucosamine. Mechanistically,
the first and fourth week. (Fenichel, 45) it is proposed that antibodies in Sydenham
chorea can induce signal transduction involv-
10. (A) An excessive level of unconjugated free ing calcium calmodulin dependent protein
bilirubin in the blood causes kernicterus during kinase II activation resulting in increased tyro-
the neonatal period. Unconjugated free bilirubin sine hydroxylase and dopamine. (Wolf 491496)
is neurotoxic, especially to the basal ganglia and
hippocampus. Seizures may occur from the 13. (A) Fructose-1.6-diphosphatase (FDPase) defi-
third day of birth during the second phase of ciency is a rare in autosomal recessive inher-
bilirubin encephalopathy. The newborn shows ited disease characterized by hyperventilation,
increasing tone with opisthotonos. Urea cycle seizure, hypoglycemic attacks or even coma,
disturbances are related to a defect in the lactic acidosis, nausea, and tremor. The absence
118 3: Pediatrics

of FDPase impairs the gluconeogenesis; this glucose concentration and normal anion gap.
explains the occurrence of lactic acidosis and Plasma quantitative amino acid analysis differ-
the accumulation of some amino acids. In the entiates the specific urea disorder. (Fenichel, 67)
absence of FDPase, the organism depends on
alimentary glucose and glycogenolysis to main- 16. (C) The clinical features of urea cycle distur-
tain a normal blood glucose level This results in bance are due to ammonia intoxication.
life-threatening attacks of hypoglycemia asso- Prorgressive lethargy, vomiting, and hypotonia
ciated with metabolic acidosis. (Fenichel, 45) develop the first day after delivery, even before
the initiation of protein feeding. Progressive loss
14. (B) Glycine encephalopathy results from a of consciousness and seizures follow on subse-
defect of glycine cleaving system. It is an auto- quent days. Vomiting and lethargy correlate well
somal recessive disorder. Affected newborns with a plasma ammonia concentration greater
are normal at birth but become irritable and than 120 mol/L, coma correlates with concen-
refuse feeding 6 to 8 hours after delivery. The trations greater than 180 mol/L, and seizures
onset is usually within 48 hours, but delays by correlate with concentration greater than 300
few weeks occur in milder allelic forms. mol/L. Death follows quickly in untreated
Hiccupping is an early and continuous feature. newborns. (Fenichel, 67)
Progressive lethargy, hypotonia, respiratory dis-
turbances, and myoclonic seizures follow. Some 17. (D) Neonatal hypoglycemia is defined as a
newborns survive the acute illness, but mental whole blood glucose concentration of less than
retardation, epilepsy, and spasticity characterize 20 mg/dL in premature and low-birth-weight
the subsequent course. (Fenichel, 5) newborns, less than 30 mg/dL in term new-
borns during the first 72 hours, and less than 40
15. (A) The patient described in this question devel- mg/dL in full-term newborns after 72 hours.
oped lethargy and hyperammonemia without (Fenichel, 10)
organic academia; the serum glucose was
normal. These findings are suggestive of urea 18. (B) Within the broad spectrum of dysgenetic
cycle disturbance. Six inborn errors in the urea abnormalities in the posterior fossa, the most
cycle have been described. Five of these repre- common lesions involve impaired development
sent a lesion at each of the five steps in the con- of the vermis associated with increased CSF
version of ammonia to urea. They include spaces. The most striking of these anomalies is
argininosuccinic aciduria, citrullinuria, hyper- DandyWalker malformation (DWM), in which
argininemia, and two conditions termed hyper- the enlarged CSF space results from cystic dis-
ammonemia, the more common one attributable tention of the fourth ventricle, with complete or
to a defect of ornithine transcarbamylase (OTC) partial agenesis of the cerebellar vermis,
and the other the result of a defect in mitochon- hypoplasia of the cerebellar hemispheres, and
drial carbamyl phosphate synthetase (CPS). The enlargement of the posterior fossa with elevation
genes for all components of the urea cycle have of the torcula and anterior displacement of the
been cloned. Additionally, a deficiency of N- brainstem; hydrocephalus develops in most
acetylglutamate synthetase has been reported. cases. The pathogenetic mechanism(s) leading to
This enzyme is responsible for the formation of the DWM remain poorly understood. In one
N-acetylglutamate, a required activator for mito- view, the cyst develops after failed incorporation
chondrial CPS. More recently, two genetic of the anterior membranous area into the
defects affecting the citrulline and ornithine choroid plexus and failed or delayed develop-
transporters have also been documented. ment of the foramen of Magendie in the poste-
The diagnosis of urea cycle disturbances is rior membranous area. Isolated inferior vermian
strongly suspected by the presence of clinical hypoplasia may occur with normal cerebellar
manifestations of ammonia intoxicationa hemispheres and brainstem. This lesion appears
blood ammonia concentration of 150 mol/L to represent an arrested incomplete dow growth
or higherassociated with a normal serum of the vermis, leaving an enlarged midline CSF
Answers: 1420 119

space that may be mistaken for a cystic lesion. istic cortical actin structure at the leading edge
Joubert syndrome is an autosomal recessive con- of migrating cells. In this respect, FLNA is
dition that presents with hypotonia, distur- believed to be essential for mammalian cell
bances in respiratory (hyperventilation and locomotion by stabilizing loose microfilament
central apnea) and oculomotor control, and later nets. Recent studies have suggested that fil-
psychomotor disturbances. The essential brain amin helps tether neural progenitors to the
morphology of Joubert syndrome includes ver- ventricular zone and that it is rapidly degraded
mian hypoplasia, impaired axonal decussation in the neural progenitor cells and highly
(with a deep interpeduncular notch), and thick, expressed in migratory neurons, consistent with
abnormally oriented superior cerebellar pedun- a role for initial migration. FLNA also appears
cles. Together, these features produce the neuro- to control migrating cell shape. However, the
radiologic picture known as the molar tooth identification of a second causative gene for
sign. Microcephaly vera refers to a reduction in PH due to autosomal recessive mutations in
brain size in the absence of other gross structural the ARFGEF2 gene has raised the possibility
abnormalities both within and outside of the brain. that this cortical malformation may not entirely
Clinically, affected patients have mental retarda- reflect a disorder in cell motility. ARFGEF2
tion and, infrequently, epilepsy. Periventricular encodes an ADP-ribosylation factor guanine
heterotopia refers to clusters of neuronal nodules exchange factor that converts GDP to GTP and
ectopically located along the walls of the lateral thereby activates the ADP-ribosylation factor.
ventricles and beneath an otherwise normal- The ADP-ribosylation factors have been impli-
appearing cortex. Clinically, affected patients cated in vesicle transport; thus mutations in
present with seizures in late adolescence, are ARFGEF2 are thought to disrupt certain trans-
generally of normal intelligence, but may have membrane proteins or adhesion molecules and
learning problems (dyslexia). Disruption of the also, potentially, neuroblast migration. (Lian,
actin cytoskeleton and impairment in cell motil- 614620).
ity have long been thought to be the major
cause of X-linked periventricular heterotopias. 20. (A) Subcortical band heterotopia is character-
Classical lissencephaly (type I lissencephaly) is ized by heterotopic neurons positioned midway
characterized by the loss of the folds of the brain between the gyrencephalic cortex and underly-
(sulci and gyri), an abnormally thick cortex, and ing the subventricular zone. Females harboring
the loss of cortical lamination. Most (70% to 80%) mutations in the X-linked doublecortin (DCX)
of lissencephaly can be attributed to deletions in gene develop subcortical band heterotopia,
the microtubule binding lissencephaly 1 (LIS1) whereas males develop classical lissencephaly.
and doublecortin (DCX) genes. Clinically, affected Females have two X chromosomes; because of
individuals tend to develop severe epilepsy X-inactivation, only some neurons lose dou-
(early onset at 6 months of age, infantile spasms), blecortin function. Presumably, those neurons
as well as profound mental retardation, diffuse with the mutant DCX protein fail to migrate into
hypotonia, and, later, spastic quadriplegia. (Lian, the cortex and form the underlying heterotopic
614620; Limperopoulos, 621627) band. The severity of epilepsy and develop-
mental delay in affected individuals correlates
19. (D) Disruption of the actin cytoskeleton and inversely with the thickness of the subcortical
impairment of cell motility have long been band heterotopia. The DCX protein is thought to
thought to be the major cause of X-linked play a key role in neural progenitor motility. It
periventricular heterotopia (PH) due to muta- may exert its effect on neuronal migration
tions in filamin A (FLNA). FLNA is an actin- through its polymerization with microtubules.
binding phosphoprotein that can promote actin RNA interferencemediated knockdown of
branching, tether large actin filaments, and DCX shows abnormal positioning of cortical
hold them in a perpendicular arrangement. The neurons within the intermediate zone and white
resulting three-dimensional orthogonal net- matter as well as inappropriate neocortical
work of actin filaments represents a character- lamina in rats. While genetic DCX mouse
120 3: Pediatrics

mutants do not demonstrate a clear cortical phe- or may not be unilateral. The quality is gener-
notype, loss of DCX function disrupts the rostral ally described as pounding, pulsing, and throb-
migratory stream and delays neuronal migra- bing, but the key feature is its intensity.
tion along this pathway. DCX appears to be Activities will be interrupted. Photophobia
required for nuclear translocation and mainte- and/or phonophobia are common and often
nance of bipolar morphology during migration prompt the adolescent to seek a quiet, dark
of these cells. (Lian, 614620). place to rest or even to sleep, as sleep often
produces significant relief. Nausea, vomiting,
21. (D) Cobblestone lissencephaly (or type II and abdominal pain may be the most disabling
lissencephaly) is characterized as a HARD(E) features, as a student with headache may be
syndrome [hydrocephaly (H), agyria (A) and able to stay in the classroom with pain, but the
retinal dysplasia (RD), with or without onset of nausea or vomiting necessitates a visit
encephalocele (E)]. Based on the gradient of to the school nurse. A migraine headache typ-
syndrome severity, the autosomal recessive ically last hours, even days (1 to 72 hours), but
cobblestone lissencephalies can be divided into does not generally occur more frequently than
three subclasses ranging from the more mild 6 to 8 times per month. More than 8 to 10
Fukuyama congenital muscular dystrophy attacks per month must warrant consideration
affecting primarily the Japanese population to of alternative diagnoses. (Lewis, 207246)
the moderate Finnish muscle-eye-brain disor-
der and the most severe WalkerWarburg 24. (E) Evaluation of the child with a complaint of
syndrome generally leading to early postnatal headache must include a thorough medical his-
death. Four genes have been associated with tory and complete physical, including a neu-
type II lissencephaly (POMT1 and POMT2 for rological examination. The clinicians first
WalkerWarburg syndrome, POMGnT1 for priority is to eliminate secondary (and poten-
Finnish muscle-eye-brain disorder, and fukutin tially life-threatening) causes of headache, such
for Fukuyama congenital muscular dystrophy); as tumors, infection, intoxication, or hydro-
each of these is implicated in the glycosylation cephalus. The headache history itself will, in
of [alpha]-dystroglycan. (Lian, 614620) most instances, yield the necessary informa-
tion to suggest a secondary cause of headache.
22. (C) By age 15, over 75% of children have expe- Age below 3 years, morning or nocturnal
rienced at least one headache. The prevalence headache, morning or nocturnal vomiting,
of recurrent headaches is 2.5% at 7 years and headache increased by Valsava or straining,
15% at 15 years. Of headaches that are acute explosive onset, progressive worsening over
and recurrent, migraines and tension-type time (chronic progressive pattern), declining
headaches (TTHs) are the first and second most school performance or personality changes,
prevalent. One study found a prevalence of and altered mental status warrant the per-
5.1% for TTH in children aged 14 to 18. (Walker, formance of ancillary diagnostic testing to
248254) exclude an organic cause. (Lewis, 207246)

23. (A) Migraine without aura is the most frequent 25. (D) Familial hemiplegic migraine (FHM) is an
form of migraine seen in pediatrics, accounting uncommon autosomal-dominant form of mi-
for 60% to 85% of all migraine. Families or graine with aura caused by a missense muta-
patients may recognize prodromal features: tion in the calcium channel gene (CACNA1A)
mood changes (euphoria to depression), irri- mapping to chromosome 19p13 in about 50% of
tability, lethargy, yawning, food cravings, or the families. Mapping to chromosome 1q31 has
increased thirst. Perhaps the most frequent been reported in other families with FHM.
heralding feature is a change in behavioral pat- Clinically, FHM is a migraine headache her-
terns or withdrawal from activities. The alded by an aura, which has stroke-like qual-
headache phase begins gradually and the pain ities producing some degree of hemiparesis.
is usually frontal or temporal in location; it may (Lewis, 207246)
Answers: 2130 121

26. (C) The migraine variants represent a hetero- lized for those in whom nausea and/or vomit-
geneous group of disorders characterized by ing represent a significant disability and/or pre-
headache accompanied by disturbing neuro- vents oral intake. (Walker, 248254)
logical signs, such as hemiparesis, altered con-
sciousness, nystagmus, or ophthalmoparesis. 29. (B) Despite a similar insult mechanism, preterm
Basilar-type migraine (BM), also known as basi- and term brains respond very differently to
lar artery or vertebrobasilar migraine, is the hypoxia/ischemia. Major regional differences
most common of the migraine variants. It is esti- in the distribution of injury, network excitability,
mated to represent 3% to 19% of all migraines. and long-term outcome have been observed in
The wide range of frequency relates to the rigor these two populations. Although these differ-
of the definition. Some authors included any ences are not absolute, the white matter of the
headache with dizziness within the spectrum preterm brain exhibits a higher degree of sus-
of BM, whereas others require the presence of ceptibility while the gray matter of the term
objective signs or symptoms of posterior fossa brain is more reflective of injury. These relative
involvement before establishing this diagnosis. susceptibilities underlie the prevalence of
The onset of BM tends to occur in young chil- periventricular leukomalacia (PVL) in the
dren with a mean age of 7 years, although the preterm infant and that of hypoxic encephalopa-
clinical entity probably appears as early as 12 to thy, seizures, and stroke in the term infant.
18 months as episodic pallor, clumsiness, and (Jensen, 628633)
vomiting in the condition know as benign
paroxysmal vertigo. Affected patients will have 30. (A) Confusional arousals, sleep terrors, and
attacks of intense dizziness, vertigo, visual dis- sleepwalking are the key NREM sleep para-
turbances, ataxia, and diplopia. The key features somnias. They are also termed arousal parasom-
of BM include vertigo, nausea and vomiting, nias as they appear during partial arousals from
ataxia, visual field defect, tinnitus, dysarthria, NREM sleep. Typically, they occur at the time of
and weakness. These transient features may last transition from deep NREM (stage N3) sleep
for minutes or up to an hour and are then fol- into the lighter stages of NREM sleep and
lowed by the headache phase. (Lewis, 207246) appear time-locked to the first third of night
sleep because N3 sleep is most abundant at this
27. (A) Vertigo is the most frequent manifestation time of the night. Sleep deprivation and fever
of vertebrobasilar migraine (73%). Nausea and can trigger all three forms of arousal parasom-
vomiting is seen in 30% to 50% of cases, ataxia nia. A genetic predisposition and acquired dis-
in 43% to 50%, visual field defects in 43%, turbances that trigger an increase in shifts from
diplopia in 30%, tinnitus in 13%, and weak- slow-wave sleep to lighter stages of sleep (such
ness (hemiplegia, quadriplegia, diplopia) in as sleep-disordered breathing, periodic limb
20%. (Lewis, 207246) movement, gastroesophageal reflux), combined
with a vulnerable age (generally 2 to 12 years),
28. (A) The American Academy of Neurology seem to precipitate arousal parasomnias.
(AAN) recommends a trial of ibuprofen as first- Separation anxiety may be a predisposing factor
line pharmacological treatment for migraine in for both sleep terrors and sleepwalking. Cyclic
children. Acetaminophen should be utilized in alternating patterns, which are periodic EEG
those with allergy, intolerance to nonsteroidal events of NREM sleep characterized by
anti-inflammatory drugs (NSAIDs), or with con- repeated and spontaneous high-voltage EEG
traindications to NSAID use. For those over 12 periods that recur at regular intervals of up to
years of age who fail ibuprofen and/or aceta- 2 minutes in duration, are a reliable marker of
minophen, it is reasonable to move to suma- unstable sleep and as such are increased during
triptan nasal spray. For those under age 12, one the slow-wave sleep of children with sleep
source proposes repeating the initial nonspe- terrors.
cific analgesic in 2 hours if symptoms do not The age of onset of sleepwalking is typi-
resolve. Additionally, antiemetics can be uti- cally between 5 and 10 years. Mild episodes in
122 3: Pediatrics

which a toddler sits up and crawls around the vation of these regions, which appears to nor-
bed or the child walks quietly in sleep to come malize with stimulant medication. (Casey,
and stand by the bed of the parents may go 119124)
unnoticed. Other children may become agitated
and run around the house. Some patients have 33. (A) Apneic spells in an otherwise normal-
injured themselves by unconsciously carrying appearing newborn are sign of brainstem
out dangerous behaviors like jumping out of a immaturity and do not point to a pathological
second-story window. Others may unknow- condition. The sudden onset of apnea and
ingly risk injury with behaviors such as walking states of decreased consciousness, especially
outdoors on a cold winter night and consequent in premature newborns, suggests an intracra-
exposure to hypothermia. Autonomic dysfunc- nial hemorrhage with brainstem compression.
tion may occur in the form of sweating and Apneic spells are almost never a seizure man-
flushing of the face. Some patients exhibit a ifestation unless they are associated with tonic
combination of sleep terrors and sleepwalking, deviation of the eyes, tonic stiffening of the
although one manifestation or another will pre- body, or characteristic limb movement.
dominate. The main differential diagnosis of (Fenichel, 3)
arousal parasomnias is nocturnal seizures.
Nocturnal frontal lobe seizures are due to muta- 34. (B) West syndrome is a triad of infantile spasms,
tions in the genes CHRNA2, CHRNA4, or developmental retardation or regression, and
CHRNB2, which regulate the expression of hypsarrhythmia on EEG. The syndrome pres-
nicotinic acetylcholine receptors. A full EEG ents in infants aged between 6 and 18 months.
montage should be incorporated into the Presence of a hypsarrhythmic EEG confirms the
polysomnogram for the investigation of any diagnosis of infantile spasms. EEG patterns may
nocturnal events suspected to be seizures. evolve over time; they initially appear in the
(Kotagal, 659665) sleep EEG record and subsequently present
during the awake state. Hypsarrhythmia is seen
31. (A) Structural MRI studies of abnormalities in in 75% of patients with West syndrome. It con-
children with ADHD have reliably shown sists of diffuse giant waves (high voltage, above
smaller than normal regional brain volumes. In 400 V) with a chaotic background of irregular,
addition to an overall reduction in total brain multifocal spikes and sharp waves and very
volume, four major findings regarding regional little synchrony between the cerebral hemi-
differences were notable. Relative to controls, spheres. During sleep, the EEG may display
individuals with ADHD showed smaller MRI- bursts of synchronous polyspikes and waves.
based volumes of the basal ganglia, specifically A pseudoperiodic pattern may be evident.
in the caudate nucleus and globus pallidus; Persistent slowing or epileptiform discharges in
larger posterior regions and smaller anterior the hypsarrhythmic background may be present
brain regions; a smaller cerebellar vermis; and and may represent an area of focal dysfunction.
smaller white matter tracts. A recent meta- Hypsarrhythmia rarely persists beyond the age
analysis of structural imaging findings showed of 24 months. It may evolve into slow spike-
reliable regional reductions in the caudate, cere- and-wave discharges. (Fenichel, 20)
bellar vermis, and corpus callosum. Longitudinal
MRI-based anatomic studies of individuals with 35. (D) Neurofibromatosis type 1 (NF1) is the most
ADHD suggest that these abnormalities are pres- common form of autosomal dominant phako-
ent early in childhood. (Casey, 119124) matoses. Although many cases are heritable,
approximately 30% to 50% arise de novo from
32. (C) Functional MRI imaging studies of ADHD spontaneous mutations. The diagnosis of NF1 is
show that multiple neural systems are involved made on the basis of clinical features, requiring
in this disorder, including the prefrontal cortex, the presence of at least two of the following
caudate nucleus, cerebellum, and parietal major criteria: six or more caf au lait spots, axil-
cortex. Most of these studies show hypoacti- lary or inguinal freckling, two or more cutaneous
Answers: 3138 123

neurofibromas, one plexiform neurofibroma, amphetamines, with dosage titration based on


characteristic bony defects, optic glioma, two or side effects and clinical response. However
more Lisch nodules of the iris, or a first-degree modafinil, a wakefulness-promoting agent,
relative with NF1. Brain tumors occur with was recently found to be effective in the treat-
increased frequency and are exclusively astro- ment of children with narcolepsy. Although its
cytic in NF1 patients. Nearly 15% of patients mechanism of action is unclear, the drug is rec-
have optic gliomas; however, a rapid deterio- ommended as first-line therapy for narcolepsy
ration of visual acuity is not common. Never- in adults. It also showed efficacy in 10 of 12
theless, these tumors require careful monitoring children with narcolepsia. Several studies have
because they can progress to the point where outlined its possible effect on dopamine, adren-
they compromise vision. The brainstem and aline, noradrenaline, serotonin, and gamma-
cerebellum are common sites of tumors aminobutyric acid (GABA) systems. The
although they can also be supratentorial. Most elimination half-life is 13.8 hours and the max-
tumors identified on MRI are grade I astrocy- imum concentration is achieved in 2 to 4 hours.
tomas and do not progress. Unidentified bright The most common adverse events are mild,
signals are areas of increased signal in the basal including headache (13%), nervousness (8%),
ganglia, thalamus, brainstem, and cerebellum and nausea (5%), with no evidence of toler-
visible on T2-weighted images. They are ance. Methylphenidate is a widely used, potent
thought to be either hamartomas or prolifera- stimulant. It primarily blocks the reuptake of
tions of blood vessels with surrounding monoamines (mainly dopamine) and, unlike
increased tissue edema. They occur more com- amphetamines, does not inhibit the vesicular
monly in children but decrease in size and fre- monoamine transporter. Clinical experience
quency with age. (Lee, 135141) shows that methylphenidate improves daytime
sleepiness in narcolepsy patients at daily doses
36. (A) Narcolepsy is a rare disorder characterized of 10 to 100 mg. The duration of action is
by symptoms of excessive daytime sleepiness 4 hours and the elimination half-life is 6 hours.
(EDS), which varies through the day and is The short-acting effect of methylphenidate is
associated with almost daily naps. A history of useful in cases where modafinil must be sup-
cataplexy (sudden loss of muscle tone triggered plemented at a specific time of the day or in sit-
by emotion) is a specific and common symp- uations where maximum alertness is required.
tom. Other symptoms include hypnagogic hal- Amphetamines have been used to treat nar-
lucinations, sleep paralysis, and disturbed colepsy since 1935. They promote monoamine
nocturnal sleep. The etiology of narcolepsy is (catecholamine but also serotonin) release
unknown; however, human and animal data through multiple mechanisms. At low doses,
suggest that it is associated with abnormalities amphetamines produce a reverse efflux of
in the hypocretin system, with loss of hypocre- dopamine (and other monamines) through
tin neurons in the hypothalamus of humans monoaminergic reuptake sites. At higher doses,
with narcolepsy and cataplexy. Hypocretin they also inhibit vesicular monoamine trans-
appears to stabilize and prevent inappropriate porters and monoamine oxidase. Cytotoxicity
changes in conscious state, such as the rapid for dopaminergic neurons can occur at high
switches between wakefulness and REM sleep doses in animals. Amphetamines are very effec-
and disordered sleep patterns characteristic of tive against sleepiness in narcolepsy but also
narcolepsy. (Kothare, 666675; Dauvilliers 499451; induce frequent adverse effects, including irri-
Keam, 699703) tability, aggressiveness, insomnia, hyperten-
sion, and abnormal movements. (Kothare,
37. (D) In pediatric patients, stimulant medica- 666675; Dauvilliers 499451; Keam, 699703)
tions for the treatment of narcolepsy are used
with the goal of providing optimal alertness 38. (C) Sodium oxybate is a GABA B receptor ago-
during school hours and other social situations. nist currently approved by the U.S. Food and
Most clinicians prescribe methylphenidate or Drug Administration (FDA) for the treatment
124 3: Pediatrics

of narcolepsy with cataplexy in patients above ity of individuals experience daytime seizures.
16 years of age. According to the American Onset ranges from infancy to adulthood. About
Academy of Sleep Medicine (AASM) practice 80% of individuals develop ADNFLE in the first
parameters, sodium oxybate is effective for two decades of life, with a mean age of 10 years.
treatment of cataplexy, daytime sleepiness, and Neurological examination is normal and intellect
disrupted sleep due to narcolepsy and may be is usually preserved. Within a family, the mani-
effective for treatment of hypnagogic halluci- festations of the disorder may vary considerably.
nations and sleep paralysis. Although the ADNFLE is lifelong but not progressive. As an
administration of sodium oxybate is associ- individual reaches middle age, attacks may
ated with a modulation of serotoninergic, become milder and less frequent. Molecular
dopaminergic, and opioid activity, along with genetic testing reveals mutations in CHRNA4
an increase in slow-wave sleep, its pharmaco- or CHRNB2 encoding, respectively, the 4 and
logic action on cataplexy remains unknown. 2 subunits of the neuronal nicotinic acetyl-
When used for the treatment of narcolepsy, choline receptor in some 20% to 30% of individ-
one-half of a therapeutic dose of sodium oxy- uals with a positive family history and less than
bate is administered at bedtime and repeated 5% of individuals with a negative family history.
2.54.0 h later, providing effective plasma con- Juvenile absence epilepsy (JAE) is believed to
centrations throughout the night while ensur- represent a smaller fraction of absence epilep-
ing that the majority of the drug has been sies and makes up about 0.2% to 3.0% of child-
eliminated when the patient awakens in the hood epilepsy, with a prevalence of 0.1 per
morning. (Kothare, 666675; Dauvilliers, 499451; 100,000 persons in the general population. The
Keam, 699703) etiology for JAE is genetic; allelic association with
a glutamate receptor gene (GRK1) polymor-
39. (E) Absence epilepsy represents about 10% to phism has been demonstrated in some families.
15% (range 1.4% to 13.0%) of childhood epilepsy Benign neonatal familial convulsions (BNFCs)
cases. Large epidemiological studies in Europe have Mendelian transmission and are consid-
and the United States have shown an incidence ered an autosomal dominanttransmitted
of about 6 to 8 per 100,000 in children aged 0 to epilepsy within complete penetrance. Significant
15 years, with a prevalence of 5 to 50 per 100,000 clinical and genetic heterogeneity exists. BNFC
in the general population. Peak age of onset is 6 belongs to the group of genetic disorders caused
to 7 years with a typical range of 2 to 13 years. by a channelopathy. The defect is in the voltage-
Less than 3% are younger than 2 years of age at dependent potassium channel (KCNQ1,
onset. Girls are more often affected by a 3:2 to 2:1 KCNQ2, and KCNQ3). The loci corresponding to
ratio. Family history of epilepsy is present in these areas have been identified on both chro-
about 15% to 45% of cases. The etiology for mosomes 20q13 and 8q24. The KCNQ gene reg-
childhood absence epilepsy (CAE) is genetic, ulates neuronal excitability by controlling the
with complex, multifactorial inheritance. To date, duration of the action potential and reducing the
linkage in some families has been correlated with M-current by 20% to 25%, thereby creating
defects in both the GABAA receptor [gamma]2 hyperexcitability. Benign childhood epilepsy
subunit and the voltage-gated Ca2 channel with centrotemporal spikes (BECTS) is the most
[alpha]1A subunit (CACNA1A). Autosomal common partial epilepsy of childhood and
dominant nocturnal frontal lobe epilepsy accounts for 16% of epilepsies beginning before
(ADNFLE) is characterized by clusters of noc- age 15 and 24% of epilepsies between ages 5 and
turnal motor seizures, which are often stereo- 14. At least two thirds of all idiopathic partial
typed and brief (5 seconds to 5 minutes in epilepsies are classified as BECTS. The preva-
duration). They vary from simple arousals from lence is 1 to 2 per 1,000. Onset of seizures typi-
sleep to dramatic, often bizarre hyperkinetic cally occurs between ages 3 and 16 years, with a
events with tonic or dystonic features. Affected peak frequency around 5 to 8 years. Rare cases of
individuals may experience aura. Retained BECTS may present as early as 1 year of age.
awareness during seizures is common. A minor- There is a slight male predominance, and 60% of
Answers: 3941 125

children with this syndrome are boys. The probably the best-known association with
genetic basis for BECTS is complicated. autism and is the leading cause of inherited
Centrotemporal spikes are present in EEG in mental retardation. Current estimates are that
11.0% of the siblings of the probands but in only FRAX accounts for 1% to 2% of all cases of
1.6% of the general population. The centrotem- autism spectrum disorder (ASD), similar to the
poral spikes in the EEG pattern have an autoso- rate of duplication (15q). Recently, associations
mal dominant inheritance with incomplete between autism and the premutation carrier
penetrance and age dependency but does not status of FRAX and FRAX mosaics with partial
necessarily lead to BECTS. A recent multicenter methylation have been reported as well.
linkage study using monozygotic twins failed Tuberous sclerosis complex (TSC) also is not a
to show any concordant data. The low absolute major cause of autism, occurring in only 1% to
prevalence of BECTS in siblings (2.3%) and 4% of large unselected autism samples,
family members makes it difficult to determine although the frequency is higher (8% to 14%) in
whether the etiology is purely genetic. The most autism patients with epilepsy. The genes for
current opinion on the genetic cause of BECTS TSC are located on chromosomes 9q34 (TSC1)
therefore is that major nonfamilial environmen- and 16p13 (TSC2). Rett syndrome is a rare dis-
tal factors determine the development of the dis- order (1 in 10,000 to 15,000) now known to be
ease. (Bergqvist, 106120) a single-gene disorder caused by mutations in
the MECP2 gene on the X chromosome. The
40. (A) The etiology of juvenile myoclonic epilepsy frequency of MECP2 mutations or duplications
(JME) is genetic, but inheritance is complex, with in ASD has been variable but may be as high as
significant heterogeneity. Multiple genes have 1% to 2% in populations at academic centers.
been linked to JME. A defect in the GABAA- Joubert syndrome and related cerebellar dis-
gated chloride channel was associated with one orders (JSRD) are autosomal recessive disor-
family with JME (GABARA1). The GABAA- ders characterized by congenital ataxia, diffuse
gated chloride channel mediates fast inhibition; hypotonia, developmental delay, abnormal res-
a defect could reduce GABA current and thereby piratory patterns, and oculomotor apraxia. The
increase neuronal hyperexcitability. A defect in pathognomonic neuroradiological finding in
the calcium channel CACNB4 is linked to the T- JSRD is the presence of the molar tooth sign
type calcium current and the synchronized (MTS) on brain imaging, which is the result of
activity of the thalamus. Finally, defects in cerebellar vermis hypoplasia (CVH), thick and
nonion channel ligands such as EFHC1 and maloriented superior cerebellar peduncles, and
BRD2 have been associated with some families an abnormally deep interpeduncular fossa. The
with JME. The functions of these genes are not genetic basis of JSRD has been partially eluci-
clear at this time. (Bergqvist, 106120) dated with the identification of three causative
genes and two other unique loci where the
41. (B) Autism is also observed in many single- genes have not yet been found. The JBTS1 locus
gene disorders. Duplication of 15q is probably (OMIM %213300), located on 9q34.3, is associ-
the most important known cause of idiopathic ated with Joubert syndrome (JS, herein refer-
autism in the maternally derived duplication of ring to the pure form of the disease with
a region on proximal 15q, which overlaps with restricted involvement of retinal, renal, or other
the region involved in PraderWilli and system). The JBTS2 locus (OMIM %608091),
Angelman syndromes. These duplications located on 11cent, is associated with extreme
occur at a rate of 1% to 3%. The patients with phenotypic variability, including frequent reti-
15q duplication syndrome almost always meet nal dystrophy and renal involvement. The
the diagnostic criteria for autistic disorder; as a JBTS3 locus (OMIM #608629), on 6q23.3 and
group, however, they show more significant due to mutations in the AHI1 gene, is associ-
cognitive impairment coupled with gross ated with JS and variably associated with cor-
motor delays, hypotonia, epilepsy, and mild facial tical polymicrogyria. However, there are rare
dysmorphisms. Fragile-X syndrome (FRAX) is patients with renal cystic disease with AHI1
126 3: Pediatrics

(JBTS3) mutations. Mutations were observed unilateral clonic or tonicclonic seizures,


in about 8% to 11% of patients with JSRD. The mostly with fever, in the first year of life. Later,
JBTS4 locus (OMIM #609583), on 2q13 and due other seizure types occur, including myoclonus,
to mutations in NPHP1, was first implicated in atypical absences, and partial seizures. Develop-
juvenile nephronophthisis. Mutations were mental delay progressively appears from the
subsequently identified in 2% to 3% of patients second year. Seizures remain fever-sensitive
with JSRD, exclusively in those with renal and tend to evolve to status epilepticus. This
involvement. The JBTS5 locus (OMIM sensitivity to fever seizures led to the identifi-
#610188), on 12q21.3 and due to mutations in cation of mutations in SCN1A, which account
the CEP290 gene, is associated with pleiotropic for 75% of Dravet syndrome cases. Seizures in
forms of the disease including severe retinal Dravet syndrome are highly pharmacoresis-
and renal involvement. (Geschwind, 4964; Zaki, tant. The combination of clusters of spasms,
556565) psychomotor deterioration, and hypsarrhyth-
mia defines West syndrome, which occurs
42. (B) Malignant migrating partial seizure in mainly between 3 and 12 months of age.
infancy (MMPSI) is an epileptic encephalopa- Cryptogenic cases account for 9% to 15% of the
thy characterized by onset in the first 6 months cases, the rest being symptomatic. The symp-
of life, of rapidly progressive partial seizures tomatic cases are associated with several pre-
that become subcontinuous. Their onset natal, perinatal, and postnatal factors. Various
migrates from one area of the cortex to the brain dysgeneses (lissencephaly, hemimega-
other and major deterioration of the psy- lencephaly, focal cortical dysplasia, septal dys-
chomotor abilities appears. Early seizures have plasia, or callosal agenesis), chromosomal
motor and autonomic components, later (including Downs syndrome, del1p36) or
seizures are more polymorphic, varying from single gene (mutations of ARX or STK9 gene)
one seizure to the next in a given patient. causes are reported. Cryptogenic late-onset
Seizures last several minutes longer than usual epileptic spasms is a cryptogenic, late-onset
partial seizures in infancy. They tend to be epileptic spasms beginning between 12 and 48
more frequently generalized as time goes by. months of age; they have a particular pattern
Myoclonus is rare and spasms are exceptional. that is intermediate between West and
By the end of the first year of life, seizures LennoxGastaut syndromes. EEG does not
become almost continuous and occur in clus- show classical hypsarrhythmia but a temporal
ters: seizures lasting several weeks, during or temporofrontal slow-wave or spike focus
which the infant deteriorates considerably, and combined with slow spike waves. Ictal events
followed by disappearance of seizures during combine spasms in clusters, tonic seizures, and
a few weeks with slow improvement of the atypical absences. Ictal EEG discloses a gener-
condition. Microcephaly progressively occurs. alized high-voltage slow wave followed by dif-
To date, no cause has been identified by histor- fuse voltage attenuation with superimposed
ical, biochemical, radiological, or histological fast activity, typical of the epileptic spasms,
investigations. No familial case or consanguin- occurring in clusters.
ity has been reported. A genetic study failed to Childhood epileptic encephalopathy
find mutations in sodium (SCN1A, SCN2A), (LennoxGastaut syndrome [LGS]) constitutes
potassium (KCNQ2, KCNQ3), and chloride 1% to 4% of childhood epilepsies. The mean
(CLCN2) ion channels in three children with age of epilepsy onset is 26 to 28 months. The
migrating partial seizures in infancy. (Nabbout, syndrome is characterized by multiple types
161166) of seizures, mental retardation or regression,
and abnormal EEG with generalized slow
43. (D) The clinical picture of the patient described spike-and-wave discharges (1.5 to 2 Hz). The
in this vignette is suggestive of Dravet syn- most common seizure types are tonicaxial,
drome. It is characterized by the occurrence, atonic, and absence seizures, but myoclonic,
in an otherwise normal infant, of generalized or generalized tonicclonic, and partial seizures
Answers: 4248 127

can be observed. Seizures often are resistant to may produce a delayed compression of the
therapy. (Nabbout, 161166) brainstem. (Fenichel, 414; Rivkin, Anderson, and
Kaye, 5156)
44. (B) Epileptic encephalopathy with suppres-
sion bursts is a severe condition that begins 46. (B) Subarachnoid hemorrhage results from
within the first 3 months of life. EEG demon- tearing of the superficial veins by shearing
strates bursts of paroxysmal activity (poly- forces during a prolonged delivery with the
spikes) lasting several seconds and alternating head engaged. Seizures occurring in a normal
with episodes of flat or low amplitude tracing, newborn in the first or second day of life may
a combination called suppression bursts. This be the only clinical manifestation. (Fenichel,
pattern is present in both awake and sleep 414; Rivkin, Anderson, and Kaye, 5156)
states or mainly during sleep. It is associated
with partial seizures variably combined with 47. (C) Cerebral venous thrombosis may compli-
myoclonus or spasms. The classification of the cate sepsis, asphyxia, and coagulopathy.
International League Against Epilepsy recog- Superior sagittal thrombosis may occur without
nizes two conditions with suppression bursts: known predisposing factors. Head MRI is the
early infantile epileptic encephalopathy (EIEE) best examination to assess the involved vessels.
or Ohtahara syndrome and neonatal myoclonic (Fenichel, 414; Rivkin, Anderson, and Kaye, 5156)
encephalopathy (EME). In EIEE, there are
spasms and the suppression burst pattern is 48. (D) The decarboxylation of leucine, isoleucine,
often asymmetric, mainly affecting the side of and valine is accomplished by a complex
a cortical malformation, usually hemimega- enzyme system, branched-chain alpha-
lencephaly or focal cortical dysplasia. Aicardi ketoacid dehydrogenase. Deficiency of this
syndrome, olivarydentate dysplasia, and enzyme causes maple syrup urine disease.
schizencephaly are other conditions in which Affected newborns are normal at birth but
such tracings are encountered. In EME, there is develop poor feeding and vomiting during the
no radiological evidence of a brain lesion, the first week of life, in addition to hypotonia.
patients exhibit erratic and massive myoclonus, Convulsion and hypoglycemia are frequent
and there is familial recurrence. Nonketotic complications. The correction of hypoglycemia
hyperglycinemia, Menkes disease, pyridoxine does not improve the clinical condition. The
and pyridoxal phosphate dependencies, and diagnosis is suspected by the odor of maple
glutamate transporter defect may be involved syrup found in the urine, sweat, and cerumen.
and share excess of glutamate transmission. It is confirmed by finding increased plasma
Seizures onset in benign infantile focal epilepsy concentration of the three branched-chain
with midline spikes and waves during sleep in amino acids or enzyme deficiency in peripheral
infants between 4 and 30 months of age. Seizure leukocytes. The urine contains a high level of
manifestations are typical, characterized by branched-chain amino acids and their keto
cyanosis, staring, and rare lateralizing signs of acids. These keto acids may be detected by
short duration. There is a strong EEG marker, a adding 2,4 dinitrophenylhydrazine to the
spike followed by a bell-shaped slow wave, urine; if the result is the formation of yellow
localized in the midline regions that is present in precipitate, maple syrup urine disease is
all subjects only during sleep. (Nabbout, 161166) the most likely diagnosis. Lowering of the
branched-chain amino acids may be attempted
45. (A) Subdural hemorrhage is the consequence by exchange transfusions, peritoneal dialysis,
of a tear in the tentorium near its junction with and special diet. Isovaleric acidemia is a rare
the falx. Excessive vertical molding of the head autosomal recessive condition related to a defi-
in vertex presentation or anteroposterior elon- ciency of isovaleryl CoA dehydrogenase.
gation of the head in face and brow presenta- Clinical manifestations include lethargy, vom-
tion may be the cause of the tear. The iting, convulsions, and severe acidosis in the
accumulation of blood in the posterior fossa first few days of life. The characteristic odor of
128 3: Pediatrics

sweaty feet may be present. The diagnosis is spike-wave complexes on EEG, and mental
established by demonstrating marked eleva- retardation) the most likely diagnosis. Mental
tion of isovaleric acid or its metabolites in retardation will appear later, since more than
urine. 90% of patients will be mentally retarded by the
Glycine encephalopathy is an autosomal age of 5 years. Sixty percent of patients with
recessive disorder caused by a defect of the LennoxGastaut syndrome may have an iden-
glycine cleaving system. Affected newborns tified underlying cause and 20% have a his-
are normal at birth but become irritable with tory of infantile spasms. EEG during a tonic
hiccuping usually within 48 hours to several seizure may show a one-spike wave per second
weeks. Myoclonic seizures, hypotonia, and followed by generalized rapid discharges with-
lethargy may follow. The diagnosis is made out postictal depression. Seizures are difficult
after seeing hyperglycinemia and a high to control. Valproate and clonazepam are the
glycine concentration in the CSF. drugs of choice. Felbamate, lamotrigene, and
Carbamoyl phosphate synthetase defi- topiramate have shown promise as add-on
ciency may cause progressive lethargy, vomit- drugs in refractory cases. (Fenichel, 2223)
ing, and hypotonia in the first day of life. The
diagnosis is made by demonstrating serum 51. (C) Infantile spasms are age-dependent
hyperammonemia without organic acidemia. myoclonic seizures that are always seen before
Bilirubin encephalopathy results from the the age of 1 year, with a peak age of onset
neurotoxic effect of a high unconjugated free between 4 and 7 months. An underlying cause
bilirubin level. It may cause hypotonia, is found in 75% of cases. Perinatal asphyxia,
lethargy, and seizures. The branched-chain congenital malformations, and tuberous scle-
amino acid level in the serum is normal but rosis are common causes. The association of
the bilirubin level is high. (Behrman, 410412; infantile spasms, agenesis of the corpus callo-
Fenichel, 46) sum or other midline cerebral malformations,
and retinal malformation is called Aicardi syn-
49. (D) DiGeorge syndrome is a congenital drome. Pertussis immunization is not a cause
hypoplasia of organs derived from the third of infantile spasms. Spasms can be extensor or
and fourth pharyngeal pouches. It results in flexor movements and generally occur in clus-
hypoplasia or agenesis of the thymus and ters after the infant awakens from sleep. During
parathyroid glands, the auricle, and the external the early stages of infantile spasm, the EEG
auditory canal; congenital cardiac anomalies; may show hypsarrhythmia and a chaotic and
cleft palate; and short stature. It is associated continuously abnormal background of very
with microdeletion of chromosome 22q11. The high voltage and slow waves and spikes.
main features are hypocalcemia, seizures, con- Within few weeks, greater interhemispheric
genital heart disease, lymphocytopenia, and synchrony replaces the original chaotic pattern
multiple minor anomalies. Affected newborns of hypsarrhytmia. ACTH or corticosteroids
may die from cardiac causes during the first may be effective in the control of infantile
month. Frequent infections due to a defect of spasms. (Fenichel, 1920)
cell-mediated immunity and failure to thrive
may complicate the course of the surviving 52. (D) The symptoms of this patient are consis-
newborn. (Behrman, 694; Fenichel, 9) tent with the diagnosis of narcolepsy and cat-
aplexy: a sleep disorder characterized by
50. (C) The patient described in this case devel- abnormal latency from sleep onset to REM
oped symptoms of tonic seizures and atypical sleep. REM sleep is attained in less than 20
absence seizures. The EEG pattern is compati- minutes. Hypotonia and dreaming occur nor-
ble with absence or atonic seizures. The asso- mally during REM sleep. In narcolepsy and
ciation of these symptoms to the EEG findings cataplexy, these phenomena occur during
makes LennoxGastaut syndrome (which is wakefulness. This may induce hypnagogic
characterized by the triad of seizure, slow hallucinations: vivid, frightening visual and
Answers: 4955 129

auditory perceptions occurring at the transi- At least 70% of these patients have an associated
tion between wakefulness and sleep. The seizure disorder. The aphasia may be primarily
sudden loss of tone in cataplexy may be receptive or expressive, and auditory agnosia
induced by excitement; the paralysis most com- may be so severe that the child is unaware of
monly affects the face or hands more than the everyday sounds. Hearing is normal, but behav-
total body. Sleep paralysis, a generalized hypo- ioral problems, including irritability and poor
tonia occurring in the transition between sleep attention span, are particularly common. Formal
and wakefulness, complicates two thirds of testing often shows normal performance and
cases of narcolepsy and cataplexy and is gen- visuospatial skills despite poor language. The
erally experienced once or twice each week. seizures are of several types, including focal or
The diagnosis of narcolepsy and cataplexy is generalized tonicclonic, atypical absence, partial
made by a multiple sleep latency test showing complex, and occasional myoclonic. High-ampli-
a REM onset sleep latency of less than 4 to 5 tude spike-and-wave discharges predominate
minutes. Pharmacological treatment of nar- and tend to be bitemporal but can be multifocal
colepsy has depended on the use of central or generalized. In the very early stages of the
nervous system stimulants to increase wake- condition, the EEG findings may be normal. The
fulness, vigilance, and performance. The med- spike discharges are always more apparent
ications considered effective in the treatment of during non-REM sleep; thus a child suspected to
narcolepsy include dextroamphetamine, pemo- have LandauKleffner syndrome should have
line, methylphenidate, and methamphetamine. an EEG during sleep, particularly if the awake
These stimulants are associated with sympath- record is normal. CT and MRI studies typically
omimetic side effects, limitations in efficacy, yield normal results. Microscopic examination
and negative effects on nighttime sleep. of surgical specimens has shown minimal gliosis
Modafinil, a new wakefulness-promoting but no evidence of encephalitis. Progressive
agent, has been shown to be effective and is myoclonic epilepsy is unlikely to be the diagno-
well tolerated. (U.S. Modafinil in Narcolepsy sis because of the speech involvement in this
Multicenter Study Group, 4348) patient and the absence of myoclonus, cerebellar
ataxia, or involuntary movements. Temporal
53. (A) Lafora disease is a progressive myoclonic lobe tumor is a rare diagnostic possibility that
epilepsy probably transmitted by autosomal may cause seizures with speech disturbance. The
recessive inheritance. The age of onset is normal head MRI rules out that diagnosis. In the
between 11 and 18 years. Mental retardation Rasmussen syndrome, the affected children have
appears early in the course of the disease, progressive motor seizures that are resistant to
whereas ataxia, spasticity, and involuntary anticonvulsive treatment. Progressive hemiple-
movements occur late. In later stages of the dis- gia may develop in the body side of seizures and
ease, EEG may show nonspecific generalized may persist after seizures have stopped.
polyspike discharges that are not activated by (Behrman, 19981999; Fenichel, 30)
sleep. Seizures are generally refractory to most
anticonvulsant drugs as the disease progresses. 55. (B) The most likely diagnosis in this case is
Inclusion bodies, an aggregate of filaments com- GuillainBarr syndrome (GBS), which is the
posed of polyglucosans, are seen on liver most common cause of acquired paralysis in
biopsy. In rare cases, they may be absent even in an otherwise well child. A nonspecific viral or
late stages of the disease. (Fenichel, 2829) bacterial infection may occur prior to the onset
of paralysis, but in 25% of cases there is no
54. (D) This case describes a previously normal antecedent of infection or vaccination. Typically,
child with progressive loss of language skills the onset of weakness is insidious, following
associated with a seizure disorder. The most an ascending progression from lower extremi-
likely diagnosis is LandauKleffner syndrome. ties to upper extremities, and may involve the
This is a condition of unknown cause, more trunk or the cranial nerves. Pain in the extrem-
common in boys, with a mean onset at 5.5 years. ities is reported in 80% of cases. The patient in
130 3: Pediatrics

this case developed symmetric weakness in all ent with other rheumatic fever manifestations
extremities that caused an ataxic gait, decreased during an acute episode, or it may present in
deep tendon reflexes, and no sensory level: isolated form in the so-called pure chorea. Its
these are key findings for the diagnosis of GBS. etiology and pathophysiological mechanisms
The facial paresis reported in this patient reflects are still unclear, although its relation with a pre-
the involvement of the facial nerve, which is vious group A beta-hemolytic streptococcus
the most frequent cranial nerve affected by this infection is well established. There is also evi-
disease. Respiratory failure, unstable blood dence of the participation of immunological
pressure, and arrhythmia are less common but mechanisms in its pathogenesis, such as the
grave complications of GBS. Activation of T finding of serum anticaudate and subthalamic
cells, cytokine synthesis, demyelination of the nuclear antibodies and an increase in IgG levels
peripheral nervous system by antibodies, and in the CSF of patients with chorea. (Goldenberg
axonal damage by antiglycolipid antibodies are et al., 152157; Nausieda et al., 331334)
key features of the pathogenesis of Guillain
Barr syndrome. 57. (C) Prophylactic treatment of epilepsy is indi-
Tick-bite paralysis results from a neuro- cated if there is major motor seizure, if two or
muscular blockage by a tick-produced neuro- more seizures occur in close temporal proxim-
toxin. It can cause paralysis that may mimic ity, or if the seizures are associated with falls.
GBS, but it is unlikely to be considered a first The age and situation of the patient may influ-
diagnosis in a child living in Philadelphia in ence the decision to prescribe antiseizure med-
January. ications. Most authors agree to discontinue
Edrophonium, a cholinesterase inhibitor, anticonvulsant medication if the patient is free
is used for the diagnosis of myasthenia gravis. of major motor seizures or absence seizures for
The type of weakness described in this patient 1 or 2 years with normal EEG and brain MRI
is unlikely to be seen in myasthenia gravis, in studies. In Janz syndrome, a variant of
which ptosis, extraocular muscle paresis, dys- myoclonic epilepsy, antiseizure medications
phagia, and fluctuant weakness are prominent should not be stopped even after a remission as
features. In myasthenia, the EMG may show a long as 3 years, because seizures often recur.
decremental response on repetitive stimula- Seizure medications should be a lifelong treat-
tion. Impairment of pupillary response, paresis ment if the seizures are caused by permanent
of the extraocular muscles, dysphagia, hypo- damage of the brain, as in many cases of cere-
tonia, descending paralysis, and constipation bral palsy or operated arteriovenous malfor-
are classic findings in botulism. The diagnosis mations. (Callaghan, Garrett, and Goggin, 942946;
is made by an EMG showing an incremental Shinnar et al., 534565)
response to repetitive stimulation and by iso-
lation in the stool of Clostridium botulinum or 58. (E) The patient described in this case has the
its toxin. (Bradshaw and Jones, 500506; Ropper, classic signs of Erb palsy, a paralysis of the mus-
11301136) cles innervated by the C5-C6 roots. It occurs in
approximately 0.6% of all vaginal deliveries;
56. (A) Sydenham chorea (chorea minor, St. Vitus shoulder dystocia is typically diagnosed when
dance, rheumatic encephalitis), described by there is impaction of the anterior fetal shoulder
Thomas Sydenham in 1686, is considered the behind the symphysis pubis. The spinal roots
most common cause of chorea in a school-age have approximately one-tenth the tensile
child. Clinically, it is characterized by involun- strength of the peripheral nerves because of
tary movements, hypotonia, dysarthria, emo- lesser amounts of collagen and the absence of
tional disorders, and less frequently by other epineurial and perineurial sheaths in the roots.
neurological manifestations, such as weakness Therefore, the nerve roots are the weak link in
and headache. The motor disorders may be gen- the nerve rootspinal, nerveplexus complex,
eralized or unilateral, in which circumstance and nerve root. Avulsion from the spinal cord
they constitute a hemichorea. Chorea may pres- may result from the severe traction injury
Answers: 5661 131

caused by a shoulder dystocia. Brachial plexus vary as the years pass. While tics are common in
injury complicates approximately 8% to 23% of childhood, they are usually a transient phe-
shoulder dystocia cases. Nearly 80% of brachial nomenon; if persistent, they tend to improve
plexus injuries involve the nerve roots of C5 to during adolescence. Tourette syndrome is the
C6. Clinical findings in Erb palsy may include most severe form of the spectrum of tic disorders
C5 root avulsion signs, which result in virtually and is defined by the presence of both motor
complete paralysis of the rhomboids and spina- and vocal tics for a duration of a year or more,
tus muscles and a varying degree of weakness with onset before the age of 18 years. The vast
of the deltoid, biceps, brachioradialis, and ser- majority of cases are idiopathic. There is a close
ratus anterior, which receive additional inner- relationship between tics and obsessive
vation from C6. More than 90% of these injuries compulsive disorder. The reported prevalence of
will resolve by 1 year of life, with only a 5% to obsessivecompulsive disorder in Tourette syn-
8% rate of persistent nerve injury. Dejerine drome patients is approximately 50%, and there
Klumpke palsy occurs less frequently than Erb is also an increased prevalence of obsessive
palsy and involves the lower trunk of the compulsive disorder in relatives of tic patients.
brachial plexus, C8, and T1 roots. It accounts for (Weeks, Tujanski, and Brooks, 401408)
only 2.5% of brachial plexus injuries. Clinical
findings include weakness in the flexor of the 60. (A) Grade I cerebral bruit is usually a physio-
wrist and fingers, absent grasp reflex, and pos- logical murmur heard in hyperdynamic states
sible unilateral Horner syndrome. C3 to C4 and such as anemia or fever. Cerebral bruit may
T2 to T4 nerve root lesions will not affect the indicate an arteriovenous malformation of the
muscles of the upper extremity. (Pollack et al., vein of Galen or increased intracranial pressure
236246) by a subdural effusion or other causes. Of the
five choices mentioned in this question, cerebral
59. (C) The patient described in this case devel- bruit is the least reliable sign of increased intrac-
oped repetitive sniffing and grunting over a erebral pressure. Setting-sun sign is a down-
period of time, most likely related to verbal tics. ward deviation of the eyes. It is thought to be
Tics are defined as sudden, rapid, recurrent, and caused by compression of the upward gaze
nonrhythmic stereotyped movements or vocal- center in the upper part of the brainstem by the
izations. Motor tics are characterized as simple dilated third ventricle. Setting-sun sign, strabis-
(such as eye blinking, facial grimacing, and head mus, bulging of fontanelle, and splitting of the
turning) or complex (such as jumping, thump- sutures are signs of increased intracranial pres-
ing, echopraxia, and copropraxia). Simple vocal- sure in a 4-month-old patient. (Fenichel, 9194)
izations can consist of grunts, coughing, and
throat clearing, while examples of complex 61. (C) Tourette syndrome arises during childhood
vocalizations include echolalia, word repeti- or early adolescence, usually between the ages of
tions, and coprolalia. Tics are unusual examples 2 and 15 years, with a mean age of 7 years. It
of movement disorders in that the abnormal occurs in boys four times as often as in girls. The
movements are often preceded by a feeling of most common initial symptoms are motor tics
inner tension and a compulsion to move, or in involving the cranial region, especially around
some cases by unpleasant focal sensory symp- the eyes. Subsequently, patients develop a con-
toms, sometimes termed sensory tics. The sub- stellation of motor and vocal tics of either a
sequent performance of the tic temporarily simple or complex nature. Tics occur many times
relieves the sensory symptoms. Some individu- a day, usually daily for at least a year, and there
als claim that their tics are voluntary, in response is never a tic-free period of more than 3 consec-
to dysphoric sensory symptoms. Tics can usu- utive months. Tics are increased during time of
ally be suppressed but only at the expense of stress and disappear during sleep. Patients may
increasing internal tension, which often causes suppress their tics for a period of time, which
a rebound exacerbation of the tic. The type, leads to increased dysphoric sensation. Two
severity, location, and frequency of tics often points about the course of Tourette syndrome
132 3: Pediatrics

are notable. One is the changing display of tics, vignette has three risk factors that increase his
and the other is the tendency toward periodic risk of developing epilepsy in later life from
remissions and exacerbations. A high frequency approximately 4% to 10% or 15%. (Annegers et al.,
of various behavioral abnormalities attends 493498)
Tourette syndrome, and these are often the most
disabling aspects of the clinical picture. Given the 64. (B) The patient described in the vignette devel-
usual age of onset, they may translate into poor oped choreoathetotic movements associated
school performance caused by disruptive activ- with episodes of ballismus and akathisia. The
ity and attentional difficulty; such patients may most common cause of new-onset chorea in that
be relegated to special education. (Behrman, 81; age is Sydenham chorea. It occurs most fre-
Fenichel, 294296) quently in females between the age of 5 and 15
years. Cardiac disease may not be evident at the
62. (D) The patient described in this case has the time of chorea. Careful cardiac examination with
features of fascioscapulohumeral syndrome. echocardiogram and ECG is recommended to
This is an autosomal dominant myopathy that assess the extent of cardiac involvement. The
affects mainly the muscles of the face, shoulder differential diagnosis of Sydenham chorea may
girdle, and upper arms. The gene, localized in include ADHD because restlessness may be con-
chromosome 4q35, has a complete penetrance fused with mild chorea. The careful history
but variable expression. The patient may give a given by the parents, the cardiac findings, and
history of inability to whistle or difficulty using the relatively short-term progression of the dis-
straws or inability to fully close his eyes during ease (a few days to weeks compared with the
sleep. As the extraocular muscles are intact, months-to-years chronic progression of ADHD)
Bells phenomenon remains and consequently suggest Sydenham chorea. Tourette syndrome is
the cornea is protected. The diagnosis is made a combination of motor and verbal tics. These
by the association of insidious muscle weak- latter findings are not seen in Sydenham chorea.
ness involving the shoulder girdle and the face, Wilson disease may present with chorea. The
normal or mildly increased creatine kinase level, association of KayserFleischer rings, low ceru-
and myopathic pattern on EMG. Histological loplasmin level, and liver function abnormalities
examination may show inflammatory signs in suggest the diagnosis of hepatolenticular degen-
addition to the degenerative changes. Retinal eration. Vascular accident is rare in this age. It
telangiectasia, exudation, and detachment are could be considered in the differential diagnosis
the most severe retinal vascular abnormalities in the case of unilateral chorea. An MRI of the
that may be seen by angiography in most brain is helpful to rule out a stroke. (Fenichel,
patients with fascioscapulohumeral syndrome. 285286)
(Fenichel, 336337; Patterson and Gomez, 7382)
65. (E) Prompt recovery after spells is most sug-
63. (B) The patient described in the vignette pre- gestive of absence seizures rather than partial
sented with a febrile seizure. Typically, it is a complex seizure. Urinary incontinence is rare in
generalized type of seizure, occurring between absence seizure and is occasionally present in
the age of 6 months and 3 years, at a tempera- complex partial seizure. The spells last 5 to 10
ture greater than 38C. The risk factors that seconds in absence seizure and up to a few min-
may increase the recurrence of febrile seizures utes in complex partial seizure. Automatism
or the development later in life of epilepsy may complicate absence seizure when pro-
include long duration of seizure (more than 15 longed and is more frequently present in
minutes), a known developmental disorder, a complex partial seizures. An EEG reveals gen-
positive family history of febrile seizure or eralized three-per-second spike-wave dis-
epilepsy, focal seizures, repeated febrile charges in absence seizures, whereas in
seizures within a single illness, and occurrence complex partial seizure it shows (in 60% of
of febrile seizures outside the usual age range cases) variably located focal spikes. (Fenichel,
(6 months to 3 years). The patient in the 2628; Panayiotopoulos, 351355)
Answers: 6270 133

66. (B) Athetosis, a distal slow writhing move- herniation as part of a Chiari II malformation or
ment of the extremities, is usually a manifesta- aqueductal stenosis, which occurs in approxi-
tion of damage to the basal ganglia. Bilirubin mately 80% of cases and leads to a noncom-
encephalopathy selectively involves the globus municating hydrocephalus that may need a
pallidus and subthalamic nuclei and charac- shunt within the first 2 weeks of life.
teristically produces athetotic cerebral palsy. Orthopedic or physical therapy referral may
Perinatal asphyxia has become the most be appropriate with complications such as tal-
frequent cause of athetotic cerebral palsy ipes equinovarus. (Behrman, 19841985)
because kernicterus has become less common
with better prevention and management of 69. (C) The patient described in this vignette has a
hyperbilirubinemia in the newborn period. long, thin face because of wasting of the tempo-
(Fenichel, 811) ral and masseter muscles; there is also hand
intrinsic muscles wasting with muscle relaxation
67. (B) The diagnosis of cerebral palsy is difficult difficulties. These findings are compatible with
to establish before the age of 6 months because the diagnosis of myotonic dystrophy. It is an
abnormalities in tone, reflexes, or involuntary autosomal dominant multisystem disorder with
movements rarely manifest during the new- variable penetrance. The disease is caused by
born period. The primary reason for this is that amplification of an unstable DNA region in
most of the movements observed in newborns chromosome 19. In addition to the striated
are of reflex origin and not under voluntary muscle, which is primarily involved in this dis-
control. The maturation of the cortex allows ease, smooth muscle of the digestive tract, car-
the clinical picture of cerebral palsy to emerge diac muscle, the endocrine system, the immune
clearly. Also, tone and reflex abnormalities system, vision (cataract), and intelligence may be
occurring after a perinatal insult to the brain affected. Myotonia can be demonstrated by per-
may falsely suggest permanent damage to the cussion of the thenar eminence, which remains
central nervous system, since they may dimpled at the site of the percussion with thumb
improve after 2 to 12 months. The presence of abduction for several seconds. In this vignette,
progressive neurological deficit excludes the the myotonia is demonstrated by the use of wrist
diagnosis of cerebral palsy because the latter flexors to force the flexors of the fingers to open.
condition is defined as an abnormal control of In Duchenne muscular dystrophy, muscle deficit
movement and posture that begins early in life in dystrophin causes an unsteady gait in males
and is not the result of an underlying progres- always before the age of 5 years. Myasthenia
sive condition. Clues to a progressive disorder gravis, WerdnigHoffman disease, and chronic
may include mental or motor regression, neu- demyelinating polyradiculopathy are unlikely
rocutaneous signs, and skeletal anomalies. A to cause myotonia. (Fenichel, 190191)
low Apgar score at 5 minutes does not correlate
with high risk of developing cerebral palsy or 70. (C) Pseudotumor cerebri is a chronic condi-
any other neurological disease. (Fenichel, tion characterized by an increase in intracranial
269270; Taft, 411418) pressure, normal CSF content, and normal
brain with normal or small ventricles on brain
68. (B) Myelomeningocele is a congenital defect imaging studies. A specific cause can usually be
of spinal cord closure. It is most commonly found in children younger than 6 years, while
located in the lumbar region and may contain most idiopathic cases occur after the age of
spinal cord or nerve roots involved in the inner- 12 years. Administration of tetracycline or
vation of the urinary bladder. To avoid uro- nalidixic acid has been postulated as a cause of
logic problems and ensure timely and pseudotumor cerebri. Hypervitaminosis A as
appropriate intervention, the genitourinary well as hypovitaminosis A has been docu-
system must be evaluated in all infants who mented as a cause of pseudotumor cerebri, but
have this malformation. Myelomeningocele not furosemide, acetazolamide, phenobarbital,
may be accompanied by cerebellar tonsillar or ampicillin administration. (Fenichel, 113114)
134 3: Pediatrics

71. (E) The most sensitive and one of the earliest the ventricles. The most likely diagnosis in this
signs of critically increased intracranial pressure case is ventriculitis, a common complication in
is a decreased level of consciousness. Headache, gram-negative meningitis in newborns.
vomiting, irritability, and abducen nerve palsy Necrotic bits of choroid plexus can block the
may develop sequentially. Headache is caused cerebral aqueduct, making the lateral cerebral
by traction on the intracranial arteries. Pain ventricles a closed space. The diagnosis is con-
fibers from supratentorial intracranial vessels firmed by a neurosurgical ventricular puncture
are innervated by the trigeminal nerve and pain that might yield the infecting organism.
is referred to the eyes, forehead, and temple. Subdural empyema usually occurs in an infant
Infratentorial vessels are innervated by cervical with a severe gram-negative meningitis. Head
nerves and pain is referred to the occiput and CT with contrast will reveal the subdural col-
neck. Papilledema is a passive swelling of the lection of fluid surrounded by an enhancing
optic disk, probably caused by the obstruction of rim. Brain abscess is an uncommon complica-
venous return from the retina and nerve head. It tion of bacterial meningitis in the newborn. It
does not develop in all patients with acute may occur as a complication of severe
increased intracranial pressure. Early papilledema Haemophilus influenzae infant meningitis.
is asymptomatic; only when it is advanced or Surgical drainage is indicated if the abscess is
chronic does the patient experience a transitory large and accessible. (Smith, 1118)
decrease in vision. This preservation of visual
acuity may differentiate papilledema from a pri- 74. (A) The frequency and duration of headaches
mary optic nerve disturbance such as optic neu- described in this vignette suggest the diagnosis
ritis. (Fenichel, 9194) of chronic tension-type headaches. A pressure
or tightening quality that is dull and nonpul-
72. (A) The patient described in the vignette satile is typical. Depression is a common
showed the emergence of a left sided focal comorbidity of this condition and should be
seizure during the course of his meningitis, assessed appropriately. The frequency of the
followed by the persistence of left sided weak- headaches in this case warrants prophylactic
ness despite the improvement of his meningi- treatment. Amitriptyline has been the most
tis. A cerebral ischemic event is the most likely successful medication with patients who have
cause. It complicates the course of bacterial tension-type headaches. Sedation that may
meningitis in 2% to 19% of cases and may be interfere with daytime activity and anticholin-
due to focal brain ischemia from venous throm- ergic side effects can be avoided by adminis-
bosis or arterial vasculitis. Brain abscess is an trating the drug at bedtime and using low
uncommon complication of bacterial meningi- dosages. (Fenichel, 85)
tis in children and unlikely to be the right diag-
nosis in this case, where all signs of infection 75. (A) Familial hemiplegic migraine is character-
improved. Transient paresis may complicate a ized by a sudden onset of hemiplegia or
focal seizure. However, such a neurological hemisensory loss that is usually followed by a
deficit classically improves within a few hours. contralateral headache. The trait is transmit-
Persistent motor deficit suggests an underlying ted by autosomal dominant inheritance. The
structural lesion rather than Todd paralysis. gene is located on chromosome 19p. Attacks
Subdural empyema and hydrocephalus with are stereotyped, occur primarily in childhood
increased intracranial pressure may complicate or adolescence, and may be precipitated by
meningitis. These entities are expected to cause minor head trauma. The hemiplegia, although
generalized rather than focal seizures. (Fenichel, more severe in the face and arm, affects the leg
107109) and may be present on alternate sides during
different episodes. Aphasia may occur when
73. (C) The patient in the vignette has a clinical the dominant hemisphere is affected. Stupor,
worsening of his symptoms, with ependymal confusion, and psychosis may complicate
rim enhancing and fluid of different contrast in attacks. The episode may last 2 or 3 days and
Answers: 7179 135

may suggest a stroke-like syndrome. The neu- dren who experience a second seizure will expe-
rological deficit usually resolves completely, rience a third one within the next 6 to 12 months.
but permanent sequelae, such as gaze-evoked (Berg et al., 11221127; Fenichel, 1718; Verity and
nystagmus, may persist between attacks. Golding, 13731376)
(Fenichel, 251)
78. (C) LaCrosse virus is the most common cause
76. (A) Abdominal migraine refers to the condi- of encephalitis due to California subgroup
tion of children who have recurrent abdominal viruses in the United States. Most cases were
pain, nausea, and vomiting as well as recurrent reported in Wisconsin and Minnesota before
headaches. This problem ceases by the teenage 1984. Later epidemics occurred in Indiana.
years and is often replaced by more conven- Small woodland mammals serve as a reservoir
tional headaches. Benign paroxysmal vertigo is and mosquitoes as the vector. Eastern equine
seen predominantly in children between 2 and encephalitis is a perennial infection of horses
6 years of age. Such episodes, which last only from New York to Florida. Human cases follow
minutes, are characterized by the sudden onset epidemics in horses. Wild birds serve as a reser-
of vertigo, pallor, and nystagmus. There is a voir and mosquitoes as a vector. Japanese B
positive family history of migraine. Most encephalitis is a major form of encephalitis in
patients will develop a typical migraine by ado- Asia and is an important health hazard to non-
lescence. Recurrent episodes of head tilt asso- immunized travelers during summer months.
ciated with headache, nausea, and vomiting St. Louis encephalitis is endemic in the western
are characteristic of paroxysmal torticollis, an United States. The vector is a mosquito and
uncommon benign disorder. Ocular migraine birds are the major reservoir. Western equine
consists of episodes of transient monocular encephalitis is a rare disorder. All recent cases
blindness in a patient who has previously had have been reported in North Dakota, South
migraine attacks or has a history of migraine in Dakota, and Canada. (Fenichel, 5658)
the family. Most attacks last for only minutes,
but permanent ocular changes have been 79. (C) The patient described in the vignette devel-
reported. No choreoathetotic movement disor- oped complex febrile seizure complicated by
der, sleepwalking, recurrent chest pain, or right-sided hemiparesis and periodic lateralizing
recurrent urinary retention is attributed to epileptiform discharge. These findings are con-
migraine variant. (Singer, 94101) sistent with the diagnosis of herpes encephalitis.
Herpes simplex is the single most common cause
77. (D) In a child with febrile seizure, the risk of of nonepidemic encephalitis and accounts for
developing subsequent epilepsy increases if 10% to 20% of cases. The annual incidence is esti-
there is a family history of epilepsy or a history mated at 2.3 cases per million people. Thirty-
of febrile seizures in parents or siblings. A history one percent of cases occur in children. Primary
of complex febrile seizure, which is defined as a infection is the most common cause of encephali-
seizure that lasts longer than 15 minutes or that tis in children, and only 22% of patients give a
occurs in a prolonged series for more than 30 history of recurrent labial herpes infection. The
minutes, may increase the risk of having subse- diagnosis is made by the examination of CSF,
quent epilepsy. A preexisting neurological dis- which may show a pleocytosis in 97% of cases.
ease such as cerebral palsy or developmental Many red blood cells, up to 500/mm3, may be
delay may also increase the risk of epilepsy. present, with a median protein concentration of
Children who have one of the above risk factors 80 mg/dL. The demonstration of lateralizing
have a 2% chance of developing epilepsy by the epileptiform discharges on EEG is considered
age of 7 years. Those who have two or more risk presumptive evidence of herpes encephalitis.
factors have a 10% chance. Having two febrile However, MRI has proved to be an early indica-
seizures in a single year does not increase the tor of herpes encephalitis; the T2-weighted
risk of developing epilepsy but increases the risk images show increased signal intensity in one
of having a third febrile seizure, as 50% of chil- or both temporal lobes. The identification of the
136 3: Pediatrics

organism in the CSF has been made possible by nial pressure by improving jugular venous
a polymerase chain reaction, which obviates the drainage. Mannitol and glycerol are the most
need for brain biopsy to confirm the diagnosis. widely used osmotic diuretics in the United
Measles encephalitis is a rare complication of States. These agents remain in the plasma and
measles, since compulsory immunization has create an osmotic gradient that draws water
almost eliminated natural measles infection in from the brain into the capillaries. Hypothermia
the United States. Symptoms of encephalitis are between 27C and 31C reduces cerebral blood
usually abrupt, following the rash by 1 day to 3 flow. It is frequently used with pentobarbital,
weeks, and are characterized by lethargy, which which also decreases cerebral flow and edema
may rapidly progress to coma; generalized formation at a dosage causing burst suppres-
seizure occurs in 50% of patients. Hemiplegia, sion on EEG. (Fenichel, 9697)
ataxia, involuntary movement disorders, and
acute transverse myelitis may occur. 81. (A) The patient described in this question has
Postinfectious encephalomyelitis is unlikely to Parinaud syndrome, which results from dys-
be the diagnosis in this case. The clinical picture function of the midbrain due to periaqueductal
would be lethargy and weakness followed by compression from a pineal region tumor.
declining consciousness, seizures, optic neuritis, Tumors of the pineal region are most frequently
and/or transverse myelitis. Postinfectious derived from germ cells. They are more fre-
encephalomyelitis is a demyelinating disorder quent in boys than girls and generally become
that occurs during or after a systemic viral illness symptomatic during the second decade.
and is presumed to be an immune-mediated dis- Symptoms of pineal tumors are caused either
ease. The diagnosis is based on a T2-weighted by tumor mass effect on local tissues or by
MRI scan that shows a marked increase in signal hydrocephalus (from the tumor blocking the
intensity throughout the white matter. Reye syn- normal CSF drain pathway). Head MRI is a
drome is a systemic disorder of mitochondrial valuable tool in assessing the location and
function that occurs during or following a viral extension of pineal tumors; in some cases it
infection and/or after the use of salicylate. The may suggest the tumors histological type.
clinical picture may progress from vomiting and Germinomas are isodense and have irregular
lethargy to flaccid coma. Typical blood abnor- margins. Teratomas may appear lobulated and
malities include hypoglycemia, hyperammone- have both hyperdense and multicystic areas.
mia, and increase in hepatic enzymes. CSF is Tumors that spread into the ventricles and
normal except for increased pressure. EEG show intense contrast enhancement are likely
shows diffuse encephalopathy. The diagnosis is to be malignant. Tumors with abundant amounts
confirmed by liver biopsy, which may, on elec- of calcium are likely to be benign. Pineal ger-
tron microscopy, show characteristic mitochon- minomas are more radiosensitive than other
drial abnormalities. St. Louis encephalitis may pineal tumors, with a 5-year survival rate up to
present with headache, fever, and a spectrum of 80%. (Fenichel, 101)
neurological illness that varies from aseptic
meningitis to severe encephalitis. Decreased con- 82. (C) The patient described in this question has
sciousness is common, but seizures or focal neu- the features of fragile-X syndrome. It is the
rological disturbances are rare. (Fenichel, 5661) most common chromosomal cause of mental
retardation in boys. The fragile-X gene (FMR-
80. (D) Hyperventilation immediately reduces the 1) has been isolated, cloned, and characterized.
intracranial pressure through vasoconstriction, It contains a trinucleotide sequence (CGG) that
induced by lowering the arterial pressure of in the normal genome is repeated from 6 to 55
carbon dioxide. However, excessive lowering times. In persons with the fragile-X syndrome,
of carbon dioxide below 25 mmHg is con- this repeat is expanded (amplified) to several
traindicated because it may cause brain hundred copies (full mutation), whereas
ischemia. Elevation of the head of the bed 30 to asymptomatic carriers for fragile X carry
45 degrees above horizontal decreases intracra- between 50 and 230 copies (premutation). The
Answers: 8084 137

premutation tends to remain stable during is 1 in 600 to 700 births, and this condition
spermatogenesis but frequently expands to a accounts for approximately 10% of all cases in
full mutation during oogenesis. All male indi- every large series of cases of severe mental retar-
viduals with a full mutation but only 53% of dation. Familiarity with the condition permits
female individuals with a full mutation are its recognition at birth, but it becomes more
mentally impaired. The condition is relatively obvious with advancing age. The round head,
common, second only to Downs syndrome as open mouth, stubby hands, slanting palpebral
a genetic cause of mental retardation. It is a fissures, and short stature impart an unmistak-
clinically subtle dysmorphic syndrome. The able appearance. The ears are low-set and oval,
male patient has a long face, prominent brow, with small lobules. The palpebral fissures slant
somewhat square chin, large floppy ears, slightly upward and outward owing to the
and macroorchidism without any obvious evi- presence of medial epicanthal folds that partly
dence of endocrine dysfunction. Although cover the inner canthi (hence the old term mon-
macroorchidism can be present at birth, it is golism). The bridge of the nose is poorly devel-
difficult to recognize in the prepubertal boy, as oped and the face is flattened (hypoplasia of
are most of the other physical features. the maxillae). The tongue is usually enlarged,
Approximately 10% of patients have a head heavily fissured, and protruded. Graywhite
circumference greater than the 97th percentile, specks of depigmentation are seen in the irides
and the fragile-X syndrome may mimic the fea- (Brushfield spots). The little fingers are often
tures of cerebral gigantism. A number of clini- short (hypoplastic middle phalanx) and incurved
cal features reflect connective tissue dysplasia. (clinodactyly). The fontanels are patent and slow
These include hyperextensible finger joints, flat to close. The hands are broad, with a single
feet, aortic root dilatation, and mitral valve pro- transverse (simian) palmar crease and other char-
lapse. A PraderWilli phenotype has also been acteristic dermal markings. Lenticular opacities
encountered. The neurological picture is high- and congenital heart lesions (septal and other
lighted by retarded language development and defects) as well as gastrointestinal abnormalities
hyperactivity. Delayed motor development is (stenosis of duodenum) are frequent. Hypotonia
seen in some 20% of male patients, and seizures of limbs is a prominent finding. At first, the Moro
have been experienced by 25% to 40%. These response is reduced or absent, and feeding is dif-
are major motor or partial complex seizures; as ficult. Most affected children do not walk until 3
a rule they respond well to anticonvulsant ther- to 4 years of age; their acquisition of speech is
apy. Because folic acid antagonists must be delayed, but over 90% talk by 5 years. The intel-
added to the culture medium (to create a folate- ligence quotient (IQ) is variable; that of a large
free culture) to detect the abnormality, high group follows a Gaussian curve with the median
doses of folate have been used to treat children IQ being 40 to 50 and the range 20 to 70. (Ropper
with fragile-X syndrome, with some behavioral and Brown, chapter 38)
improvement. (Fenichel, 120; Menkes, 244247)
84. (D) The patient described in this question has
83. (B) Hypotonia, mongoloid facies, flat nape of the features of trisomy 18. The disorder is seen in
neck, and Brushfield spots are features of tri- 1 in 4,000 live births, more in females, with an
somy 21. Downs syndrome is the most average maternal age of 34 years. It is character-
common chromosomal abnormality affecting ized by slow growth, occasional seizures, severe
live-born children. It results from the presence mental retardation, hypertonia, ptosis and lid
of three copies of chromosome 21 rather than abnormalities, low-set ears, small mouth, mot-
the normal two copies. Affected individuals tled skin, clenched fists with index fingers over-
most often exhibit mild to moderate mental lapping the third finger, syndactyly, rocker-
retardation and have characteristic facial and bottom feet, shortened big toe, ventricular septal
physical abnormalities as well as possible con- defect, umbilical and inguinal hernias, short ster-
genital defects in the cardiac, visual, gastroin- num, small pelvis, and small mandible. (Ropper
testinal, and endocrine systems. The frequency and Brown, chapter 38)
138 3: Pediatrics

85. (A) Cri-du-chat syndrome is caused by a dele- gangliosides. As a result of this deficiency, GM2
tion in short arm of chromosome 5. It is char- ganglioside accumulates in the cerebral cortical
acterized by an abnormal cry, like that of a neurons, Purkinje cells, retinal ganglion cells,
kitten, severe mental retardation, hyper- and, to a lesser extent, larger neurons of the
telorism, epicanthal folds, brachycephaly, brainstem and spinal cord. (Ropper and Brown,
moon face, antimongoloid slant of palpebral chapter 37)
fissures, micrognathia, hypotonia, and strabis-
mus. (Ropper and Brown, chapter 38) 88. (A) GM1 gangliosidosis is an autosomal reces-
sive lysosomal storage disorder characterized by
86. (D) Hutchinson triaddefined as the combi- the generalized accumulation of GM1 ganglio-
nation of deafness, interstitial keratitis, and side, oligosaccharides, and the mucopolysac-
peg-shaped upper incisorsis characteristic of charide keratan sulfate (and their derivatives).
congenital syphilis. The more common features The amount and type of residual activity deter-
in symptomatic infants with congenital syphilis mine whether the phenotype is generalized gan-
are condylomata lata, periostitis or osteochon- gliosidosis, as in GM1 gangliosidosis (deficiency
dritis, persistent rhinorrhea, and maculopapu- of the lysosomal hydrolase acid b-galactosidase
lar rash. (Fenichel, 122) causes GM1 gangliosidosis) or visceral storage
of mucopolysaccharidosis with little brain dis-
87. (B) TaySachs disease is an autosomal reces- ease, as in Morquio disease type B. Three clini-
sive disease, mostly of Jewish infants of eastern cal subtypes of GM1 gangliosidosis exist,
European (Ashkenazic) background. The dis- classified by age of onset as infantile form, juve-
ease becomes apparent in the first weeks and nile form, and adult form. In the infantile form,
months of life, almost always by the fourth the infants appear abnormal at birth. They have
month. The first manifestations are a regres- dysmorphic facial features, like those of indi-
sion of motor activity and an abnormal startle viduals with the mucopolysaccharidoses:
to acoustic stimuli, accompanied by listless- depressed and wide nasal bridge, frontal boss-
ness, irritability, and poor reactions to visual ing, hypertelorism, puffy eyelids, long upper
stimuli. These are followed by a progressive lip, gingival and alveolar hypertrophy,
delay in psychomotor development or regres- macroglossia, and low-set ears. These features,
sion (by 4 to 6 months), with inability to roll with the bone changes mentioned below,
over and sit. At first, axial hypotonia is promi- account for the term pseudo-Hurler. Other indi-
nent, but later spasticity and other corti- cations of the disease are the onset of impaired
cospinal tract signs and visual failure become awareness and reduced responsivity in the first
evident. Degeneration of the macular cells days or weeks of life; lack of psychomotor devel-
exposes the underlying red vascular choroid opment after 3 to 6 months; hypotonia and later
surrounded by a whitish gray ring of retinal hypertonia with lively tendon reflexes and
cells distended with ganglioside. The resulting Babinski signs. Seizures are frequent. The head
appearance is of the cherry-red spot with optic size is variable (microcephaly more often than
atrophy. In the second year, there are tonic macrocephaly). Loss of vision, coarse nystag-
clonic or minor motor seizures and an increas- mus and strabismus, macular cherry-red spots
ing size of the head and diastasis of sutures with (in half the cases), flexion pseudocontractures of
relatively normal-sized ventricles; in the third elbows and knees, kyphoscoliosis, and enlarged
year, the clinical picture is one of dementia, liver and sometimes enlarged spleen are the
decerebration, and blindness. Cachexia becomes other important clinical findings. Radiographic
increasingly severe and death occurs at 2 to abnormalities include subperiosteal bone for-
4 years. The EEG becomes abnormal in the early mation, midshaft widening and demineraliza-
stages (paroxysmal slow waves with multiple tion of long bones, and hypoplasia and beaking
spikes). The basic enzymatic abnormality is a of the thoracolumbar vertebrae. Vacuoles are
deficiency of hexosaminidase A, which nor- seen in 10% to 80% of blood lymphocytes and
mally cleaves the N-acetylgalactosamine from foam cells in the urinary sediment. The juvenile
Answers: 8591 139

subtype is marked by a slightly later age of onset spasticity) in combination with reduced output
and clinical variability in the classic physical of speech and mental regression. At first, the
features. The adult subtype is marked by normal tendon reflexes are usually brisk, but later, as
early neurological development with no physi- the peripheral nerves become more involved,
cal stigmata and subsequent development of a the tendon reflexes are decreased and eventu-
slowly progressive dementia with parkinsonian ally lost. Or, there may be variable hypotonia
features, extrapyramidal disease, and dystonia. and areflexia from the beginning, or spasticity
(Fenichel 129130; Ropper and Brown, chapter 37) may be present throughout the illness, but with
hyporeflexia and slowed conduction velocities.
89. (E) Infantile NiemannPick disease is an auto- Signs of mental regression may be apparent
somal recessive disease. Two thirds of the from the onset or appear after the motor disor-
affected infants have been of Ashkenazi Jewish der has become established. Later, there is
parentage. The onset of symptoms in the usual impairment of vision, sometimes with squint
type A disease is between 3 and 9 months of and nystagmus; intention tremor in the arms,
age, frequently beginning with marked enlarge- and dysarthria; dysphagia and drooling; and
ment of liver, spleen, and lymph nodes and optic atrophy (one third of patients), sometimes
infiltration of the lungs; rarely there is jaundice with grayish degeneration around the maculae.
and ascites. Cerebral abnormalities are definite Seizures are rare, and there are no somatic
by the end of the first year, often earlier. The abnormalities. The head size is usually normal,
usual manifestations are loss of spontaneous but rarely there is macrocephaly. Progression to
movements, lack of interest in the environment, a bedridden quadriplegic state without speech
axial hypotonia with bilateral corticospinal or comprehension occurs over a 1- to 3-year
signs, blindness and amaurotic nystagmus, and period, somewhat more slowly in late-onset
a macular cherry-red spot (in about one quarter types. The CSF protein is elevated. There is
of the patients). Seizures may occur but are rel- widespread degeneration of myelinated fibers
atively late. There is no acoustic-induced startle in the cerebrum, cerebellum, spinal cord, and
or myoclonus, and head size is normal or peripheral nerves. The presence of metachro-
slightly reduced. Loss of tendon reflexes and matic granules in glial cells and engorged
slowed conduction in peripheral nerves have macrophages is characteristic and enables the
been recorded but are rare. Protuberant eyes, diagnosis to be made from a biopsy of a periph-
mild hypertelorism, slight yellowish pigmen- eral nerve. (Fenichel, 132; Ropper and Brown, chapter 37)
tation of oral mucosa, and dysplasia of dental
enamel have also been reported but are rare. 91. (C) Krabbe disease is a rapidly progressive
Most patients succumb to an intercurrent infec- demyelinating disorder of infants caused by a
tion by the end of the second year. (Fenichel, 132; deficit in the enzyme galactocerebrosidase. The
Ropper and Brown, chapter 37) enzyme normally degrades galactocerebroside
to ceramide and galactose. The deficiency results
90. (D) Metachromatic leukodystrophy is a lyso- in the accumulation of galactocerebroside; a
somal (sphingolipid) storage disease. The basic toxic metabolite, psychosine, leads to the early
abnormality, localized in chromosome 22, is destruction of oligodendrocytes and depletion
the absence of the gene for enzyme arylsulfa- of lipids in the cerebral white matter. The onset
tase A, a deficiency of which prevents the con- is usually before the sixth month and often
version of sulfatide to cerebroside (a major before the third month (10% after 1 year). Early
component of myelin) and results in an accu- manifestations are generalized rigidity, loss of
mulation of the former. The disease is trans- head control, diminished alertness, frequent
mitted as an autosomal recessive trait and vomiting, irritability and bouts of inexplicable
usually becomes manifest between the first and crying, and spasms induced by stimulation.
fourth years of life. The disease in the infantile With increasing muscular tone, opisthotonic
form is characterized clinically by progressive recurvation of the neck and trunk develops.
impairment of motor function (gait disorder, Later signs are adduction and extension of the
140 3: Pediatrics

legs, flexion of the arms, clenching of the fists, tance. The onset of neurological difficulty in
hyperactive tendon reflexes, and Babinski signs. more than half of these patients is in the first
Later still, the tendon reflexes are depressed or year of life, mostly before the sixth month; but
lost but Babinski signs remain, an indication late-onset forms, with great heterogeneity of
that neuropathy is added to corticospinal presentation as late as early adulthood, are also
damage. This finding, shared with some of the known. Neurological symptoms often appear
other leukodystrophies, is of diagnostic value. subacutely or abruptly, sometimes precipitated
Blindness and optic atrophy supervene. by a febrile illness or a surgical operation. In
Convulsions occur but are rare and difficult to infants, loss of head control and other recent
distinguish from tonic spasms. Myoclonus in motor acquisitions, hypotonia, poor sucking,
response to auditory stimuli is present in some anorexia and vomiting, irritability and contin-
cases. The head size is normal or, rarely, slightly uous crying, generalized seizures, and
increased. In the last stage of the disease, which myoclonic jerks constitute the usual clinical pic-
may occur from one to several months after the ture. If the onset is in the second year, there is
onset, the child is blind and usually deaf, delay in walking, ataxia, dysarthria, psychomo-
opisthotonic, irritable, and cachectic. Most tor regression, tonic spasms, characteristic res-
patients die by the end of the first year; survival piratory disturbance (episodic hyperventilation,
beyond 2 years is unusual, although a consider- especially during infections, and periods of
able number of cases of later onset have been apnea, gasping, and quiet sobbing), external
reported. (Fenichel 132; Ropper and Brown, chapter 37) ophthalmoplegia, nystagmus, and disorders of
gaze (like those of Wernicke disease), paralysis
92. (C) Leigh disease is a syndrome of progres- of deglutition, and abnormal movements of the
sive dystrophy primarily affecting neurons of limbs (particularly dystonia but also jerky and
the brainstem, thalamus, basal ganglia, and choreiform movements). Mild cases, showing
cerebellum. The disease is transmitted by an mainly developmental delay, have been mis-
autosomal recessive or X-linked inheritance. taken for cerebral palsy. Peripheral nerves are
The disease may be caused by an enzyme involved in some cases (areflexia, weakness,
deficit, either in pyruvate metabolism or in res- atrophy, and slowed conduction velocities of
piratory chain complexes. Onset of the disease peripheral nerves); in a few, autonomic failure
occurs in 60% of cases in the first year. The ini- is the most prominent feature. In some chil-
tial symptoms are developmental delay, fail- dren, the disease is episodic; in others, it is
ure to thrive, hypotonia, and seizures. intermittently progressive and quite protracted,
Intercurrent infection or a heavy carbohydrate with exacerbation of neurological symptoms
meal may worsen symptoms. During infancy in association with nonspecific infections.
the patient may show three typical features: (Fenichel 134; Ropper and Brown, chapter 37).
respiratory disturbance, hypotonia, and ocular
motility abnormalities. Hypotonia results from 94. (C) Gaucher disease is a lipid storage disease
a combination of peripheral neuropathy and characterized by the deposition of glucocere-
disturbed cerebellar function. Ocular motility broside in cells of the macrophagemonocyte
disturbance varies from nystagmus to oph- system. The disorder results from the defi-
thalmoplegia. Respiratory disturbance can be ciency of a specific lysosomal hydrolase, glu-
characterized by CheyneStokes breathing, cocerebrosidase (also termed acid beta-
ataxic breathing, or central hyperventilation. glucosidase, glucosylceramidase). The disease
(Fenichel, 134) is characterized by a continuum of phenotypes.
The severity is extremely variable; some
93. (D) Subacute necrotizing encephalomyelopa- patients present in childhood with virtually all
thy (Leigh disease) is a familial or sporadically the complications of Gaucher disease, while
occurring mitochondrial disorder with a wide others remain asymptomatic into the eighth
range of clinical manifestations. Only some of decade of life. Gaucher disease has traditionally
the cases display a maternal pattern of inheri- been divided into the following three clinical
Answers: 9298 141

subtypes, delineated by the absence or pres- transporting ATPase, ATP7A, that is currently
ence of neurologic involvement and its pro- thought to result in a failure of absorption of
gression: type I, nonneuronopathic form; type copper from the gastrointestinal tract and a
II, acute neuronopathic form; and type III, profound deficiency of tissue copper. Poor
chronic neuronopathic form. feeding and failure to gain weight, instability of
Glucosylceramide, the accumulated glycol- temperature (mainly hypothermia), and
ipid, is primarily derived from the phagocyto- seizures become apparent in early infancy.
sis and degradation of senescent leukocytes (Fenichel, 137138; Ropper and Brown, chapter 37)
and, to a lesser extent, from erythrocyte mem-
branes. The glycolipid storage gives rise to the 96. (E) ChediakHigashi syndrome is an autoso-
characteristic Gaucher cells, macrophages mal recessive disorder resulting from lack of
engorged with lipid with a crumpled-tissue- regulation of the fusion of primary lympho-
paper appearance and displaced nuclei. The cytes. Recurrent infection during infancy, skin
factors that contribute to neurological involve- and hair pigmentation defects, peripheral neu-
ment in patients with types II and III disease ropathy, seizures, and developmental retarda-
are still unknown but may be related to the tion may be seen in affected patients. (Rowland,
accumulation of a cytotoxic glycolipid, gluco- 10611062)
sylsphingosine, in the brain due to the severe
deficiency of glucocerebrosidase activity. 97. (B) NiemannPick syndrome is a group of dis-
In type II Gaucher disease, the clinical man- orders grouped together on the basis of the
ifestations are characterized by a rapid neu- overlapping pathology and biochemistry into
rodegenerative course with extensive visceral four groups. Groups A and B are primary
involvement and death within the first 2 years sphingomyelinase deficiencies, whereas groups
of life. Patients with this type may present at birth C and D are allelic disorders, whose primary
or during infancy with increased tone, seizures, defect is not deficiency of a lysosomal hydrolase,
strabismus, and organomegaly. Disruption of the but is in intracellular lipid trafficking. Patients
epidermal layers of the skin, observed on skin with types A and B NiemannPick disease have
biopsy findings, may manifest before the onset deficient activity of the sphingomyelin-cleaving
of neurological symptoms, but this may not enzyme acid sphingomyelinase. (Rowland, 629)
always be clinically apparent. There is a failure
to thrive and stridor due to laryngospasm is 98. (B) The association of skin lesions and neuro-
typical in individuals with type 2 disease. The logical symptoms suggests a neurocutaneous
progressive psychomotor degeneration leads disorder. The association of developmental
to death, usually caused by respiratory com- delay, seizures, and hypopigmented area is
promise. highly suggestive of tuberous sclerosis. The
The clinical manifestation of type III Gaucher disease is transmitted by an autosomal domi-
disease varies widely; it can present in infancy or nant inheritance with a genetic linkage to chro-
childhood. In addition to organomegaly and mosome 9 and 16. The seizures and mental
bony involvement, individuals with type III dis- retardation are caused by disturbed histogene-
ease have neurological involvement. Slowing of sis in the brain. Leaf-shaped hypochromic nevi
the horizontal saccades, an oculomotor finding, are present in only about 18% of patients.
is often the sole neurological manifestation. Some Besides the skin, other organs may be affected,
patients develop myoclonic epilepsy, exhibit such as the retina, kidney, bone, and lungs.
learning disabilities, or develop dementia. Neurofibromatosis type I is unlikely. It is char-
(Fenichel, 128129; Ropper and Brown, chapter 37) acterized by the presence of at least two of the
following: six or more caf au lait spots greater
95. (A) Menkes disease is a rare disorder, inherited than 5 mm (in prepubertal age), two or more
as a sex-linked recessive trait. The manifesta- neurofibromas, freckling in the axillary or
tions of this disease are attributable to one of inguinal region, optic glioma, two or more iris
numerous known mutations in a copper- hamartomas, bone dyplasia, and a first-degree
142 3: Pediatrics

relative with neurofibromatosis type I. The parenchymal cells of the kidney, liver, and
patient in the vignette does not correspond to brain. High concentrations of intracellular
these criteria. (Fenichel, 135136) galactose-1-phosphate can function as a com-
petitive inhibitor of phosphoglucomutase. This
99. (D) Bilateral acoustic neurinomas are a char- inhibition transiently impairs the conversion
acteristic of type II neurofibromatosis. The of glycogen to glucose and produces hypo-
other conditions listed in the question are char- glycemia. Injury to parenchymal cells may
acteristic of type I neurofibromatosis. Other begin prenatally in the affected fetus by
features of type I neurofibromatosis include a transplacental galactose derived from the diet
first-degree relative with this diagnosis, neu- of the heterozygous mother or by endogenous
rofibroma, and freckling in the inguinal or axil- production of galactose in the fetus.
lary region. (Fenichel, 134135) The diagnosis of galactose-1-phosphate
uridyl transferase deficiency should be con-
100. (B) Rett syndrome is an X-linked dominant sidered in newborns with any of the following
disorder affecting girls almost exclusively (it features: jaundice, hepatomegaly, vomiting,
is lethal to the male fetus). It has a prevalence hypoglycemia, convulsions, lethargy, irritabil-
of 1 to 2 per 20,000. Development proceeds nor- ity, feeding difficulties, aminoaciduria,
mally until approximately 1 year of age, at cataracts, or vitreous hemorrhage. The pre-
which time language and motor development liminary diagnosis of galactosemia is made by
regress and acquired microcephaly becomes demonstrating a reducing substance in several
apparent. These girls present with midline urine specimens collected while the patient is
hand-wringing and unusual sighing. Autistic receiving human milk, cows milk, or another
behaviors are typical. Postmortem examina- formula containing lactose. The deficient activ-
tions have revealed greatly reduced brain size ity of galactose-1-phosphate uridyl transferase
and weight as well as a reduced number of is demonstrable in hemolysates of erythro-
synapses. The Rett syndrome Diagnosis cytes, which also exhibit increased concentra-
Criteria Work Group has divided the diagnos- tions of galactose-1-phosphate.
tic criteria into necessary criteria, supportive Krabbe disease is a rapidly progressive
criteria, and exclusion criteria. Severe progres- demyelinating disorder caused by a deficit of
sive dementia is a part of the necessary criteria. the activity of galactosylceramidase. Diffuse
Exclusionary criteria include intrauterine demyelination suggests the diagnosis on MRI;
growth retardation, microcephaly at birth, prolonged motor nerve conduction velocity
organomegaly or sign of storage disease, and increased cerebrospinal fluid protein con-
retinopathy, optic atrophy, and evidence of tent are supportive. The diagnosis is confirmed
acquired identifiable neurological disease. by showing decreased galactosylceramidase
(Behrman, 2034; Fenichel, 13) activity in leukocytes.
Gaucher disease is a multisystemic lipido-
101. (E) The association of vomiting, hepatomegaly, sis characterized by hematological problems,
cataract, and the presence of reducing sub- organomegaly, and skeletal involvement. It is
stances in the urine, especially after feeding, is the most common lysosomal storage disease
highly suggestive of galactosemia. It is a seri- and the most prevalent genetic defect among
ous disease with an early onset of symptoms. Ashkenazi Jews. There are three clinical sub-
Its incidence is around 1 per 60,000. It results types, delineated by the absence or presence
from galactose-1-phosphate uridyl transferase and progression of neurological manifestations:
deficiency. The newborn infant normally type 1 or the adult, nonneuronopathic form;
receives up to 20% of caloric intake as lactose, type 2, the infantile or acute neuronopathic
which consists of glucose and galactose. form; and type 3, the juvenile form. All sub-
Without the transferase enzyme, the infant is types are autosomal recessive traits. Gaucher
unable to metabolize galactose-1-phosphate, disease type 2 is much less common and does
the accumulation of which results in injury to not have a striking ethnic predilection. It is char-
Answers: 99104 143

acterized by a rapid neurodegenerative course 103. (E) Spinal muscular atrophy is a genetic disorder
with extensive visceral involvement and death, in which anterior horn cells in the spinal cord
often within the first 2 years of life. It presents and motor nuclei of the brainstem are progres-
in infancy with increased tone, strabismus, and sively lost. Two clinical syndromes of infantile
organomegaly. Failure to thrive and stridor due spinal muscular atrophy can be distinguished:
to laryngospasm are typical. After a several- spinal muscular atrophy type I, which is the
year period of psychomotor regression, death acute fulminant form appearing in the first 6
occurs secondary to respiratory complications. months, and spinal muscular atrophy type II,
TaySachs disease results from the defi- which is the more chronic form. Affected new-
ciency of hexosaminidase activity and the lyso- borns in spinal muscular atrophy type I have
somal accumulation of GM2 gangliosides, generalized weakness more proximal than
particularly in the central nervous system. distal, hypotonia, and areflexia. Facial expres-
Patients with clinical manifestations of the sion is relatively well preserved, as are extraoc-
infantile form of TaySachs disease present ular movements. Despite intrauterine hypotonia,
with loss of motor skills, increased startle reac- arthrogryposis is not present. Creatine kinase is
tion, and the presence of macular pallor and a normal or mildly elevated. The diagnosis is
cherry-red spot on retinoscopy. Macrocephaly, established by a histological examination that
not associated with hydrocephalus, may shows hypertrophy of type I fibers by the
develop. In the second year of life, seizures myosin ATPase reaction. Prenatal diagnosis can
requiring anticonvulsant therapy develop. be accomplished by DNA analysis of chorion
Mucopolysaccharidoses result from a deficit of villus biopsy. (Fenichel, 173174)
enzymes involved in the catabolism of der-
matan sulfate, heparan sulfate, or keratin sul- 104. (D) The association of constipation, poor feed-
fate. The clinical picture of these diseases is ing, and incremental response to repetitive
different from the clinical picture presented in stimulation between 20 and 50 Hz is highly
the vignette. (Behrman, 475476; Fenichel, 13) suggestive of infantile botulism. It is an age-
limited disorder in which Clostridium botulinum
102. (E) Zellweger syndrome, or cerebrohepatore- is ingested, colonizes the intestinal tract, and
nal syndrome, is a rare and lethal disorder. It is produces toxin in situ. The exotoxin prevents
inherited as an autosomal recessive trait. It rep- the release of acetylcholine, causing a cholin-
resents the prototype of a group of peroxisomal ergic blockade of skeletal muscles and the end
disorders that have overlapping symptoms, organs innervated by autonomic nerves.
signs, and biochemical abnormalities. Infants Infected infants at the age of 4 weeks are usu-
with this syndrome have dysmorphic facies con- ally living in a dusty environment adjacent to
sisting of frontal bossing and a large anterior construction or agricultural soil disruption. The
fontanel. The occiput is flattened, and the exter- prodromal signs are poor feeding and consti-
nal ears are abnormal. A high-arched palate, pation. Typically, the newborn may present
excessive skinfolds of the neck, severe hypotonia, with diffuse hypotonia, ptosis, dysphagia,
and areflexia are usually evident. Examination of weak cry, and dilated pupils that react slug-
the eyes reveals searching nystagmoid move- gishly to light. Electrophysiology studies show
ments, bilateral cataracts, and optic atrophy. an incremental response to repetitive stimula-
Generalized seizures become evident early in tion at a frequency between 20 and 50 Hz. The
life, associated with severe global developmen- diagnosis is confirmed by isolation of organ-
tal delay and significant bilateral hearing loss. isms or toxin from the stool. Infantile botulism
The cause of the severe neurologic abnormalities may suggest GuillainBarr syndrome. The
is related to an arrest of migrating neuroblasts clinical differential diagnosis may be difficult,
during early development, resulting in cerebral but electrophysiology testing establishes the
pachygyria with neuronal heterotopia. Patients diagnosis. Infantile botulism differs from infan-
with Zellweger syndrome rarely survive beyond tile spinal muscular atrophy by the early
1 year of age. (Behrman, 439440) appearance of facial and pharyngeal weakness,
144 3: Pediatrics

the presence of ptosis and dilated pupils, and 107. (A) Myotonic dystrophy (DM) is a complex
the occurrence of severe constipation. Infants multisystemic disorder linked to two different
with generalized myasthenia do not have genetic loci. Myotonic dystrophy type 1 (DM1)
dilated pupils, absent reflexes, or severe con- is caused by an expansion of a CTG repeat
stipation. Lowe syndrome, or oculocerebrore- located in the 3 untranslated region (UTR) of
nal syndrome, is an X-linked disease DMPK (myotonic dystrophy protein kinase)
characterized by hypotonia and hyporeflexia on chromosome 19q13.3. Myotonic dystrophy
and sometimes cataracts. Later in infancy type 2 (DM2) is caused by an unstable CCTG
mental retardation and defects in urine acidifi- repeat in intron 1 of ZNF9 (zinc finger protein
cation appear. (Behrman, 947995) 9) on chromosome 3q21. Therefore, both DM1
and DM2 are caused by a repeat expansion in
105. (D) Myotonic dystrophy is a multisystem dis- a region transcribed into RNA but not trans-
order transmitted by autosomal dominant lated into protein. The discovery that these two
inheritance. It is caused by an unstable DNA distinct mutations cause largely similar clinical
triplet on chromosome 19 that repeats 50 to syndromes put emphasis on the molecular
several thousand times in successive genera- properties they have in common, namely, RNA
tions. The number of triplets correlates with transcripts containing expanded, nontranslated
the severity of the disease. Repeat size changes repeats. The mutant RNA transcripts of DM1
from mother to child are greater than from and DM2 aberrantly affect the splicing of the
father to child. For this reason, the mother is the same target RNAs, such as chloride channel 1
most often affected parent when a child has a (ClC-1) and insulin receptor (INSR), resulting
myotonic dystrophy. The main feature during in their shared myotonia and insulin resistance.
pregnancy is reduced fetal movements and (Cho, 195204)
polyhydramnios. Prominent clinical features
in the newborn include facial diplegia, gener- 108. (C) Duchenne muscular dystrophy is the most
alized muscular hypotonia with more proxi- common hereditary neuromuscular disease
mal than distal weakness, and arthrogryposis. affecting all races and ethnic groups. This dis-
Myotonia is not elicited by percussion and may ease is inherited as an X-linked recessive trait.
not be demonstrable by EMG. (Fenichel, 167) The abnormal gene is on the X chromosome at
the Xp21 locus and is one of the largest genes
106. (A) Duchenne and Becker muscular dystro- identified. The defected gene product is a
phies are variable phenotypic expressions of a reduced muscle content of a the structural pro-
gene defect at the Xp21 site. The abnormal gene tein dystrophin. Muscle dystrophin is found
produces a reduced muscle content of dys- on the plasma membrane surface in skeletal
trophin, a structural muscle protein. In muscle fibers, on the surfaces of plasma mem-
Duchenne muscular dystrophy, the dystrophin brane and transverse tubules of cardiac muscle
content is less than 3% of normal, whereas in fibers, and on smooth muscle membranes.
Becker muscular dystrophy, the dystrophin con- Cortical dystrophin is found in the hippocam-
tent is between 3% and 20% of normal. The inci- pus, amygdala, thalamus, hypothalamus, and
dence of Duchenne dystrophy is 1 per 3,500 neocortex, and a Purkinje cell isoform is found
male births. The initial feature of the disease is in the cerebellum. (Goetz, 776778, 787789;
gait disturbance. Toe-walking and frequent Fenichel, 177178)
falling before the age of 5 years are typical. The
decline in motor function is linear throughout 109. (A) Pompe disease, also referred to as glycogen
childhood. Motor function appears static storage disease type II or acid maltase defi-
between the age of 3 and 6 years because of cere- ciency, is a rare autosomal recessive disorder
bral maturation. The immediate cause of death caused by mutations in the gene that encodes
may be an arrhythmia, aspiration, and intercur- for [alpha]-glucosidase (GAA). GAA cleaves
rent infection. Respiratory insufficiency is a con- [alpha] 1,4 and 1,6 linkages in glycogen to
tributing factor in most cases. (Fenichel, 177178) release glucose. Deficiency of this enzyme
Answers: 105113 145

results in the accumulation of glycogen in var- with an estimated prevalence of 1 per 100,000.
ious tissues. Pompe disease can present in early HypoPP is also autosomal dominant. However,
infancy, childhood, or adulthood. In general, one third of cases are sporadic, and penetrance
the clinical presentation is related to the amount is only 50% in women. Most patients note that
of functioning enzyme present. Severe cases of some form of atypical strenuous exercise or exer-
Pompe disease in infants are usually associated tion followed by rest or sleep usually precipi-
with less than 1% enzyme activity (which is the tates an attack. Other aggravating factors
limit of assay sensitivity), and most untreated include heavy meals rich in carbohydrates and
patients die before reaching 1 year of age. In sodium, alcohol consumption, exposure to cold,
juvenile or adult patients, residual activity can and emotional stress. During peak weakness,
usually be detected and the disease presents reflexes are absent and the muscle is electrically
with debilitating muscle weakness and respi- unexcitable. Myotonia is usually not present but
ratory problems. Pompe disease is caused by may be found in the eyelids in some patients.
mutations in the acid GAA gene on chromo- The underlying genetic defect in approximately
some 17q25.2-q25.3. Almost 200 mutations have 70% of patients is a mutation in the calcium
been identified. The severity of the clinical phe- channel gene CACNA1S on chromosome 1q31.
notype is at least in part felt to be related to the In 12% of cases, sodium channel gene SCN4A
residual enzyme activity. However, absent mutations has been identified. In approximately
enzyme function has recently been documented 20% of patients, neither mutation will be identi-
in adult-onset cases. In general, nonsense muta- fied; the cause for these cases remains unknown.
tions, or insertions or deletions, leading to dis- (Hudson, 547563; Saperstein, 260269; Fenichel,
ruption of the reading frame result in absence of 194196)
functioning enzyme. Missense and splice site
mutations may result in absence of enzyme 112. (B) DejerineSottas syndrome (DSS) or
activity or allow some functioning enzyme to be CharcotMarieTooth 3 was originally
translated. (Katzin, 421431) described as a severe demyelinating neuropa-
thy of infancy and childhood associated with
110. (D) Hyperkalemic periodic paralysis (hyperPP) slow nerve conduction studies, elevated CSF
is an autosomal dominant disorders due in protein, marked clinical weakness, and hyper-
most cases to mutations in the SCN4A (SkM1) trophic nerves with onion-bulb formation.
adult skeletal muscle sodium channel gene on Recent progress in genetics demonstrated that
chromosome 17q23-25. Typical attacks in DSS includes a group of patients with different
hyperPP are characterized by generalized or gene defects. DSS-A or CMT1A is heterozy-
focal muscle weakness, often precipitated by gous for PMP-22 point mutations located on
rest after strenuous exercise. The facial and res- chromosome 17 with autosmal dominant
piratory muscles are usually spared. Attacks transmission. DSS-B or CMT1B is caused by P0
vary in frequency and severity but usually last point mutations located on chromosome 1q22
between minutes and an hour. The serum with recessive or dominant transmission. In
potassium may be normal during an acute DSS-C, the transmission is autosomal domi-
attack. Interictally, there may be clinical or elec- nant and the gene defect is located on chro-
tromyography evidence of myotonia. Lid lag mosome 8q23-q24. In CMT 4F, the gene defect
(lagging of upper eyelid on downward gaze) is located on chromosome 19q13 and in DSS-
may also be seen. Some patients develop a EGR2, the gene defect is located on chromo-
fixed proximal muscle weakness although it some 10q21.1-q22.1. (Fenichel, 184185)
remains unclear whether this is related to the
number or severity of attacks. (Hudson, 547563; 113. (D) EmeryDreifuss muscular dystrophy type
Saperstein, 260269; Fenichel, 194196) 1 is characterized by early contractures and
cardiomyopathy. Generally, it is transmitted by
111. (B) Hypokalemic periodic paralysis (hypoPP) is X-linked inheritance. The gene is located in
the most frequent form of periodic paralysis, Xq28. The abnormal gene product is emerin.
146 3: Pediatrics

The onset of symptoms occurs in patients sweating, and the improvement of stiffness by
between 5 and 15 years of age. The earliest fea- exercise make the diagnosis of neuromyotonia
ture of the disease is the development of con- unlikely. The absence of skeletal deformities
tractures in the flexor of the elbows, the ankle rules out SchwartzJampel syndrome. Stiff-
tendon, and the extensors of the hand. person syndrome is an autoimmune condition
Contractures are followed by muscle weakness that is extremely rare in children. It is charac-
and wasting first in the biceps and triceps mus- terized by involuntary truncal muscle tight-
cles, then in the deltoid and other shoulder ness without spinal deformity. Abdominal wall
muscles. The peroneal muscles are severely rigidity and contraction of the thoracolumbar
affected. The progression of symptoms is slow paraspinal muscles cause a hyperlordosis that
and the condition usually stabilizes by the age is characteristic of the disease. The patient
of 20 years. All patients develop cardiomyopa- described in the vignette does not have the fea-
thy, which may lead to atrial paralysis, brady- tures of stiff-person syndrome. (Fenichel,
cardia, or syncope. (Fenichel, 190191) 154155)

114. (A) The patient described in the vignette has 116. (C) The patient described in this case has pain
signs of chronic polyneuropathy, cerebellar and muscle weakness associated with mild
ataxia, decreased deep tendon reflexes, and rhabdomyolysis and failure to generate ammo-
night blindness. These signs are highly sug- nia on an ischemic exercise test. These features
gestive of Refsum disease. It is a hereditary are suggestive of myoadenylate deaminase
motor and sensory polyneuropathy type IV, deficiency, an autosomal recessive disease. The
which is an autosomal recessive disorder defective gene is located on chromosome 1p.
caused by an inborn error in the metabolism of Myoadenylate deaminase deficiency most
phytanic acid. The clinical picture may include, commonly presents with isolated muscle weak-
beside the symptoms described in the vignette, ness, fatigue, and myalgias following moder-
progressive hearing loss, cardiomyopathy, ate-to-vigorous exercise. Myalgias may be
ichthyosis, and pes cavus. The diagnosis is con- associated with an increased serum creatine
firmed by showing reduced oxidation of phy- kinase level and detectable electromyelo-
tanic acid in cultured fibroblasts. (Fenichel, graphic abnormalities. Muscle wasting or his-
186187) tologic changes on biopsy are absent. The age
of onset may be as early as 8 months of life. The
115. (A) The patient described in this case has a enzyme defect has been identified in asympto-
generalized myotonic syndrome. The associa- matic family members. The disorder may be
tion of generalized muscle hypertrophy and screened for by performing an exercise test. The
myotonic discharges on EMG examination is normal elevation of venous plasma ammonia
suggestive of myotonia congenita. following exercise that is seen in normal sub-
Transmission of the disorder is either autoso- jects is absent in myoadenylate deaminase defi-
mal dominant or recessive. The abnormal gene ciency. The final diagnosis is made by
is located on chromosome 17q23-35. Clinical histochemical or biochemical assays of a muscle
features are stereotyped. At rest, muscles are biopsy. The absence of external ophthalmople-
stiff, with difficulty in moving, which improves gia, pigmentary degeneration of the retina, and
with activity. The myotonia causes generalized heart block make the diagnosis of KearnsSayre
muscle hypertrophy, which gives the infant a syndrome unlikely. Brody disease is caused by
Herculean appearance. The diagnosis is estab- a deficiency of calcium-activated ATPase in
lished by EMG, which shows repetitive dis- the sarcoplasmic reticulum. Stiffness becomes
charges at rates of 20 to 80 Hz when the needle worse with exercise. Ischemic exercise results
is inserted into the muscle or on voluntary con- are normal. Muscle biopsy results reveal type II
traction (myotonic discharges). Muscle biopsy atrophy. Biochemical studies confirm the diag-
shows the absence of type II fibers. The absence nosis. The age of onset and the absence of
of involuntary muscle twitching, excessive myoclonic seizures rule out Menkes syndrome,
Answers: 114120 147

a disorder of intestinal copper transport that thocytes, low cholesterol, and low triglyceride
starts at the age of 3 months. Most patients die levels. Ataxia telangiectasia is a multisystem
before the age of 18 months. Carnitine palmitoyl disorder affecting the nervous and immune
transferase deficiency causes exercise intoler- systems. It is transmitted by autosomal reces-
ance and reduced production of ketone bodies sive inheritance. The abnormal gene is located
in the blood or urine during fasting. (Behrman, on chromosome 11. Clinical features include
492; Fenichel, 207) chronic sinopulmonary infections, truncal
ataxia, oculomotor apraxia, and telangiectasias.
117. (A) The association of limb ataxia with photo- Ramsay Hunt syndrome is a progressive degen-
sensitivity indicates the diagnosis of Hartnup eration of the dentate nucleus and superior cere-
disease. This is a disorder transmitted by auto- bellar peduncle characterized by myoclonus
somal recessive inheritance. It is caused by a and cerebellar ataxia without elevation of very
defect of neutral amino acid transport in cells of long chain fatty acids. (Behrman, 20322033;
the proximal cell tubules and small intestine. Fenichel, 145146)
The result is massive aminoaciduria and reten-
tion of amino acids in the small intestine, where 119. (E) The clinical picture described in this ques-
they may be converted into absorbed toxic tion is suggestive of Harp syndrome. This is a
products. Clinical features include limb ataxia, genetic disorder transmitted by autosomal
nystagmus, decreased tone, and pellagra-like recessive inheritance. Clinical features of the
skin lesions after exposure to sunlight because disease include retinitis pigmentosa, mild
of nicotinamide deficiency. Mental change may mental abnormalities, dystonic dysarthria, and
occur, ranging from emotional instability to decreased facial expression. Head MRI shows
delirium. Administration of nicotinamide may an eye-of-the-tiger appearance of the pallidum.
prevent the rash. (Fenichel, 222) Acanthocytes and echinocytes are seen in
preparations of washed erythrocytes. (Behrman,
118. (E) Progressive ataxia with an increased blood 20202023; Ching, 16731674; Goetz, 725)
level of very long chain fatty acids occurs in
adrenoleukodystrophy. This is an X-linked dis- 120. (B) The patient described in this vignette pres-
order associated with the accumulation of sat- ents a clinical picture highly suggestive of
urated very long chain fatty acids and HallervordenSpatz disease. This is a rare
progressive dysfunction of the adrenal cortex degenerative disorder inherited as an autoso-
and nervous system white matter. Excess hexa- mal recessive trait. Linkage analysis indicates
cosanoic acid is the most striking and charac- that the gene is located on chromosome 20p13.
teristic feature. This accumulation of fatty acids The condition usually begins during childhood
is due to genetically determined deficient and is characterized by progressive dystonia,
degradation of fatty acids, which is a normal rigidity, and choreoathetosis. Spasticity, exten-
peroxisomal function. Sulfatide lipidosis is a sor plantar responses, dysarthria, and intellec-
disorder of central and peripheral myelin tual deterioration become evident during
metabolism caused by deficient activity of the adolescence, and death usually occurs by early
enzyme arylsulfatase A. Abetalipoproteinemia is adulthood. Head MRI shows lesions of the
a disorder of lipid metabolism transmitted by globus pallidus, including low signal intensity
autosomal recessive inheritance. Apolipoprotein in T2-weighted images (corresponding to iron
B is essential for the synthesis and integrity of pigments) and an anteromedial area of
low-density and very low density lipoproteins. high signal intensity or eye-of-the-tiger sign
Its absence results in fat malabsorption and a (corresponding to areas of vacuolation).
progressive deficiency of vitamins A, E, and K. Neuropathological examination indicates
Clinical features include progressive cerebel- excessive accumulation of iron-containing pig-
lar ataxia, delayed psychomotor development, ments in the globus pallidus and substantia
and retinitis pigmentosa. Laboratory features nigra. (Clement, 2631; Behrman, 20202023; Goetz,
include severe anemia, the presence of acan- 725)
148 3: Pediatrics

121. (E) Sydenham chorea is the most common


REFERENCES
acquired chorea of childhood and is the sole
neurological manifestation of rheumatic
Ahdab-Barmada M, Moossy J. The neuropathology of ker-
fever. The pathogenesis of Sydenham chorea
nicterus in the premature neonate: diagnostic problems.
is probably an autoimmune response of the J Neuropathol Exp Neurol. 1984;43:45-56.
central nervous system to group A strepto- Behrman RE. Nelson Textbook of Pediatrics. 16th ed.
coccal organisms. The majority of children Philadelphia: Saunders; 2000.
with Sydenham chorea have antineuronal Bergqvist A, Christina G. Idiopathic pediatric epilepsy
antibodies, which develop in response to syndromes. Continuum: lifelong learning in neurology.
group A beta-hemolytic streptococcal infec- Epilepsy. 2007;13(4):106-120.
tions. Antineuronal antibodies cross-react Bradshaw DY, Jones HR Jr. Guillain Barre syndrome in
with the cytoplasm of subthalamic and cau- children: clinical course, electrodiagnosis, and prognosis.
date nuclei neurons. (Behrman, 20322033; Muscle Nerve. 1992;15:500-506.
Fenichel, 285286) Callaghan N, Garrett A, Goggin T. Withdrawal of anti-
convulsant drugs in patients free of seizures for two
years. A prospective study. N Engl J Med. 1988; 318:
122. (B) The combination of encephalopathy and
942-946.
progressive calcification of the basal ganglia is Casey, BJ, Nigg JT, Durston S. New potential leads in the
seen in Fahr disease. Affected children may biology and treatment of attention deficit-hyperactivity
have dwarfism, senile appearance, retinitis disorder. Curr Opin Neurol. 2007;20(2):119-124.
pigmentosa, mental retardation, choreoa- Ching, KH et al. HARP syndrome is allelic with pantothen-
thetotic movements, ataxia, dysarthria, and ate kinase-associated neurodegeneration. Neurology.
seizures. Head CT may show calcification that 2002;58(11):1673-1674.
appears first in the dentate nuclei and pons, Cho DH, Tapscott SJ. Myotonic dystrophy: emerging mech-
then in the basal ganglia, and finally in the anisms for DM1 and DM2. Biochim Biophys Acta.
corpus callosum. (Behrman, 20322033; Fenichel, 2007;1772(2):195-120.
284) Clement F et al. Neurodegeneration with brain iron accu-
mulation: clinical, radiographic and genetic hetero-
geneity and corresponding therapeutic options. Acta
123. (A) Tardive dyskinesia is characterized by
Neurol Belg. 2007;107(1):26-31.
stereotypical facial movements, particularly by Dauvilliers Y, Arnulf I, Mignot E. Narcolepsy with cata-
lip smacking and protrusion and retraction of plexy. Lancet. 2007;369:499-451.
the tongue. These are drug-induced choreiform Fenichel GM. Clinical Pediatric Neurology. A Signs and
movements. The condition is most often asso- Symptoms Approach. 5th ed. Philadelphia: Saunders; 2005.
ciated with drugs used to modify behavior or Geschwind DH, Spence SJ. Genetics of autism. Continuum:
with antiemetics. It can occur in children with Lifelong Learning in Neurology. 2008;14(2)(Neurogenetics):
asthma treated with theophylline. (Behrman, 49-64.
20322033; Fenichel, 287) Goetz CG, Pappert EJ. Textbook of Clinical Neurology,
Philadelphia: Saunders; 1999.
124. (D) Neuroacanthocytosis is a rare disorder Goldenberg J, Ferraz MB, Fonseca AS, Hilario MO, Bastos
W, Sachetti S. Sydenham chorea: clinical and laboratory
with autosomal dominant, recessive, or even X-
findings. Analysis of 187 cases. Rev Paul Med. 1992;110(4):
linked inheritance. It is manifested by chorea,
152-157.
tics, dystonia, parkinsonism, self-mutilatory Hudson A.J, Ebers GC, Bulman DE. The skeletal muscle
behavior, amyotrophy, areflexia, and elevated sodium and chloride channel diseases. Brain. 1995;118
creatine phosphokinase. Among the most dis- (Pt 2):547-563.
tinguishing features of neuroacanthocytosis is Jensen FE. Developmental factors regulating susceptibility
an eating dysfunction due to orolingual dysto- to perinatal brain injury and seizures. Curr Opin Pediatr.
nia that is manifested by expulsion of food 2006;18:628-633.
from the mouth by a protruding tongue. Katzin LW, Amato AA. Pompe disease: a review of the
Involuntary vocalizations and parkinsonism current diagnosis and treatment recommendations in
also occur. (Behrman, 20202023; Fenichel, 284285; the era of enzyme replacement therapy. J Clin Neuro-
Goetz, 725) muscul Dis. 2008;9:421-431.
Answers: 121124 149

Keam S, Walker MC. Therapies for narcolepsy with or Ropper AH, Brown RH. The inherited metabolic diseases of
without cataplexy: evidence-based review. Curr Opin the nervous system. In: Ropper AH, Brown RH: Adams
Neurol. 2007;20:699-703. and Victors Principles of Neurology. 8th ed. Chapter 37.
Kotagal S. Parasomnias of childhood. Curr Opin Pediatr. Available at http://www.accessmedicine.com/content.
2008;20:659-665. aspx?aID=975151
Kothare SV, Kaleyias J. Narcolepsy and other hypersom- Ropper AH, Brown RH. Developmental diseases of the
nias in children. Curr Opin Pediatr. 2008;20:666-675. nervous system. In: Ropper AH, Brown RH: Adams and
Lee MJ, Stephenson DA. Recent developments in neurofi- Victors Principles of Neurology. 8th ed. Chapter 38.
bromatosis type 1. Curr Opin Neurol. 2007;20(2):135-141. Available at http://www.accessmedicine.com/content.
Lewis DW. Headaches in children and adolescents. Curr aspx?aID=975888
Probl Pediatr Adolesc Health Care. 2007;37:207-246. Rosenbloom L. Diagnosis and management of cerebral
Lian G, Sheen V. Cerebral developmental disorders. Curr palsy. Arch Dis Child. 1995;72:350-354.
Opin Pediatr. 2006;18:614-620. Rowland LP, Merritt HH. Merritts Neurology. Rowland
Limperopoulos C, du Plessis AJ. Disorders of cerebellar LP, ed. Philadelphia: Lippincott Williams & Wilkins;
growth and development. Curr Opin Pediatr. 2006;18: 2005.
621-627. Saperstein DS. Muscle channelopathies. Semin Neurol.
Menkes JH, Sarnat HB, Maria BL. Child Neurology. 7th ed. 2008;28(2):260-269.
Philadelphia: Lippincott Williams & Wilkins; 2006. Shinnar S, Berg AT, Moshe SL, Kang H, ODell C, Alemany M,
Nabbout R, Dulac O. Epileptic syndromes in infancy and et al. Discontinuing antiepileptic drugs in children with
childhood. Curr Opin Neurol. 2008;21:161-166. epilepsy: a prospective study. Ann Neurol. 1994;35:534-545.
Nausieda PA, Grossman BJ, Koller WC, Weiner WJ, Taft LT. Cerebral palsy. Pediatr Rev. 1995;16:411-408.
Klawans HL. Sydenham chorea: an update. Neurology. U.S. Modafinil in Narcolepsy Multicenter Study Group.
1980;30:331-334. Randomized trial of modafinil as a treatment for the exces-
Nelson KB. Epidemiology and etiology of cerebral palsy. sive daytime somnolence of narcolepsy: treatment modal-
In: Capture AJ, Accardo PJ, eds. Developmental Disabilities ities for narcolepsy. Neurology. 1998;50(2 Suppl 1):S43-S48.
in Infancy and Childhood. 2nd ed. Baltimore: Brookes; Walker DM, Teach SJ. Emergency department treatment of
1996:73-80. primary headaches in children and adolescents. Curr
Panayiotopoulos CP. Typical absence seizures and their Opin Pediatr. 2008;20:248-254.
treatment. Arch Dis Child. 1999;81:351-355. Weeks RA. Turjanski N. Brooks DJ. Tourettes syndrome: a
Pidcock FS, Graziani LJ, Stanley C, Mitchell DG, Merton D. disorder of cingulate and orbitofrontal function? QJM.
Neurosonographic features of periventricular echoden- 1996;89:401-408.
sities associated with cerebral palsy in preterm infants. Wolf DS, Singer HS. Pediatric movement disorders: an
J Pediatr. 1990;116:417-422. update. Curr Opin Neurol. 2008;21:491-496.
Rivkin MJ, Anderson ML, Kaye EM. Neonatal idiopathic Zaki MS, Abdel-Aleem A, Abdel-Salam G, Marsh SE,
cerebral venous thrombosis: an unrecognized cause of Silhavy JL, Barkovich, AJ, et al. The molar tooth sign: a
transient seizures or lethargy. Ann Neurol. 1992;32: new Joubert syndrome and related cerebellar disorders
51-56. classification system tested in Egyptian families.
Ropper AH. The GuillainBarr syndrome. N Engl J Med. Neurology. 2008;70(7):556-565.
1992;326:1130-1136.
This page intentionally left blank
CHAPTER 4

Neurophysiology, Epilepsy, Evoked


Potentials, and Sleep Disorders
Questions

1. What is the frequency of the posterior domi- 4. Which of the following cardinal features of
nant rhythm at the age of 3 months? stage II sleep is not seen in stage I sleep?
(A) 9 Hz (A) Alpha attenuation
(B) 7 Hz (B) Positive occipital sharp transient
(C) 6 Hz (C) Sleep spindles
(D) 8 Hz (D) Vertex sharp transient
(E) 4 Hz (E) Increased frontocentral beta rhythm

2. Which of the following statements is true about 5. Which of the following statements is true about
mu rhythm? photomyoclonic response?
(A) Mu activity is increased with move- (A) It occurs in the occipital area when the
ments of the contralateral arm. flashing light evokes facial muscle con-
(B) Mu amplitude is usually higher than the traction.
amplitude of alpha rhythm. (B) It is enhanced in case of barbiturate
(C) It is always unilateral. withdrawal.
(D) Mu activity should be considered (C) It is reduced in case of ethanol with-
normal even if it is persistent in a region drawal.
of focal slowing. (D) Stimulation rate of 2 Hz is most effective
(E) Mu activity may slow gradually with in producing a photomyoclonic
aging. response.
(E) The amplitude of muscle contraction
3. Which of the following drugs have the least decreases as the photic stimulation con-
effect on beta rhythm? tinues.
(A) Phenobarbital
6. Well-formed synchronous sleep spindles
(B) Valproic acid appear at the age of
(C) Clonazepam
(A) 1 month
(D) Chloral hydrate
(B) 3 months
(E) Amitriptyline
(C) 4 months
(D) 6 months
(E) 2 years

151
152 4: Neurophysiology, Epilepsy, Evoked Potentials, and Sleep Disorders

7. During the neonatal period, trace discontinue 12. Which of the following is true about delta
first appears during the gestational age of brushes?
(A) 25 weeks (A) They represent the fusion of underlying
(B) 30 weeks delta transient with a superimposed
(B) 32 weeks rhythmic fast activity.
(C) 35 weeks (B) They occur only during active sleep.
(D) 37 weeks (C) They disappear by the age of 1 year.
(E) 40 weeks (D) Frontal delta brushes are frequent at any
age.
8. During the neonatal period, trace alternant first (E) They are more common in the occipital
appears during the gestational age of area at the conceptional age of 29 weeks.

(A) 32 weeks 13. Sleep spindles appear for the first time at a con-
(B) 33 weeks ceptional age of
(C) 34 weeks
(A) 32 weeks
(D) 35 weeks
(B) 36 weeks
(E) 36 weeks
(C) 40 weeks
9. During the neonatal period, activit moyenne (D) 46 weeks
first appears during the gestational age of (E) 50 weeks

(A) 32 weeks 14. Which of the following is true about active


(B) 33 weeks sleep in a full-term neonate?
(C) 34 weeks
(A) Eyes are always closed.
(D) 35 weeks
(B) There is increased muscle tone.
(E) 36 weeks
(C) There is no body or facial movement.
10. During the neonatal period, continuous slow- (D) Respiration is irregular.
wave sleep first appears during the gestational (E) Active sleep comprises 30% of the time
age of spent in sleep by a full-term neonate.

(A) 40 weeks 15. Photomyogenic response is


(B) 41 weeks
(A) an epileptiform discharge triggered by
(C) 42 weeks photic stimulation
(D) 43 weeks (B) a spike-like driving response
(E) 44 weeks (C) of noncerebral origin
(D) less frequent than the flashing stimulus
11. An asynchronous burst of hemispheric activity
is defined as activity in one hemisphere leading (E) felt to be an abnormal response to high-
the other by more than intensity light

(A) 5 seconds 16. Posterior predominant activity during wake-


(B) 0.5 second fulness blocked by eye opening is suggestive of
(C) 1 second (A) alpha activity
(D) 1.5 seconds (B) alpha rhythm
(E) 3 seconds (C) temporal transient
(D) Bancaud phenomenon
(E) posterior slowing of youth
Questions: 726 153

17. Failure of alpha rhythm to block on eye open- (D) bihemispheric lateral epileptiform dis-
ing is suggestive of charges
(A) alpha activity (E) the burst suppression pattern
(B) alpha rhythm
23. Benzodiazepine overdose typically shows an
(C) temporal transient EEG pattern of
(D) Bancaud phenomenon
(A) triphasic waves
(E) posterior slowing of youth
(B) widespread high-amplitude beta activity
18. Sail wave is also called (C) burst suppression pattern
(D) alpha coma
(A alpha activity
(E) spontaneous epileptiform spike and
(B) alpha rhythm
wave
(C) temporal transient
(D) Bancaud phenomenon 24. In which stage of Rett syndrome does the EEG
(E) posterior slowing of youth show background slowing with focal spike or
sharp waves, most commonly over the cen-
19. Sylvian theta activity that seems to be related to troparietal region?
normal aging is suggestive of
(A) Stage 1
(A) alpha activity (B) Stage 2
(B) alpha rhythm (C) Stage 3
(C) temporal transient (D) Stage 4
(D) Bancaud phenomenon (E) None of the above
(E) posterior slowing of youth
25. Which of the following is true about EEG
20. Triphasic waves are not seen in guidelines in the determination of brain death
in adults?
(A) renal failure
(B) hepatic failure (A) Minimum of four scalp and two ear elec-
trodes
(C) hyponatremia
(B) Electrode impedances between 1,000
(D) hypoglycemia
and 100 ohms
(E) hypoparathyroidism
(C) Instrument sensitivity set at 20 V/mm
21. Which of the following EEG waves is the least (D) Interelectrode distance of 8 cm or less
likely to be seen in anoxic encephalopathy? (E) Monitoring for artifact, EMG, and ECG
(A) Burst suppression pattern 26. Which of the following EEG patterns has the best
(B) Periodic spike or sharp waves prognosis in case of anoxic encephalopathy?
(C) Alpha coma pattern
(A) Nearly isoelectric record
(D) Delta brushes
(B) Dominant alpha rhythm with scattered
(E) Bihemispheric epileptiform discharges theta activity
(C) Dominant theta activity with rare alpha
22. The EEG pattern most commonly seen in dial-
activity
ysis dementia is
(D) Invariant low-amplitude delta activity
(A) the bilateral spike-and-wave complex unresponsive to stimulus
(B) triphasic waves (E) Continuous polymorphic slow delta
(C) the alpha pattern waves with little activity and fast fre-
quency
154 4: Neurophysiology, Epilepsy, Evoked Potentials, and Sleep Disorders

27. In which of the following conditions does alpha (D) wicket spikes
coma cause the abnormal alpha rhythm to have (E) benign epileptiform transients of sleep
some reactivity and to be more prominent over
the posterior head region? 32. Which of the following is true of lambda
(A) Ventral pons ischemic stroke waves?
(B) Cardiorespiratory arrest (A) Monophasic triangular waves in the
(C) Barbiturate overdose occipital region.
(D) Benzodiazepine overdose (B) Small sharp spikes seen in adults during
(E) Methaqualone overdose drowsiness without distortion of the
background activity.
28. A relatively low voltage mixed (2 Hz to 7 Hz) fre- (C) Arc-like waves typically occurring in
quency EEG with episodic rapid eye movements trains that are often mistaken for a tem-
and absent or reduced chin EMG activity are poral spike.
suggestive of which of the following sleep (D) They may represent an evoked cerebral
stages? response to visual stimulus produced
from shifts of images across the retina in
(A) Stage N1
the course of saccadic eye movements.
(B) Stage N2
(E) Rolandic alpha activity.
(C) Stage N3
(D) Wakefulness 33. Rolandic alpha activity is also called
(E ) Rapid eye movement (REM) sleep
(A) mu rhythm
29. Monophasic triangular waves in the occipital (B) positive occipital sharp transients
region define (C) lambda waves
(D) wicket spikes
(A) mu rhythm
(E) benign epileptiform transients of sleep
(B) positive occipital sharp transients
(C) lambda waves 34. The metabolism of phenytoin slows substan-
(D) wicket spikes tially when its serum concentration reaches
(E) benign epileptiform transients of sleep
(A) 10 g/dL
30. Small sharp spikes seen in adults during (B) 15 g/dL
drowsiness without distortion of the back- (C) 20 g/dL
ground activity define (D) 25 g/dL
(A) mu rhythm (E) 30 g/dL
(B) positive occipital sharp transients
35. Which of the following drugs reduces the level
(C) lambda waves of serum phenytoin?
(D) wicket spikes
(A) Rifampin
(E) benign epileptiform transients of sleep
(B) Cimetidine
31. Arc-like waves typically occur in trains. They (C) Chloramphenicol
are often mistaken for a temporal spike and (D) Isoniazid
are suggestive of (E) Dicumarol
(A) mu rhythm
(B) positive occipital sharp transients
(C) lambda waves
Questions: 2744 155

36. Which of the following drugs alters the free 41. N9 evoked potential is generated by
phenytoin level without altering the total meas-
(A) Erbs point
ured serum level?
(B) caudal medial lemniscus
(A) Sulfonamides (C) primary sensory cortex from median
(B) Chlorenphenicol nerve stimulation
(C) Disulfiram (D) lumbar cord
(D) Antacids (E) cortical response from posterior tibial
(E) Salicylates nerve stimulation

37. Which of the following antiepileptic drugs is a 42. N22 evoked potential is generated by
calcium channel blocker?
(A) Erbs point
(A) Phenytoin (B) caudal medial lemniscus
(B) Carbamazepine (C) primary sensory cortex from median
(C) Ethosuximide nerve stimulation
(D) Phenobarbital (D) lumbar cord
(E) Valproic acid (E) cortical response from posterior tibial
nerve stimulation
38. The addition of valproic acid to a chronic reg-
imen of phenytoin causes immediate 43. Breach rhythm is linked to which of the fol-
lowing EEG variety?
(A) valproic acid toxicity
(B) phenytoin toxicity (A) Sleep spindle
(C) a higher serum concentration of pheny- (B) Skull defect causes enhanced beta activity
toin (C) Presence of occipital sharp transient
(D) a lower serum concentration of pheny- (D) Delta slowing in 20% to 50% of its record
toin (E) Unilateral failure of alpha wave block-
(E) a lower serum concentration of valproic ing on eye opening
acid
44. Which of the following is not a side effect of
39. Wave III evoked potential is generated by valproic acid?
(A) the distal part of the auditory nerve (A) Hair loss
(B) cortical response to visual evoked poten- (B) Weight gain
tials (C) Essential tremor
(C) trapezoid body (D) Acute pancreatitis
(D) midbrain (E) Aplastic anemia
(E) Erbs point

40. P100 evoked potential is generated by


(A) distal part of the auditory nerve
(B) cortical response to visual evoked poten-
tials
(C) trapezoid body
(D) midbrain
(E) Erbs point
156 4: Neurophysiology, Epilepsy, Evoked Potentials, and Sleep Disorders

45. Which of the following is true of carba- (A) a physiological effect of carbamazepine
mazepine? (B) carbamazepine intoxication
(A) It is highly soluble in water. (C) an idiosyncratic reaction to carba-
(B) In the naive patient, the half-life of the mazepine
drug is about 30 hours; it decreases to 10 (D) psychogenic symptoms not related to
hours within a few weeks. carbamazepine administration
(C) Skin rash is seen less with carba- (E) none of the above
mazepine than with phenytoin.
(D) Hypernatremia may complicate the 49. Which of the following is true of lamotrigine?
chronic use of carbamazepine. (A) The half-life of the drug is not affected
(E) It may cause elevation of hepatic by other antiepileptic drugs.
enzymes that predispose to hepatitis. (B) It acts by prolonging inactivation of volt-
age-sensitive calcium channels.
46. Which of the following antiseizure medications (C) Cutaneous rash may occur in 10% of
is not metabolized by the liver? cases.
(A) Zonisamide (D) Its antiseizure activity correlates with its
(B) Levetiracetam ability to inhibit dihydrofolate reductase
(C) Topiramate activity.
(D) Lamotrigine (E) It is about 95% protein-bound.
(E) Tiagabine
50. In case of renal failure, which of the following
drugs needs the least adjustment?
47. A 45-year-old man with a history of seizure dis-
order and ethanol abuse is admitted to the neu- (A) Ethosuximide
rology floor because of phenytoin intoxication. (B) Carbamazepine
His admission phenytoin level is 45 g/mL. On (C) Phenytoin
the second and third days of admission, his
(D) Phenobarbital
levels drop to 35 and 25 g/mL, respectively.
What would be his level on the fourth day of (E) Topiramate
admission?
51. Which of the following antiepileptic drugs is
(A) 15 g/mL not removed by dialysis?
(B) 10 g/mL (A) Ethosuximide
(C) Unpredictable because phenytoin fol- (B) Phenobarbital
lows zero-order kinetics
(C) Gabapentin
(D) Unpredictable because phenytoin fol-
(D) Phenytoin
lows first-order kinetics
(E) Lamotrigine
(E) Unpredictable because the patient is an
ethanol abuser 52. Which of the following drugs is the treatment
of choice for infantile spasm?
48. A 25-year-old man is diagnosed with a com-
plex partial seizure and started on carba- (A) Carbamazepine
mazepine 200 mg PO qid. In the first week of (B) Adrenocorticotropic hormone (ACTH)
treatment, he developesd blurred vision, nys- (C) Phenobarbital
tagmus, dizziness, fatigue, and headache that (D) Topiramate
progressively improve. He asks about these (E) Phenytoin
symptoms 2 weeks later. His carbamazepine
level was 8 g/mL. The initial symptoms were
due to
Questions: 4561 157

53. As a side effect, phenytoin may cause 57. As a side effect, felbamate may cause
(A) somnolence, headaches, and ataxia (A) somnolence, headaches, and ataxia
(B) poor memory, cognitive impairment, (B) aplastic anemia
and Dupuytren contracture with chronic (C) cutaneous rash in up to 10% of cases,
use especially with concomitant administra-
(C) cutaneous rash in up to 10% of cases, tion of valproic acid
especially with concomitant administra- (D) weight gain in 50% of cases
tion of valproic acid (E) hyponatremia
(D) weight gain in 50% of cases
(E) cerebellar atrophy and gum hypertro- 58. As a side effect, zonisamide may cause
phy with long-term use
(A) somnolence, headaches, and ataxia
54. As a side effect, lamotrigine may cause (B) aplastic anemia
(C) hyponatremia
(A) somnolence, headaches, and ataxia
(D) weight gain in 50% of cases
(B) poor memory, cognitive impairment,
and Dupuytren contracture with chronic (E) renal stones
use
59. As a side effect, gabapentin may cause
(C) cutaneous rash in up to 10% of cases,
especially with concomitant administra- (A) somnolence, headaches, and ataxia
tion of valproic acid (B) poor memory, cognitive impairment, and
(D) weight gain in 50% of cases Dupuytren contracture with chronic use
(E) cerebellar atrophy and gum hypertro- (C) cutaneous rash in up to 10% of cases,
phy with long-term use especially with concomitant administra-
tion of valproic acid
55. As a side effect, phenobarbital may cause (D) weight gain in 50% of cases
(A) somnolence, headaches, and ataxia (E) cerebellar atrophy and gum hypertro-
(B) poor memory, cognitive impairment, phy with long-term use
and Dupuytren contracture with chronic
use 60. As side effect, valproic acid may cause
(C) cutaneous rash in up to 10% of cases, (A) somnolence, headaches, and ataxia
especially with concomitant administra- (B) poor memory, cognitive impairment, and
tion of valproic acid Dupuytren contracture with chronic use
(D) weight gain in 50% of cases (C) cutaneous rash in up to 10% of cases,
(E) cerebellar atrophy and gum hypertro- especially with concomitant administra-
phy with long-term use tion of valproic acid
(D) weight gain in 50% of cases
56. As a side effect, carbamazepine may cause (E) cerebellar atrophy and gum hypertro-
(A) somnolence, headaches, and ataxia phy with long-term use
(B) aplastic anemia
61. Which of the following drugs is removed by
(C) cutaneous rash in up to 10% of cases,
dialysis?
especially with concomitant administra-
tion of valproic acid (A) Carbamazepine
(D) weight gain in 50% of cases (B) Topiramate
(E) hyponatremia (C) Valproic acid
(D) Phenytoin
(E) None of the above
158 4: Neurophysiology, Epilepsy, Evoked Potentials, and Sleep Disorders

62. Which of the following antiepileptic drugs does 67. What is the most likely EEG pattern seen in a
not inhibit voltage-dependent sodium channels? 56-year-old man with acute left parietal
ischemic stroke?
(A) Tiagabine
(B) Phenytoin (A) Left triphasic wave
(C) Carbamazepine (B) Beta asymmetry
(D) Valproic acid (C) Generalized temporal theta
(E) Zonisamide (D) Hypsarrhythmia
(E) PLEDs (periodic lateralizing epilepti-
63. The overall risk of having a baby with a major form discharges)
malformation from a mother taking a single
antiepileptic drug is 68. Which of the following is an inhibitor neuro-
transmitter?
(A) 2%
(B) 5% (A) Aspartic acid
(C) 10% (B) Cysteic acid
(D) 15% (C) Glutamic acid
(E) 20% (D) GABA
(E) Homocystic acid
64. Which of the following drugs increases the
level of phenytoin? 69. Hyperventilation is most helpful in which of
the following?
(A) Reserpine
(B) Sucralfate (A) A 65-year-old man with recurrence of
(C) Amiodarone stroke
(D) Verapamil (B) An 8-year-old girl with staring spells
(E) Erythromycin (C) A 40-year-old woman with history of
depression
65. Which of the following drugs decreases the (D) A 53-year-old woman with history of
level of carbamazepine? cerebral aneurysm
(E) A 10-year-old boy with history of atten-
(A) Cimetidine
tion deficithyperactivity disorder
(B) Fluoxetine
(C) Isoniazid 70. Which of the following is true of REM para-
(D) Theophylline somnia?
(E) Propoxyphene (A) It shows marked female predominance.
(B) It usually occurs in preschool children.
66. Which of the following drugs may increase
EEG beta activity? (C) It often occurs in the first portion of
sleep.
(A) Olanzapine (D) It results from lack of normal atonia of
(B) Phenytoin REM sleep.
(C) Carbamazepine (E) It responds well to treatment with a tri-
(D) Topiramate cyclic antidepressant.
(E) Lorazepam
Questions: 6278 159

71. Which of the following is more characteristic of 75. The most frequent inducing factor for reflex
complex partial seizure rather than absence epilepsy is
seizure?
(A) menstruation
(A) 3-Hz spike and wave on EEG (B) visual stimuli
(B) Photic stimulation inducing seizure (C) music
activity in 10% to 30% of cases (D) eating
(C) Brief period of confusion, emotional dis- (E) bathing in hot water
turbance, and headache in the postictal
phase. 76. Which of the following is true about West syn-
(D) Hyperventilation most likely increasing drome?
the seizure activity in the EEG
(A) Phenobarbital is the treatment of choice.
(E) Most likely age of onset in childhood or
early adulthood (B) EEG typically shows burst suppression
pattern.
72. Which of the following is more suggestive of (C) Seizures remit only on antiepileptic
lateral temporal seizure than medial temporal medications.
seizure? (D) Patients typically show spasms that take
the form of sudden brief contractions of
(A) A history of febrile seizure
the head, neck or trunk, usually in flex-
(B) Autonomic signs or symptoms ion but sometimes in extension.
(C) Motionless stare (E) Long-term prognosis is good, with
(D) Structured hallucination of visual type improvement of mental retardation after
during the aura period the spasms cease.
(E) Postictal confusion
77. In a benign rolandic epilepsy
73. Which of the following is not characteristic of
(A) seizures have a strong tendency to occur
frontal seizure?
during wakefulness
(A) Frequent attacks with clustering (B) typical seizures are primarily generalized
(B) Presence of psychiatric aura (C) typical EEG shows focal high-amplitude
(C) Absence of postictal confusion midtemporal spikes
(D) Frequent secondary generalization (D) cognitive function is usually mildly
(E) EEG showing no focal abnormalities affected
ictally or interictally (E) seizures may persist for 5 years after
their onset in 90% of cases
74. The incidence of epilepsy is greater in which of
the following degenerative diseases? 78. LennoxGastaut syndrome is characterized by
(A) Alzheimer disease (A) onset after the age of 10 years
(B) Pick disease (B) multiple seizures
(C) Huntington disease (C) absence of mental retardation
(D) Wilson disease (D) seizures precipitated by hyperventilation
(E) Amyotrophic lateral sclerosis (E) a good response to antiepileptic drugs
160 4: Neurophysiology, Epilepsy, Evoked Potentials, and Sleep Disorders

79. Which of the following is true about childhood 82. What is the appropriate EEG pattern of tem-
absence epilepsy? poral lobe epilepsy?
(A) Monozygotic twins develop absence as (A) Generalized epileptiform discharges of
frequently as dizygotic twins. 3 Hz maximum in the parasagittal
(B) Hyperventilation may precipitate region. Normal background. Photic stim-
absence more than photic stimulation. ulation often induces occipital spikes.
(C) Ten percent of patients with childhood (B) Bitemporal spikes are usually maximal
absence epilepsy develop generalized over the temporal frontal region. Ictal
tonicclonic seizures within 5 to 10 years EEG shows rhythmic theta at onset.
after the onset of absence seizures. (C) Posterior 1.5 to 3 spikes and slow waves
(D) Typical EEG in childhood absence per second discharges that usually atten-
epilepsy shows generalized bilateral uate with eye opening.
synchronous, symmetric 3-Hz spikes (D) Hypsarrhythmia.
and slow-wave discharges with abnor- (E) Triphasic large-amplitude spikes maxi-
mal background. mum in the centrotemporal area.
(E) By adulthood, remission occurs in only
20% of cases. 83. What is the appropriate EEG pattern of infan-
tile spasm?
80. Which of the following is not a progressive
(A) Slow disorganized background. EEG
myoclonic epilepsy?
with superadded 1- to 2.5-Hz general-
(A) Juvenile myoclonic epilepsy ized interiorly predominant and slow-
(B) Lafora disease wave discharges.
(C) UnverrichtLundborg disease (B) Bitemporal spikes are usually maximal
(D) Juvenile neuronal ceroid lipofuscinosis over the temporal frontal region. Ictal
EEG shows rhythmic theta at onset.
(E) Myoclonic epilepsy with ragged-red
fibers (C) Posterior 1.5 to 3 spikes and slow waves
per second discharges that usually atten-
81. What is the appropriate EEG pattern of idio- uate with eye opening.
pathic generalized epilepsy? (D) Hypsarrhythmia.
(E) Triphasic large-amplitude spikes maxi-
(A) Generalized epileptiform discharges of
mum in the centrotemporal area.
3 Hz maximum in the parasagittal
region. Normal background. Photic
84. What is the appropriate EEG pattern of benign
stimulation often induces occipital
rolandic epilepsy?
spikes.
(B) Bitemporal spikes are usually maximal (A) Slow, disorganized background. EEG
over the temporal frontal region. Ictal with superadded 1- to 2.5-Hz general-
EEG shows rhythmic theta at onset. ized interiorly predominant and slow-
(C) Posterior 1.5 to 3 spikes and slow waves wave discharges.
per second discharges that usually atten- (B) Bitemporal spikes are usually maximal
uate with eye opening. over the temporal frontal region. Ictal
(D) Hypsarrhythmia. EEG shows rhythmic theta at onset.
(E) Triphasic large-amplitude spikes maxi- (C) Posterior 1.5 to 3 spikes and slow waves
mum in the centrotemporal area. per second discharges that usually atten-
uate with eye opening.
(D) Hypsarrhythmia.
(E) Triphasic large-amplitude spikes maxi-
mum in the centrotemporal area.
Questions: 7990 161

85. What is the appropriate EEG pattern of benign 88. Seizure and impaired cognitive function occurs
occipital epilepsy? in GM1 gangliosidosis. The disorder is charac-
terized by
(A) Slow, disorganized background. EEG
with superadded 1- to 2.5-Hz general- (A) reduced N-acetylneuraminidase activity
ized interiorly predominant and slow- in leukocytes
wave discharges. (B) spongiform leukodystrophy with defi-
(B) Bitemporal spikes are usually maximal ciency in aspartoacylase
over the temporal frontal region. Ictal (C) polyglucosans found in the peripheral
EEG shows rhythmic theta at onset. muscle and liver
(C) Posterior 1.5 to 3 spikes and slow waves (D) sphingomyelinase activity is decreased
per second discharges that usually atten- in leukocytes
uate with eye opening. (E) reduced beta galactosidase activity in
(D) Hypsarrhythmia. leukocytes
(E) Triphasic large-amplitude spikes maxi-
mum in the centrotemporal area. 89. Seizure and impaired cognitive function occurs
in NiemannPick disease. The disorder is char-
86. What is the appropriate EEG pattern of acterized by
LennoxGastaut syndrome?
(A) reduced N-acetylneuraminidase activity
(A) Slow, disorganized background. EEG in leukocytes
with superadded 1- to 2.5-Hz general- (B) spongiform leukodystrophy with defi-
ized interiorly predominant and slow- ciency in aspartoacylase
wave discharges. (C) polyglucosans found in the peripheral
(B) Bitemporal spikes are usually maximal muscle and liver
over the temporal frontal region. Ictal (D) sphingomyelinase activity decreased in
EEG shows rhythmic theta at onset. leukocytes
(C) Posterior 1.5 to 3 spikes and slow waves (E) reduced beta galactosidase activity in
per second discharges that usually atten- leukocytes
uate with eye opening.
(D) Hypsarrhythmia. 90. Seizure and impaired cognitive function occurs
(E) Triphasic large-amplitude spikes maxi- in Gaucher disease. The disorder is character-
mum in the centrotemporal area. ized by
(A) reduced N-acetylneuraminidase activity
87. Seizure and impaired cognitive function occurs
in leukocytes
in GM2 gangliosidosis. The disorder is charac-
terized by (B) spongiform leukodystrophy with defi-
ciency in aspartoacylase
(A) reduced N-acetylneuraminidase activity (C) reduced beta glucocerebrosidase activity
in leukocytes in leukocytes
(B) spongiform leukodystrophy with defi- (D) sphingomyelinase activity decreased in
ciency in aspartoacylase leukocytes
(C) polyglucosans found in the peripheral (E) reduced beta galactosidase activity in
muscle and liver leukocytes
(D) sphingomyelinase activity is decreased
in leukocytes
(E) reduced hexosaminidase activity in
leukocytes
162 4: Neurophysiology, Epilepsy, Evoked Potentials, and Sleep Disorders

91. Seizure and impaired cognitive function occurs (C) reduced hexosaminidase activity in
sialidosis. The disorder is characterized by leukocytes
(A) reduced N-acetylneuraminidase activity (D) sphingomyelinase activity decreased in
in leukocytes leukocytes
(B) spongiform leukodystrophy with defi- (E) polyglucosans found in the peripheral
ciency in aspartoacylase muscle and liver
(C) reduced beta glucocerebrosidase activity
95. Seizure and impaired cognitive function occurs
in leukocytes
in globoid leukodystrophy. The disorder is
(D) sphingomyelinase activity decreased in characterized by
leukocytes
(E) reduced beta galactosidase activity in (A) defective lignoceroyl CoA
leukocytes (B) spongiform leukodystrophy with defi-
ciency in aspartoacylase
92. Seizure and impaired cognitive function occurs (C) reduced hexosaminidase activity in
in the presence of Lafora bodies. The disorder leukocytes
is characterized by (D) sphingomyelinase activity decreased in
(A) reduced N-acetylneuraminidase activity leukocytes
in leukocytes (E) reduced galactocerebroside beta-
(B) spongiform leukodystrophy with defi- galactosidase in leukocytes
ciency in aspartoacylase
96. Seizure and impaired cognitive function occurs
(C) reduced beta glucocerebrosidase activity
in Canavan disease. The disorder is character-
in leukocytes
ized by
(D) sphingomyelinase activity decreased in
leukocytes (A) defective lignoceroyl CoA
(E) polyglucosans found in the peripheral (B) spongiform leukodystrophy with defi-
muscle and liver ciency in aspartoacylase
(C) reduced hexosaminidase activity in
93. Seizure and impaired cognitive function occurs leukocytes
in metachromatic leukodystrophy. The disor- (D) sphingomyelinase activity decreased in
der is characterized by leukocytes
(A) reduced N-acetylneuraminidase activity (E) reduced galactocerebroside beta-
in leukocytes galactosidase in leukocytes
(B) spongiform leukodystrophy with defi-
ciency in aspartoacylase 97. Which of the following is an X-linked disorder
characterized by abnormal copper metabolism?
(C) reduced beta glucocerebrosidase activity
in leukocytes (A) MELAS syndrome (myopathy,
(D) sphingomyelinase activity is decreased encephalopathy, lactic acidosis, stroke)
in leukocytes (B) Nonketotic hyperglycinemia
(E) reduced arylsulfatase activity (C) Wilson disease
(D) Porphyria
94. Seizure and impaired cognitive function occurs (E) Menkes disease
in adrenoleukodystrophy. The disorder is char-
acterized by
(A) defective lignoceroyl CoA
(B) spongiform leukodystrophy with defi-
ciency in aspartoacylase
Questions: 91105 163

98. Which of the following disorders is character- 103. Which of the following causes of seizure carries
ized by the formation of KayserFleischer rings? the best prognosis after surgical resection?
(A) MELAS syndrome (myopathy, (A) Hippocampal sclerosis
encephalopathy, lactic acidosis, stroke) (B) Large arteriovenous malformation
(B) Nonketotic hyperglycinemia (C) Gross cortical dysplasia
(C) Wilson disease (D) Small low-grade glioma
(D) Porphyria (E) Trauma
(E) Menkes disease
104. Which of the following polysomnographic
99. Which of the following is characterized by patterns is observed in REM sleep behavior
reduced activity of glycine cleavage enzyme? disorder?
(A) MELAS syndrome (myopathy, (A) Short sleep latency, excessive disruption
encephalopathy, lactic acidosis, stroke) of sleep with frequent arousals, reduced
(B) Nonketotic hyperglycinemia total sleep time, excessive body move-
(C) Wilsons disease ment, and reduced slow-wave sleep
(D) Porphyria (B) Reduction of slow-wave sleep, reduced
REM and total sleep time, increased
(E) Menkes disease
sleep latency, and increased number of
awakenings during sleep
100. Which of the following is characterized by a
respiratory chain enzymes defect? (C) Reduced total sleep time, decreased
REM and slow-wave sleep, reduction of
(A) MELAS syndrome (myopathy, sleep spindle and K complexes, and
encephalopathy, lactic acidosis, stroke) increased sleep fragmentation
(B) Nonketotic hyperglycinemia (D) Absence of muscle atonia during REM
(C) Wilson disease sleep, with increased muscle tone activ-
(D) Porphyria ity in upper and lower limbs
(E) Menkes disease (E) Periodic limb movements

101. Which of the following is characterized by 105. Which of the following polysomnographic pat-
exacerbation of seizures when carbamazepine terns is observed in multiple system atrophy?
is used? (A) Short sleep latency, excessive disruption
(A) MELAS syndrome (myopathy, of sleep with frequent arousals, reduced
encephalopathy, lactic acidosis, stroke) total sleep time, excessive body move-
(B) Nonketotic hyperglycinemia ment, and reduced slow-wave sleep
(C) Wilson disease (B) Reduction of slow-wave sleep, reduced
REM and total sleep time, increased
(D) Porphyria
sleep latency, and increased number of
(E) Menkes disease
awakenings during sleep
(C) Reduced total sleep time, decreased
102. Failure of oral contraception may be caused by
REM and slow-wave sleep, reduction of
which of the following antiepileptic drugs?
sleep spindle and K complexes, and
(A) Vigabatrin increased sleep fragmentation
(B) Carbamazepine (D) Absence of muscle atonia during REM
(C) Valproate sleep, with increased muscle tone activ-
(D) Gabapentin ity in upper and lower limbs
(E) Benzodiazepine (E) Periodic limb movements
164 4: Neurophysiology, Epilepsy, Evoked Potentials, and Sleep Disorders

106. Which of the following polysomnographic pat- 108. Which of the following polysomnographic pat-
terns is observed in restless leg syndrome? terns is observed in Alzheimer disease?
(A) Short sleep latency, excessive disruption (A) Short sleep latency, excessive disruption
of sleep with frequent arousals, reduced of sleep with frequent arousals, reduced
total sleep time, excessive body move- total sleep time, excessive body move-
ment, and reduced slow-wave sleep ment, and reduced slow-wave sleep
(B) Reduction of slow-wave sleep, reduced (B) Reduction of slow-wave sleep, reduced
REM and total sleep time, increased REM and total sleep time, increased
sleep latency, and increased number of sleep latency, and increased number of
awakenings during sleep awakenings during sleep
(C) Reduced total sleep time, decreased (C) Reduced total sleep time, decreased
REM and slow-wave sleep, reduction of REM and slow-wave sleep, reduction of
sleep spindle and K complexes, and sleep spindle and K complexes, and
increased sleep fragmentation increased sleep fragmentation
(D) Absence of muscle atonia during REM (D) Absence of muscle atonia during REM
sleep, with increased muscle tone activ- sleep, with increased muscle tone activ-
ity in upper and lower limbs ity in upper and lower limbs
(E) Periodic limb movements (E) Periodic limb movements

107. Which of the following polysomnographic pat- Questions 109 through 115
terns is observed in narcolepsy?
Link each EEG pattern to the appropriate description.
(A) Short sleep latency, excessive disruption
(A) Hepatic failure
of sleep with frequent arousals, reduced
total sleep time, excessive body move- (B) Hypsarrhythmia
ment, and reduced slow-wave sleep (C) K complexes
(B) Reduction of slow-wave sleep, reduced (D) Stage I of sleep
REM and total sleep time, increased (E) Absence seizures
sleep latency, and increased number of (F) Porencephaly
awakenings during sleep (G) Normal EEG pattern at the conceptional
(C) Reduced total sleep time, decreased age of 29 weeks
REM and slow-wave sleep, reduction of
sleep spindle and K complexes, and
increased sleep fragmentation
(D) Absence of muscle atonia during REM
sleep, with increased muscle tone activ-
ity in upper and lower limbs
(E) Periodic limb movements
109. Fig. 4-1; EEG 1

Fp2-F8

F8-T4

T4-T6

T6-02

Fp1-F7

F7-T3

T3-T5

T5-01

T10-T4

T4-C4

C4-Cz

CZ-C3

C3-T3

T3-T9

Fp2-F4

F4-C4

C4-P4

Fp1-F3

F3-C3

C3-P3

Fz-Cz

CZ-Pz

EK1-EK2
TCP
60 Hz Notch LF = 1.0 HF = 70 100 uV 10 S
165
166

110. Fig. 4-2; EEG 2

A A
EKG-RE C C E A
500 80 Hz 1 Q Q Y C
5.0 15.0 3 3 E Q
: : S 3
:
H H C
F7-F8 I R L H
50 C O I
5.0 6.0 C 1 S C
U 4 E C
RE-A2 P 9 D U
S P
S
LE-A1

T5-T6
50
5.0 70.0
Fp2-T4

T4-02

Fp1-T3

T3-01

Fp2-C4

C4-02

Fp1-C3

C3-01

A2-T4

T4-C4

C4-Tz

Cz-C3

C3-T3

T3-A1

NEONATAL 60 Hz Notch LF = 1.0 HF = 70 150 uV 20 S


111. Fig. 4-3; EEG 3

T4-F8
F8-Fp2

Fp2-Fp1

Fp1-F7
F7-T3

A2-T4
T4-C4
C4-Cz

Cz-C3

C3-T3
T3-A1

T4-T6

T6-02

02-01

01-T5
T5-T3
F4-Fz

Fz-F3
P4-Pz

Pz-P3

Pg2-Pg1
80 Hz
1.0 70.0

EKG-X
500 50 Hz 1
1.0 70.0
CIRCLE LF = 1.0 HF = 70 300 uV 10 S
167
168

112. Fig. 4-4; EEG 4

Fp2-F8

F8-T4

T4-T6

T6-02

Fp1-F7

F7-T3

T3-T5

T5-01

A2-T4

T4-C4
C4-Cz

Cz-C3

C3-T3

T3-A1

Fp2-F4
F4-C4

C4-P4

Fp1-F3
F3-C3

C3-P3

EkG-X 1
70 1
5.0 70.0
Pg2-Pg1
100
TCP 60 Hz Notch LF = 1.0 HF = 70 70 uV 10 S
113. Fig. 4-5; EEG 5

Fp2-A2

F8-A2
T4-A2

T6-A2

Fp1-A1

F7-A1

T3-A1

T5-A1

F4-A2

C4-A2

P4-A2

O2-A2

F3-A1

C3-A1

P3-A1

O1-A1

Fz-A1

Cz-A1

Pz-A1

Az-A1

EKG-X1
400 1
5.0 70.0
Pg2-Pg1
100
REF EAR 60 Hz Notch LF = 1.0 HF = 70 150 uV 10 S
169
170

114. Fig. 4-6; EEG 6

Fp2-F8

F8-t4

T4-T6

T6-02

Fp1-F7
F7-T3

T3-T5

T5-01

A2-T4

T4-C4
C4-Cz

Cz-C3

C3-T3

T3-A1

Fp2-F4
F4-C4
C4-P4

Fp1-F3

F3-C3

C3-P3

EKG-X1
500 60 Hz
1.0 70.0

Pg2-Pg1
100 60 Hz
1.0 70.0 60 Hz Notch LF = 1.0 HF = 70 70 uV 10 S
TCP
115. Fig. 4-7; EEG 7

Fp2-A2

F8-A2

T4-A2

T6-A2
Fp1-A1

F7-A1

T3-A1

T5-A1

F4-A2

C4-A2

P4-A2

02-A2

F3-A1

C3-A1

P3-A1

01-A1

Fz-A1

Cz-A1

Pz-A1

A2-A1

EKG-X1
500 1
5.0 70.0 60 Hz Notch LF = 1.0 HF = 70 100 uV 10 S
REF EAR
171
172 4: Neurophysiology, Epilepsy, Evoked Potentials, and Sleep Disorders

116. Tetrodotoxin (C) Hair follicle


(D) Nuclear bag fiber
(A) blocks the opening of voltage-gated
sodium channels (E) Merkel cell
(B) inactivates voltage-gated potassium
121. Gap junction channels
channels
(C) prolongs the duration of action potential (A) provide high-resistance synaptic trans-
(D) prolongs the opening of voltage-gated mission
potassium channels (B) provide the ultrastructural components
(E) prolongs the opening of voltage-gated of electrical synapses
sodium channels (C) are usually involved in a unidirectional
synapse
117. Retrograde axonal transport (D) cause significant delay in synaptic trans-
mission
(A) occurs at a speed rate of 1 mm/day
(E) activate chemical transmitters in the
(B) is based on the action of kinesin
synaptic transmission
(C) carries actin from the cell body toward
axonal terminals
122. The process by which photons are detected and
(D) is used by the rabies virus to reach the the information is transduced into an electro-
neural cell body chemical signal is performed by
(E) is used by radioactively labeled amino
acids when injected into neuronal cells (A) rods
body to be transported to the correspon- (B) ganglion cells
ding axons (C) bipolar cells
(D) horizontal cells
118. Astrocyte cells (E) amacrine cells
(A) propagate action potential
123. Rods
(B) are derived from the mesoderm
(C) are responsible for neuron myelination (A) have a resting potential of 70 mV
in the central nervous system (B) are hyperpolarized in response to light
(D) are able to become phagocytic scav- stimulus
engers in the central nervous system (C) release epinephrine in response to light
(E) provide a pathway for neuronal migration (D) are responsible for color vision
(E) are the only photoreceptors present in
119. Merkel cells the fovea
(A) are activated by motion
124. Slow-twitching fibers
(B)are crucial to reading Braille
(C) are rapidly adapting receptors (A) contract with a higher level of force than
(D) Ia fibers are the corresponding afferent fast-twitching fibers
fibers (B) are resistant to fatigue
(E) respond to joint position (C) contain large stores of glycogen
(D) are the predominant muscle fibers in the
120. Which of the following receptors is a proprio- gastrocnemius muscle
ceptive receptor? (E) use exclusively anaerobic metabolism
(A) Meissner corpuscule
(B) Pacinian corpuscle
Questions: 116131 173

125. Nerve cooling may cause 128. In recording a sensory nerve action potential,
the reduction of high-frequency (low-pass)
(A) an increase in nerve conduction velocity
filter from 10,000 to 500 Hz results in
(B) a dispersion of sensory nerve action
potential (A) reduction of sensory nerve action poten-
(C) a reduction in distal motor latency tial onset latency
(D) an increase in sensory nerve action (B) reduction of sensory nerve action poten-
potential amplitude tial amplitude
(E) a reduction in the amplitude of the com- (C) reduction of sensory nerve action poten-
pound muscle action potential tial peak latency
(D) reduction of sensory nerve action poten-
126. What is the effect of increasing a low-frequency tial duration
(high-pass) filter from 10 to 300 Hz on the (E) no effect on sensory nerve action poten-
response of the sensory nerve action potential? tial response
(A) Prolonged onset distal latency of sen-
129. In recording a compound muscle action poten-
sory nerve action potential response
tial, the reduction of high-frequency (low-pass)
(B) Reduction of sensory nerve action filter from 10,000 to 500 Hz results in
potential amplitude
(C) Prolongation of sensory nerve action (A) reduction in compound muscle action
potential response peak latency potential amplitude
(D) An increase in the negative spike dura- (B) an increase in compound muscle action
tion of sensory nerve action potential potential amplitude
response (C) reduction in compound muscle action
(E) No effect on the morphology, duration, potential onset latency
and latencies of the sensory nerve action (D) prolongation of compound muscle
potential response action potential onset latency
(E) no effect on compound muscle action
127. In recording a compound muscle action poten- potential response
tial, the increase of low-frequency (high-pass)
filter from 1 to 100 Hz results in 130. F latency response depends on the patients
(A) reduction of onset latency of compound (A) age
muscle action potential (B) gender
(B) prolongation of onset latency of com- (C) weight
pound muscle action potential (D) height
(C) reduction of compound muscle action (E) temperature
potential amplitude
(D) prolongation of compound muscle 131. The cooling of a single myelinated fiber results
action potential peak latency in
(E) no effect on compound muscle action
(A) increased nerve fiber excitability
potential amplitude or latencies
(B) reduced nerve fiber excitability
(C) increased nerve fiber transmembrane
resistance
(D) reduced nerve fiber transmembrane
resistance
(E) no effect on nerve fiber excitability and
transmembrane resistance
174 4: Neurophysiology, Epilepsy, Evoked Potentials, and Sleep Disorders

132. As the nerves temperature declines, 134. Which of the following is true about the waves
illustrated in Figure 4-8?
(A) action potential amplitude increases
(B) action potentials rise and fall times (A) They are obtained by submaximal long-
declines duration (1-ms) nerve stimulation.
(C) nerve conduction velocity is reduced (B) Sensory nerve fibers constitute the affer-
(D) absolute refractory period decreases ent pathway that generates these waves.
(E) relative refractory period decreases (C) Their latencies are specifically affected in
S1 radiculopathy.
133. In mammalian nerves, decreasing temperature (D) They may be absent or have prolonged
latency in acute inflammatory demyeli-
(A) significantly alters sodium channel acti- nating polyneuropathy.
vation
(E) They are the electrical correlate of ankle
(B) significantly slows sodium channel inac- jerk.
tivation
(C) significantly alters potassium channel
activation
(D) significantly slows potassium channel
inactivation
(E) significantly alters calcium channel acti-
vation

L ULNAR

5 mV

Vertical Cursors
F min 24.00 ms 480.0 V

F min

FIG. 4-8
Questions: 132136 175

Vertical Cursors
H Lat 25.00 ms 35.0 V

FIG. 4-9

135. Which of the following is true about the waves 136. Figure 4-10 illustrates a motor nerve conduc-
illustrated in Figure 4-9? tion study of a median nerve, stimulating at
the wrist and the elbow and recording at the
(A) They are obtained by supramaximal
abductor pollicis brevis. The most likely diag-
nerve stimulation.
nosis suggested by Figure 4-10 is
(B) Motor nerve fibers are the afferent path-
way that generate these waves. (A) carpal tunnel syndrome
(C) They can be obtained from sensory (B) congenital demyelinating polyneuropa-
nerves. thy
(D) They are usually preserved in S1 radicu- (C) acquired demyelinating polyneuropathy
lopathy. (D) axonal polyneuropathy
(E) They are the electrical correlate of ankle (E) myasthenia gravis
jerk.

R MEDIAN - APB

1 3 5 Wrist 1
4 50ms 5mV 71 mA
2

1 3
Elbow 2 5
4
50ms 5mV 65 mA
Dist Vel Lat Amp
Segments cm m/s ms mV
Wrist - APB 8 10.35 2.6
Elbow - Wrist 23 20.8 21.40 2.6

FIG. 4-10
176 4: Neurophysiology, Epilepsy, Evoked Potentials, and Sleep Disorders

137. Figure 4-11 illustrates a motor nerve conduc- R ULNAR - ADM

tion study of a ulnar nerve, stimulating at the


wrist and the elbow and recording at the 2
abductor digiti minimi. The most likely diag-
nosis suggested by Figure 4-11 is 1 3 5 1
Wrist
4 30ms 5mV 100mA
(A) carpal tunnel syndrome
(B) congenital demyelinating polyneuropa-
thy
2
(C) acquired demyelinating polyneuropathy 1 3 5 B.Elbow 2
4
(D) axonal polyneuropathy
50ms 5mV 100mA
(E) myasthenia gravis
2 5
1 3 A.Elbow 3
138. Figure 4-12 illustrates a motor nerve conduc- 4

tion study of a median nerve, stimulating at 50ms 5mV 81mA


the wrist and the elbow and recording at the Dist Vel Lat Amp
abductor pollicis brevis. The most likely diag- Segments cm m/s ms mV
Wrist - ADM 8 3.40 3.2
nosis suggested by Figure 4-12 is
B.Elbow - Wrist 21 18.9 14.50 1.3
(A) carpal tunnel syndrome A.Elbow - B.Elbow 13 16.5 22.40 1.1

(B) congenital demyelinating polyneuropa- FIG. 4-11


thy
(C) acquired demyelinating polyneuropathy
(D) axonal polyneuropathy
(E) myasthenia gravis

R MEDIAN - APB

1 3 Wrist 1
5
4 50ms 5mV 32mA

1 3 5 Elbow 2
4
50ms 5mV 75mA
Lat Amp
Dist Vel ms mV
Segments cm m/s
8.70 5.6
Wrist - APB 8
13.25 5.1
Elbow - Wrist 21 46.2

FIG. 4-12
Answers and Explanations

1. (E) The posterior dominant rhythm is located antihistamines, and neuroleptics may enhance
over the occipital, parietal, and posterior tem- beta activity in routine EEG. Valproic acid does
poral regions of both hemispheres. The typical not modify beta activity. (Aminoff, 43)
alpha frequency is 8 to 13 Hz, usually 9 to 11 Hz
in adults. Age affects alpha frequency. A reac- 4. (C) Stage II sleep has all the features of stage I
tive posterior dominant rhythm first appears at sleep with the addition of sleep spindles and K
the age of 3 to 4 months. The typical frequency complexes. Sleep spindles can be located cen-
at that age is 3.5 to 4.5 Hz. trally at a frequency of 14 to 15 Hz. They can be
frontally located, especially in younger individ-
Frequency of Posterior Dominant uals, often at a frequency of 12 Hz. Spindles first
Rhythm by Age appear at the age of 2 months at a frequency of
8 to 12 Hz. They are asynchronous before the
Age Frequency in Hz age of 18 months. They later become symmetric
34 months 3.54.5 in amplitude and frequency. They become less
1 year 57 frequent with aging. They are seen in stages II
2 years 68
3 years 79 and III of sleep.
79 years 9 K complexes are diphasic waves, which
Midteens 10 consist of an initial sharply contoured transient
(Spehlman, 170173) followed by high-amplitude slow waves, usu-
ally of delta frequency, maximal in the fronto-
2. (E) Mu rhythm is a centrally located alpha fre- central region. They are seen in sleep stages II to
quency activity that diminishes with move- IV. They appear at the age of 5 months. Vertex
ment of the contralateral arm or body. Tactile sharp transients can be seen in the deeper por-
stimuli and the thought of movement can also tion of stage I of sleep and in stages II and III of
attenuate mu activity. Its morphology is often sleep. They are diphasic sharp transients max-
arch-shaped and the rhythm can be unilateral imal centrally at C3 and C4 with prominent
or bilateral. Mu activity may slow with aging. negative phase reversal at the midline in the
It should be considered abnormal if it is per- coronal bipolar montages. They are first seen at
sistent or nonreactive in a region of focal slow- the age of 2 months with a synchronous and
ing. (Spehlman, 191192) symmetric appearance. A consistent lateraliza-
tion of vertex waves is abnormal. Positive occip-
3. (B) Beta rhythm is defined by a frequency ital sharp transients are surface-positive,
greater than 13 Hz. It is best seen when the bisynchronous sharp transients occasionally fol-
subject is relaxed or drowsy. Beta activity can lowed by low-amplitude surface negativity.
increase with cognition or stage I sleep, espe- They occur in stages I or II, predominantly in
cially in children. Beta activity is enhanced by the occipital region singly or in runs. Alpha
the use of barbiturates, benzodiazepines, and attenuation and increased frontocentral beta are
chloral hydrate. Occasionally, antidepressants, features of stages I and II. (Spehlman, 203204)

177
178 4: Neurophysiology, Epilepsy, Evoked Potentials, and Sleep Disorders

5. (B) Intermittent photic stimulation is used as 10. (E) By 44 to 46 weeks of conceptional age, the
an activating procedure in the diagnostic eval- trace discontinue and trace alternant gradually
uation of epilepsy. A photomyoclonic response yield to more mature and continuous slow-
occurs frontally when the flashing light evokes wave sleep during which nonstop high voltage
muscle contraction of the facial musculature and activity dominate the tracing. (Spehlman,
about 50 to 60 ms after each flash. The ampli- 159170)
tude of this muscle artifact increases as photic
stimulation continues. The response can be 11. (D) Lombroso proposed an operational defi-
enhanced in case of barbiturate or ethanol nition of interhemispheric synchrony that has
withdrawal. Photic stimulation rates from been widely adopted. In a synchronous burst,
about 12 to 18 Hz are most effective in pro- the signal begins in both hemispheres within
ducing a photomyoclonic response. (Spehlman, 1.5 seconds. Excessive interhemispheric asyn-
448449) chrony of burst activity may be seen in prema-
turity, diffuse encephalopathy, and cerebral
6. (E) Sleep spindles consist of sinusoidal waves, dysgenesis. (Lombroso, 460474)
from 12 to 15 Hz, which develop in the central
regions during the first few weeks of life and 12. (A) Delta brushes are the quintessential wave-
are well established by the age of 6 to 8 weeks. forms of prematurity. The brush represents a
Spindle bursts are commonly asynchronous in pattern of the simultaneous fusion of two sub-
the two hemispheres until the age of 2 years. units, the underlying delta transient and the
(Aminoff, 108) superimposed rhythmic fast activity. Brushes
occur during wakefulness, active sleep, and
7. (A) The neonatal EEG is characterized by an quiet sleep. At a very premature age, brushes are
age-specific bioelectric rhythm. Between 24 and more common in the rolandic region. When they
28 weeks of conceptional age, the entire EEG is achieve their highest expression, between 32 and
essentially discontinuous. Whether the child is 34 weeks of conceptional age, brushes are seen in
clinically awake or asleep, active or quiet, the the occipital, temporal, and rolandic regions.
EEG itself is composed of discontinuous back- Frontal brushes are rare at any age. By the age of
ground. Stimulation of the newborn or changes 1 month after term, brushes largely disappear as
in spontaneous state provoke minor qualita- greater maturity emerges. (Aminoff, 86)
tive changes in the discontinuity of the back-
ground. (Spehlman, 159170) 13. (D) By 44 to 46 weeks of conceptional age, quiet
sleep sheds its immature trace alternant pat-
8. (E) After 32 weeks of age, EEG discontinuity is tern and establishes its mature foundation as
mostly confined to quiet sleep. The voltage of continuous slow-wave sleep. Sleep spindles
the interburst activity remains less than 25 V. appear from the midline or the central region.
With advancing maturity, the voltage of the They are asynchronous between the two hemi-
interburst period exceeds 25 V. The label spheres and remain so until the age of 2 years.
assigned to this more mature discontinuous (Spehlman, 175)
quiet sleep activity is trace alternant. It is seen
by the age of 36 weeks. The distinction between 14. (D) A full-term neonate expresses the follow-
trace discontinue and trace alternant is arbi- ing behavioral sleep states:
trary except for the higher interburst voltage.
1. Waking state: the full-term neonate spends
(Spehlman, 159170)
one-third of the time in this state. It is char-
acterized by eye opening, body and facial
9. (E) Activit moyenne appears during wakeful-
movements, muscle activity, and irregular
ness, by 36 to 40 weeks of gestational age. It
breathing.
is formed by a continuous low- to medium-
2. Active sleep: the full-term neonate spends
voltage mixed-frequency signal. (Spehlman,
50% to 60% of the time in this state. It is
159170)
Answers: 521 179

characterized by closed eyes, most of the that occur singly or in brief trains over the tem-
time with occasional opening, slower facial poral region. Temporal transients are usually
and body movements, decreased muscle maximal over the mid-Sylvian area. They are
tone, and irregular respiration. seen in normal subjects after the age of 40, with
3. Quiet sleep represents 30% to 40% of sleep left-side preponderance, and occur in about
time with regular respiration, few body and 35% of individuals between the fifth and sixth
facial movements, and muscle tone similar decades. They appear to be related to a normal
to that of the waking state. aging process, although some studies suggest
4. Transitional sleep. cerebrovascular ischemia as the origin of this
activity. (Spehlman, 215; Aminoff, 4243, 109)
(Spehlman, 165170)

20. (E) Triphasic waves are 100- to 300-V positive


15. (C) The photomyogenic response is a non-
sharp waves preceded and followed by lower-
cerebral response characterized by brief repet-
amplitude negative waves. They are bilaterally
itive muscle spikes in the frontal leads with the
synchronous, generalized in distribution, and
same frequency as the flash stimulus. This
usually predominant in the frontal region.
may be associated with fluttering and twitch-
Triphasic waves occur more commonly from
ing movements of the forehead and eyelid
hepatic encephalopathy than from other causes.
muscles. The photomyogenic response shows
They may occur in hypoglycemia, renal failure,
recruitment and ceases when the photic stim-
hyponatremia, hypercalcemia, hyperthyroidism,
ulus stops. It is most prominent when the eyes
drug intoxication, and anoxia. There is no spe-
are closed but can occur when the eyes are
cific EEG finding in hypoparathyroidism. EEG
open. It is believed by some to represent a
may show irregular high-voltage delta activity
physiological response to high-intensity light.
that is increased on hyperventilation.
(Aminoff, 5354)
Paroxysmal abnormalities may also occur
during wakefulness and sleep. (Spehlman, 327)
16. (B) Alpha activity is an activity in the range of
8 to 13 Hz, whereas alpha rhythm is activity in
21. (D) Patients with anoxic encephalopathy
the same range occurring during wakefulness
exhibiting the following EEG patterns are
over the posterior head region; it is present
gravely ill and may have a poor prognosis.
when the eyes are closed and the patient relaxed
and is blocked by eye opening or alerting the The burst suppression pattern consists of
patient. (Spehlman, 215; Aminoff, 4243, 109) generalized high-voltage, mixed-frequency
waveforms of variable duration, usually
17. (D) The Bancaud phenomenon results from with admixed spike and sharp waves.
failure of alpha rhythm to block on one side of The periodic spikes or sharp waves are usu-
the brain. It indicates an abnormality on the ally seen at a rate of 0.5 to 2 Hz. They are
side that fails to attenuate. (Spehlman, 215; often associated with multifocal or bilateral
Aminoff, 4243, 109) myoclonus.
The alpha coma pattern in cerebral anoxia
18. (E) Posterior slowing of youth consists of consists of monotonous unreactive alpha
single 2- to 4-Hz triangular-contoured slow activity with diffuse frontal predominance.
waves (also called sail waves) interspersed This pattern carries a poor prognosis in
with alpha activity over the posterior head anoxic encephalopathy.
regions and occurring maximally over the A pattern of bihemispheric periodic lateral-
occipital head regions. (Spehlman, 215; Aminoff, ized epileptiform discharges also carries a
4243, 109) poor prognosis. About 60% of patients show-
ing this pattern do not recover.
19. (C) Temporal transients consist of episodic Delta brushes are seen in prematurity.
slow-wave components ranging from 2 to 5 Hz
(Spehlman, 389392; Prior, 770)
180 4: Neurophysiology, Epilepsy, Evoked Potentials, and Sleep Disorders

22. (A) Dialysis dementia is a progressive condi- 25. (E) Electrocerebral inactivity or electrocerebral
tion that develops in patients on chronic silence is defined as no EEG activity of more
hemodialysis. Characterized by confusion, pro- than 2 V in recording from scalp electrode
gressive dementia, dysarthria, myoclonus, and pairs 10 cm or more apart with interelectrode
seizure, it is caused by the toxic accumulation impedances less than 10,000 but more than 100
of aluminum in the brain. Hughes and col- ohms. Ten guidelines for EEG recordings are
leagues found bilateral spike-and-wave com- recommended:
plexes in 77% of patients with dialysis dementia.
1. A minimum of eight scalp electrodes
Based on the presence or absence of that pattern,
should be utilized.
91% of the patients, along with 91% of their
2. Interelectrode impedances should be less
EEGs, were correctly placed by Hughes and
than 10,000 but more than 100 ohms.
coworkers into the clinical category of dialysis
3. The integrity of the entire recording system
dementia either with or without chronic
should be tested.
encephalopathy. (Hughes and Schreeder, 11481154)
4. Interelectrode distances should be at least
10 cm.
23. (B) Benzodiazepines acutely induce wide-
5. Sensitivity must be increased from 7 V/
spread fast activity with a maximum in the cen-
mm to at least 2 V/mm for at least 30
tral and frontal regions. This persists during
minutes of the recording, with inclusion of
wakefulness until stages I and II of sleep and
appropriate calibrations.
becomes more conspicuous when the alpha
6. Filter settings should be appropriate for
rhythm disappears. (Spehlman, 428)
the assessment of electrocerebral silence.
7. Additional monitoring techniques should
24. (B) Rett syndrome, a neurodegenerative dis-
be employed when necessary.
order of unknown cause, occurs exclusively in
8. There should be no EEG reactivity to
girls and has a prevalence of approximately 1
intense somatosensory, auditory, or visual
per 15,000 to 22,000. The hallmarks of Rett
stimuli.
syndrome are repetitive hand-wringing move-
9. Recordings should be made only by a qual-
ments and loss of purposeful and spontaneous
ified technologist.
use of the hands; these may not appear until
10. A repeat EEG should be performed if there
2 to 3 years of age. Autistic behavior is a typ-
is doubt about electrocerebral silence.
ical finding in all patients. Generalized
tonicclonic convulsions occur in the majority (Aminoff, 688)
and are usually well controlled by anticon-
vulsants. The EEG in Rett syndrome has been 26. (B) Hockaday and coworkers studied the EEG
extensively studied. In stage 1 of the disease, of patients with anoxic encephalopathy and
the corresponding EEG is normal or may classified the abnormalities into five grades
show minimal slowing of the background. In according to the final outcome. The prognosis
stage 2, a rapidly destructive stage, back- for patients belonging to grade 1 was favorable.
ground slowing of the EEG is evident. Focal Severity increases and prognosis worsens from
spikes or sharp waves appear, most com- grade 1 to grade 5. In grade 1, the EEG shows
monly over the centroparietal regions but reactive alpha rhythm with scattered theta
sometimes bilateral, multifocal, or diffuse. By activity. In grade 2, the EEG shows dominant
stage 3, or the plateau stage, diffuse intermit- theta activity with rare alpha activity. In stage 3,
tent 2- to 4-Hz slow waves are superimposed there are continuous polymorphic slow delta
on background slowing. In stage 4, the motor waves with little activity and faster frequency.
deterioration stage, the EEG is dominated by In stage 4, there is invariant low-amplitude
monorhythmic 4- to 5-Hz activity; there are delta activity unresponsive to stimulus or any
frequent multifocal and generalized slow activity with a suppression interval of 1 second
spike wave discharges during both waking or more. Stage 5 shows a nearly or completely
and sleep. (Behrman, 2034; Verma et al., 395401) isoelectric record. (Hockaday et al., 575)
Answers: 2233 181

27. (A) Alpha coma in patients with brainstem raised gently, the globes are usually seen to be
lesions is most commonly seen in vascular dis- exotropic and the pupils are even smaller than
ease, with lesions noted in the ventral pons, spar- before, but with retained responses to light. An
ing the brainstem tegmentum. This abnormal additional stage of the sleep cycle, which fol-
alpha rhythm tends to be more reactive than lows the others intermittently throughout the
that seen after hypoxia or drug overdose and is night, is associated with further reduction in
most prominently seen over the posterior head muscle tone except in the extraocular muscles
region. (Spehlman, 438) and with bursts of REMs; thus the term REM
sleep designates this stage. The EEG becomes
28. (E) Five stages of sleep, representative of two desynchronized (i.e., it has a low-voltage, high-
alternating physiological mechanisms, have frequency discharge pattern). The first three
been defined. In each stage, the electrical activ- stages of sleep are called NREM sleep or syn-
ity of the brain occurs in organized and recur- chronized sleep; the last stage, in addition to
ring cycles, referred to as the architecture of REM sleep, is variously designated as fast-wave,
sleep. As the electrophysiological stages of sleep nonsynchronized, or desynchronized sleep.
progress, sleep becomes deeper, meaning that (Ropper and Samuels, Chapter 19)
arousal requires a more intense stimulus.
Relaxed wakefulness with the eyes closed is 29. (B) Positive occipital sharp transients of sleep
accompanied in the EEG by posterior alpha are monophasic triangular waves in the occip-
waves of 9 to 11 Hz (cycles per second) and ital regions. They may appear at the end of
intermixed low-voltage fast activity of mixed stage 1 of sleep in many subjects. (Spehlman,
frequency. Except for the facial muscles, the 191197)
EMG is silent when the patient is sitting or lying
quietly. With drowsiness, as the first stage of 30. (E) Benign epileptiform transients of sleep are
sleep sets in, the eyelids begin to droop, the eyes small sharp spikes seen mainly in adults
may rove slowly from side to side, and the during drowsiness. Unlike true epileptiform
pupils become smaller. As the early stage of discharges, they do not distort the background
sleep evolves, the muscles relax and the EEG activity. They disappear in deeper levels of
pattern changes to one of progressively lower sleep. (Spehlman, 191197)
voltage and mixed frequency with a loss of
alpha waves; this is associated with slow, rolling 31. (D) Wicket spikes are arc-like waves that typ-
eye movements and is called stage 1 sleep. As ically occur in train. When they occur as a
this changes into stage 2 sleep, 0.5- to 2-s bursts single waveform, they are often mistaken for a
of biparietal 12- to 14-Hz waves (sleep spindles) temporal spike. They are not associated with a
and intermittent high-amplitude, central-pari- subsequent slow wave and do not disrupt the
etal sharp slow-wave complexes appear (vertex background. (Spehlman, 191197)
waves). The American Academy of Sleep
Medicine (AASM) recommends the following 32. (D) Lambda waves occur over the occipital
staging: stage W (wakefulness); stage N1 head regions when the subject is engaged
(nonrapid eye movements [NREM sleep] or actively at something that arouses his or her
NREM 1, formerly stage 1); stage N2 (NREM 2, interest. These waves appear to represent an
formerly stage 2); stage N3 (NREM 3, combining evoked cerebral response to a visual stimulus
former stages 3 and 4, or slow-wave sleep); and produced from shifts of images across the
stage R (REM sleep). The essential difference retina in the course of saccadic eye movements.
between this new nomenclature and the one (Spehlman, 191197)
familiar to most neurologists is that stage N3
now represents slow-wave sleep, replacing stage 33. (A) Mu rhythm has been called rolandic alpha
3 and stage 4 sleep, composed of an increasing activity and consists of 7- to 11-Hz arc-shaped
proportion of high-amplitude delta waves (0.75- waveforms, usually unilateral, present over
V, 0.5- to 2-Hz in the EEG). If the eyelids are the central region. It seems to be related to
182 4: Neurophysiology, Epilepsy, Evoked Potentials, and Sleep Disorders

functions of the sensorimotor cortex. It is not anorexia, and headache. In rare cases, behav-
attenuated by eye opening but by active move- ioral changes may be seen, including psychosis.
ment or the thought of movement of the con- Drugdrug interactions are minimal. (Pollard et
tralateral extremity. (Spehlman, 191197) al., 91105)

34. (B) Phenytoin toxicity may occur unexpectedly 38. (B) Some antiepileptic drugs are bound sub-
because its metabolism slows substantially stantially to serum proteins. Bound drug is not
when a serum concentration of approximately relevant to drug effect and is thus considered
15 g/dL is reached due to zero-order kinetics. inactive, as it cannot cross the bloodbrain
In this state of slow metabolism, even small barrier. An interaction that displaces the drug
adjustments in dosage may lead to very large from its binding site immediately increases the
increases in serum concentration. (Pollard et al., active fraction of the drug. This can cause tran-
91105) sient toxicity. However, the free fraction is also
available for metabolism, which, therefore, will
35. (A) Phenytoin, because of its properties of occur at a higher rate. After several days of
hepatic enzyme induction and also its own equilibration, the total concentration of the dis-
metabolism via cytochrome P450, may produce placed drug, as measured by serum levels, is
drugdrug interactions. Some examples of clin- typically lower, but the free fraction is the same
ically significant interactions include the ability as before the drug was displaced. The most fre-
of dicumarol, chloramphenicol, sulfonamides, quently encountered example is the displace-
isoniazid, disulfiram, and cimetidine to pro- ment of phenytoin from serum proteins by
duce competitive or noncompetitive inhibition valproate. The addition of valproate to a
of hepatic metabolism and to increase serum chronic regimen of phenytoin will result in an
phenytoin levels, whereas rifampin and immediately higher free fraction of phenytoin.
antacids may decrease them. Phenytoin, in turn, The patient therefore can experience symptoms
may increase clearance of oral contraceptives, of phenytoin toxicity without a higher total
quinidine, chloramphenicol, and dicumarol. serum concentration of phenytoin. (Pollard et
(Pollard et al., 91105) al., 91105)

36. (E) Salicylates may alter protein binding, caus- 39. (C) The brainstem evoked potentials are sig-
ing an increase in free (active) phenytoin with- nals generated in the auditory nerve and brain-
out altering the total measured serum level. stem following an acoustic stimulus. A normal
(Pollard et al., 91105) brainstem evoked response has five waves.
Waves I and II correspond to the activation of
37. (C) Ethosuximide (Zarontin), a T-type calcium the distal and proximal parts of the auditory
channel blocker, is an antiepileptic agent with a nerve, respectively. Wave III corresponds to the
very narrow therapeutic indication. It is effec- activation of the superior olive and trapezoid
tive only as a first-line agent for absence body. The generators of wave IV and V are
seizures occurring in isolation, a condition seen located in the upper pons and midbrain.
primarily in childhood. Occasionally, in pri- (Aminoff, 421536)
mary generalized epilepsy with seizure types
other than absence where valproate acid is inef- 40. (B) The clinical interpretation of the pattern
fective, the addition of ethosuximide improves reversal visual evoked potential is based on
seizure control. It is in this setting that most measurement of the latency of the cortical
adult use of ethosuximide occurs. Ethosuximide response from the P100 component, the major
is primarily cleared through hepatic metabo- positive wave having a nominal latency of 100
lism, although a small portion (less than 20%) is ms in normal subjects. (Aminoff, 421536)
renally eliminated. The most common side
effects noted with ethosuximide use include 41. (A) Median nerve somatosensory evoked
nausea and abdominal discomfort, drowsiness, potentials show, in normal cases, N9 generated
Answers: 3447 183

at Erbs point; N14 generated in the cervi- microsomal enzymes. In naive patients, intro-
comedullary region, probably in the caudal duction of the drug is associated with a half-life
medial lemniscus; and N20 in the cortical area of 30 hours, decreasing within days to weeks to
processing the stimulus. (Aminoff, 421536) 10 to 20 hours. A low dose of the drug should
be introduced first, when starting the treat-
42. (D) Somatosensory evoked potential obtained ment; then it should be increased slowly to
from stimulation of the posterior tibial nerve avoid toxicity. Skin rash occurs in 15% of
shows N22 potential generated in the gray patients treated with carbamazepine, whereas
matter of the lumbar cord and P38 generated in it occurs in only 10% of those treated with
the cortical area close to the leg representation. phenytoin. Carbamazepine has a slight antidi-
(Aminoff, 421536) uretic effect, which may result in mild to mod-
erate hyponatremia. Mild elevation of hepatic
43. (B) Sleep spindles and K complexes are seen in enzymes may occur with carbamazepine use in
stage II or III of sleep. K complexes consist 5% to 10% of cases. This elevation neither pre-
simply of a sharp vertex wave associated with dicts nor predisposes to hepatitis, which is pre-
a sleep spindle. Positive occipital transients of sumed to be an immune hypersensitivity
sleep are seen in the deep stage I of sleep. In reaction. (Bazil and Pedley,1998, 135162)
stage III of sleep, delta slowing is seen in 20% to
50% of sleep. The enhancement of the ampli- 46. (B) Levetiracetam is not extensively metabo-
tude of beta activity in the presence of a skull lized in humans. The major metabolic pathway
defect is called a breach rhythm. The Bancaud is the enzymatic hydrolysis of the acetamide
phenomenon results from failure of the alpha group, which produces the carboxylic acid
rhythm to attenuate by eye opening. (Aminoff, metabolite (24%) and is not dependent on any of
4546, 54) the liver cytochrome P450 isoenzymes. The
major metabolite is inactive in animal seizure
44. (E) Nausea, vomiting, and gastrointestinal dis- models. Its plasma half-life in adults is 7 1
tress are among the most frequent adverse hour and is unaffected by either dose or
effects of valproic acid. Other common side repeated administration. It is eliminated from
effects include weight gain in 50% of cases, the systemic circulation by renal excretion as
hair loss, and a tremor in 10% of cases. unchanged drug, which represents 66% of the
Hyperammonemia may occur; it is usually administered dose. The mechanism of excretion
asymptomatic but can cause encephalopathy in is glomerular filtration with subsequent partial
the absence of hepatic dysfunction. Mild to tubular reabsorption. Levetiracetam elimination
moderate hepatic dysfunction may occur in is correlated with creatinine clearance.
40% of cases. It is dose-related and reversible. Levetiracetam clearance is reduced in patients
Fatal hepatitis may occur as an idiosyncratic with impaired renal function.
reaction, especially in children under the age of Zonisamide is 70% metabolized by the liver.
2 years who are on multiple antiseizure med- Topiramite is 30% metabolized by the liver,
ications. Acute pancreatitis can be induced by whereas lamotrigine and tiagabine are more
valproic acid in children more than adults. than 90% metabolized by the liver. (Patsalos,
Aplastic anemia is seen after treatment with 123129)
felbamate. (Bazil and Pedley, 1998, 135162; Dichter
and Brodie, 15311590) 47. (C) Phenytoin has zero-order kinetics. This
means that its process of utilization is not
45. (B) Carbamazepine has poor solubility in linear. The plasma drug concentration and
water, a factor that has inhibited the develop- clearance are independent of drug dose.
ment of a parenteral form for clinical use. Furthermore, in this case, one cannot reliably
Carbamazepine is metabolized by the liver. Its predict the time that will be required for pheny-
pharmacokinetics are characterized by the toin concentration to fall into the therapeutic
phenomenon of autoinduction of hepatic range. (Bazil and Pedley, 1998, 135162)
184 4: Neurophysiology, Epilepsy, Evoked Potentials, and Sleep Disorders

48. (B) The patient described in the vignette has lar dose by 50% is needed in cases of renal
early symptoms of carbamazepine intoxication. failure or dialysis. (Bazil and Pedley 2003,
The therapeutic level of carbamazepine is 3852)
explained by the phenomenon of autoinduc-
tion of the drug from its own metabolism by 51. (D) All these drugs are removed by dialysis
hepatic microsomal enzymes. The patient was except phenytoin. In case of phenytoin over-
started initially on a full dose of the drug, dose, forced fluid diuresis, peritoneal dialysis,
which caused signs of carbamazepine intoxi- exchange transfusion, hemodialysis, and
cation. Autoinduction of the drug decreases its plasmapheresis are ineffective. They produce
serum level to a therapeutic range and little renal elimination and pose a danger of
improves the clinical symptoms. It is recom- fluid overload. (Bazil and Pedley 2003, 3852)
mended that carbamazepine be started at a low
dose (100 mg PO bid) and increased progres- 52. (B) West syndrome is an age-dependent
sively. (Bazil and Pedley, 2003, 3852) epilepsy with a characteristic EEG pattern
known as hypsarrhythmia; it is also accompa-
49. (C) Lamotrigine was originally developed nied by infantile spasms and psychomotor
because of an observation that some anticon- retardation. Infantile spasms occur in 24 to 42
vulsants have antifolate properties. Although of 100,000 births. Spasms and psychomotor
lamotrigine weakly inhibits dihydrofolate retardation appear in the first year of life in
reductase, there is no correlation between 85% of cases, the majority between 3 and 7
this activity and its anticonvulsant action. months. Associated abnormalities include gen-
Lamotrigine is about 55% bound to protein. It eralized hypotonicity, microcephaly, paralysis,
undergoes extensive liver metabolism. Its rate ataxia, blindness, and deafness. The typical
of elimination is influenced by concomitant EEG reveals hypsarrhythmia, a chaotic pattern
administration of hepatic inhibitors or induc- of high-amplitude slow waves with multifocal
ers. When lamotrigine is given in monotherapy, epileptiform discharges and poor interhemi-
the elimination half-life is about 25 hours. spheric synchrony. ACTH is the treatment of
When it is given with enzyme inducers (carba- choice for infantile spasm. The dose and
mazepine, phenytoin, phenobarbital), its half- duration of steroid therapy has not been stan-
life is reduced to 15 hours. When given with an dardized. The most common treatment is
enzyme inhibitor (valproic acid), the half-life ACTH 40 IU/day administered intramuscu-
increases to 60 hours. Rash may complicate larly. Approximately 75% of patients achieve
lamotrigine administration in 10% of cases. It initial seizure control with this regimen. The
acts through prolonging inactivation of volt- response rate tends to increase with duration of
age-sensitive sodium channels. (Bazil and Pedley, therapy. Within 2 months of remission, how-
2003, 3852) ever, approximately 30% to 50% of patients
suffer relapse after the first course of steroids.
50. (C) Phenytoin has minimal renal excretion and (Goetz, 10701072)
does not require adjustment in case of renal
failure. Ethosuximide has renal excretion of 53. (E) Phenytoin can cause a range of dose-
10% to 20%. A decrease of 25% of the regular related and idiosyncratic adverse effects.
dose is needed if the glomerular filtration is Reversible cosmetic changes (gum hypertro-
less than 10 mL/min. Phenobarbital has a renal phy, acne, hirsutism, and facial coarsening),
excretion of 25% to 30%. The dosing interval although often mild, can be troublesome.
must be increased by 50% to 100% in cases of Neurotoxic symptoms (drowsiness, dysarthria,
renal failure. Carbamazepine has minimal renal tremor, ataxia, and cognitive difficulties) may
excretion, but its dose must be decreased by occur when the plasma drug concentration
25% in cases of renal failure with glomerular fil- exceeds 20 g/mL. Cerebellar atrophy may
tration less than 10 mL/min. Topiramate has occur with chronic use. (Brodie and Dichter,
renal excretion of 70%. A decrease in the regu- 168175; Dichter and Brodie, 15831590)
Answers: 4863 185

54. (C) The most common side effects of lamotrig- 58. (E) Zonisamide is a weak carbonic anhydrase
ine use are headache, nausea and vomiting, inhibitor that is probably responsible for an
dizziness, diplopia, and ataxia. Tremor can be increased incidences of symptomatic renal
troublesome at high dosages. In up to 10% of stones, although the rate was similar to placebo
patients in add-on trials, a rash may develop, in clinical trials. (Brodie and Dichter, 168175;
which subsequently disappears in some patients Dichter and Brodie, 15831590)
despite continued therapy. In a few patients,
however, the rash is more serious, and fever, 59. (A) Gabapentin is typically well tolerated. The
arthralgias, and eosinophilia occur. In rare cases most common side effects are headache, fatigue,
(less than 1%), StevensJohnson syndrome ataxia, and somnolence. (Brodie and Dichter, 168175;
develops. The concomitant administration of Dichter and Brodie, 15831590)
valproic acid with lamotrigine may increase the
likelihood of a serious rash; the gradual intro- 60. (D) The common side effects of valproic acid
duction of lamotrigine may lower the likelihood are dose-related tremor, weight gain due to
of skin reactions. (Brodie and Dichter, 168175; appetite stimulation, thinning or loss of hair
Dichter and Brodie, 15831590) (usually temporary), and menstrual irregular-
ities, including amenorrhea. Sedation is
55. (B) The main limitation of phenobarbital use is unusual, although stupor and encephalopathy
its propensity to alter cognition, mood, and occur in rare cases. The incidence of hepato-
behavior. The drug can cause fatigue in adults, toxic effects, histologically evident as a
and insomnia, hyperactivity, and aggression in microvesicular steatosis, is less than 1 in 20,000
children (and sometimes in the elderly). Memory, patients, but such effects are of concern in chil-
mood, and learning capacity may be subtly dren under 3 years of age who are receiving
impaired. Depression, arthritic changes, and multiple antiepileptic drugs. Approximately
Dupuytrens contracture can be associated prob- 20% of all patients receiving the drug have
lems. (Brodie and Dichter, 168175; Dichter and Brodie, hyperammonemia without hepatic damage.
15831590) This effect is usually asymptomatic but occa-
sionally can cause confusion, nausea, and vom-
56. (E) Carbamazepine can cause a range of idio- iting. (Brodie and Dichter, 168175; Dichter and Brodie,
syncratic reactions; the most common is a mor- 15831590)
billiform rash, which develops in about 10% of
patients. Less common skin eruptions include 61. (B) Carbamazepine, valproic acid, and pheny-
erythema multiforme and StevensJohnson syn- toin are not removed by dialysis. (Bazil and
drome. Reversible mild leukopenia is common Pedley, 2003, 3852)
but does not generally require discontinuation
of therapy. Blood dyscrasias and toxic hepatitis 62. (A) Tiagabine acts as an inhibitor of GABA
are rare. At high plasma concentrations, carba- reuptake into presynaptic nerve terminals.
mazepine has an antidiuretic hormonelike Because GABA is a major inhibitory neuro-
action; the resulting hyponatremia is usually transmitter in the central nervous system, the
mild and asymptomatic. But if the plasma increase of available GABA through this mech-
sodium concentration falls below 125 mmol/L, anism may be responsible for tiagabines anti-
there may be confusion, peripheral edema, and convulsant action. (Bazil and Pedley, 2003, 3852)
decreasing control of seizures. Orofacial dyski-
nesias and cardiac arrhythmias are additional 63. (B) The overall risk of having a baby with a
rare side effects. (Brodie and Dichter, 168175; Dichter major malformation is about 2% to 3% in
and Brodie, 15831590) healthy populations. This is increased to 5% to
6% in women with epilepsy taking a single
57. (B) Felbamate may cause aplastic anemia and antiepileptic drug, perhaps to 6% to 7% if the
hepatic toxicity. (Brodie and Dichter, 168175; drug is carbamazepine or valproic acid. This
Dichter and Brodie, 15831590) rate may be doubled by the use of two or more
186 4: Neurophysiology, Epilepsy, Evoked Potentials, and Sleep Disorders

drugs or by a very high plasma level of in mammals and is formed by GABA, glycine,
antiepileptic drugs. (Bazil and Pedley, 2003, 3852) taurine, and beta-alanine. (Cooper, 126)

64. (C) Many drugs may increase phenytoin levels. 69. (B) Hyperventilation is an activation proce-
Serum level determinations for phenytoin are dure used to bring out focal slowing or epilep-
especially helpful when possible drug interac- tiform abnormalities. The incidence and
tions are suspected. Drugs that may increase intensity of the hyperventilation response
phenytoin serum levels include alcohol (acute depend on age, blood sugar level, and cerebral
intake), amiodarone, chloramphenicol, chlor- oxygen supply. Hyperventilation responses are
diazepoxide, diazepam, dicumarol, disulfiram, most common and extensive in children and
estrogens, H2-antagonists, halothane, isoniazid, teenagers. In children, spike and wave dis-
methylphenidate, phenothiazines, phenylbuta- charges of 3 Hz are particularly seen in absence
zone, salicylates, succinimides, sulfonamides, seizures and are particularly sensitive to hyper-
tolbutamide, and trazodone. (Bazil and Pedley, ventilation; in many cases, they appear only
2003, 3852) during this procedure. The 8-year-old boy with
episodes of staring most likely has absence
65. (D) Theophylline, a CYP3A4 inducer, can seizures. Hyperventilation may be the only
increase carbamazepine metabolism, resulting procedure to induce spike and wave dis-
in the potential for decreased plasma carba- charges, considering this patients age and the
mazepine levels. (Bazil and Pedley, 2003, 3852) type of seizure that he is most likely to have.
(Spehlman, 219222)
66. (E) Barbiturates and benzodiazepines are
potent beta activators. Chloral hydrate is 70. (D) REM parasomnias usually occur in middle-
another activator of beta activity. Occasionally, aged or elderly patients and show a marked
neuroleptics, antidepressants, and antihista- male predominance. They are due to a lack of
mines can increase beta activity in routine EEG. normal atonia of REM sleep. Consequently, they
Drug-induced beta activity is usually slower occur more often in the later portion of sleep.
than physiological beta. (Spehlman, 428429) During REM sleep, patients may have an
increase in the severity or frequency of frag-
67. (E) Periodic lateralizing epileptiform dis- mentary myoclonus. Although REM sleep
charges (PLEDs) are EEG patterns showing behavior disorders may occur in healthy elderly
complexes consisting of di- or polyphasic spike subjects, they are also seen with tricyclics, alco-
or sharp waves. They commonly appear in a hol abuse, or central nervous system diseases
wide distribution on one side of the head. They affecting the pathways controlling REM atonia,
are uni- or bilateral but with a clear maximum such as multisystem atrophy. REM parasom-
on one side. Clinical conditions causing PLEDs nias, like other REM sleep disorders, respond
are mainly acute cerebral infarcts but they also well to clonazepam. (Duncan, 21)
include encephalitis, cerebral tumors, menin-
gitis, cerebral abscess, and other conditions 71. (C) Absence seizures usually occur in child-
causing acute destruction of an area of the hood or early adulthood, whereas complex par-
brain. PLEDs usually disappear after days or tial seizures occur at any age. Absence seizures
weeks from the acute phase of the damage. are idiopathic and generalized, while complex
(Spehlman, 321) partial seizures can be cryptogenic or caused by
focal pathology. The duration of attacks is short
68. (D) Amino acids have been separated into two in absence seizures, less than 30 seconds, while
general classes: excitatory and inhibitory. The it is longer in cases of complex partial seizures.
former group depolarizes neurons in mam- There is no postictal state in absence seizures,
malian cells and is formed by aspartic acid, whereas in complex partial seizures confusion,
cysteic acid, glutamic acid, and homocystic emotional disturbance, or headache usually
acid. The latter group hyperpolarizes neurons occurs in the postictal state. The EEG may show
Answers: 6476 187

spikes and slow waves induced by hyperven- and the inaccessibility of scalp electrodes to the
tilation or, less frequently, by photic stimulation medial and inferior frontal lobe surfaces.
in absence seizures. Photosensitivity induces (Duncan, 47)
spikes and waves in only 10% to 30% of
patients with absence seizures. In complex par- 74. (A) Epilepsy, although a relatively common
tial seizures, photic stimulation does not induce symptom of cerebral degenerative diseases, is
seizure activity on EEG, while hyperventila- seldom the predominant clinical problem.
tion has a modest effect. (Duncan, 35) Seizures develop in up to 33% of cases of
Alzheimer disease late in the course of the ill-
72. (D) There is a considerable overlap between the ness. Pick disease rarely causes seizures.
clinical and EEG features of mesiobasal and lat- Epilepsy occurs in 5% of cases of Huntington
eral temporal lobe epilepsy. In lateral temporal disease, especially in late stages and the juve-
seizures, there is usually a detectable underlying nile rigid form. Epilepsy occurs in 6% of cases
structural pathology like glioma, angioma, of Wilson disease and may be the presenting
hamartoma, neural migration defect, and post- feature. (Duncan, 5556)
traumatic changes. There is no association with
a history of febrile seizure. Consciousness may 75. (B) Reflex seizures are attacks precipitated by
be preserved longer than in mesial temporal a specific stimulus, such as touch, musical tune,
epilepsy. Typical aura includes structured hal- a particular movement, reading, stroboscopic
lucination of visual, auditory, gustatory, or olfac- light patterns, or complex visual images. The
tory forms. Illusions of size, shape, weight, most common reflex epilepsies are those
distance, or sound may occur. Automatisms induced by visual stimuli such as flashing
may occur unilaterally and have more motor lights and moving patterns. The other answers
manifestations than does mesial temporal mentioned cause seizures less frequently.
epilepsy. Motor arrest or absence is most often (Duncan, 6667)
seen in mesial temporal epilepsy. Postictal phe-
nomena and autonomic changes may occur in 76. (D) West syndrome is defined by the clinical
both mesial and lateral temporal epilepsy. triad of infantile spasms, arrest of psychomotor
(Duncan, 4446) development, and hypsarrhythmia on EEG. Its
incidence is estimated to be 1 per 4,000 to 6,000
73. (B) The clinical and EEG features of frontal lobe live births. The identified causes of infantile
seizures overlap with those of complex partial spasms are divided into prenatal (cerebral dys-
seizures of temporal lobe origin. Nevertheless, genesis, genetic disorders, intrauterine infec-
there are a number of core features that are tion), perinatal (anoxic injury, head trauma,
strongly suggestive of frontal lobe origin. infection), and postnatal (metabolic disorders,
Typically, the seizures are frequent, with a trauma, infection). No etiology (cryptogenic)
marked tendency to cluster. A brief nonspecific can be identified in as many as 40% of cases.
cephalic aura may occur, which is a vague sen- The onset of almost all cases occurs before age
sation of dizziness, strangeness, and headache. 1 year, with peak onset between 3 and 7 months
Automatisms are present. They are typically of age. The spasms occur in clusters and are
gestural, highly excited, violent, or bizarre, lead- characterized by sudden flexor or extensor
ing to the misdiagnosis of nonepileptic attacks. movements of the trunk. They have a
Postictal recovery is rapid, with a shorter period myoclonic quality but are somewhat longer in
of postictal confusion than in temporal epilepsy. duration. Developmental arrest and regression
Frontal lobe seizures are more likely to general- begins with or before the spasms. EEG shows
ize secondarily than are partial seizures of tem- hypsarrhythmia on interictal EEG recordings.
poral lobe origin. The EEG in frontal lobe In the waking record, hypsarrhythmia consists
epilepsy may be normal ictally or interictally, of disorganized high-voltage slow waves,
partly because of the large area of frontal cortex spikes, and sharp waves that occur diffusely
covered by the relatively few scalp electrodes with a somewhat posterior predominance.
188 4: Neurophysiology, Epilepsy, Evoked Potentials, and Sleep Disorders

The prognosis for children with infantile from 1 to 8 years of age, with a peak incidence
spasms is extremely poor. Although the spasms between 3 and 5 years of age. Fifty percent of
cease, 70% to 90% of infants develop mental cases have severe mental retardation, preced-
retardation and 35% to 60% have chronic ing the onset of seizures 20% to 60% of the time.
epilepsy. However, the major determinant of In addition, disturbances of behavior and per-
outcome is the underlying cause of the spasms. sonality are common. The EEG reveals 2.0- to
Only 5% of children develop normally, and 2.5-Hz slow spike-wave complexes and multi-
these are mainly from the cryptogenic group. focal spikes superimposed on abnormal back-
Corticosteroids, in the form of adrenocorti- ground activity. The complexes are rarely
cotropic hormone (ACTH), prednisone, or induced by hyperventilation or photic stimula-
prednisolone, are the treatment of choice for tion. Seizures may be precipitated by drowsi-
infantile spasms. Benzodiazepines, in the form ness or stimulation. In general, seizures respond
of clonazepam, provide some benefit but are poorly to anticonvulsant treatment, and poly-
not as effective as corticosteroids. Valproic acid therapy is usually required. Benzodiazepines
is also effective, but the risk of hepatotoxicity in and valproic acid are the most effective agents,
this age group must be considered. Vigabatrin although the former may precipitate tonic
has been used to treat infantile spasms effec- status. Sedation should be minimized because
tively. (Bradley, 17571758; Duncan, 7374) of the tendency for seizures to increase in sleep.
Phenytoin and rectal diazepam are effective for
77. (C) Rolandic epilepsy is a well-defined child- serial tonic seizures and status epilepticus.
hood syndrome accounting for up to 15% of all Refractory cases may benefit from the keto-
childhood epilepsies. The age of onset is genic diet or corpus callosotomy, which
between 3 and 13 years with a peak incidence reduces tonic and atonic seizures in some cases.
between 5 and 10 years. The seizures are typi- (Goetz, 10721073)
cally focal, involving the face and oropharynx,
often with secondary generalization, and have 79. (B) Absence seizures begin between the ages of
a strong tendency to occur during sleep. The 3 and 12 years. Patients are more commonly
motor phenomena are usually associated with female than male and there is a strong genetic
sensory disturbances and clonic unilateral jerk- predisposition. Monozygotic twins develop
ing of the upper limb. In about two thirds of absences in 75% of pairs, as do 5% of dizygotic
patients, secondarily generalized tonicclonic twins. Typical absence seizure is characterized
seizures occur, almost always during sleep. by sudden behavioral arrest and unresponsive-
There is generally no associated neurological ness that may be accompanied by eyelid or facial
abnormality, and intellect is usually normal. clonus; automatisms; and autonomic, tonic, or
The diagnosis of rolandic epilepsy can be made atonic features. The interictal EEG in patients
by the typical clinical features and confirmed with typical absence seizures reveals general-
by characteristic EEG findings. These consist of ized 3.0-Hz spike-wave complexes superim-
remarkably focal high-amplitude midtempo- posed on normal background activity. Bursts of
ral spikes or spike-wave discharges. There is a generalized 3.0- to 4.0-Hz spike-wave complexes
striking contrast between the active EEG and slowing to 2.5 to 3.0 Hz are observed during
the benign clinical picture. The prognosis of seizures. Typical absence seizures are commonly
rolandic epilepsy is good. Only 10% of patients precipitated by hyperventilation and infre-
will continue to have seizures 5 years after quently by photic stimulation. The prognosis of
onset of the disease. (Duncan, 7879) absence seizure is good. Absences become less
frequent through adolescence and about 80%
78. (B) LennoxGastaut syndrome is responsible remit by adulthood. (Duncan, 8485; Goetz,
for 2% to 3% of childhood epilepsies. The syn- 10631064)
drome is characterized by multiple seizure
types, slow spike-wave complexes, and diffuse 80. (A) Progressive myoclonic epilepsies represent
cognitive dysfunction. Seizure onset ranges a group of disorders of various etiology that
Answers: 7789 189

collectively account for 1% of all epilepsy syn- regions. The ictal scalp EEG often shows rhyth-
dromes. The natural history varies with the spe- mic theta at the onset that may be bilateral or
cific disorder from mild neurological localized in the affected temporal lobe. (Duncan,
impairment to severe disability progressing to 115119)
death in early childhood.
The disorders comprising the progressive 83. (D) In infantile spasms, the resting EEG shows
myoclonic epilepsies may be classified into the a characteristic disorganized high-voltage pat-
following categories: (1) those with well- tern, with generalized attenuation during
defined biochemical defects such as sialidosis; spasms. This pattern may be seen unilaterally
(2) those with a known pathological or bio- or bilaterally. (Duncan, 115119)
chemical marker yet poorly defined mecha-
nism, such as Lafora disease and myoclonic 84. (E) Benign rolandic epilepsy is characterized
epilepsy with ragged-red fibers; (3) the degen- by unilateral or bilateral triphasic, large-ampli-
erative diseases such as UnverrichtLundborg tude spikes that are maximum in the central or
disease (Baltic myoclonus) and denta- centrotemporal region without background
torubropallidoluysian atrophy. abnormalities. (Duncan, 115119)
Juvenile myoclonic epilepsy is distinct from
progressive myoclonic epilepsy. It usually 85. (C) In benign occipital epilepsy, the EEG
appears around puberty and is characterized shows posterior 1.5- to 3-Hz spike and slow-
by seizures with bilateral single or repetitive wave discharges, singly or in long runs. These
arrhythmic, irregular myoclonic jerks, predom- may be lateralized and usually attenuate with
inantly in the arms. Jerks may cause some eye opening. (Duncan, 115119)
patients to fall suddenly. No disturbance of con-
sciousness is noticeable. The disorder may be 86. (A) In LennoxGastaut syndrome, the back-
inherited and sex distribution is equal. Often ground EEG is usually slow and disorganized,
there are generalized tonicclonic seizures and, with superimposed 1- to 2.5-Hz generalized,
less often, infrequent absences. The seizures anteriorly predominant spike and slow-wave
usually occur after awakening and are often discharges. (Duncan, 115119)
precipitated by sleep deprivation. Interictal and
ictal EEGs have rapid, generalized, often irreg- 87. (E) GM2 gangliosidosis is an autosomal reces-
ular spike waves and polyspike waves; there is sive condition caused by a deficit in the activ-
no close phase correlation between EEG spikes ity of hexosaminidase A. Its clinical picture
and jerks. Frequently, the patients are photo- includes seizures, dementia, blindness, and a
sensitive. Response to appropriate drugs is cherry-red spot in the retina. (Behrman, 20302932;
good. (Duncan, 9092; Goetz, 10731074) Duncan, 160164; Goetz, 1074)

81. (A) The key EEG features of idiopathic gener- 88. (E) GM1 gangliosidosis usually presents with
alized epilepsy are generalized epileptiform dis- failure to thrive in infants, hepatosplenomegaly,
charges that often have a 3-Hz frequency; mental regression, and seizures. Later onset
usually maximum in the anterior parasagitall forms result in seizures, cognitive decline, spas-
regions, normal background, and often photo- ticity, extrapyramidal rigidity, and ataxia. The
sensitive. Photic stimulation may induce occip- key diagnostic test is reduced leukocyte beta-
ital spikes, occipital spikes and slow waves with galactosidase activity. (Behrman, 20302932; Duncan,
frontal or parietooccipital regions, and general- 160164; Goetz, 1074)
ized spike and slow-wave discharges. (Duncan,
115119) 89. (D) NiemannPick disease is characterized in
type A by infant hepatosplenomegaly, slow
82. (B) Temporal lobe epilepsy is characterized in development, loss of skills, spasticity, and
the EEG interictally by spikes that are usually seizures. In type C, NiemannPick disease is
maximal over the temporal or frontotemporal characterized by tonicclonic seizures, ataxia,
190 4: Neurophysiology, Epilepsy, Evoked Potentials, and Sleep Disorders

and dementia. Sphyngomyelinase activity in alized tonicclonic seizures, dementia, and


both types is deficient. (Behrman, 20302932; visual failure. (Behrman, 20302932; Duncan,
Duncan, 160164; Goetz, 1074) 160164; Goetz, 1074)

90. (C) In Gaucher disease, beta-galactocerebrosi- 92. (E) Lafora disease usually presents with the
dase activity is reduced in leukocytes. Infantile phenotype of progressive myoclonic epilepsy
Gaucher disease is an autosomal recessive con- with cerebellar ataxia, dementia, and person-
dition. The onset of the neuronopathic form is ality changes. Death occurs within 10 to 15
usually before 6 months and frequently before years from the onset. Lafora bodies consist of
3 months. Oculomotor apraxia and bilateral polyglucosans and are found in peripheral
strabismus are early signs and are accompa- nerve, liver, and muscle. (Behrman, 20302932;
nied by rapid loss of head control, ability to Duncan, 160164; Goetz, 1074)
roll over and sit, and purposeful movements of
the limbsalong with apathy, irritability, fre- 93. (E) Metachromatic leukodystrophy is trans-
quent crying, and difficulty in sucking and mitted in an autosomal recessive pattern and
swallowing. In some cases, progression is results from a deficiency of arylsulfatase A.
slower, with acquisition of single words by the Enzyme deficiency leads to an accumulation
first year, bilateral corticospinal signs, persist- of the sulfatides, especially cerebroside sulfate.
ent retroflexion of the neck, and strabismus. The gene for arylsulfatase A has been localized
Laryngeal stridor and trismus, diminished to chromosome 22q, and a wide range of muta-
reaction to stimuli, smallness of the head, rare tions has been described. (Behrman, 20302932;
seizures, normal optic fundi, enlarged spleen Duncan, 160164; Goetz, 1074)
and slightly enlarged liver, poor nutrition, yel-
lowish skin and scleral pigmentation, osteo- 94. (A) Adrenoleukodystrophy is a disorder with
porosis, vertebral collapse and kyphoscoliosis, several clinically and genetically distinct forms.
and sometimes lymphadenopathy complete It is a progressive disease with symptoms refer-
the clinical picture. The important laboratory able to myelin loss from the central nervous
findings are an increase in serum acid phos- system and peripheral nerves as well as adrenal
phatase and characteristic histiocytes (Gaucher insufficiency. In general, forms with earlier onset
cells) in marrow smears and liver and spleen have a more rapid course. The X-linked form
biopsies. A deficiency of glucocerebrosidase in usually presents in the early school years with
leukocytes and hepatocytes is diagnostic; glu- neurological symptoms and adrenal insuffi-
cocerebroside accumulates in the involved tis- ciency. The disease is rapidly progressive and
sues. The characteristic pathological feature is fatal. In individuals with later onset, the course
the Gaucher cell, 20 to 60 m in diameter, with is more protracted; when it develops in adults, it
a wrinkled appearance of the cytoplasm and is usually a slowly progressive disorder with
eccentricity of the nucleus. These cells are predominantly peripheral nerve involvement
found in the marrow, lungs, and other viscera; developing for a period of decades. The
neuronal storage is seldom evident. In the adrenoleukodystrophy gene encodes a member
brain, the main abnormality is a loss of nerve of the ATP-binding transporter family of pro-
cellsparticularly in the bulbar nuclei but also teins. The disease is characterized by the inabil-
in the basal ganglia, cortex, and cerebellum ity to properly catabolize very long chain fatty
and a reactive gliosis that extends into the acids within peroxisomes because of a deficit of
white matter. (Behrman, 20302932; Duncan, lignoceroyl CoA, with elevation of levels of very
160164; Goetz, 1074; Ropper and Samuels, Chapter 37) long chain fatty acids in serum. (Behrman,
20302932; Duncan, 160164; Goetz, 1074)
91. (A) Sialidosis is a complex of two types of
autosomal recessive disorders associated with 95. (E) Krabbe disease or globoid cell leukodys-
deficiencies of N-acetylneuraminidase. The trophy is a rare autosomal recessive neurode-
clinical picture includes myoclonic and gener- generative disorder characterized by severe
Answers: 9097 191

myelin loss and the presence of globoid bodies form of leukodystrophy. The cause of the disease
in the white matter. The gene for Krabbe dis- is a deficit of the enzyme aspartoacylase, which
ease is located on chromosome 14q24.3-q32.1. leads to the accumulation of N-acetylaspartic
The disease results from a marked deficiency of acid in the brain. The onset is early, usually rec-
the lysosomal enzyme galactocerebroside beta- ognizable in the first 3 months of life and some-
galactosidase. The onset of the disorder is usu- times in the first neonatal weeks. There is either
ally before the sixth month and often before a lack of development or rapid regression of
the third month (10% after 1 year). Early man- psychomotor function, loss of sight and optic
ifestations are generalized rigidity, loss of head atrophy, lethargy, difficulty in sucking, irritabil-
control, diminished alertness, frequent vomit- ity, reduced motor activity, hypotonia followed
ing, irritability and bouts of inexplicable crying, by spasticity of the limbs with corticospinal
and spasms induced by stimulation. With signs, and an enlarged head. There are no vis-
increasing muscular tone, opisthotonic recur- ceral or skeletal abnormalities but a variable sen-
vation of the neck and trunk develops. Later sorineural hearing loss has been found. Seizures
signs are adduction and extension of the legs, occur in some cases. The CSF is usually normal
flexion of the arms, clenching of the fists, but the protein is slightly elevated in some cases.
hyperactive tendon reflexes, and Babinski The disease is characterized by the increased
signs. Later still, the tendon reflexes are urinary excretion of N -acetyl-L-aspartic acid
depressed or lost but Babinski signs remain, (NAA), which may be used as a biochemical
an indication that neuropathy is added to cor- marker. It reflects the basic enzyme abnormality,
ticospinal damage. Blindness and optic atro- a deficiency of aminoacylase II, which catalyzes
phy supervene. Convulsions occur but are rare the breakdown of NAA. On computed tomog-
and difficult to distinguish from tonic spasms. raphy (CT) scans there is attenuation of cere-
Myoclonus in response to auditory stimuli is bral and cerebellar white matter in an enlarged
present in some cases. The head size is normal brain with relatively normal-size ventricles. The
or, rarely, slightly increased. In the last stage of characteristic pathological changes are an
the disease, which may occur up to several increase in brain volume and weight, spongy
months after the onset, the child is blind and degeneration in the deep layers of the cerebral
usually deaf, opisthotonic, irritable, and cachec- cortex and subcortical white matter, widespread
tic. Most patients die by the end of the first depletion of myelin involving the convolutional
year and survival beyond 2 years is unusual, more than the central white matter, loss of
although a considerable number of cases of Purkinje cells, and hyperplasia of Alzheimer
later onset have been reported (see below). The type II astrocytes throughout the cerebral cortex
deficient lysosomal enzyme in Krabbe disease and basal ganglia. Adachi and coworkers have
is galactocerebrosidase (GALC; also called demonstrated an abnormal vacuolar accumu-
galactosylceramide beta-galactosidase); it nor- lation of fluid in astrocytes and between split
mally degrades galactocerebroside to ceramide myelin lamellae; they have suggested that the
and galactose. The deficiency results in the loss of myelin is secondary to these changes.
accumulation of galactocerebroside; a toxic (Behrman, 20302932; Duncan, 160164; Goetz, 1074;
metabolite, psychosine, leads to the early Ropper and Samuels, Chapter 37)
destruction of oligodendrocytes and depletion
of lipids in the cerebral white matter. The 97. (E) Menkes disease is an X-linked disorder of
globoid cell reaction, however, indicates that copper metabolism. This rare disorder is inher-
impaired catabolism of galactosylceramide is ited as a sex-linked recessive trait. Poor feeding
also important. (Behrman, 20302932; Duncan, and failure to gain weight, instability of tem-
160164; Goetz, 1074; Ropper and Samuels, Chapter 37) perature (mainly hypothermia), and seizures
become apparent in early infancy. The manifes-
96. (B) Canavan disease, an autosomal recessive tations of this disease are attributable to one of
disorder characterized by spongy degeneration numerous known mutations in a copper-trans-
of the white matter of the brain, leads to a severe porting adenosine triphosphatase (ATPase),
192 4: Neurophysiology, Epilepsy, Evoked Potentials, and Sleep Disorders

ATP7A, that is attributed to a failure of absorp- MELAS cases are related to a mitochondrial
tion of copper from the gastrointestinal tract and mutation occurring at the 3243 site of the mito-
a profound deficiency of tissue copper. chondrial gene or, in a few instances, at an
Furthermore, because copper fails to cross the alternative locus that also codes for a segment
placenta, a severe reduction of copper in the of transfer RNA. Maternal inheritance is
brain and liver is evident from birth. In this common but sporadic cases are well known.
sense, the abnormality of copper metabolism is (Ropper and Samuels, Chapter 37)
the opposite of that in Wilson disease. Ropper and
Samuels, Chapter 37) 101. (D) Carbamazepine inhibits uroporphyrino-
gen-1 synthase, which increases the frequency
98. (C) In Wilsons disease, seizures occur in about of seizures in case of porphyria, a condition
6% of cases and may be associated with disor- caused by a defect in uroporphyrinogen-1 syn-
ders of movement, psychiatric and behavioral thase, coproporphyrinogen oxidase, and pro-
derangements, and impaired cognition. A slit- toporphyrine oxydase in erythrocytes. (Duncan,
lamp examination of the cornea shows a 165166)
KayserFleischer ring, which is a peripheral
corneal deposition of copper involving 102. (B) Carbamazepine increases the rate of
Descemets membrane. (Duncan, 165166) metabolism of steroid hormones by inducing
the activity of hepatic microsomal enzymes.
99. (B) In nonketotic hyperglycinemia, severe This will reduce the plasma estrogen level in
epilepsy and mental retardation with abnor- women taking carbamazepine and birth control
mal EEG are the key features. Glycine concen- pills. As a consequence, there is a reduction of
tration is very high in plasma and urine contraceptive efficacy. Vigabatrin, valproate,
without ketosis or acidosis. Reduced glycine benzodiazepines, and gabapentin do not
cleavage enzyme activity is demonstrated in induce hepatic enzymes. (Duncan, 268)
liver biopsy. (Duncan, 165166)
103. (D) The nature of the underlying pathology is
100. (A) MELAS syndrome is a mitochondrial dis- a key determinant of outcome of epilepsy sur-
ease caused by a defect in respiratory chain gery. Small low-grade glioma, dysembryolastic
enzyme deficiencies. Patients with this syn- neuroepithelial tumors, and small cryptic vas-
drome have normal early development fol- cular malformations carry the best prognosis
lowed by poor growth, focal or generalized for epilepsy surgery, with about 70% to 80% of
seizures, and recurrent acute episodes that cases being rendered seizure-free following ade-
resemble strokes or prolonged transient quate resection. In an MRI-based study, 62% of
ischemic attacks. The stroke deficits often patients with hippocampal sclerosis became
improve but in some cases lead to a progressive seizure-free after surgery. Large or complex vas-
encephalopathy. Some have hemicranial cular malformations, particularly if they have
headaches that cannot be distinguished from bled and resulted in deposition of hemosiderin
migraine, and others suffer repetitive vomit- or traumatic injuries, have a poorer prognosis
ing or episodic lactic acidosis. If there is a char- after epilepsy surgery, with less than 50% of
acteristic feature it is the unusual clinical patients rendered seizure-free. Cortical dyplasia
pattern of focal seizures, sometimes prolonged, carries a poor prognosis for surgery, presum-
which herald a stroke and produce an unusual ably because such abnormalities represent a
radiographic pattern of infarction involving more diffuse process than is usually apparent.
the cortex and immediate subcortical white Only a small percent of patients with gross cor-
matter. The CT scan may also show numerous tical dysplasia become seizure-free following
low-density regions that have no clinical cor- epilepsy surgery. (Duncan, 359360)
relates. Most patients have ragged red fibers in
muscle but only rarely is there weakness or 104. (D) The characteristic polysomnographic find-
exercise intolerance. Approximately 80% of ings in REM behavior disorder (RBD) consists
Answers: 98117 193

of the absence of muscle atonia and the pres- 112. (C)


ence of increased EMG activity in the upper
and lower limbs. (Bradley, 18071814) 113. (E)

105. (B) In multiple system atrophy, polysomnog- 114. (D)


raphy shows a reduction of slow-wave, REM,
and total sleep time, increased sleep latency, 115. (A)
and increased number of awakenings during
sleep. (Bradley, 18071814) Explanations 109 through 115

106. (E) In restless leg syndrome, polysomnogra- EEG 1 shows generalized paroxysmal fast
phy documents sleep disturbance and periodic activity that may be seen in porencephaly.
limb movements in sleep, which is found in at Reduced EEG amplitude, focal slow waves,
least 80% of patients. Diagnosis of periodic and focal epileptiform activity over areas of
limb movements in sleep is based on an index atrophic brain may be also seen. EEG 2 shows
gauging this (the number of periodic limb a pattern of discontinuous background activity
movements in sleep per hour of sleep); periodic with a series of bursts separated by lower volt-
limb movements in sleep of up to five is con- age interburst periods. This is characteristic of
sidered normal. High periodic limb move- trace discontinue, a normal EEG pattern for a
ments in sleep with arousal are considered 29-week-old premature baby.
more significant than movements without Hypsarrhythmia corresponds to the EEG
arousal. (Bradley, 18071814) pattern seen in EEG 3. It is a disorganized EEG
background seen in West syndrome. EEG 4
107. (A) Narcolepsy rarely begins before adoles- shows K complexes, which are diphasic waves
cence and is characterized by paroxysmal consisting of an initial sharply contoured tran-
attacks of irrepressible daytime sleep, which is sient followed by a high-amplitude slow wave,
sometimes associated with transient loss of usually of delta frequency. EEG 5 shows 3-Hz
muscle tone. Overnight polysomnography burst of spikes and waves that may be seen in
findings include short sleep latency, excessive typical absence seizure. EEG 6 shows a pattern
disruption of sleep with frequent arousals, of positive occipital transients, which are sur-
reduced total sleep time, excessive body move- face-positive bisynchronous sharp transients.
ments, and reduced slow-wave sleep. (Bradley, They occur in stage I of sleep. EEG 7 shows
18071814) triphasic waves, a pattern seen in hepatic fail-
ure, renal failure, and drug intoxication.
108. (C) In Alzheimer disease (AD), the essential (Spehlman, 327, 31, 175, 201205, 254255)
features of sleep architectural alterations are
reduced total sleep time, decreased REM and 116. (A) Tetrodotoxin clogs the sodium-permeable
slow-wave sleep, reduction of sleep spindles pore by binding tightly to a specific site on the
and K complexes, increased nighttime awak- outside of the channel. It blocks all sodium-
enings, and sleep fragmentation. There is a dependent action potentials and therefore is
high frequency of sleep apnea in those with usually fatal when ingested. (Bear, 89)
AD compared with age-matched controls.
(Bradley, 18071814) 117. (D) Axonal transport from the nerve terminals
to the cell body is called retrograde transport,
109. (F) whereas axonal transport occurring from the
cell body to terminals is called anterograde
110. (G) transport and is dependent on a protein called
kinesin. Retrograde transport allows the
111. (B) neuron to respond to molecules such as growth
factors, which are taken up near the axon
194 4: Neurophysiology, Epilepsy, Evoked Potentials, and Sleep Disorders

terminal by either pinocytosis or receptor- slow adapting and are active so long as the stim-
mediated endocytosis. Retrograde transport ulus is present. For that reason they are crucial
along axonal microtubules is driven by the pro- to reading Braille. Receptors such as Meissner
tein dynein rather than by kinesin. After repli- corpuscles respond to each initial application or
cating in muscle tissue, at a site of a bite, the removal of a stimulus but fail to respond during
rabies virus is then transported in a retrograde maintained stimulation. They are called a rap-
direction to the cell bodies of neurons inner- idly adapting receptors. (Haines, 264265)
vating muscles. Anterograde transport is used
by radioactively labeled amino acids when 120. (D) Muscle spindles such as nuclear bag fibers
injected into neuronal cell bodies. They will be are small encapsulated sensory receptors that
incorporated into neuronal proteins and trans- have a spindle-like or fusiform shape and are
ported in an anterograde direction. The axons located within the fleshy part of the muscle.
containing the radiolabeled proteins can then Their main function is to signal changes of the
be detected by autoradiography. (Haines, 1920) length of the muscle within which they reside.
Changes in the length of muscles are closely
118. (E) Unlike neurons, astrocytes do not propa- associated with changes in the angles of the
gate action potentials. However, they provide joint that the muscles cross. Thus, muscle spin-
neurons with structural support and maintain dles can be used by the central nervous system
the appropriate microenvironment essential for to sense relative positions of the body seg-
neuronal function. Astrocytes associated to ments. (Kandel, 718)
oligodendrocytes and microglia form the major
types of glial cells in the central nervous 121. (B) Gap junction channels provide the ultra-
system. Astrocytes and oligodendrocytes are structural components of electrical synapse. At
derived from the neuroectoderm, whereas electrical synapses, the gap junction channels
microglial cells are derived from the meso- that connect the pre- and postsynaptic cells pro-
derm. Oligodendrocytes are responsible for vide a low-resistance (high-conductance) path-
central nervous system myelination. During way for electrical current to flow between the
development, astrocytes, in the form of radial two cells. Electrical synapse provides a cyto-
glial cells, provide a pathway for neuronal plasmic continuity between a pre- and postsy-
migration. Microglial cells are the immune naptic cells, and a synaptic transmission is
effector cells of the central nervous system. usually bidirectional and without any delay. At
They are able to become phagocytic scavenger chemical synapses, there is no direct continuity
of the central nervous system when it suffers or direct low-resistance pathway between pre-
injury. Activated migroglial cells migrate to the and postsynaptic cells. The synaptic transmis-
site of damage, where they proliferate and sion is unidirectional, using a chemical trans-
phagocytose cell debris. (Haines, 259228) mitter that is released from presynaptic vesicles
to activate postsynaptic receptors. (Kandel,
119. (C) The first step in evoking somatic sensations 176177)
is the activation of peripheral mechanorecep-
tors. Cutaneous tactile receptors are located in 122. (A) The rods and cones of the retina are
the basal epidermis and dermis of the glabrous responsible for photoreception, a process by
and hairy skin. Merkel cells are low threshold which photons are detected and the informa-
unencapsulated mechanoreceptors signal tonic tion is transduced into an electrochemical
events such as discrete small indentation in the signal. (Haines, 314)
skin. They provide input to type II afferent sen-
sory fibers related to both displacement and 123. (B) Rod cells are photoreceptor cells in the
velocity of a stimulus. They produce sensation retina responsible for night vision. Named for
to touch and pressure. They have small receptive their cylindrical shape, rods are concentrated at
field size. They are also capable of encoding the outer edges of the retina and are used in
stimulus intensity or duration because they are peripheral vision. There are about 120 million
Answers: 118126 195

rod cells in the human retina. At rest, in the they contract with greater force than postural
dark, sodium ions flow into the rods outer seg- muscles but for shorter period of time. The
ment. This high resting level of sodium per- fast-twitch muscle fibers are divided into two
meability results in a relatively high resting types: the first type, the fast fatigable type IIB
potential for rod cells, about 40 mV. These muscle fibers, contain large stores of glycogen
sodium channels of the outer segment mem- that provide the energy necessary to produce
brane, which are normally open, close in relatively greater amount of force compared
response to increased calcium or a reduction in with slow-twitch muscle fibers. The second
cyclic guanosine monophosphate. This drives type is type IIA, which is an intermediate
the membrane potential away from the sodium between the type I slow-twitch and type II fast-
equilibrium potential and toward the potas- twitch muscle fiber. Muscles generally contain
sium equilibrium potential, and the rod cell is a mixture of motor units; the proportions vary
hyperpolarized in response to light stimulus. In according to the demands placed on the
the absence of light, the photoreceptor termi- muscle. The gastrocnemius muscle is a
nals constantly release the transmitter gluta- dynamic, powerful muscle used in running
mate at these synapses. The arrival of a and jumping. It is considered a fast-twitch
light-induced wave of hyperpolarization muscle and contains type IIB muscle fibers
causes a transient reduction in this tonic release innervated by large-diameter, rapidly conduc-
of glutamate. The perception of color is tion axons. (Haines, 381382)
achieved in humans through color receptors
containing pigments carried by cone cells. 125. (D) Nerve cooling significantly slows sodium
(Haines, 314315) channel inactivation, which causes prolonga-
tion of the absolute refractory period. This
124. (B) Motor units can be divided into two cate- results in a reduction in nerve conduction veloc-
gories (slow-twitch and fast-twitch) based on ity, a prolongation of motor nerve distal latency,
the metabolic and physiological properties of and an increase in the amplitude of sensory
the muscle fibers and their innervations. Type nerve action potentials. (Dumitru, 188189)
I, slow-oxidative, slow-twitch, or red muscle
is dense with capillaries and is rich in mito- 126. (B) Most commercially available instruments
chondria and myoglobin, giving the muscle for nerve conduction studies have variable low-
tissue its characteristic red color. It can carry and high-frequency filter systems that can be
more oxygen and sustain aerobic activity. adjusted by the clinician to optimize the fre-
Because of the ability of type I muscle fibers to quency content of the signal under investiga-
utilize glucose and oxygen from the blood- tion and limit undesirable noise. This low- and
stream, these fibers can generate abundant high-frequency filter combination constitutes a
adenosine triphosphate (aerobic metabolism) window through which to observe a relatively
and fuel the contractile apparatus for long limited frequency domain, referred to as a
periods of contraction time, making these handwidth. The frequencies above and below
motor units resistant to fatigue. The trade-off, the handwidths limitations are severely atten-
however, is that these muscle fibers can gen- uated by the low- and high-frequency filters,
erate only relatively small levels of force or respectively. Increasing the low-frequency
tension. The postural muscles, such as deep (high-pass) filter from 10 to 300 Hz reduces the
back muscles, are composed predominantly of amplitude of sensory nerves action potential,
this fiber type. These muscles may contract at shortens its peak latency, and reduces the dura-
low level of tension but for exceedingly long tion of the negative spike. The onset latency
periods of time. In contrast, the type II or fast- remains unchanged; however, the morphology
twitch muscle fibers generate much higher of the sensory nerves action potential become
levels of force but for comparatively brief peri- triphasic. The waveforms initial departure
ods of time. Muscles used during strenuous from the baseline occurs over a relatively short
exercise are examples of type II muscle fibers: period of time. This portion of the waveform
196 4: Neurophysiology, Epilepsy, Evoked Potentials, and Sleep Disorders

has a minimal amplitude contribution from the same motor axons back to the original target
low frequencies contained in the potential. muscle. F response latency is shorter in the
Because elevating the low-frequency filter does arms than in the legs because the length of
not affect high frequencies, the onset latency nerve traveled is less. Taller patients have
remains unchanged. (Dumitru, 8283) longer F responses than do shorter patients.
Thus, F latency depends on the distal motor
127. (C) The biological waveforms of the com- latency, the conduction velocity, and the height
pound muscle action potential have both low- of patient. (Dumitru, 238241)
and high-frequency subcomponents. The high
frequencies of most waveforms are contained 131. (B) One of the most profound factors influ-
in the portions of the potential that change rap- encing nerve conduction studies is tempera-
idly, as during the rise time or inflection points. ture. The cooling of a single myelinated fiber
Increasing the low-frequency (high-pass) filter results is reduction of its excitability without
from 1 to 100 Hz removes more low frequencies any effect on transmembrane resistance.
from the motor waveform. This reduces the (Dumitru, 188189)
amplitude of the compound nerves action
potential, shortens its peak latency, reduces the 132. (C) The action potentials amplitude increases
duration of the negative spike, and increases as the nerves temperature declines. In addition
the number of phases of motor response. to amplitude, the action potentials rise and fall
However, the onset latency is not significantly times are also increased. The conduction veloc-
affected. (Dumitru, 83) ity is reduced with nerve temperature decline.
(Dumitru, 188189)
128. (B) Lowering the high-frequency filter results
in the removal of some high-frequency compo- 133. (B) The reduction of nerve temperature causes
nents from the sensory response waveform, a prolongation of the absolute refractory period
leaving it with a relatively lower-frequency con- mainly through a slowing of sodium channels
tent below the upper cutoff limit, which is 500 inactivation. (Dumitru, 188189)
Hz in this question. This delays distal and peak
latencies, reduces the amplitude of the sensory 134. (D) Figure 4-8 illustrates the F wave response.
nerve action potential, and prolongs the dura- The afferent and efferent pathways are carried by
tion of the negative spike. (Dumitru, 83) the stimulated motor nerve. The response is
obtained by supramaximal stimulation of a
129. (D) In recording compound muscle action motor nerve. F latency depends on the distal
potentials, a reduction in the high-frequency motor latency, the conduction velocity, and the
(low filter) component from 10,000 to 500 Hz height of the tested patient. In acute inflamma-
causes a delay in the onset and peak latency, tory demyelinating polyneuropathy, motor nerve
mild amplitude reduction, and longer negative demyelination dramatically reduces the nerve
spike duration. (Dumitru, 8384) conduction velocity, which results in absence or
prolonged latency of the F response. (Preston,
130. (D) The F response is generated by supra- 4850)
maximal stimulation of a motor nerve. It is pro-
duced by an antidromic action potential 135. (E) Figure 4-9 illustrates the H reflex. It is
traveling centripetally toward the spinal cord. elicited by stimulating the tibial nerve in the
At the level of the corresponding anterior cells, popliteal fossa, recording the gastrocsoleus
this antidromic action potential establishes a muscle. The circuit of H reflex involves Ia
persistent or second action potential at the level muscle spindles as sensory afferents and the
of either the perikaryon or its axon hillock. The motor neurons and their axons as efferents. It is
action potential produced at this location is car- obtained by submaximal stimulation of the
ried in a centrifugal or orthodromic direction tibial nerve. It is the electrical correlate of ankle
along the entire length of one or more of the jerk. H reflex may be absent or have prolonged
Answers:127138 197

latency in case of S1 radiculopathy and gener- Behrman RE, Kliegman R, et al. Nelson Textbook of Pediatrics.
alized neuropathy. (Preston, 5356) Philadelphia: Saunders; 2000.
Cooper JR, Bloom FE, Roth RH. The Biochemical Basis of
136. (B) Figure 4-10 illustrates a motor median Neuropharmacology. 7th ed. New York: Oxford University
nerve conduction study recording from the Press; 1996.
Dichter MA. Brodie MJ. New antiepileptic drugs. N Engl J
abductor pollicis brevis. The median nerve
Med. 1996;334:1583-1590.
distal latency (normally 4.5 ms or less) was pro-
Dumitru D, Amato AA, Zwarts MJ. Electrodiagnostic
longed in the demylination range. The median Medicine. 2nd ed. Philadelphia: Hanley & Belfus; 2002.
nerve conduction velocity (normally 49 m/s) Duncan JS, Shorvon SD, Fish DR. Clinical Epilepsy. New
was reduced in the demyelination range. The York: Churchill Livingstone; 1995.
absence of compound muscle action potential Goetz CG, Pappert EJ. Textbook of Clinical Neurology.
dispersion or conduction block is suggestive of Philadelphia: Saunders; 1999.
congenital demylinating polyneuropathy. Hockaday JM, Potts F, Epstein E, et al. Electroencephalo-
(Dumitru, 904905) graphic changes in acute cerebral anoxia from cardiac or
respiratory arrest. Electroencephalogr Clin Neurophysiol.
137. (C) Figure 4-11 illustrates a motor nerve con- 1965;575:1965.
duction study of a ulnar nerve, stimulating at Hughes JR, Schreeder MT. EEG in dialysis encephalopathy.
Neurology. 1980;30:1148-1154.
the wrist and the elbow and recording at the
Lombroso CT. Quantified electrographic scales on 10
abductor digiti minimi. There is a dramatic
preterm healthy newborns followed up to 40-43 weeks
reduction of the nerve conduction velocity in of conceptional age by serial polygraphic recordings.
the demyelinating range with conduction block Electroencephalogr Clin Neurophysiol. 1979;46:460-474.
and dispersion of the compound muscle action Patsalos PN. The pharmacokinetic characteristics of leve-
potential. These findings are suggestive of tiracetam. Methods Find Exp Clin Pharmacol. 2003;25:123-
acquired demyelinating polyneuropathy. 129.
(Dumitru, 951952) Pollard JR, Delanty N. Antiepileptic drug interactions.
Continuum: Lifelong Learning in Neurology 2007;13(4)
138. (A) Figure 4-12 illustrates a motor nerve con- (Epilepsy):91-105.
duction study of a median nerve, stimulat- Preston DC, Shapiro BE. Electromyography and Neuromuscular
ing at the wrist and the elbow and recording Disorders: Clinical-Electrophysiologic Correlations. 2nd ed.
Philadelphia: Elsevier Butterworth-Heinemann; 2005.
at the abductor pollicis brevis. The median
Prior PF. Scott DF. Outcome after severe brain damage.
nerve distal latency is prolonged (normal 4.5
Lancet. 1073;1:770.
ms or less), suggesting a median nerve dys- Reynolds BA, Weiss S. Generation of neurons and astro-
function at the wrist consistent with the diag- cytes from isolated cells of the adult mammalian central
nosis of carpal tunnel syndrome. (Dumitru, nervous system. Science. 1992;255:1707-1710.
10611063) Ropper AH, Samuels MA. Inherited metabolic diseases of
the nervous system. In: Ropper AH, Samuels MA: Adams
and Victors Principles of Neurology. 9th ed. Chapter 37.
REFERENCES Available at: http://www.accessmedicine.com/content.
aspx?aID=3636356
Aminoff MJ. Electrodiagnosis in Clinical Neurology. 4th ed. Ropper AH, Samuels MA. Sleep and its abnormalities. In:
New York: Churchill Livingstone; 1999. Ropper AH, Samuels MA: Adams and Victors Principles of
Bazil CW, Pedley TA. Advances in the medical treatment Neurology. 9th ed. Chapter 19. Available at: http://www.
of epilepsy. Annu Rev Med. 1998;49:135-162. accessmedicine.com/content.aspx?aID=3636356
Bazil CW, Pedley TA. Clinical pharmacology of antiepilep- Spehlman R. EEG Primer. Amsterdam: Elsevier/North-
tic drugs. Clin Neuropharmacol. 2003;26:38-52. Holland Biomedical; 1981.
Bear MF, Connors BW, Paradiso MA. Neuroscience: Verma, NP, Chheda, RL, Nigro MA, Hart ZH. Electro-
Exploring the Brain. 3rd ed. Baltimore: Lippincott encephalographic findings in Rett syndrome.
Williams & Wilkins, 2007. Electroencephalogr Clin Neurophysiol. 1986;64:394-401.
This page intentionally left blank
CHAPTER 5

Neuromuscular Diseases
Questions

1. Vasculitic neuropathy may occur following (C) influenza virus vaccine


(A) hepatitis C infection (D) tetanus and diphtheria toxoids
(B) botulism (E) rubella virus vaccine
(C) neomycin administration
5. Respiratory failure in myotonic dystrophy type
(D) Campylobacter jejuni infection 1 (DM1)
(E) amiodarone administration
(A) does not parallel the muscular manifesta-
2. Human immunodeficiency virus (HIV) tions of DM1
associated GuillainBarr syndrome (GBS) (B) is usually triggered by pulmonary embo-
occurs most often lus
(C) may be the first manifestation of DM1
(A) when retroviral therapy is withdrawn
(D) correlates in occurrence with the severity
(B) when the CD4 cell count is less than
of the genetic defect
100/mm3
(E) causes nocturnal dyspnea and hypoventi-
(C) when the CD4 cell count is less than
lation correlating with the severity of
50/mm3
limb weakness
(D) early in the course of the disease or at the
time of seroconversion 6. Mutation in which of the following myelin pro-
(E) following immune reconstitution teins genes causes predominantly axonal neu-
ropathy?
3. The most frequent HIV neuropathy is
(A) Myelin-oligodendrocyte glycoprotein
(A) mononeuritis multiplex (B) Myelin-associated glycoprotein
(B)distal sensory polyneuropathy (C) PMP-22 protein
(C) motor polyneuropathy (D) Myelin protein zero (MPZ)
(D) acute inflammatory demyelinating (E) Early growth response-2
polyneuropathy
(E) chronic inflammatory demyelinating
polyneuropathy

4. The most reported vaccine associated with


GuillainBarr syndrome is
(A) rabies vaccine
(B) hepatitis vaccine

199
200 5: Neuromuscular Diseases

7. Which of the following is the first step in acti- 10. In traumatic nerve injury, neuropraxia is defined
vation of botulinum toxin (BTX)? by
(A) A release of a zinc endopeptidase into the (A) complete axonal degeneration; disruption
cytoplasm of all connective tissue elements
(B) A proteolytic cleavage of the BTX (B) focal demyelination; block of nerve con-
polypeptide chain into a 100-kDa heavy duction without axonal degeneration
chain (H) and a 50-kDa light chain (L) (C) axonal degeneration with intact
linked by a disulfide bond (S-S) endoneurium
(C) Internalization of the BTX by energy- (D) axonal degeneration and endoneurial dis-
dependent endocytosis ruption with intact perineurium
(D) Enzymatic cleavage by the light chain of (E) axonal degeneration and endoneurial
BTX of selected proteins that are critical and perineurial disruption with intact
for fusion of the presynaptic acetylcholine epineurium
vesicle with the presynaptic membrane
(E) Binding of the light chain of the toxin to 11. Which of the following is true of neuropraxia?
the presynaptic plasma membrane of the
(A) At least 6 months is needed for some
motor axon terminal
recovery.
8. The classic neurophysiological finding in botu- (B) Remyelination is the main mechanism of
lism is recovery.
(C) Collateral sprouting is the main mecha-
(A) reduction of the compound muscle action nism of recovery.
potential (CMAP), which increases after
(D) Regeneration from the proximal site of
repetitive high-frequency nerve stimula-
injury is the main mechanism of recovery.
tion
(E) It commonly results from axonal lacera-
(B) reduction of the compound muscle action
tions.
potential (CMAP), which increases after
repetitive low-frequency nerve stimula-
12. A 50-year-old man developed progressive prox-
tion
imal lower extremities weakness, dry month,
(C) absence of sensory responses on nerve and easy fatigability over several months. His
conduction studies weakness is worsened by hot weather. The result
(D) absence of motor response after repetitive of 20-Hz repetitive nerve stimulation of the right
high-frequency nerve stimulation abductor digiti minimi is shown in Figure 5-1.
(E) reduction of the conduction velocity in This suggests
motor nerves in the demyelinating range
(A) a normal response
9. In which of the following disorders does elec- (B) LambertEaton myasthenic syndrome
trical myotonia occur (on EMG testing) without (C) botulism
clinical myotonia? (D) myotonic dystrophy
(E) myasthenia gravis
(A) Hyperkalemic periodic paralysis
(B) SchwartzJampel syndrome
(C) Paramyotonia congenita
(D) Acid maltase deficiency
(E) Myotonic dystrophy type 2 (proximal
myotonic myopathy)
Questions: 715 201

Motor Nerve cond/5 site R n. Ulnaris


(C) chronic inflammatory demyelinating
5 mY/D 3 ms/D
30. 0 .ms
Segm.
no.
Time
ms
Dist.
cm
CV
m/s
ANALYSE polyneuropathy
1-R. 2.8 4.5
2-1:
3-1:
0.3
3.9
4.5
23.5
150.0
60.2
A: Acquire (D) hereditary neuropathy with liability to
4-1:
5-1:
2.8 0.0
0.0
0.0 B:
pressure palsies
: 0.0 C: Printout
(E) schwannoma

Site Lat. Amp. Area Stimulus


no.
1:
ms
2.8
mV
2.3
ms*mV
6.7
mA
20.00
14. The most likely diagnosis suggested by Figure
2:
3:
2.5
6.7
7.5
2.8 *
21.8
9.0
20.00
25.00 5-3 is
4: 0.0 0.0 0.0 0.00

(A) normal peripheral nerve structure


(B) Wallerian degeneration
10 sec facilitation
(C) segmental peripheral nerve demyelina-
DANTEC tion
A (D) amyloid polyneuropathy
R.Abd Ha1 (E) necrotizing vaculitis
2.8 fU

2 mo Train 1
B
FIG. 5-1. (Reproduced with permission from Amato AA, Russell JA.
Neuromuscular Disorders. New York: McGraw-Hill; 2008.)

13. Figure 5-2 is consistent with the diagnosis of


(A) motor neuron disease
FIG. 5-3. (Reproduced with permission from Amato AA, Russell JA.
(B) diabetic neuropathy Neuromuscular Disorders. New York: McGraw-Hill; 2008.)

15. Figure 5-4 is suggestive of


(A) acute inflammatory demyelinating
polyneuropathy
(B) CharcotMarieTooth disease type 1
(C) amyloidosis
(D) polyarteritis nodosa
(E) diabetic neuropathy

FIG. 5-2. (Reproduced with permission from Amato AA, Russell JA.
Neuromuscular Disorders. New York: McGraw-Hill; 2008.)
202 5: Neuromuscular Diseases

A B
FIG. 5-4. See color insert. (Reproduced with permission from Amato AA, Russell JA. Neuromuscular Disorders. New York: McGraw-Hill; 2008.)

16. Primary lateral sclerosis (PLS) is characterized by (C) multifocal motor neuropathy
(A) asymmetric leg weakness, which is fre- (D) chronic inflammatory demylinating
quently the initial manifestation of the polyneuropathy
disease (E) CharcotMarieTooth neuropathy type 1
(B) the fact that genitourinary symptoms
never occur in PLS 19. Spinal muscular atrophy type I (Werdnig
Hoffman disease) is characterized by
(C) a life expectancy similar to that of
patients with amyotrophic lateral sclero- (A) severe weakness of facial muscles
sis (ALS) (B) preservation of deep tendon reflexes
(D) the fact that PLS never progresses to ALS (C) abdominal breathing
(E) the fact that bulbar muscles are initially (D) severe mental retardation
affected in PLS in 80% of cases (E) delay in the capability of sitting inde-
pendently
17. Hereditary spastic paraplegia is caused by a
mutation in the gene coding for 20. A 35-year-old man consults the neurologist
(A) superoxide dismutase because of muscle cramping and fatigue. His
(B) hexominidase A deficiency physical examination is significant for limb fas-
ciculations and gynecomastia. His CPK level is
(C) senataxin
at 450 UI/L (normal is less than 170 UI/L). The
(D) microtubule associated tau most likely diagnosis is
(E) atlastin
(A) bulbospinal muscular atrophy
18. Hereditary spastic paraplegia may have a clini- (B) inflammatory myopathy
cal presentation similar to that of (C) adult-onset spinal muscular atrophy
(A) GuillainBarr syndrome (D) ALS
(B) copper deficiency (E) multifocal motor neuropathy
Questions: 1627 203

21. Duplication of the peripheral myelin protein 25. Acute motor axonal neuropathy is characterized
(PMP-22) gene causes by
(A) CharcotMarieTooth type 1A disease (A) the lengthy recovery time, which is usu-
(B) CharcotMarieTooth type 1B disease ally longer than for acute inflammatory
(C) CharcotMarieTooth type 1C disease demyelinating polyneuropathy
(D) CharcotMarieTooth type 1E disease (B) the mortality rate, which is close to 20%
(E) CharcotMarieTooth type 2B1 disease (C) respiratory failure requiring mechanical
ventilation, which may be seen in up to
22. De novo deletion of the gene coding for PMP-22 one third of patients
causes (D) the fact that proximal muscles are often
more severely affected than distal mus-
(A) CharcotMarieTooth type 3 disease cles
(DejerineSottas disease)
(E) the fact that there is no autonomic dys-
(B) CharcotMarieTooth type 2A1 disease function
(C) CharcotMarieTooth type 1A disease
(D) hereditary neuropathy with liability to 26. Anti-GM1 antibodies are most frequently seen in
pressure palsies
(A) chronic inflammatory demyelinating
(E) CharcotMarieTooth type 1C disease polyneuropathy
(B) multifocal motor neuropathy
23. CharcotMarieTooth type 1B disease is caused
by a mutation in the gene coding for (C) MillerFisher syndrome
(D) acute inflammatory demyelinating
(A) myelin protein zero (MPZ) polyneuropathy
(B) connexin 32 (E) acute sensory neuropathy affecting small
(C) neurofilament light chain fibers
(D) heat-shock 27-kDa protein-1
(E) early growth response-2 protein (ERG2) 27. The most likely diagnosis suggested by Figure
5-5 is
24. A 27-year-old man developed a progressively (A) a dorsal scapular nerve lesion
worsening unsteady gait and a visual deficit
(B) a long thoracic nerve lesion
most prominent at night. Neurological exami-
nation demonstrated cerebellar ataxia, reduced (C) an axillary nerve lesion
pinprick and vibration sensation in all extremi- (D) a thoracodorsal nerve lesion
ties, and the absence of deep tendon reflexes (E) a musculocutaneous nerve lesion
throughout. Ophthalmological examination
revealed retinitis pigmentosa. Cerebrospinal
fluid examination was significant for an elevated
concentration of protein. These finding are asso-
ciated with
(A) reduced galactosidase activity
(B) reduced galactosidase activity
(C) reduced arylsulfatase A activity
(D) deficiency of high-density lipoprotein
(E) impaired -oxidation of phytanic acid
204 5: Neuromuscular Diseases

29. The molecular abnormality associated with the


neurophysiological findings illustrated in Figure
5-7 is
(A) expansion of unstable polymorphic
cytosinethymineguanine (CTG)
trinucleotide repeats
(B) mutation in the gene coding for superox-
ide dismutase
(C) de novo deletion of the gene coding for
PMP-22
(D) hexominidase A deficiency
(E) duplication of the PMP-22 gene

FIG. 5-5. (Photo courtesy of Steven A. Greenberg, MD. Reproduced with


permission from Greenberg SA, Amato AA. EMG Pearls. Philadelphia:
Hanley & Belfus; 2004.)

28. Figure 5-6 is suggestive of


(A) inclusion body myositis
(B) dermatomyositis
(C) congenital muscular dystrophy
(D) mitochondrial myopathy
FIG. 5-7. (Reproduced with permission from Amato AA, Russell JA.
(E) metabolic myopathy Neuromuscular Disorders. New York: McGraw-Hill; 2008.)

FIG. 5-6. See color insert. (Reproduced with permission from Amato AA, Russell JA.
Neuromuscular Disorders. New York: McGraw-Hill Medical; 2008.)
Questions: 2833 205

30. The most likely diagnosis suggested by Figure


5-8 is
(A) vasculitic neuropathy
(B) hereditary neuropathy
(C) dermatomyositis
(D) amyloid myopathy
(E) inclusion body myositis

FIG. 5-9. See color insert. (Reproduced with permission from Amato AA,
Russell JA. Neuromuscular Disorders. New York: McGraw-Hill Medical,
2008.)

32. Mutation of the ryanodine receptor 1 (RYR1)


gene is responsible for
(A) myotonic dystrophy
(B) CharcotMarieTooth type 1A disease
(C) motor neuron disease
(D) spinal muscular atrophy
A (E) malignant hyperthermia

33. A 10-year-old boy had difficulties with walking


since the age of 1 year. His CPK level is 20 times
the upper limit of normal and his muscle
biopsy showed the presence of staining with
antidystrophin antibodies. The most likely
diagnosis is
(A) Becker muscular dystrophy
(B) Duchenne muscular dystrophy
(C) myotonic dystrophy
(D) nemaline myopathy
(E) inclusion body myositis
B
FIG. 5-8. See color insert. (Reproduced with permission from Amato AA,
Russell JA. Neuromuscular Disorders. New York: McGraw-Hill; 2008.)

31. Figure 5-9 is an ATPase 4.5 staining of a muscle


biopsy. The most likely diagnosis is
(A) mitochondrial myopathy
(B) steroid myopathy
(C) myotonic dystrophy
(D) nemaline myopathy
(E) dermatomyositis
206 5: Neuromuscular Diseases

34. A 35-year-old man suddenly developed pro- (C) intavenous immunoglobulin


found weakness of all his extremities. Physical (D) prednisone
examination demonstrated severe quadriplegia (E) methotrexate
with preservation of respiration, facial and eyes
movements, and absence of deep tendon 38. In myoadenylate deaminase deficiency, the fore-
reflexes. His laboratory workup was significant arm ischemic test would be expected to produce
for a serum potassium level of 2 mEq/L. The which of the following results?
most likely cause of these finding is
(A) A fivefold increase in the levels of ammo-
(A) acute peripheral nerve demyelination nia and lactate
(B) mutation in the calcium channel gene (B) A fivefold increase in the level of lactate
(C) mutation in the chloride channel gene with no change in the level of ammonia
CLcN1 (C) No change in the levels of lactate and
(D) mutation in the potassium channel gene ammonia
KCNJ2 (D) No change in lactate level and a fivefold
(E) expansion of unstable polymorphic increase in the level of ammonia
cytosinethymineguanine (CTG) (E) A less than threefold increase in the level
trinucleotide repeats of ammonia and a fivefold increase in the
level of lactate
35. The association of facial weakness and ptosis
without ophtalmoplegia is highly suggestive of 39. The most frequent malignancy associated with
(A) myasthenia gravis subacute sensory neuronopathy is
(B) acide maltase deficiency (A) adenocarcioma of the lung
(C) myotonic dystrophy type1 (B) small cell lung cancer
(D) KearnsSayre syndrome (C) breast cancer
(E) thyrotoxic paralysis (D) ovarian cancer
(E) lymphoma
36. A 14-year-old boy was referred to the neurolo-
gist by his soccer coach because of difficulties in 40. Deletion of D4Z4 repeats is associated with
running at the beginning of the game, which
improved 20 minutes later. Neurological exam- (A) fascioscapulohumeral dystrophy
ination was normal except that the boy had dif- (B) myotonic dystrophy
ficulty in relaxing his grip, which improved (C) myotonia congenital
when he was asked to grip the examiners hand (D) hereditary neuropathy with a tendency
several times. The most likely diagnosis associ- toward pressure palsies
ated with these clinical findings is
(E) CharcotMarieTooth neuropathy type
(A) myotonic dystrophy IA
(B) myotonia congenita
(C) paramyotonia congenita 41. Which of the following descriptions best char-
acterizes endplate noise?
(D) myasthenia gravis
(E) LambertEaton myasthenic syndrome (A) Regularly occurring spikes fire at 0.5 to 10
Hz with a sound similar to rain on a roof
37. The presence of anti-Jo-1 antibodies in a 48-year- and an initial positive deflection. They
old woman who was diagnosed with dermato- are not specific for muscle fiber damage.
myositis precludes the use of (B) The spike is formed by an initial brief
positive wave followed by a slow nega-
(A) cyclosporine
tive phase. It may be seen in cases of den-
(B) mycophenolate mofetil ervation.
Questions: 3446 207

(C) Low-amplitude monophasic potentials 44. Which of the following descriptions best char-
fire at 20 to 40 Hz and have a characteris- acterizes fasciculation?
tic hissing sound on EMG.
(A) There is a spontaneous involuntary dis-
(D) Spikes wax and wane in both amplitude charge of an individual motor unit.
and frequency.
(B) They are 150-Hz decrementing discharges
(E) Brief, irregular spikes have a negative ini- of a single motor unit that have a charac-
tial deflection and a crackling sound on teristic pinging sound on EMG.
EMG.
(C) It occurs from depolarization of a single
muscle fiber, followed by ephaptic spread
42. Which of the following descriptions best char-
to adjacent denervated fibers.
acterizes endplate spike?
(D) They are rhythmically grouped repetitive
(A) Regularly occurring spikes fire at 0.5 to 10 discharges of the same motor unit, often
Hz with a sound similar to rain on a tin noted in radiation plexitis.
roof and an initial positive deflection. (E) Brief, irregular spikes have a negative initial
They are not specific for muscle fiber deflection and a crackling sound on EMG.
damage.
(B) The spike is formed by an initial brief 45. Which of the following descriptions best char-
positive wave followed by a slow nega- acterizes complex repetitive discharges?
tive phase. It may be seen in cases of den-
ervation. (A) There is a spontaneous involuntary dis-
charge of an individual motor unit.
(C) Low-amplitude monophasic potentials
fire at 20 to 40 Hz and have a characteris- (B) They are 150-Hz decrementing discharges
tic hissing sound on EMG. of a single motor unit that have a charac-
teristic pinging sound on EMG.
(D) Spikes wax and wane in both amplitude
and frequency. (C) It occurs from depolarization of a single
muscle fiber, followed by ephaptic spread
(E) Brief, irregular spikes have a negative initial
to adjacent denervated fibers.
deflection and a crackling sound on EMG.
(D) They are rhythmically grouped repetitive
43. Which of the following descriptions best char- discharges of the same motor unit, often
acterizes positive sharp waves? noted in radiation plexitis.
(E) Brief, irregular spikes have a negative initial
(A) Regularly occurring spikes fire at 0.5 to 10 deflection and a crackling sound on EMG.
Hz with a sound similar to rain on a tin
roof and an initial positive deflection. 46. Which of the following descriptions best char-
They are not specific for muscle fiber acterizes myokymic discharges?
damage.
(B) The spike is formed by an initial brief (A) There is a spontaneous involuntary dis-
positive wave followed by a slow nega- charge of an individual motor unit.
tive phase. It may be seen in cases of den- (B) They are 150-Hz decrementing discharges
ervation. of a single motor unit that have a charac-
(C) Low-amplitude monophasic potentials teristic pinging sound on EMG.
fire at 20 to 40 Hz and have a characteris- (C) It occurs from depolarization of a single
tic hissing sound on EMG. muscle fiber, followed by ephaptic spread
(D) Spikes wax and wane in both amplitude to adjacent denervated fibers.
and frequency. (D) They are rhythmically grouped repetitive
(E) Brief, irregular spikes have a negative ini- discharges of the same motor unit, often
tial deflection and a crackling sound on noted in radiation plexitis.
EMG. (E) Brief, irregular spikes have a negative initial
deflection and a crackling sound on EMG.
208 5: Neuromuscular Diseases

47. Which of the following descriptions best char- (C) Hyperthyroidism myopathy
acterizes neuromyotonic discharges? (D) Disuse atrophy
(A) There is a spontaneous involuntary dis- (E) Upper motor neuron disease
charge of an individual motor unit.
(B) They are 150-Hz decrementing discharges 51. Which of the following is characteristic of
of a single motor unit that have a charac- myotonic muscular dystrophy type 1?
teristic pinging sound on EMG. (A) The mutation in the myotonic dystrophy
(C) It occurs from depolarization of a single gene is an extension of the trinucleotide
muscle fiber, followed by ephaptic spread CTG.
to adjacent denervated fibers. (B) The abnormal gene is located on chromo-
(D) They are rhythmically grouped repetitive some 17.
discharges of the same motor unit, often (C) Cardiac conduction abnormalities are
noted in radiation plexitis. rarely seen.
(E) Brief, irregular spikes have a negative ini- (D) Testicular hypertrophy is seen in most
tial deflection and a crackling sound on cases of myotonic dystrophy.
EMG. (E) There is no clinical myotonia.

48. Which of the following needle EMG parame- 52. The third cranial (oculomotor) nerve is most fre-
ters is seen in acute neuropathic axonal dam- quently affected in cases of
age?
(A) diphtheria
(A) Increased duration of the motor unit
(B) sarcoidosis
action potential
(C) diabetes mellitus
(B) Increased amplitude of the motor unit
action potential (D) Lyme disease
(C) Increased phases of the motor unit action (E) porphyria
potential
(D) Decreased activation of the motor unit 53. Primary axonopathy, with secondary demyeli-
action potential nation and abnormal marker of the connexin-32
gene is a characteristic of
(E) Decreased interference pattern on maxi-
mum voluntary effort (A) hereditary sensory and motor neuropathy
type I
49. On needle EMG, early recruitment of the motor (B) hereditary sensory and motor neuropathy
unit action potential is seen in the case of type II
(A) stroke (C) hereditary sensory and motor neuropathy
(B) chronic inflammatory demyelinating type III
polyneuropathy (D) hereditary sensory and motor neuropathy
(C) acute inflammatory demyelinating type IV
polyneuropathy (E) X-linked CharcotMarieTooth disease
(D) acute myopathy
54. Which of the following myopathies is most
(E) early reinnervation following severe
likely to be associated with arrhythmia?
denervation
(A) Centronuclear myopathy
50. Which of the following conditions is commonly (B) Nemaline myopathy
associated with type I muscle fiber atrophy? (C) Acid maltase deficiency
(A) Corticosteroid-induced myopathy (D) Carnitine deficiency
(B) Myotonic muscular dystrophy (E) KearnsSayre syndrome
Questions: 4763 209

55. The asymmetric and bilateral slowly progres- (C) Upper motor neuron dysfunction is
sive weakness of wrists, ulnar finger flexors, observed in most cases.
and knee extensors with normal facial muscles (D) Nerve conduction studies show conduc-
in a 55-year-old male is highly suggestive of tion block and temporal dispersion.
(A) inclusion body myositis (E) It is treated effectively with corticosteroids.
(B) acid maltase deficiency
60. Which of the following excludes a diagnosis of
(C) myotonic dystrophy chronic inflammatory demyelinating polyradicu-
(D) polymyositis loneuropathy?
(E) nemaline myopathy
(A) Hyporeflexia or areflexia in four limbs
56. Point mutation in the muscle chloride channel (B) Sensory level
gene causes (C) Evidence of demyelination and remyeli-
nation on histological examination of the
(A) hyperkalemic periodic paralysis nerve
(B) myotonia congenita (D) Prolonged distal latencies in two or more
(C) hypokalemic periodic paralysis nerves in nerve conduction studies
(D) paramyotonia congenita (E) Negative VDRL in the cerebrospinal fluid
(E) none of the above
61. Which of the following characteristics is com-
57. Which of the following differentiate Charcot mon to multifocal motor neuropathy (MMN)
MarieTooth neuropathy type IA from Charcot and chronic inflammatory demyelinating
MarieTooth neuropathy type IB? polyradiculoneuropathy (CIDP)?
(A) It is anutosomal dominant disorder. (A) Symmetric distribution of weakness
(B) It is an autosomal recessive disorder. (B) Prolonged F wave on nerve conduction
(C) Hammertoes and pes cavus are seen in study
up to 75% of patients. (C) Normal sensory nerve conduction studies
(D) In nerve conduction studies, definite con- (D) High anti-GM1 antibody titers
duction block is characteristically present. (E) Effectively treated with prednisone
(E) PMP-22 is defective.
62. Which of the following may cause a predomi-
58. Which of the following is true of the dominant nantly sensory neuropathy?
form of X-linked CharcotMarieTooth disease? (A) Arsenic exposure
(A) It is primarily a demyelinating disorder (B) N-hexane exposure
with secondary axonal degeneration. (C) Dapsone
(B) Mental retardation is commonly seen. (D) Pyridoxine intoxication
(C) The autosomal recessive form is more fre- (E) Nitrofurantoin
quent than the autosomal dominant form.
(D) Cranial nerves are frequently involved. 63. Which of the following is true of polymyalgia
(E) The connexin-32 gene is defective. rheumatica?
(A) It is the most common cause of pain in
59. Which of the following is true of multifocal young adults.
motor neuropathy?
(B) It is more common in males than females
(A) Anti-GQ1b antibodies are elevated in by a ratio of 3 to 1.
about 50% of patients. (C) High creatine kinase is commonly found.
(B) Bulbar function and cranial nerves are (D) The sedimentation rate is high.
affected early. (E) The response to steroids is poor.
210 5: Neuromuscular Diseases

64. Which of the following muscles is spared in (D) Spasticity is one of the clinical signs.
facioscapulohumeral dystrophy? (E) The defective gene expresses a
(A) Latissimus dorsi serine/threonine kinase.
(B) Trapezius
69. Which of the following statements is true of
(C) Rhomboid hyperkalemic periodic paralysis?
(D) Serratus anterior
(A) It is an autosomal recessive disease.
(E) Deltoid
(B) It is caused by a mutation on 1q32.
65. Which of the following myopathies is caused (C) The first symptoms usually occur at birth.
by a defective synthesis of dysferlin and is linked (D) Muscle exercise with subsequent rest may
to an abnormal gene located on chromosome trigger symptoms.
2p13? (E) Spironolactone is an effective treatment.
(A) Miyoshi myopathy
(B) Nonaka myopathy (distal myopathy with 70. A 7-month-old male died following a progres-
rimmed vacuoles) sive neurological illness over 6 weeks, with som-
nolence, blindness, deafness, and generalized
(C) Bethlem myopathy
limb spasticity. Autopsy showed bilateral sym-
(D) EmeryDreifuss muscular dystrophy metric necrotic lesions of the thalamus, pons,
(E) Oculopharyngeal muscular dystrophy inferior olive, and spinal cord. The most likely
diagnosis is
66. In Duchenne muscular dystrophy, the biochem-
ical defect is based on a deficit of which of the (A) Leber hereditary optic neuropathy
following proteins? (B) mitochondrial neurogastrointestinal
encephalopathy
(A) Dystrophin
(C) Leigh syndrome
(B) Myotonia protein kinase
(D) Alpers disease
(C) Laminin alpha 2
(E) myoclonic epilepsy with ragged-red
(D) Myelin basic protein fibers
(E) Ryanodine receptor channels
71. A 25-year-old male developed recurrent
67. Which of the following myopathies is related to episodes of nausea, vomiting, and diarrhea con-
a chloride channel defect? comitant with decreased ocular motility, weak-
(A) Becker myotonia ness, and numbness in the lower extremities.
Neurological examination demonstrated sym-
(B) Myotonic dystrophy
metric extraocular ophthalmoplegia. EMG/NCS
(C) Paramyotonia congenita showed a generalized sensorimotor neuropa-
(D) Hyperkalemic periodic paralysis thy. The most likely diagnosis is
(E) Potassium-aggravated myopathy
(A) Leber hereditary optic neuropathy
68. Which of the following is true of congenital (B) mitochondrial neurogastrointestinal
myotonic dystrophy? encephalopathy
(C) Leigh syndrome
(A) Myotonia is the cardinal clinical sign in
(D) Alpers disease
the neonatal period.
(E) myoclonic epilepsy with ragged-red
(B) It is caused by an abnormal, unstable trin-
fibers
ucleotide repeat on chromosome 17.
(C) Neonatal respiratory failure never occurs.
Questions: 6477 211

72. A 25-year-old male developed a painful decrease (C) myoclonic epilepsy with ragged-red
of his visual acuity with decreased ocular motil- fibers
ity over 6 weeks. Neurological examination (D) KearnsSayre syndrome
demonstrated bilateral external ophthalmople- (E) mitochondrial encephalomyelopathy
gia, bilateral centrocecal scotoma, abnormal with lactic acidosis and stroke-like
color vision, bilateral optic atrophy, and a visual episodes
acuity of 20/400 bilaterally. The most likely
diagnosis is 75. A 17-year-old youth came to the emergency room
(A) Leber hereditary optic neuropathy because of an episode of syncope. He reported a
(B) mitochondrial neurogastrointestinal decrease in his visual acuity and ocular motility
encephalopathy over the preceding weeks. Neurological exami-
nation demonstrated bilateral retinal degenera-
(C) Leigh syndrome
tion, mild bilateral extraocular ophthalmople-
(D) Alpers disease gia, and mild cerebellar ataxia. The rest of the
(E) myoclonic epilepsy with ragged-red physical examination was unremarkable. ECG
fibers showed a Mobitz type II atrioventricular block.
The most likely diagnosis is
73. A 2-year-old infant was brought to the emer-
gency room by her mother because of intractable (A) Leigh syndrome
seizures, developmental delay, failure to thrive, (B) Alpers disease
and episodes of vomiting. Neurological exami- (C) myoclonic epilepsy with ragged-red
nation showed developmental delay and gen- fibers
eralized hypotonia. CT scan of the head showed (D) KearnsSayre syndrome
bilateral occipital and temporal hypodensities (E) mitochondrial encephalomyelopathy
with cortical atrophy. EEG showed generalized with lactic acidosis and stroke-like
slow-wave activity. Lab workup showed episodes
increased liver enzymes. Liver biopsy showed
fatty degeneration. The most likely diagnosis is 76. A 30-year-old man developed progressive
(A) Leber hereditary optic neuropathy ataxia, myoclonic seizures, and severe myopa-
(B) mitochondrial neurogastrointestinal thy. The most likely diagnosis is
encephalopathy (A) Leigh syndrome
(C) Leigh syndrome (B) Alpers disease
(D) Alpers disease (C) myoclonic epilepsy with ragged-red
(E) myoclonic epilepsy with ragged-red fibers
fibers (D) KearnsSayre syndrome
(E) mitochondrial encephalomyelopathy
74. A 15-year-old youth consulted the neurologist with lactic acidosis and stroke-like
because of recurrent headache, vomiting, and episodes
transient right hemiparesis. He also reported two
episodes of generalized tonicclonic seizures. 77. KearnsSayre syndrome is characterized by
Neurological examination showed no focal
deficits. MRI of the head revealed multiple sub- (A) hypotonia
acute ischemic lesions in small vessels of the (B) retinal degeneration
parietal lobes. Lab workup showed elevated lac- (C) age of onset after 20 years
tic acid levels in serum and cerebrospinal fluid. (D) mytonia
The most likely diagnosis is
(E) seizure
(A) Leigh syndrome
(B) Alpers disease
212 5: Neuromuscular Diseases

78. Cardiac conduction defects are frequent features (D) Magnesium-containing drugs may
of improve the symptoms of myasthenia
gravis.
(A) KearnsSayre syndrome
(E) A repetitive stimulation study at 50 Hz
(B) Leigh syndrome
will show a decremental response.
(C) myoclonic epilepsy with ragged-red
fibers 81. Which of the following is characteristic of
(D) mitochondrial encephalomyelopathy LambertEaton myasthenic syndrome?
with lactic acidosis and stroke-like
episodes (A) The N type of voltage-gated calcium
channel is detected in less than 10% of
(E) the dominant form of progressive exter-
patients with LambertEaton myasthenic
nal ophthalmoplegia
syndrome associated with malignancy.
79. A 30-year-old man consulted the neurologist (B) The first symptom is usually facial weak-
because of exercise intolerance, especially when ness.
lifting heavy weights and walking uphill. He (C) Autonomic symptoms are usually rare.
also complained of myalgia, premature fatigue, (D) An incremental response at a rate of 3 Hz
and muscle swelling (relieved by rest). He is highly suggestive.
reported increased shortness of breath and pal- (E) Postactivation stimulation may be seen
pitations on exercise. Neurological examination after voluntary exercise of 10 seconds or
was normal. Laboratory evaluation showed nor- after a tetanic stimulation of 20 to 50 Hz.
mal pyruvate and lactate levels in serum, myo-
globinuria with exercise, CK 1200 UI/ML, nor- 82. A 25-year-old man developed an acute episode
mal CBC and liver function tests, and normal of nausea, vomiting, diarrhea, and abdominal
electrolytes. The administration of epinephrine pain, followed a few hours later by diplopia,
induced a normal rise of the blood sugar. Which dysarthria, and progressive lower extremity
of the following is true of this condition? weakness. Neurological examination demon-
(A) There is a decrease in the ADP level in the strated bilateral external ophthalmoplegia,
muscle cells during exercise. dilated pupils, paraparesis with absent deep
tendon reflexes in the lower extremities, and a
(B) Glucose infusion may cause a substantial
normal sensory examination. Nerve conduction
drop in the patients exercise capacity.
study showed decreased compound muscle
(C) It is caused by a myophosphorylase defi- action potentials of lower extremity muscles.
ciency. Needle EMG showed small amplitude and short
(D) It is caused by a phosphofructokinase duration of recruited motor unit action poten-
deficiency. tials under voluntary contraction. Repetitive
(E) Phosphorous magnetic spectroscopy may nerve stimulation showed a decrement at 3 Hz
detect a high phosphomonoester peak. and an incremental response at 50 Hz. The most
likely diagnosis is
80. Which of the following is true of acquired myas-
(A) botulism
thenia gravis?
(B) tetanus
(A) The loss of acetylcholine receptors results (C) venom poisoning
in increased postsynaptic sensitivity to
(D) LambertEaton myasthenic syndrome
acetylcholine.
(E) organophosphate poisoning
(B) Within 1 year of onset, the disease
remains purely ocular in 80% of cases.
(C) Bulbar symptoms are present in about
16% of patients at the onset of the disease.
Questions: 7891 213

83. Which of the following drugs does not exacer- 88. The most frequent cranial nerve involved in dia-
bate the neuromuscular blockade in myasthenia betes is the
gravis?
(A) glossopharyngeal nerve
(A) Magnesium sulfate (B) vestibulocochlear nerve
(B) Tobramycin (C) oculomotor nerve
(C) Quinidine (D) trigeminal nerve
(D) Ciprofloxacin (E) facial nerve
(E) Acyclovir
89. The olfactory nerve as well as the vestibulo-
84. Which of the following hereditary myasthenic cochlear nerve are most commonly affected cra-
disorders has autosomal dominant penetrance? nial nerves in

(A) Familial infantile myasthenia (A) acute inflammatory demyelinating


(B) Limb-girdle myasthenia polyneuropathy
(C) Benign congenital myasthenic syndrome (B) Refsum disease
with facial dysmorphism (C) MillerFisher syndrome
(D) Slow-channel myasthenic syndrome (D) Sjgren syndrome neuropathy
(E) Acetylcholine deficiency syndrome (E) Wegener granulomatosis

85. Which of the following drugs may cause a 90. A predominantly motor neuropathy is seen in
necrotizing myopathy? cases of

(A) Lovastatin (A) pyridoxine neuropathy


(B) Amiodarone (B) paraneoplastic neuropathy
(C) Zidovudine (C) spinocerebellar degeneration
(D) D penicillamine (D) dapsone-induced neuropathy
(E) Colchicine (E) a deficiency in vitamin E neuropathy

86. The most frequent cranial nerve involved in sar- 91. Which of the following is not true about the
coidosis is the safety factor in neuromuscular transmission?

(A) glossopharyngeal nerve (A) It is defined by the difference between the


(B) vestibulocochlear nerve membrane potential and the threshold
potential for initiating an action potential.
(C) oculomotor nerve
(B) Postsynaptic folds form a high-resistance
(D) trigeminal nerve
pathway and increase the action potential
(E) facial nerve threshold.
(C) The loss of synaptic folds increases the
87. The most frequent cranial nerve involved in
safety factor.
diphtheria is the
(D) Myasthenia gravis, like all neuromuscu-
(A) glossopharyngeal nerve lar transmission disorders, is character-
(B) vestibulocochlear nerve ized by a compromise of the safety factor.
(C) oculomotor nerve (E) The conduction properties and density of
(D) trigeminal nerve acetylcholine receptors contribute to the
(E) facial nerve safety factor.
214 5: Neuromuscular Diseases

92. Which of the following suggests that CD4 T (D) Only myasthenia gravis patients have
helper cells have a major role in the pathogene- CD4 T cells that react against self-
sis of myasthenia gravis? antimuscle acetylcholine receptors; nor-
mal patients CD4 T cells do not react to
(A) Most antiacetylcholine receptor antibod- self-antigens.
ies in myasthenia gravis patients are
(E) Myoid cells are the only thymic cells that
high-affinity IgG; their synthesis requires
express acetylcholine receptor sequences.
CD4 and T helper factors.
(B) Acetylcholine receptors reactivate CD4 94. Which of the following is true of the mechanism
cells from the blood, and the thymus of of action of corticosteroids in the treatment of
myasthenia gravis patients has a T-cell myasthenia gravis?
cytotoxic function.
(C) Thymectomy does not modify the reactiv- (A) Stimulation of lymphocyte proliferation
ity of blood T cells against acetylcholine (B) Stimulation of antigen processing by
receptors. macrophages
(D) In vitro treatment of CD4 T cells from (C) Reduction of acetylcholine receptor syn-
the blood of myasthenia gravis patients thesis in the muscle
with anti CD4+ antibodies increases the (D) Redistribution of lymphocytes from cir-
reactivity of T cells to acetylcholine recep- culation
tors. (E) Increasing lymphocyte differentiation
(E) In experimental autoimmune myasthenia
gravis, suppression of the synthesis of 95. In the pathogenesis of dermatomyositis, recent
pathogenic antiacetylcholine receptor microarray studies have demonstrated upregu-
antibodies requires CD4 cells. lation of genes induced by
(A) interferon
93. Which of the following is true of the role of the
thymus in myasthenia gravis? (B) interferon
(C) antibodies to specific endothelial antigen
(A) Ten percent of patients with myasthenia
(D) macrophage
gravis have follicular hyperplasia of the
thymus. (E) major histocompatibility complex (MHC)
class I
(B) Acetylcholine receptors are expressed
only in the thymus of these patients.
(C) Thymic myoid cells expressing acetyl-
choline receptors, or antigenically similar
proteins may act as antigen-presenting
cells.
Answers and Explanations

1. (A) Vasculitic neuropathy may complicate the advanced stages of AIDS. Clinical and electro-
course of hepatitis C. The disorder is often physiological studies suggest that DSP is pre-
painful and asymmetric skeletal muscle weak- dominantly an axonal neuropathy. In some
ness is prominent. Central nervous system dis- patients, DSP may coexist with a toxic neuropa-
ease may accompany the neuropathy. Palpable thy associated with the use of specific nucleoside
purpura, which is due to a leukocytoclastic vas- antiretrovirals. Inflammatory demyelinating
culitis, is often seen in the legs in a distal greater polyneuropathies such as chronic inflammatory
than a proximal distribution. The tempo of the demyelinating polyneuropathy (CIDP) and acute
vasculitic neuropathy may be subacute, chronic, inflammatory demyelinating polyneuropathy
or acute on chronic. Progression to multiorgan (AIDP) are less common but can occur in the set-
failure may occur, especially when the baseline ting of HIV infection, often during seroconver-
viral load is high (greater than 2 to 3 million sion, before AIDS or immunosupression appears.
ge/mL serum) or if the hepatitis C virus geno- Rarely, AIDP may occur after severe CD4+ deple-
type is 1a and 1b. (Souayah, Neurol Neurophysiol tion. Progressive polyradiculopathies are fre-
Neurosci 5; Khella, 101106) quently associated with CMV infection, mostly
in the very late stages of AIDS, when the CD4+
2. (D) Several types of peripheral neuropathy are cell count is below 50/mm3. Mononeuritis mul-
associated with human immunodeficiency virus tiplex (MM) may also present in the early stages
(HIV) infection. GBS occurs most often at the of AIDS as a manifestation of a vasculitic neu-
time of seroconversion or in the early course of ropathy (sometimes associated with hepatitis B
infections where the CD4+ cell count is rela- or C) or during the late stages as the result of
tively preserved. Occasionally, GBS is reported CMV infection. (Cornblath, 446450; Souayah,
in an advanced stage of acquired immunodefi- 143145)
ciency syndrome (AIDS) with a CD4+ cell count
less than 50/mm3 or during immune reconsti- 4. (C) Because of fears of an influenza pandemic,
tution. (Souayah, J Neuroimmunol 188, 143145). the 1976 National Influenza Immunization
Program A/New Jersey swine flu influenza
3. (B) With the effectiveness of antiretroviral treat- (A/NJ/1976/H1N1) vaccination campaign was
ment and the consequent decline in the incidence designed to immunize almost the entire adult
rates of central nervous system infections and population in the United States as well as children
HIV dementia, HIV-associated neuropathies at risk for serious influenza virus infection. The
have become the most common neurological dis- program was stopped after reports of vaccine-
orders associated with AIDS. The most common associated GBS. The A/NJ/1976 vaccine was
HIV-associated neuropathy is distal sensory shown to be associated with the development of
polyneuropathy (DSP), a disorder characterized GBS, with attributable risk estimates for GBS in
mostly by sensory symptoms, often including the 6 weeks after vaccination ranging from 4.9 to
spontaneous or evoked pain with a subacute and 11.7 cases per million adult vaccinees. Studies of
chronic course usually developing during the subsequent influenza vaccines used after 1976 in

215
216 5: Neuromuscular Diseases

general detected no significant increase in the form causes severe neuropathy in infancy with
overall risk for GBS in adult vaccinees, although delayed motor milestones, slow conduction
a borderline statistically significant elevated risk velocities in the demyelinating range, and pre-
of less than one excess case per million adult dominant demyelination on nerve biopsy. It
vaccinees was reported during the 19921993 seems that mutations that significantly disturb
and 19931994 influenza seasons combined. The the tertiary structure of MPZ are responsible for
1976 swine flu vaccine induced anti-GM1 anti- this phenotype. The late-onset form presents in
bodies in mice, as did vaccines from 1991 to 1992 adulthood with a neuropathy that is slowly pro-
and 2004 to 2005. These preliminary studies sug- gressive; it includes axonal features to a greater
gests that influenza vaccines contain structures extent than demyelinating features on electro-
that can induce anti-GM1 antibodies after inocu- diagnostic and nerve biopsy studies. Mutations
lation into mice. (Nachamkin, 226233; Souayah, that subtly affect the MPZ structure may inter-
Vaccine 25, 52535255) fere with Schwann cellaxon interaction and
cause this phenotype. (Souayah, J Neurol Sci 263,
5. (B) Acute respiratory failure may complicate 177179)
the course of myotonic dystrophy type 1 (DM1)
and is the cause of death in 30% to 75% of 7. (B) The action of BTX involves a four-step
patients. Acute respiratory failure may occur in process. Step 1: after botulinum toxin is acti-
DM1 patients undergoing anesthesia and it may vated by proteolytic cleavage of the polypep-
rarely be the revealing symptom in DM1 tide chain into a 100-kDa heavy chain (H) and a
patients undergoing surgery. It was also 50-kDa light chain (L) linked by a disulfide bond
reported in a series of patients, in the first of (S-S), the heavy-chain (H) domain of the toxin
whom ventilatory failure was the initial pres- binds to the presynaptic plasma membrane of
entation of DM1 without the provocative chal- the motor axon terminal. Step 2: the toxin com-
lenge of anesthesia or surgery. Respiratory dys- plex is then internalized by energy-dependent
function may have a central origin with endocytosis. Step 3: the light chain (L), a zinc
decreased respiratory drive or a peripheral ori- endopeptidase, is released into the cytoplasm.
gin as in the setting of aspiration pneumonia Step 4: the light chain cleaves various compo-
secondary to poor cough reflex and weakness of nents of SNARE, including SNAP 25 (BTX A),
the diaphragmatic, pharyngeal, and masticatory VAMP/synaptobrevin (BTX B), or syntaxin
muscles. Ventilatory failure generally parallels (BTX C), and thus prevents the fusion of acetyl-
the development of limb weakness in DM1 choline synaptic vesicle with the plasma mem-
patients, although the diaphragm may be more brane. This blocks the release of the neurotrans-
involved than limb muscles and cause supine mitter into the synaptic cleft, causing local
dyspnea and nocturnal hypoventilation. No cor- chemodenervation. (Jankovic, 951957)
relation was found between the occurrence of
ventilatory failure and CTG repeat number. 8. (A) BTX prevents acetylcholine release from the
(Souayah, J Clin Neuromuscul Dis 9, 252255) presynaptic neuromuscular junction. BTX binds
to a receptor at the neuromuscular junction. It is
6. (D) MPZ is a transmembrane protein with then cleaved into two protein chains and moves
extracellular and intracellular domains respon- into the cytoplasm, where it interferes with the
sible for myelin compaction and adherence of neuroexocytosis apparatus within the cell. This
adjacent wraps of myelin sheets. It is also results in a dose-dependent chemodenervation
involved in the signal transduction cascade of the injected muscle. Mild and transitory gen-
responsible for interaction between the Schwann eralized weakness has been reported after focal
cell and axon as well as in the regulation of injection of BTX at a therapeutic dose. The clas-
myelin-specific gene expression. MPZ mutations sic neurophysiological finding in botulism is
cause hereditary neuropathy with phenotypic reduction of the compound muscle action poten-
clustering into two major clinical, electrodiag- tial (CMAP), which increases after repetitive
nostic, and pathological entities. The early-onset high-frequency nerve stimulation. On needle
Answers: 512 217

EMG, the common findings are spontaneous axonotmesis of varying degrees. Focal ischemia
activity and small motor unit potentials of short also may cause axonotmesis. Neurotmesis com-
duration and increased jitter and blocking on monly results from lacerations or less commonly
single-fiber EMG. However, in a case report of from severe crush or stretch injuries. (Quan,
a high-dose injection of BTX A, no motor 4551)
response was obtained either with single stim-
ulation or high-frequency repetitive stimula- 11. (B) Neurapraxia most often results from com-
tion. On needle EMG, all sampled muscles were pression and subsequent focal demyelination.
silent, and even the physiological endplate The time required for recovery in traumatic
spikes and noise were absent. They reflect a nerve injury depends on the type of injury and
severe chemodenervation and total depletion the relative contributions of three possible
of acetylcholine release in the neuromuscular modes of recovery: remyelination, collateral
junction in response to nerve excitation. sprouting from surviving axons, and axonal
(Souayah, Neurology 67, 18551856) regeneration. Restoration of impulse conduc-
tion after neurapraxia depends on remyelina-
9. (D) Myotonia on clinical examination is always tion of the affected site. Of all nerve injuries,
associated with myotonic discharges on EMG. neurapraxia generally recovers most quickly,
The converse is also nearly always true with one usually taking 6 to 8 weeks. Axonotmesis recov-
notable exception. Acid maltase disease consis- ers by two processes. Lesions involving less than
tently shows myotonic potentials on EMG with 20% to 30% of motor axons may recover fully by
absent clinical myotonia. Adult-onset acid mal- collateral sprouting of remaining axons over 2 to
tase deficiency (glycogenosis type II) is a glyco- 6 months. With more extensive injury, surviving
gen storage disease presenting with truncal and axons cannot fully supply the denervated mus-
proximal limb weakness; it is slowly progressive cle. Nerve regeneration from the proximal axon
over the years. Death is usually caused by weak- stump at the site of injury must compensate for
ness of respiratory muscles. Occasionally, the the remainder. When more than 90% of axons
presenting weakness is diaphragmatic. EMG are injured, regeneration becomes the predom-
shows evidence of multiple spontaneous dis- inant mechanism of recovery. The timing of
charges including myotonic discharges, fibril- recovery depends on the distance of the lesion
lation potentials, positive sharp waves, complex from the denervated target muscle. Proximal
repetitive discharges (CRDs), and small motor regeneration occurs at a rate of 6 to 8 mm/day,
unit action potentials. (Miller, 293299) whereas distal regeneration occurs at 1 to
2 mm/day. The prerequisite for regeneration is
10. (B) In 1943, Seddon proposed a classification of an intact Schwann cell basal lamina tube to
peripheral nerve injuries that is still useful today. guide and support axonal growth to the appro-
Under this system, three types of injuriesneu- priate target muscle. Schwann cell tubes remain
ropraxia, axonotmesis, and neurotmesisare viable for 18 to 24 months after injury. If the
described. The mildest, neurapraxia, is the inabil- axon does not reach its target muscle within this
ity of nerve fibers to conduct an action potential time, these supporting elements degenerate and
despite axonal continuity. Loss of axonal conti- effective regeneration cannot occur. (Quan, 4551)
nuity without associated disruption of the fasci-
cular connective tissue elements is referred to as 12. (B) LambertEaton myasthenic syndrome (LEMS)
axonotmesis. Neurotmesis describes the most is a rare condition in which weakness results
severe injury, with disruption of the entire nerve, from an abnormality of acetylcholine (ACh)
including all glial and connective tissue sup- release at the neuromuscular junction. LEMS
ports. results from an autoimmune attack against volt-
Neurapraxia, or type 1 injuries, most often age-gated calcium channels (VGCCs) on the
result from compression and subsequent focal presynaptic motor nerve terminal. Cancer is
demyelination. More severe closed trauma, present when the weakness begins or is later
such as crush or stretch injuries, may cause found in 40% of patients with LEMS. This is
218 5: Neuromuscular Diseases

usually a small cell lung cancer (SCLC), although 14. (C) Figure 5-3 illustrates teased nerve fibers
LEMS has also been associated with non-SCLC, with segmental demyelination (a short demyli-
lymphosarcoma, malignant thymoma, or carci- nating internode). This may be seen in chronic
noma of the breast, stomach, colon, prostate, inflammatory demyelinating polyneuropathy.
bladder, kidney, or gallbladder. (Amato, 89)
Clinical manifestations frequently precede
cancer identification. In most cases, the cancer 15. (B) Figure 5-4A and B (respectively showing a
is discovered within the first 2 years after onset semithin section and the same as seen under elec-
of LEMS and within 4 years in virtually all tron microscopy) illustrates onion-bulb forma-
cases. tion. With sequential episodes of demyelination
Symptoms usually begin insidiously. Many and remyelination, concentric tiers of Schwann
patients have symptoms for months or years cell processes accumulate around the axons,
before the diagnosis is made. Weakness is the forming onion bulbs. They are typically seen in
major symptom, with proximal muscles more hereditary demyelinating neuropathies such as
affected than distal ones (especially in the lower CharcotMarieTooth disease types 1, 3, and 4.
limbs). Respiratory muscles are not usually They can also be seen in chronic inflammatory
affected, Most patients have a dry mouth, which demyelinating polyneuropathy. (Amato, 89)
frequently precedes other symptoms of LEMS.
Some patients have other manifestations of auto- 16. (B) Primary lateral sclerosis (PLS) is character-
nomic dysfunction, including impotence in ized by exclusive or predominant upper motor
males and postural hypotension. Approximately neuron dysfunction, in which spasticity is the pre-
20% of patients note that weakness and fatigue dominant source of impairment. Approximately
are exacerbated by hot weather and during the 2% to 5% of amyotrophic lateral sclerosis (ALS)
course of taking hot bath. cases begin with a PLS phenotype. The average
Repetitive nerve stimulation at a low rate of onset is at about 50 years of age, 10 years ear-
(2 to 3 Hz) often yields a decremental response lier than typical ALS. The legs are involved first
that is maximal between the first and second in 75% of PLS cases, making it impossible to run
responses and continues to decline until the or hop effectively. In 10% of PLS cases, the upper
fourth, fifth, or sixth response. Repetitive nerve extremities are the first region to become symp-
stimulation at a low rate (20 Hz) is illustrated tomatic. In approximately 15% of PLS cases, bul-
in Figure 5-1. It demonstrates a dramatic and bar muscles are affected initially. In most cases,
serial increase of compound muscle action onset is asymmetric. Genitouronary symptoms
potential (CMAP) amplitude from the abnor- such as urinary urgency and urgency inconti-
mally small to the normal CMAP range. nence do occur in PLS (as opposed to the
(Dumitru, 11771181) absence of genitourinary involvement in ALS),
presumably on the basis of detrusorsphincter
13. (D) Figure 5-2 shows an analysis of teased nerve dyssynergia from upper motor neuron involve-
fibers. By this method, individual myelinated ment. The life expectancy of individuals with
fibers are separated from the nerve fascicles and PLS is considerably longer than that of those
lightly stained, allowing the integrity and thick- with ALS, since the average duration of PLS
ness of the myelin sheath to be examined. Figure ranges from 7 to 14 years. Some 80% of PLS
5-2 illustrates focal thickening and folding of patients who evolve into ALS do so within the
the myelin sheath in the perinodal or intermodal first 4 years of their disorder. (Amato, 101)
area, leading to the formation of tamacula.
Tamacula (the Latin word for sausage) are com- 17. (E) Hereditary spastic paraplegias (HSPs) com-
monly seen in hereditary neuropathy with a ten- prise a cluster of inherited neurological disorders
dency for pressure palsies and occasionally in characterized principally by lower extremity
other forms of CharcotMarieTooth disease to spasticity and weakness due to a length-
develop. (Amato, 8889) dependent retrograde axonopathy of corti-
cospinal motor neurons. The most common
Answers: 1323 219

form of autosomal dominant HSP is caused by bospinal muscular atrophy (Kennedy disease). It
mutations in the SPG4/SPAST gene, encoding is an X-linked adult-onset of spinal muscular
spastin. Mutations in the gene encoding the atrophy. Affected patients develop bulbar or
large oligomeric GTPase atlastin-1 are responsi- proximal weakness at a median age of onset of
ble for SPG3A. SPG3A is the second most fre- 44 years. Initial symptoms are usually nonspe-
quent gene mutated in autosomal dominant cific and include muscle cramping, tremor,
HSPs, accounting for approximately 10% of gynecomastia, fatigue, and mild CPK elevation.
cases. SPG3A HSP is pure and almost indistin- The clinical manifestations stem from lower cra-
guishable from SPG4 HSP except that it usually nial nerve motor nuclei and anterior horn cells
begins earlier, in childhood or adolescence. of the spinal cord. The weakness is symmetric
(Amato, 116; Depienne, 674680) and progresses insidiously. In 10% of cases, the
initial symptoms include difficulty in swallow-
18. (B) Copper is an essential trace element ing, chewing, or speaking. Facial weakness and
required by all life forms. It is a component of dropped jaw may occur. The disorder results
key metalloenzymes that play a critical role in from a mutation of the androgen receptor gene
the structure and function of the nervous sys- on the X chromosome. (Amato, 125126)
tem. Cytochrome c oxidase is a component of
the mitochondrial respiratory chain, superox- 21. (A) Approximately 85% of people with Charcot
ide dismutase is an important antioxidant, and MarieTooth type 1A disease have a 1.5-
dopamine [beta]-hydroxylase is important in megabase duplication within chromosome
the catecholamine biosynthetic pathway. Copper 17p11.2-12, wherein the gene for PMP-22 lies.
deficiency may cause signs of myelopathy and (Amato, 165)
peripheral neuropathy. The clinical signs of
myelopathy are similar to those seen in HSP and 22. (D) Hereditary neuropathy with a tendency to
include ataxic gait, spasticity, and increased develop pressure palsies is inherited in an auto-
deep tendon reflexes. Signs of peripheral neu- somal dominant manner. It is characterized by
ropathy are weakness, and reduced or lost vibra- the occurrence of a painless numbness and
tion sensation and joint position. (Amato, 307) weakness in the distribution of a single nerve,
although multiple neuropathies and cranial neu-
19. (C) Spinal muscular atrophy type I (Werdnig ropathies can occur. The age of onset is usually
Hoffman disease) is the most severe form of spinal within the second and third decade. The most
muscular atrophy. Its clinical manifestations are commonly affected sites are the median nerve at
evident within the first 6 months of life. Affected the wrist, the ulnar nerve at the elbow, the radial
patients are hypotonic with a symmetric, gener- nerve at the arm, and the peroneal nerve at the
alized, or proximally predominant pattern of head of the fibula. Approximately 85% of cases
weakness. Facial weakness is mild and extraocu- of hereditary neuropathy with a tendency to
lar muscles are spared. Fasciculations are seen in develop pressure palsies are caused by a dele-
the tongue but rarely in limb muscles. Deep ten- tion of one copy of the PMP-22 gene . The neu-
don reflexes are typically absent. Abdominal ropathy may be also caused by a mutation of the
breathing, a weak cry, and a poor suck are com- PMP-22 gene, resulting in loss of function of the
monplace. There is no intellectual impairment. PMP-22 protein. (Amato, 167168).
Children with spinal muscular atrophy type I
are distinguished from those with other types of 23. (A) Approximately 20% of patients with Charcot
spinal muscular atrophy by the fact that they MarieTooth type 1 (CMT1) have CMT1B, which
never develop the ability to sit independently. is caused by a mutation in the myelin protein
(Amato, 123124) zero (MPZ) gene located on chromosome 1q22-
23. Mutation in the connexin-32 gene on chro-
20. (A) The association of gynecomastia, fascicula- mosome Xq13 causes CMT1X. Mutations in the
tions, fatigue, and mild CPK elevation in a neurofilament light chain gene located on chro-
young male is suggestive of X-linked bul- mosome p13-21 causes CMT1F. Mutation in the
220 5: Neuromuscular Diseases

heat-shock 27-kDa protein-1 gene located on patients. Sensory signs or symptoms are absent;
chromosome 7q11-q21 causes CMT2F, and however autonomic dysfunction may occur. The
mutation in the early growth response-2 pro- median time to recovery is similar to that
tein (ERG2) gene causes CMT3. (Amato, 162, seen with acute inflammatory demyelinating
166167) polyneuropathy; most affected patients have a
good recovery within a year. The mortality rate
24. (E) The patient described in the vignette devel- is less than 5%. (Amato, 222)
oped a combination of peripheral neuropathy,
cerebellar ataxia, retinitis pigmentosa, and ele- 26. (B) Multifocal motor neuropathy (MMN) is an
vated proteins in the cerebrospinal fluid. These immune-mediated demyelinating neuropathy
findings are suggestive of Refsum disease, a rare characterized clinically by asymmetric weak-
disorder inherited as an autosomal recessive ness and atrophy, typically in the distribution of
trait that has its onset in late childhood, adoles- peripheral nerves. The prevalence of MMN
cence, or early adult life. Diagnosis is based remains unknown but is estimated to be about 1
on a combination of clinical manifestations per 100,000. The protracted and chronic nature
retinitis pigmentosa, ataxia, and chronic of the disease means that the incidence is very
polyneuropathycoupled with the metabolic much lower than this. MNN occurs more com-
marker of the disease: an increase in blood phy- monly in males (3:1) and presents mainly
tanic acid. Phytanic acid accumulates because of between 20 and 50 years of age, although up to
a deficiency of the peroxisomal enzyme phy- 20% of cases may present later. Typically, a
tanoyl-coenzyme A (CoA) hydroxylase. The patient presents with progressive or sometimes
deficiency is caused by mutations in one of two stepwise asymmetric weakness affecting the
disparate genes. Cardiomyopathy and neuro- upper limbs. If the presentation is late, there may
genic deafness are present in most patients. The be a history of static weakness or spontaneous
polyneuropathy is sensorimotor, distal, and remission. Weakness of a single nerve or nerve
symmetric in distribution, affecting the legs branch, such as a radial or posterior interosseous
more than the arms. Although the nerves may nerve, or a nerve territory, as seen in median or
not be palpably enlarged, hypertrophic ulnar intrinsic hand weakness, is common early
changes with onion-bulb formation are invari- on, but weakness may spread to affect one or
able pathological features. The metabolic defect more other limbs, sometimes becoming conflu-
has been found to lie in the utilization of dietary ent. Patients frequently complain of cramps,
phytol; a failure of oxidation of phytanic acid often outside clinically affected areas, fatigue,
a branched-chain tetramethylated 16-carbon fatty and twitching. Involvement of cranial or respi-
acidthat accumulates in the absence of activity ratory nerves is unusual but reported. Mild sen-
of the enzyme phytanoyl-CoA-hydroxylase. sory symptoms and signs are not uncommon
Clinical diagnosis is confirmed by the finding of and should not exclude the diagnosis. At an
increased phytanic acid in the blood of a patient early stage, weakness without wasting in iden-
with a chronic, mainly sensory neuropathy; the tifiable nerve territories is typical. Occasionally,
normal level is less than 0.3 mg/dL, but in neurogenic muscle hypertrophy is evident, espe-
patients with this disease it constitutes 5% to cially in areas affected by cramps. Fasciculation
30% of the total fatty acids in serum lipids. or, rarely, myokymia is seen, which may be exac-
(Amato, 199; Ropper and Samuels, chapter 46) erbated by exercise. Some 22% to 84% of patients
with MMN have IgM serum antibodies directed
25. (C) Acute motor axonal neuropathy presents as against gangliosides, mainly GM1 but also asialo-
an abrupt onset of generalized weakness. It may GM1 and GM2; however, the importance of these
occur in children as well as adults. The distal antibodies in the pathogenesis of MMN is
limb muscles are often more severely affected unclear. (Amato, 248249; Lunn, 249358)
than the proximal ones. Cranial nerve deficits
and respiratory failure requiring mechanical 27. (B) Figure 5-5 illustrates winging of the right
ventilation can be seen in up to one third of scapula which may be enhanced by having the
Answers: 2432 221

patient flex the arm forward at the shoulder. charges are easily differentiated from fibrilla-
The figure shows that the whole scapula is wing- tions and positive sharp waves by their charac-
ing with the inferior angle rotated medially. teristic waxing and waning of both frequency
These findings are suggestive of neuropathy and amplitude. They can occur in some
affecting the right long thoracic nerve. This myopathies and occasionally in denervation
nerve arises by three roots from the fusion of the from any cause. However, the myotonic dis-
fifth, sixth, and seventh spinal roots to inner- charges in denervation are usually single brief
vate the serratus anterior. The roots from C5 and runs and are never the predominant waveform.
C6 pierce the scalenus medius, while the C7 root (Amato, 648)
passes in front of the muscle. The nerve
descends behind the brachial plexus and axillary 30. (C) Figure 5-8A shows a macular erythematous
vessels, resting on the outer surface of the ser- rash over the extensor surface of the knuckles
ratus anterior. Owing to its long, relatively suggestive of Gottron papules, as seen in der-
superficial course, it is susceptible to injury matomyostis. Such papules are found over bony
either by direct trauma or stretch. Injury has prominences, particularly the metacarpopha-
been reported in almost all sports, typically from langeal joints, the proximal interphalangeal
a blow to the ribs underneath an outstretched joints, and/or the distal interphalangeal joints.
arm. The long thoracic nerve can also be dam- Papules may also be found overlying the elbows,
aged during surgery for breast cancer, specifi- knees, and/or feet. The lesions consist of slightly
cally radical mastectomies that involve removal elevated violaceous papules and plaques. A
of axillary lymph nodes. Injury to the nerve can slight scale and, occasionally, a thick psoriasi-
result from carrying a heavy bag over the shoul- form scale may be present. These lesions may
der for a prolonged time. Damage to the dorsal resemble those of lupus erythematosus or psori-
scapular nerve may lead to winging of the asis. Other dermatological signs of dermato-
scapula, with the inferior angle rotated laterally. myostis includes dilated capillary loops evident
An axillary nerve lesion may cause paralysis of in the nail beds (Figure 5-8B); heliotrope rash,
the teres minor and deltoid muscles, resulting in which is a violaceous eruption on the upper eye-
impaired shoulder abduction and loss of sensa- lids, often with swelling; the shawl (or V) sign,
tion over a small part of the lateral upper arm. which is a diffuse, flat, erythematous lesion over
A thoracodorsal nerve lesion causes weakness of the chest and shoulders or in a V over the ante-
the latissimus dorsi muscle, which results in rior neck and chest, worsened with UV light;
impaired ability to adduct, medially rotate, and and an erythematous lesion similar to the shawl
extend the upper arm. A musculocutaneous sign but located in other areas, such as the malar
nerve lesion causes weakness of elbow flexion region and the forehead. (Amato, 688)
and supination of the forearm. It also causes a
discrete sensory disturbance on the radial side of 31. (B) Figure 5-9 demonstrates atrophy of type 2B
the forearm. (Amato, 398) fibers. These findings are characteristic of steroid
myopathy, the most common endocrine myopa-
28. (C) Figure 5-6 shows hematoxylin-and-eosin thy. (Amato, 725726)
staining of a muscle biopsy. It demonstrates fiber
size variability and increased endomysial and 32. (E) Malignant hyperthermia is a syndrome
perimysial connective tissue, consistent with a rather than a specific disorder; it is characterized
dystrophic process. (Amato, 548) by severe muscle rigidity, myoglobinuria, fever,
tachycardia, cyanosis, and cardiac arrhythmias
29. (A) Figure 5-7 shows myotonic discharges. This precipitated by depolarizing muscle relaxants
is a spontaneous discharge of a muscle fiber and inhalational anesthetic agents. In suscepti-
in which the amplitude and frequency of the ble individuals, these drugs can induce a dras-
potential wax and wane. An individual tic and uncontrolled increase in skeletal muscle
myotonic potential may have either a positive oxidative metabolism, which overwhelms the
wave or brief spike morphology. Myotonic dis- bodys capacity to supply oxygen, remove
222 5: Neuromuscular Diseases

carbon dioxide, and regulate body temperature, recognized that in about 10% of patients, hypoPP
eventually leading to circulatory collapse and is due to mutations in SCN4A, a voltage-gated
death if not treated quickly. Susceptibility to sodium channel Nav1.4 found at the neuromus-
malignant hyperthermia is often inherited as cular junction. Variants of HypoPP have been
an autosomal dominant disorder, for which associated with mutations in KCNE3, a voltage-
there are at least six genetic loci of interest, gated potassium channel. (Amato, 671672)
most prominently the ryanodine receptor gene
(RYR1) (chromosome 19q13.1). Ryanodine 35. (C) Ptosis and facial weakness are frequent
receptor mutations are found in at least 25% of manifestations of the classic form of myotonic
known malignant hyperthermia-susceptible dystrophy type 1. Although extaocular muscle
individuals in North America. (Amato, 675676; weakness may occur, it is usually asymptomatic.
Litman, 29182924) External ophthalmoplegia is a rare sign in
myotonic dystrophy type 1. (Amato, 647649)
33. (A) Becker muscular dystrophy (BMD) is a
milder form of dystrophinopathy. It can be dis- 36. (B) The phenotype described in this vignette is
tinguished from Duchenne muscular dystrophy highly suggestive of myotonia congenita in an
(DMD) by its slower rate of progression and by autosomal recessive form.
dystrophin analysis. Most patients with BMD Myotonia congenita may be inherited as an
experience difficulties initially between 5 and autosomal dominant (Thomsen) or recessive
15 years of age; however, onset in the third or (Becker) trait. It has been reported that the same
fourth decade or even later can occur. By defi- mutation may be inherited in a dominant fash-
nition, patients with Beckers dystrophy ambu- ion in one family and be recessive in another.
late beyond the age of 15 years. Serum CK lev- Mutations in CLCN1, the gene encoding the
els are elevated often 20 to 200 times the upper human skeletal muscle chloride channel, were
limit of normal. BMD may be distinguished his- subsequently shown to cause recessive and
tologically from DMD with immune staining, dominant myotonia. In Thomsen disease, symp-
whichusing carboxyl-terminal antibodies on toms begin in early childhood. The myotonia is
muscle membranesdemonstrates the presence generalized and the lower limbs tend to be more
of dystrophin in most cases of BMD. In contrast, severely affected. There may be marked muscle
immunostaining with antibodies directed against hypertrophy. Myotonia that diminishes with
the carboxyl terminal of dystrophin is usually exercise (warmup phenomenon) is characteris-
negative in DMD. (Amato, 537538) tic of both dominant and recessive disease.
Becker disease, or recessive generalized myoto-
34. (B) The patient in this vignette has signs and nia, is the only skeletal muscle channelopathy
symptoms suggestive of hypokalemic periodic inherited as an autosomal recessive trait.
paralysis (hypoPP). This is an autosomal domi- Although symptoms do not usually begin until
nant disorder and is the most common form of the second decade, the myotonia tends to be
primary periodic paralysis, with an estimated more severe. The condition tends to progress
prevalence of 0.4 to 1 in 100,000. Attacks of gen- slowly until the age of 30 to 40 years. Disability
eralized weakness may be triggered by rest after may result from severe myotonic stiffness of the
exercise and by a carbohydrate-rich meal the lower limbs, which are often markedly hyper-
preceding day; they may last hours to days. trophied. (Amato, 655658)
Weakness may be mild and limited to certain
muscle groups or may involve more severe full- 37. (E) Anti Jo-1 antibodies are the most common
body paralysis. Loss of deep tendon reflexes antisynthetase antibodies and are directed against
during an attack is characteristic of hypoPP cytoplasmic translational proteins. These anti-
Recovery is usually sudden when it occurs. bodies are associated with interstitial lung disease
Causal mutations were first identified in the in as many as 20% of patients with inflammatory
[alpha]-subunit of the skeletal-muscle calcium myopathy. Since methotrexate may cause pul-
channel gene CACNA1S (hypoPP1). It was later monary fibrosis, its administration in patients
Answers: 3343 223

with dermatomysitis and anti Jo-1 antibodies is include subclinical sensorineural hearing loss
not recommended. (Amato, 681688) and retinal telangectasias. More than 95% of
cases of FSHD are associated with the deletion
38. (B) The forearm ischemic exercise test is an of integral copies of a tandemly repeated 3.2-kb
important step in the diagnosis of muscle energy unit (D4Z4 repeat) at the subtelomeric region
disorders. In most normal patients, serum lactate 4q35 of the human genome, of which a normal
concentrations are raised more than 20 mg/dL chromosome will include between 11 and 150
and serum ammonia concentrations are raised repetitions of D4Z4. (Amato, 552554)
more than 100 g/dL. If neither lactate nor
ammonia concentration increases during the test, 41. (C) Endplate noise is generated at the endplate
the subject did not exercise strenuously and the region. It represents normal spontaneous activ-
test should be repeated. Failure to increase ity and is manifest as low-amplitude monopha-
ammonia concentrations more than 100 g/dL sic negative potentials that fire at 20 to 40 Hz. It
above baseline coupled with normal elevation of has the characteristic of a seashell or hissing
lactate response suggests myoadenylate deami- sound on EMG, representing miniature endplate
nase deficiency. This is a recessive genetic meta- potentials. Endplate noise is a normal finding in
bolic disorder that affects approximately 1% all individuals when the EMG needle is near the
to 2% of populations of European descent. neuromuscular junction. (Preston and Shapiro, 610)
Myoadenylate deaminase is an enzyme that con-
verts adenosine monophosphate (AMP) to ino- 42. (E) Endplate spikes are another normal wave-
sine monophosphate (IMP), freeing an ammonia form that can occur when the needle is near the
molecule in the process. It is a part of the meta- neuromuscular junction. They are brief, irregu-
bolic process that converts sugar, fat, and protein lar spikes with an initial negative deflection for
into cellular energy. If myoadenylate deaminase is each spike. They have a crackling, buzzing, or
deficient, excess AMP builds up in the cell and is sputtering sound on EMG and are caused by
eventually transported by the blood to the liver to irritation of the terminal axon twig by the EMG
be metabolized or to the kidneys to be excreted. needle, resulting in a depolarization of axon
Clinically, the typical history is intermittent mus- twigs. These propagate across the neuromuscu-
cles pain and weakness. (Amato, 609611) lar junction to produce motor fiber action poten-
tials. (Preston and Shapiro, 610)
39. (B) Small cell lung cancer is the most common
malignancy that causes subacute sensory neu- 43. (B) Both fibrillation potentials and positive
ronopathy, but cases of carcinoma of the esoph- sharp waves represent the extracellular record-
agus, breast, ovaries, and kidney as well as lym- ing of a single muscle fibers electrical activity
phoma have also been reported. (Amato, 313) occurring as a result of membrane instability.
These are the electrophysiological markers of
40. (A) Facioscapulohumeral muscular dystrophy loss of functional activity between the motor
(FSHD) is an autosomal dominant form of mus- axon and the muscle membrane. Fibrillation
cular dystrophy that initially affects the skeletal potentials are often described as producing a
muscles of the face, scapula, and upper arms. rain on a roof type of sound. They are prima-
Symptoms may develop in early childhood and rily recognized by their regular firing pattern
are usually noticeable in the teenage years, with (usually at a rate of 1 to 10 Hz); their morphol-
95% of affected individuals manifesting disease ogy is that of a single motor unit action poten-
by age 20. A progressive skeletal muscle weak- tial, and they have an initial positive deflection
ness usually develops in other areas of the body (1 to 5 ms in duration and typically 10 to 100 V
as well; often the weakness is asymmetric. Life in amplitude). Positive sharp waves have a brief
expectancy is normal, but up to 15% of affected initial positivity followed by a long negative
individuals become severely disabled and must phase. They create a dull popping sound. A
eventually use a wheelchair. Nonmuscular myotonic discharge is the spontaneous dis-
symptoms frequently associated with FSHD charge of a muscle fiber in which the potentials
224 5: Neuromuscular Diseases

amplitude and frequency wax and wane. This is denervated fibers. The number of newly rein-
characteristically seen in myotonic dystrophy, nervated fibers may exceed the normal number
myotonia congenita, and paramyotonia con- of fibers in the motor unit. This may lead to an
genita. (Preston and Shapiro, 610) increase in the duration, amplitude, and number
of phases. This process takes many weeks to
44. (A) A fasciculation potential is a spontaneous months to occur. In the acute setting, the mor-
involuntary discharge of an individual motor phology of the motor unit action potential
unit. The source generator of a fasciculation is remains normal. The only abnormality is a
the motor neuron or its axon. Fasciculations fire decreased number of motor unit action poten-
slowly, typically 0.51 Hz, and irregularly. tials in weak muscles owing to the initial loss of
Clinically, they are recognized as individual motor units. (Preston and Shapiro, 3233)
brief twitches that seldom result in significant
movement of a joint. (Preston and Shapiro, 1119) 49. (D) In myopathies, the number of functioning
muscle fibers in a motor unit decreases. Fewer
45. (C) Complex repetitive discharges occur from muscle fibers per motor unit results in shorter-
depolarization of a single muscle fiber followed duration and smaller-amplitude motor unit
by ephaptic spread to adjacent denervated action potentials. With dysfunction of the
fibers. On EMG, they are recognized as high- remaining muscle fibers, less synchronous firing
frequency, multiserrated repetitive discharges results in increased polyphasia. However, the
with an abrupt onset and termination and a number of functioning motor unit action poten-
machine-like sound. They are present in chronic tials remains normal. Thus, recruitment remains
neuropathic and myopathic disorders. (Preston normal for the level of activation. Since each
and Shapiro, 1119) motor unit contains fewer muscle fibers, each
unit generates less force. Consequently, more
46. (D) Myokymic discharges are rhythmic, motor unit action potentials, as compared with
grouped, and repetitive discharges of the same normal, are needed to generate a level of force
motor unit. Clinically, myokymia is usually rec- equivalent to the premorbid state, resulting in
ognized as the continuous, involuntary, quiver- early full recruitment. (Preston and Shapiro, 3435)
ing, rippling, or undulating movement of a mus-
cle. Myokymia is seen in a variety of conditions, 50. (B) In humans, two major types of muscle
including radiculopathy, entrapment neuropa- fibers, 1 and 2, have been defined on the basis of
thy, and demyelinating neuropathies. In a histochemistry and physiology. Type 1 fibers are
patient with a history of brachial plexopathy high in myoglobin and oxidative enzymes and
and of cancer and radiation therapy, the pres- have many mitochondria, in keeping with their
ence of myokymia is a specific although not nec- ability to perform tonic contraction; histologi-
essarily sensitive, sign supporting the diagnosis cally, they are defined by their dark staining for
of radiation plexitis rather than recurrent neo- adenosine triphosphatase (ATPase) at pH 4.2
plastic invasion. (Preston and Shapiro, 1119) but light staining at pH 9.4. Type 1 fibers are
slow-twitch red fibers. Type 2 fibers are rich in
47. (B) Neuromyotonic discharges are high-fre- glycolytic enzymes and are involved in rapid
quency (150- to 250-Hz) decremental discharges phasic contractions. They stain dark for ATPase
of a single motor unit that have a characteristic at pH 9.4 but light at pH 4.2. Type 2 fibers are
pinging sound on EMG. (Preston and Shapiro, fast-twitch white fibers. Since the motor neuron
1119) determines fiber type, all fibers of a single unit
are of the same type. These fibers are distrib-
48. (E) Acute axonal damage in a nerve causes uted randomly across the muscle, giving rise to
Wallerian degeneration after the first 4 to 7 days. the checkerboard pattern of alternating light and
This is followed by denervation of the distal dark fibers, as demonstrated especially well
muscle fibers of the involved motor units. with ATPase. Type-specific atrophy is charac-
Sprouting of the nearby axons reinnervates these teristic of some disease states. Type 2 fiber
Answers: 4452 225

atrophy is a relatively common finding and is weakness), hypotonia, and weakness of respi-
associated with inactivity or disuse. This type of ratory muscles (with absence of clinical myoto-
disuse atrophy may occur after fracture of a nia) are hallmarks. Additionally, 75% of the non-
limb and application of a plaster cast, in pyram- congenital patients and 81% of the congenital
idal tract degeneration, or in neurodegenerative patients have cardiac abnormalities, primarily
diseases. It may also occur with hyperthyroid conduction defects, demonstrated on ECG. The
myopathy and corticosteroid-induced myopa- heart is prominently involved, and the severity
thy. Type 1 fiber atrophy occurs with myotonic of cardiac symptoms does not correlate with the
muscular dystrophy, centronuclear myopathy, severity of other symptoms in this disorder.
and congenital fiber-type disproportion myopa- Central nervous system manifestations may
thy. (Karpali, 4748) include apathy, inertia, and hypersomnolence.
Structural changes in the brain are not common;
51. (C) Myotonic dystrophy type 1 (DM1) is an however, generalized atrophy and ventricular
autosomal dominant multisystem degenerative dilatation may be seen. Endocrinological abnor-
disease characterized by myotonia, progressive malities have been reported, including hyperin-
muscular weakness, gonadal atrophy, cataracts, sulinism with reduced insulin receptors, ele-
and cardiac dysrhythmias. The molecular basis vated pituitary FSH and LH, testicular atrophy
of DM1 is an unstable trinucleotide repeat (seen in 60% to 80% of patients), Leydig cell
sequencecytosine, thymine, and guanidine hyperplasia, and reduced testosterone level.
(CTG)in the protein kinaseencoding gene (Goetz, 707708)
(DMK) located at 19q13.3. The normal CTG
repeat is between 5 and 30, whereas in DM1 the 52. (C) Neurological complications of diphtheria
CTG repeat is 50 to several thousand. The size of parallel the extent of the primary infection and
the repeats correlates with the anticipation phe- are multiphasic in onset. Some 2 to 3 weeks after
nomenon as well as with the severity of symp- the onset of oropharyngeal inflammation, weak-
toms. There is an estimated prevalence of 3 to 5 ness of the posterior pharynx, larynx, and facial
per 100,000 population and an incidence of 1 in nerves occurs, causing nasal speech. Death may
8,000 live births, making it the most common occur from aspiration. Blurred vision, strabis-
adult muscular dystrophy. mus, and accommodation abnormalities are
The clinical presentation is variable, ranging manifestations of oculomotor and ciliary paral-
from a single relatively benign presentation ysis and may occur in the fifth week. The periph-
(such as cataracts) of middle age to severe eral nervous system manifestations of diphthe-
neonatal hypotonia, which can lead to death if ria include a symmetric polyneuropathy that
respiratory support is not provided. The classic appears between 10 days and 3 months after the
presentation of noncongenital DM1 includes onset of the disease, with distal weakness that
marked weakness in the face, jaw, and neck progresses proximally as well as decreased deep
muscles and milder weakness in the distal tendon reflexes. Paralysis of the diaphragm can
extremities, often perceived earlier than the ensue. Complete recovery is likely. Rarely, 2 to
myotonia. Myotonia can be elicited by a brisk 3 weeks after onset of the illness, dysfunction of
percussion of the thenar muscles, causing flex- the vasomotor centers can cause hypotension
ion opposition of the thumb with slow relax- or cardiac failure.
ation. In the advanced stage of myotonic dys- Diabetic cranial mononeuropathies are
trophy, the patient may present a characteristic caused by peripheral nerve microinfarction as
long, thin face with sunken cheeks, due to tem- well as fascicular ischemic lesions within the
poral and masseter wasting, and atrophy of the brain. Diabetic oculomotor cranial mononeu-
sternocleidomastoid, causing a swan neck and ropathies primarily involve the oculomotor
ptosis. Congenital myotonic dystrophy presents nerve, followed by the abducens nerve; the
a distinctive picture that is different from that trochlear nerve is uncommonly affected alone.
of other myotonic disorders. Facial diplegia, Clinically, the patient may report eye pain or
jaw weakness (without concomitant extremity headache followed by a diplopia. In the setting
226 5: Neuromuscular Diseases

of oculomotor involvement, pupillary sparing is somal dominant disorder with onset before the
noted in 80% to 90% of cases. second decade. Family history may be absent
Sarcoidosis may affect virtually any part of in about 20% of cases. There are two main
the nervous system. Involvement of the facial genetic variants of CMT1: CMT1A (75%) and
nerve leading to unilateral facial nerve palsy CMT1B (20%). Up to 90% of CMT1A patients
is the most commonly recognized symptom, have a tandem DNA duplication on the short
although any cranial nerve can be affected. arm of chromosome 17. Several laboratories
Unusual combinations of neurological deficits have mapped the human PMP-22 gene to chro-
affecting the central nervous system and/or mosome 17p11.2-p12. The duplication leads to
peripheral nerves should raise the clinical sus- an overexpression of the PMP-22. Histologically,
picion of sarcoidosis. CMT1A may show small-diameter axons and
The neuropathy in porphyria is primarily frequent onion bulbs.
motor. Weakness begins in the proximal mus- CMT1B is linked to chromosome 1q22-q23,
cles, arms more commonly than legs. Paresis is where the gene locus for myelin protein zero
often focal; cranial nerve involvement may (P0) gene is mapped. Histologically, CMT1B
occur, especially in the oculomotor, facial, and biopsies show loss of myelinated fibers, small
vagus nerves. onion bulbs, and tomaculous formations.
The possible neurological manifestations of CharcotMarieTooth neuropathy type 2
Lyme disease include lymphocytic meningitis (CMT2) is also an autosomal dominant condition.
with episodic headache and mild neck stiffness, It is significantly less common than CMT1 and
subtle encephalitis with difficulty with menta- accounts for one third of all autosomal dominant
tion, cranial neuropathy (particularly unilateral CMT. There are three genetic variants: CMT2A,
or bilateral facial palsy), motor or sensory CMT2B, and CMT2C. Electrophysiologically, all
radiculoneuritis, mononeuritis multiplex, cere- CMT2 patients exhibit findings of primary
bellar ataxia, and myelitis. In children, the optic axonal sensorimotor neuropathy. The motor
nerve may also be affected because of inflam- nerve conduction velocities are normal or mildly
mation or increased intracranial pressure, which slowed. Sensorimotor nerve action potential
may lead to blindness. (Crimlisk, 319328; Newman, amplitudes are reduced. CMT2A is linked to the
Rose, and Maier, 12241234; Steere, 115125) short arm of chromosome 1 (1p35-36), while
CMT2B is mapped to the long arm of chromo-
53. (E) The dominant form of X-linked Charcot some 3 (3q13-22).
MarieTooth (CMT) neuropathy accounts for CharcotMarieTooth neuropathy type 3
approximately 10% to 15% of the dominant (CMT3) is a severe neuropathy that begins in
forms of CMT neuropathy. It becomes sympto- infancy or early childhood. Occasionally, infants
matic in the first decade of life. The disorder is have neonatal hypotonia and delayed motor
related to an abnormal marker of the connexin- milestones. There is proximal and distal limb
32 gene, which is a gap junction protein involv- weakness, significant sensory ataxia, and diffuse
ing intercellular communication. Clinical man- areflexia. The peripheral nerves are enlarged
ifestations include distal muscle weakness and and palpable. Skeletal and foot abnormalities
atrophy, areflexia, distal sensory loss, pes cavus, are present. Most CMT3 cases are sporadic, and
hammertoes, and claw-hand deformity. Women the inheritance is traditionally described as auto-
are less severely affected than men. Enlarged somal recessive.
nerves are infrequent. The recessive form is rare. CharcotMarieTooth neuropathy type 4
Spasticity and pyramidal signs may also be pres- (CMT4) is rare. It also begins in infancy or child-
ent, but mental retardation is seen only when the hood. There is a delay in acquiring motor mile-
onset is in infancy. Pathologically, there is pri- stones. The distal weakness and atrophy spread
mary axonopathy with secondary demyelination. to proximal muscles in the second decade. Facial
CharcotMarieTooth neuropathy type 1 muscles may become weak; there is areflexia;
(CMT1) is the most common hereditary motor adults become wheelchair-bound. Sensory loss
and sensory neuropathy (HMSN). It is an auto- is mild. Skeletal abnormalities are common.
Answers: 5356 227

Electrophysiologically, nerve conduction veloc- nant and recessive forms that are allelic disor-
ities are slowed to 15 to 30 m/s. This helps dif- ders. The autosomal dominant form is also
ferentiate CMT4 from CMT3 and CMT2C, which known as Thomsen disease; the autosomal
also begin in childhood. CMT4 is inherited in an recessive form is known as Becker myotonia
autosomal recessive mode. The Tunisian form congenita. Both diseases are benign and associ-
was mapped to chromosome 8q13-21.1 and is ated with diffuse muscle hypertrophy and elec-
termed CMT4A. (Murakami, 233235) trical myotonia. Cold increases the myotonia,
and sustained exercise improves it (warmup
54. (E) Involvement of organs or tissues other than phenomenon). There is no involvement of the
muscle may provide helpful clues in making the heart or other organs. Patients with Thomsen
appropriate diagnosis of myopathy. Cardiac disease are not weak, but those with Becker
arrhythmias are associated with KearnsSayre myotonia congenita develop limb-girdle weak-
syndrome, Anderson syndrome, polymyositis, ness and the myotonia is more severe. Patients
and EmeryDreifuss muscular dystrophy. with myotonia congenita do not complain of
Congestive heart failure may be seen in pain, which is a feature that distinguishes them
Duchenne muscular dystrophy, Becker muscu- from those with proximal myotonic myopathy.
lar dystrophy, EmeryDreifuss myopathy, nema- The membrane defect consists of markedly
line myopathy, acid maltase deficiency, carni- reduced chloride conductance, resulting in
tine deficiency, and polymyositis. Respiratory hyperexcitability and afterdepolarization and
failure may be the presenting symptom of producing involuntary myotonic potentials.
myotonic dystrophy, centronuclear myopathy, Paramyotonia congenita and hyperkalemic
nemaline myopathy, or acid maltase deficiency. periodic paralysis are due to point mutations
Hepatomegaly may be seen in myopathies in the voltage-dependent sodium channel
associated with deficiencies in acid maltase, (SCN4A) gene on chromosome 17q23-25. These
debranching enzyme, and carnitine. (Schapira, are autosomal dominant conditions. All have
184) symptoms beginning in the first decade and con-
tinuing throughout life. Paramyotonia congenita
55. (A) The pattern of weakness described in this is characterized by paradoxical myotonia in that
case shows distal arm and proximal leg weak- the muscle symptoms increase with repetitive
ness. The distal arm weakness involves the wrist movements. This is often best observed on
and ulnar finger flexors. The proximal leg weak- repeated forced eye closure: after several
ness involves the quadriceps, which is a knee attempts, the patient cannot open the eyelids.
extensor. The asymmetry of the weakness and Muscle stiffness is worsened by cold tempera-
sparing of the face make this pattern highly sug- ture. Hyperkalemic periodic paralysis is char-
gestive of inclusion body myositis. This pattern acterized by attacks of weakness lasting no more
may also uncommonly occur in myotonic dys- than 1 or 2 hours. Attacks are precipitated by
trophy; however, unlike inclusion body myosi- fasting, by rest shortly after exercise (minutes or
tis, muscle weakness is symmetric. In acid mal- several hours), the ingestion of potassium-rich
tase deficiency, the weakness involves the trunk foods or compounds, and cold. During attacks,
and proximal limbs and the progression is slow, patients are areflexic with normal sensation and
taking years. Nemaline myopathy is mainly there is no ocular or respiratory muscle weak-
seen in infancy and early childhood and is char- ness. The serum potassium level may or may
acterized by hypotonia and muscle weakness. not be increased during the attack. Strength is
Polymyositis has a pattern of symmetric proxi- generally normal between attacks, but some
mal limb weakness involving the muscles of the patients can have mild interictal limb-girdle
hip, thigh, and shoulders. (Schapira, 311317) weakness. Episodes of weakness are rarely seri-
ous enough to require acute therapy; oral car-
56. (B) Myotonia congenita is due to point muta- bohydrates or glucose may improve weakness.
tions in the muscle chloride channel gene on Hypokalemic periodic paralysis is due to
chromosome 7q35. There are autosomal domi- abnormal muscle membrane excitability arising
228 5: Neuromuscular Diseases

from mutations in the muscle calcium channel There are two main genetic variants of
alpha-1 subunit on chromosome 1q31-32. The CMT1: CMT1A (75%) and CMT1B (20%). The
mutation produces a reduction of the calcium remaining cases are genetically more heteroge-
current in the T tubule. During attacks, there is neous. Up to 90% of CMT1A patients have
an influx of potassium into muscle cells and the tandem DNA duplication on the short arm of
muscles become electrically unexcitable. Patients chromosome 17 (17p11.2-12), causing an over-
have an increased sensitivity to the effects of expression of PMP-22, which is a 22-kDa mem-
insulin on potassium. Hypokalemic periodic brane glycoprotein localized to the compact por-
paralysis is an autosomal dominant condition. It tion of the peripheral nerve myelin. Nerve
is the most frequent form of periodic paralysis biopsy in CMT1A may show small axonal diam-
and is more common in males, with a reduced eter and onion bulbs. CMT1B is linked to chro-
female penetrance. Attacks begin by adolescence mosome 1q22-q23, where the gene locus for
and are provoked by exercise followed by sleep, myelin protein zero (P0) gene is mapped. It is a
stress, alcohol, or meals rich in carbohydrates member of the large family of adhesive mole-
and sodium. The episodes last from 3 to 24 hours. cules and plays a role in the compaction of
A vague prodrome of stiffness or heaviness in peripheral nerve myelin. Histologically, CMT1B
the legs can occur. Rarely, ocular, bulbar, and biopsies show loss of myelinated fibers, small
respiratory muscles can be involved in severe onion bulbs, and tomaculous formations.
attacks. (Schapira, 135175) (Mendell, 431436)

57. (E) CharcotMarieTooth neuropathy type 1 58. (E) The dominant form of X-linked CMT dis-
(CMT1) is the most common hereditary motor ease (CMTX) becomes manifest in the first
and sensory neuropathy (HMSN). It is an auto- decade of life. There is distal muscle weakness
somal dominant disorder with onset before the and atrophy, distal sensory loss, and areflexia.
second decade. Although family history may Pes cavus and hammertoes are common, and
not be reported in about 20% of cases, detailed claw-hand deformity may occur in the adult
investigations, including clinical and electro- male. Women are mildly affected. Enlarged
physiological evaluations of asymptomatic fam- nerves are infrequent. The recessive form is rare.
ily members, improve the yield significantly. In Spasticity and pyramidal signs are also present,
all CMT1 subtypes, 50% to 75% of patients have but mental retardation is seen only when the
pes cavus and hammertoes. There is distal mus- onset is in infancy. Pathologically, there is pri-
cle weakness and atrophy in the legs. About mary axonopathy with secondary demyelina-
65% of cases have distal upper limb involve- tion. The disorder is linked to the marker
ment. Distal sensory impairment is present but DXYS1, a marker for the connexin-32 gene. CX32
usually asymptomatic. The vibratory sensation is a gap junction protein involved in intercellu-
is most often diminished. Muscle stretch lar communication. (Mendell, 445447)
reflexes are absent in about 50% of patients.
Nerve enlargement is present in at least 25% of 59. (D) Multifocal motor neuropathy (MMN) is a
patients. Electrodiagnostic testing reveals slow- demyelinating neuropathy, presumably of
ing of nerve conduction velocities to less than autoimmune origin. The arguments in favor of
75% of the lower limit of normal in all nerves. an autoimmune origin are the presence of con-
The slowing is present in early childhood. duction blocks, as seen in chronic inflammatory
Definite conduction block is characteristically demyelinating polyneuropathy (CIDP), the pres-
absent. Compound sensory and motor action ence of anti-GM1 antibodies, and the effective-
potential amplitudes are often low in the lower ness of immunomodulating therapy. Age at
limbs. Needle EMG shows chronic neurogenic onset is highly variable, with reports of patients
motor unit action potentials mainly in the dis- in their 20s to as old as 70 years. It is more com-
tal muscles. The magnitude of axonal changes is mon in males than females. Patients have grad-
a better prognostic indicator than slowing of ual, progressive, asymmetric weakness in the
nerve conduction velocities. distribution of one or more motor nerves. The
Answers: 5761 229

duration of symptoms at presentation is usually spectrum of clinical presentation. Mandatory fea-


greater than 1 year, and durations of 20 years or tures are those required for diagnosis and should
more have been reported. Upper extremity be present in all definite cases. Supportive
involvement is more common than lower features are helpful in clinical diagnosis but
extremity involvement, and there is usually a do not by themselves make a diagnosis.
distal predominance. Exclusionary features strongly suggest alternative
Thus, the most common presentation is that diagnoses.
of a young to middle-aged male with slowly The clinical mandatory features include pro-
progressive asymmetric hand weakness over gressive or relapsing motor and/or sensory dys-
months or years. Atrophy may or may not be function of more than one limb of a peripheral
present; one hallmark of MMN is weakness out nerve (developing over at least 2 months), and
of proportion to the degree of atrophy. Bulbar hypo- or areflexia, usually of all four limbs. The
function and other cranial innervated muscles supportive clinical features include large-fiber
are usually spared. Sensory symptoms and signs sensory loss predominating over small-fiber sen-
are absent or minimal. Reflexes vary but are sory loss. The exclusionary features include
usually decreased focally in affected areas. mutilation of hands or feet, retinitis pigmentosa,
MMN lacks the upper motor neuron findings of ichthyosis, appropriate history of drug or toxic
classic ALS. Features that can help differentiate exposure (known to cause a similar peripheral
the two include multifocal demyelination on neuropathy), or family history of an inherited
electrodiagnostic studies in MMN, weakness in peripheral neuropathy, the presence of sensory
the distribution of major motor nerves in MMN, level, and unequivocal sphincter disturbance.
the presence of very high titers of anti-GM1 anti- The mandatory electrodiagnostic study features
bodies in some but not all patients with MMN, include predominance of demyelination in the
and the response of MMN to intravenous proximal nerve segments with reduced con-
immunoglobulin (IVIg) or cyclophosphamide. duction velocity and prolonged distal latency.
Corticosteroids generally have no effect on The mandatory CSF studies include a cell count
MMN or produce worsening. Plasma exchange below 10/mm3 if HIV-seronegative or below
has not been effective and may even worsen the 50/mm3 if HIV-seropositive and there is a neg-
condition. Cyclophosphamide leads to improve- ative VDRL test. Elevated protein level in the
ment in most patients. Although the effect may CSF is a supportive feature. If nerve biopsy is
last for several months after completion of the performed, the mandatory pathological features
course of cyclophosphamide, weakness often include unequivocal evidence of demyelination
recurs after discontinuation of the medication, and remyelination. The exclusionary patholog-
requiring resumption of treatment. IVIg is effec- ical features for CIDP include vasculitis, neuro-
tive in most patients with MMN, including some filamentous swollen axons, amyloid deposits,
who have been unresponsive to cyclophos- or intracytoplasmic inclusions in Schwann cells
phamide. (Mendell, 192201) or macrophages, indicating adrenoleukodystro-
phy, metachromatic leukodystrophy, globoid
60. (B) Chronic inflammatory demyelinating cell leukodystrophy, or other evidence of specific
polyneuropathy (CIDP) is a clinical diagnosis pathology. However, nerve biopsies are not
based on symptoms and signs, electrodiagnos- required to make a clinical diagnosis. (Mendell,
tic studies, cerebrospinal fluid (CSF) examina- 173191)
tion, laboratory tests appropriate to the specific
clinical situation, and, occasionally, nerve biopsy. 61. (B) Inflammatory demyelinating polyradicu-
Four features are used as the basis of diagnosis: loneuropathy (CIDP) is a chronic disorder of the
clinical, electrodiagnostic, pathological, and CSF peripheral nervous system. It may have a relaps-
studies. These are further divided into (A) ing, monophasic, or progressive course and is
mandatory, (B) supportive, and (C) exclusionary. generally steroid-responsive. Multifocal motor
While these criteria have been established for neuropathy (MMN) is characterized by a slowly
research purposes, there is a highly variable progressive, asymmetric, multifocal weakness
230 5: Neuromuscular Diseases

with atrophy that may mimic motor neuron dis- Early systemic symptoms of acute arsenic
ease, but it demonstrates features of multifocal intoxication include nausea, vomiting, and diar-
conduction block and slowing in motor nerves. rhea. Initial laboratory findings reflect abnor-
This condition represents a demyelinating neu- mal liver function and depressed bone marrow,
ropathy, which is generally treatable. sometimes with pancytopenia and basophilic
Both CIDP and MMN may have their onset stippling of red blood cells. CSF protein is ele-
in adults of all ages; CIDP rarely affects chil- vated in most patients with severe arsenic neu-
dren. Male predominance is found in both dis- ropathy. Increased urinary arsenic excretion is an
eases. Weakness tends to be symmetric in CIDP important feature of recent exposure. The half-
and asymmetric in MMN, with upper extremi- life of urinary arsenic excretion after acute expo-
ties more involved than lower extremities and sure is about 3 weeks, making it a helpful test
distal muscles more involved than proximal early after exposure. The magnitude of expo-
muscles. Large-fiber impairment is more com- sure also can be related to accumulation in hair
mon in CIDP, whereas it is minimal or absent in or nails at a later time. Serial nerve conduction
MMN. Reflexes are globally decreased in CIDP, studies in patients with arsenic neuropathy
whereas they are focally decreased or absent in demonstrate evidence of a distal dying-back
MMN. Sensory nerve studies are usually abnor- neuropathy with progressive axonal degenera-
mal in CIDP but normal in MMN. Motor nerve tion (findings confirmed on nerve biopsy).
conduction studies in both MMN and CIDP N-hexane, an organic solvent, is thought to
demonstrate acquired demyelination with con- be responsible for the neuropathy seen in glue
duction block, abnormal temporal dispersion, sniffers. The sensory component is usually min-
slowed conduction velocities, prolonged distal imal as compared with the motor component.
latencies, and prolonged F-wave latencies. Low An unusual and characteristic of this neuropa-
titers of anti-GM1 antibodies may be present in thy is that the clinical condition frequently con-
CIDP patients, whereas they are present at high tinues to deteriorate for some months after expo-
titers in about half of MMN patients. CSF pro- sure ceases. It is also characterized by a pure
tein is usually elevated in CIDP and normal or motor neuropathy or mixed neuropathy with
elevated to less than 100 mg/dL in MMN. motor predominance. Sufficient exposure to n-
Sensory nerve biopsy in CIDP may show hexane produces a dying-back sensorimotor
demyelination, axonal degeneration, mononu- neuropathy characterized by distal weakness,
clear inflammation, and endoneurial edema, stockingglove sensory loss, and absent ankle
whereas it is normal or shows minor abnormal- reflexes. Involvement may be severe, as in the
ities in MMN. Prednisone, IVIg, and plasma case example, with profound weakness and sen-
exchange are the usual treatment for CIDP, sory loss. In the majority of reports, motor signs
whereas IVIg and cyclophosphamide are usu- predominate, but pure motor neuropathy is
ally used in MMN. (Mendell, 173201) unusual and inconsistent with the known sural
nerve abnormalities. Nerve conduction studies
62. (D) Peripheral neuropathy is a common and in asymptomatic n-hexaneexposed individu-
debilitating complication of arsenic intoxication. als may be normal or demonstrate mildly
It may present as a distal symmetric axonal sen- slowed motor conduction velocities. In sympto-
sorimotor polyneuropathy with motor predom- matic patients, initial findings consist of reduced
inance. Neuropathic features begin 5 to 10 days sensory amplitudes, followed by reduced motor
after acute exposure, progressing over several amplitudes and conduction velocities, some-
weeks; they often resemble those of GBS. The times to 35% to 40% of the lower limit of normal.
neuropathy involves sensory and motor axons. The reduced conduction velocity and partial
Unlike the case in GBS, neuropathy is only one conduction block are explained by secondary
component of a systemic intoxication; other myelin changes caused in part by axonal
features provide important clues that something swelling, demonstrated in humans and experi-
other than idiopathic GBS explains the neu- mental animals in peripheral and central nerve
ropathy. fibers.
Answers: 6264 231

Dapsone produces a neuropathy character- The condition affects the older population, with
ized by weakness and muscle wasting that fre- a mean age at onset of 70 years and a female-to-
quently involves the arms more than the legs. It male incidence of 3 to 1. The disorder is charac-
is one of several toxins associated with motor terized by the indolent onset of myalgia,
involvement or motor greater than sensory stiffness, aching, and fatigue predominantly
involvement but no conduction block. Dapsone affecting the neck, shoulder, and hip region.
neuropathy is thought to reflect primary or Symptoms are typically worse in the morning,
exclusive axonal degeneration of motor fibers, when prominent stiffness occurs. Low-grade
although controversy exists regarding the pres- fever, depression, anemia, and weight loss can
ence or absence of sensory involvement. accompany the muscular manifestation.
Neuropathic toxicity of pyridoxine is dose- Laboratory evaluation reveals typically normal
related, either to long-term cumulative expo- CK level and high sedimentation rate, often to a
sure or short-term administration of large doses. value greater than 100 mm/h. Muscle biopsies
Symptoms include unpleasant distal paresthe- are invariably normal. Polymyalgia rheumatica
sias and numbness. Associated signs include occurs in approximately 50% of patients with
areflexia, profoundly reduced vibration and giant cell arteritis. Approximately 15% of
joint position sensations, with minimally patients with the diagnosis of polymyalgia
decreased pinprick sensation. With particularly rheumatica will develop giant cell arteritis.
large doses of pyridoxine, sensory loss may be Although occasional patients with polymyalgia
virtually complete, including facial and mucous rheumatica respond to nonsteroidal anti-
membrane areas, with little resolution after inflammatory drugs, most require treatment
removal from exposure. Such profound loss is with corticosteroids, which usually results in a
consistent with a sensory neuronopathy. Sensory dramatic improvement of the myalgia and stiff-
nerve conduction studies are the only tests able ness. (Schapira and Griggs, 4041)
to localize sensory loss to the periphery, but
sensory nerve action potentials persist for up to 64. (E) Facioscapulohumeral dystrophy is an
10 days after clinical sensory loss is identified. autosomal dominant disorder. The prevalence
Neuropathy is the exclusive neurotoxicity is approximately 1 in 20,000. There are two
associated with nitrofurantoin. Neuropathy distinctive patterns of progressive muscular
develops in a small proportion (less than 0.5%) weakness involving the face, scapular stabi-
of patients receiving nitrofurantoin for extended lizer, proximal arm, and peroneal muscles. The
periods (usually exceeding 1 to 2 months). first is a gradually descending autosomal dom-
Neuropathy is most common in elderly patients inant form and the second is a jump form in
with abnormal renal function, presumably which the progressive weakness jumps from
resulting in high blood levels. The neuropathy is the upper body to the peroneal muscles. The
a mixed sensorimotor polyneuropathy. Onset is age of onset is from infancy to middle age. The
with distal dysesthesias and sensory loss involv- initial weakness typically affects the facial
ing large-fiber modalities. With continued use, muscles, especially the orbicularis oculi and
motor symptoms, sign development, and sen- oris. The masseter, temporalis, extraocular, and
sory loss may be severe. Weakness may be sub- pharyngeal muscles are usually unaffected.
acute and progress to respiratory failure, super- Shoulder weakness is the presenting symptom
ficially resembling GBS. When the condition is in more than 82% of symptomatic patients.
recognized and nitrofurantoin discontinued, Involvement of the scapular fixator muscles
most patients improve or recover. (Mendell, the latissimus dorsi, trapezius, rhomboids, and
316330) serratus anteriorcauses a winging of the
scapula, a highly characteristic sign. The
63. (D) Polymyalgia rheumatica may be one of the scapula is placed more laterally than normal
more common causes of muscle pain in adults and moves upward in shoulder abduction. The
above 50 years of age. One study suggests that deltoid muscle is typically not affected.
the prevalence of the disease is 600 per 100,000. (Schapira and Griggs, 6168)
232 5: Neuromuscular Diseases

65. (A) Miyoshi myopathy is clinically character- (percussion myotonia). In classic myotonia, the
ized by autosomal recessive inheritance; the myotonia improves as muscles warm up,
onset is in early adulthood with preferential gas- whereas in paradoxical myotonia (paramyoto-
trocnemius muscle involvement and dystrophic nia) it worsens with repeated muscle contrac-
muscle pathology. The gene has been mapped to tions. Genetic linkage studies have now
chromosome 2p13 and has been cloned. The pre- pinpointed the lesions to chromosomal loci
dicted gene product has been named dysferlin. encoding specific ion channels and a protein
The location and function of dysferlin remain kinase. In sodium channel diseases, the gene
unknown. Mutations are variable and include defect is located on chromosome 17; these
insertions, deletions, altered splicing, and point include hyperkalemic periodic paralysis, paramy-
mutations. Bethlem myopathy is a rare autoso- otonia congenita, and potassium-sensitive myoto-
mal myopathy characterized by a slowly pro- nia congenita. In protein kinaserelated diseases,
gressive limb-girdle weakness from childhood the gene defect is located on chromosome 19
onward with periods of arrest for several and includes myotonic dystrophy. In chloride
decades and flexion contractures of fingers, channel diseases, the gene defect is located on
elbows, and ankles. The disease has been chromosome 7 and includes autosomal domi-
demonstrated to be due to a type VI collagen nant myotonia congenita (Thomsen myotonia),
gene defect. EmeryDreifuss muscular dystro- and autosomal recessive myotonia congenita
phy is an X-linked disorder characterized by a (Becker myotonia). (Ptacek, Johnson, and Griggs,
slowly progressive wasting and weakness of the 482489)
scapulohumeral, anterior tibial, and peroneal
muscle groups. Cardiomyopathy with conduc- 68. (E) Congenital myotonic dystrophy is an auto-
tion defects is common. The defective gene is somal dominant disorder caused by an abnor-
mapped to Xq28. Oculopharyngeal muscle dys- mal unstable expansion of a trinucleotide repeat
trophy is an autosomal dominant disease linked gene on chromosome 19. The tissues that are
to the chromosome 14q11. Nonaka myopathy is commonly involved, in addition to the skeletal
linked to chromosome 9p1q1. (Nonaka, 493499) muscle, include heart, smooth muscle, lens,
brain, and endocrine tissues. The cardinal sign of
66. (A) Duchenne-type muscular dystrophy is the adult myotonic dystrophy is myotonia. It is
most common form of dystrophy. It is inherited absent in cases of congenital myotonic dystro-
as an X-linked recessive trait and therefore pre- phy and gradually appears during childhood.
dominantly affects boys. It is a serious condition At birth, there is a frequent history of hydram-
with progressive muscle wasting and weakness nios and reduced fetal movements. Neonatal
that causes most affected boys to start using respiratory distress may occur. Other signs of
wheelchairs by age 12 and to die in their 20s. The congenital myotonic dystrophy include hypo-
associated gene in Duchenne-type muscular tonia, bilateral facial weakness, feeding diffi-
dystrophy produces dystrophin. Histochemical culty, and mental retardation. The gene defect
studies on muscle sections without muscular results from an abnormal expansion of the trin-
dystrophy indicate that dystrophin is localized ucleotide repeat (CTG) of a gene on chromo-
at the periphery of muscle fibers. It is a some 19, which codes for serine/threonine
cytoskeletal protein located beneath the sar- kinase. (Schapira and Griggs, 118124)
colemma. In Duchenne-type dystrophy, there is
dystrophin deficit and the majority of fibers fail 69. (E) Hyperkalemic periodic paralysis is an auto-
to stain for dystrophin. (Emery, 991995) somal dominant disorder that can occur with or
without myotonia or with paramyotonia. It usu-
67. (A) Myotonic muscle disorders represent a het- ally begins in the first decade of life. The attack
erogeneous group of clinically similar diseases commonly starts in the morning before break-
sharing the feature of myotonia: delayed relax- fast, lasts from 15 minutes to an hour, and then
ation of muscle after voluntary contraction spontaneously resolves. Rest often provokes an
(action myotonia) or mechanical stimulation attack, particularly if preceded by strenuous
Answers: 6573 233

exercise. Potassium loading usually precipitates symptoms is suggestive of myoneurogastroin-


an attack. Cold environment, emotional stress, testinal encephalopathy. It is defined by the com-
glucocorticoids, and pregnancy provoke or bination of chronic intestinal pseudoobstruction
worsen the attacks. The generalized weakness is with skeletal myopathy, ophthalmoplegia, and
usually accompanied by a significant increase peripheral neuropathy. The gastrointestinal
of serum potassium, up to 5 to 6 mM/L. motility disturbance manifests as chronic nau-
Sometimes, the serum potassium level remains sea, vomiting, diarrhea, and malabsorption with
within the upper normal range and rarely progressive malnutrition, often leading to death
reaches a toxic level. The frequency of the attacks in the third or fourth decade of life. Postmortem
is variable, from a few times per year to daily. changes include a severe visceral neuropathy or
The gene defect is located on chromosome 17q23 scleroderma-like changes. The peripheral sen-
(coding for the subunit of the adult human sorimotor neuropathy and skeletal myopathy
skeletal muscle sodium channel SCN4A). contribute to muscle weakness and atrophy
Preventive therapy consists of frequent meals accompanying the chronic progressive external
rich in carbohydrates and low in potassium, ophthalmoplegia (CPEO). Deafness is also com-
avoidance of fasting, strenuous work, and expo- mon, and there may be cognitive decline due to
sure to cold, and continuous use of thiazide a leukoencephalopathy. CT or MRI has shown
diuretics or acetazolamide. Some patients can extensive white matter changes in around 50%
abort or attenuate attacks by the prompt oral of the cases described, and electrical studies con-
intake of a thiazide diuretic or by inhalation firm the presence of a sensorimotor neuropathy
of an adrenergic agent that stimulates the with both axonal and demyelinating compo-
sodiumpotassium pump. (Schapira and Griggs, nents. Muscle biopsy shows numerous ragged-
143144) red fibers with a partial defect of cyclooxygenase
(COX) as the most common biochemical finding.
70. (C) The patient described in this vignette died (Schapira and Cock, 886898)
of a progressive necrotizing encephalopathy
involving the thalamus, pons, inferior olive, 72. (A) A rapid loss of vision in a young healthy
and spinal cord, with sparing of the cortex. man with external ophthalmoplegia and bilat-
These findings are suggestive of Leighs syn- eral optic atrophy, as described in this vignette,
drome, also known as subacute necrotizing points to the diagnosis of Leber hereditary optic
encephalopathy. Although it is a multisystemic neuropathy (LHON). It is recognized as the most
disorder with hepatic dysfunction and chronic common cause of isolated blindness in young
acidosis, the clinical picture is dominated by men, with an estimated incidence of 1 in 50,000.
nervous system involvement, including devel- Maternal inheritance has long been recognized,
opmental delay and psychomotor regression, and it is an obvious target in which to search for
ataxia, optic atrophy, seizures, peripheral neu- mtDNA mutations. Recovery is variable and to
ropathy, and brainstem dysfunction. Serum and some extent may be linked to the underlying
CSF lactate and pyruvate are high; in most mtDNA genotype, but most individuals remain
patients the diagnosis is further supported by a visually handicapped for life. (Schapira and Cock,
characteristic MRI showing midbrain, basal 886898)
ganglia, and brainstem lucencies with or with-
out cortical changes. Postmortem spongiform 73. (D) This question reports a 2-year-old female
degeneration is seen in the brainstem, with with intractable seizures, hypotonia, and liver
marked loss of neuronal cells and vascular pro- dysfunction. These symptoms are consistent
liferation. The cerebral and cerebellar cortices with the diagnosis of Alpers disease.
are characteristically spared. (Schapira and Cock, Alpers disease, or progressive neuronal
886898) degeneration of childhood with liver disease, is
a rare familial disorder of unknown etiology.
71. (B) The association of extraocular ophthalmo- Typically, onset of symptoms follows normal
plegia with neuromuscular and gastrointestinal delivery and early development. Infants most
234 5: Neuromuscular Diseases

often present with intractable generalized con- encephalomyopathies. Large-scale rearrange-


vulsions associated with developmental delay, ments of mtDNA, in particular deletions, are
marked hypotonia, episodes of vomiting, and found in some 40% of adult patients with mito-
failure to thrive. There may be signs of liver dis- chondrial disease. Most commonly, the resultant
ease at presentation. Investigations reveal occip- clinical picture is one of CPEO, with or without
ital and posterior temporal hypodensities and the associated features that make up Kearns
atrophy on CT scan, very slow activity of very Sayre syndrome (KSS), which are retinitis,
high amplitude spikes interspersed with lower- ataxia, cardiac conduction block, or elevated
amplitude polyspikes on EEG, absent visual CSF protein. The case described in this vignette
evoked responses, and abnormal liver histology. shows a symptomatic high-degree atrioventric-
(Schapira and Cock, 886898) ular heart block, ophthalmoplegia, retinal
abnormalities, and cerebellar signs in a young
74. (E) The patient described in this question has patient. These symptoms are highly suggestive
clinical and radiological evidence of recurrent of KSS. The diagnostic criteria of KSS include
episodes of stroke with headaches and bio- onset before the age of 20 years, CPEO, and a
chemical evidence of lactic acidosis. The most pigmentary retinopathy in association with
likely diagnosis is mitochondrial encephalomy- ataxia, heart block, or raised cerebrospinal fluid
opathy with lactic acidosis and stroke-like protein. A proximal myopathy commonly devel-
episodes. The key features of the disease include ops as the disease progresses, and there may
stroke-like episodes with headache, vomiting, also be deafness, stroke-like episodes, bulbar
and focal neurological disturbance, lactic acido- symptoms, areflexia, and lactic acidosis. Muscle
sis, and biochemical or morphological evidence biopsy may show up to 60% of all fibers lacking
of mitochondrial dysfunction on muscle biopsy. COX activity. (Schapira and Cock, 886898)
Other common features include a pigmentary
retinopathy, psychomotor deterioration, con- 76. (C) The association of cerebellar signs, seizures,
vulsions, myopathy (87%), deafness, diabetes, and severe myopathy in a 30-year-old male, as
and short stature (55%). (Schapira and Cock, described in this vignette, is suggestive of
886898) myoclonic epilepsy with ragged-red fibers. The
major clinical features of the disease are
75. (D) Mitochondria serve several important func- myoclonic epilepsy, usually with tonicclonic
tions within the cell, the most important of generalized seizures, a progressive cerebellar
which is the production of ATP by the oxidative syndrome, and a myopathy. Deafness is com-
phosphorylation system (OXPHOS). The ubiq- mon in both clinically overt cases and in other-
uity of mitochondria suggests that a defect of wise asymptomatic maternal relatives. Other
OXPHOS will affect the function of numerous features may include pes cavus, peripheral
tissues and implies that mitochondrial OXPHOS neuropathy, optic atrophy, dorsal column loss,
decrease will be multisystemic. However, dif- heart block, and, in severe cases, dementia.
ferent tissues have varying dependence on CPEO, pigmentary retinopathy, and stroke-like
OXPHOS for normal function and survival: episodes are said to be typically absent. The
brain and muscle (heart and skeletal) are highly onset is commonly in the second or third decade,
dependent, and bone and fibroblasts less so. but cases have been reported from ages 3 to
Mitochondria also have their own DNA, inher- 62 years. Early childhood development is often
ited through the maternal line, since no sperm normal, but there may be a history of muscle
containing mitochondria enter the ovum, leav- fatigue, cramps, epilepsy, or developmental
ing the embryo to develop using only maternal delay before the diagnosis becomes clear. In
mtDNA. Mutations of mtDNA have now been those with clear central nervous system involve-
associated with a large variety of clinical ment, the disease is usually progressive,
presentations, most of which involve muscle although milder cases may remain minimally
and central nervous system features and are affected for many years. (Schapira and Cock,
collectively referred to as the mitochondrial 886898)
Answers: 7479 235

77. (C) KSS is a mitochondrial disease character- (as exercise duration increases) over glucose
ized by a clinical triad: progressive external oph- until, a few hours into exercise, lipid oxidation
thalmoplegia, retinal degeneration, and onset becomes the major source of energy. At high
before the age of 20 years. It is variably associ- intensities of aerobic exercise, the proportion of
ated with cerebellar ataxia, growth failure, sen- energy derived from carbohydrate oxidation
sorineuronal deafness, heart block, and raised increases and glycogen become an important
CSF protein. Diabetes mellitus, hypoparathy- fuel. Fatigue appears when the glycogen is
roidism, and growth deficiency may occur. Both exhausted. Hence, the symptoms of patients with
ragged-red fibers and COX-negativity are pres- glycogenoses are almost invariably related to a
ent in biopsied muscle. Ninety percent of strenuous bout of exertion. In contrast, patients
patients have a large-scale rearrangement of with a disorder of lipid metabolism usually have
their muscle mitochondrial DNA. (Schapira and little difficulty with short-term intense exercise.
Griggs, 184) In this vignette, the observation that venous
pyruvate and lactate did not increase after exer-
78. (A) Cardiac conduction defects are the frequent cise pointed to a failure of the breakdown of
features of KSS, whereas hypertrophic car- glycogen to lactic acid. The administration of
diomyopathy has been reported in myoclonic epinephrine elicited a normal rise of blood glu-
epilepsy, with ragged-red fibers, mitochondrial cose, indicating intact hepatic glycogenolysis
encephalomyelopathy with lactic acidosis and and abnormal muscle glycogen metabolism. The
stroke-like episodes, Leigh syndrome, and pro- most likely diagnosis is myophosphorylase defi-
gressive external ophthalmoplegia. (Schapira and ciency, or McArdle disease. In typical cases, the
Griggs, 190191) cardinal manifestation of the disease is exercise
intolerance manifested by myalgia, premature
79. (C) The patient described in this vignette has fatigue, and weakness or stiffness of exercising
acute recurrent and reversible exercise intoler- muscles. Muscle necrosis and myoglobinuria
ance (especially during a brief, intense isometric after exercise occur in about half of the patients,
exercise or less intense but sustained exercise), and about half of these develop acute renal fail-
normal neurological examination, increased CK ure. Mild proximal limb weakness is seen in
level in the serum, myoglobinuria, and normal about one third of patients and is more com-
pyruvate and lactate levels in the serum. This is mon in older individuals. The EMG may be nor-
highly suggestive of a muscle energy deficit. mal or may show nonspecific myopathic
The fuel used by muscle depends on several fac- changes. No electrical activity is recorded by
tors, most importantly, the type, intensity, and needle EMG from maximally shortened mus-
duration of exercise but also diet and physical cles during the cramps induced by ischemic
conditioning. At rest, muscle uses predomi- exercise. Examination of a muscle biopsy speci-
nantly fatty acids, whereas the energy for men under light microscopy may show subsar-
intense aerobic exercise derives from the oxida- colemmal deposits of glycogen as bulges or blebs
tion of carbohydrate. The energy for maximal at the periphery of the fibers. Accumulation of
force generation in intense isometric exercise glycogen, between myofibrils, generally is less
derives from anaerobic metabolism, particularly marked but may be sufficient to give the fibers
anaerobic glycogenolysis. During submaximal a vacuolar appearance.
dynamic exercise, the type of fuel used by mus- In phosphofructokinase deficiency, typically,
cle depends on the relative intensity and dura- there is intolerance to intense exercise, often
tion of exercise. At low intensity, the initial accompanied by cramps of exercising muscles,
oxidative fuel is glycogen, with increasing pro- which are relieved by rest. Although a careful
portions of oxidative energy supplied by blood, history reveals that exercise intolerance is pres-
glucose, and free fatty acids as exercise dura- ent since childhood, patients usually do not
tion increases. The type of circulating substrate come to medical attention until adolescence, and
during mild exercise varies with time. There is the diagnosis is established most commonly in
a gradual increase in the use of free fatty acids young men. Symptoms are more likely to occur
236 5: Neuromuscular Diseases

with isometric exercise. The exercise intolerance and bulbar or oculobulbar in about 15% of cases.
seems to worsen with a high carbohydrate Within 2 years after onset, myasthenic syndrome
intake. The resting serum CK level is usually remains restricted to the extraocular muscles in
increased. The EMG may be normal or show only about 14% of the patients whose initial
myopathic abnormalities. Studies of 31 P- manifestations are only ocular, whereas about
nuclear MR spectroscopy show the accumula- 86% of patients develop generalized manifesta-
tion, even with mild exercise, of glycolytic inter- tions. The primary pathogenic event in MG is
mediates in the form of phosphorylated identified as an antibody-triggered acceleration,
monoesters. The accumulation of phosphory- internalization, and progressive loss of acetyl-
lated monoesters with exercise also occurs in choline receptors associated with a complement-
other defects of glycolysis but not in myophos- mediated degeneration of synaptic folds. The
phorylase deficiency. loss of acetylcholine receptors results in
Another important clinical difference decreased postsynaptic sensitivity to acetyl-
between McArdle disease and phosphofructok- choline. Repetitive stimulation studies are the
inase deficiency is related to the fact that because most commonly used electrophysiologic test of
phosphofructokinase deficiency blocks the neuromuscular transmission. In MG, the major
metabolism of glucose, these patients experi- physiological defect is the decremental response
ence a substantial drop in exercise capacity in of the compound muscle action potential to a
response to glucose infusion or high-carbohydrate train of supramaximal stimuli, at a frequency
meals. This response is termed the out of varying from 2 to 3 Hz, of a nerve innervating
wind phenomenon and is related to the fact the affected muscle. The administration of mag-
that in phosphofructokinase deficiency the nesium may depress the neuromuscular con-
muscle is highly dependent on the availability duction and worsen the symptoms of MG.
of fatty acids and ketones for oxidative metab- (Schapira and Griggs, 254266)
olism. Glucose causes an insulin-mediated inhi-
bition of triglyceride hydrolysis and a fall in 81. (E) LambertEaton myasthenic syndrome is
blood levels of the fatty acids and ketones nec- due to an impairment of presynaptic release of
essary for muscle oxidative fuel. Definitive acetylcholine at the neuromuscular junction.
diagnosis requires biochemical documentation Presynaptic loss of voltage-gated calcium chan-
of the enzyme defect in muscle or careful meas- nels is the suggested primary pathological event
urement of the enzyme activity in erythrocytes in this disease. It is postulated that the calcium
to show partial deficiency. (Schapira and Griggs, channels are the targets of autoantibodies. As a
230233) consequence, the influx of calcium into the nerve
terminal is impaired, resulting in decreased
80. (C) The cardinal clinical features of myasthenia quantal release of acetylcholine. Repetitive stim-
gravis (MG) are fluctuating weakness and ulation, which increases the external calcium
abnormal fatigability affecting all voluntary concentration, promotes calcium entry into the
muscles, with a predilection for extraocular, bul- nerve terminal, thus enhancing the acetylcholine
bar, and proximal limb muscles. Initial symp- release and neuromuscular transmission. In
toms involve the external ocular muscles in about 92% of patients with LambertEaton
approximately 50% of cases, but bulbar symp- myasthenic syndrome, antibodies directed
toms are present in 16% of cases, and more against the P/Q type of voltage-gated calcium
rarely limb muscles may also be affected ini- channels have been found with or without asso-
tially. Muscle weakness tends to be worse with ciation to neoplasms. The N type of voltage-
repeated or prolonged exercise and typically gated calcium channel antibodies are detected in
exhibits diurnal fluctuation, worsening toward 40% to 49% of all early LambertEaton myas-
the evening hours. Within 1 year of onset, the thenic syndromes and in approximately 70% of
disease remains purely ocular in about 40% of those associated with malignancies.
cases, generalized in about 35% of cases, con- In its classic presentation, the syndrome is
fined to the extremities in about 10% of cases, characterized by weakness and fatigability,
Answers: 8082 237

mostly affecting the proximal limb muscles, with and dysphagia. Swallowing ultimately becomes
minimum or moderate extraocular involvement impossible, and liquids are regurgitated through
or bulbar symptoms. Onset of symptoms usu- the nose. The voice often has a nasal quality and
ally occurs in the proximal lower limb muscles, may be hoarse. Muscular weakness may appear
which remain more predominantly involved. between the second and fourth day of the illness.
Autonomic symptoms and signs are usually At first, the limbs may feel tired and the patient
prominent, and may include dry mouth and is unable to climb stairs. This weakness may
eyes, impotence, orthostatic hypotension, and become so severe that moving about or even
hyperhidrosis. The reflexes are reduced or turning in bed is impossible. Often this muscu-
absent. Repetitive stimulation studies represent lar involvement is limited to the neck muscles,
the most specific diagnostic tool. They may so that the patient is unable to raise his or her
show the neurophysiologic characteristic of this head. Restlessness and agitation may occur;
presynaptic disorder: initial reduced compound however, consciousness is preserved.
muscle action potential amplitude with postac- The neurophysiological findings of botulism
tivation facilitation after voluntary exercise of are similar to the ones observed in Lambert
10 seconds or tetanic stimulation. There is an Eaton myasthenic syndrome. Sensory conduc-
incremental response of the amplitude of the tion studies are usually normal. In motor nerve
first compound muscle action potential of at conduction studies, the compound muscle
least 100%. Tetanic stimulation at a rate of 20 to action potential amplitudes are decreased in
50 Hz is more painful than the voluntary exer- affected muscles with normal latencies and con-
cise and is indicated in cases of absent postacti- duction velocities. Needle EMG may show
vation facilitation after voluntary activation. abnormalities at rest with fibrillations and pos-
(Schapira and Griggs, 272274) itive sharp waves. Repetitive stimulation stud-
ies may show decrement at slow rates of stimu-
82. (A) The case described in this vignette is highly lation and characteristic increment, after a brief
suggestive of botulism intoxication. Botulinum exercise or tetanic stimulation, between 20 and
toxin is one of the most potent poisons known. 50 Hz. The diagnosis is confirmed by detection
It is produced by the spores of Clostridium botu- of the neurotoxin in the patient serum or feces.
linum. The basic pathophysiology of botulism Clostridium tetani produces a powerful exo-
neurotoxicity relates to its inhibitory effect on toxin under the anaerobic conditions of wounds
acetylcholine release. Individual toxin types dif- or soil-contaminated injuries. Clinical features
fer in their affinity for neuronal tissue, with type involve both the central and the peripheral nerv-
A being the most potent, followed by type B. ous systems as well as the muscular system. The
Toxin-induced paralysis of cholinergic nerves incubation period is usually from 5 to 25 days
involves three basic steps: (1) the binding of but may be as short as a few hours. In most
exotoxin to external receptors at ganglionic cases, the clinical onset is characterized by a
synapses, postganglionic parasympathetic seemingly preferential affinity of the toxin for
synapses, and neuromuscular junction; (2) the the facial and bulbar muscles. Premonitory signs
translocated step, during which the toxin mole- may consist of a chill, headache, and restless-
cule or some portion of it passes through the ness, with pain and erythema at the site of
nerve or muscle; and (3) the paralytic step, dur- injury. A sensation of tightness in the jaw and a
ing which the release of acetylcholine is usually mild stiffness and soreness in the neck are
blocked. usually noticed within a few hours. Pain
Clinical features may include prodromal between the shoulder blades may also be pres-
symptoms with nausea, vomiting, abdominal ent. Later, the jaw becomes stiff and tight and
pain, and diarrhea. Cranial nerve signs appear trismus results. This muscular involvement soon
early, with eye symptoms being the most com- spreads to the throat muscles, producing dys-
mon. There may be both an internal and exter- phagia, and when the facial muscles are
nal ophthalmoplegia. Rapid involvement of involved, facial asymmetry and a fixed smile
other cranial nerves produces vertigo, deafness, result. As the disease progresses, muscular
238 5: Neuromuscular Diseases

hypertonicity may spread and become general- disorder of the neuromuscular junction with a
ized, involving the muscles of the trunk and curare-like effect.
extremities. The rigidity of the back muscles pro- LambertEaton myasthenic syndrome is
duces an arching of the spine that, together with diagnosed based on the triad of fatigable weak-
the retraction of the head, results in opisthotonos. ness predominantly in the proximal limb mus-
Spasms or tonic contractions occur in any mus- cles, reduced or absent reflexes, and autonomic
cle group and may be spontaneous or precipi- features. Repetitive stimulation studies show an
tated by the slightest stimulus, such as noises, incremental response on titanic stimulation.
touching the patient, or even touching the bed. Antibodies against the P/Q voltage-gated cal-
Organophosphorus compounds are power- cium channel may be detected. (Schapira and
ful inhibitors of acetylcholinesterase and pseudo- Griggs, 272280)
cholinesterase. In the human, the former enzyme
is found in nervous tissue, specifically in brain 83. (E) The deleterious effects of aminoglycosides
and spinal cord myoneural junctions (at pre- and on neuromuscular transmission have been well
postganglionic parasympathetic synapses and established for practically all agents of this fam-
at preganglionic synapses) and in some post- ily of antibiotics, which are contraindicated in
ganglionic sympathetic nerve endings. Excess patients with MG. Most aminoglycosides,
acetylcholine causes overstimulation and then including neomycin and tobramycin, exert their
depolarization blockade of cholinergic trans- effect through reduction of the number of acetyl-
mission. Two major neurophysiological features choline quanta released at the nerve terminal,
of acetylcholinesterase inhibition are repetitive after the arrival of the propagated nerve action
discharges following the compound muscle potential. Other antibiotics have been incrimi-
action potential (CMAP) and the decrement in nated in myasthenic exacerbation, including
the CMAP with repetitive stimulation. ampicillin, ciprofloxacin, perfloxacin, and nor-
Clinically, intoxication may range from floxacin. Clindamycin and lincomycin may pro-
latent, asymptomatic poisoning to a life-threat- duce a neuromuscular blockage, not reversible
ening illness, depending on the level of serum with anticholinesterases but reversible with 3,4
cholinesterase activity. A decrease of 10% to 50% aminopyridine. Quinidine can produce a wors-
of serum cholinesterase activity may not even ening of weakness in patients with MG, acting at
be clinically detectable. When levels are the nerve terminal by inhibiting acetylcholine
moderately depressed (20% normal), sweating, synthesis or release. Magnesium derivate may
cramps, tingling of the extremities, and mild worsen myasthenia symptoms by blocking cal-
bulbar weakness with fasciculation may occur. cium entry into the nerve terminal. (Schapira and
At 10% of serum cholinesterase activity, con- Griggs, 277278)
sciousness becomes depressed, and myosis with
no pupillary response to light occurs. The 84. (D) Congenital myasthenic syndromes (CMS)
patient may become cyanotic from respiratory are heterogeneous disorders arising from presy-
weakness, and pooled secretions may obstruct naptic, synaptic, or postsynaptic defects. In each
the airway. Central nervous system manifesta- CMS, the specific defect compromises the safety
tions include confusion, convulsions, depres- margin of neuromuscular transmission by one
sion of respiratory and circulatory centers, night- or more mechanisms. The European Neuro-
mares, headaches, progressive generalized muscular Centers classification of congenital
weakness, slurred speech, ataxia, and tremor. myasthenic syndrome classifies congenital
Muscarinic manifestations include bradycardia myasthenia into three groups: Type I, with auto-
with hypotension, excessive sweating and sali- somal recessive transmission, includes familial
vation, miosis and blurring of vision, nausea infantile myasthenia, limb-girdle myasthenia,
and vomiting with cramps, and wheezing with acetylcholinesterase deficiency, acetylcholine
bronchial constriction. Nicotinic manifestations receptor deficiency, and benign congenital myas-
include muscular twitching, fasciculation, and thenic syndrome with facial dysmorphism. Type
cramps. Venom poisoning may rarely cause a II, with autosomal dominant transmission,
Answers: 8391 239

includes slow-channel syndrome. Type III 89. (B) Cranial nerves may be affected in certain
includes sporadic cases. The clues for the diag- diseases that cause peripheral neuropathies.
nosis of a slow-channel myasthenic syndrome GuillainBarr syndrome (GBS) may be associ-
consist of selectively severe weakness of the ated with abducens and facial nerve paralysis,
forearm extensor muscles, repetitive compound whereas MillerFisher syndrome (a variant of
muscle action potential response to single nerve GBS) may be associated with oculomotor and
stimuli that is accentuated by edrophonium, trochlear nerve paralysis. Sjgren syndrome neu-
prolonged and biexponentially decaying minia- ropathy may be associated with trigeminal neu-
ture endplate current, and endplate myopathy. ropathy. Polyarteritis nodosa may commonly
The endplate myopathy, which results from cal- involve the oculomotor and facial nerves, less
cium overloading of the postsynaptic region, is likely the vestibulocochlear nerve. Wegener
evidenced by degeneration of junctional folds granulomatosis may commonly affect the facial
with loss of acetylcholine receptors and widen- nerve. The trigeminal and facial nerves are the
ing of the synaptic space. (Engel et al., 140156; most commonly affected in Lyme disease. The
Schapira and Griggs, 279286) trigeminal and vagus nerves are the most com-
monly affected cranial nerves in porphyria. The
85. (A) Cholesterol- and lipid-lowering agents are oculomotor nerve is the most commonly affected
associated with myopathy, occurring in less than in syphilis. Primary amyloidosis commonly
0.5% of patients on monotherapy and increasing affects the facial, the trigeminal, and the oculo-
in frequency up to 5% with combined lipid- motor nerves, whereas in Refsum disease the
lowering therapy. Patients complain of myalgia olfactory nerve as well as the vestibulocochlear
and weakness. CK concentration is elevated. nerve are most commonly affected. (Rolak, 81)
Biopsy reveals type II atrophy and myofiber
necrosis. The 3-hydroxy-3-methylglutaryl coen- 90. (D) A predominantly sensory type of neuropa-
zyme A (HMG-CoA) reductase inhibitors (such thy may be seen in cases of pyridoxine or dox-
as lovastatin) and niacin have all been impli- orubicin toxicity, sensory variants of acute and
cated in producing myopathy. The HMG-CoA chronic demyelinating polyneuropathy, IgM
reductase inhibitors produce rhabdomyolysis as paraproteinemia, paraneoplastic neuropathy,
a direct toxic effect on myocytes. Amiodarone Sjgren syndrome neuropathy, vitamin E defi-
may cause a lysosomal-related myopathy. ciency, abetalipoproteinemia, and spinocerebel-
Zidovidine may cause mitochondrial myopa- lar degeneration. A predominantly motor neu-
thy. D penicillamine may cause inflammatory ropathy is commonly seen in cases of GBS,
myopathy. Colchicine may cause antimicro- diphtheric neuropathy, dapsone-induced neu-
tubular myopathy. (Schapira and Griggs, 364366) ropathy, porphyria, and multifocal motor neu-
ropathy. (Rolak, 82)
86. (E) The facial nerve is the most commonly
affected cranial nerve in sarcoidosis, but olfac- 91. (C) A useful concept in understanding neuro-
tory, oculomotor, and trochlear nerves can also muscular transmission is the safety factor. It is
be affected. (Rolak, 81) defined as the difference between the membrane
potential and the threshold potential of initiating
87. (A) Diphtheritic neuropathy most commonly an action potential. As long as the threshold
causes glossopharyngeal nerve palsy or, less potential is reached, the action potential initi-
commonly, oculomotor cranial nerve palsies. ates muscle contraction. Several factors con-
(Rolak, 81) tribute to the safety factor: quantal release, acetyl-
choline receptor conduction properties and
88. (C) Diabetic neuropathy may be associated density, and cholinesterase activity. Postsynaptic
with oculomotor nerve palsy with conserva- folds form a high-resistance pathway that
tion of the pupillary reflexes. The trochlear, focuses endplate current flow on voltage-gated
abducens, and facial nerves can be affected. sodium channels concentrated in the depth of
(Rolak, 81) the folds. Both these factors reduce the action
240 5: Neuromuscular Diseases

potential threshold at the endplate and serve to cytes from a hyperplasic thymus. (6) There is
increase the safety factor. All disorders of neu- transfer by MG thymic explants of pathogenic
romuscular transmission are characterized by parameters in some animal studies. The human
the compromise of the safety factor for neuro- thymus may express acetylcholine receptors.
muscular transmission. The functional effect of Thymic myoid cells may have a role in anti-
reduced acetylcholine receptors is decreased AChR sensitization. This is supported by their
endplate potential. If quantal release is lowered characteristic and unusual microenvironment
(such as in cases of repetitive stimulation or in MG thymuses with lymphoid follicular
activity), the endplate potential may fall below hyperplasia. In MG thymuses but never in nor-
the threshold and the muscle action potential mal thymuses, myoid cells, which do not
will not be generated. (Schapira, 254) express class II molecules, are in intimate con-
tact with HLA-DRpositive reticulum cells close
92. (A) Several lines of evidence suggest that to, and occasionally inside, germinal centers.
CD4 T helper cells have a major role in the These histopathological findings suggest that
pathogenesis of MG: (1) Most antiacetylcholine the reticulum cells may function as antigen-pre-
receptor antibodies in MG patients are high- senting cells and acetylcholine receptor epi-
affinity IgG and their synthesis requires CD4 topes. (Moulian et al., 397406)
and T-helper factors. (2) Acetylcholine recep-
tors reactive CD4 cells from the blood, and 94. (D) Corticosteroids are the mainstay of
the thymus of MG patients have a T-helper immunosuppressive treatment of MG. They
function. (3) Thymectomy decreases the reac- have numerous effects on the immune system as
tivity of blood T cells against acetylcholine a whole, leading to generalized immunosup-
receptors. (4) In vitro treatment of CD4 T pression. The beneficial effect therapy for MG
cells from the blood of MG patients with anti appears to be related to (1) reduction of lym-
CD4 antibodies decreases the reactivity of T phocyte differentiation and proliferation; (2)
cells to acetylcholine receptors. (5) In experi- redistribution of lymphocytes from the circula-
mental autoimmune MG, the synthesis of tion into tissues that remove them from the site
pathogenic antiacetylcholine receptor anti- of immunoreactivity; (3) alteration of lym-
bodies requires CD4 cells. (Drachman, 1797 phokine function, primarily tumor necrosis fac-
1810) tors IL-1 and IL-2; (4) inhibition of macrophage
function, in particular antigen presentation and
93. (C) MG is characterized clinically by muscle processing; and (5) increased acetylcholine
weakness, enhanced by physical effort. receptor synthesis in muscle.
Although the acetylcholine receptor (AChR)
expressed on muscle is the main target of the 95. (B) Dermatomyositis has been modeled as a
disease, the thymus has long been known to be disease in which autoantibodies directed against
involved in the pathogenesis of MG. Most endothelium cause vascular injury, leading to
myasthenic patients have thymic abnormalities: ischemic myofiber damage. Myofiber injury
70% of patients have lymphoid follicular hyper- consists of perifascicular atrophy, with small,
plasia, and 10% have a thymoma. Numerous abnormal-appearing myofibers at the periphery
arguments indicate a relationship between MG, of muscle fascicles. It has been proposed that
the anti-acetylcholine receptor antibodies, and the initial injury to capillaries in dermato-
the thymus: (1) Thymic abnormalities are seen myositis results from the immune systems pro-
in seropositive patients. (2) Thymectomy has a duction of autoantibodies against an endothelial
beneficial effect. (3) Anti-AChR antibody titers antigen, causing damage to the perifascicular
are decreased after thymectomy. (4) There is a area. This area was considered vulnerable to
decreased in vitro production of anti-AChR ischemia because it is a watershed region at the
antibodies from stimulated peripheral blood periphery of fascicles. However, a pathogenic
lymphocytes. (5) There is spontaneous in vitro autoantibody to a specific endothelial antigen
production of anti-AChR antibodies by thymo- has not been identified in dermatomyositis.
Answers: 9295 241

Perifascicular muscle fibers have not been Karpati G, Hilton-Jones D, Griggs RC. Disorders of Voluntary
shown to be preferentially vulnerable to Muscle, 7th ed. Cambridge, New York: Cambridge
ischemia. In fact, experimental models suggest University Press; 2001.
that perifascicular myofibers are less vulnerable Khella SL, Souayah N. Hepatitis C: a review of its neuro-
than central muscle fibers to ischemia. In addi- logic complications. Neurologist. 2002;8:101-106.
Litman RS, Rosenberg H. Malignant hyperthermia: update
tion, no evidence has been found that such fibers
on susceptibility testing. JAMA. 2005;293:2918-2924.
are indeed damaged by ischemia.
Lunn MP, Willison HJ. Diagnosis and treatment in inflam-
Recent microarray studies assessing the matory neuropathies. J Neurol Neurosurg Psychiatry.
pattern of muscle gene expression in patients 2009;80:249-258.
with dermatomyositis have shown upregula- Mendell JR, Cornblath DR, Kissel JT. Diagnosis and Manage-
tion of genes induced by type 1 interferons ment of Peripheral Nerve Disorders. Contemporary neurol-
(interferon and ) but not type 2 interferon ogy series 59. New York: Oxford University Press; 2001.
(interferon-) as compared with normal Miller TM. Differential diagnosis of myotonic disorders.
patients. The plasmacytoid dendritic cells, nat- Muscle Nerve. 2008;37(3):293-299.
ural interferon-producing cells, have been iden- Moulian N, Wakkach A, Guyon T, Poea S, Aissaoui A,
tified in dermatomyositis muscles, suggesting Levasseur P, et al. Respective role of thymus and mus-
a possible intramuscular source of type 1 inter- cle in autoimmune myasthenia gravis. Ann N Y Acad
Sci. 1998;841:397-406.
ferons.
Murakami T, Garcia CA, Reiter LT, Lupski JR. Charcot-
Based on these findings, a revised model for
Marie-Tooth disease and related inherited neuropathies.
dermatomyositis has been proposed: endothelial Medicine (Baltimore). 1996;75(5):233-250.
cells and myofibers may be injured by the Nachamkin I, Shadomy SV, Moran AP, Cox N, Fitzgerald
chronic intracellular overproduction of one or C, et al. Anti-ganglioside antibody induction by swine
more interferon 1inducible proteins such as (A/NJ/1976/H1N1) and other influenza vaccines:
myxovirus resistance A protein. (Greenberg, insights into vaccine-associated GuillainBarr syn-
20082019) drome. J Infect Dis. 2008;198:226-233.
Newman, LS, Rose, CS, Maier, LA. Medical progress: sar-
coidosis. N Engl J Med. 1997;336:1224-1234.
Nonaka I. Distal myopathies. Curr Opin Neurol. 1999;12:493-
499.
REFERENCES Preston DC, Shapiro BE. Electromyography and Neuromuscular
Disorders: ClinicalElectrophysiologic Correlations. Boston:
Amato AA, Russell JA. Neuromuscular Disorders. New York: Butterworth-Heinemann; 2000.
McGraw-Hill; 2008. Ptacek LJ, Johnson KJ. Griggs RC. Genetics and physiology
Crimlisk HL. The little imitatorporphyria: a neuropsy- of the myotonic muscle disorders. N Engl J Med.
chiatric disorder. J Neurol Neurosurg Psychiatry. 1997;62: 1993;328:482-489.
319-328. Quan D, Bird SJ. Nerve conduction studies and elec-
Depienne C, Stevanin G, Brice A, Durr A. Hereditary spas- tromyography in the evaluation of peripheral nerve
tic paraplegias: an update. Curr Opin Neurol. 2007;20:674- injuries. University of Pennsylvania Orthopaedic Journal
680. (UPOJ). 1999;12:45-51.
Dumitru D, Amato AA, Zwarts MJ. Electrodiagnostic Rolak LA, ed. Neurology Secrets. 2nd ed. Philadelphia:
Medicine, 2nd ed. Philadelphia: Hanley & Belfus; 2002. Hanley & Belfus; 1998.
Emery AE. The muscular dystrophies. BMJ. 1998;317(7164): Ropper AH, Samuels MA. Diseases of the peripheral
991-995. nerves. In: Ropper AH, Samuels MA, eds. Adams and
Engel AG, Ohno K, Milone M, Sine SM. Congenital myas- Victors Principles of Neurology. 9th ed. Chapter 46.
thenic syndromes. New insights from molecular genetic Available at: http://www.accessmedicine.com/con-
and patch-clamp studies. Ann N Y Acad Sci. 1998;841:140- tent.aspx?aID3641268.
156. Schapira AH, Cock HR. Mitochondrial myopathies and
Greenberg SA. Proposed immunologic models of the encephalomyopathies. Eur J Clin Invest. 1999;29(10): 886-
inflammatory myopathies and potential therapeutic 898.
implications. Neurology. 2007;69:2008-2019. Schapira AHV, Griggs RC. Muscle Diseases. Blue books of
Jankovic J. Botulinum toxin in clinical practice. J Neurol practical neurology 24. Boston: Butterworth-Heinemann;
Neurosurg Psychiatry. 2004;75:951-957. 1999.
242 5: Neuromuscular Diseases

Souayah N, Karim H, Kamin SS, McArdle J, Marcus S. responsive to cyclophosphamide. Neurol Neurophysiol
Severe botulism after focal injection of botulinum toxin. Neurosci. 2006;5.
Neurology. 2006;67:1855-1856. Souayah N, Tick Chong PS, Dreyer M, Cros D,
Souayah N, Nasar A, Suri MF, Qureshi AI. GuillainBarr Schmahmann JD. Myotonic dystrophy type 1 presenting
syndrome after vaccination in United States: a report with ventilatory failure. J Clin Neuromuscul Dis. 2007;9:
from the CDC/FDA Vaccine Adverse Event Reporting 252-255.
System. Vaccine. 2007;25:5253-5255. Steere AC. Medical progress: Lyme disease. N Engl J Med.
Souayah N, Sander HW, Menkes DL, Khella SL. Hepatitis 2001;345:115-125.
C virus acute quadriparetic vasculitic neuropathy
CHAPTER 6

Behavioral Neurology
Questions

1. Which of the following biomarkers is elevated 5. The association of apathy, disinhibition, and
in the cerebrospinal fluid (CSF) of patients with eating disorder is suggestive of
Alzheimer disease (AD)?
(A) progressive supranuclear palsy
(A) Secretase enzymes (B) Huntington disease
(B) A-40 peptide (C) frontal variant of frontotemporal dementia
(C) A-42 peptide (D) SD
(D) Oligoclonal bands (E) AD
(E) Phosphorylated tau
6. In SD, the patient may develop
2. Frontotemporal dementia may be associated
(A) agnosia for faces and objects
with
(B) early parkinsonism
(A) multiple sclerosis (C) early short-term memory deficit
(B) motor neuron disease (D) apathy
(C) Parkinson disease (PD) (E) disinhibition
(D) Huntington disease
(E) Wilson disease 7. A deficit in articulatory planning resulting in an
inability to command the speech musculature
3. Degeneration of the left frontoinsular cortex is to produce sounds in a proper sequence defines
prominent in
(A) motor aphasia
(A) AD (B) sensory aphasia
(B) frontal variant of frontotemporal (C) dysarthria
dementia (D) speech apraxia
(C) nonfluent aphasia (E) perseveration
(D) semantic dementia (SD)
(E) mild cognitive impairment 8. Which of the following is true of pick bodies?
(A) They are seen in most cases of frontotem-
4. Alien limb is seen in
poral lobe dementia.
(A) progressive supranuclear palsy (B) They are composed of randomly
(B) motor neuron disease arranged filaments of tau proteins.
(C) AD (C) They contain exclusively of amyloid
(D) corticobasal degeneration precursor protein.
(E) PD (D) They are rarely found in the hippocampus.
(E) Most often, they are located in the pyram-
idal cells of layer V.

243
244 6: Behavioral Neurology

9. The most important genetic mutation in fron- (D) Retrieval deficittype memory impair-
totemporal dementia is located on ment that responds to clues
(A) chromosome 17q21-22 (E) Impaired visuospatial function
(B) chromosome 4
15. Which of the following drugs for AD is an N-
(C) chromosome X methyl-D-aspartic acid (NMDA) receptor anta-
(D) chromosome 19p13 gonist?
(E) chromosome 9q21-q22
(A) Donepezil
10. HMPAO-SPECT studies in SD demonstrated (B) Rivastigmine
severe hypoperfusion of the (C) Galantamine
(D) Memantine
(A) bilateral frontal lobes
(E) Vitamin E
(B) bilateral anterior temporal lobes
(C) bilateral parietal lobes 16. Which of the following is true of mild cognitive
(D) bilateral orbitofrontal cortex impairment (MCI)?
(E) insula
(A) The prevalence of MCI is between 12%
and 15%.
11. Serotonin dysfunction has been reported in
(B) It impairs activities of daily living.
(A) HIV dementia (C) There is moderate impairment of cogni-
(B) mild cognitive impairment tive function.
(C) mild AD (D) The preservation of memory function
(D) severe AD excludes the diagnosis of MCI.
(E) SD (E) Within a year after the diagnosis of MCI is
established, 50% of patients progress to AD.
12. The major risk factor for AD is
17. Which of the following predicts that mild cog-
(A) age nitive impairment will progress to dementia?
(B) family history of AD
(A) ApoE2 carrier
(C) female gender
(B) Atrophic hippcampi on MRI
(D) low education level
(C) Elevated A level in the CSF
(E) head trauma
(D) Reduction in the level of tau in the CSF
13. Mutations causing early onset of AD have been (E) Hypometabolism of the frontal lobe on
localized in the fluorodeoxyglucose positron emission
tomography (FDG-PET) studies
(A) ApoE 2 gene
(B) presenilin-1 gene 18. Six months after being diagnosed with PD, a
(C) Parkin gene 63-year-old man noticed a progressive and
(D) tau protein gene fluctuating decline in his memory and cogni-
(E) Notch3 gene tive function as well as visual hallucinations
that interfered with his social and occupational
14. Which of the following changes in cognitive function. This patient most likely has
function is consistent with normal aging rather (A) AD
than AD? (B) PD dementia
(A) Loss of insight (C) dementia with Lewy bodies (DLB)
(B) Apathy (D) frontotemporal dementia
(C) Anomia (E) mild cognitive impairment
Questions: 928 245

19. Alpha-synuclein pathology occurs with PD, 24. It has been well established that on MRI spec-
multisystem atrophy, and troscopy, patients with AD show
(A) Huntington disease (A) an elevated level of lactate
(B) AD (B) a reduced level of choline
(C) vascular dementia (C) a reduced level of N-acetyl aspartate
(D) HIV dementia (D) a reduced level of phosphocreatine
(E) DLB (E) a reduced level of myoinositol

20. Orthostatic hypotension is a prominent feature 25. Donepezil demonstrates more benefit in the
of progression from mild cognitive impairment to
definite dementia in
(A) corticobasal degeneration
(B) DLB (A) APOE4-negative carriers
(C) motor neuron disease (B) APOE4-positive carriers
(D) AD (C) APOE2-negative carriers
(E) mild cognitive impairment (D) APOE2-positive carriers
(E) Carriers of the Notch3 gene mutation
21. Which of the following clinical manifestation
favors the diagnosis of vascular dementia ver- 26. The most common psychiatric manifestation in
sus other forms of dementia? mild cognitive impairment is
(A) Visual hallucination (A) delusions
(B) Memory loss sufficient to interfere with (B) visual hallucinations
activities of daily living (C) depression
(C) Stepwise deterioration (D) euphoria
(D) Severe autonomic dysfunction (E) disinhibition
(E) Parkinsonism
27. The most pervasive neuropsychiatric symptom
22. The association of gait apraxia, early urinary of AD is
incontinence, and progressive cognitive decline
(A) depression
to confluent deep white matter changes on mag-
netic resonance imaging (MRI) is suggestive of (B) apathy
(C) anxiety
(A) normal-pressure hydrocephalus
(D) delusions
(B) HIV dementia
(E) hallucinations
(C) Notch3 gene mutation
(D) Binswanger syndrome 28. The most common neuropsychiatric symptom
(E) Parkinson dementia of DLB is
(A) auditory hallucinations
23. Which of the following is a factor favoring lack
of cognitive improvement after shunting of (B) visual hallucinations
normal-pressure hydrocephalus? (C) delusions
(D) anxiety
(A) Anomia
(E) agitation
(B) Mild impairment in cognition
(C) Secondary normal-pressure hydrocephalus
(D) Gait disturbance preceding cognitive
impairment
(E) Long duration of gait abnormality
246 6: Behavioral Neurology

29. Impulsivity and tactless conduct are seen at an 34. Injury to which of the following locations is most
early stage of critical in causing memory impairment in AD?
(A) mild cognitive impairment (A) Hippocampus
(B) AD (B) Amygdala
(C) vascular dementia (C) Basal forebrain cholinergic system
(D) DLB (D) Brainstem monoaminergic system
(E) frontotemporal dementia (E) Neocortex

30. The gene defect of familial frontotemporal 35. In AD, the enzyme responsible for the endopro-
dementia with amyotrophic lateral sclerosis is teolytic cleavage of amyloid precursor protein
located on chromosome to generate N-terminal of A peptide is
(A) 17q21-22 (A) -secretase
(B) 3 (B) BACE1
(C) 9q21-22 (C) BACE2
(D) 1 (D) -secretase
(E) 14 (E) none of the above

31. Progranulin mutations have been associated 36. Which of the following is true of the -amyloid
with peptide?
(A) frontotemporal dementia (A) The soluble -amyloid peptide is the
(B) HIV dementia major constituent of the senile plaque.
(C) vascular dementia (B) -secretase cleaves the N terminal of
(D) AD amyloid precursor protein.
(E) DLB (C) P3 deposits are found in patients with
AD.
32. Filament-containing neuronal processes in the (D) AD patients have a reduced glial reaction
distal axons are as compared with normal patients.
(E) Cox1 expression is increased in AD.
(A) neurofibrillary tangles
(B) neuropil threads 37. In early-stage AD, neuropsychological tests
(C) dystrophic neurites show a defect in
(D) senile plaques (A) remote memory
(E) Lewy bodies (B) immediate memory
(C) recent memory
33. AD was associated with mutations of gene cod-
ing for presenilin 2, which is located on (D) concrete reasoning
(E) calculation
(A) chromosome 21
(B) chromosome 19 38. Positron emission tomography shows that the
(C) chromosome 14 most severe reduction in cerebral metabolism
(D) chromosome 1 in cases of AD is located in the
(E) chromosome 17 (A) temporal association cortex
(B) frontal association cortex
(C) motor cortex
(D) parietal association cortex
(E) basal ganglia
Questions: 2947 247

39. In AD, the most common psychiatric symptom 44. In progressive supranuclear palsy, high-den-
is sity neurofibrillary tangles are least likely to be
seen in the
(A) depression
(B) visual hallucination (A) striatum
(C) auditory hallucination (B) thalamus
(D) delusion (C) pallidum
(E) verbal aggressiveness (D) dentate nucleus
(E) prefrontal cortex
40. Posterior cortical atrophy, a dementia syndrome
with early prominent visual and visuospatial 45. Which of the following is true of neurofibrillary
disturbances, is most frequently seen in tangles?
(A) corticobasal degeneration (A) Neurofibrillary tangles are more specific
(B) AD to AD than neuritic plaques.
(C) CreutzfeldtJakob disease (B) They contain reactive astrocytes and
(D) subcortical gliosis microglia.
(E) Huntington disease (C) Their main location is the leptomeningeal
and superficial cortical vessels.
41. Which of the following characteristics is a fea- (D) Neurofibrillary tangles begin in the
ture of frontotemporal dementia that distin- transentorhinal cortex and progress to the
guishes it from AD? limbic cortex to reach the neocortical
areas.
(A) Hyperorality
(E) They are made up of dense granules that
(B) Impairment in executive function react with antineurofilament antibodies.
(C) Decreased verbal memory
(D) Visuospatial short-term memory 46. Which of the following is true of neurotrans-
(E) Association with chromosome 4 mitter disturbance in AD?
(A) There is a dramatic increase in the level of
42. Which of the following neurodegenerative dis- choline acetyltransferase activity in the
eases is associated with the aggregation of tau nucleus basalis of Meynert.
isoforms without exon 10?
(B) Acetylcholinesterase activity is reduced.
(A) Supranuclear palsy (C) The number of M1 muscarinic receptors
(B) Corticobasal degeneration is decreased.
(C) AD (D) M2 muscarinic and nicotinic receptors are
(D) Pick disease preserved.
(E) Familial frontotemporal dementia (E) The level of GABA activity is increased
and correlates with the severity of the
43. Which of the following is a feature of progres- disease.
sive supranuclear palsy?
47. Early onset of familial AD has been associated
(A) Aphasia with
(B) Unilateral dystonia
(A) chromosome 21
(C) Marked slowing of vertical saccades fol-
lowed by the development of vertical (B) chromosome 4
supranuclear gaze palsy (C) chromosome 6
(D) Hallucinations not related to medications (D) chromosome 12
(E) Onset of symptoms at the age of 30 years (E) chromosome 17
248 6: Behavioral Neurology

48. Early HIV dementia is associated with 53. Alpha-synuclein aggregates are found in
(A) impaired retrieval (A) Pick disease
(B) impaired calculation (B) multiple-infarct dementia
(C) impaired attention (C) Huntington disease
(D) impaired language (D) spinocerebellar atrophy
(E) impaired recognition memory (E) multiple system atrophy

49. The most characteristic neuropathological fea- 54. In brain injury, irritability and disinhibition are
ture of HIV dementia is seen in which of the following areas of the
frontal lobe?
(A) neurofibrillary tangles
(B) periventricular demyelination (A) Dorsolateral frontal lobe
(C) caudate atrophy (B) Frontal eye field
(D) multinucleated giant cells (C) Orbitofrontal area
(E) neuritic plaques (D) Medial frontal area
(E) Supplementary motor area
50. Which of the following is a feature of progres-
sive nonfluent aphasia that differentiates it 55. Which of the following is an implicit form of
from dementia of frontal type? memory?
(A) Phonological paraphasic errors are an (A) Episodic memory
early feature. (B) Semantic memory
(B) Apathy, lack of motivation, and mental (C) Memory that needs a deliberate conscious
flexibility are extremely common. effort
(C) There is a isproportion between poor (D) Priming
executive function and preservation of (E) Short-term memory
memory and language functions.
(D) Atrophy of the left polar temporal lobe is 56. Associative visual agnosia is caused by a lesion
commonly seen. in the
(E) Atrophy of the orbitomedial cortex is uni-
(A) posterior parietal cortex
versal.
(B) occipital cortex
51. Which of the following cognitive functions is (C) inferotemporal cortex
the least affected in case of DLB? (D) association area of the frontal cortex
(A) Memory (E) thalamus
(B) Visuospatial function
57. A perceptive visual agnosia is caused by a
(C) Executive function lesion in the
(D) Attention
(A) posterior parietal cortex
(E) Construction ability
(B) occipital cortex
52. The least supportive of the diagnosis of DLB is (C) inferotemporal cortex
(D) association area of the frontal cortex
(A) progressive cognitive decline
(E) thalamus
(B) spontaneous parkinsonism
(C) neuroleptic hypersensitivity
(D) auditory hallucinations
(E) multiple falls
Questions: 4864 249

58. Prosopagnosia is caused by a lesion in the 62. Which of the following is true of long-term
potentiation in the mossy fiber pathway?
(A) posterior parietal cortex
(B) occipital cortex (A) GABA is the neurotransmitter used by
(C) inferotemporal cortex synaptic terminals of mossy fibers.
(D) association area of the frontal cortex (B) The blockade of NMDA receptors affects
long-term potentiation in mossy fibers.
(E) thalamus
(C) Long-term potentiation in mossy fibers is
59. Source amnesia is caused by a lesion in the dependent on presynaptic calcium.
(D) Cooperativity is a typical feature of long-
(A) posterior parietal cortex term potentiation in mossy fibers.
(B) occipital cortex (E) Long-term potentiation in mossy fibers
(C) inferotemporal cortex requires the concomitant activation of
(D) association area of the frontal cortex pre- and postsynaptic cells.
(E) thalamus
63. Conduction aphasia may result from a lesion
60. The process that makes information more sta- located in the
ble for long-term storage is called (A) supramarginal gyrus
(A) encoding (B) internal capsule
(B) consolidation (C) corpus callosum
(C) storage (D) angular gyrus
(D) retrieval (E) orbitofrontal areas
(E) priming
64. A patient demonstrating the use of a tooth-
61. Which of the following is true of the hippocam- brush at the level of the chest has
pal pathways involved in the storage of explicit (A) sensory neglect
memory?
(B) ideomotor apraxia
(A) There are projections from the entorhinal (C) motor neglect
cortex to the pyramidal cells of CA3. (D) hemispatial neglect
(B) Mossy fibers project to pyramidal cells of (E) limb kinetic apraxia
CA3.
(C) Pyramidal cells of CA1 project to CA3.
(D) Entorhinal cortex cells project to CA1 cells.
(E) Mossy fibers project to the entorhinal
cortex cells.
Answers and Explanations

1. (E) Elevations of CSF total tau and phosphory- FTLD syndromes within each pedigree. They
lated tau (P-tau) and a reduction of CSF found that FTD-MND was the most heritable of
A[beta]-42 levels are characteristic of AD. the syndromes, although even here only 37% of
Recent studies provide evidence that a reduc- cases occurred in pedigrees with an autosomal
tion in the ratio of A[beta]-42 to P-tau may help dominant pattern of inheritance while 41% had
distinguish patients with early AD from those no family history. Most of the responsible genes
with frontotemporal dementia and identify in the pedigrees with an apparent single-gene
those with mild cognitive impairment who defect could not be identified on screening for
later develop AD. (Hansson, 165173, 228234; recognized mutations, implying that many loci
Schoonenboom, 15801584; Schott, 552558) remain to be described. (Goldman, 18171819;
Knibb, 565571; Neary, 15461554)
2. (B) Frontotemporal dementia (FTD) or fron-
totemporal lobar degeneration (FTLD) is a 3. (C) Frontotemporal lobar degeneration
group of related neurodegenerative conditions (FTLD) has been divided into three clinical syn-
that present as a disturbance of behavior or lan- dromes: frontal variant of frontotemporal
guage. A division of FTLD into three subgroups dementia (FTD), semantic dementia (SD), and
is now widely accepted, particularly since the nonfluent aphasia (NFA), based primarily on
publication in 1998 of diagnostic consensus the relative degeneration seen in the frontal
criteria. The first subgroup is the frontal or and temporal lobes and the right and left hemi-
behavioral variant FTD (fvFTD or bvFTD, spheres. Individuals with FTD show primarily
or confusingly sometimes just FTD), which right frontal involvement and present with
accounts for about half of FTLD cases. The oth- behavioral abnormalities such as disinhibition,
ers are progressive nonfluent aphasia (PNFA) apathy, emotional blunting, and lack of insight.
and SD, which often present as a fluent pro- Patients with NFA have selective left frontoin-
gressive aphasia but are due to a deficit of con- sular degeneration and present with agramma-
ceptual knowledge rather than of language. tism, hesitant, nonfluent speech output, and
The motor syndromes of corticobasal degener- speech apraxia. With SD, which is a temporally
ation (CBD), progressive supranuclear palsy predominant disorder, two syndromes emerge:
(PSP), and motor neuron disease (MND) may SD patients with predominantly left temporal
also be associated with FTLD features and degeneration have profound anomia associated
pathology; some authorities see these as part of with progressive loss of conceptual knowledge
the same spectrum. FTLD has a strong genetic of words, while predominantly right temporal
component but only a small proportion of cases cases are associated with deficits in empathy and
show simple Mendelian inheritance. Goldman knowledge about peoples emotions. (Viskontas
and colleagues have examined this issue in 87108, 528545)
detail by selecting FTLD pedigrees from their
genetic counseling service and stratifying them 4. (D) Corticobasal degeneration (CBD) is defined
according to the strength of association of the by the presence of asymmetric parkinsonism

250
Answers: 19 251

with dystonia, rigidity, limb apraxia, and an nouns remains intact, but deficits in the under-
alien limb; it is a motor disorder defined by standing of grammar are common. Many pa-
involuntary hand or arm movements that occur tients exhibit speech apraxia, which is charac-
either in addition to or instead of a planned or terized by a deficit in articulatory planning,
willed movement. Pathological examination resulting in an inability to command the speech
demonstrates neuronal inclusions with tau pres- musculature to produce sounds in a proper
ent in astrocytes and neurons. CBD brains have sequence. (Viskontas 87108)
ballooned neurons that may be found through-
out the neocortex but mostly in the superior 8. (B) The pathological feature of Pick disease is
frontal and parietal lobes, including primary severe cortical atrophy caused by severe and
motor or sensory cortex. Neuronal loss and glio- often complete loss of large pyramidal cells in
sis is also visible in affected regions, often in the cortical layer III and the small pyramidal and
basal ganglia. (Boeve, 795800; Dickson, 935946) nonpyramidal cells of layer II. The remaining
neurons also show two distinctive histological
5. (C) Frontotemporal dementia (FTD) is charac- features: swelling (called a ballooned or Pick
terized by a multitude of behavioral changes cell) and an inclusion within the perikaryon,
that often herald its onset. These include alter- most often in layer II (Pick body). Pick bodies
ations in social decorum and personal regula- are usually found in the limbic (greatest con-
tion, including disinhibition, apathy, overeat- centration is in the amygdala and hippocam-
ing, emotional blunting, personality changes pus, including the dentate gyrus), paralimbic,
toward coldness and submissiveness, repetitive and ventral temporal lobe cortex but may also
motor behaviors, and impairment in judgment be seen in anterior frontal and dorsal temporal
and insight. With these behavioral changes, lobes. Pick bodies are composed of randomly
deficits in executive control emerge and arranged filaments of the tau protein, which is
patients have problems with planning, organiz- an axonal protein involved in microtubule
ing, shifting patterns, and generating ideas. assembly. Only a minority of patients diag-
Approximately 15% of patients develop amy- nosed with Pick disease will show the classic
otrophic lateral sclerosis, and extrapyramidal Pick pattern at autopsy. Furthermore, even
deficits are common. (Viskontas, 87108) among patients with tau inclusions, the major-
ity will not show the classic Pick body, which
6. (A) SD is a temporally predominant syndrome stains positive with silver (argyrophilic) stains.
that attacks either the left or the right temporal (Mann, 605614; Viskontas, 87108)
lobe asymmetrically. In classic SD, patients
show problems with word finding, sometimes 9. (A) The first important genetic mutations in
with nouns more than verbs. Speech remains FTLD were seen in tau. This syndrome is called
fluent, but anomia worsens and patients show FTD with parkinsonism linked to chromosome
trouble not only in naming words but even in 17 (FTLD-17). FTLD-17 turned out to be associ-
recognizing them. Compulsive interest in visu- ated with mutations in the exon or intron
ally appealing objects is common, sometimes regions of the tau gene localized to 17q21-22.
leading to compulsive playing of card games Three of the mutations account for more than
such as solitaire. As the disease spreads to the half of the genetically characterized cases cur-
right side patients, begin to have problems rec- rently reported in the literature. These three
ognizing emotions in others. Eventually, proso- mutations are the P301L, associated with the
pagnosia and multimodality agnosia for objects classic FTD phenotype; exon 10 5 splice-site
develop. (Viskontas, 87108) 16, associated with a syndrome that includes
memory or language impairment and parkin-
7. (D) Nonfluent aphasia has an insidious onset. sonism; and N279K, with features of parkin-
It emerges with a decreased output of words sonism and PSP, also called pallidopontoni-
while, soon thereafter, shortened phrase lengths gral degeneration. The tau protein binds to
and deficits in articulation develop. The use of microtubules, thereby facilitating microtubule
252 6: Behavioral Neurology

assembly. Via this mechanism, the protein main- 12. (A) The major risk factor for AD is aging.
tains the stability of cytoskeletal structural ele- Although the illness has very rarely been
ments. In a healthy brain, tau is soluble and reported to occur in patients in their 20s and
expressed as six major protein isoforms gener- 30s, onset of clinical symptoms is uncommon
ated by alternative splicing of the gene on chro- until the 50s, with prevalence rapidly increasing
mosome 17q21. Abnormal tau protein may alter to age 65, when 1% to 2% of the population is
its binding affinity. (Hong, 19141917; van Slegten- affected. By age 75, the prevalence is estimated
horst, 461471; Viskontas, 87108; Wilhelmsen, 1591165) at 15%; by age 85, it has been estimated to be
present in 35% to 50% of the general population,
10. (B) On HMPAO-SPECT, patients with SD with some studies suggesting that the preva-
show severe bilateral but asymmetric hypoper- lence continuing to climb, such that the majority
fusion in the anterior temporal lobes. Bilateral of individuals in their 90s show clinical signs of
atrophy of the anterior temporal lobes is well at least mild dementia. The second major risk
illustrated on structural MRI scans, which may factor for AD is family history, with approxi-
differentiate SD from AD. In fact, detailed volu- mately 20% of patients with AD having one or
metric measurements may show that hip- more siblings or parents affected and a pattern
pocampal atrophy is more severe in SD than in of autosomal inheritance. In families with AD,
AD; it is usually asymmetric, accompanied by several genetic mutations have been identified
more severe atrophy of the amygdala as well as that seem to be causative for the disease, such as
the temporal pole and the fusiform and infero- mutations in the amyloid precursor protein
lateral temporal gyri. (EdwardsLee, 10271040; gene, mutations in the presenilin-1 (PS-1) gene
Galton, 216225; Mummery, 6173; Viskontas, 87108) on chromosome 14 and the presenilin-2 (PS-2)
gene on chromosome 1, and the APOE poly-
11. (D) The pathogenesis of AD involves a cascade morphism [varepsilon]4 gene. Other factors
of mechanisms that include -amyloid deposi- associated with differential risk for AD are gen-
tion and its toxic effect, neuroinflammation, der and education. Women are at modestly
abnormal phosphorylation of tau, free radical greater risk for AD, even with adjustment for
toxicity, disturbed calcium homeostasis, synap- their greater survival to older ages. In several
tic loss, cholinergic dysfunction, neuronal loss, studies, a higher educational level has been
and norepinephrine and serotonin dysfunction. associated with reduced risk for AD or later
Interestingly, although considerable evidence onset of dementia. Head trauma has been sug-
suggests that -amyloid and its toxic effects on gested as a risk factor for AD, but studies have
the brain may play the key role in initiating the been muddled by wide differences in reported
pathophysiological cascade of processes that series in the criteria applied to define significant
lead to AD, the progression of clinical dementia previous history of head trauma. (Farlow, 3968)
seems to correlate more closely with the num-
ber of neurofibrillary tangles and/or loss of 13. (B) Early onset of AD is caused by a mutation
synapses. As the disease progresses, a cascade in the amyloid precursor protein (APP) gene
of pathological processes involving different that codes for the -amyloid proteins. It is
mechanisms occurs, including free radical for- thought that these mutations cause abnormal
mation and inflammation. Neuronal systems as -amyloid metabolism, resulting in chronically
defined by neurotransmitters are differentially higher levels of this proteina process leading
affected. The cholinergic system is particularly to AD. Onset of clinical symptoms typically
susceptible to deterioration, and cholinergic occurs in the late 30s to 60 years of age. The
deficiency has been correlated with the clinical other mutations causing early-onset disease
progression of AD. Similarly, as AD progresses, have been localized to the presenilin-1 (PS-1)
glutaminergic, noradrenergic, and serotonergic gene on chromosome 14 and the presenilin-2 (PS-
system deficiencies develop; these have been 2) gene on chromosome 1. The presenilins have
associated with further cognitive deterioration been found to operate in a complex that acts
and/or behavioral abnormalities. (Farlow, 3968) functionally as -secretase, which is critically
Answers: 1017 253

involved in slicing APP to produce -amyloid. Cooperative Study GroupMCI study showed
(Farlow, 3968) no benefit versus placebo. (Farlow, 3968)

14. (D) Changes in cognition, behavior, and global 16. (A) Mild cognitive impairment (MCI) involves
functioning may occur with normal aging and an abnormal process that probably represents
should be differentiated from changes related the prodromal stages of a dementing condition;
to AD. In normal aging, there is a retrieval- as such it is fundamentally different from the
deficit type of memory impairment (which extremes of normal aging. It is characterized by
responds well to clues and multiple-choice memory impairment for age and education
questions), retained insight, preservation of with preserved general cognitive function and
activities of daily living, minor delays in word intact activities of daily living. The prevalence
finding, and preservation of visuospatial func- of MCI is probably in the range of 12% to 15%
tion and social engagement. among individuals 65 years of age and older;
In AD, there is an amnestic-type of memory the incidence rates are in the range of 1% per
impairment that does not respond to clues, loss year, similar to the figures for AD.
of insight, impaired activities of daily living, The original criteria developed for MCI
apathy, anomia, and impaired visuospatial were centered on memory impairment and
function and social engagement. (Farlow, 3968) designed to characterize the early stages of an
AD-like process. However, as the field has
15. (D) Current therapies for AD are primarily expanded, it has become apparent that not all
symptomatic; they are focused on treating patients with MCI evolve to AD. Therefore, the
either cognitive or behavioral symptoms. No criteria have been expanded to include many
therapy has been proven to delay biological types of intermediate cognitive impairments
progression of the disease. Cholinesterase inhi- that may be precursors to a variety of dement-
bitors were developed as a treatment after it ing disorders. Once the determination of MCI
was recognized that cholinergic deficiency has been made, the patient may be classified in
worsens in parallel with deterioration of mem- the amnestic MCI subtype if memory impair-
ory and other cognitive functioning. Donepezil ment is part of the clinical picture or in the non-
is an inhibitor that is more selective for acetyl- amnestic MCI group if the patient does not have
cholinesterase; rivastigmine inhibits both significant memory dysfunction but has deficits
acetylcholinesterase and butyrylcholinesterase; in other cognitive domains such as language,
and galantamine, in addition to inhibiting executive function, or visuospatial skills.
cholinesterase, apparently modulates stimula- If patients meet the criteria for amnestic
tion at nicotinic receptors. Memantine belongs MCI, the progression rate is likely to be in the
to a second class of drugs that work by antago- range of 10% to 15% per year. However, in com-
nizing glutamate at the NMDA receptor, poten- munity studies, where a more heterogeneous
tially improving signal transmission and pre- patient population exists, the rates may be
venting excess calcium from rushing into lower, perhaps in the range of 8% to 10% per
neurons with glutamate stimulation, thus pro- year. (Petersen, 1538)
viding neuroprotection. In patients with moder-
ate to severe disease, memantine mildly im- 17. (B) An individual with mild cognitive impair-
proves cognitive deficits and also activities of ment is more likely to progress to dementia or
daily living and behavior. In a large double- AD if he
blind, placebo-controlled trial, vitamin E was is an APOE4 carrier.
originally found to delay functional deteriora- has atrophic hippocampi on MRI.
tion, nursing home placement, and death in has an elevated tau level in the CSF.
patients with moderate to severe AD by approx- has a reduced A level in the CSF.
imately 25%. However, no cognitive benefits has hypometabolism of the temporoparietal
were seen in the group taking vitamin E. Also, lobe on FDG-PET studies.
the vitamin E group in the Alzheimers Disease
254 6: Behavioral Neurology

has a positive amyloid imaging on PET scan. tured soluble protein, -synuclein can aggregate
has clinical manifestation of severe MCI. to form insoluble fibrils in pathological condi-
(Petersen, 1538)
tions characterized by Lewy bodies, such as PD,
DLB, and multiple system atrophy. Alpha-synu-
18. (C) The patient described in this vignette clein is the primary structural component of
developed a fluctuating decline in cognitive Lewy body fibrils. (Jellinger, 12191235; Saito,
742749)
function with visual hallucinations shortly
after being diagnosed with PD. These findings
are suggestive of DLB. Cognitive symptoms in 20. (B) Autonomic impairment is well documented
DLB usually have a gradual onset and progres- in DLB and also in other synucleinopathies such
sion. A prominent feature is fluctuation of cog- as PD and multiple system atrophy. The most
nitive function, which may be difficult to iden- serious symptoms include orthostatic hypoten-
tify. sion and syncope, but excess salivation, altered
Cognitive symptoms in DLB include forget- sweating, and seborrhea may also occur. Pathol-
fulness; impaired judgment, organization, and ogy in several pathwayssuch as carotid sinus
planning; getting lost; and trouble with spatial sensitivity, cardiac autonomic denervation, and
perception. Parkinsonism varies in DLB and central autonomic pathway dysfunctionmay
may be more subtle than that found in idio- predispose patients with DLB to syncope.
(Galasko, 6986)
pathic PD. Many studies have noted a prepon-
derance of axial signs, including gait difficulty
and postural instability, with less common rest 21. (C) Memory loss sufficient to interfere with
tremor, which resembles the postural instability activities of daily living may have a stepwise
gait disorder subtype of PD that is more likely course of deterioration associated with a patchy
to point to the development of cognitive impair- distribution of deficits, preservation of con-
ment. Patients with DLB are predisposed to sciousness, and focal neurological signs and
falling for a number of reasons, including symptoms suggestive of vascular dementia.
(Chui, 109143)
parkinsonism, impaired postural (righting)
reflexes, and autonomic impairment as well as
general problems such as those associated with 22. (D) Binswanger syndrome comprises the com-
aging, deconditioning due to lack of activity, bination of severe, confluent deep white matter
and dementia. Recurrent complex visual hallu- changes together with a slowly progressive
cinations are among the most helpful features in decline in cognition and gait. The white matter
DLB. Patients typically report seeing people, changes are postulated to result from chronic
animals, or insects and can sometimes describe hypoperfusion, incomplete infarction, and de-
them in great detail. DLB patients with visual myelination in periventricular and deep white
hallucinations have more severe visuospatial border zones. Clinically, Binswanger syndrome
dysfunction than those without. Other clinical is characterized by a slowly progressive decline
manifestation of DLB include delusions, in cognition, gait apraxia, and early urinary
depression, apathy, rapid eye movement (REM) incontinence. Although this symptom triad
behavior disorder, autonomic dysfunction, agi- may be confused with normal-pressure hydro-
tation, and anxiety. (Galasko, 6986) cephalus, Binswanger syndrome can usually be
distinguished by the accompanying diffuse
19. (E) Alpha-synuclein is a protein of unknown cerebral atrophy and confluent deep white mat-
function primarily found in neural tissue, where ter changes. (Chui, 109143)
it is seen mainly in presynaptic terminals.
Expressed mainly in the neocortex, hippocam- 23. (A) Factors favoring clinical improvement after
pus, substantia nigra, thalamus, and cerebellum, the shunting of normal-pressure hydrocephalus
it is predominantly a neuronal protein but can include secondary normal-pressure hydro-
also be found in glial cells. Normally, an unstruc- cephalus, gait disturbance preceding cognitive
impairment, mild impairment in cognition,
Answers: 1827 255

short duration of cognitive impairment, and clin- APOE4 carrier subgroup. (Petersen, 1538; Petersen
ical improvement following lumbar puncture. et al., 23792388)
Factors associated with lack of clinical improve-
ment after shunting include diffuse cerebral atro- 26. (C) Neuropsychiatric symptoms are common in
phy and extensive white matter disease on MRI, mild cognitive impairment (MCI). The symp-
moderate or severe cognitive impairment, his- toms with highest prevalence were found to be
tory of ethanol abuse, presence of aphasia, and depression, apathy, irritability, anxiety, and
history of cognitive impairment preceding gait agitation. Psychotic symptoms (delusions and
disturbance. Long duration of gait abnormalities hallucinations), euphoria, and disinhibition were
is of unproved significance. (Chui, 109143) relatively rare. The presence and severity of neu-
ropsychiatric symptoms in MCI correlate with
24. (C) Proton magnetic resonance spectroscopy the degree of cognitive and functional impair-
(1H-MRS) is a noninvasive technique that allows ment. (Apostolova, 165179)
the assessment of specific brain metabolites. The
protein-containing metabolites most commonly 27. (B) Apathy is the most pervasive neuropsychi-
measured include N-acetylaspartate (NAA), atric symptom in AD, affecting 42% of patients
which provides a marker of neuronal density; with mild, 80% of those with moderate, and
myoinositol (MI), which provides a marker of 92% of those with advanced AD. It is thought to
glial cell activity; and choline (Cho), which is reflect frontosubcortical dysfunction and dis-
thought to reflect the level of membrane turn- connection of the anterior cingulate cortex from
over. Although absolute levels of metabolite sig- other cortical and subcortical areas. It presents
nal can be quantified, results vary greatly across with loss of interest in previously enjoyed activ-
subjects and study centers; therefore a reference ities, including hobbies, social outings, or
signal is usually measured. This can be either the spending time with family; aloofness, dimin-
signal of water or, more commonly, the level of ished spontaneity and emotional behavior; and
the metabolite creatine plus phosphocreatine reduced motivation. Anxiety, another early fea-
(Cr), which is thought to be unaffected in ture of AD, is frequently felt as apprehension
patients with dementia. It has been well estab- and inner feeling of nervousness with or with-
lished that patients with AD show a decrease in out associated autonomic manifestations such
the level of NAA in a number of brain regions, as tachycardia, perspiration, xerostomia, and
including the posterior cingulate and temporal, angina-like chest tightness. The prevalence of
parietal, and frontal lobes, compared with nor- anxiety among cognitively normal elderly per-
mal controls. In contrast, the levels of MI/Cr sons is around 6%. Agitation and irritability fre-
increase in patients with AD. (Whitwell, 180203) quently co-occur. Depression is very common in
AD, occurring in 10% of mild, 40% to 60% of
25. (B) In the Alzheimers Disease Cooperative moderate, and 60% or more of patients with
Study, patients with mild cognitive impairment severe AD. The symptoms are rarely severe
(MCI) were randomized to one of three treat- enough to merit diagnosis of major depressive
ment groups: donepezil (10 mg per day), vitamin disorder; rather, they represent the less severe
E (2000 IU per day), or placebo. The study had dysphoria or minor depression.
projected that the progression rate for the sub- Psychosis in AD presents with hallucina-
jects with amnestic mild cognitive impairment tions, delusions, or delusional misidentifica-
(aMCI) would be 10% to 15% per year and the tions. Psychotic symptoms may be medication-
study was powered to reduce the rate of progres- or delirium-induced or, in the case of visual
sion by 33%. The results indicated that neither of hallucinations, triggered by poor visual acuity.
the two active treatment arms was able to reduce Psychotic symptoms occur more frequently in
the risk of progressing to AD over the entire 36 the moderate and advanced stages of AD. As
months. However, donepezil reduced the risk of many as 10% to 20% of patients with AD experi-
progression to AD for the first 12 months of the ence hallucinations, which are most frequently
study in all subjects and up to 24 months in the visual. (Mega, 130135; Apostolova, 165179)
256 6: Behavioral Neurology

28. (B) DLB is the second most common neurode- Elastase digests progranulin within intergranulin
generative dementia of older adults. It is a dis- linkers to form eight individual granulin pep-
order in which a detailed neuropsychiatric tidesa process regulated by the secretory leuko-
evaluation is of utmost importance, as visual cyte protease inhibitor, which complexes pro-
hallucinations are a core diagnostic criterion granulin, preventing elastase-mediated digestion
and delusions and prominent early depression into fragments. Progranulin is involved in a range
are supportive features. Up to 98% of patients of cellular processes, including epithelial cell
with DLB experience some neuropsychiatric growth regulation, tumor growth and invasion in
symptoms in the course of their illness. These vivo, host defense and wound repair, develop-
symptoms, along with cognitive fluctuations ment, inflammation, and signal transduction
and extrapyramidal symptoms, differentiate involving the extracellular regulated kinase sig-
DLB from other common types of dementia, naling pathways. In human brain, progranulin
such as AD and vascular dementia (VaD). mRNA is expressed at low levels, although in situ
Visual hallucinations in DLB are brightly col- hybridization has demonstrated expression in
ored three-dimensional representations of peo- specific neuronal cells, including Purkinje cells,
ple and animals. Other common themes are pyramidal cells of the hippocampus, and some
insects, fire, children, objects, and birds. These cerebral cortical neurons. Baker and colleagues
images are frequently animated and may also analyzed more than 80 candidate genes in 43 kin-
speak or make noise (i.e., they co-occur with dreds with frontotemporal lobar degeneration
auditory hallucinations). Visual hallucinations with ubiquitin-positive inclusions (FTLD-U)
are sometimes more pronounced in the within a 6.19-Mb region on 17q21 and identified
evening, when the lack of strong sensory stim- seven mutations in the gene encoding progran-
ulation and solitude promote their appearance. ulin. Subsequently, several mutations in the pro-
(Apostolova, 165179) granulin gene have been identified and associ-
ated with FTDL-U. (Baker, 442:916919; Bhandari,
29. (E) Frontotemporal dementia is an insidious, 17151719; Cruts, 920924; Gass, 29883001; Le Ber,
relentless disorder manifesting with early 846855)
prominent behavioral disturbances and per-
sonality changes. Impulsivity, tactless conduct, 32. (C) In AD, many neurons exhibit fibrillary
antisocial trends, disinhibition, lack of concern accumulations in the cytoplasm, including
with social norms, loss of interpersonal bound- neurofibrillary tangles (NFTsneurofibrillary
aries, apathy, self-centeredness, and lack of pathology in cell bodies and proximal den-
empathy are hallmark features of this disorder. drites), neuropil threads (filamentous accumula-
Obsessivecompulsive and stereotyped behav- tions in dendrites), dystrophic neurites (filament-
iors also are common. (Apostolova, 165179) containing neuronal processes, particularly
distal axons/terminals), and senile plaques of
30. (C) Several studies of families with amyotro- which the -amyloid peptide is the major con-
phic lateral sclerosis associated with fron- stituent. Ultrastructurally, fibrillary inclusions
totemporal dementia (in one study, SOD1 represent intracellular accumulations of straight
mutations had been excluded) identified a sub- filaments and paired helical filaments; both are
set of cases linked to chromosome 9q21-22. composed principally of hyperphosphorylated
(Sikkink, 693698) isoforms of tau, a low-molecular-weight, micro-
tubule-associated protein. Because hyperphos-
31. (A) Progranulin is a 68.5-kDa (90-kDa when it is phorylated tau species bind poorly to micro-
heavily glycosylated on sodium dodecyl sulfate tubules and alter microtubule stability, this
Western blot), 589amino acid pluripotent biochemical modification could affect other cyto-
secreted glycoprotein composed of 12 exons cov- skeletal constituents, intracellular transport, cel-
ering 3.7 kb. It is made up of seven tandem lular geometry, and/or neuronal viability. (Price,
repeats of the 12-cysteine granulin domain. 461493)
Answers: 2838 257

33. (D) The inheritance of one of the following may originate from cells of the central nervous
genes may predispose the carrier to an increased system as well as from the peripheral nervous
risk of AD: system. The amyloid precursor protein is
cleaved in its N and C terminials by and sec-
Polymorphic variant of the apoE gene on
retases, respectively. P3 fragment, a major com-
chromosome 19
ponent of diffuse plaques, originates from the
Presenilin 1 gene located on the long arm of
cleavage of the amyloid precursor protein by an
chromosome 14
secretase. P3 deposits are seen in normal aging
Presenilin 2 gene isolated and mapped to
as well as in AD patients. P3 deposits in normal
chromosome 1
aging patients lack abnormal neurites and dis-
Amyloid precursor protein gene linked to
play a reduced glial reaction compared with P3
chromosome 21.
deposits in AD. Cyclooxygenase enzymes 1 and
(Price, 461493) 2, biosynthesis inflammatory mediators, are
constitutively expressed and mitogen-induced,
34. (A) The disease process in AD selectively dam- respectively. Cyclooxygenase-2 level is sensitive
ages brain regions and neural circuits critical to IL-1, IL-2, and TNF proinflammatory inter-
for cognition and memory, including neurons in leukins. The expression of cyclooxygenase 2, but
the neocortex, hippocampus, amygdala, basal not cyclooxygenase 1, increases in cases of AD,
forebrain cholinergic system, and brainstem especially in neurons that are destined for apop-
monoaminergic nuclei. The severity of memory tosis. (Halliday, 18)
impairments and density of senile plaques cor-
relates with levels of synaptophysin (a presy- 37. (C) Deficits in recent memory are typically the
naptic vesicle protein) in the hippocampus of first symptom of AD and may be clinically
individuals with AD. Alterations in the basal reported when the patient misplaces objects,
forebrain cholinergic system are also believed to repeats questions and statements, and forgets
contribute to memory difficulties and to deficits names. Both verbal and visuospatial memory
in arousal/attention. (Price, 461493) changes occur. Impairments in visuospatial
memory are often experienced as getting lost.
35. (B) AD is characterized by a progressive depo- Early in the disease, these deficits in memory
sition of neurofibrillary and A-amyloid tangles are primarily for recent information. On exami-
in many areas of the brain, particularly the hip- nation, AD patients have marked difficulty
pocampus and cerebral cortex. Endoproteolytic remembering word lists, stories, and designs.
cleavage of amyloid precursor protein by and As the disease progresses, the cognitive impair-
secretase generates toxic A peptides. BACE1 ment becomes more diffuse, with reduced abil-
and BACE2 are two secretases involved in the ity to do multiple tasks or carry out complex
generation of amyloid precursor protein. The mental tracking, decreased concentration, diffi-
secretion of A peptide is abolished in cultures culty with mental arithmetic, and a decline in
of BACE1-deficient embryonic cortical neurons. abstract reasoning. Remote memory impair-
BACE1 is the principal neuronal protease ments also emerge, with the oldest memories
required to cleave amyloid precursor protein at tending to be the most stable. Semantic memory
1 and 11 sites that generate the N-terminal of as well as implicit memory become impaired as
A peptides. (Cai, 233234) the disease progresses. (Kramer, 447454)

36. (B) The -amyloid peptide is the major con- 38. (D) Positron emission tomography (PET)
stituent of senile plaques. It is present in a soluble examines regional cerebral metabolic rates for
nontoxic form in all human brains. It undergoes oxygen and glucose and has proved to be an
conformational changes and becomes relatively effective means of studying brain functioning
insoluble in AD. Soluble -amyloid peptide is in dementia patients. PET studies have consis-
mainly formed by 40amino acid sequences and tently found that association cortex, primarily
258 6: Behavioral Neurology

in posterior regions, is most severely affected in cantly better in FTD patients, suggesting that
AD. Primary sensory and motor cortices, basal FTD patients mainly experience retrieval diffi-
ganglia, thalamus, and cerebellum are rela- culties provided that encoding is controlled. Ver-
tively spared. The parietal lobe has the largest bal and visuospatial short-term memories are
reductions in metabolism. Patients with mild to both decreased in AD, whereas only verbal
moderate AD have reductions in rate of metab- memory is decreased in FTD. Language compre-
olism that range from 23% to 39% for parietal hension profiles in FTD are mainly character-
association cortex, 15% to 30% for temporal ized by sentence comprehension difficulties
association cortex, and 15% to 21% for frontal caused by impaired processing of grammatical
association cortex. Metabolic rate shows fur- phrase structure as well as a relatively selective
ther reductions as the severity of the dementia impairment in action naming. FTD patients per-
worsens. (Kramer, 447454) form better than AD patients on construction
and calculation and have greater impairment in
39. (A) Depression is the most common psychiatric executive than in memory tasks. FTD has been
symptom associated with AD. It occurs in 15% linked to chromosome 17q21-22 in a population-
to 20% of AD patients. As the dementia ad- based study. Huntington disease has been
vances, delusions become more common. More linked to chromosome 4. (Pasquier, 417427)
severely demented AD patients may have visual
and auditory hallucinations as well as restless- 42. (D) Tau proteins are the basic components of
ness, irritability, repetitive behavior, disturbed neurofibrillary neuronal inclusions that affect
sleep patterns, and verbal aggression. (Kramer, numerous causes of dementia. They stabilize
447454) microtubules, which play an important role in
intraneuronal transport. They are formed by
40. (B) Pathological studies have shown that the six isoforms resulting from the translation of
most frequent cause of posterior cortical atro- exons 2, 3, and 10. Tau proteins may be abnor-
phy is AD. This syndrome can also be seen in mally phosphorylated and aggregate into neu-
subcortical gliosis and CreutzfeldtJakob dis- ronal inclusions. These inclusions may be bio-
ease. Careful postmortem study has shown that chemically different from one type of dementia
relative to typical AD cases, patients who ini- to another. In AD, all six tau isoforms (with and
tially present with prominent visual symptoms without exon 10) are aggregated. This is not
have higher densities of plaques and tangles in specific to AD; the same pattern of tau aggrega-
primary and visual association areas and rela- tion is seen in postencephalitic parkinsonism,
tively fewer lesions in the prefrontal cortex. NiemannPick type C disease, and Downs syn-
(Kramer, 447454) drome. In progressive supranuclear palsy and
corticobasal degeneration, there is an aggrega-
41. (A) Frontotemporal dementia (FTD) is charac- tion of tau isoforms with exon 10, mainly in the
terized by the prominence of behavioral abnor- frontal subcortical and cortical areas. They are
malities such as loss of personal awareness, mainly located in small pyramidal cells and
hyperorality, stereotyped and perseverative astrocytes of layers II and III of the cerebral
behavior, and progressive reduction of speech cortex. In familial frontotemporal dementia,
with conservation of spatial orientation. Loss of tau isoforms with exon 10 are aggregated. In
spatial orientation is more prominent in AD than Pick disease, there is aggregation of two main
in frontotemporal dementia, where inappropri- tau variants lacking exon 10: tau 55 and tau 64.
ate behavior is more prominent. Informed- They are found in the frontotemporal areas and
based questionnaire as well as other behavioral involve the small pyramidal cells of neocortical
and neuropsychological tests serve to differenti- layers II and III as well as granule cells of the
ate frontotemporal dementia from AD. Free dentate gyrus. (Pasquier, 417427)
recall does not differ between AD and fron-
totemporal dementia patients, but the benefit 43. (C) Progressive supranuclear palsy is character-
from semantic cueing and recognition is signifi- ized by early postural instability, supranuclear
Answers: 3946 259

vertical gaze palsy, parkinsonism insensitive to Clinical history compatible with progressive
levodopa therapy, pseudobulbar palsy, and sub- supranuclear palsy.
cortical dementia. Histological features include
(Litvan, 4148)
degeneration in different areas of the basal gan-
glia and brainstem. The presence of early insta-
45. (D) The pathological characteristics of AD
bility and multiple falls during the first year of
include neurofibrillary tangles, neuritic plaques,
symptom onset in a patient with parkinsonism
loss of synapses and neurons, granulovacuolar
should point to the diagnosis of progressive
degeneration, amyloid angiopathy, and non-
supranuclear palsy, although early instability
amyloid (A), plaque-like deposits (AMY)
may be seen in cases of multiple system atrophy
plaques. Neurofibrillary tangles are formed by
and corticobasal degeneration. Marked slowing
paired helical filaments that occupy the cell
of vertical saccades is followed by the develop-
body and may extend to the dendrites but not
ment of vertical supranuclear gaze palsy. This
to the axon. These filaments are arranged to
distinguishes progressive supranuclear palsy
form a tubule that contains abnormally phos-
from corticobasal degeneration and multisys-
phorylated tau protein. They are preferentially
tem atrophy. In corticobasal degeneration, the
located in large pyramidal neurons, particularly
saccades may have increased latency but nor-
those with long ipsilateral corticalcortical con-
mal speed and are equally affected in the verti-
nections. Neurofibrillary tangles are produced in
cal and horizontal planes. In multiple system
the transentorhinal cortex in the beginning of the
atrophy, the saccades have normal speed and
disease and progress to limbic cortical regions to
latency. Frontal lobe signs including apathy,
reach the neocortical areas. The pattern of pro-
impaired abstract thought, decreased verbal
gression of neurofibrillary tangles correlates with
fluency, and imitation behavior are seen early
the early memory deficit seen in AD. Neurofibril-
in the course of progressive supranuclear palsy,
lary tangles are not specific to AD; they are seen in
as well as prominent swallowing and speech
progressive supranuclear palsy, postencephalitic
difficulties. Red flags against the diagnosis of
PD, and subacute sclerosing panencephalitis.
supranuclear palsy include the presence of
Neuritic plaques are more specific to AD than
aphasia, onset earlier than age 40, duration of
neurofibrillary tangles and are formed by a cen-
the disease of more than 20 years, presence of
tral immunoreactive amyloid core surrounded by
cortical dementia or cortical sensory or visual
dystrophic neurons, which contain paired helical
deficit, hallucinations not due to medications,
filaments, normal glial processes, abnormal
and maintained response to levodopa replace-
organelles, reactive astrocytes, and microglia.
ment. (Litvan, 4148)
Amyloid angiopathy involves leptomeningeal
and superficial cortical vessels in Alzheimer
44. (B) Progressive supranuclear palsy is character-
patients. Granulovacuolar degeneration involves
ized on neuropathological examination by neu-
the pyramidal cell layer of the hippocampus
ronal loss, gliosis, and the presence of neurofib-
with the presence in the cytoplasm of the
rillary tangles and/or neuropil threads in
pyramidal cell of vacuoles. (Cummings, S2S17)
specific areas of the basal ganglia and brainstem.
The National Institute of Neurological Disorders
46. (B) The nucleus basalis of Meynert is
and Stroke (NINDS) neuropathological criteria
affected early in the course of AD. It is a
for typical progressive supranuclear palsy are:
major source of choline acetyltransferase,
High density of neurofibrillary tangles and which is responsible for the synthesis of
neuropil threads in at least three of the fol- acetylcholine. There is a marked and consis-
lowing areas: pallidum, subthalamic nucleus, tent decrease of choline acetyltransferase and
substantia nigra, or pons. acetylcholine synthesis as well as a reduction
Low to high density of neurofibrillary tan- of the activity of acetylcholinesterase, the
gles or neuropil threads in at least three of enzyme responsible of the degradation of
the following areas: striatum, oculomotor acetylcholine. M1 muscarinic receptors located
complex, medulla, or dentate nucleus. in the hippocampus and in the upper and
260 6: Behavioral Neurology

lower levels of the cerebral cortex are rela- involving both the frontal and temporal lobes.
tively preserved in AD as compared with M2 The nonfluent output relates to breakdown in
muscarinic receptors located in the brainstem the phonological and grammatical aspects of
and nucleus basalis, which are markedly language. These deficits affect production and
reduced. Nicotinic receptors, serotonin, nor- comprehension. An early feature is phonologi-
epinephrine, GABA, and somatostatin are also cal, as opposed to semantic, paraphasic errors.
reduced. (Cummings, S2S17) Buccofacial apraxia is also a common feature:
patients are unable to perform tasks such as
47. (A) Early onset of familial AD has been linked licking their lips or blowing out matches on
to mutations in chromosomes 21, 14, and 1. It is command. In spite of profound language dys-
inherited as an autosomal dominant disease. function, these patients often continue to main-
Mutations in chromosome 21 involve the amy- tain an independent lifestyle without significant
loid precursor gene affecting the processing of behavioral or social disturbance.
the amyloid precursor protein. Mutations in Dementia of the frontal type presents with
chromosome 14 involve the presenilin gene neuropsychiatric symptoms rather than neu-
and cause an increase of the production of the ropsychological deficits. Patients become dis-
amyloid- peptide. Mutations in chromosome tractible and impulsive yet lack mental flexibility.
1 involve presenilin 2 gene and also cause an Apathy and lack of motivation are extremely
increase of the production of the amyloid- common; social skills degenerate, with tactless-
peptide. Mutations on chromosomes 17, 12, ness, lack of emotional warmth, and disinhibited
and 6 are considered genetic risk factors in AD. behavior. The key feature of dementia of the
(Cummings, S2S17) frontal type on neuropsychological examination
is a disproportionately poor performance on tests
48. (A) Early symptoms of HIV dementia are sub- sensitive for frontal lobe function (executive func-
tle and may be confused with psychiatric com- tion) in the absence of a significant memory, lan-
plaints, the effect of substance abuse, or delir- guage, or visuospatial disorder. The orbitomedial
ium. They are characterized by the prominence frontal lobes are affected earlier than dorsolateral
of subcortical involvement. These symptoms lobes in dementia of the frontal type. Since classic
may include: frontal lobe tasks reflect dorsolateral function
rather than orbitomedial pathology, patients with
Memory impairment, both verbal and non-
gross behavioral changes may perform normally
verbal.
on these frontal lobe tests for a number of years.
Impaired manipulation of acquired knowl-
Neuroimaging studies may show left peri-
edge.
sylvian atrophy in progressive nonfluent apha-
Impaired retrieval, and general slowing of
sia and orbitomedial frontal lobe atrophy in
psychomotor speed and thought processes.
dementia of the frontal type. (Nestor, 439446)
Attention, language, and recognition memory
are relatively preserved. (McArthur, 129150) 51. (A) DLB is characterized by a progressive cog-
nitive decline sufficient to interfere with social
49. (D) Multinucleated giant cells are characteris- or occupational function. Fluctuation of cogni-
tically seen in HIV dementia. Their presence tive function as well as well-formed and com-
correlates with the degree of dementia and the plex visual hallucinations are common features
detection of HIV DNA. (McArthur, 129150) of the disease. Spontaneous parkinsonism is
the final core feature of DLB that leads to the
50. (A) Progressive nonfluent aphasia, in contrast diagnosis being considered. In early stages of
to the fluent language disorder of SD, is charac- the disease, there is a distinctive profile that
terized by a nonfluent, Broca-like aphasia, ulti- distinguishes DLB from other types of demen-
mately leading to a state of mutism. Pathologi- tia, as the trend over time is for the develop-
cal studies show diffuse left perisylvian atrophy ment of global impairment. The typical early
Answers: 4755 261

profile is one of disproportionate involvement synuclein inclusions and the tau-positive neu-
of attention, executive, and visuospatial domains. ronal inclusion bodies in Pick disease do not
Visual deficits affect perceptual, spatial, and con- have synuclein associated with them. (Schulz,
structive abilities; when matched for degree of 433439)
dementia, memory function is superior to that
seen in AD. (Nestor, 439446) 54. (C) Frontal lobe function is characterized by the
presence of five parallel but independent cir-
52. (D) The consensus criteria for antemortem cuits defined by their distinct major reciprocal
diagnosis of DLB are divided into mandatory subcortical connections. Each circuit involves a
criteria, core features, supportive criteria, and frontal lobe area, specific projections to striatal
criteria against the diagnosis. The only manda- regions, continuation to globus pallidus, return
tory criterion for the diagnosis of DLB is pro- to the thalamus, and then back to the frontal
gressive cognitive decline that interferes with region of origin. There are two motor circuits,
social and occupational function. The core fea- one involving the supplementary motor area
tures for the diagnosis of DLB include fluctuat- and the second the frontal eye fields. Three cir-
ing state with significant variations in attention cuits determine cognitive and affective behav-
and alertness, spontaneous motor features of iors initiating in three separate regions of the
parkinsonism, and recurrence of hallucina- prefrontal cortex: dorsolateral, lateral orbital,
tions, particularly of the visual type. Support- and medial frontal/anterior cingulate. Distinct
ive criteria for the diagnosis include repeated cognitive and behavioral profiles are associated
falls, syncope, transient loss of consciousness, with lesions in the last three separate circuits.
neuroleptic hypersensitivity, systematized delu- Dorsolateral prefrontal lesions produce deficits
sions, and nonvisual hallucinations. Criteria in verbal and nonverbal fluency, decreased
against the diagnosis of DLB include the pres- problem solving and set shifting, and reduced
ence of evidence of other physical or neurolog- learning and retrieval. Orbitofrontal lesions
ical illness sufficient to explain the clinical fea- cause disinhibition and irritability. Medial
tures. (Nestor, 439446) frontal/anterior cingulate lesions result in apa-
thy and decreased initiative. Damage at any
53. (E) Alpha-synuclein is a 140 amino acid pro- point in each circuit will produce similar
tein of unknown function that is abundantly deficits. Lesions in the subcortical segments of
expressed in the brain, where it is located in these anatomical systems often cause mixed
presynaptic nerve terminals with little staining syndromes because of the proximity of the sub-
of nerve cell bodies and dendrites. cortical structures involved in the different cir-
An -synuclein 35-residue has been found cuits. The rationale for splitting the frontal lobes
in the nonamyloid component of AD plaques. into these separate operating systems is sup-
This 35-residue segment is referred to as a ported by parallel anatomical observations. The
nonamyloid component (NAC) and -synu- cortical portions of these systems have different
clein as a NAC precursor protein. NAC was connections with posterior cortical areas.
the second component, after the Alzheimer- (Alexander, 427437)
protein, to be found in extracellular AD
plaques. Alpha-synuclein aggregates are found 55. (D) Long-term memory is divided into two
not only in the Lewy bodies of PD but also in types: implicit or nondeclarative memory and
the cortical Lewy bodies of LBD and in glial explicit or declarative memory. Implicit mem-
cytoplasmic inclusions throughout the brain ory is unconscious memory of how to do some-
in multiple system atrophy. Although -synu- thing, as in training reflexive motor or percep-
clein deposits occur in several neurodegener- tual skills, whereas explicit memory involves
ative diseases, this is not a ubiquitous phe- factual knowledge of people, places, things,
nomenon after neuronal damage; brains of and what these facts mean. Explicit memory
patients with multi-infarct dementia have no involves a deliberate, conscious effort to associate
262 6: Behavioral Neurology

multiple pieces of information in a highly 58. (C) Prosopagnosia is defined by the inability to
flexible way. In contrast, implicit memory is recognize familiar faces or learn new faces and
more rigid, tightly connected to the original is caused by a lesion in the inferotemporal cor-
stimulus conditions under which learning tex. (Kandell, 12361237)
occurred. Explicit memory is further divided
into episodic memory, a memory of events and 59. (D) Source amnesia is caused by damage to
personal experience, and semantic memory, a the association areas of the frontal lobes. These
memory for facts. areas of the cortex are responsible for the long-
Priming is a form of nondeclarative memory term storage of episodic knowledge. A patient
in which the recall of words or objects is with source amnesia has a tendency to forget
improved by prior exposure to words or objects. how the information was acquired. (Kandell,
Memory of procedural skills and habits as well 12361237)
as habituation and sensitization are part of the
implicit memory and are linked, respectively, to 60. (B) Explicit memory is processed by at least
the striatum and reflex pathways. Classical and four distinct types of processing: encoding,
operant conditioning are parts of associative consolidation, storage, and retrieval. Encoding
learning, which is a part of implicit memory. The refers to the processes by which newly learned
emotional response of the classical and operant information is processed when first encoun-
conditioning involves the amygdala, whereas tered. The quality of the encoding is critical for
skeletal musculature response involves the cere- the integration and storage of newly acquired
bellum. The amygdala is involved in affective information. Consolidation refers to the pro-
aspects of memory which is related to implicit as cesses that alter the newly stored and still labile
much as explicit memory. Explicit memory is information to make it more stable for long-
acquired through a processing in one or more of term storage. It involves the expression of
the three polymodal association cortices (the genes and the synthesis of proteins that give
prefrontal, limbic, and parietooccipitotemporal structural changes necessary for stable storage
cortices) that synthesizes visual, auditory, and of the information. Storage refers to mecha-
somatic information. The association cortices nisms and sites by which memory is retained
then convey the information in series to the over time. Retrieval refers to processes that per-
parahippocampal and perirhinal cortices, the mit the recall and use of stored information.
entorhinal cortex, the dentate gyrus, the hip- (Kandell, 1237)
pocampus, the subiculum, and finally back to
the entorhinal cortex. The information is then 61. (B) Three major hippocampal pathways are
sent back from the entorhinal cortex to the involved in the processing of explicit memory:
parahippocampal and perirhinal cortex and the perforant pathway, which projects from the
finally back to the polymodal association areas entorhinal cortex to the granule cells of the den-
of the neocortex. (Kandell, 11281132) tate gyrus; the mossy fiber pathway, which con-
tains the axons of the granule cells and runs to
56. (A) Associative visual agnosia results from the pyramidal cells in the CA3 region of the hip-
damage to the posterior parietal cortex. The pocampus; and the Schaffer collateral pathway,
patient cannot name objects but can identify which consists of the excitatory collaterals of the
them by selecting the correct drawing and can pyramidal cells in the CA3 region and ends on
faithfully reproduce detailed drawings of the the pyramidal cells of the CA1 region. (Kandell,
object. (Kandell, 12361237) 1259)

57. (B) A perceptive visual agnosia is caused by a 62. (C) The mossy fiber pathway consists of the
lesion in the occipital lobe and surrounding axons of the granule cells of the dentate gyrus.
region. The patient is unable to draw objects The mossy fiber terminals release glutamate
but can identify them if appropriate perceptual as a neurotransmitter, which binds to both
cues are made available. (Kandell, 12361237) NMDA and non-NMDA receptors. NMDA
Answers: 5664 263

receptors have a minor role in synaptic plas-


REFERENCES
ticity. The blockage of NMDA receptors as
well as the postsynaptic influx of calcium has
Alexander MP, Stuss DT. Disorders of frontal lobe func-
no effect on long-term potentiation in mossy
tioning. Semin Neurol. 2000;20:427-437.
fiber pathways. However, the presynaptic cal- Apostolova LG, Cummings JL. Psychiatric manifestations
cium influx has been found to play a major in dementia. Continuum: Lifelong Learning in Neurology.
role in mossy fiber long-term potentiation. 2007;13(2):(Dementia)165-179.
Cooperativity (the process of activating sev- Baker M, Mackenzie IR, Pickering-Brown SM, et al. Muta-
eral afferent axons together) as well as associa- tions in progranulin cause tau-negative frontotemporal
tivity (the concomitant activation of pre- and dementia linked to chromosome 17. Nature. 2006;442:
postsynaptic cells to adequately depolarize 916-919.
the postsynaptic cell) are distinctive features Bhandari V, Palfree RG, Bateman A. Isolation and
of long-term potentiation in the Schaffer col- sequence of the granulin precursor cDNA from human
lateral pathway. (Kandell, 1260) bone marrow reveals tandem cysteine-rich granulin
domains. Proc Natl Acad Sci USA. 1992;89:1715-1719.
Boeve BF, Maraganore DM, Parisi JE, et al. Pathologic het-
63. (A) Conduction aphasia is characterized by flu-
erogeneity in clinically diagnosed corticobasal degen-
ent speech with paraphasic errors, with con- eration. Neurology. 1999;53:795-800.
served comprehension and impaired repetition. Cai H, Wang Y, McCarthy D, Wen H, Borchelt DR, Price
It can be caused by lesions in a variety of loca- DL, et al. BACE1 is the major -secretase for generation
tions including the supramarginal gyrus as well of A peptides by neurons. Nat Neurosci. 2001;4:233-234.
as by interruption of fiber tracts lying deep to Chui HC, Brown NN. Vascular cognitive impairment.
the sensory cortex in the parietal lobe. (Saffran, Continuum: Lifelong Learning in Neurology. 2007;13(2):
409418) (Dementia)109-143.
Cruts M, Gijselinck I, van der Zee J, et al. Null mutations
64. (B) Patients with ideomotor apraxia make sev- in progranulin cause ubiquitin-positive frontotemporal
eral types of errors when performing skilled, dementia linked to chromosome 17q21. Nature.
2006;442: 920-924.
purposive limb movements. The most com-
Cummings JL, Vinters HV, Cole GM, Khachaturian ZS.
mon errors in ideomotor apraxia are spatial
Alzheimers disease: etiologies, pathophysiology, cog-
errors. One type of spatial error involves the nitive reserve, and treatment opportunities. Neurology.
failure to position the hand in an appropriate 1998;51(S1):S2-S17; discussion S65-S67.
posture (e.g., closed-fist posture for drinking Dickson DW, Bergeron C, Chin SS, et al. Office of Rare
from a cup). A second type of spatial error Diseases neuropathologic criteria for corticobasal
involves the failure to orient the movement degeneration. J Neuropathol Exp Neurol. 2002;61:935-946.
toward an imagined object (e.g., demonstrat- Edwards-Lee T, Miller BL, Benson DF, et al. The temporal
ing the use of a toothbrush at the level of the variant of frontotemporal dementia. Brain. 1997;120:
chest). A third type of spatial error involves the 1027-1040.
failure to coordinate joint movement (e.g., Farlow MR. Alzheimers disease. Continuum: Lifelong
demonstrating a screwdriver by rotating at the Learning in Neurology. 2007; 13(2):(Dementia)39-68.
Galasko MD, Douglas R. Dementia with lewy bodies.
shoulder instead of at the elbow). Another
Continuum: Lifelong Learning in Neurology. 2007;13(2):
common apraxic error involves using a body
(Dementia)69-86.
part as if it were the imagined tool (e.g., Galton CJ, Patterson K, Graham K, et al. Differing pat-
extending the finger to represent the blade of terns of temporal atrophy in Alzheimers disease and
the screwdriver instead of positioning the semantic dementia. Neurology. 2001;57:216-225.
hand around the handle of the screwdriver). Gass J, Cannon A, Mackenzie IR, et al. Mutations in pro-
Apraxic patients may also make sequencing granulin are a major cause of ubiquitin-positive fron-
errors (e.g., demonstrating the use of a key by totemporal lobar degeneration. Hum Mol Genet.
rotating the wrist, then extending the arm) and 2006;15: 2988-3001.
timing errors such as failure to coordinate Goldman JS, Farmer JM, Wood EM, et al. Comparison of
speed with the spatial aspects of the gesture. family histories in FTLD subtypes and related
(Alexander, 427437; Ochipa, 417478) tauopathies. Neurology. 2005;65:1817-1819.
264 6: Behavioral Neurology

Halliday G, Robinson SR, Shepherd C, Kril J. Alzheimers Ochipa C, Gonzalez Rothi LJ. Limb apraxia. Semin Neurol.
disease and inflammation: a review of cellular and ther- 2000;20:471-478.
apeutic mechanisms. Clin Exp Pharmacol Physiol. 2000;27: Pasquier F, Delacourte A. Non-Alzheimer degenerative
1-8. dementias. Curr Opin Neurol. 1998;11:417-427.
Hansson O, Buchhave P, Zetterberg H, Blennow K, Petersen, RC. Mild cognitive impairment. Continuum:
Minthon L, Warkentin S. Combined rCBF and CSF bio- Lifelong Learning in Neurology. 2007;13(2):(Dementia)15-
markers predict progression from mild cognitive impair- 38.
ment to Alzheimers disease. Neurobiol Aging. 2009;30: Petersen RC, Thomas RG, Grundman M, et al. Vitamin E
165-173. and donepezil in the treatment of mild cognitive
Hansson O, Zetterberg H, Buchhave P, Londos E, impairment. N Engl J Med. 2005;352:2379-2388.
Blennow K, Minthon L. Association between CSF bio- Price DL, Tanzi RE, Borchelt DR, Sisodia SS. Alzheimers
markers and incipient Alzheimers disease in patients disease: genetic studies and transgenic models. Annu
with mild cognitive impairment: a follow-up study. Rev Genet. 1998;32:461-493.
Lancet Neurol. 2006;5:228-234. Saffran EM. Aphasia and the relationship of language and
Hong M, Zhukareva V, Vogelsberg-Ragaglia V, et al. brain. Semin Neurol. 2000;20:409-418.
Mutation-specific functional impairments in distinct Saito Y, Ruberu NN, Sawabe M, et al. Lewy bodyrelated
tau isoforms of hereditary frontotemporal dementia alpha-synucleinopathy in aging. J Neuropathol Exp Neu-
with parkinsonism-17. Science. 1998;282:1914-1917. rol. 2004;63:742-749.
Jellinger KA. Lewy bodyrelated alpha-synucleinopathy in Schoonenboom NS, Pijnenburg YA, Mulder C, et al. Amy-
the aged human brain. J Neural Transm. 2004;111:1219- loid beta(1-42) and phosphorylated tau in CSF as mark-
1235. ers for early-onset Alzheimer disease. Neurology.
Kandell ER, Schwartz JH, Jessell TM, eds. Principles 2004;62:1580-1584.
of Neural Science. 4th ed. New York: McGraw-Hill; Schott JM, Kennedy J, Fox NC. New developments in
2000. mild cognitive impairment and Alzheimers disease.
Knibb JA, Kipps CM, Hodges JR. Frontotemporal demen- Curr Opin Neurol. 2006;19:552-558.
tia. Curr Opin Neurol. 2006;19:565-571. Schulz JB, Dichgans J. Molecular pathogenesis of move-
Kramer JH, Miller BL. Alzheimers disease and its focal ment disorders: are protein aggregates a common link
variants. Semin Neurol. 2000;20:447-454. in neuronal degeneration? Curr Opin Neurol. 1999;12:
Le Ber I, van der Zee J, Hannequin D, et al. Progranulin 433-439.
null mutations in both sporadic and familial frontotem- Sikkink S, Rollinson S, Pickering-Brown SM. The genetics
poral dementia. Hum Mutat. 2007;28:846-855. of frontotemporal lobar degeneration. Curr Opin Neu-
Litvan I. Diagnosis and management of progressive rol. 2007;20:693-698.
supranuclear palsy. Semin Neurol. 2001;21:41-48. van Slegtenhorst M, Lewis J, Hutton M. The molecular
McArthur JC, Sacktor N, Selnes O. Human immunodefi- genetics of the tauopathies. Exp Gerontol. 2000;35:461-471.
ciency virusassociated dementia. Semin Neurol. 1999; Viskontas IV, Possin KL, Miller BL. Symptoms of fron-
19:129-150. totemporal dementia provide insights into
Mega MS, Cummings JL, Fiorello T, Gornbein J. The spec- orbitofrontal cortex function and social behavior. Ann
trum of behavioral changes in Alzheimers disease. N Y Acad Sci. 2007;1121:528-545.
Neurology. 1996;46:130-135. Viskontas I, Miller B. Frontotemporal dementia. Contin-
Mummery CJ, Patterson K, Wise RJ, et al. Disrupted tem- uum: Lifelong Learning in Neurology. 2007;13(2): (Demen-
poral lobe connections in semantic dementia. Brain. tia)87-108.
1999;122:61-73. Whitwell J, Jack CR Jr. Neuroimaging in dementia. Con-
Neary D, Snowden JS, Gustafson L, et al. Frontotemporal tinuum: Lifelong Learning in Neurology. 2007;13(2):
lobar degeneration: a consensus on clinical diagnostic (Dementia)180-203.
criteria. Neurology. 1998;51:1546-1554. Wilhelmsen KC, Lynch T, Pavlou E, et al. Localization of
Nestor P, Hodges J. Non-Alzheimer dementias. Semin disinhibitiondementiaparkinsonismamyotrophy
Neurol. 2000;20:439-446. complex to 17q21-22. Am J Hum Genet. 1994;55:1159-1165.
CHAPTER 7

Cerebrovascular Diseases
Questions

1. After administering intravenous recombinant 4. The main cause of neural death in the core of an
tissue-type plasminogen activator (t-PA) with- ischemic stroke is
out complication, prophylaxis for deep venous
(A) mitochondrial dysfunction
thrombosis with subcutaneous unfractionated
heparin may be started (B) free radical production
(C) apoptosis
(A) immediately
(D) metabolic acidosis
(B) in 24 hours
(E) necrosis
(C) in 4 days
(D) in 7 days 5. A 46-year-old woman with a history of atrial fib-
(E) in 10 days rillation developed a sudden onset of right-sided
weakness and slurred speech. The patient was
2. Ischemic penumbra tissue has a cerebral blood seen in the emergency room within 90 minutes of
flow higher than symptom onset and found to be eligible for treat-
ment with t-PA. Prior to the initiation of this
(A) 3 mL/100 mg per minute
treatment, which of the following tests should be
(B) 5 mL/100 mg per minute ordered?
(C) 7 mL/100 mg per minute
(A) Urine toxicology screen
(D) 10 mL/100 mg per minute
(B) Blood alcohol level
(E) 15 mL/100 mg per minute
(C) Chest x-ray
3. On magnetic resonance imaging (MRI), the (D) Arterial blood gas
area of diffusion/perfusion mismatch in acute (E) Blood glucose
stroke corresponds to the
6. Which of the followings is true of poststroke
(A) area of the brain with irreversible
infection?
ischemic damage
(B) area of the brain with reversible ischemic (A) It is an uncommon complication in the
damage first 5 days after stroke onset.
(C) healthy brain tissue (B) Chest infection occurs exclusively in
(D) hemorrhagic brain lesion patients with dysphagia.
(E) area of the brain with cerebral blood flow (C) Age greater than 65 years is an independ-
less than 5 mL/100 mg per minute ent predictor of pneumonia.
(D) Cellulitis is the most frequent infectious
complication in the first week after stroke.
(E) Prophylactic administration of antibiotics
is strongly recommended in brainstem
stroke.

265
266 7: Cerebrovascular Diseases

7. Which of the following presentations exposes a 9. A few days after developing a left-sided
stroke patient to a high risk of dysphagia? headache, a 42-year-old man suddenly devel-
oped ptosis of the left eye, slurred speech, and
(A) Cortical blindness
right-sided weakness. His angiogram (Figure
(B) Brainstem stroke 7-1) is suggestive of
(C) Asonognosia
(A) basilar artery stenosis
(D) Age greater than 65 years
(B) left vertebral artery dissection
(E) Preservation of consciousness
(C) left carotid artery dissection
8. A 55-year-old man developed acute right-sided (D) left middle cerebral artery occlusion
weakness with aphasia. He was assessed in the (E) complicated migraine
emergency room within 2 hours of symptoms
onset and found to be eligible for recombinant 10. A 67-year-old woman with a history of hyper-
tissue-type plasminogen activator (rt-PA). tension suddenly developed left-sided weak-
However, his blood pressure was 220/110. The ness. Her angiogram (Figure 7-2) is suggestive of
most recommended drug to lower his blood
(A) anterior cerebral artery occlusion
pressure before being treated with rt-PA is
(B) middle cerebral artery occlusion
(A) Lasix (C) posterior cerebral artery occlusion
(B) Enalapril (D) vertebral artery occlusion
(C) Losartan (E) basilar artery occlusion
(D) Labetalol
(E) Prazosin 11. The angiogram in Figure 7-3 demonstrates
(A) carotid artery stenosis
(B) vertebral artery stenosis

FIG. 7-2
FIG. 7-1
Questions: 714 267

(D) vertebral artery occlusion


(E) basilar artery occlusion

13. After 9 hours of sleep, a 56-year-old man awoke


with right-sided weakness and dysarthrias. A
computed tomography (CT) scan showed early
ischemic changes in the left internal capsule.
The most appropriate treatment for this patient
would be
(A) heparin
(B) aspirin
(C) intravenous rt-PA
(D) intra-arterial rt-PA
(E) no anticoagulation because of the risk of
hemorrhagic transformation
FIG. 7-3
14. Subhyaloid hemorrhage is seen in association
with
(C) middle cerebral artery stenosis (A) subarachnoid hemorrhage
(D) basilar artery stenosis (B) seizure
(E) anterior cerebral artery stenosis (C) metabolic encephalopathy
(D) basilar artery occlusion
12. The angiogram in Figure 7-4 demonstrates (E) ischemic encephalopathy
(A) anterior cerebral artery occlusion
(B) middle cerebral artery occlusion
(C) posterior cerebral artery occlusion

FIG. 7-4
268 7: Cerebrovascular Diseases

15. A 44-year-old woman developed a transitory 19. An ischemic central nervous system lesion
loss of vision in her right eye. Her fundoscopic causing ipsilateral ataxia and weakness of the
examination demonstrated a fibrin platelet muscles of mastication with contralateral hemi-
embolus in a branch of the retinal artery. The plegia of face and body as well as loss of all
most likely diagnosis is sensory modalities is located in the
(A) multiple sclerosis (A) medial medulla
(B) optic neuritis (B) lateral medulla
(C) complicated migraine (C) ventral pontine
(D) central nervous system vasculitis (D) lower dorsal pontine
(E) amaurosis fugax (E) upper dorsal pontine

16. Radiological and postmortem studies indicate 20. An ischemic central nervous system lesion
that preoperative strokes are predominantly causing ipsilateral Horner syndrome and
severe tremor with contralateral loss of all sen-
(A) thrombotic sory modalities is located in the
(B) hemorrhagic
(A) lower dorsal pontine
(C) lacunar
(B) upper dorsal pontine
(D) due to hypoperfusion
(C) ventral midbrain
(E) embolic
(D) dorsal midbrain
17. Which of the following is true of subarachnoid (E) dorsolateral midbrain
hemorrhage?
21. An ischemic central nervous system lesion
(A) The mean age at presentation of patients causing ipsilateral oculomotor paresis with
with subarachnoid hemorrhage is 35 ptosis and dilated pupils and contralateral
years. hemiplegia including the lower face is located
(B) The risk for a male is 1.5 times that for a in the
female.
(A) lower dorsal pontine
(C) Most deaths from subarachnoid hemor-
rhage occur within the first 24 hours. (B) upper dorsal pontine
(D) Altered level of consciousness on admis- (C) ventral midbrain
sion points to a poor prognosis. (D) dorsal midbrain
(E) Diabetes is a major modifiable risk factor (E) dorsolateral midbrain
for subarachnoid hemorrhage.
22. Spontaneous dissection of the carotid or verte-
18. The most frequent location of an intracranial bral artery in young patients accounts for
aneurysm is the (A) 1% of all ischemic strokes
(A) posteroinferior cerebellar artery (B) 1.5% of all ischemic strokes
(B) basilar tip (C) 2% of all ischemic strokes
(C) posterior communicating artery (D) 3% of all ischemic strokes
(D) internal carotid bifurcation (E) 10% of all ischemic strokes
(E) pericallosal artery
Questions: 1529 269

23. In cases of carotid dissection, the most fre- (A) Subarachnoid hemorrhage may appear
quently affected cranial nerve is the isodense to brain parenchyma if the
serum hemoglobin level is below 12 g/dL.
(A) facial cranial nerve
(B) Modern CT scanning has 93% sensitivity
(B) glossopharyngeal cranial nerve
for the diagnosis of subarachnoid hemor-
(C) trigeminal cranial nerve rhage when done in the first 12 hours.
(D) spinal accessory cranial nerve (C) Standard MRI is more sensitive than CT
(E) hypoglossal cranial nerve in detecting subarachnoid hemorrhage.
(D) The presence of xanthochromia in the
24. A 27-year-old man with a history of migraine cerebrospinal fluid of patients with a neg-
developed a right-sided headache and right ative head CT scan is the primary crite-
anterolateral cervical pain 4 days after chiro- rion for the diagnosis of subarachnoid
practic manipulation of the neck. One day after hemorrhage.
becoming symptomatic, he consulted a neurol-
(E) Less than 10% of patients with subarach-
ogist because of transient right-eye blindness
noid hemorrhage have a cardiac arrhyth-
that resolved within a few hours. Neurological
mia.
examination demonstrated right miosis, ptosis,
and mild right tongue deviation. The most
28. Which of the following is true of the study
likely diagnosis is
Warfarin-Aspirin Recurrent Stroke Study
(A) vertebral artery dissection (WARSS) comparing aspirin to warfarin in pre-
(B) basilar artery occlusion venting the recurrence of ischemic strokes?
(C) complicated migraine (A) It was a randomized double-blind multi-
(D) carotid artery dissection center study comparing aspirin to war-
(E) cluster headache farin in preventing the recurrence of
cardioembolic strokes.
25. The chain of events that produces the abnor- (B) The only endpoint of this study was
malities seen on diffusion-weighted MRI in death from an ischemic stroke.
acute stroke includes (C) Aspirin showed a higher risk of major
(A) an increased apparent diffusion coeffi- bleed than warfarin when used for pre-
cient venting the recurrence of nonembolic
ischemic strokes.
(B) glial and neuronal swelling
(D) Warfarin was superior to aspirin in pre-
(C) activation of the sodium potassium
venting the recurrence of nonembolic
pumps
ischemic strokes.
(D) intracellular accumulation of potassium
(E) Warfarin was equivalent to aspirin in pre-
(E) an increase in extracellular space venting the recurrence of nonembolic
ischemic strokes.
26. A lacunar stroke located in the subthalamic
nucleus causes 29. Currently, the most consistent and important
(A) hemichoreahemiballismus predictor of stroke is
(B) pure motor hemiparesis (A) hypertension
(C) dysarthria and clumsy hand syndrome (B) diabetes
(D) ataxic hemiparesis (C) high serum cholesterol
(E) agnosia (D) smoking
(E) obesity
27. Which of the following is true of the diagnosis
of subarachnoid hemorrhage?
270 7: Cerebrovascular Diseases

30. Which of the following is true of the use of (A) Hypertension


thrombolytics in the treatment of acute ischemic (B) Ethanol abuse
stroke? (C) Cerebral amyloid angiopathy
(A) The use of rt-PA showed a significant (D) Cholesterol greater than 200 mg/dL
benefit in the first 24 hours. (E) Mutation of the gene coding for coagula-
(B) For every 100 patients treated with rt-PA, tion factor XIII
an additional 11 patients have a favorable
outcome as compared with 100 patients 34. Which of the following is true about the man-
not treated with rt-PA. agement of spontaneous intracerebral bleed?
(C) The administration of rt-PA did not show (A) Since a marked increase of intracerebral
any benefit on stroke due to small vessel pressure is seen in all patients with
disease. intracerebral hemorrhage, the early use of
(D) Within the first 36 hours of rt-PA, signifi- hyperventilation or osmotic agents is rec-
cant intracerebral bleed was present in ommended.
0.6% of cases and was comparable to that (B) Corticosteroids may be used in intracere-
in the placebo group. bral bleed if an osmotic agent or hyper-
(E) Administration of rt-PA is contraindi- ventilation fails to reduce the intracranial
cated if the patient is over 75 years of age. pressure.
(C) Early surgical evacuation of hematoma
31. Which of the following is not true of serum homo- from the basal ganglia or pons has a bet-
cysteine level and the risk of ischemic stroke? ter prognosis than medical treatment.
(A) The relative risk of stroke in patients with (D) Early craniotomy is recommended in
an abnormal homocysteine level is 1.8 cases of cerebellar hematoma.
and depends on the level of homocys- (E) Long-term use of antiseizure medications
teine in the serum. is recommended because most seizures
(B) Homocysteine has a mitogenic effect on occur after the first 24 hours following
vascular smooth muscle. formation of the hematoma.
(C) Elevated homocysteine increases the
activity of coagulation factor XII. 35. The least common presentation of an arteriove-
(D) Statins reduces the plasma homocysteine nous malformation of the brain is
level by 25%. (A) intracerebral hemorrhage
(E) The progression of atherosclerotic carotid (B)seizure
plaque may be decreased by lowering the (C) headache
homocysteine level.
(D) focal neurological deficit without signs of
underlying hemorrhage
32. Positron emission tomography (PET) studies in
ischemic stroke show that the blood flow in (E) progressive neurological deficit without
necrotic tissue is less than signs of underlying hemorrhage

(A) 30 mL/100 gr per minute 36. Which of the following MR spectroscopy peaks
(B) 24 mL/100 gr per minute typically increases in acute ischemic stroke?
(C) 18 mL/100 gr per minute (A) Lactate peak
(D) 12 mL/100 gr per minute (B) Creatine peak
(E) 6 mL/100 gr per minute (C) N-acetyl aspartate peak
(D) Choline peak
33. Which of the following is the most important
risk factor for spontaneous intracerebral hem- (E) None of the above
orrhage?
Answers and Explanations

1. (B) Without the use of heparin prophylaxis, 3. (B) In acute ischemic stroke, the loss of blood
deep venous thrombosis (DVT) of a lower flow from the occluded vessel leads to a time-
extremity may occur in up to half of patients dependent compartmentalization of the ischemic
with hemiplegic stroke. The highest incidence brain into tissue that is irreversibly damaged
occurs between the second and seventh day (ischemic core), tissue that is functionally
poststroke. Elderly patients and those immobi- impaired but structurally intact and thus
lized after stroke appear to be at highest risk. potentially salvageable (penumbra), and tissue
The greatest clinical concern related to proxi- that is hypoperfused but not threatened under
mal DVT is fatal pulmonary embolism (PE). normal circumstances (oligemic brain). The
Estimates of early deaths attributable to PE timing for recanalization of occluded vessels in
range from 13% to 25% and occur most fre- acute stroke was based on the concept that the
quently between the second and fourth week. ischemic penumbra has a short lifespan, being
Measures to prevent DVT should be routine for rapidly incorporated into the core within hours
all patients with ischemic stroke admitted to of the onset of the acute stroke. Several investi-
the hospital. The use of low-intensity anticoag- gators have estimated the penumbra based on
ulation for DVT prophylaxis is recommended diffusion/perfusion MRI (diffusion-weighted
for all immobilized patients with stroke. Anti- imaging [DWI]/perfusion-weighted imaging
coagulants should not be used for 24 hours [PWI]) mismatch in acute stroke. This was
after administration of thrombolytic therapy. In based on the concept that the diffusion abnor-
patients with primary intracerebral hemor- malities are presumed to represent an approxi-
rhage, initiation of anticoagulation for DVT mation of the irreversible ischemic lesion and
prophylaxis is often delayed for 3 to 4 days. the perfusion abnormalities are presumed to
However, definitive evidence to guide manage- represent the brain territory at risk, the area of
ment after intracerebral hemorrhage is not mismatch between DWI and PWI is considered
available. (Barrett, 6179) a territory still viable but at risk of undergoing
infarction and corresponds theoretically to the
2. (C) The ischemic penumbra presents tissue concept of ischemic penumbra. (Jovin, 2845)
that is functionally impaired but structurally
intact and, as such, potentially salvageable. It 4. (E) In acute ischemic stroke, neuronal cell
corresponds to a high cerebral blood flow death occurs as a result of two main mecha-
(CBF) limit of 17 mL/100 mg per minute to nisms: necrosis and apoptosis. Necrosis occurs
22 mL/100 mg per minute and a low CBF limit mainly as a consequence of disruption of cellu-
of 7 mL/100 mg per minute to 12 mL/100 mg lar homeostasis due to energy failure and is
per minute. Salvaging this tissue by restoring accompanied by cellular swelling, membrane
its flow to nonischemic levels is the aim of lysis, inflammation, vascular damage, and
acute stroke therapy. (Jovin, 2845) edema formation. Apoptosis, or programmed
cell death, is characterized by cell shrinkage,

271
272 7: Cerebrovascular Diseases

chromatin clumping, and cytoplasmic blebbing sidered to be independent predictors of pneumo-


and is not associated with inflammation or sec- nia. Pneumonia may occur even without frank
ondary injury to surrounding brain. These two aspiration events or dysphagia. Pneumonia may
distinct types of neuronal death appear to rep- occur in 5.6% of patients after stroke and has been
resent opposite poles of a spectrum that coexist associated with a significantly increased cost of
within the ischemic brain, with necrosis being hospitalization and likelihood of extended care
the main mechanism of neuronal injury in the requirements upon discharge. A study performed
ischemic core and apoptosis being the main by Chamorro and associates assessing prophylac-
mechanism of neuronal injury in the penumbra tic antibiotics to prevent infection after stroke
where, because of the milder degree of does not support their routine use. Urinary tract
ischemia, sufficient energy is produced to infection is common among hospitalized patients
allow for expression of new proteins that medi- with stroke, especially in patients with in-
ate apoptosis. (Jovin, 2845; Bhardwaj, 160167) dwelling bladder catheters, which are often
placed in immobilized patients for urinary reten-
5. (E) Before the administration of thrombolytic tion or incontinence after stroke. Less common
therapy, ancillary tests are performed to supple- infections in patients with stroke include cellulitis
ment the clinical impression based on the bed- (in association with catheter placement or pres-
side assessment. These tests may help to distin- sure ulcers) and cholecystitis. (Adams, 16551711;
guishing condition that may mimic a stroke Azzimondi, 20402043; Chamorro, 14951500; Katzan,
(e.g., hypoglycemia) or identifying conditions 19381943; Kwan, 331338; Sellars, 22842291)
that contraindicate thrombolytic therapy.
Recently published guidelines by Adams and 7. (B) Dysphagia may occur after a stroke in 51% to
coworkers recommend routine laboratory test- 55% of patients. Hemispheric lesions may cause
ing of blood glucose, electrolytes, complete motor impairment of the face, lips, or tongue
blood count, prothrombin time, activated partial and/or attentional deficits that can compromise
thromboplastin time, international normalized normal swallowing function. Brainstem lesions
ratio, and renal function. Testing for stool guaiac can impair the normal coordination of pharyn-
is not routinely recommended unless an indica- geal swallow, laryngeal elevation, glottic closure,
tion exists (e.g., melena or hematochezia). Car- and cricopharyngeal relaxation. Other risk fac-
diac enzymes and a 12-lead ECG are recom- tors of dysphagia after a stroke include impaired
mended for all stroke patients. The utility of consciousness, difficulty or inability to sit
routine chest radiography as part of the acute upright, shortness of breath, slurred speech, facial
stroke evaluation is limited and currently not weakness or droop, expressive aphasia, weak
routinely recommended. Cerebral spinal fluid or wet cough, and hoarse voice. (Barrett et al.,
examination is not routinely indicated unless the 6179; Martino, 27562763)
patient complains of severe headache, which
raises the possibility of subarachnoid hemor- 8. (D) Attempts to lower blood pressure before rt-
rhage or central nervous system infection. Urine PA administration may be undertaken to obtain
toxicology screen, blood alcohol level, arterial the required parameter of less than 185/110
blood gas, or pregnancy tests may be indicated mmHg. The American Heart Association recom-
when the clinical history is limited or uncertain. mends the use of intravenous labetalol, nicardip-
(Adams, 16551711; Barrett et al., 1327) ine, or nitropaste patch to manage hypertension
before rt-PA administration. (Khatri, 4660)
6. (C) Strokes may be complicated by infections,
which are common complications during the first 9. (C) The patient described in this vignette has an
5 days after hospital admission and are associated angiogram significant for a flame-shaped
with worse short-term outcomes. Age greater carotid occlusion. This finding, associated the
than 65 years, dysarthria or expressive aphasia, clinical manifestation described in the vignette,
modified Rankin Scale score of 4 or greater, and is suggestive of left carotid dissection. Some
failure of the bedside water swallow test are con- patients with carotid dissection may develop
Answers: 512 273

warning attacks of unilateral cranial or facial carotid and proximal to its bifurcation. An occlu-
pain followed within minutes to days by signs of sion here blocks the flow in the small deep pene-
ischemia in the territory of the internal carotid trating vessels as well as superficial cortical
artery. The pain is nonthrobbing and centered branches; alternatively, an occlusion at the distal
most often in and around the eye; less often, it is end of the stem blocks the orifices of the divisions
in the frontal or temporal regions, angle of the of the artery in the sylvian sulcus but leaves unaf-
mandible, or high anterior neck over the carotid fected the more proximal deep penetrating ves-
artery. A unilateral Horner syndrome is often sels. The picture of total occlusion of the stem is
present. Cervical bruitsometimes audible to one of contralateral hemiplegia (face, arm, and
the patientamaurosis fugax, faintness and leg), hemianesthesia, and homonymous hemi-
syncope, and facial numbness are less common anopia (due to infarction of the lateral geniculate
symptoms. Most of the patients described by body), with deviation of the head and eyes
Mokri and coworkers presented with one of two toward the side of the lesion; in addition, there is
distinct syndromes: (1) unilateral headache a variable but usually global aphasia with left
associated with an ipsilateral Horner syndrome hemispheric lesions and anosognosia and amor-
or (2) unilateral headache and symptoms of phosynthesis with a right-sided ones. Once fully
delayed focal cerebral ischemia. Some patients established, the motor, sensory, and language
have evidence of involvement of the vagus, deficits remain static or improve very little as
spinal accessory, or hypoglossal nerve; these months and years pass. If the patient is globally
nerves lie in close proximity to the carotid artery aphasic for a prolonged period of time, he or she
and are nourished by small branches from it. will seldom ever again communicate effectively.
(Ropper, chapter 34; Mokri, 5960, 677680) Occlusion of the stem of the middle cere-
bral artery by a thrombus, contrary to conven-
10. (B) The angiogram in Figure 7-2 demonstrated tional teaching, is relatively infrequent (less
occlusion of the proximal part of the middle cere- than 10% of middle cerebral artery occlusions
bral artery (MCA). The MCA is by far the largest in our experience); cerebral embolism is a more
cerebral artery and is the vessel most commonly common cause. Pathological studies over the
affected by cerebrovascular accident (CVA). The years have shown that most carotid occlusions
MCA has superficial and deep hemispheral are thrombotic, whereas most middle cerebral
branches. Through its cortical branches, it sup- occlusions are embolic. (Ropper, chapter 34)
plies the lateral convexity, part of the cerebral
hemisphere. Its cortical territory encompasses (1) 11. (D) The angiogram in Figure 7-3 demonstrated
the cortex and white matter of the lateral and infe- basilar artery stenosis. Patients with basilar
rior parts of the frontal lobeincluding motor artery stenosis may develop transient verte-
areas 4 and 6, contraversive centers for lateral brobasilar ischemic attacks. Symptoms of ver-
gazeand motor speech area of Broca (dominant tebrobasilar circular insuffiency include binoc-
hemisphere); (2) the cortex and white matter of ular visual loss, vertigo, weakness, gait ataxia,
the parietal lobe, including the sensory cortex and dysarthria, and diplopia. (Adams, 82)
the angular and supramarginal convolutions;
and (3) superior parts of the temporal lobe and 12. (E) The basilar artery supplies a wedge of the
insula, including the sensory language areas of pons on either side of the midline with its para-
Wernicke. The deep penetrating or lenticulostri- median branches, the lateral two third of the
ate branches of the middle cerebral artery supply pons, and the middle and superior cerebellar
the putamen, part of the head and body of the peduncles through its short circumferential
caudate nucleus, outer globus pallidus, posterior branches, the cerebellar hemispheres through
limb of the internal capsule, and corona radiata. its long circumferential branches, and the high
The middle cerebral artery may be occluded in its midbrain and medial subthalamic regions
stem (the term M1 is used by radiologists to through its several paramedian branches.
denote this portion of the vessel)the portion Basilar artery occlusion due to thrombosis
that is distal to its origin at the distal internal may arise from occlusion of the basilar artery
274 7: Cerebrovascular Diseases

itself, occlusion of both vertebral arteries, and subhyaloidal, or vitreal blood. The classic pres-
occlusion of a single vertebral artery when it is entation is in the subhyaloidal space. Reports
the only one of adequate size. When there is em- have shown an incidence of 10% to 50% of
bolism, the clot usually lodges at the upper bifur- intraocular hemorrhage with subarachnoid hem-
cation of the basilar or in one of the posterior orrhage. This association is statistically associ-
cerebral arteries. The syndrome of basilar artery ated with the severity of the subarachnoid hem-
occlusion reflects damage to the corticospinal orrhage based on the HuntHess classification
and corticobulbar tracts, cerebellum, middle and system of subarachnoid hemorrhages. The inci-
superior cerebellar peduncles, medial and lateral dence of vitreous hemorrhage is much lower
lemnisci, spinothalamic tracts, medial longitudi- (3% to 13%). Several mechanisms of subhyaloid
nal fasciculi, pontine nuclei, vestibular and hemorrhage have been proposed: (1) A sudden
cochlear nuclei, descending hypothalamospinal increase in intracranial pressure (ICP) forces
sympathetic fibers, and the third through eighth blood from the subarachnoid space directly into
cranial nerves (the nuclei and their segments the preretinal space. (2) A sudden rise in ICP is
within the brainstem). (Ropper, chapter 34) thought to decrease venous return to the cav-
ernous sinus from the veins draining the globe.
13. (B) In this vignette, the patient woke up with The increased retinal venous pressure results in
right side weakness after 9 hours of sleep. For a stasis followed by vessel rupture. (3) A sudden
patient who awakens with symptoms, the time rise in ICP obstructs both the retinochoroidal
of onset is considered to be the time when anastomoses and the central retinal vein due to
the patient went to bed, which in this case is out- a rapid effusion of CSF through the communi-
side the time range for intravenous or intra- cation of the subarachnoid space with the optic
arterial thrombolytic therapy. In two large ran- nerve sheath. This produces an acute decrease
domized trials, the use of aspirin (160 or 300 in venous drainage from the retina and results
mg/day) initiated within 48 hours after the onset in stasis and hemorrhage. (Adams, 85; Castano-
of stroke and continued for 2 weeks or until dis- Duque, 10811083)
charge led to reduced rates of death or depend-
ency at discharge or at 6 months, probably by 15. (E) The association of monocular transitory
means of reducing the risk of recurrent ischemic visual loss with the presence of a fibrin platelet
stroke. In both trials, the routine use of aspirin embolus in a branch of the retinal artery on fun-
was recommended as secondary prevention after doscopic examination is suggestive of amauro-
the first few weeks. Although the benefit was sis fugax. It is the most specific symptom of
small (77 patients would need to be treated to pre- transient ischemia in the carotid circulation.
vent a poor outcome in 1 patient), aspirin is inex- The usual cause the transitory visual loss is
pensive, has a good safety profile, and appears to embolization to the central retinal artery or its
be effective across the range of patients with branches. (Adams, 85)
ischemic stroke. (CAST, 16411649; Chen 12401249;
International Stroke Trial Collaborative Group, 15691581) 16. (E) Radiological and postmortem studies indi-
cate that perioperative strokes are predomi-
14. (A) Subhyaloid hemorrhage is seen in associa- nantly ischemic and embolic. In a study of
tion with subarachnoid hemorrhage (SAH). The 388 patients with stroke after coronary artery
combination of SAH and subhyaloid hemor- bypass grafting (CABG), hemorrhage was
rhage is known as Terson syndrome. It was reported in only 1% of patients; 62% had embolic
defined by the occurrence of vitreous hemor- infarcts. The timing of embolic strokes after sur-
rhage in association with subarachnoid hemor- gery had a bimodal distribution. Approximately
rhage. Terson syndrome now encompasses any 45% of perioperative strokes were identified
intraocular hemorrhage associated with intra- within the first day after surgery. The remain-
cranial hemorrhage and elevated intracranial ing 55% occurred after uneventful recovery
pressure. Intraocular hemorrhage includes the from anesthesia from the second postoperative
development of subretinal, retinal, preretinal, day onward. Early embolism results especially
Answers: 1319 275

from manipulation of the heart and aorta or 18. (C) Intracranial aneurysms are common lesions;
release of particulate matter from the cardiopul- autopsy studies indicate prevalence in the adult
monary bypass pump. Delayed embolism is population between 1% and 5%. Associated con-
often attributed to postoperative atrial fibrilla- ditions include autosomal dominant polycystic
tion, myocardial infarction resulting from an kidney disease, fibromuscular dysplasia, Marfan
imbalance between myocardial oxygen supply syndrome, EhlersDanlos syndrome type IV,
and demand, and coagulopathy. Surgical trauma and arteriovenous malformations of the brain.
and associated tissue injury result in hypercoag- An estimated 5% to 40% of patients with autoso-
ulability. (Selim, 706713; Likosky, 28302834) mal dominant polycystic kidney disease have
intracranial aneurysms and 10% to 30% of
17. (D) Nontraumatic subarachnoid hemorrhage is patients have multiple aneurysms. The most fre-
a neurological emergency characterized by the quent location of intracranial aneurysms is the
extravasation of blood into the spaces covering anterior communicating artery (30%), followed
the central nervous system that are filled with by the posterior communicating artery (25%),
cerebrospinal fluid. The leading cause of non- middle cerebral artery (20%), internal carotid
traumatic subarachnoid hemorrhage is rupture bifurcation (7.5%), basilar tip (7%), pericallosal
of an intracranial aneurysm, which accounts for artery (4%), and posterior inferior cerebellar
about 80% of cases and is associated with a high artery (3%). (Brisman, 928939)
rate of death and complications. It accounts for
2% to 5% of all new strokes and affects 21,000 to 19. (E) The brainstem is formed by three main divi-
33,000 people each year in the United States. sions: the medulla, the pons, and the midbrain.
The incidence of the disorder has remained sta- The medulla is the rostral extension of the spinal
ble over the past 30 years, and although it varies cord. It contains the inferior olivary nucleus as
from region to region, the aggregate worldwide well as the nucleus of the lower cranial nerves.
incidence is about 10.5 cases per 100,000 person- The hypoglossal nucleus is located near the ven-
years. The incidence increases with age, with a trolateral portion of the central canal. The
mean age at presentation of 55 years. The risk nucleus ambiguus (the nucleus of the glossopha-
for women is 1.6 times that of men, and the risk ryngeal nerve, vagus nerve, and spinal accessory
for blacks is 2.1 times that of whites. The aver- nerves) is located within the medullary reticular
age case fatality rate for subarachnoid hemor- formation, ventromedial to the nucleus and
rhage is 51%, with approximately one third of spinal tract of the trigeminal nerve.
survivors needing lifelong care. Most deaths The dorsal motor nucleus of the vagus is
occur within 2 weeks after the ictus, with 10% located dorsolaterally to the hypoglossal
occurring before the patient receives medical nucleus. The nucleus of the tractus solitarius
attention and 25% within 24 hours after the (nucleus of the sensory facial, glossopharyn-
event. The major factors associated with poor geal, and vagus nerves) lies anterolateral to the
outcome are the patients level of consciousness motor nucleus of the vagus nerve. The postero-
on admission, his or her age, and the amount of lateral to the solitary tracts lie medial and infe-
blood shown by initial CT of the head. The rior to the vestibular nuclei; caudal to them are
major identified modifiable risk factors include the dorsal and ventral cochlear nuclei of the
cigarette smoking, hypertension, cocaine use, vestibulocochlear nerve. The nucleus gracilis
and heavy alcohol use. Patients with a family and nucleus cuneatus are located in the poste-
history of first-degree relatives with subarach- rior funiculi of the dorsal medulla and give off
noid hemorrhage are also at a higher risk. Heri- fibers that decussate in the medial lemniscus.
table connective tissue disorders associated Medial medullary syndrome (Dejerine syn-
with the presence of intracranial aneurysm and drome) is caused by an occlusion of the anterior
subarachnoid hemorrhage include polycystic spinal artery or its parent vertebral artery. The
kidney disease, EhlersDanlos syndrome type syndrome causes ipsilateral paresis of the
IV, pseudoxanthoma elasticum, and fibromus- tongue due to damage of the hypoglossal nerve,
cular dysplasia. (Suarez, 387396) which deviates toward the lesion; contralateral
276 7: Cerebrovascular Diseases

hemiplegia sparing the face due to damage of arteries and the superior cerebellar arteries). The
the corticospinal tract; and contralateral loss of ventral pontine syndrome (MillardGubler syn-
position and vibratory sensation due to damage drome) is caused by paramedian infarction of the
of the medial lemniscus. pons. This results in ipsilateral paresis of the lat-
The lateral medullary syndrome (Wallen- eral rectus from damage to the abducens nerve,
berg syndrome) may be caused by an occlusion causing diplopia; in addition, there is ipsilateral
of the vertebral artery or posterior inferior cere- paresis of the upper and lower face from damage
bellar artery. The damage is located in the dorso- to the facial cranial nerve and contralateral hemi-
lateral medulla and inferior cerebellar peduncle. plegia from damage to the corticospinal tract.
The clinical features of lateral medullary syn- The lower dorsal pontine syndrome (Foville
drome include ipsilateral loss of pain and tem- syndrome) is caused by a lesion in the dorsal
perature sensation of the face due to damage of tegmentum of the lower pons. The affected
the descending spinal tract and nucleus of the patient may develop ipsilateral paresis of the
trigeminal nerve; ipsilateral paralysis of the upper and lower halves of the face from damage
palate, pharynx, and vocal cords with dysarthria to the nucleus or fibers of the facial nerve and
and dysphagia due to damage of the nuclei and ipsilateral horizontal gaze palsy from damage of
fibers of the glossopharyngeal and vagus nerves; the paramedian pontine reticular formation
ipsilateral Horner syndrome due to damage of and/or the abducens nerve nucleus. The upper
the descending sympathetic tract fibers; ipsilat- dorsal pontine syndrome is caused by obstruc-
eral ataxia and dysmetria due to damage of the tion of the long circumferential branches of the
inferior cerebellar peduncle and cerebellum; con- basilar artery and results in ipsilateral ataxia and
tralateral loss of pain and temperature sensation coarse intention tremor (the superior and middle
due to damage of the spinothalamic tract; and cerebellar peduncles). There is also contralateral
vertigo, nausea, vomiting, and nystagmus due to body sensory loss to all modalities from damage
damage of the vestibular nuclei. to the medial lemniscus and spinothalamic tract.
The pons lies rostral to the medulla. It has When the lesion extends to the ventral part of the
two components: a dorsal part (the tegmen- pons, contralateral hemiparesis, including the
tum) and a ventral part (the basis pontis). The face, occurs from damage to the corticospinal
tegmentum is largely composed of the pontine tract. (Afifi, 141146, 179186, 227234; Rolak, 112121)
reticular formation. Cranial nerve nuclei in
the pons include the nucleus of the abducens 20. (E) The midbrain, which is the smallest and the
nerve (located in the dorsomedial pons, dorso- most rostral component of the brainstem, plays
lateral to the paramedian pontine reticular for- an important role in the control of eye move-
mation), the motor nucleus of the facial nerve ments and coordination of visual and auditory
(situated ventrolaterally), the main motor and reflexes. The midbrain may be divided into three
sensory nucleus of the trigeminal nerve, the parts: the tectum, the tegmentum, and the cere-
superior and inferior salivatory nuclei, and the bral peduncles. The dorsal tectum contains the
lacrimal nucleus. Tracts within the pons corpora quadrigemina, made up of four
include the medial longitudinal fasciculus, the rounded eminences arranged in pairs, the supe-
medial lemniscus, and the corticospinal, the cor- rior and inferior colliculi. The tegmentum con-
ticobulbar, the corticopontine, the spinothala- tains ascending and descending tracts, reticular
mic, the ventral spinocerebellar, the rubrospinal, nuclei, and well-delineated nuclear masses. The
and the lateral tectospinal tracts. cerebral peduncles are ventral and contain corti-
The basilar artery is the principal source of copontine fibers in their medial fifth, corti-
blood flow to the pons. It gives off three types of cospinal tract fibers in their middle three-fifths,
branches: the paramedian arteries, the short cir- and temporopontine fibers in their lateral fifth.
cumferential arteries, and the long circumferen- The substantia nigra is a pigmented layer pos-
tial arteries (which supply the pontine tegmen- sessing melanin granules; dorsal to the peduncle
tum and the dorsolateral quadrant of the pons, and ventral to the red nucleus, it is composed
together with the anterior inferior cerebellar of the dorsal zona compacta and ventral zona
Answers: 2024 277

reticulata. The nucleus of the trochlear nerve is young and middle-aged populations, it accounts
located in the ventral part of the central gray for 10% to 25% of ischemic strokes. Spontaneous
matter at the level of the inferior colliculus. The dissections of the carotid and vertebral arteries
nucleus of the oculomotor nerve lies rostral to affect all age groups, but there is a distinct peak in
the trochlear nucleus beneath the superior col- the fifth decade of life. Although there is no over-
liculus. Mesencephalic tracts include the crus all sex-based predilection, women are, on aver-
cerebri, the dentatorubrothalamic tract, the age, about 5 years younger than men at the time
medial tegmental tract, the posterior commis- of the dissection. (Schievink, 898906)
sure, the median longitudinal fasciculus, the
spinothalamic tract, and the median lemniscus. 23. (E) Cranial nerves are affected in about 12% of
The vascular supply of the midbrain includes carotid artery dissections. The hypoglossal
the paramedian and circumferential branches of nerve is the most commonly affected cranial
the basilar artery. nerve, followed by other lower cranial nerves.
Dorsolateral midbrain syndrome is caused The oculomotor nerve, trigeminal nerve, and
by infarction of the territory of the circumferen- facial nerve are less affected. (Schievink, 898906)
tial arteries and results in (1) ipsilateral Horner
syndrome; (2) ipsilateral severe tremor from 24. (D) The patient described in this vignette has a
damage to the superior cerebellar peduncle; triad of signs and symptoms that include uni-
and (3) contralateral loss of all sensory modali- lateral headache, unilateral oculosympathetic
ties from damage to the spinothalamic and palsy, and transient monocular blindness
medial lemniscus tracts. (Afifi, 141146, 179186, (which may be considered in this context as a
227234; Rolak, 112121) sign of retinal ischemia). The occurrence of this
constellation of signs and symptoms after chi-
21. (C) Ventral midbrain syndrome (Weber syn- ropractic manipulation is highly suggestive of
drome) is caused by an occlusion of median and carotid artery dissection. Hyperextension or
paramedian perforating branches and may result rotation of the neck may also precipitate carotid
in ipsilateral oculomotor paresis, ptosis, and dissection. It is estimated that about 1 in 20,000
dilated pupils from damage to the fascicle of the of spinal manipulations are associated with a
third nerve. In addition, there is a contralateral stroke by carotid or vertebral artery dissection.
hemiplegia, including the lower face, from dam- Unilateral headache develops in two thirds of
age to the corticospinal tract. Dorsal midbrain patients. It may mimic a migraine headache or a
syndrome (MoritzBenedikt syndrome) results subarachnoid hemorrhage, but most commonly
from a lesion in the midbrain tegmentum caused it is a frontotemporal headache with gradual
by occlusion of the paramedian branches of the onset, often above the ipsilateral eye. Miosis and
basilar or posterior cerebral arteries or both. It ptosis reflect oculosympathetic palsy and are
results in ipsilateral oculomotor paresis, ptosis, seen in less than 50% of patients with carotid dis-
and dilated pupil from damage to the third section. A cranial nerve lesion is detected in only
nerve. Contralateral involuntary movements 12% of cases of carotid artery dissection. The
also occur, such as intention tremor, ataxia, and hypoglossal nerve is the most frequently affected
chorea, from damage to the red nucleus. There is cranial nerve. Transient monocular blindness is
a contralateral hemiparesis with extension of the the most common sign of retinal ischemia seen in
lesion ventrally and contralateral hemianesthesia patients with spontaneous carotid dissection.
with extension of the lesion laterally to the Permanent blindness from ischemic optic neu-
spinothalamic tracts and the medial lemniscus. ropathy or occlusion of the retinal artery is rare.
(Afifi, 141146, 179186, 227234; Rolak, 112121) Symptoms and signs of retinal or cerebral
ischemia are reported in 50% to 95% of carotid
22. (E) Spontaneous dissection of the carotid or ver- artery dissections. Ultrasound or MR angiogra-
tebral artery accounts for about 2% of all ischemic phy of the carotid artery is useful for an initial
strokes. Its annual incidence in some studies assessment, but carotid angiography remains the
ranges from 2.5% to 3% per 100,000. However, in gold standard. (Schievink, 898906)
278 7: Cerebrovascular Diseases

25. (B) Diffusion-weighted imaging is a technique 27. (D) The typical presentation of subarachnoid
that permits in vivo measurement of the transla- hemorrhage includes a sudden onset of severe
tional mobility of water along the particular headache, frequently described as the worst
direction of the used diffusion-sensitizing gradi- headache of the patients life. Nausea, vomiting,
ent in tissue. The apparent diffusion coefficient transient loss of consciousness, or leg buckling
that quantifies water mobility is reduced in may accompany the headache. Physical exami-
ischemic tissue. The drop in brain perfusion nation may show nuchal rigidity, retinal hemor-
caused by an acute ischemic event induces an rhages, papilledema, third nerve palsy (in case of
energy deficit, with failure of the sodium/potas- posterior communicating artery aneurysm),
sium pump. This results in the accumulation of sixth nerve palsy, bilateral weakness in legs (or
sodium inside the neural and glial cells, which abulia in case of aneurysm of the anterior com-
causes an intracellular influx of water inside municating artery), and/or aphasia hemiparesis
these cells. This influx of water from the extracel- (or left-sided visual neglect in the case of a mid-
lular space into the intracellular space causes dle cerebral artery aneurysm). Arrythmias are a
swelling of the neuronal and glial cells and a frequent complication of subarachnoid bleeding.
reduction of the extracellular volume space. Andreoli and coworkers reported arrhythmias in
Thus, water mobility is hindered as a conse- 91% of patients diagnosed with spontaneous
quence of the production of longer diffusion subarachnoid hemorrhage and investigated
paths and the apparent diffusion coefficient of prospectively with 24-hour Holter monitoring.
water is decreased. (Schievink, 898906) This study did not demonstrate a correlation
between the frequency and severity of cardiac
26. (A) Lacunar strokes are small infarcts of the arrhythmias and the neurologicl condition, the
noncortical parts of the cerebellum and the site and extent of intracranial blood on CT scan,
brain stem that result from occlusion of pene- or the location of the ruptured vessel.
trating branches of large cerebral arteries, most CT scan of the head without contrast and
commonly the middle cerebral, basilar, and with thin cuts through the base of the brain is the
posterior cerebral arteries and less commonly first recommended test. In retrospective studies,
the anterior cerebral and vertebral arteries. The the sensitivity of modern third-generation CT
size of a lacunar stroke ranges from 3 mm to scanners for detecting subarachnoid hemorrhage
2 cm. Pathological studies show that the pene- is 100% in the first 12 hours and 93% in the first 24
trating vessels might be obstructed either by hours. Although MRI technology is continually
small embolic particles or lipohyalinosis. Signs advancing and can detect aneurysms, standard
of lacunar stroke depend on the anatomical MRI is inferior to CT for the detection of acute
location of the ischemic lesion. Hemiballismus subarachnoid hemorrhage. Lumbar puncture
is caused by an infarct or hemorrhage in the should be performed in a patient whose clinical
subthalamic nucleus. Pure motor hemiparesis presentation suggests subarachnoid hemorrhage
involves the face, arm, and leg without sensory and whose CT scan is negative, equivocal, or
deficit, aphasia, agnosia, apraxia, or visual technically inadequate. After aneurysmal hemor-
field defect. The most frequent location of the rhage, erythrocytes invade the subarachnoid
lesion is the posterior limb of the internal cap- space and then gradually lyse to release hemoglo-
sule and less frequently the corona radiata, bin, which is metabolized to the pigmented mole-
pons, and medullary pyramid. Infarction of cules, oxyhemoglobin (reddish pink) and biliru-
the basis pontis at the junction of the upper bin (yellow), resulting in xanthochromia. The
third and inferior two thirds from obstruction of presence of xanthochromia is considered by
the paramedian branch of the basilar artery many authors to be the primary criterion for a
causes dysarthria clumsy hand syndrome. diagnosis of subarachnoid hemorrhage in pa-
Ataxia hemiparesis results from a lacunar lesion tients with negative CT scans, although some
located in the corona radiata, internal capsule, authors assert that the presence of erythrocytes,
or pons. (Fisher, 871876; Fisher and Bogousslavsky, even in the absence of xanthochromia, is more
108) accurate. (Andreoli, 558564; Edlow, 2936)
Answers: 2531 279

28. (E) WARSS was a multicenter, double blind, direct endothelial damage. Tobacco may pro-
randomized trial that compared the effect of mote arterial rupture by increasing blood pres-
warfarin (at a dose adjusted to produce an inter- sure.
national normalized ratio of 1.4 to 2.8) and Diabetes may increase the risk of stroke
aspirin (325 mg per day) on the combined pri- independently of hypertension, dyslipopro-
mary endpoints of recurrent ischemic stroke or teinemia, and obesity. Diabetes may increase
death from any cause within 2 years. The two the risk of stroke by promoting atherogenesis.
randomized study groups were similar with Obesity, elevated serum cholesterol, and lack of
respect to baseline risk factors. No significant physical activity are also risk factors for stroke.
differences were found between the treatment (Bronner, 13921400)
groups in any of the outcomes measured. The
rates of major hemorrhage were low (2.22 per 30. (B) rt-PA is produced endogenously in physio-
100 patient-years in the warfarin group and 1.49 logical concentrations by endothelial cells. It is
per 100 patient-years in the aspirin group). Also, relatively fibrin-specific. The NINDS trial ran-
there were no significant treatment-related dif- domized 624 patients (312 each to placebo and
ferences in the frequency of major hemorrhage intravenous rt-PA) within 3 hours after stroke
according to the cause of the initial stroke. The symptom onset. The trial had two parts: Part 1
authors concluded that over a 2-year period, (in which 301 patients were enrolled) tested
there was no difference between aspirin- and whether rt-PA demonstrated a clinical effect
warfarin-treated patients in the rate of major within 24 hours of the onset of stroke (primary
hemorrhage or in the prevention of recurrent endpoint). Part 2 (in which 333 patients were
ischemic stroke (for a population with noncar- enrolled) used a global test statistic to assess
dioembolic stroke) or death. (Mohr, 14441451) clinical outcome at 3 months. There was no sig-
nificant difference between the drug treatment
29. (A) Hypertension is reported to be the most con- and placebo groups in the percentages of
sistently powerful predictor of stroke. It is found patients with neurological improvement at 24
to be a contributing factor in about 70% of hours. A benefit was observed for the rt-PA
strokes. Hypertension promotes stroke by aggra- group at 3 months for all outcome measures.
vating atherosclerosis in the aortic arch and cer- For every 100 patients treated with rt-PA, an
vicocerebral arteries, causing arteriosclerosis and additional 11 to 13 patients will have a favor-
lipohyalinosis in the small-diameter, penetrating able outcome as compared with 100 not treated
end arteries of the cerebrum. For people of all with rt-PA. The benefit did not vary by stroke
ages and both sexes, higher levels of both systolic subtype at baseline because not only cardioem-
and diastolic blood pressure have been associ- bolic strokes but also small-vessel ischemic
ated with an increased incidence of ischemic and strokes benefited from thrombolytic treatment.
hemorrhagic stroke. Treatments not only for Symptomatic intracerebral hemorrhage within
severe hypertension but also mild-to-moderate 36 hours after the onset of stroke occurred in
hypertension have been associated with a de- 6.4% of patients given rt-PA but only in 0.6% of
crease in the incidence of stroke. patients given placebo ( p 0.001). Neverthe-
Cigarette smoking is also a major cause of less, severe disability and death were higher in
both ischemic and hemorrhagic stroke. The rela- the nontreated group. (Schellinger, 18121818)
tive risk of stroke for smokers, as compared with
nonsmokers, is estimated to be close to 1.51. Cig- 31. (D) A prospective population-based cohort
arette smoking may increase the risk of stroke by study of patients without a history of stroke who
modifying blood coagulability (increasing blood were followed for a median of 9.9 years identi-
levels of fibrinogen and other clotting factors, fied homocysteine level as an independent risk
increasing platelet aggregability and hemat- factor for stroke. This relative risk increases with
ocrit), modifying the lipid profile of smokers the level of homocysteine up to 1.82. Homo-
(decreasing the high-density lipoprotein choles- cysteine may act by inducing endothelial
terol level), and promoting atherosclerosis by dysfunction and altering the antithrombotic
280 7: Cerebrovascular Diseases

properties of the endothelium with enhance- with (epsilon)2 and (epsilon)4 alleles of the
ment of the activity of factors XII and V. It may apolipoprotein E gene. (Qureshi et al., 14501460)
also have a mitogenic effect on vascular smooth
muscle cells. Statins do not lower homocysteine 34. (D) Several days after intracerebral hemor-
levels, but dietary folic acid may reduce the level rhage, the patient may develop obstructive
of homocysteine by approximately 25% and hydrocephalus from the mass effect caused by
vitamin B12 may reduce it by an additional 7%. It the hematoma and the edematous tissue sur-
remains unknown whether lowering homocys- rounding it. The resulting increased intracranial
teine prevents important atherosclerotic vascu- pressure may subsequently cause central nerv-
lar events. However, an uncontrolled study has ous system herniation, which remains the chief
shown that lowering homocysteine through the secondary cause of death in the first few days
combination of folic acid 2.5 mg/day, vitamin B6 after intracerebral hemorrhage. Marked eleva-
25 mg/day, and vitamin B12 250 g/day reduces tion of the intracranial pressure occurs with
the progression of atherosclerosis as measured massive intracerebral hemorrhages because the
by carotid plaque area. (Hankey, 95102) intracranial volume cannot expand. However,
localized intracerebral hemorrhage may occur
32. (D) PET classifies three regions within the dis- without significant increase in global intracra-
turbed vascular territory. The core zone of nial pressure.
ischemia, which usually becomes necrotic, has The use of hyperventilation and osmotic
blood flow below 12 mL/100 gr per minute agents is discouraged in the absence of evidence
and a cerebral metabolic rate for oxygen of a critical rise in intracranial pressure and should
(CMRO2) below 60 mol/100 gr per minute. be reserved for patients with impending cerebral
The penumbral region has a flow rate between herniation. This strategy is supported by some
12 and 22 mL/100 gr per minute (it is a dynamic experimental studies showing improvement of
area where the tissue is still viable but can the blood flow and cerebral metabolism when
either become necrotic or recover; it is charac- high intracranial pressure is lowered in cases of
terized by increased oxygen extraction frac- intracranial herniation but no benefit with moder-
tion). The third region is an area of hypopofu- ate pressure elevations. Corticosteroids should be
sion (flow 22 mL/100 gr per minute), which avoided because randomized trials have failed to
is not primarily damaged by the lack of blood demonstrate their efficacy in patients with an
supply. (Heiss, 6775) intracerebral hemorrhage.
Surgical evacuation of a hematoma is indi-
33. (A) Hypertension is the most important risk cated to reduce the mass effect. However, there
factor for spontaneous intracerebral hemor- is no sustained benefit from evacuation of basal
rhage. It increases the risk of intracerebral ganglionic, thalamic, and pontine hemorrhages.
hemorrhage, particularly in untreated people Cerebellar hematomas are unique from a sur-
55 years of age or younger and those who gical perspective because they can be approached
smoke. Excessive use of alcohol also increases without causing substantial damage to higher
the risk of intracerebral hemorrhage by impair- cortical or primary motor pathways. Thus early
ing coagulation and directly affecting the craniotomy is recommended in patients with a
integrity of cerebral vessels. Low serum cho- cerebellar hematoma that causes compression
lesterol (less than 4.1 mmol/L), especially of the brainstem because the rate of neurologic
when associated with hypertension or hyper- deterioration after cerebellar hemorrhage is
coagulable status, is a less well established risk very high and unpredictable. Most patients
factor for intracerebral hemorrhage. Cerebral with intracerebral bleed do not require long-
amyloid angiopathy, which is characterized by term antiseizure medications, since most
the deposition of (beta)-amyloid protein in the seizures occur at the onset of the hemorrhage or
blood vessels of the cerebral cortex and lep- in the first 24 hours. Anticonvulsants can usu-
tomeninges, is another risk factor for intracere- ally be discontinued after the first month in
bral hemorrhage, particularly in elderly persons patients who have had no further seizures.
Answers: 3236 281

Patients who have a seizure more than 2 weeks decrease, or remain unchanged. A lactate dou-
after the onset of an intracerebral hemorrhage blet peak is seen at 1.33 ppm. It is not normally
are at higher risk for further seizures and may detected within the brain. Its concentration rises
require long-term prophylactic treatment with when the glycolytic rate exceeds the tissues
anticonvulsants. (Qureshi et al., 14501460) capacity to catabolize or remove it from the
brain and into the circulation. The presence of
35. (E) Intracranial hemorrhage is the most com- lactate may thus represent a perfusion mismatch
mon clinical presentation of arteriovenous mal- and may possibly point to salvageable tissue.
formation, with a reported frequency ranging (Saunders, 334345)
from 30% to 82%. Several factors increase the
risk of a first hemorrhage: a small malforma- REFERENCES
tion, exclusively deep venous drainage, and
high intracranial pressure resulting in high Adams HP Jr, del Zoppo G, Alberts MJ, et al. Guidelines for
pressures to the feeding arteries or restriction of the early management of adults with ischemic stroke: a
venous outflow. Seizures that are not caused by guideline from the American Heart Association/
hemorrhage are the initial symptom in 16% to American Stroke Council, Clinical Cardiology Council,
53% of patients. The majority of seizures are Cardiovascular Radiology and Intervention Council,
partial or complex partial; grand mal seizures and the Atherosclerotic Peripheral Vascular Disease
account for 27% to 35% of seizures. Headache and Quality of Care Outcomes in Research Interdisci-
is the presenting symptom in 7% to 48% of plinary Working Groups: the American Academy of
Neurology affirms the value of this guideline as an edu-
patients, without distinctive features such as
cational tool for neurologists [published errata appear
frequency, duration, or severity. Focal neuro-
in Stroke 2007;38(6):e38, 2007;38(9):e96]. Stroke. 2007;
logical deficits without signs of underlying 38(6):1655-1711.
hemorrhage have been reported in 1% to 40% Adams HP. Principles of Cerebrovascular Disease. New York:
of patients, and only a few of them (4% to 8%) McGraw-Hill Medical; 2007:564[4].
have well-documented progressive neurologi- Afifi AK, Bergman RA, Ronald A, eds. Functional Neu-
cal deficits. (Anonymous, 18121818) ranatomy: Text and Atlas. New York: McGraw-Hill; 2005.
Andreoli A, di Pasquale G, Pinelli G, Grazi P, Tognetti F,
36. (A) Magnetic resonance spectroscopy (MRS) is Testa C. Subarachnoid hemorrhage: frequency and
a noninvasive in vivo technique that allows the severity of cardiac arrhythmias: a survey of 70 cases
measurement of histochemical cell components. studied in the acute phase. Stroke. 1987;18:558-564.
Specific cell types or structures have metabolites Anonymous. Arteriovenous malformations of the brain
in adults. N Engl J Med. 1999;340(23):1812-1818.
that give a change in proton MRS peaks that
Azzimondi G, Bassein L, Nonino F, et al. Fever in acute
may reflect a loss of a specific cell component.
stroke worsens prognosis: a prospective study. Stroke
The methyl resonance of N-acetyl aspartate 1995;26(11):2040-2043.
(NAA) produces a sharp peak at 2.01 ppm. It Barrett KM, Khatri P, Jovin TG. Complications of ischemic
acts as a specific neuronal marker and reflects stroke: prevention and management. Continuum: Life-
neuronal integrity. In cases of acute ischemia, the long Learning in Neurology. 2008;14(6):(Acute Ischemic
NAA peak declines, reflecting neuronal loss. Stroke)61-79.
Creatine and phosphocreatine have specific Barrett KM, Levine JM, Johnston KC. Diagnosis of stroke
MRS signals. They are found in both neurons and stroke mimics in the emergency setting. Contin-
and glial cells and act as a phosphate transporter uum: Lifelong Learning in Neurology. 2008;14(6):(Acute
system and energy buffer within the cell. In Ischemic Stroke)13-278.
acute ischemic stroke, there is a reduction in the Bhardwaj A, Alkayed NJ, Kirsch JR, Hurn PD. Mecha-
nisms of ischemic brain damage. Curr Cardiol Rep.
peak of creatine and phosphocreatine, reflecting
2003;5(2):160-167.
the disturbance of cellular energy metabolism.
Brisman JL, Song JK, Newell DW. Cerebral aneurysms. N
The trimethylamine resonance of the choline- Engl J Med. 2006;355:928-939.
containing component is present in 3.2 ppm. It is Bronner LL, Kanter DS, Manson JE. Medical progress: pri-
a marker of cell membrane integrity. Also, in mary prevention of stroke. N Engl J Med. 1995;333:1392-
acute ischemia, the choline peak may increase, 1400.
282 7: Cerebrovascular Diseases

CAST (Chinese Acute Stroke Trial) Collaborative Group. coronary artery bypass graft surgery. Stroke. 2003;34:
CAST: randomised placebo-controlled trial of early 2830-2834.
aspirin use in 20,000 patients with acute ischaemic Martino R, Foley N, Bhogal S, et al. Dysphagia after
stroke. Lancet. 1997;349:1641-1649. stroke: incidence, diagnosis, and pulmonary complica-
Castano-Duque CH, Pons-Irazazabal LC, Lopez-Moreno tions. Stroke. 2005;36(12):2756-2763.
JL. [Subarachnoid hemorrhage associated to sub- McCarron MO, Alberts MJ, McCarron P. A systematic
hyaloid hemorrhage: Terson syndrome.] Rev Neurol. review of Tersons syndrome: frequency and prognosis
1997;25:1081-1083. after subarachnoid haemorrhage. J Neurol Neurosurg
Chamorro A, Horcajada JP, Obach V, et al. The Early Sys- Psychiatry. 2004;75:491-493.
temic Prophylaxis of Infection After Stroke study: a Mohr JP, Thompson JLP, Lazar RM, et al. A comparison of
randomized clinical trial. Stroke. 2005;36(7):1495-1500. warfarin and aspirin for the prevention of recurrent
Chen ZM, Sandercock P, Pan HC, et al. Indications for ischemic stroke. N Engl J Med. 2001;345:1444-1451.
early aspirin use in acute ischemic stroke: a combined Mokri B. Carotid arterial dissection as a cause of severe
analysis of 40 000 randomized patients from the Chi- brain infarction in young adults. J Stroke Cerebrovasc
nese Acute Stroke Trial and the International Stroke Dis. 1996;6(2):59-60.
Trial. Stroke. 2000;31:1240-1249. Mokri B, Sundt TM Jr, Houser OW. Spontaneous internal
Edlow JA, Caplan LR. Avoiding pitfalls in the diagnosis of carotid dissection, hemicrania, and Horners syn-
subarachnoid hemorrhage. N Engl J Med. 2000;342:29-36. drome. Arch Neurol. 1979;36(11):677-680.
Fisher CM. Lacunar strokes and infarcts: a review. Neurol- Qureshi AI, Tuhrim S. Broderick JP. Batjer HH. Hondo H.
ogy. 1982;32:871-876. Hanley DF. Spontaneous intracerebral hemorrhage. N
Fisher M, Bogousslavsky J, eds. Current Review of Cerebro- Engl J Med. 2001;344:1450-1460.
vascular Disease. 4th ed. Philadelphia: Current Medi- Rolak LA, ed. Neurology Secrets. 2nd ed. Philadelphia:
cine; 2001. Hanley & Belfus; 1998.
Hankey GJ, Eikelboom JW. Homocysteine and stroke. Ropper AH, Brown RH. Cerebrovascular diseases. In:
Curr Opin Neurol. 2001;14:95-102. Ropper AH, Brown RH, eds. Adams and Victors Princi-
Heiss WD, Forsting M, Diener HC. Imaging in cere- ples of Neurology. 8th ed. Chapter 34. Available at http://
brovascular disease. Curr Opin Neurol. 2001;14(1):67-75. www.accessmedicine.com/content.aspx?aID=973845
International Stroke Trial Collaborative Group. The Inter- Saunders DE. MR spectroscopy in stroke. Br Med Bull
national Stroke Trial (IST): a randomised trial of aspirin, Stroke. 2000;56:334-345.
subcutaneous heparin, both, or neither among 19,435 Schellinger PD, Fiebach JB, Mohr A, Ringleb PA, Jansen
patients with acute ischaemic stroke. Lancet. 1997;349: O, Hacke W. Thrombolytic therapy for ischemic
1569-1581. strokea review. Part IIntravenous thrombolysis.
Jovin TG, Demchuk AM, Gupta R. Pathophysiology of Crit Care Med. 2001;29:1812-1818.
acute ischemic stroke. Continuum: Lifelong Learning in Schievink WI. Spontaneous dissection of the carotid and
Neurology. 2008;14(6):(Acute Ischemic Stroke)28-45. vertebral arteries. N Engl J Med. 2001;344:898-906.
Katzan IL, Dawson NV, Thomas CL, et al. The cost of pneu- Selim M. Perioperative stroke. N Engl J Med. 2007;356:706-
monia after acute stroke. Neurology. 2007;68(22):1938-1943. 713.
Kwan J, Hand P. Infection after stroke is associated with Sellars C, Bowie L, Bagg J, et al. Risk factors for chest
poor short-term outcome. Acta Neurol Scand. 2007;115 infection in acute stroke: a prospective cohort study.
(5):331-338. Stroke. 2007;38(8):2284-2291.
Likosky DS, Marrin CA, Caplan LR, et al. Determina- Suarez JI, Tarr RW, Selman WR. Aneurysmal subarach-
tion of etiologic mechanisms of strokes secondary to noid hemorrhage. N Engl J Med. 2006;354:387-339.
CHAPTER 8

Infections of the Nervous System


Questions

1. Which of the following pathological features worsening stiffness in his lower extremities. He
correlates best with the severity of AIDS demen- has also noticed increasing urinary frequency and
tia? urgency over the last 6 months. On neurological
examination, the patient displays mild parapare-
(A) The number of nodules containing
sis of lower extremities with spasticity and brisk
macrophages, lymphocytes, and microglia
reflexes in the left knee and ankle as well as bilat-
(B) Multinucleated giant cells eral Babinski signs. Sensory examination shows
(C) Cortical atrophy moderate loss of proprioception in the legs with-
(D) Neuronal loss out a sensory level.
(E) Macrophage activation CSF (CSF) analysis shows a protein level of
63 mg/dL, WBCs at 5/mm3, 100% lymphocytes,
2. A 40-year-old man diagnosed with HIV-associ- and glucose 50 mg/dL. CSF viral and bacterio-
ated dementia is able to perform basic activities logic studies are normal. Vitamin B12 level is
of self-care but cannot work or maintain the normal. Magnetic resonance imaging (MRI) of
more demanding aspects of daily life. He is able the cervical, thoracic, and lumbar spine with
to ambulate. In which stage of HIV-associated gadolinium enhancement is normal. Somato-
dementia Memorial Sloan Kettering (MSK clini- sensory evoked potentials show a prolongation
cal staging system) can the patient be classified? of the central conduction time. The most likely
diagnosis is
(A) Stage 0
(B) Stage 1 (A) lymphoma
(C) Stage 2 (B) cytomegalovirus myelopathy
(D) Stage 3 (C) AIDS-associated vacuolar myelopathy
(E) Stage 4 (D) HIV myelitis
(E) bacterial paraspinal abscess
3. Neuropsychological tests in the initial stage of
AIDS dementia may show 5. The most common type of peripheral neuropa-
thy in AIDS patients is
(A) selective memory loss; impaired retrieval
(B) severe attention deficit (A) distal symmetric polyneuropathy
(C) impaired calculation (B) acute inflammatory demyelinating
polyneuropathy
(D) language impairment
(C) mononeuropathy multiplex
(E) preservation of manipulation of acquired
knowledge (D) autonomic neuropathy
(E) progressive polyradiculopathy
4. A 35-year-old HIV-positive man consults the neu-
rologist because of increasing difficulty walking
over the last 6 months. He reports progressively

283
284 8: Infections of the Nervous System

6. Which of the following drugs causes a pure sen- (E) start empirical treatment for toxoplasmo-
sory neuropathy as a neurotoxic side effect? sis combined with corticosteroids
(A) Stavudine
9. Which of the following CSF features is com-
(B) Isoniazide monly found in cryptococcal meningitis in HIV
(C) Vinca alkaloids patients?
(D) Ethambutol
(A) Decreased opening pressure
(E) Dapsone
(B) Hypoglycorrhachia
7. Which of the following statements is true of pro- (C) Cryptococcal antigen
gressive multifocal leukoencephalopathy? (D) Increased protein
(E) Visualization of Cryptococcus neoformans
(A) SV 40 virus is the most frequent cause of
organisms on India ink smear
the disease.
(B) Approximately 50% of patients with 10. The most commonly affected cranial nerve in
AIDS will develop progressive multifocal tuberculosis in non-HIV-infected patients is the
leukoencephalopathy during the course
of their disease. (A) oculomotor nerve
(C) The parietooccipital region is the site of (B) abducens nerve
predilection for progressive multifocal (C) trochlear nerve
leukoencephalopathy. (D) facial nerve
(D) Gait disturbance is the most frequent clin- (E) vestibulocochlear nerve
ical manifestation of progressive multifo-
cal leukoencephalopathy. 11. A 24-year-old man with a history of HIV infec-
(E) Polymerase chain reaction (PCR) testing tion consults the neurologist because of a chronic
of the CSF has low specificity for the headache. Neurological examination is normal.
diagnosis of progressive multifocal Computed tomography (CT) scan of the head
leukoencephalopathy. with contrast shows a ring-enhancing lesion in
the left parietal area, which is confirmed by mag-
8. A 31-year-old right-handed man with a history of netic resonance imaging (MRI) of the head with
HIV infection developed headaches and right- gadolinium enhancement. The most appropri-
sided weakness progressing over 1 week. ate diagnosis or therapeutic approach is to
Physical examination demonstrates mild right
(A) proceed to a biopsy to establish the diag-
hemiparesis with increased deep tendon reflexes,
nosis
right Babinski sign, and a fever of 38.5C. MRI of
the head with contrast shows a single ring- (B) start the patient on corticosteroids
enhanced lesion in the left basal ganglia and (C) start empirical antibiotic therapy
internal capsule surrounded by edema. The (D) start empirical antitoxoplasmosis treatment
lesion is hypoactive on thallium-201 single pho- (E) start the patient on intravenous acyclovir
ton emission computed tomography (SPECT).
The patients CD4 lymphocyte count is 10/L. 12. The most frequent presenting symptom of pri-
Toxoplasmsa gondii serology is positive. The most mary central nervous system (CNS) lymphoma
appropriate approach to the treatment of this in HIV patients is
patient is to
(A) impaired cognition
(A) start corticosteroids alone (B) seizures
(B) schedule a stereotactic biopsy (C) hemiparesis
(C) start radiation therapy (D) aphasia
(D) start antitoxoplasmosis therapy without (E) cranial nerve palsy
corticosteroids
Questions: 620 285

13. The most common cause of an intracranial (D) Haemophilus influenzae


space-occupying mass with contrast enhance- (E) Neisseria meningitides
ment in AIDS patients is
(A) primary CNS lymphoma 18. Which of the following is true of leprosy?
(B) bacterial abscess (A) Mycobacterium leprae has a preference for
(C) fungal abscess body parts with higher temperature than
(D) toxoplasmosis the core body.
(E) metastatic brain tumor (B) Loss of light touch is the first manifesta-
tion of sensory impairment.
14. The most frequent abnormal finding on retinal (C) Peripheral nerve thickening results from
examination in AIDS patients is bacterial multiplication within the neu-
ron.
(A) cotton-wool spots
(D) The median nerve is the most frequently
(B) cytomegalovirus (CMV) retinitis affected peripheral nerve in tuberculoid
(C) optic atrophy leprosy.
(D) swollen optic nerve (E) Impaired cell-mediated immunity causes
(E) toxoplasmal retinitis lepromatous leprosy.

15. The abnormal findings most frequently noted in 19. Which of the following is true of botulism?
eastern equine encephalitis are located in the
(A) Toxin production by Clostridium botu-
(A) brainstem linum colonizing the gut is the most
(B) cerebellum frequent cause of adult botulism.
(C) periventricular white matter (B) Botulism toxin blocks acetylcholine
(D) basal ganglia receptors.
(E) meninges (C) The N-terminal of the heavy chain of bot-
ulinum toxin governs the internalization
16. Which of the following statements is true about of the toxin into the motor neuron.
subacute sclerosing panencephalitis? (D) Cognition is usually affected in the early
stage of the disease.
(A) It is a slow CNS infection with herpes
(E) Sensory examination is typically altered.
simplex virus.
(B) The onset of the disease is characterized 20. Which of the following is true of tetanus?
by a rapid onset of dementia.
(C) Myoclonic jerks are seen in the second (A) Tetanospasmin inhibits glutamate release
stage of the disease. in the spinal cord.
(D) Brain biopsy, if performed in the early (B) Tetanus toxin travels to the anterior horn
stage of the disease, shows a mild inflam- cells by retrograde axonal transport.
mation limited to the cortex area. (C) The heavy chain of tetanus toxin blocks
(E) The electroencephalogram (EEG) normal- exocytosis.
izes in the myoclonic phase. (D) Spasms are caused by sympathetic
blockade.
17. The most frequent cause of bacterial meningitis (E) Autonomic dysfunction rarely compli-
among children aged less than 3 months is cates the course of the disease.
(A) group B Streptococcus
(B) Listeria monocytogenes
(C) Streptococcus pneumoniae
286 8: Infections of the Nervous System

21. The most common cause of viral encephalitis in (A) Streptococcus pneumoniae
the United States is (B) Haemophilus influenzae type b
(A) arbovirus (C) Staphylococcus aureus
(B) herpesvirus (D) Acinetobacter calcoaceticus
(C) measles virus (E) Listeria monocytogenes
(D) mumps virus
27. The most common cause of bacterial meningitis
(E) enterovirus
in an immunocompetent adult is
22. Repetitive sharp wave complexes over the tem- (A) Streptococcus pneumoniae
poral lobe are commonly seen in (B) Haemophilus influenzae type b
(A) HIV encephalitis (C) Staphylococcus aureus
(B) subacute sclerosing panencephalitis (D) Acinetobacter calcoaceticus
(C) herpes simplex encephalitis (E) Listeria monocytogenes
(D) CreutzfeldtJakob disease
28. The most likely cause of bacterial meningitis in
(E) renal failure
a 40-year-old man 2 days after undergoing a
ventriculostomy is
23. The most frequent neurological manifestation
of poliovirus infection is (A) Streptococcus pneumoniae
(A) transverse myelitis (B) Haemophilus influenzae type b
(B) paralytic illness (C) Staphylococcus aureus
(C) cerebellitis (D) Acinetobacter calcoaceticus
(D) aseptic meningitis (E) Listeria monocytogenes
(E) seizures
29. What is the most appropriate antibiotic therapy
for the treatment of hospital-acquired meningi-
24. The most likely cause of bacterial meningitis in a
tis in a 55-year-old man with severe neutrope-
40-year-old man with a history of splenectomy is
nia?
(A) Streptococcus pneumoniae
(A) Ceftriaxone 2 gr every 12 hours
(B) Haemophilus influenzae type b
(B) Vancomycin 0.5 gr every 6 hours and
(C) Staphylococcus aureus ampicillin 2 gr every 4 hours
(D) Acinetobacter calcoaceticus (C) Cefotaxime 2 gr every 4 hours and
(E) Listeria monocytogenes vancomycin 0.5 gr every 6 hours
(D) Vancomycin 0.5 gr every 6 hours and
25. The most likely cause of bacterial meningitis in ceftazidine 2 gr every 8 hours
a 56-year-old man with a history of heavy
(E) Ampicillin 2 gr every 4 hours and
ethanol abuse is
gentamicin 6 mg/kg per day
(A) Streptococcus pneumoniae
(B) Haemophilus influenzae type b 30. The presence of owl eye intranuclear inclu-
(C) Staphylococcus aureus sions, with focal necrosis in the basal ganglia
and thalamus is highly suggestive of
(D) Acinetobacter calcoaceticus
(E) Listeria monocytogenes (A) herpes encephalitis
(B) CMV encephalitis
26. The most likely cause of bacterial meningitis in (C) EpsteinBarr meningoencephalitis
a 60-year-old woman with a history of heart (D) varicella zoster encephalitis
transplantation is
(E) La Crosse virus encephalitis
Questions: 2139 287

31. The most frequent cause of viral meningitis is (D) anthrax


(A) enterovirus (E) Venezuelan equine encephalitis
(B) CMV virus
37. Which of the following statements is true about
(C) herpesvirus CreutzfeldtJakob disease?
(D) arbovirus
(A) The cerebrospinal fluid (CSF) opening
(E) West Nile virus
pressure is usually high.
32. The most common cause of epidural abscess in (B) CSF protein 14-3-3 has a high sensitivity
immunocompetent patients is and specificity for patients with the
diagnosis of progressive dementia.
(A) Klebsiella pneumoniae (C) The CSF protein level is higher than
(B) Staphylococcus 100 mg/dL.
(C) Streptococcus (D) Brain MRI may show hypersignal in the
(D) fungal infection internal capsule in 80% of cases.
(E) Mycobacterium tuberculosis (E) Early in the course of the disease, the
EEG usually shows triphasic synchro-
33. The most frequent sign of neurocysticercosis is nous sharp-wave complexes.
(A) seizure
38. Which of the following statements is true of vari-
(B) headache cella zoster virus?
(C) visual disturbance
(A) The virus can be cultured from human
(D) hemiplegia
ganglia during the latent period.
(E) ataxia
(B) Neurons are the primary site of the latent
virus.
34. The most common neurological complication of
chronic Chagas infection is (C) The incidence of recurrent zoster is
around 30%.
(A) seizure (D) The abducens cranial nerve is the cranial
(B) irritability nerve most commonly affected by vari-
(C) stupor cella zoster virus.
(D) dementia (E) Facial RamsayHunt syndrome has a bet-
(E) cardioembolic stroke ter prognosis for recovery than Bells
palsy.
35. Which of the following HIV complications is the
least affected by antiretroviral therapy? 39. Which of the following is true of fatal insomnia?
(A) Progressive multifocal leukoencephalopa- (A) Cognitive function is affected early in the
thy course of the disease.
(B) AIDS dementia (B) EEG typically shows periodic discharges.
(C) CNS toxoplasmosis (C) The absence of spongiform changes on
(D) Cryptococcal meningitis neuropathology excludes the diagnosis.
(E) CMV polyradiculopathy (D) PET scan shows decreased blood flow in
the thalamus early in the course of the
36. Hemorrhagic meningitis is caused by the organ- disease.
ism leading to (E) Sleep studies are not necessary for the
diagnosis of fatal insomnia as the disease
(A) botulism
is always clinically obvious.
(B) brucellosis
(C) Q fever
288 8: Infections of the Nervous System

40. Which of the following is true of Japanese bellar ataxia and bilateral pyramidal syndrome.
encephalitis? Over the next 3 months, the patient progressed
to akinetic mutism. She died 5 months after the
(A) The disease is limited to the pediatric
onset of symptoms. The most likely diagnosis is
population.
(B) Ninety percent of patients affected by (A) CreutzfeldtJakob disease
Japanese encephalitis will recover fully. (B) GSS disease
(C) The thalamus and basal ganglia are the (C) Kuru
sites of intense inflammation in affected (D) variant CreutzfeldtJakob disease
patients. (E) none of the above
(D) Serologic tests in Japanese encephalitis
have a low sensitivity and specificity. 44. A 40-year-old man died after 4 years of pro-
(E) A progressive ascending paralysis is the gressive cerebellar ataxia, pyramidal syndrome,
most frequent form of Japanese and dementia. Postmortem examination showed
encephalitis seen in children. an abundant prion protein (PrP), amyloid
plaques in cerebral and cerebellar cortexes,
41. A 40-year-old man developed progressive wors- as well as multicentric prionprotein amyloid
ening of anxiety and depressed mood, with plaques. The most likely diagnosis is
auditory and visual hallucinations. Neurological
(A) CreutzfeldtJakob disease
examination demonstrated mild bilateral cere-
bellar syndrome. His clinical status deteriorated (B) GSS disease
progressively over the next 13 months, with (C) Kuru
decreased cognitive function and chorea fol- (D) variant CreutzfeldtJakob disease
lowed by myoclonus. The most likely diagnosis (E) none of the above
is
(A) CreutzfeldtJakob diseas 45. The most common cause of brain abscess in
immunocompetent patients is
(B) GerstmannStrausslerScheinker (GSS)
disease (A) anaerobic bacteria
(C) Kuru (B) Staphylococcus aureus
(D) variant CreutzfeldtJakob disease (C) fungi
(E) none of the above (D) Pseudomonas aeruginosa
(E) protozoa
42. A 20-year-old man died after 12 months of pro-
gressive cerebellar ataxia without alteration of 46. The gold standard method for the diagnosis of
the cognitive function until the late stage of the herpes encephalitis is
disease, when the patient was obtunded. The
most likely diagnosis is (A) brain MRI
(B) brain MRI spectroscopy
(A) CreutzfeldtJakob disease
(C) brain biopsy
(B) GSS disease
(D) polymerase chain reaction (PCR) testing
(C) Kuru of the CSF for herpes DNA
(D) variant CreutzfeldtJakob disease (E) culture of the herpesvirus in the CSF
(E) none of the above
47. The most common cause of spinal epidural
43. A 60-year-old woman developed a progressive abscess is
onset of fatigue, insomnia, and ill-defined pain
over several weeks. This was followed by pro- (A) Staphylococcus aureus
gressive mental deterioration and myoclonus. (B) Candida albicans
Neurological examination demonstrated cere- (C) Salmonella
Questions: 4050 289

(D) Nocardia asteroides (C) echovirus


(E) Escherichia coli (D) herpes simplex virus
(E) mumps virus
48. Which of the following is a classic CSF abnor-
mality in tuberculous meningitis? 50. The most common neurological complication of
(A) Hemorrhagic CSF HIV infection is
(B) Low CSF opening pressure (A) seizure
(C) Lymphocytic pleocytosis (B) CNS toxoplasmosis
(D) Elevated CSF glucose concentration (C) myelopathy
(E) Normal CSF protein concentration (D) distal sensory polyneuropathy
(E) progressive multifocal
49. The most frequent cause of viral meningitis is leukoencephalopathy
(A) La Crosse virus
(B) West Nile virus
Answers and Explanations

1. (E) Cerebral atrophy involving the frontotem- Stage 2 (moderate severity): ability to per-
poral areas is a common finding in patients with form basic activities of self-care but cannot
HIV dementia. It does not correlate with the work or maintain the more demanding
severity of the dementia. Multiple microglial aspects of daily life. The patient may ambu-
nodules containing macrophages, lymphocytes, late independently.
and microglia may be seen in HIV dementia. Stage 3 (severe disability): major intellectual
Scattered in the gray and white matter of the or motor incapacity.
brain, they are more common in the white mat- Stage 4: nearly vegetative status. (Price and
ter, subcortical gray of the thalamus, basal gan- Brew, 10791083)
glia, and brainstem. They are not specific to HIV
dementia and do not correlate with its severity. 3. (B) Neuropsychological testing adds to the neu-
Other neuropathological findings include neu- rological evaluation of suspected HIV dementia
ronal loss, dendritic changes, myelin pallor by virtue of being sensitive to mild or early
(which corresponds to changes of the blood symptoms of HIV-related cognitive impairment.
brain barrier), and multinucleated giant cells. In addition to quantifying the severity of any
The presence of these cells correlates with the cognitive symptoms, it can also provide infor-
degree of dementia and the detection of HIV-1 mation regarding the overall pattern of cognitive
DNA. However, the intensity of macrophage impairment. Attention, calculation, and lan-
activation does appear to correlate best with the guage are not usually affected in HIV dementia
severity of dementia. (Glass, 22302237; Glass, in its initial stage, fitting with the subcortical
755762) pattern of involvement. Impaired memory (ver-
bal and nonverbal), impaired manipulation of
2. (C) In 1988, Price and colleagues developed the acquired knowledge, memory loss selective for
following clinical staging of the severity of HIV- impaired retrieval, and deficits in psychomotor
associated dementia: speed are characteristic of HIV dementia and
are typically more severe than deficits in other
Stage 0: normal mental and motor function.
cognitive domains. (McArthur, 129150)
Stage 0.5 (equivocal subclinical): absent, min-
imal, or equivocal symptoms without impair-
4. (C) The patient described in the vignette demon-
ment of work or capacity to perform activities
strates clinical evidence of an insidiously pro-
of daily life. Mild signs such as snout response,
gressive thoracic or high lumbar myelopathy
slowed ocular, or extremity movements may
because of the spastic paraparesis, bladder dys-
be present. Gait and strength are normal.
function, and corticospinal and posterior column
Stage 1 (mild severity): ability to perform all
signs without a sensory level and normal spinal
but the most demanding activities of daily
cord MRI. The differential diagnosis includes:
life with unequivocal evidence of functional
intellectual or motor impairment. The patient CMV myelopathy: a rare cause of myelopathy
can walk without assistance. in HIV patients. It usually presents as an acute

290
Answers: 18 291

or subacute radiculomyelitis involving the infection, injury from cytokine effects, metabolic
lower lumbar sacral roots and cauda equina. abnormalities, or the medications used to control
It is characterized clinically by the acute or the HIV infection. (Simpson, 769785)
subacute development of a flaccid paraplegia
and incontinence. The patient in this case did 6. (A) Toxicity from drugs used to treat HIV infec-
not have signs of radiculopathy and had slow tion is among the major causes of peripheral neu-
progression of his symptoms, which make the ropathy in these patients. Didanosine, stavudine,
diagnosis of CMV myelopathy unlikely. and zalcitabine are antiretroviral drugs that
HIV myelitis may appear as an acute or sub- may cause a sensory neuropathy. Isoniazid and
acute complication of AIDS. A rare complica- ethambutol cause sensorimotor neuropathy.
tion of AIDS, it is characterized by features of Vinca alkaloids and dapsone cause a mixed
transverse myelitis with CSF pleocytosis. It motor more than a sensory neuropathy. (Simpson,
is unlikely to be the diagnosis in this clinical 769785)
case considering the clinical picture, espe-
cially the CSF results. 7. (C) Progressive multifocal leukoencephalopa-
AIDS-associated vacuolar myelopathy is a thy was initially described on the basis of its dis-
slowly progressive disease of the spinal cord. tinctive neuropathological features of demyeli-
It is prevalent in more than 30% in pathologi- nation, giant astrocytes, and oligodendrocytes
cal series, although only a minority of these with abnormal nuclei. Subsequently, a papo-
patients are symptomatic. Pathological features vavirus referred to as JC virus was identified in
of the disease include the presence of intra- affected oligodendrocytes and astrocytes.
myelinic and periaxonal vacuoles in the lat- JC virus infection typically remains latent until
eral and posterior columns of the spinal cord. there is impairment of cellular immunity.
It appears late in the course of the illness. The Approximately 5% of all AIDS patients will
patient may report erectile or bladder dys- develop progressive multifocal leukoencephalo-
function and mild paraparesis in the early pathy. Its clinical manifestations are dependent
stage of the disease. Neurological examination on the location of the infection. Hemiparesis is
typically demonstrates a spastic asymmetric the most common clinical manifestation of pro-
paraparesis, increased deep tendon reflexes in gressive multifocal leukoencephalopathy in
the lower extremities, hyperreflexia, and the AIDS patients. Other common clinical manifes-
absence of a sensory level. MRI may be normal tations are gait disturbance, speech and language
or may show atrophy of the thoracic and cer- disorders, cognitive dysfunction, and cortical
vical spinal cord. CSF examination may show blindness. MRI of the head is the most sensitive
mild pleocytosis or increased protein level. technique to show the demyelinating lesion
Somatosensory evoked potentials may show involving the white matter in the subcortical U
increased central conduction time. The patient fibers with a predilection for the parietooccipital
described in the vignette does have signs of areas. The lesions are not typically enhancing,
AIDS-associated vacuolar myelopathy. although up to 9% may have peripheral
Other causes of lower extremity weakness in enhancement around them. PCR of the CSF for
HIV patientssuch as syphilis, tuberculosis, the diagnosis of progressive multifocal leukoen-
cryptococcosis, aspergillosis, vitamin B12 cephalopathy has high sensitivity and specificity
or folate deficiency, and lymphomaare that reaches 100%. The prognosis of progressive
unlikely to be the diagnosis in this case con- multifocal leukoencephalopathy is ominous.
sidering the clinical features and the results of Death occurs between 1 and 18 months after the
ancillary tests. (DiRocco, 151155) onset of symptoms. (Berger, 5968)

5. (A) Distal symmetric polyneuropathy is the 8. (D) In this vignette, the differential diagnosis is of
most common form of peripheral neuropathy a single brain lesion in an HIV patient with posi-
in HIV-infected patients. The proposed mecha- tive toxoplasmosis serology. The main differential
nisms of this neuropathy include direct HIV diagnoses are toxoplasmosis encephalitis and
292 8: Infections of the Nervous System

primary CNS lymphoma. Although this patients raise the possibility of CNS toxoplasmosis. The
head MRI does not present the classic multiple diagnostic workup should include a toxoplas-
space-occupying enhancing lesions of toxoplas- mosis antibody titer of the serum and the CSF;
mosis encephalitis, the combination of a positive also, if possible, a thallium SPECT study should
serology of toxoplasmosis, a CD4 cell count of be done to rule out a hyperactive lesion (more
less than 200/L, and decreased thallium-201 suggestive of lymphoma). The most appropriate
SPECT imaging that is more suggestive of toxo- therapeutic approach is to start the patient on
plasmosis encephalitis than lymphoma. The rec- antitoxoplasmosis therapy for 2 weeks and
ommendations of the American Academy of thereafter assess the patient clinically and radi-
Neurology Qualified Standards Subcommittee are ologically. In case of improvement, the patient
to institute empirical antibiotic therapy for the should be maintained on prophylactic antibi-
presumptive toxoplasmosis and obtain a follow- otics for life after completion of the induction
up imaging study. If the patient shows a clinical therapy. If there is no improvement, stereotactic
and radiological response to this treatment, he or biopsy should be considered. The use of corti-
she can be presumed to have CNS toxoplasmo- costeroids has not been shown to be beneficial
sis. The patient should be maintained on sup- for the treatment of CNS toxoplasmosis and may
pressive antibiotics for life after finishing the delay the diagnosis of primary CNS lymphoma.
induction therapy. If the patient does not show It should be used with caution and rapidly
improvement on antibiotics after 2 weeks, he or tapered. (Luft, 211222)
she is eligible for stereotactic biopsy. The empir-
ical use of corticosteroids is not recommended 12. (A) The most frequent presenting symptom of
for toxoplasmosis treatment. The addition of cor- central nervous lymphoma is cognitive or men-
ticosteroids has not been shown to improve neu- tal status impairment, which is seen in 60% of
rological outcome and may interfere with the cases. Hemiparesis or aphasia is seen in 35% of
assessment of a possible primary CNS lymphoma. cases. Seizures are seen in 15% of cases at pres-
(Anonymous, 2126) entation. Cranial nerve palsy is seen in only 10%
of cases at presentation. (Baumgartner, 206211)
9. (A) CSF abnormalities in cryptococcal menin-
gitis in HIV patients may be subtle. The opening 13. (D) Toxoplasmosis is the most common cause of
pressure is elevated in two thirds of cases and an intracranial space-occupying mass in AIDS
may exceed 400 mm H2O. Mild pleocytosis is patients, followed by primary CNS lymphoma.
seen in 6% to 30% of cases. Hypoglycorrhachia Toxoplasmosis lesions are most frequently
is found in 8% to 76% of cases. Increased protein found in the cerebral cortex, basal ganglia, and
is observed in 35% to 70% of cases. Visualization graywhite matter junction. (Chinn, 694654)
of organisms on India ink smear is seen in 72%
to 94% of cases. Successful culture of C. neofor- 14. (A) The most frequent retinal lesion in AIDS
mans in and visualization of cryptococcal anti- patients is cotton-wool spots, seen in 75% of these
gen in the CSF may be seen in up to 100% of cases. It is a buildup of exoplasmic material at the
cases. (Churck, 794799) site of a nerve fiber layer infarct. When these spots
are coupled with hemorrhage or capillary abnor-
10. (B) Cranial nerves are affected in 15% to 40% of malities, it is called AIDS retinopathy. Axonal loss
immunocompetent patients with tuberculosis. in the optic nerve has been estimated at 40% in
The abducens nerve is the most commonly patients dying from AIDS in the absence of oppor-
affected, followed in descending order by the tunistic infection of the retina. CMV retinitis may
oculomotor, trochlear, facial, optic, vestibulo- affect 25% to 35% of patients with AIDS. Toxo-
cochlear, vagus, accessory, and hypoglossal plasmosis retinitis, optic atrophy, and optic edema
nerves. (Lincoln, 807823) are less frequently seen. (Gagliuso, 6386)

11. (D) The presence of a ring-enhancing mass on 15. (D) Eastern equine encephalitis is a life-threat-
CT scan of the head in an HIV patient should ening mosquito-borne arboviral infection found
Answers: 919 293

principally along the East and Gulf Coasts of the disorganized, with high amplitude and random
United States. Cases have occurred sporadically dysrhythmic slowing. (Garg, 6370)
and in small epidemics. Diagnosing eastern
equine encephalitis is difficult because its symp- 17. (A) The likelihood of infection with a specific
toms are nonspecific and confirmation requires pathogen in cases of bacterial meningitis is
either specific serologic findings or the isolation related in large part to the age of the patient.
of the virus in CSF or brain tissue. Neuroradio- Group B Streptococcus is the most frequent
graphic abnormalities are common and best pathogen among immunocompetent patients
visualized by MRI. The abnormal findings in- aged less than 3 months. Neisseria meningitidis is
clude focal lesions in the basal ganglia (found in the predominant pathogen among children aged
71% of patients on MRI and in 56% on CT), thal- 2 to 18 years, and Streptococcus pneumoniae is
ami (found in 71% on MRI and in 25% on CT), most likely among adults. (Schuchat, 970976)
and brain stem (found in 43% on MRI and in 9%
on CT). Periventricular white matter changes 18. (E) Leprosy, a primary peripheral nerve and
and cortical lesions as well as meningeal en- skin infection, is caused by the acid-fast bac-
hancement are less common. (Deresiewicz et al., terium Mycobacterium leprae. M. leprae has a pref-
18671874) erence for parts of the body with a temperature
of 7C to 10C lower than the core body
16. (C) Subacute sclerosing panencephalitis (SSPE) temperature. It has a long incubation period,
is a slowly progressing CNS infection caused between 6 months and 40 years. Sensory impair-
by measles virus that results in progressive ment proceeds in a predictable sequence, with
inflammation and sclerosis of the brain. The inci- loss of temperature sensation first, followed by
dence of the disease in the United States fell pain and then touch. Vibration and propriocep-
from 0.61 case per million for persons under the tion are spared. Impaired cell-mediated immu-
age of 20 years to 0.06 case in 1980, most likely nity causes lepromatous leprosy, a more dis-
related to measles vaccination. Patients with seminated infection than the tuberculoid leprosy
subacute sclerosing panencephalitis generally of patients with conserved cellular immunity.
have a history of typical measles with full recov- Nerve damage results from bacterial multipli-
ery. The symptoms of SSPE follow on average by cation within Schwann cells or granulomatous
about 7 years. The clinical course of the disease damage to the perineurium. The ulnar nerve is
is divided arbitrarily into four stages. Its onset is the most frequently affected in cases of tuber-
usually insidious, marked by subtle changes in culoid leprosy. (Bradley, 13321334)
behavior and deterioration of school or work,
followed by a frank dementia. The appearance 19. (C) Botulism is caused by the blockade of periph-
of massive repetitive myoclonic jerks marks the eral cholinergic transmission by a neurotoxin
onset of stage 2 of the disease. Cerebellar ataxia, (secreted by Clostridium botulinum). It can be
dystonia, retinopathy, and optic atrophy may acquired either by contaminated food or by an
appear. As the disease progresses, the myoclonic infected wound spreading the toxin in the blood-
jerks tend to disappear, while the dementia pro- stream. Gut colonization by C. botulinum causes
gresses to stupor and coma. Death occurs after infantile botulism and less commonly adult
a mean of 18 months. Brain biopsy in the early botulism. Botulinum toxin blocks acetylcholine
stages may show mild inflammation of the release from the presynaptic membrane, leading
meninges and a panencephalitis involving cor- to the paralytic and autonomic symptoms.
tical and subcortical gray as well as white mat- Botulinum toxin is formed by heavy and light
ter. EEG is useful in supporting the diagnosis of chains. The C-terminal region of the heavy chain
SSPE. Early in the course of the disease, it may binds tightly and specifically to the presynaptic
be normal or show moderate nonspecific slow- membrane, whereas the N-terminal domain gov-
ing. In the myoclonic phase, the EEG shows erns internalization of the toxin into the motor
suppression-burst episodes. Later in the course neuron, which protects the toxin from neutralizing
of the disease, the EEG becomes increasingly antibodies. After translocation into the cytosol,
294 8: Infections of the Nervous System

the liberated light chain (a zinc endopeptidase) complexes occur in subacute sclerosing panen-
targets various protein-mediating exocytoses, cephalitis. Triphasic waves at higher periodic
causing irreversible blockade at peripheral frequency are seen in CreutzfeldtJakob disease.
cholinergic synapses. The early symptoms of (Bradley, 1358)
botulism include diplopia, ptosis, dysarthria,
and dysphagia. Respiratory muscles as well as 23. (D) Poliovirus one of the most virulent mem-
extraocular and limb muscles are affected sym- bers of the enterovirus group, is the causative
metrically. Cognitive function is conserved unless agent of acute anterior poliomyelitis. It has a
there are metabolic changes from respiratory fail- tropism for motor neurons of the spinal cord
ure. Sensation is typically normal. Reflexes are and brainstem. The most frequent clinical man-
decreased or absent. (Bradley, 1384) ifestation of poliovirus is aseptic meningitis
(seen in 8% of cases). Paralytic illness is seen in
20. (B) Clostridium tetani secretes tetanospasmin, a 1% of all cases. Other less frequent clinical man-
neurotoxin that inhibits the release of GABA and ifestations include cerebellitis, transverse myelitis,
glycine (which are inhibitory neurotransmitters) and facial paresis. (Bradley, 19911994)
in the brainstem and spinal cord. Tetanus neuro-
toxin is formed by heavy and light chains linked 24. (B) Haemophilus influenzae type b causes menin-
together by a disulfide bond. The light chain, a gitis in immunocompromised patients (e.g.,
zinc endopeptidase, is responsible for blocking postsplenectomy and in chronic lung diseases).
exocytosis. Tetanus neurotoxin travels to the ante- (Roos, 12; 2002)
rior horn cells of the spinal cord by retrograde
axonal transport, penetrates the intrasynaptic 25. (D) Acinetobacter calcoaceticus and other gram-
space, and enters inhibitory neurons. Impaired negative bacilli cause meningitis in alcoholic
exocytosis in these spinal inhibitory neurons patients. (Roos, 12; 2002)
and in the intermediolateral column of the
spinal cord causes muscle contractions and auto- 26. (E) Listeria monocytogenes causes meningitis in
nomic dysfunction, respectively. The cardinal immunosuppressed patients (e.g., after organ
clinical features of tetanus include muscle rigid- transplantation). (Roos, 12; 2002)
ity and spasms, which may be triggered by a
sensory stimulus, movement, or emotion. Auto- 27. (A) The most common causative organisms of
nomic dysfunction is frequent in tetanus and community-acquired bacterial meningitis are
includes fever, tachycardia, hypertension, and Streptococcus pneumoniae and Neisseria meningi-
other signs of sympathetic irritation. (Bradley, tidis. (Roos, 12; 2002)
13401342)
28. (C) Staphylococcus aureus causes meningitis fol-
21. (B) The most common cause of focal viral lowing invasive neurosurgical procedures such
encephalitis in the United States is herpes sim- as ventriculostomy. (Roos, 12; 2002)
plex virus type 1. Other causes of viral encepha-
litis are varicella zoster virus (VZV), enterovirus, 29. (D) Bacterial meningitis is a medical emergency.
mumps, measles, and Lacrosse virus. (Bradley, The goal is to begin antibiotic therapy within
13581359) 60 minutes of a patients arrival in the emer-
gency room. Empiric antimicrobial therapy is
22. (C) The EEG in acute viral encephalitis may initiated in patients with suspected bacterial
show patterns suggesting a specific diagnosis. meningitis before the results of CSF Grams stain
Repetitive sharp-wave complexes over the tem- and culture are known. S. pneumoniae and N.
poral lobes or periodic lateralized epileptiform meningitidis are the most common etiological
discharges are recorded in herpes simplex type organisms of community-acquired bacterial
1 encephalitis and rare cases of infectious meningitis. Because resistant S. pneumonia
mononucleosis encephalitis. Periodic slow-wave appeared after the development of penicillin,
Answers: 2033 295

empiric therapy of community-acquired bacter- 31. (A) Enteroviruses are responsible for the over-
ial meningitis should include a third-generation whelming majority (85% to 95%) of cases of
cephalosporin (e.g., ceftriaxone or cefotaxime) or acute viral meningitis. Less commonly, entero-
a fourth-generation cephalosporin (cefepime) viruses produce encephalitis or polio-like flaccid
and vancomycin. Ampicillin and gentamicin paralysis. Most infections occur in the summer
should be added to the empiric regimen for cov- and early fall (May through October), although
erage of Listeria monocytogenes in individuals sporadic cases occur all year round. Children
with impaired cell-mediated immunity due to are more frequently infected than adults,
chronic illness, organ transplantation, preg- although infections in older individuals tend to
nancy, AIDS, malignancies, or immunosuppres- be more severe. (Rotbart, 971981)
sive therapy. In hospital-acquired meningitis
and particularly meningitis following neuro- 32. (B) Staphylococcus aureus has been traditionally
surgical procedures, staphylococci and gram- and remains the main pathogen in epidural
negative organisms (including P. aeruginosa), are abscess, accounting for over 60% of the isolates.
the most common etiological organisms. In these Other gram-positive pathogens that may cause
patients, empiric therapy should include a com- epidural abscess less frequently include Staphy-
bination of vancomycin and ceftazidime. lococcus epidermidis, streptococci (alpha and beta
Ceftazidime should be substituted for ceftriax- hemolytic), and anaerobes. Gram-negative
one or cefotaxime in neurosurgical patients and pathogens are increasing in frequency (second to
in neutropenic patients, as P. aeruginosa may be Staphylococcus), perhaps reflecting an increasing
the meningeal pathogen, and ceftazidime is the proportion of iatrogenic infections. Myco-
only cephalosporin with sufficient activity bacterium tuberculosis and pathogenic fungi also
against P. aeruginosa in the CNS. (Roos, 710; 2005) account for a significant percentage of cases.
(Bradley, 12381329)
30. (B) CMV meningoencephalitis is largely a dis-
ease of congenitally infected newborns and 33. (A) Neurocysticercosis infection is pleomorphic
immunocompromised individuals, including owing to individual differences in the number
organ transplant recipients and those with and location of lesions and in the severity of the
advanced HIV infection. In immunocompromised hosts immune response to the parasites. Seizures
individuals, infection results predominantly from are the most common clinical manifestation of the
reactivation of latent virus. It is generally associ- disease, occurring in more than 70% of cases.
ated with evidence of CMV infection in other Indeed, in endemic regions, the presence of adult-
organs, including the eye, lungs, gastrointestinal onset epilepsy is highly suggestive of neurocys-
tract, and other organs. In individuals infected ticercosis. Focal neurological signs (pyramidal
with AIDS, CMV can produce multifocal CNS tract signs, sensory deficits, cerebellar ataxia, signs
disease with involvement of the spinal cord, nerve of brainstem dysfunction, and involuntary move-
roots, ventricular and subependymal regions, and ments) occur in 20% to 30% of patients with neu-
both gray and white matter. Infection often shows rocysticercosis. These manifestations usually fol-
a predilection for deep gray structures, including low a subacute or chronic course, making the
basal ganglia and thalamus, producing regions differential diagnosis with neoplasms or other
of focal necrosis and hemorrhage. Microglial nod- infections of the CNS difficult on clinical grounds.
ules (dense focal aggregates of microglial cells However, focal signs may occur abruptly in
and macrophages) and Cowdry type A intranu- patients who develop a cerebral infarct as a com-
clear inclusions are characteristic of CMV, plication of cysticercotic angiitis. Some patients
although both are seen in a variety of other infec- present with intracranial hypertension that may
tions. Owls eye inclusions (large basophilic be associated with seizures, focal neurological
intranuclear inclusions separated from the nuclear signs, or intellectual deterioration. Hydrocephalus
membrane by a thin halo) are virtually diagnostic is the most common cause of this syndrome. In
of CMV infection. (McCutchan, 747754) these cases, clinical manifestations have a
296 8: Infections of the Nervous System

subacute onset and a slowly progressive course is suggested by the study as a possible explana-
that may be punctuated by episodes of sudden tion for the modest impact of HAART on ADC.
loss of consciousness related to movements of the (Dore, 12491253)
head when the cause of hydrocephalus is a fourth
ventricle cyst. Intracranial hypertension also 36. (D) Anthrax meningitis is a rare complication
occurs in patients with cysticercotic encephalitis, that can occur with any form of anthrax.
a severe form of the disease resulting from a mas- Meningeal symptoms are usually accompanied
sive cysticercal infection of the brain parenchyma by fever, myalgias and vomiting and less fre-
that induces an intense immune response from the quently by seizures or delirium. The CSF is typi-
host. (Bradley, 13921393) cally hemorrhagic and culture for Bacillus anthracis
is positive. Imaging of the CNS may reveal sub-
34. (E) Acute Chagas disease is usually character- arachnoid, intracerebral, or intraventricular hem-
ized by an inoculation chagoma in the orbital orrhage with leptomeningeal enhancement, while
region (Romaas sign) and mild constitutional pathology reveals hemorrhage of the lep-
symptoms. However, some children, HIV-infected tomeninges known as a cardinals cap.
individuals, and immunosuppressed patients Brucellosis is associated with low back pain
may develop severe encephalitis during the acute in 60% of infected people and can be associated
phase of the disease. Chagasic encephalitis is with vertebral osteomyelitis, intervertebral disk
characterized by irritability, stupor progressing infection, sacroiliac infection, or paravertebral
to coma, seizures, focal neurological signs (related abscess. The nervous system is involved in
to granuloma formation), and a mononuclear approximately 5% of people with brucellosis;
pleocytosis in the CSF. Most of these patients die the condition is then referred to as neurobrucel-
during the acute disease, and survivors are usu- losis. Acute neurobrucellosis typically includes
ally left with epilepsy and intellectual impair- symptoms of meningeal irritation and may be
ment. In patients with chronic infection, the most accompanied by seizures or coma. Chronic neu-
common neurologic complication is a cardioem- robrucellosis may include meningoencephalitis,
bolic stroke related to the development of cardiac demyelination, cranial neuropathy (most often
arrhythmias or ventricular aneurysms. The terri- involving the vestibulocochlear nerve), myelo-
tory of the middle cerebral artery is the most fre- radiculitis, or cerebral arteritis. The CSF exami-
quently affected, and infarcts may be located in nation in neurobrucellosis almost always reveals
the parietal, frontal, and temporal lobes or in the a moderate elevation of protein and a lympho-
basal ganglia. The actual prevalence of stroke in cytic pleocytosis.
Chagas disease is unknown; however, some stud- Severe headache is present in most people
ies have shown that between 9% and 36% of with symptomatic Q fever, but infection of the
patients with chagasic cardiomyopathy develop a nervous system is uncommon. Infection of the
cerebral infarct. (Bradley, 13921393) CNS usually manifests as an acute aseptic
meningitis and/or encephalitis and may be
35. (B) Potent antiretroviral therapy may be less accompanied by cranial nerve palsy, seizures,
effective in preventing HIV-1associated demen- mental status change, or coma. Examination of
tia than other HIV-1-related complications. A the CSF typically reveals a pleocytosis (usually
study from Australia compared the effect of lymphocytic, but it can also be neutrophilic) in
highly active antiretroviral therapy (HAART) 50% of patients as well as an elevated protein
against AIDS dementia complex (ADC) relative and negative bacterial culture in nearly all
to its effect on other initial AIDS-defining ill- patients. Neuroimaging may reveal hypodense
nesses (ADIs). The study demonstrated a pro- lesions in the subcortical white matter.
portional increase in ADC compared with other Botulism typically presents with bilateral
ADIs. A marked increase in the median CD4 cell cranial nerve palsies and symmetric descend-
count at ADC diagnosis has occurred since the ing paralysis. Symptoms often include blurred
introduction of HAART in Australia. Poor pen- vision, diplopia, dry mouth and throat, dys-
etration of antiretroviral medication in the CNS phagia, and dysphonia. Sensory deficits do not
Answers: 3440 297

occur. Deep tendon reflexes may be present or keratitis, which is a potential cause of blindness
absent. Respiratory failure is a common com- if not recognized and treated promptly. When
plication. CSF examination should be normal. the seventh cranial nerve is involved, there is
Venezuelan equine encephalitis (VEE) causes weakness of all facial muscles on one side, along
encephalitis in only 4% of children and less than with rash in the ipsilateral external ear (zoster
1% of adults, and typically occurs after a few oticus) or hard palate. Zoster oticus and periph-
days or a week of prodromal illness. CSF exam- eral facial weakness together constitute the
ination typically reveals a lymphocytic pleocy- RamsayHunt syndrome. Recovery from facial
tosis. (Bradley, 1346) weakness or paralysis is reported to be less com-
plete than in idiopathic Bells palsy. Palsies of
37. (B) Study of the CSF in CreutzfeldtJakob dis- other cranial nerves occur less frequently. (Gilden,
ease shows that it has a normal opening pres- 635645)
sure, does not have an increase in cells or abnor-
mal levels of immunoglobulin, and has a normal 39. (D) In its most characteristic presentation, fatal
or mildly elevated protein content. Partial insomnia causes an untreatable condition that
sequencing of these proteins has shown that sometimes lasts for weeks or months. The insom-
they matched a normal brain protein known as nia is followed by dysautonomia, ataxia, and vari-
14-3-3, and a rapid CSF immunoassay for the able pyramidal and extrapyramidal signs, with
protein has proved useful in the diagnosis. The relative sparing of cognitive function until late
sensitivity of the test is 96% and the specificity in the course. The dysautonomias may include
is 99%. Elevated levels of this protein are found episodic alterations in blood pressure, heart rate,
in the CSF of patients with viral encephalitis temperature, respiratory rate, and secretions. The
and during the first month after a cerebrovas- EEG shows diffuse slowing rather than periodic
cular accident. Early in the course of the dis- discharges. A sleep study is valuable to document
ease, the EEG may be normal or show nonspe- a shortening of total sleep time if insomnia is not
cific slowing. Later in the disease, periodic clinically obvious. Positron emission tomography
biphasic or triphasic synchronous sharp-wave (PET) shows a reduction in metabolic activity or
complexes are superimposed on a slow back- blood flow to the thalamus relatively early in the
ground rhythm in most patients, but these char- disease. The neuropathologic features of fatal
acteristic complexes may disappear as the insomnia include neuronal loss and astrogliosis
myoclonus subsides in the terminal phase of the within the thalamus, inferior olives and, to a lesser
disease. The results of brain imaging are usually degree, the cerebellum. The lack of spongiform
normal in the early stages of the disease. MRI changes does not exclude the diagnosis. The
may show hyperintense signals in the basal gan- protease-resistant prion protein (PrP) is detectable
glia on T2-weighted images. (Johnson, 1994) in the brains of affected patients but is usually
present only in small amounts and is often
38. (B) After it has produced chickenpox, VZV restricted to specific regions such as the thalamus
becomes latent in ganglia along the entire neu- and temporal lobe. (Mastrianni, 337352)
raxis. The virus cannot be cultured from human
ganglia, although viral DNA was detected by 40. (C) Japanese encephalitis (JE) virus is one of the
PCR in human trigeminal and thoracic ganglia. most important causes of viral encephalitis
Most studies indicate that neurons are the pri- worldwide, with an estimated 50,000 cases and
mary or possibly exclusive site of latent virus. 15,000 deaths annually. The clinical manifesta-
Other studies have detected the presence of the tions appear predominantly in children and
virus in the perineuronal satellite cells. The inci- young adults. Older adults seem to be affected
dence of recurrent zoster in immunocompetent when epidemics occur in new locations. The
patients is less than 5%. The trigeminal nerve is clinical features include a nonspecific prodro-
the most common cranial nerve affected by VZV. mal stage followed by headaches, nausea, vom-
When the ophthalmic division of the trigeminal iting, behavioral changes, altered state of con-
nerve is affected, it is frequently accompanied by sciousness, and often seizures. A dull mask-like
298 8: Infections of the Nervous System

face with wide, staring eyes, tremor, choreoa- ataxia. In sharp contrast to CreutzfeldJakob
thetosis, head nodding, and rigidity are also disease, dementia is often absent, although in
found. Approximately one third of patients die, the terminal stages the faculties of many patients
and 50% of the survivors have severe neu- are impaired. (Collinge, 519550)
ropsychiatric sequelae. In addition, Japanese
encephalitis virus was recently found to cause 43. (A) The classic (sporadic) CreutzfeldJakob dis-
acute flaccid paralysis in Vietnamese children. ease is a rapidly progressive multifocal demen-
The weakness is usually asymmetric, and the tia, usually with myoclonus. Onset usually
lower extremities are more often affected than occurs in individuals between 45 and 75 years of
the upper. Electrophysiological studies have age, with peak onset between 60 and 65 years.
localized the site of damage to the anterior horn Around 70% of those afflicted die less than
cells. The clinical and pathological features are 6 months after onset of symptoms. Prodromal
therefore similar to those of poliomyelitis. The features are seen in approximately one third of
diagnosis of JE infection is made serologically. cases and include fatigue, insomnia, depression,
The presence of anti-JE virus immunoglobulin weight loss, headaches, general malaise, and ill-
M in the CSF has a sensitivity and specificity in defined pain. In addition to mental deterioration
excess of 95%. Pathological studies demonstrate and myoclonus, frequent additional neurologi-
that the thalamus, basal ganglia, and midbrain cal features include extrapyramidal signs, cere-
are heavily affected, providing anatomical cor- bellar ataxia, pyramidal signs, and cortical blind-
relates for the tremor and dystonias that char- ness. (Collinge, 519550)
acterize Japanese encephalitis. (Hinson, 369374;
Solomon, 405415) 44. (B) The presence of chronic cerebellar ataxia
with pyramidal signs and dementia is highly
41. (D) Variant CreutzfeldJakob disease has a clin- suggestive of GSS. This diagnosis is supported
ical presentation in which behavioral and psy- by the presence of multicentric PrP amyloid
chiatric disturbances predominate; in some cases, plaques. (Collinge, 519550)
there are marked sensory phenomena. Initial
referral is often to a psychiatrist and the most 45. (A) Most brain abscesses in immunocompetent
prominent feature is depression, but anxiety, patients are bacterial in origin. The most com-
withdrawal, and behavioral changes are also mon etiologic organisms are microaerophilic
frequent. Other features include delusions, emo- streptococci and anaerobic bacteria. Other com-
tional lability, aggression, insomnia, and audi- mon organisms include S. aureus, Clostridium,
tory and visual hallucinations. In most patients, Enterobacteriaceae, P. aeruginosa and Haemophilus.
a progressive cerebellar syndrome develops, (Roos, 30)
with gait and limb ataxia. Dementia usually
develops later in the clinical course. Myoclonus 46. (D) The actual gold standard for the diagnosis
is seen in most patients, in some cases preceded of herpes encephalitis is CSF PCR for herpes
by chorea. The most remarkable neuropatho- simplex virus DNA, replacing brain biopsy.
logical feature of variant CreutzfeldJakob dis- CSF viral cultures for herpesvirus are almost
ease are the abundant PrP amyloid plaques in always negative. CSF herpes PCR studies have
the cerebral and cerebellar cortexes. (Collinge, been reported negative in the first few days of
519550) infection. If the diagnosis of herpes encephali-
tis is strongly suspected clinically, CSF PCR
42. (C) Kuru affects both sexes, and onset of disease should be repeated if the first test was negative.
ranges from the age of 5 years to over 60 years. (Roos, 70)
The mean clinical duration of illness is
12 months, with a range of 3 months to 3 years; 47. (D) About half of subdural empyemas are due
the course tends to be shorter in children. The to S. aureus, which represent the most common
central clinical feature is progressive cerebellar cause of spinal epidural abscess. (Roos, 345)
Answers: 4150 299

48. (C) The classic cerebrospinal findings in tuber- Dore GJ, et al: Changes to AIDS dementia complex in the
culosis meningitis include: era of of highly active antiretroviral therapy. AIDS.
1999;13:1249-1253.
an elevated opening pressure. Gagliuso DJ, Teich SA, Friedman AH, Orellana J. Ocular
a lymphocytic pleocytosis of 10 to 500/mm2. toxoplasmosis in AIDS patients. Trans Am Ophtalmol Soc.
a reduced glucose concentration. 1990;88:63-86.
an elevated protein concentration. (Roos, 345) Garg RK. Subacute sclerosing panencephalitis. Postgrad
Med J. 2002;78:63-70.
49. (C) Enteroviruses are the most common cause Gilden DH, KleinschmidtDeMasters BK, LaGuardia JJ,
of viral meningitis. The enteroviruses include Mahalingam R, Cohrs RJ. Neurologic complications of
the coxsackieviruses, echoviruses, polioviruses, the reactivation of varicella zoster virus. N Engl J Med.
2000;342:635-645.
and human enteroviruses 68 to 71. (Roos, 55)
Glass JD, Wesselingh SL, Selnes OA, McArthur JC. Clinical-
neuropathologic correlation in HIV-associated dementia.
50. (D) HIV-associated distal sensory polyneuropa- Neurology. 1993;43:2230-2237.
thy is the most common neurological complica- Glass JD, Fedor H, Wesselingh SL, McArthur JC.
tion of HIV infection affecting over 30% of patients Immunocytochemical quantitation of human immun-
during their lives, with pathological evidence of odeficiency virus in the brain: correlations with demen-
neuropathy in most patients at autopsy. (Roos, 103) tia. Ann Neurol. 1995;38:755-762.
Hinson VK, Tyor WR. Update on viral encephalitis. Curr
Opin Neurol. 2001;14:369-374.
Johnson RT, Gibbs CJ Jr. CreutzfeldtJakob disease and
related transmissible spongiform encephalopathies.
REFERENCES N Engl J Med. 1998;339:1994-2004.
Lincoln E, Sordillo S, Davies P. Tuberculosis meningitis in
Anonymous. Evaluation and management of intracranial children: a review of 167 untreated and 73 treated
mass lesions in AIDS: Report of the Quality Standards patients with special reference to early diagnosis.
Subcommittee of the American Academy of Neurology. J Pediatr. 1960;57:807-823.
Neurology. 1998;50:21-26. Luft BJ, Hafner R, Korzun AH, et al: Toxoplasmosis
Baumgartner JE, Rachlin JR, Beckstead JH, et al. Primary encephalitis in AIDS. Clin Infect Dis. 1992;15:211-222.
central nervous system lymphomas: natural history and Mastrianni JA. Roos RP. The prion diseases. Semin Neurol.
response to radiation therapy in 55 patients with 2000;20:337-352.
acquired immunodeficiency syndrome. J Neurosurg. McArthur JC. Sacktor N. Selnes O. Human immunodefi-
1990;73:206-211. ciency virusassociated dementia. Semin Neurol.
Berger JR, Paull L, Lanska D, Whiteman M. Progressive 1999;19:129-150.
multifocal leukoencephalopathy in patients with HIV McCutchan JA. Cytomegalovirus infections of the nervous
infection. J Neurovirol. 1998: 4:59-68. system in patients with AIDS. Clin Infect Dis. 1995;20:747-
Bradley WG, Daroff RB, Fenichel GM. Neurology in Clinical 754.
Practice: Principles of Diagnosis and Management. Oxford, Price RW. Brew BJ. The AIDS dementia complex. J Infect
UK: Butterworth-Heinemann; 2000. Dis. 1988;158:1079-1083.
Chinn RJS, Wilkinson ID, Hall-Craggs MA, et al. Toxo- Roos KL. Meningitis. In: Infections of the Nervous System. AAN
plasmosis and primary central nervous system lym- Courses. 2002.
phoma in HIV infection: diagnosis with MR spec- Roos KL. Principles of Neurologic Infectious Diseases. New
troscopy. Radiology. 1995;197:654-694. York: McGraw-Hill; 2005.
Churck SL, Sande MA. Infection with Cryptococcus neofor- Rotbart HA. Enteroviral infections of the central nervous
mans in the acquired immunodeficiency syndrome. system. Clin Infect Dis. 1995;20:971-981.
N Engl J Med. 1989;321:794-799. Schuchat A, Robinson K, Wenger JD, Harrison LH, Farley
Collinge J. Prion diseases of human and animals: their causes M, Reingold AL, et al. Bacterial meningitis in the United
and molecular basis. Annu Rev Neurosci. 2001;24:519-550. States in 1995. Active Surveillance Team. N Engl J Med.
Deresiewicz RL, Thaler SJ, Hsu L, Zamani AA. Clinical 1997;337:970-976.
and neuroradiographic manifestations of eastern equine Simpson DM, Taglia M. Neurological manifestations of
encephalitis. N Engl J Med. 1997;336:1867-1874. HIV-infection. Ann Intern Med. 1994;121:769-785.
Di Rocco A. Diseases of the spinal cord in human immun- Solomon T, Dung NM, Kneen R, Gainsborough M, Vaughn
odeficiency virus infection [review]. Sem Neurol. DW, Khanh VT. Japanese encephalitis. J Neurol Neurosurg
1999;19:151-155. Psychiatr. 2000:68:405-415.
This page intentionally left blank
CHAPTER 9

Neuroimmunology
Questions

1. Which of the following statements is true of (C) cerebellum


relapsingremitting multiple sclerosis (MS)? (D) brainstem
(A) The female predominance is approxi- (E) periventricular white matter
mately 4 to 1.
(B) Slowly evolving upper motor neuron 4. Which of the following is true of the mechanism
syndrome of the legs is typical of the of action of interferon in MS?
relapsingremitting form of MS. (A) It decreases the antigen presentation.
(C) An increase in body temperature may (B) It decreases cytokine production gov-
exacerbate symptoms. erned by type 2 helper T cells.
(D) The presence of oligoclonal bands (C) It reduces the secretion of interferon 10.
reduces the risk of recurrence. (D) It competes with myelin basic protein for
(E) Women with predominantly sensory presentation on MHC class I molecules.
symptoms have a more serious prognosis. (E) It increases the passage of the immune
cells across the bloodbrain barrier.
2. Which of the following statements is true of the
influence of genetic factors in MS? 5. Which of the following is characteristic of chronic
(A) The concordance rate of MS in monozy- inactive MS plaques?
gotic twins is equal to that among dizy- (A) They are hypercellular infiltrates mainly
gotic twins. composed of macrophages, T lympho-
(B) The absolute risk of MS in the first-degree cytes, and microglia.
relative of a patient with MS is not differ- (B) They have well-demarcated areas of
ent from the risk in the general popula- hypocellularity with myelin pallor.
tion.
(C) Shadow plaques are seen in the center of
(C) The HLA DR2 allele increases the risk of the demyelinating lesion.
MS.
(D) B lymphocytes tend to concentrate in the
(D) The mode of transmission of MS is auto- periventricular region
somal recessive.
(E) Local precipitation of immunoglobulin
(E) HLA DR and DQ polymorphisms are and complement is seen in areas of
associated with the course and severity of myelin damage.
the disease.

3. Demyelinating lesions in MS are least likely to


occur in the
(A) thalamus
(B) optic nerve

301
302 9: Neuroimmunology

6. Once autoreactive T cells have gained entry into (A) 2 weeks


the central nervous system (CNS), they invade (B) 1 year
the extracellular matrix aided by their secretion of (C) 5 years
(A) metalloproteinases (D) 10 years
(B) adhesion molecules (E) 20 years
(C) tumor necrosis factor (TNF)-
(D) interleukin (IL)-6 11. Which of the following mechanisms does not
contribute to the clinical remission in relapsing
(E) TNF-
remitting MS?
7. Autoreactive T lymphocytes respond to puta- (A) Resolution of inflammation
tive MS antigens through the formation of a (B) Redistribution of axolemmal sodium
molecular complex involving all of the following channels
except (C) Remyelination
(A) oligodendrocytes (D) Accumulation of amyloid precursor
(B) macrophages proteins
(C) T lymphocytes (E) Compensatory adaptation of the CNS
(D) perivascular monocytes
12. Which of the following is the most likely deter-
(E) microglia
minant of disability in MS?
8. Which of the following agents has been sug- (A) Number of enhancing lesions on T1
gested to cause immunological injury in MS? magnetic resonance imaging (MRI) with
contrast
(A) HIV virus
(B) Number of T2 MRI lesions
(B) Herpesvirus
(C) Positron emission tomography studies of
(C) Escherichia coli
brain activity
(D) Cytomegalovirus (CMV)
(D) Number of T1 hypodense lesions in the
(E) JC virus brain
(E) Total white matter axonal status
9. Which of the following is true of axonal injury in
MS?
13. In MR spectroscopy, axonal integrity correlates
(A) Less than 10% of axons are lost in chroni- with which of the following peaks?
cally demyelinating cervical spinal cord
(A) Choline
plaque.
(B) Lactate
(B) Acute active MS plaque does not demon-
strate axonal transection. (C) N-acetylaspartate
(C) Axonal loss may underlie the neurologi- (D) Creatinine
cal deficit during primary or secondary (E) None of the above
progressive MS.
(D) The accumulation of amyloid precursor 14. The decrease in NAA peak on MR spectroscopy
protein identifies intact axons in actively in MS is a marker of
demyelinating MS. (A) axonal regeneration
(E) Acute axonal injury correlates with the (B) axonal loss
number of CD4 T lymphocytes. (C) demyelination
(D) astrocyte activity
10. Transected axons are identified during the pro-
gression of MS as early as (E) oligodendrocyte activity
Questions: 623 303

15. Which of the following is true of neurophysio- 19. Which of the following drugs is used to improve
logical tests in MS? fatigue in MS?
(A) EEG studies are abnormal in less than (A) Carbamazepine
10% of patients with MS. (B) Pregabalin
(B) In case of cognitive deficit, electroen- (C) Amantadine
cephalography (EEG) may show (D) Gabapentil
increased activity in the frontal lobe.
(E) Duloxetine
(C) P 300 event-related potential latency cor-
relates with the degree of white matter 20. Which of the following myelin sheath proteins is
disease. found in the CNS as well as in the peripheral
(D) Evoked potentials are very sensitive in nervous system?
detecting the spatial distribution of MS
lesions. (A) Protein zero (P0)
(E) The evaluation of middle latency audi- (B) Peripheral myelin protein 22 (PMP22)
tory evoked potentials does not affect the (C) Myelin basic protein
sensitivity of brainstem auditory evoked (D) Proteolipid protein
potentials. (E) Oligodendrocyte specific protein

16. Which of the following is not associated with 21. Which of the following cytokines is not pro-
unfavorable prognosis in MS? duced by Th2 lymphocytes?
(A) Male sex (A) IL-4
(B) Younger age of onset (B) IL-5
(C) Motor or cerebellar signs at onset (C) Interferon (IFN)-
(D) Early disability (D) IL-10
(E) Incomplete remission after the first attack (E) IL-13

17. The first line of treatment of spasticity in MS is 22. Upregulation of IL-4 is most likely seen in which
(A) the use of reciprocal motion exercises phase of experimental autoimmune encephalitis?
(B) baclofen (A) Induction
(C) clonidine (B) Demyelination
(D) cyproheptadine (C) Relapse
(E) dantrolene (D) Epitope spreading
(E) Remission
18. Which of the following antispasticity medica-
tions acts as an 2 sympathetic agonist with 23. Which of the following is a proinflammatory
effects on polysynaptic reflexes? cytokine?
(A) Tizanidine (A) IL-4
(B) Dantrolene (B) IL-10
(C) Baclofen (C) TNF-
(D) Cyproheptadine (D) IFN-
(E) Phenol (E) TGF-
304 9: Neuroimmunology

24. Which of the following is the correct combination 29. Demyelinating polyneuropathy is associated
of a paraneoplastic syndrome and the antineuronal with
antibody associated with that syndrome?
(A) anti-Ri antibody
(A) LambertEaton myasthenic (B) antiamphiphysin antibody
syndrome/anti-MAG (C) anti-CV2 antibody
(B) Myasthenia gravis (MG)/anti-CV2 (D) anti-Ma2 antibody
(C) Limbic encephalitis/anti-Ma2 (E) anti-MAG antibody
(D) Peripheral neuropathy/antiamphiphysin
(E) Cerebellar degeneration/anti-MAG 30. Anti-CV2 antibody is associated with
(A) thymoma related sensory neuronopathy
25. Which of the following paraneoplastic antibod-
ies is not associated with small cell lung cancer (B) cerebellar degeneration
(SCLC)? (C) LambertEaton myasthenic syndrome
(D) testicular cancer limbic encephalitis
(A) AntiP/Q type VGCC antibody
(E) demyelinating neuropathy
(B) Anti-Hu antibody
(C) Anti-MAG antibody 31. Antiamphiphysin antibody is most commonly
(D) Antiamphiphysin antibody associated with
(E) Anti-Ri antibody
(A) MG
26. MG is associated with (B) neuromyotonia
(C) stiff-man syndrome
(A) antiP/Q type voltage-gated calcium
(D) SCLCrelated encephalomyelitis
channels
(E) thymoma-related sensory neuronopathy
(B) antiacetylcholine receptor antibody
(C) anti-Hu antibody 32. Cerebellar degeneration is most commonly asso-
(D) antivoltage-gated potassium channel ciated with
antibody
(A) antiacetylcholine receptor antibody
(E) anti-Yo antibody
(B) anti-Hu antibody
27. Neuromyotonia is associated with (C) antivoltage-gated potassium channel
antibody
(A) antiP/Q type voltage-gated calcium
(D) anti-Yo antibody
channels
(E) anti-Ri antibody
(B) antiacetylcholine receptor antibody
(C) antiamphiphysin antibody 33. Anti-Ma2 antibody is commonly associated with
(D) antivoltage-gated potassium channel
antibody (A) cerebellar degeneration
(E) anti-Yo antibody (B) LambertEaton myasthenic syndrome
(C) testicular cancer limbic encephalitis
28. Anti-Ri antibodies are associated with (D) demyelinating neuropathy
(A) cerebellar degeneration (E) opsoclonus
(B)LambertEaton myasthenic syndrome
34. LambertEaton myasthenic syndrome is com-
(C) testicular cancer limbic encephalitis monly associated with
(D) chronic inflammatory demyelinating
polyneuropathylike (CIDP-like) (A) antiP/Q type voltage-gated calcium
neuropathy channels
(E) opsoclonus (B) anti-acetylcholine receptors antibody
Questions: 2441 305

(C) anti-Hu antibody 39. Which of the following antineuronal antibodies


(D) antivoltage-gated potassium channel is associated with testicular cancer?
antibody (A) Anti-Tr antibodies
(E) anti-Yo antibody (B) Antiacetylcholine receptors antibodies
(C) Anti-CV2 antibodies
35. The most frequent cause of anti-Yo antibodies is
(D) Amphiphysin antibodies
(A) breast cancer (E) Anti-Ma2 antibodies
(B) ovarian cancer
(C) SCLC 40. Which of the following antineuronal antibodies
(D) bladder cancer is not associated with limbic encephalitis?
(E) uterine cancer (A) Anti-Hu antibodies
(B) Anti-Ma2 antibodies
36. Which of the following is true of paraneoplastic
(C) Anti-CV2 antibodies
encephalomyelitis?
(D) Anti-PCA2 antibodies
(A) It usually antedates the diagnosis of (E) Anti-Yo antibodies
breast cancer.
(B) It is commonly associated with anti-CV2 41. A 60-year-old right-handed woman came to the
antibodies. emergency room because she had recently
(C) It is frequently associated with irregular, developed an unsteady gait, dizziness, and
continuous large-amplitude conjugate double vision, progressing over several weeks.
saccades in all directions of gaze. Neurological examination showed generalized
(D) It may be associated with central severe ataxia, more prominent in the trunk, and
hypoventilation. mild dysarthria. Ocular examination showed
(E) Patients with paraneoplastic opsoclonus ocular flutter and abnormal
encephalomyelitis may develop seizures visual tracking. MRI of the head was normal.
but not nonconvulsive status. Laboratory evaluation showed the presence of
antineuronal antibodies. Which of the follow-
37. Anti-Hu antibodies are seen most frequently in ing is true of the patient condition?

(A) SCLC (A) Diffuse loss of pyramidal cells is the


pathological hallmark of the patients
(B) breast cancer
condition.
(C) prostate cancer
(B) Anti-Hu antibody is the most likely type
(D) neuroblastoma of antineuronal antibody found in this
(E) sarcoma patient.
(C) SCLC is the most likely malignancy that
38. Opsoclonus seen in conjunction with abnor- is causing this patients symptoms.
malities in ocular motility is most commonly
(D) Inflammatory infiltrate involving the
associated with
tegmentum of the pons and mesen-
(A) anti-Ma antibody cephalon may be seen with cerebellar
(B) anti-Yo antibody degeneration.
(C) anti-Hu antibody (E) Immunosuppressive treatment does not
(D) anti-Ri antibody reverse these symptoms.
(E) antiamphiphysin antibody
306 9: Neuroimmunology

42. A 50-year-old man developed a progressive 46. In children, the most frequent cause of opso-
increase of muscle stiffness, rigidity, lumbar lor- clonusmyoclonus syndrome is
dosis, and urinary incontinence. If his symp-
(A) lymphoma
toms are caused by a paraneoplastic syndrome,
which of the following is true? (B) neuroblastoma
(C) SCLC
(A) The use of clonazepam will not improve
(D) thymoma
the rigidity.
(E) breast cancer
(B) SCLC is the major cause of the syndrome.
(C) Antiamphiphysin antibodies are the 47. Antiglial nuclear antibody is commonly seen in
major antineuronal antibodies found in
the serum of this patient. (A) SCLC
(D) Antiglutamic acid decarboxylase (GAD) (B) astrocytoma
is found in 70% of patients with a similar (C) oligodendroglioma
paraneoplastic condition. (D) lymphoma
(E) Treatment of the primary tumor will not (E) glioblastoma multiforme
improve the patients neurological symp-
toms. 48. Which of the following is true of dermato-
myositis?
43. In acute inflammatory demyelinating polyneu-
(A) It is the most common inflammatory
ropathy, the most frequently detected antigan-
myopathy after the age of 50 years.
gliosides antibody is
(B) There is asymmetric muscle involvement.
(A) IgG antibody against GM1 (C) It is associated with the diagnosis of
(B) antibody against GQ1b malignancy in 45% of cases.
(C) antibody antiglycolipids (D) It affects men 3 times more than women.
(D) anti-EVB antibody (E) Irregular rimmed vacuoles are present in
(E) anti-GD1a antibody up to 70% of muscle fibers.

44. Which of the following is suggestive of the mech- 49. According to the McDonald criteria, what addi-
anism of action of intravenous immunoglobu- tional data are needed to confirm the diagnosis
lins in the treatment of demyelinating polyneu- of MS in a 45-year-old woman who developed
ropathy? two partially reversible attacks of blurred vision,
ataxic gait, and left leg weakness over 3 months?
(A) Downregulation of Th2 cytokine produc-
tion (A) A third clinical attack
(B) T-cell activation (B)Two brain MRI-detected lesions
(C) Complement activation (C) None
(D) Immunoglobulin Fab receptor blockade (D) Presence of oligoclonal bands on cere-
(E) Stimulation of immunoglobulin production brospinal fluid (CSF) examination
(E) Abnormal visual evoked potentials
45. Anti-N-methyl-D-aspartate (NMDA) receptor
associated encephalitis is usually seen in con- 50. Which of the following is the strongest genetic
junction with factor influencing susceptibility to MS?
(A) SCLC (A) HLA DRB1
(B) lymphoma (B) APOE gene
(C) ovarina teratoma (C) Alpha synuclein gene
(D) neuroblastoma (D) Dystrophin gene
(E) breast cancer (E) Merlin gene
Questions: 4260 307

51. Which of the following infectious agents is most 56. Which of the following autoantibodies is asso-
likely to play a contributory role in the patho- ciated with neuromyelitis optica?
genesis of MS?
(A) Aquoporin-4 antibody
(A) HIV virus (B) Antisulfatide antibody
(B) EpsteinBarr virus (C) Anti-MAG antibody
(C) Campylobacter jejuni (D) Anti-GAD antibody
(D) Mumps virus (E) Anti-Hu antibody
(E) Rubella virus
57. Which of the following syndromes has the least
52. Which of the following medications acts by likely chance of having a paraneoplastic origin?
blocking CD20 receptor on circulating B cells?
(A) Stiff-man syndrome
(A) Rituximab (B) Cerebellar degeneration
(B) Intravenous immunoglobulin (C) Limbic encephalitis
(C) Glatiramer (D) Motor neuron disease
(D) IFN- (E) Brainstem encephalitis
(E) Corticosteroids
58. MRI of the brain is useful in which of the fol-
53. Which drug has the following mechanism lowing paraneoplastic syndromes?
of action: stimulation of anti-inflammatory
(A) Subacute sensory neuronopathy
cytokine production and inhibition of synthesis
and transport of matrix metalloproteinases? (B) Limbic encephalitis
(C) LambertEaton myasthenic syndrome
(A) Rituximab
(D) Stiff-man syndrome
(B) Intravenous immunoglobulin
(E) Brain stem encephalitis
(C) Glatiramer
(D) IFN- 59. Which of the following differentiates MS from
(E) Mitoxantrone postinfectious encephalomyelitis?
(A) Commonly monophasic
54. Which medication acts by intercalating with
DNA and suppressing lymphocytes? (B) Equal gender incidence
(C) Seizure seen in 50% of cases
(A) Rituximab
(D) Optic neuritis
(B) Intravenous immunoglobulin
(E) Preserved level of consciousness
(C) Glatiramer
(D) IFN- 60. Which of the following factors may increase the
(E) Mitoxantrone risk of developing MS after isolated transverse
myelitis?
55. Which drug has activity against circulating
(A) Complete transverse myelitis
immune cell integrins?
(B) Asymmetric sensory and motor deficit on
(A) Rituximab neurological examination
(B) Intravenous immunoglobulin (C) Normal CSF examination.
(C) Glatiramer (D) Limited nonconfluent intramedullary
(D) Natalizumab lesions on spinal MRI
(E) Mitoxantrone (E) Normal multimodality evoked potentials
Answers and Explanations

1. (C) In relapsingremitting MS, the type of MS analysis and MRI, in particular, have simplified
present in 80% of patients, symptoms and signs the diagnostic process. The relapsing forms are
typically progress over a period of several days, considered clinically definite when neurological
stabilize, and then often improve spontaneously dysfunction becomes disseminated in space and
or in response to corticosteroids within weeks. time. Studies of the natural history of the disease
Relapsingremitting MS typically begins in the have provided important prognostic informa-
second or third decade of life and has a female tion. Ten percent of patients do well for more
predominance of approximately 2 to 1. The ten- than 20 years and are thus considered to have
dency for corticosteroids to speed recovery from benign MS. Approximately 70% will have sec-
relapses often diminishes with time. Persistent ondary progression. Frequent relapses in the
signs of CNS dysfunction may develop after a first 2 years, progressive course from the onset,
relapse, and the disease may progress between male sex, early permanent motor or cerebellar
relapses. Twenty percent of affected patients findings, and presence of oligoclonal bands in
have primary progressive MS, which is charac- the CSF are associated with the more severe
terized by a gradually progressive clinical course course of the disease. Women and patients with
and a similar incidence among men and women. predominantly sensory symptoms and optic
Relapsingremitting MS typically starts with neuritis have a more favorable prognosis.
sensory disturbances, unilateral optic neuri- (Noseworthy, 938952)
tis, diplopia (internuclear ophthalmoplegia),
Lhermittes sign, limb weakness, clumsiness, 2. (C) Evidence that genetic factors have a sub-
gait ataxia, and neurogenic bladder and bowel stantial effect on susceptibility to MS is unequiv-
symptoms. Many patients describe fatigue that ocal. The concordance rate of 31% among
worsens in the afternoon and is accompanied by monozygotic twins is approximately 6 times the
physiological increases in body temperature. rate among dizygotic twins (5%). The absolute
Prominent cortical signs (aphasia, apraxia, risk of the disease in a first-degree relative of a
recurrent seizures, visual-field loss, and early patient with MS is less than 5%; however, the
dementia) and extrapyramidal phenomena only risk in such relatives is 20 to 40 times the risk in
(chorea and rigidity) rarely dominate the clinical the general population. The HLA-DR2 allele
picture. substantially increases the risk of MS. The mag-
Patients who have primary progressive MS nitude of the relative risk depends on the fre-
often present with a slowly evolving upper quency of the HLA-DR2 allele in the general
motor neuron syndrome of the legs. Typically, population. The mode of transmission of genetic
this variant worsens gradually, and quadri- susceptibility to MS is complex. Most cases are
paresis, cognitive decline, visual loss, brain stem sporadic, despite the clear excess risk among
and cerebellar syndromes, bowel, bladder, and the relatives of patients. Investigators have used
sexual dysfunction may develop. The diagnosis the usual genetic approaches to identify genes
is based on established clinical and, when associated with an increased risk of MS. HLA-
necessary, laboratory criteria. Advances in CSF DR and DQ polymorphisms are not associated

308
Answers: 17 309

with the course and severity of MS despite their of autoreactive T cells and antigen-nonspecific
substantial contribution to disease susceptibility. monocytes. Macrophages and lymphocytes form
(Noseworthy, 938952) prominent perivascular cuffs and invade the
parenchyma, whereas plasma cells and B cells
3. (A) The pathological hallmark of chronic MS is tend to concentrate in the perivascular region
the demyelinated plaque, which consists of a well- only. Most lymphocytes within plaques are T
demarcated hypocellular area characterized by cells, including both CD4 (helper) and CD8
the loss of myelin, relative preservation of axons, (cytotoxic) cells. The CD4 cells can be func-
and formation of astrocytic scars. Lesions have a tionally divided into Th1 (which secrete proin-
predilection for the optic nerves, periventricular flammatory cytokines, such as TNF- and IFN-)
white matter, brainstem, cerebellum, and spinal or Th2 (which secrete antiinflammatory cytokines
cord white matter; they often surround one or such as IL-4, -5, and -6).
several medium-sized vessels. Although the Chronic plaques display well-demarcated
lesions are usually round or oval, they often have areas of hypocellularity with myelin pallor or
finger-like extensions along the path of small- or loss. There are varying degrees of axonal loss,
medium-sized blood vessels (Dawsons fin- usually most obvious in the center of the lesion.
gers). Inflammatory cells are typically perivas- There is typically a persistent but minor inflam-
cular in location, but they may diffusely infiltrate matory response, with only a few scattered
the parenchyma. The composition of the inflam- perivascular lymphocytes present, although
matory infiltrate varies depending on the stage of plasma cells may occasionally be prominent.
demyelination. In general, it is composed of lym- Shadow plaques are circumscribed regions
phocytes and macrophages; the latter predomi- where axons maintain uniformly thin myelin
nate in active lesions. (Noseworthy, 938952) sheaths. They may occur within acute plaques or
at the edge of chronic ones. These plaques rep-
4. (D) IFN--1a and glatiramer acetate reduce the resent areas of remyelination and are macro-
frequency of relapses of MS. IFN- may delay scopic evidence that the CNS white matter pos-
the progression of disability in patients with sesses the means for self-repair. Shadow plaques
minor disability who have a relapsing form of are seen in conjunction with actively demyeli-
MS. The specific mechanisms of action of these nating lesions that retain viable oligodendro-
agents in MS are incompletely understood. The cytes in the plaque center. (Wingerchuk, 263281)
interferons reduce the proliferation of T cells
and the production of TNF-, decrease antigen 6. (A) An intact bloodbrain barrier allows lim-
presentation, alter cytokine production to favor ited passage of T lymphocytes that may not have
ones governed by type 2 helper T (Th2) cells, antigen specificity. This may be initiated by the
increase the secretion of IL-10, and reduce the interaction of adhesion molecules expressed on
passage of immune cells across the bloodbrain the surface of lymphocytes with complemen-
barrier by means of their effects on adhesion mol- tary integrins present on the endothelium,
ecules, chemokines, and proteases. Glatiramer resulting in T-cell rolling and adherence to the
acetate may promote the proliferation of Th2 luminal surface. Examples of such molecules
cytokines, compete with myelin basic protein for include vascular cell adhesion molecule (VCAM)
presentation on MHC class II molecules, alter the and intercellular adhesion molecule (ICAM).
function of macrophages, and induce antigen- After crossing the bloodbrain barrier, activated
specific suppressor T cells. (Noseworthy, 938952) T cells invade the extracellular matrix, aided by
their secretion of matrix metalloproteinases,
5. (B) MS plaques may be characterized as active which degrade myelin components as well as
or inactive. The presence in macrophages of acti- type IV collagen matrix and regulate cytokine
vation markers and specific myelin degradation production. (Wingerchuk, 263281)
products is suggestive of active plaque. Macro-
phages are numerous in active lesions, which 7. (A) Autoreactive T cells respond to putative MS
are hypercellular and contain patchy infiltrates autoantigens presented by antigen-presenting
310 9: Neuroimmunology

cells (APCs) through formation of a trimolecu- myelin antigens. This explanation has been used
lar complex involving perivascular monocytes, to implicate herpesvirus in MS pathogenesis,
microglia and macrophages, parenchymal lym- although a latent viral infection, rather than
phocytes, and possibly astrocytes. These cells mimicry, could also potentially result in demyeli-
express MHC molecules. There are two principal nation and oligodendroglial loss. The T-cell
types of MHC molecules: class I (includes HLA- receptor normally maintains an extremely high
A, -B, and -C) and class II (includes HLA-DR, - level of cross-reactivity, probably to balance the
DP, and -DQ). These molecules bind peptide requirement to recognize nonself antigens and to
antigens as part of the processing they require reduce the possibility of loss of self-tolerance.
for presentation to different T lymphocytes. The concept of molecular mimicry remains spec-
Lymphocytes of the CD4 type recognize anti- ulative. (Wingerchuk, 263281)
gens in conjunction with MHC class II mole-
cules; whereas CD8 lymphocytes recognize 9. (C) It has long been recognized that axons are
antigens in the context of MHC class I mole- relatively but not absolutely spared in MS, espe-
cules. The trimolecular complex is completed cially early in the disease. Recent pathological
by interaction with the T-cell receptor. and noninvasive radiological studies have
MS seems primarily to be a disease involv- focused attention on how, early in the disease,
ing immune responses to antigens presented by axons may be injured or lost, the possible con-
class II molecules, although class I mechanisms tribution of axonal injury to clinical disability,
are receiving increased attention. The interac- and the development of progressive MS. Not
tion between a CD4 T lymphocyte and an APC surprisingly, axonal density is reduced in chronic
results in antigen-specific signaling; however, plaques. Whereas estimates of axonal number
T-cell activation requires the presence of cos- and density are challenged by the variable pres-
timulatory molecules. Costimulatory molecules ence of edema, myelin loss, atrophy, and inflam-
CD28 and CTLA-4 are present on the T-cell sur- matory cell infiltrates, between 50% and 80% of
face. These molecules interact with their ligands axons may be lost in chronically demyelinated
B7-1 and B7-2 to promote activation; when cervical spinal cord plaques. The accumulation
absent, the T cellAPC interaction results in T- of -amyloid precursor protein identifies dam-
cell apoptosis and limitation of the immune aged axons in actively demyelinating MS
response. When the costimulatory molecules lesions. Acute, active MS plaques may also
and their respective ligands are present, intra- demonstrate axonal transection, swelling, for-
cellular signaling pathways involving phos- mation of terminal spheroids, and regenerative
phorylation-dependent enzymes and second sprouting.
messenger systems activate secretory and pro- Others have confirmed early axonal loss in
liferative mechanisms within the T lymphocyte. the early inflammatory phases of the disease,
(Wingerchuk, 263281) even in the absence of demonstrable primary
demyelination. Axonal loss is irreversible and
8. (B) Dysregulation of the immune system may probably underlies the worsening neurological
contribute to the initiation or propagation of a deficits that accrue in the primary and second-
pathological state in MS by autoreactive T cells. ary progressive forms of the disease; clinical
The causative autoantigen(s) in MS is still not progression correlates with brain atrophy in
known; however, the leading candidates are both of these forms of MS. The mechanisms of
myelin protein constituents. The role of other axonal injury are largely unknown. In particular,
myelin components is less well studied. it is not clear whether inflammatory effects may
Molecular mimicry has been hypothesized to damage axons directly or whether they operate
explain immunological injury in autoimmune primarily through a pathway that includes
diseases. Under this schema, antigens present demyelination. Recently, it was found that acute
in or originating from an exogenous pathogen axonal injury correlates with the number of
activate T cells. These cells then induce CNS macrophages and CD8 T lymphocytes within
demyelination by recognizing cross-reactive plaques but not with TNF- or nitric oxide
Answers: 813 311

synthase expression. This suggests that contributor to atrophy in MS, although demyeli-
axonal injury is not solely due to demyelina- nation and reduced axon diameter may also
tion. Glutamate-driven excitotoxic mechanisms decrease tissue volume. Axonal pathology and
may be operative as well. (Wingerchuk, 263281) loss is not restricted to MS lesions, as all axons
will undergo Wallerian degeneration distal to
10. (A) Histopathologic studies of MS brains have the site of axonal transection of the MS lesion.
demonstrated axonal injury in lesions undergo- Axonal loss in normal-appearing white mat-
ing inflammatory demyelination. Axonal ovoids ter (NAWM) has been quantified in a number of
(which are characteristic of newly transected recent autopsy studies. Axonal density was
axons) and extensive accumulation of the amy- reported to be reduced by 19% to 42% in the lat-
loid precursor protein (APP) have been reported eral corticospinal tract of MS patients with lower
in active lesions and at the border of chronic limb weakness. Axonal loss was investigated in
active lesions. APP is detected immunohisto- NAWM from cervical spinal cords of patients
chemically only in axons with impaired axonal with SP-MS. The average reduction in axonal
transport. This result indicates not only axonal density in these samples was as much as 57%. As
dysfunction within inflammatory MS lesions NAWM constitutes the greatest proportion of
but also suggests that many of the axons are white matter in MS patients and as levels of
transected. Importantly, these changes are the neuron-specific marker N-acetyl aspartate
observed in patients with a short duration of (NAA) in NAWM show a strong correlation
disease. A morphological investigation quanti- with disability, the possibility has been raised
fied axonal ovoids in MS brains with disease that total white matter axonal status may be a
durations from 2 weeks to 27 years. The results more precise determinant of disease progres-
of the study not only confirm that axonal tran- sion than the presence and characteristics of
section is abundant during the early stages of the individual lesions. (Bjartmar, 271278)
disease but also demonstrate that the density of
transected axons correlates with inflammatory 13. (C) The clinical importance of axonal degener-
activity in the lesions. Because APP accumula- ation in MS suggests that neuronal markers
tion correlates with number of macrophages and could be useful for noninvasive monitoring of
CD8 T lymphocytes but not with expression of disease progression and efficiency of therapy
putative mediators of demyelination such as in these patients. In this respect, measurement
TNF- and inducible nitric oxide synthase, it is of NAA by proton magnetic resonance spec-
suggested that axonal damage in MS lesions troscopy (MRS) is a promising tool. NAA
might not be directly proportional to demyeli- appears relatively specific for neurons and neu-
nating activity. (Bjartmar, 271278; Trapp, 278285) ronal processes in vivo, although expression by
oligodendrocyte progenitors and oligodendro-
11. (D) Four mechanisms may contribute to clinical cytes in vitro has been reported. Reduced levels
remission in MS: resolution of the inflamma- of NAA as determined by MRS have been
tion, redistribution of axolemmal sodium chan- demonstrated in a number of neurodegenerative
nels, remyelination, and compensatory adapta- disorders, including MS. At acute stages of MS,
tion of the CNS. (Bjartmar, 271278) reduced NAA occurs primarily in lesions, is
partly reversible, and correlates with reversible
12. (E) In MS, the correlation between clinical dis- functional impairment. Over time, NAA appears
ability and atrophyas revealed by MRI of the to decrease irreversibly in normal-appearing
cerebellum, spinal cord, and cerebrumhas white matter (NAWM), indicating that axonal
been interpreted as a reflection of axonal loss. In loss or damage occurs outside MS lesions.
secondary progressive MS (SP-MS), cervical Reduced white matter NAA correlates with
spinal cord atrophy averages 25% to 30%. In a increased disability over time.
group of RR-MS patients with mild to moderate These results demonstrate a side-to-side
disability followed over 2 years, brain atrophy correlation between NAA levels, motor impair-
increased yearly. Axonal loss is a conceivable ment, and conduction times, conforming with
312 9: Neuroimmunology

the view that axonal pathology in NAWM is a ment of the sensory and motor pathways or to
likely determinant of disease progression in MS. objectify vague symptoms. Previous studies
In theory, reduced NAA in MS tissue could indicated that in isolated syndromes, approxi-
reflect multiple mechanisms, including reversible mately one third of patients have subclinical
neuronal/axonal damage due to inflammatory involvement of sensory pathways revealed by
demyelination, altered neuronal metabolism EPs, mostly by visual-evoked potentials (VEPs)
related to activity, axonal atrophy, or axonal loss. and somatosensory-evoked potentials (SEPs).
In order to differentiate between these possibil- In a study of 112 patients with isolated optic
ities, NAA is measured by high-performance neuritis, 34.1% had abnormal extravisual EPs;
liquid chromatography at autopsy in MS spinal however, the contribution of neurophysiological
cord white matter and correlated with axonal techniques in demonstrating spatial dissemina-
loss as determined by immunohistochemistry. tion of the lesions was quite poor: only 4% of
NAA is significantly reduced in chronic inactive patients with abnormal extravisual EPs had nor-
MS lesions compared with MS nonlesion and mal brain MRI.
control white matter, and the reduction corre- The major limiting factor on the usefulness
lates with total axonal volume and axonal den- of EPs in detecting subclinical involvement is
sity. These results demonstrate that reduced that the presence of a lesion is revealed only if it
NAA levels in inactive lesions correspond to affects pathways explored by neurophysiologi-
substantial axonal loss and support axonal loss cal investigations. Moreover, a significant pro-
as a major cause of decreased white matter portion of the fibers must be affected to produce
NAA in secondary progressive MS. (Bjartmar, recordable modifications of EPs. The evaluation
271278) of middle latency AEPs complemented by brain-
stem AEPs (BAEP) in a group of 30 clinically
14. (B) The measurement of N-acetyl aspartate definite MS patients increased the sensitivity of
(NAA) as a neuronal marker by proton MRS is a the test from 60% to 83%, suggesting that the
valuable tool for assessing the progression of MS. validation of middle latency AEPs could estab-
Reduced NAA occurs in MS lesions and becomes lish criteria for a more comprehensive evalua-
irreversible as the disease progresses. In theory, tion of the auditory system.
reduced NAA in MS tissue could reflect multiple The EEG, which is the expression of multi-
mechanisms, including reversible neuronal/ ple neuronal network interactions affected by
axonal damage due to inflammatory demyelina- white matter damage, may be used as an indi-
tion, altered neuronal metabolism related to activ- cator of the global status of such interactions.
ity, axonal atrophy, or axonal loss. In order to dif- Spectral analysis of the EEG revealed abnor-
ferentiate between these possibilities, Bjartmar malities in 40% to 79% of MS patients; the main
and colleagues studied the NAA levels in MS changes were an increase of slow frequencies
spinal cord white matter by high-performance and decrease of the alpha band, which is related
liquid chromatography at autopsy. They found a to cognitive dysfunction. Event-related poten-
correlation between the NAA level and the axonal tials (ERPs) are brain waves related to stimulus
loss as determined by immunohistochemistry. processing. P300, the most widely studied ERP,
NAA was significantly reduced in chronic inactive is a positive wave recorded over the scalp when
MS lesions compared with MS white matter with- subjects discriminate stimuli differing in some
out lesion and control white matter. The reduction physical dimension. It is thought to represent a
in NAA concentration was found to correlate with closure of the evaluation process stimulus, and
total axonal volume and axonal density. These its latency has been proposed as an indicator of
results demonstrate that reduced NAA levels in information processing speed. This process is
inactive lesions correspond to substantial axonal electively affected in MS, and P300 latency is
loss. (Bjartmar, 271278) increased in MS patients. The increase in P300
latency is correlated with cognitive impairment
15. (C) In the early phases of MS, evoked poten- and the degree of white matter involvement.
tials (EPs) are used to detect subclinical involve- (Leocani, 255261)
Answers: 1419 313

16. (B) Factors associated with unfavorable prog- of its direct muscle effect of preventing excita-
nosis in MS include: tioncontraction coupling. If use of dantrolene is
continued, liver function should be monitored at
Male sex
least every 3 months; liver toxicity can occur in
Older age at onset
rare instances. If the usual antispasticity med-
Motor or cerebellar signs at onset
ications fail or are contraindicated, various other
Short interval between initial and second
medications can be tried. Clonidine hydrochlo-
attack
ride or cyproheptadine hydrochloride (Periactin),
High relapse rate in early years
serotonin, acetylcholine, and histamine antago-
Incomplete remission after first relapses
nists have been reported to reduce MS-related
Early disability
spasticity. Selective botulinum toxin injections
High lesion load detected by early MRI of
are also used. (Stolp-Smith, 11841196)
the brain
(Polman, 490494) 18. (A) Tizanidine is a centrally acting -2-
sympathetic agonist pharmacologically similar
17. (A) Approximately 55% of patients with MS to clonidine with effects on polysynaptic reflex
have detectable spasticity. This is defined as arcs. Tizanidine has been shown to reduce spas-
increased resistance to passive range of motion ticity in several placebo-controlled clinical trials
of the limb, which can be associated with exag- and has had efficacy similar to that of baclofen.
gerated withdrawal to noxious stimuli, spasms, Muscle weakness occurs less frequently with
clonus, and hyperreflexia. Spasticity can inter- tizanidine than with baclofen. The most com-
fere with volitional movement, cause pain, and mon side effects are drowsiness, dry mouth, and
disrupt sleep or activities of daily living. The orthostatic hypotension. Liver function abnor-
first line of treatment of spasticity involves sim- malities rarely occur. (Stolp-Smith, 11841196)
ple physical measures including stretching, use
of reciprocal motion exercises such as exercy- 19. (C) Fatigue is a common problem for patients
cling, and, for some patients, passive standing in with MS. Pharmacological therapy may be help-
a standing frame. ful when other medical problems that may cause
When these measures fail, orally adminis- fatigue, such as anemia or hypothyroidism, are
tered pharmacological agents may be necessary excluded. Amantadine hydrochloride is the
and in most cases are sufficient to manage the neg- most widely used medication for MS-related
ative manifestations of increased muscle tone. The fatigue. Its mechanism of action in MS is un-
classically used medications include baclofen, known, but the drug has central dopaminergic
benzodiazepines, and dantrolene sodium. The activity that may be relevant. Pemoline, also
first two drugs can cause drowsiness, and all used for MS-related fatigue, has had a response
three can increase fatigue and weakness. Careful rate of approximately 50% in some studies.
dose titration is therefore critical. The dose of In a prospective open label study, modafanil
baclofen, a gamma-aminobutyric acid (GABA) was found to significantly improve fatigue and
agonist, should be titrated slowly because patients sleepiness in patients with MS. Unlike the higher-
with MS may be more sensitive than other patients dose regimen required in narcolepsy, a low-dose
to the side effects of drowsiness and weakness. If regimen of modafinil was found to be effective
the response to baclofen is insufficient or if this and well tolerated by MS patients. Potassium
drug causes intolerable side effects, diazepam or channel blocking agents, such as 4-aminopyri-
dantrolene may be substituted or added. In dine and 3,4-diaminopyridine, may also prove
patients with fatigue or drowsiness from baclofen, to be effective for MS-related fatigue. With use of
diazepam may exacerbate these symptoms. these agents, the major toxic effect of these drugs
Use of diazepam should be avoided in those at high serum levels is the occasional occurrence
patients with a tendency toward depression or of generalized tonicclonic seizures. Patients
a history of substance abuse. Dantrolene can may experience increased fatigue and a
then be used but may cause weakness because decline in neurological function due to warm
314 9: Neuroimmunology

environments (Uhthoffs phenomenon). Thus, acterized being Ma1 and Ma2. The expression of
remaining in a cool environment can enhance these proteins is highly restricted to neurons
function. (Stolp-Smith, 11841196; Zifko, 983987) and spermatogenic cells of the testis. The anti-Ta
(anti-Ma2) antibodies are present in the serum
20. (C) The myelin sheaths of the CNS and periph- and spinal fluid of patients with paraneoplastic
eral nervous system contain distinct sets of pro- limbic and brainstem encephalitis associated
teins, but myelin basic protein is found in the with testicular cancer. These antibodies recog-
myelin sheaths of both. In the peripheral nerv- nize epitopes mainly contained in Ma2 (a 40-
ous system, the compact myelin contains protein kDa neuronal protein). Limbic encephalitis is
0 (P0), peripheral myelin protein (PMP22), and frequently associated with small cell lung cancer
myelin basic protein (MBP); whereas the non- (SCLC) and is characterized clinically by suba-
compact myelin contains ecadherin, myelin- cute confusion, amnesia, and psychiatric symp-
associated glycoprotein (MAG), and connexin toms. MRI usually reveals increased signal in
32 (Cx32). In the CNS, myelin contains prote- the medial temporal lobes on T2-weighted
olipid protein (PLP), oligodendrocyte-specific images and CSF may show a mild lymphocytic
protein (OSP), myelinoligodendrocyte basic pleocytosis. Limbic encephalitis can occur as an
protein, and myelin basic protein. (Arroyo, isolated syndrome or as part of a multifocal dis-
118) order associated with cerebellar, brainstem,
spinal cord, and dorsal root ganglion involve-
21. (C) As CD4 lymphocyte responses develop ment. The largest study of paraneoplastic limbic
in response to immune stimulation and T-cell encephalitis revealed that anti-Hu antibodies
populations become divided toward the pro- only occurred in 50% of the patients who had an
duction of Th1 or inflammatory cytokines ver- associated SCLC. The antibody-positive
sus Th2 or regulatory cytokines. The paradig- patients are more likely to have involvement of
matic Th1 cytokine is IFN-, IL-4 being the other areas of the nervous system and to die
defining Th2 cytokine. IL-12 is implicated in from the neurological disorder rather than pro-
driving responses toward Th1 cytokine patterns: gression of the cancer. (Dalmau, 405408; Rees,
IFN-, IL-2, granulocyte-macrophage colony- 633637)
stimulating factor (GM-CSF), IL-12 itself, and
the B cells and macrophage cytokines. IL-4 25. (C) SCLC is by far the tumor most commonly
implicated in Th2 cytokine patterns switch: associated with paraneoplastic encephalomyelitis
IL-3, IL-5, IL-10, and IL-13. (Ransohoff, 1314) (PEM). Nearly all patients display signs and
symptoms of multifocal involvement of the CNS
22. (E) Experimental autoimmune encephalitis may and dorsal root ganglia. The most common clini-
pursue a relapsingremitting course. The remis- cal manifestation of PEM is a disabling subacute
sion may be caused by natural termination of the sensory neuronopathy (SSN). A high percentage
T-cell response through apoptosis or the action of patients with PEM/SSN have polyclonal IgG
of regulatory cytokines, including upregulation anti-Hu antibodies. These antibodies produce dif-
of Th2 cytokines such as IL-4. (Ransohoff, 24) fuse staining of the nuclei and, to a lesser degree,
the cytoplasm of all neurons in the brain, spinal
23. (C) Proinflammatory cytokines include IL-1,, cord, dorsal root ganglia, and autonomic gan-
IL-2, IL-3, IL-6 (which has pro- and anti- glia.
inflammatory proprieties), IL-12, TNF-, IFN- Ninety percent of patients with paraneo-
, LT-, G-CSF, and GM-CSF. Anti-inflammatory plastic cerebellar degeneration (PCD) have
cytokines include IL-4, IL-10, IL-13, IFN-, and SCLC, Hodgkin lymphoma, or carcinomas of
TGF-. (Ransohoff, 37) the breast, ovary, or female genital tract. Patients
typically have a subacute onset and progression
24. (C) A new family of paraneoplastic antigens of pancerebellar dysfunction. In addition to the
(the Ma proteins) has recently been identified. cerebellar deficits, many patients show symp-
There are at least five Ma proteins, the best char- toms or signs of multifocal PEM, including
Answers: 2028 315

lethargy, cognitive deterioration, bulbar palsy, 27. (D) Neuromyotonia arises from peripheral
and limb weakness. nerve hyperexcitability. Patients present with
The most prevalent autoantibodies in patients muscle twitching and myokymiaa continu-
with PCD are high-titer and polyclonal IgG anti- ous undulating, rippling of the muscles
Purkinje cell antibodies (also called anti-Yo anti- described as a bag of worms. Other features
bodies). Anti-Ri antibodies are seen in paraneo- include stiffness, painful cramps worsened by
plastic opsoclonus. SCLC and breast carcinoma attempted muscle contraction, hyperhidrosis,
together account for approximately 70% of adults muscle hypertrophy, and pseudomyotonia, a
with paraneoplastic opsoclonus. Antiamphiphysin myotonic-like slow relaxation of muscle after
antibodies have been detected in the serum and voluntary contraction without percussion
CSF of a few patients with SCLC and PEM mainly myotonia. Neuromyotonia on needle EMG is
manifesting as SSN without rigidity. demonstrated by spontaneous, continuous,
LambertEaton myasthenic syndrome high- frequency (150 to 300 Hz) doublet, triplet,
(LEMS) occurs in around 2 of every 1,000 cancer or multiplet single motor unit discharges.
patients and is characterized by limb weakness, Acquired neuromyotonia is considered an auto-
usually of the lower limbs; it is commonly asso- immune or paraneoplastic syndrome. Antibodies
ciated with autonomic dysfunction. The deep to voltage-gated potassium channel antibody
tendon reflexes are reduced but show facilitation (VGKC) are often present and seem to result in
after exercise. Sixty percent of all cases are asso- motor nerve hyperexcitability and increased
ciated with underlying malignancy; in 40%, the acetylcholine release, acting at the distal
LEMS occurs as an autoimmune condition in its motor nerve, terminal arborization, or both.
own right. Nonparaneoplastic cases of LEMS Paraneoplastic neuromyotonia primarily
occur more commonly in middle-aged women. occurs with thymoma and occasionally with
When cancer is identified, it is usually SCLC, MG. It is also seen in SCLC and rarely in
although cancer of the prostate or cervix has Hodgkin disease. VGKC-Abs are frequently
been described. Antibodies against voltage-gated present, but neuomyotonia is also reported
calcium channels are present in most patients. with anti-Hu antibodies and SCLC. (Toothaker,
Anti-MAG (myelin-associated glycoprotein) anti- 2133)
bodies have been seen in Waldenstrm
macroglobulinemia and are associated with 28. (E) Opsoclonusmyoclonus (OM) comprises
peripheral neuropathy. Immunoglobulin M anti- myoclonic jerks of the limbs and trunk, with
bodies seem to have a higher pathogenicity for opsoclonusinvoluntary, arrhythmic, high-
polyneuropathy than immunoglobulin G or amplitude, multidirectional saccades. Opsoclonus
immunoglobulin A antibodies. (Dropcho, 246261) may be constant, even during sleep, and may
cause oscillopsia or blurring and oscillation of
26. (B) Myasthenia gravis (MG) is associated with vision. OM is often associated with cerebellar
three types of acetycholine receptor antibodies: ataxia, most often in children; it is commonly
binding, blocking, and modulating. The binding referred to as OMataxia syndrome, although
antibody test is the most sensitive; 90% of patients adult forms exist. Pediatric OM etiology is
with generalized MG and 50% of those with ocu- diverse, including para- and postinfectious, toxic,
lar myasthenia have positive tests. If the test and paraneoplastic causes. Pediatric OM is para-
yields a negative result, then an acetycholine neoplastic in 40% of patients, always associated
receptormodulating antibody may increase the with neuroblastoma. Some 2% to 3% of neurob-
diagnostic yield slightly; however, the test suffers lastoma patients develop OM. Among adults,
higher rates of false positives. Acetycholine recep- paraneoplastic OM and anti-Ri antibodies are
torblocking antibodies do not help in making associated, usually in women with underlying
the diagnosis of MG because they are found in breast cancer. The Ri antibody recognizes the
only 1% of MG patients without acetycholine RNA-binding protein Nova, which is strictly neu-
receptor binding antibodies, making them of lim- ron-specific and may regulate neuronal RNA
ited diagnostic utility. (Kusner, 231239) metabolism. (Toothaker, 2133)
316 9: Neuroimmunology

29. (E) In approximately 50% of patients with neu- whom antibodies are detected, anti-Yo is the
ropathy and IgM monoclonal gammopathy, the most common antibody in PCD. With anti-Yo,
M-protein reacts with the myelin-associated gly- cerebellar symptoms are usually present in iso-
coprotein (MAG) and other glycoconjugates in lation, often leaving patients with significant
nerves bearing the carbohydrate epitope HNK-1. long-term disability resulting from irreversible
High titers of anti-MAG IgM antibodies are Purkinje cell destruction. (Toothaker, 2133)
almost invariably associated with a chronic,
slowly progressive, predominantly sensory 33. (C) Limbic or brainstem encephalitis may be
demyelinating neuropathy. (Nobile-Orazio, 710717; seen with testicular cancer and is associated with
Gondim, 902904; Ropper, 16011605) the synthesis of anti-Ma2 antibodies. (Ransohoff,
94104)
30. (A) Anti-CV2 is seen in thymoma or SCLC, caus-
ing sensory neuronopathy or encephalomyelitis. 34. (A) LambertEaton myasthenic syndrome is
(Ransohoff, 94104) seen in SCLC and is associated with the synthe-
sis of anti-P/Q type voltage-gated calcium chan-
31. (C) Stiff-man syndrome has both a nonparane- nel antibodies. (Ransohoff, 94104)
oplastic and paraneoplastic variant, both auto-
immune. Glutamic acid decarboxylase (GAD) 35. (B) Anti-Yo antibodies are markers of paraneo-
antibodies are associated with the nonparaneo- plastic cerebellar degeneration. The associated
plastic variant, usually in patients with other tumors include ovarian cancer (60%) and other
autoimmune diseases, particularly the diabetes gynecologic tumors (5%), breast cancer (30%),
mellitus type. Paraneoplastic stiff-man syn- and other cancers (5% lung and bladder). Low
drome is clinically similar to the nonparaneo- titers of anti-Yo antibodies may also be detected
plastic form, although arm involvement may be in less than 5% of patients with ovarian cancer
more prominent. Amphiphysin antibodies are without neurological symptoms. The target
most commonly detected in paraneoplastic stiff- antigens of the anti-Yo antibodies are several
man syndrome, usually in breast cancer patients, 34- and 62-kDa proteins expressed predomi-
although SCLC and Hodgkin disease are also nantly in the cytoplasm of Purkinje cells of the
reported. (Toothaker, 2133) cerebellum and to a lesser degree in neurons of
the molecular layer and large neurons in the
32. (D) Paraneoplastic cerebellar degeneration brainstem. (Ransohoff, 102)
(PCD) was the first paraneoplastic syndrome to
be recognized. Cerebellar symptoms, including 36. (D) Paraneoplastic encephalomyelitis (PEM) is
truncal and appendicular ataxia, nystagmus, characterized by neuronal loss and inflammatory
and dysarthria begin abruptly, progress over infiltrates in multiple areas of the nervous system.
weeks to months, and then stabilize, usually PEM antedates the diagnosis of cancer, almost
leaving the patient significantly impaired, always a SCLC; the majority of patients harbor
unable to walk or sit unassisted, and unable to serum and CSF Hu (ANNA-1) antibodies and,
perform fine motor tasks such as writing or eat- less frequently, CV2 (CRMP5), amphiphysin, Ri
ing. The degree and probability of severe impair- (ANNA-2), and other less well characterized
ment correlates somewhat with the underlying onconeural antibodies. PEM may present with
cancer and type of antineuronal antibody pres- classic neurological syndromes, such as limbic
ent. Malignancies commonly associated with encephalitis, or less frequently with symptoms
PCD include gynecologic cancers, such as breast that do not initially raise the suspicion of a para-
and ovarian, SCLC, and Hodgkin disease. Breast neoplastic syndrome. In this setting, the detec-
or gynecological cancer is usually detected with tion of onconeural antibodies is crucial to make
anti-Yo positivity. It targets the cdr2 antigen, the diagnosis. Patients with PEM and Hu anti-
normally expressed on Purkinje cells in the cere- bodies may present with epilepsia partialis con-
bellum and aberrantly expressed in ovarian and tinua or nonconvulsive status epilepticus as the
breast cancers. Present in 38% of patients in first and predominant manifestation. Acquired
Answers: 2940 317

central hypoventilation leading to loss of auto- are encoded by different genes. Ma1 is expressed
matic respiration with preserved voluntary in brain and testis, while Ma2 is expressed only
breathing (Ondines curse) occurs with in the brain. Antibodies that react with both Ma1
medullary lesions. Less than 5% of PEM patients and Ma2 are called anti-Ma. Anti-Ma antibodies
with Hu antibodies never develop cancer after are associated predominantly with brainstem
long-term follow up. Whether PEM patients and cerebellar dysfunction and several types of
without detectable cancer represent true para- cancer, including lung, breast, colon, and parotid
neoplastic cases in which the tumor is destroyed glands. Antibodies that react only with Ma2 are
by the immune response is presently unclear. called anti-Ma2 or anti-Ta. The detection of anti-
(Graus, 732737) Ma2 antibodies is usually associated with limbic
and brainstem encephalitis; 80% of these
37. (A) Anti-Hu antibodies are markers of parane- patients have germ cell tumors and 20% have
oplastic encephalomyelitis, sensory neuronopa- other tumors, including lung and breast can-
thy, and autonomic dysfunction. The detection cers. Anti-CV2 antibodies are associated with
of these antibodies in patients with focal symp- paraneoplastic cerebellar degeneration and
toms, such as limbic encephalopathy or cere- encephalomyelitis. The causal tumors are SCLC
bellar dysfunction, indicates that although these and thymoma. The target antigen of anti CV2
areas are the main targets of the immune antibodies is a set of 62- to 66-kDa proteins
response, the neuropathological substrate is a expressed in neurons and oligodendrocytes. The
more diffuse encephalomyelitis. Since in 80% of detection of antiamphiphysin in patients with
patients with an anti-Huassociated syndrome neurological symptoms of unknown cause
the causal tumor is a SCLC, a chest CT scan is is suggestive of a paraneoplastic origin. Antia-
mandatory for patients with anti-Hu antibodies mphiphysin antibodies may be seen in breast
and an as yet undiagnosed cancer. Rarely, other cancer and SCLC. Amphiphysin is a major anti-
tumors have been found to be associated with gen associated with paraneoplastic stiff-man
anti-Hu antibodies, including breast cancer, syndrome, although some patients develop
prostate cancer, neuroblastoma, and small cell paraneoplastic encephalomyelitis and sensory
cancer of unknown origin. The targets antigens neuronopathy. Anti-Tr antibodies are associated
of the anti-Hu antibodies are a family of neu- with paraneoplastic cerebellar degeneration and
ronal specific RNA-binding proteins expressed Hodgkin lymphoma. In adult brain, the Tr anti-
predominantly in the nuclei of neurons of the gen is expressed predominantly in the Purkinje
central and peripheral nervous system. cell cytoplasm and dendrites. Antiacetylcholine
(Ransohoff, 102) receptors antibodies are associated with thy-
moma. (Ransohoff, 103)
38. (D) Anti-Ri antibodies are associated with para-
neoplastic cerebellar and brainstem encephalopa- 40. (E) Paraneoplasticlimbic encephalitis is a dis-
thy characterized by opsoclonus and other abnor- order characterized by the subacute develop-
malities of ocular motility. The most commonly ment of depression, irritability, seizures, and
associated tumor is breast cancer. The anti-Ri anti- short-term memory loss. Symptoms usually pre-
bodies react with neuronal proteins located in cede or lead to the diagnosis of the primary
the nuclei of neurons in the CNS but not in the tumor. Typical MRI findings of paraneoplastic
peripheral nervous system. (Ransohoff, 103) limbic encephalitis include uni- or bilateral
mesial temporal lobe abnormalities that are best
39. (E) Anti-Ma and anti-Ma2 antibodies are mark- seen on T2-weighted images.
ers of paraneoplastic syndromes involving the The tumor most frequently involved is lung
limbic region, brainstem, and cerebellum. The cancer. Other tumors include germ-cell tumors of
target antigens are a family of brain cancer tes- the testis, breast cancer, thymoma, and immature
ticular proteins that include Ma1, Ma2, and sev- teratoma of the ovary. Antineuronal antibodies
eral other uncharacterized members. These pro- associated with limbic encephalitis include
teins are highly homologous to each other and anti-Hu, anti-Ma2, anti-CV2, and anti-PCA2
318 9: Neuroimmunology

antibodies. Pathological findings include lymphoma may cause the production of anti-Tr
perivascular and interstitial inflammatory infil- antibodies, leading to cerebellar degeneration
trates, neuronal loss, and microglial proliferation in relatively young patients. Improvement of
that predominates in the limbic system. symptoms may result from treatment of the
Neurological symptoms usually develop over lymphoma. (Ransohoff, 107109)
days or weeks; they then stabilize, leaving the
patient with severe short-term memory loss. In 42. (C) The patient described in this vignette has
contrast to other paraneoplastic diseases of the symptoms highly suggestive of stiff-man syn-
CNS, this disorder may improve with treatment drome with sphincter dysfunction. The syndrome
of the tumor. (Ransohoff, 107) may be idiopathic or a paraneoplastic manifesta-
tion of breast, colon, and Hodgkin lymphoma.
41. (D) The patient described in this vignette has When the syndrome is not associated with a can-
symptoms of cerebellar and mesencephalic dys- cer, the major autoantigen is GAD and around
function. The presence of antineuronal antibod- 70% of patients develop diabetes. When the syn-
ies is suggestive of paraneoplastic syndrome drome is caused by paraneoplastic manifestation
affecting the brainstem and cerebellum. Anti- of a cancer, antiamphiphysin antibodies are often
neuronal antibodies associated with paraneo- found in the serum and the CSF of affected
plastic cerebellar dysfunction include anti-Ri, patients. The use of clonazepam or diazepam may
anti-Tr, anti-Yo, anti-Ma, anti-CV2, anti-GluR1, improve the rigidity. Some authors include the
and anti-PCA2 antibodies. A number of clinical improvement of rigidity with the use of diazepam
immunology correlates have been suggested for as the diagnostic criteria to maintain stiff-man
some of these antibodies. syndrome as the diagnosis of such rigidity. The
The patient in this case has distinctive clin- treatment of the primary tumor, as well as the use
ical findings suggestive of an association with of steroids, may cause definitive improvement of
anti-Ri antibodies. Up to 75% of patients with the stiff-man syndrome. (Ransohoff, 113)
anti-Ri antibodies have opsoclonus, ocular flut-
ter, and dysmetria, the latter two developing 43. (A) The improvement of symptoms of acute
when the opsoclonus subsides. Patients may inflammatory demyelinating polyneuropathy
also develop nystagmus and abnormal visual (AIDP) by plasmapheresis, the presence of cir-
tracking. Ataxia predominates in the trunk and culating antibodies directed against peripheral
may cause severe gait difficulty and multiple nerve antigens, and the deposition of immuno-
falls. Treatment of the primary tumor, which is globulins and complements in the myelinated
usually a breast cancer, or the use of immune fibers are highly suggestive of humoral factors
suppression may result in neurological improve- involved in the pathogenesis of this polyneu-
ment. Pathological examination may show ropathy. Several circulating antibodies against
perivascular and interstitial inflammatory infil- myelin have been found in patients with AIDP.
trates involving the tegmentum of the pons and The anti-GM1 antibody is the most frequent
mesencephalon, with extensive degeneration of antiganglioside antibody detected in serum of
cerebellar Purkinje cells. patients with AIDP. Some authors reported
Patients with paraneoplastic cerebellar increased titers of anti-GD1a in the axonal form
degeneration and anti-Yo antibodies generally of AIDP. The anti-GM1 antibody has been
present with progressive disabling cerebellar reported in the motor form of AIDP as well as in
syndrome over a few days or weeks. The most acute motor axonal polyneuropathy. Anti-GQ1
frequent cause of this syndrome is ovarian and antibodies are invariably associated with the
other gynecological cancers, followed by breast MillerFisher variant of AIDP. Antiglycolipide
cancer. Treatment of the primary tumor rarely antibodies have been associated with AIDP,
improves the cerebellar symptoms. Cerebellar including antibodies against Campylobacter
syndrome associated with anti-Hu antibodies is jejuni, Mycoplasma pneumoniae, Haemophilus
generally caused by SCLC. Treatment of the influenzae, cytomegalovirus, and Epstein-Barr
tumor may improve the symptoms. Hodgkin virus. (Ransohoff, 126)
Answers: 4149 319

44. (A) The therapeutic effect of intravenous sion mediated by voltage-gated potassium chan-
immunoglobulin on demyelinating polyneu- nel (VGCC) antibodies. The predominant initial
ropathy has been established. The suggested symptoms include proximal weakness of the
mechanisms are downregulation of Th2 cytokine lower limbs, dry mouth, and transient ptosis.
production, immunoglobulin Fc receptors block- SCLC is detected in up to 50% of LEMS patients.
ade, inhibition of T-cell activation, nonspecific At present, there are no biological markers that
binding of activated complement, and anti- can determine which LEMS patients are parane-
idiopathic suppression of autoantibodies. oplastic. However, a previous study conducted
(Ransohoff, 128) by Graus and colleagues showed that 43% of
patients with LEMS and SCLC had an antibody,
45. (C) Anti-N-methyl-D-aspartate (NMDA) recep- called antiglial nuclear antibody (AGNA), defined
torassociated encephalitis is a recently described by the immunoreactivity with the nuclei of the
disorder that usually affects young women with Bergmann glia of the cerebellum. While the fre-
teratoma of the ovary. A few days after pro- quency of AGNAs was higher than expected in
dromic fever or headache, most patients develop LEMS patients with SCLC, no patient with idio-
a syndrome that predictably evolves in stages, pathic LEMS had these antibodies. (Graus, 732737)
including prominent psychiatric symptoms (agi-
tation, delusional thoughts, hallucinations) or, 48. (C) Inclusion body myositis (IBM) is the most
less frequently, short-term memory loss, seizures, common inflammatory myopathy after the age of
progressive unresponsiveness (catatonia-like 50 years. It is characterized by an insidious onset
stage), central hypoventilation, autonomic insta- of asymmetric weakness involving the quadri-
bility (fluctuations of blood pressure, tempera- ceps, volar forearm muscles, and ankle dorsi-
ture, and cardiac rhythm), orofacial dyskinesias, flexors. Up to 25% of patients with IBM have an
limb choreoathetosis, and dystonia. The tumor associated autoimmune disease, but there is no
(mature or immature teratoma) is often missed or increased association with malignancy or lung
mistaken for a benign or physiological cyst of and heart abnormalities. IBM affects men three
the ovary. Detection of CSF and serum antibod- times more often than women, whereas non-
ies to NR1/NR2 heteromers of the NMDA recep- paraneoplastic varieties of dermatomyositis and
tor is characteristic of this disorder. The disorder polymyositis affect women twice as often as men.
can also occur in men or women without a Paraneoplastic dermatomyositis, however, is
detectable tumor. (Graus, 732737) slightly more frequent in men than women.
Polymyositis is associated with the diagnosis of
46. (B) Opsoclonus is a disorder of the ocular motil- malignancy in up to 28% of patients; dermato-
ity characterized by irregular, continuous, large- myositis is associated with malignancy in up to
amplitude conjugate saccades in all directions of 45% of cases. In IBM, pathological examination
gaze. Paraneoplastic opsoclonusmyoclonus may shows irregular rimmed vacuoles in up to 70% of
be the presenting symptom in about 2% to 5% of muscle fibers. Eosinophilic inclusions are found
children with neuroblastoma. In adults, parane- in the cytoplasm and nuclei, and CD8 T-cell
oplastic opsoclonus usually affects women with endomysial infiltrate may be seen in the muscle
breast or fallopian cancer in association with Ri fibers. In dermatomyositis, pathological find-
antibodies and patients with SCLC without any ings are characterized by perivascular and peri-
characteristic antibodies. Children with idio- fascicular infiltrates, predominantly formed by
pathic or paraneoplastic opsoclonusmyoclonus B lymphocytes, macrophages, and CD4 T cells.
syndrome develop antibodies against unknown Early and prominent capillary changes in der-
membrane antigens of neuroblastoma cell lines matomyositis suggest the importance of humoral
and cerebellar granular neurons detected by flow factors in the pathogenesis of the disease.
cytometry. (Graus, 732737) (Ransohoff, 135138)

47. (A) LambertEaton myasthenic syndrome 49. (C) The McDonald criteria allow for the diag-
(LEMS) is a disorder of neuromuscular transmis- nosis of MS based on clinical presentation alone
320 9: Neuroimmunology

if multiple attacks accompanied by clinical evi- seronegative. In addition, some investigators


dence of at least two lesions can be identified. have found that EBV antibody titers are higher
The definition of an attack is generally accepted during MS relapses. Among individuals
to be the development of neurologic symptoms seropositive for EBV capsid antigen, almost
likely caused by an inflammatory demyelinating three times more patients with MS recalled
lesion, lasting at least 24 hours, and supported having had clinical infectious mononucleosis.
by objective findings. When there are two or Furthermore, patients who report having had
more attacks with objective clinical evidence of infectious mononucleosis before age 18 have an
two or more lesions, there is no need for addi- eightfold higher risk of MS than those who did
tional data when the McDonald criteria are used. not recall having had infectious mononucleosis.
Although the patient in this vignette does not (Green, 6385)
need additional tests to confirm her diagnosis,
extreme caution is needed before making the 52. (A) Rituximab is a mousehuman chimeric anti-
diagnosis of MS if tests such as head MRI and body directed against the CD20 precursors.
CSF examination are undertaken and are nega- It induces antibody-dependent cell- and
tive. Since many patients early in the disease complement-mediated cytotoxicity in these cells
course may not meet the strict clinical defini- and therefore prevents the formation of new
tion for MS, the McDonald criteria allow for the antibody-secreting cells and reduces titers of
use of ancillary testing such as MRI, CSF analy- autoantibodies. (Onrust, 7988)
sis, and visual evoked potentials to satisfy the
requirements for dissemination in space and 53. (D) The mechanism of action of IFN- in MS is
time. (Rinker, 1334) not completely understood. IFN- receptor
binding induces the expression of numerous
50. (A) The HLA-DRB1 gene located within the proteins (including neopterin, 2-microglobulin,
major histocompatibility complex (MHC) super- MxA, IL-10) responsible for the pleiotropic
locus on chromosome 6p21 is the strongest bioactivities of IFN-. Immunomodulatory
genetic factor identified as influencing MS sus- effects of IFN- include the enhancement of
ceptibility. The association of MS with HLA suppressor T-cell activity, reduction of proin-
genes, specifically DRB1*1501 allele, has been a flammatory cytokine production, downregula-
consistent finding across nearly all populations. tion of antigen presentation, inhibition of lym-
Studies in multicase families confirm the known phocyte trafficking into the CNS, and reduction
association with the HLA class II DR2 haplotype of matrix metalloproteinase production.
(HLA-DRB1*1501-DQA1*0102-DQB1*0602), pri- IFN- is a polypeptide, normally produced
marily in populations of northern European by fibroblasts, that has antiviral and antiprolif-
descent. The exact mechanisms by which the erative effects. Binding of IFN- to its receptor
DRB1 gene influences susceptibility to MS remain induces a complex transcriptional response. In
undefined but are likely related to the physio- immune cells (the most likely target of IFN-s
logical function of HLA molecules, including therapeutic effect in MS), IFN- reduces anti-
antigen binding and presentation, and T-cell gen presentation and T-cell proliferation, alters
repertoire determination by negative selection of cytokine and matrix metalloproteinase (MMP)
high-avidity autoreactive T cells within the expression, reduces proinflammatory cytokine
embryonic thymic environment. (Oksenberg, production, inhibits lymphocyte trafficking into
375387; Rinker, 1334) the CNS, and restores suppressor function.
(Markowitz, S8S11)
51. (B) Among possible infectious agents suggested
to play a role in MS pathogenesis, EpsteinBarr 54. (E) Mitoxantrone is an anthracenedione com-
virus (EBV) currently appears the most likely to pound that intercalates with DNA and modu-
play a contributory role. First, almost all patients lates the immune system through a variety of
with MS are seropositive for EBV antibodies, mechanisms. It has potent anti-inflammatory
while 5% to 10% of the general population is and immunomodulating properties. It suppresses
Answers: 5059 321

both B and T lymphocytes and has more effects antibodies and is seen in Hodgkins disease as
on helper subsets than on suppressor subsets, well as in breast and colon cancers. Paran-
resulting in a downregulation of the inflamma- eoplasticlimbic encephalitis is associated with
tory cascade. Effects on B cells lead to a decrease the synthesis of anti-Hu, anti-MA-2, anti-CV2,
in the rate and magnitude of B-cell function, and anti-PCA2 antibodies and is seen in SCLC
thereby decreasing antibody formation. In addi- and germ cell tumor of testis. Brainstem encephali-
tion, mitoxantrone has a marked suppressive tis may be seen in testis cancer, whereas cere-
effect on macrophage function. Because bellar degeneration is associated with anti-Yo
macrophages are found in large numbers in antibodies and is seen in breast and ovarian can-
acute lesions, their suppression may be associ- cers. Motor neuron syndrome is rarely reported
ated with a decrease in the extent of tissue dam- to be associated with malignancies. (Ransohoff,
age caused by inflammation. (Jeffery, 1924) 94113)

55. Natalizumab (Tysabri) is a humanized mono- 58. (B) Limbic encephalitis is the most consistent
clonal antibody specifically designed for use in paraneoplastic disorder associated with MRI
MS. Natalizumab binds to the 4 subunit of 41 abnormalities. On T2-weighted images, abnor-
and 47 integrins that are present on most mal signals may be seen in the mesiotemporal
leukocytes other than neutrophils. Antibody lobes unilaterally or bilaterally. On T1 sequences,
binding blocks the interaction with comple- the temporal limbic regions may be hypointense
mentary endothelial molecules known as vas- and atrophic and may sometimes enhance with
cular cell adhesion molecules and thereby contrast injection. (Ransohoff, 106107)
inhibits the migration of leukocytes across the
bloodbrain barrier. (Mehta, 144180) 59. (E) Postinfectious encephalomyelitis is an acute
disseminated encephalomyelitis and a monopha-
56. (A) Neuromyelitis optica (NMO) is a severe sic polyregional syndrome; it is temporally
autoimmune inflammatory demyelinating dis- related to an infection or vaccination and is most
ease of the CNS that predominantly affects the common in children. Compared with MS,
spinal cord and optic nerves. The recent descrip- patients with postinfectious encephalomyelitis
tion of NMO-IgG, a highly disease-specific have a monophasic course and 70% have reported
autoantibody found in NMO and NMO-related a precipitating event in the weeks preceding the
diseases (i.e., relapsing optic neuritis and longi- acute phase, whereas in MS the time course of
tudinally extensive transverse myelitis) but the disease is multiphasic and preceding events
absent in the classic form of MS, convincingly are uncommon. MS most commonly affects
demonstrated that NMO is a specific disease young adults, with female predominance,
and not just a subtype of MS. In contrast to pro- whereas postinfectious encephalomyelitis
totypical MS, NMO active lesions displayed affects both male and female children equally.
perivascular immunoglobulin deposition, com- Clinically, postinfectious encephalomyelitis
plement activation, and polymorphonuclear has an abrupt onset. Bilateral optic neuritis is more
(neutrophils and eosinophils) infiltration, thus commonly seen than unilateral optic neuritis,
supporting the importance of antibody-medi- seizures are seen in 50% of patients, and the level
ated pathogenesis in NMO. of consciousness is frequently affected. Complete
The target antigen of NMO-IgG was identi- transverse myelitis with areflexia is seen more in
fied as aquaporin-4 (AQP4), the main water postinfectious encephalomyelitis than in the
channel protein in the CNS, expressed on astro- incomplete transverse myelitis more frequently
cyte end-feet at the bloodbrain barrier (BBB) seen in MS. The disease onset of MS has a suba-
and the brainCSF barrier. (Lennon, 473477; cute pattern: seizures are seen in less than 5% of
Saikali, 132135) cases, the level of consciousness is generally con-
served, and optic neuritis occurs unilaterally
57. (D) Paraneoplastic stiff-man syndrome is asso- rather than bilaterally. Head MRI commonly
ciated with the production of antiamphiphysin shows a conservation of the periventricular
322 9: Neuroimmunology

area in postinfectious encephalomyelitis, whereas Kusner LL, Puwanant A, Kaminski HJ. Ocular myasthenia:
in MS, a periventricular area is frequently diagnosis, treatment, and pathogenesis. Neurologist.
affected. Increased cell count may be seen in 2006;12:231-239.
both postinfectious encephalomyelitis and MS, Lennon VA, Kryzer TJ, Pittock SJ, Verkman AS, Hinson SR.
whereas oligoclonal bands are more commonly IgG marker of opticspinal multiple sclerosis binds to the
aquaporin-4 water channel. J Exp Med. 2005;202(4):473-477.
seen in MS. (Burks, 91)
Leocani L, Comi G. Neurophysiological investigations in
multiple sclerosis. Curr Opin Neurol. 2000;13:255261.
60. (D) Transverse myelitis may be an isolated clin- Markowitz CE. Interferon-beta: mechanism of action and
ical syndrome where the affected patient has an dosing issues. Neurology. 2007;68:S8S11.
increased risk of developing MS. Certain fea- Mehta LR, Goodman AD. Disease-Modifying Therapies.
tures of transverse myelitis are helpful in pre- Continuum: Lifelong Learning in Neurology. 2007;13(5):
dicting the likelihood of MS. Complete trans- (Multiple Sclerosis)144-180.
verse myelitis carries a risk lower than 14%, Nobile-Orazio E, Meucci N, Baldini L, Di Troia A, Scarlato
whereas incomplete transverse myelitis carries G. Long-term prognosis of neuropathy associated with
a risk that approximates 70%. Other features anti-MAG IgM M-proteins and its relationship to
that increase the risk of developing MS after immune therapies. Brain. 2000;123( Pt 4):710-717.
transverse myelitis include asymmetric sensory Noseworthy JH, Lucchinetti C, Rodriguez M, Weinshenker
BG. Multiple sclerosis. N Engl J Med. 2000;343:938952.
or motor findings, abnormal CSF or brain MRI
Oksenberg JR, Barcellos LF. Multiple sclerosis genetics:
findings, spinal MRI showing limited noncon-
leaving no stone unturned. Genes Immunol. 2005;6:375-
fluent intramedullary lesions, and abnormal 387.
multimodality-evoked potentials. (Burks, 93) Onrust SV, Lamb HM, Balfour JA. Rituximab. Drugs. 1999;
58(1):79-88; discussion 9-90.
Polman CH, Uitdehaag BM. Drug treatment of multiple
sclerosis. BMJ. 2000;321:490-494.
REFERENCES Ransohoff R. Neuroimmunology. Continuum. 2001;7(3):
3-145.
Arroyo EJ, Scherer SS. On the molecular architecture of Rees J. Paraneoplastic syndromes. Curr Opin Neurol.
myelinated fibers. Histochem Cell Biol. 2000;113:1-18. 1998;11:633-637.
Bjartmar C, Trapp BD. Axonal and neuronal degeneration Rinker JR II, Cross AH. Diagnosis and differential diagno-
in multiple sclerosis: mechanisms and functional conse- sis of multiple sclerosis. Continuum: Lifelong Learning in
quences. Curr Opin Neurol. 2001;14:271-278. Neurology. 2007;13(5):(Multiple Sclerosis)13-34.
Burks JS, Johnson KP. Multiple Sclerosis: Diagnosis, Medical Ropper AH, Gorson KC. Neuropathies associated with
Management, and Rehabilitation. New York: Demos Medical, paraproteinemia. N Engl J Med. 1998;38:1601-1605.
2000. Saikali P, Cayrol R, Vincent T. Anti-aquaporin-4 auto-
Dalmau JO, Posner JB. Paraneoplastic syndromes. Arch antibodies orchestrate the pathogenesis in neuromyelitis
Neurol. 1999;56:405-408. optica. Autoimmun Rev. 2009;9(2):132-135.
Dropcho EJ. Principles of paraneoplastic syndromes. Ann Stolp-Smith KA, Carter JL, Rohe DE, Knowland DP III.
N Y Acad Sci. 1998;841:246-261. Management of impairment, disability, and handicap
Gondim FA, De Sousa EA, Latov N, Sander HW, Chin RL, due to multiple sclerosis. Mayo Clin Proc. 1997;72:1184-
Brannagan TH. Anti-MAG/SGPG associated neuropa- 1196.
thy does not commonly cause distal nerve temporal dis- Toothaker TB, Rubin M. Paraneoplastic neurological syn-
persion. J Neurol Neurosurg Psychiatry. 2007;78:902-904. dromes: a review. Neurologist. 2009;15:21-33.
Graus F, Dalmau J. Paraneoplastic neurological syndromes: Trapp BD, Peterson J, Ransohoff RM, et al. Axonal tran-
diagnosis and treatment. Curr Opin Neurol. 2007;20:732- section in the lesions of multiple sclerosis. N Engl J Med.
737. 1998;338:278-285.
Green A, Waubant E. Genetics and epidemiology of mul- Wingerchuk DM, Lucchinetti CF, Noseworthy JH. Multiple
tiple sclerosis. Continuum: Lifelong Learning in Neurology. sclerosis: current pathophysiological concepts. Lab Invest.
2007;13(5):(Multiple Sclerosis)63-85. 2001;81:263-281.
Jeffery DR, Herndon R. Review of mitoxantrone in the Zifko UA, Rupp M, Schwarz S, Zipko HT, Maida EM.
treatment of multiple sclerosis. Neurology. 2004;63: Modafinil in treatment of fatigue in multiple sclerosis.
S19-S24. Results of an open-label study. J Neurol. 2002;249:983-987.
CHAPTER 10

Neuropharmacology and Neurochemistry


Questions

1. Which of the following substances is not an 5. Which of the following drugs blocks postsy-
amino acid or biogenic amine neurotransmitter? naptic nicotinic cholinergic receptors?
(A) Dopamine (A) Curare
(B) Acetylcholine (B) Soman
(C) Histamine (C) Dimethylphenyl piperazinium
(D) Glycine (D) Oxotemorine
(E) Epinephrine (E) Vesamicol

2. Which of the following drugs blocks the trans- 6. Which of the following drugs is a nicotinic
port of acetylcholine into vesicles? receptor agonist?
(A) Curare (A) Curare
(B) Soman (B) Soman
(C) Dimethylphenyl piperazinium (C) Dimethylphenyl piperazinium
(D) Oxotemorine (D) Oxotemorine
(E) Vesamicol (E) Vesamicol

3. Which of the following drugs promotes the 7. Which of the following drugs is a muscarinic
release of acetylcholine from vesicles? receptor antagonist?
(A) Atropine (A) Atropine
(B) Physostigmine (B) Physostigmine
(C) Hemicolinium-3 (C) Hemicolinium-3
(D) Botulinum toxin (D) Botulinum toxin
(E) Beta-bungarotoxin (E) Beta-bungarotoxin

4. Which of the following drugs blocks the release 8. Which of the following drugs is a presynaptic
of acetylcholine from vesicles? muscarinic agonist?
(A) Atropine (A) Curare
(B) Physostigmine (B) Soman
(C) Hemicolinium-3 (C) Dimethylphenyl piperazinium
(D) Botulinum toxin (D) Oxotemorine
(E) Beta-bungarotoxin (E) Vesamicol

323
324 10: Neuropharmacology and Neurochemistry

9. Which of the following drugs is a reversible (C) the conduction of each open channel
acetylcholinesterase inhibitor? (D) the driving force that acts on ions
(A) Atropine (E) the total number of endplate channels
(B) Physostigmine
15. Which of the following blocks the action of
(C) Hemicolinium-3
gamma aminobutyric acid (GABA) at postsy-
(D) Botulinum toxin naptic receptors?
(E) Beta-bungarotoxin
(A) Allylglycine
10. Which of the following drugs is an irreversible (B) Flumazenil
acetylcholinesterase inhibitor? (C) Phenobarbital
(A) Curare (D) Diazepam
(B) Soman (E) Picrotoxin
(C) Dimethylphenyl piperazinium
16. Which of the following is a GABA A agonist at
(D) Oxotemorine postsynaptic receptors?
(E) Vesamicol
(A) Gabaculine
11. Which of the following drugs is a competitive (B) Muscimol
inhibitor of choline uptake? (C) Nipecotic acid
(A) Atropine (D) Baclofen
(B) Physostigmine (E) Phaclofen
(C) Hemicolinium-3
17. Which of the following is a GABA transaminase
(D) Botulinum toxin inhibitor?
(E) -bungarotoxin
(A) Gabaculine
12. The rate-limiting step for the synthesis of (B) Muscimol
dopamine is (C) Nipecotic acid
(A) tyrosine hydroxylase (D) Baclofen
(B) aromatic amino acid decarboxylase (E) Phaclofen
(C) pteridine reductase
18. Which of the following inhibits glutamic acid
(D) dopamine -hydroxylase decarboxylase?
(E) phenylethanolamine-N-methyl
transferase (A) Allylglycine
(B) Flumazenil
13. Serotonin is derived from (C) Phenobarbital
(A) histidine (D) Diazepam
(B) tryptophan (E) Picrotoxin
(C) dopamine
19. Which of the following drugs increases the fre-
(D) tyrosine quency of GABA A receptor opening?
(E) glutamate
(A) Allylglycine
14. Ligand-gated channel opening for acetylcholine (B) Flumazenil
does not depend on (C) Phenobarbital
(A) the value of the membrane potential (D) Diazepam
(B) the probability that the channel is open (E) Picrotoxin
Questions: 928 325

20. Which of the following drugs prolongs the dura- 25. Which of the following statements is true of the
tion of opening of the GABA A receptors? molecular mechanism of cocaine addiction?
(A) Allylglycine (A) Methadone is a powerful medication
(B) Flumazenil against cocaine addiction.
(C) Phenobarbital (B) Cocaine, by blocking the dopamine reup-
(D) Diazepam take transporter, increases the postsynap-
tic concentration of dopamine.
(E) Picrotoxin
(C) The dopamine transporter system is not
21. Which of the following drugs reverses the action necessary for the mechanism of cocaine
of benzodiazepine agonists and has no phar- addiction.
macological effect when administered alone? (D) D1 dopamine agonists stimulate cocaine-
seeking behavior.
(A) Allylglycine
(E) D2 receptor agonists may decrease
(B) Flumazenil episodes of craving for cocaine.
(C) Phenobarbital
(D) Diazepam 26. A 10-year-old boy was treated with ethosux-
(E) Picrotoxin imide for several months because of absence
seizures. Which of the following side effects
22. Which of the following drugs is a GABA B might this patient develop?
receptors agonist? (A) Hyponatremia
(A) Gabaculine (B) Megaloblastic anemia
(B) Muscimol (C) Acute pancreatitis
(C) Nipecotic acid (D) Fatigue
(D) Baclofen (E) Ataxia
(E) Phaclofen
27. A 45-year-old man was treated with phenytoin
23. Which of the following drugs is a GABA B for several years because of a seizure disorder.
receptor antagonist? Which of the following side effects might this
patient develop?
(A) Gabaculine
(B) Muscimol (A) Hyponatremia
(C) Nipecotic acid (B) Megaloblastic anemia
(D) Baclofen (C) Acute pancreatitis
(E) Phaclofen (D) Fatigue
(E) Ataxia
24. Which of the following drugs is a GABA uptake
inhibitor? 28. A 25-year-old man, diagnosed with primary
generalized seizures, was recently switched
(A) Gabaculine to valproic acid. His valproic acid level is
(B) Muscimol 50 g/mL. Which of the following side effects
(C) Nipecotic acid might this patient develop?
(D) Baclofen
(A) Hyponatremia
(E) Phaclofen
(B) Impotence
(C) Acute pancreatitis
(D) Fatigue
(E) Ataxia
326 10: Neuropharmacology and Neurochemistry

29. A 55-year-old man was started on primidone (D) cimetidine


for the treatment of an essential tremor. Which (E) prednisone
of the following side effects might this patient
develop? 34. Which of the following antiepileptic drugs can
(A) Hyponatremia have its concentration increased by the con-
comitant administration of aspirin?
(B) Impotence
(C) Acute pancreatitis (A) Phenobarbital
(D) Megaloblastic anemia (B) Valproic acid
(E) Ataxia (C) Carbamazepine
(D) Lamotrigine
30. A 60-year-old man was started on phenobarbi- (E) Primidone
tal 4 months ago after undergoing brain surgery
for astrocytoma. Which of the following side 35. Which of the following drugs is a tyrosine
effects might this patient develop? hydroxylase inhibitor?
(A) Hyponatremia (A) Pargyline
(B) Impotence (B) Amphetamine
(C) Acute pancreatitis (C) Phentolamine
(D) Dupuytren contracture (D) Tropolone
(E) Ataxia (E) Alpha methyltyrosine

31. A 35-year-old man with a history of complex 36. Which of the following drugs causes norepi-
partial seizure was being treated with carba- nephrine storage depletion?
mazepine. She consulted the neurologist because
of chronic headache and blurred vision. Which (A) Tropolone
of the following side effects might this patient (B) Alpha methyltyrosine
develop? (C) Clonidine
(A) Hyponatremia (D) Desipramine
(B) Impotence (E) Reserpine
(C) Acute pancreatitis
37. Release of norepinephrine is caused by
(D) Dupuytren contracture
(E) Ataxia (A) pargyline
(B) amphetamine
32. Which of the following drugs decrease the (C) phentolamine
serum level of carbamazepine? (D) tropolone
(A) Phenytoin (E) alpha-methyltyrosine
(B) Valproic acid
(C) Verapamil 38. Which of the following drugs is a presynaptic 2
adrenergic autoreceptor stimulator?
(D) Erythromycin
(E) Isoniazid (A) Tropolone
(B) Alpha methyltyrosine
33. Phenytoin may reach a toxic level in the serum (C) Clonidine
of a patient using _________ at the same time. (D) Desipramine
(A) valproic acid (E) Reserpine
(B) ethosuximide
(C) cyclosporine
Questions: 2953 327

39. Which of the following drugs is a postsynaptic 44. For a patient with seizures and acute porphyria,
-adrenergic receptor blocker? the most appropriate antiepileptic drug is
(A) Pargyline (A) felbamate
(B) Amphetamine (B) topiramate
(C) Phentolamine (C) lamotrigine
(D) Tropolone (D) tiagabine
(E) Alpha methyltyrosine (E) gabapentin

40. Which of the following drugs is a norepineph- 45. The mechanism of action of cocaine in the cen-
rine reuptake inhibitor? tral nervous system is
(A) Tropolone (A) inhibition of tyrosine hydroxylase
(B) Alpha methyltyrosine (B) inhibition of the storage of dopamine
(C) Clonidine (C) inhibition of dopamine reuptake
(D) Desipramine (D) inhibition of monoamine oxidase
(E) Reserpine (E) inhibition of catechol-O-methyltrans-
ferase
41. Which of the following drugs is a monoamine
oxidase (MAO) inhibitor? Questions 46 through 53

(A) Pargyline Link the following:


(B) Amphetamine (A) D1 dopamine receptors
(C) Phentolamine (B) D2 dopamine receptors
(D) Tropolone (C) Both
(E) Alpha methyltyrosine (D) Neither

42. Which of the following drugs is a catechol-O-


46. Stimulation of adenylate cyclase.
methyl transferase inhibitor?
(A) Pargyline 47. Enhancement of potassium conductance.
(B) Amphetamine
(C) Phentolamine 48. The number of dopamine receptors increases in
cases of tardive dyskinesia.
(D) Tropolone
(E) Alpha methyltyrosine 49. Postmortem studies have shown an increased
number of dopamine receptors in schizophrenic
43. D1 dopamine receptors exceed the number of patients.
D2 dopamine receptors as well as the number of
other types of dopamine receptors in the 50. It has the highest affinity to quinpirol.
(A) substantia nigra
(B) caudate nucleus 51. It has the highest affinity to clozapine.
(C) hippocampus
52. Bromocriptine is an agonist.
(D) amygdala
(E) ventral tegmental area 53. Sulpiride is an antagonist.
328 10: Neuropharmacology and Neurochemistry

54. Which of the following is true of amyloid pre- (D) agranulocytosis


cursor protein (APP)? (E) akathisia
(A) The dominant isoform of APP contains
58. Which of the following is true of the pharmaco-
the protease inhibitor region.
logical properties of clozapine?
(B) APP undergoes a fast axonal transport to
the synaptic region to interact with the (A) It has a higher affinity to D2 than to D1
extracellular matrix. dopamine receptors.
(C) APP secretase is associated with the (B) It is a potent D4 dopamine receptor blocker.
amyloidogenic form of APP. (C) It inhibits c-fos expression.
(D) Beta secretases cleave the C terminal of (D) It has serotonin agonist activity.
APP and do not participate in the amy- (E) It activates the same dopaminergic neu-
loidogenic process. rons stimulated by haloperidol.
(E) Normal cellular metabolism does not syn-
thesize the A region of APP. 59. What is the mechanism of action of buspirone?
(A) It interacts with 5-HT1A receptors.
55. Compared with the nigrostriatal system, the
mesoprefrontal dopamine system is character- (B) It inhibits serotonin reuptake.
ized by (C) It interacts with 5-HT2 receptors.
(D) It blocks histamine reuptake.
(A) the presence of dopamine autoreceptors
(E) It is a potent D1 dopamine receptor
(B) lack of development of biochemical toler-
antagonist.
ance following chronic antipsychotic
drug administration 60. Which of the following drugs does not increase
(C) greater increase in the responsiveness to the level of lithium when both are administrated
dopamine agonists concomitantly?
(D) a lower turnover rate of transmitter
(A) Ibuprofen
dopamine
(B) Furosemide
(E) a lower rate of physiological activity
(C) Aspirin
56. Which of the following is true of the serotonin- (D) Lisinopril
ergic receptors? (E) None of the above
(A) The 5-HTA2 A receptors are densely
61. The primary neurotransmitter of postganglionic
located in the raphe nuclei.
sympathetic neurons for sweat glands is
(B) The activation of 5-HT1 receptors induces
an increase of adenylate cyclase. (A) acetylcholine
(C) The inhibitory effect of serotonin in the (B) norepinephrine
central nervous system is mediated by (C) glutamate
5-HT1 receptors. (D) aspartate
(D) The activation of 5-HT2 receptors induces (E) serotonin
the opening of potassium channels.
(E) The 5-HT3 receptors mediate fast excita- 62. Stimulation of the dorsomedial nucleus of the
tion requiring a coupling to G protein. hypothalamus results in
(A) aggressive behavior
57. The most serious side effect of clozapine is
(B) decreased feeding
(A) tardive dyskinesia (C) increased feeding
(B) neuroleptic malignant syndrome (D) increased blood pressure
(C) acute dystonia (E) adjustment of the circadian clock phase
Questions: 5471 329

63. Corticotropin-releasing hormone is produced 68. Functional MRI is based on


by which of the following hypothalamic nuclei?
(A) measuring changes in the oxygenation
(A) Anterior nucleus state of hemoglobin within a local brain
(B) Arcuate nucleus region
(C) Dorsomedial nucleus (B) computing the distribution of radiola-
(D) Supraoptic nucleus beled compounds involved in neuronal
metabolism or cerebral blood flow
(E) Medial preoptic nucleus
(C) T1 relaxation time
64. The inhibitory neurotransmitter of the cerebellar (D) T2 relaxation time
superficial stellate cell is (E) water protons activity in the brain tissue
(A) dopamine
69. Which of the following cerebellar cells is an exci-
(B) serotonin tatory interneuron?
(C) taurine
(A) Basket cell
(D) aspartate
(B) Stellate cell
(E) acetylcholine
(C) Golgi cell
65. Which of the following neurotransmitters is (D) Granule cell
released by the axons of cerebellar Purkinje (E) Pyramidal cell
cells?
70. Which of the following is a catecholamine neu-
(A) Dopamine
rotransmitter?
(B) GABA
(C) Glutamate (A) Glutamate
(D) Aspartate (B) Dopamine
(E) Serotonin (C) Acetylcholine
(D) Serotonin
66. The suprachiasmatic nucleus of the hypothala- (E) Glycine
mus plays an important role in
71. In the adult central nervous system, new neu-
(A) drinking behavior
rons are continuously being generated in the
(B) sexual arousal
(C) the sleepwake cycle (A) frontal lobe
(D) temperature regulation (B) hippocampal dentate gyrus
(E) parasympathetic activation (C) thalamus
(D) caudate nucleus
67. The component of the magnetic resonance imag- (E) cerebellum
ing (MRI) signal that depends on the direction of
diffusion of water protons within the central
nervous system tissue produces
(A) T1-weighted images
(B) T2-weighted images
(C) functional MRI
(D) diffusion-weighted MRI
(E) MR spectroscopy
330 10: Neuropharmacology and Neurochemistry

72. Differentiation of NSCs in neurospheres into 74. Which of the following is true of the nerves
neurons could be increased by absolute refractory period?
(A) leukemia inhibitory factor (A) It follows the relative refractory period.
(B) platelet derived growth factor (B) It is caused by reduced opening of potas-
(C) activation of the Notch signaling sium channels.
pathway (C) It results from residual inactivation of
(D) fibroblast growth factors sodium channels.
(E) transfection with v-myc oncogenes (D) It results from the opening of chloride
channels.
73. In glial cell membranes, the vast majority of rest- (E) An action potential may be triggered dur-
ing channels are permeable to ing the absolute refractory period, when a
stimulus stronger than those normally
(A) sodium ions only
required to reach threshold is applied.
(B) potassium ions only
(C) chloride ions only
(D) calcium ions only
(E) calcium and chloride ions
Answers and Explanations

1. (B) A substrate is accepted as a neurotransmit- presynaptically to high-affinity recognition sites


ter when it is present in the presynaptic termi- on the cholinergic nerve terminals and decreas-
nal, synthesized by neurons, and released in ing the release of acetylcholine, causing a neu-
amounts sufficient to exert an effect on the post- romuscular blocking effect. (Cooper, 151177)
synaptic neurons or effector organ, mimics its
endogenous action when given exogenously, 5. (A) Nicotinic receptors are blocked by curare
and has a specific mechanism for its removal and hexamethonium. Curare is an example of a
from the site of action. Biogenic amine neuro- nondepolarizing muscle relaxant that blocks the
transmitters include dopamine, norepinephrine, nicotinic acetylcholine receptor. The main toxin
epinephrine, serotonin, and histamine. Amino of curare, d-tubocurarine, occupies the same
acid neurotransmitters include GABA, glycine, position on the receptor as acetylcholine with an
and glutamate. Acetylcholine is the only accepted equal or greater affinity and elicits no response,
low-molecular-weight amine transmitter that is making it a competitive antagonist. The anti-
not an amino acid or derived directly from one. dote for curare poisoning is an acetylcholi-
(Kandell, 280) nesterase inhibitor such as physostigmine. By
blocking acetylcholine degradation, acetyl-
2. (E) Acetylcholine is synthesized from the com- cholinesterase inhibitors raise the amount of
bination of Acetyl CoA and choline in a reaction acetylcholine at the neuromuscular junction; the
catalyzed by a choline acetyltransferase. Acety- accumulated acetylcholine will then correct for
lcholine is then transported in vesicles into the the effect of the curare by activating the recep-
cholinergic synapse. This transport can be tors not blocked by toxin at a higher rate. (Cooper,
blocked by vesamicol and induces the deple- 151177)
tion of acetylcholine from the cholinergic vesicle.
(Cooper, 151177) 6. (C) Nicotinic receptors are blocked by curare
and hexamethonium, whereas dimethylphenyl
3. (E) The release of acetylcholine in the choliner- piperazinium acts as ganglionic nicotinic recep-
gic synapse is promoted by -bungarotoxin, a tors agonist. (Cooper, 151177)
form of bungarotoxin that is fairly common in
some snake venom. The target of this neuro- 7. (A) Cholinergic receptors fall into two cate-
toxin is the presynaptic terminal, whereby gories: muscarinic and nicotinic. Muscarinic
binding to proteins, most commonly actinit receptors that exhibit a slow response time are
causes release of acetylcholine and subsequent coupled to G proteins and are linked to phos-
exhaustion of acetylcholine stores in the nerve phoinositide hydrolysis or cyclic AMP as a sec-
terminal. (Cooper, 151177) ond messenger. Presynaptic or postsynaptic
muscarinic receptors are blocked by atropine.
4. (D) The release of acetylcholine at the choliner- Atropine and related compounds compete with
gic synapse is inhibited by botulinum toxin and acetylcholine and other muscarinic agonists
magnesium. Botulinum toxin acts by binding for a common binding site on the muscarinic

331
332 10: Neuropharmacology and Neurochemistry

receptor. Based on the position of retinol in the 12. (A) Dopamine is synthesized from the essential
mammalian rhodopsin structure, binding site amino acid tyrosine pathway involving five
for competitive antagonists and acetylcholine enzymes. The first enzyme is tyrosine hydrox-
likely is in a cleft formed by several of the recep- ylase. It is an oxidase that converts tyrosine to
tors seven transmembrane helices. An aspartic L-dopa. This is the rate-limiting step of
acid present in the N-terminal portion of the dopamine synthesis and requires reduced pteri-
third transmembrane helix of all five muscarinic dine as a cofactor, which is generated from
receptor subtypes is believed to form an ionic pteridine by a pteridine reductase. L-dopa
bond with the cationic quaternary nitrogen in decarboxylase produces dopamine after decar-
acetylcholine and the tertiary or quaternary boxylation of L-dopa. Dopamine is converted
nitrogen of the antagonists. Since antagonism to norepinephrine by dopamine -hydroxylase.
by atropine is competitive, it can be overcome if Norepinephrine is methylated to epinephrine
the concentration of acetylcholine at receptor by phenylethanolamine-N-methyl transferase.
sites of the effector organ is increased suffi- (Kandell, 282283)
ciently. (Cooper, 151177)
13. (B) Serotonin is derived from the hydroxyla-
8. (D) Oxotremorine is a presynaptic muscarinic tion of tryptophan by a tryptophan hydroxylase
agonist that inhibits the evoked release of acetyl- followed by decarboxylation of hydroxytrypto-
choline. (Cooper, 151177) phan by a 5-hydroxytryptophan decarboxylase.
(Kandell, 283284)
9. (B) Acetylcholine is deactivated in the cholin-
ergic synapse when it is hydrolyzed into choline 14. (A) The stimulation of a motor nerve releases
and acetate by acetylcholinesterase, which may acetylcholine into the synaptic cleft, where it
be inhibited reversibly by physostigmine. By diffuses to bind and activate acetylcholine recep-
interfering with the metabolism of acetylcholine, tors. The activation of postsynaptic acetylcholine
physostigmine indirectly stimulates both nico- produces a rapid increase in the endplate cur-
tinic and muscarinic receptors. (Cooper, 151177) rent. After deactivation of acetylcholine, the ran-
dom closure of the opened channels causes the
10. (B) Acetylcholine is deactivated in the cholin- endplate current to decay smoothly. The end-
ergic synapse when it is hydrolyzed into choline plate current depends on the number of acetyl-
and acetate by acetylcholinesterase, which may choline channels available for activation. The
be inhibited reversibly by physostigmine or irre- probability that a channel will be open depends
versibly by soman. (Cooper, 151177) on the concentration of acetylcholine at the chan-
nel, the conduction of each open channel, and
11. (C) Acetylcholine is synthesized from choline the driving force acting on the ions. The postsy-
and a donated acetyl group from acetyl-CoA by naptic acetylcholine receptor channels open by
the action of choline acetyltransferase (ChAT). the binding of acetylcholine, not by a change in
Thus, decreasing the amount of choline available voltage; therefore, the value of the membrane
to a neuron will decrease the amount of acetyl- potential does not influence the endplate cur-
choline produced. Choline reuptake, for further rent. (Kandell, 190)
acetylcholine synthesis, may be inhibited by a
competitive blocker such as hemicolinium-3. It 15. (E) GABA is formed by the decarboxylation of
is a drug that blocks the reuptake of choline by glutamic acid by a glutamic acid decarboxylase,
the high-affinity transporter ChAT (encoded in an enzyme located in the central nervous system
humans by the gene SLC5A7) at the presynapse. and the retina. Allyglycine is an inhibitor of glu-
The reuptake of choline is the rate-limiting step tamic acid decarboxylase. GABA is metabolized
in the synthesis of acetylcholine; hence, hemi- by transamination by GABA-transaminase,
cholinium decreases the synthesis of acetyl- yielding succinic semialdehyde and regenerat-
choline. It is therefore classified as an indirect ing glutamate. GABA receptors fall into two
acetylcholine antagonist. (Cooper, 151177) major types: GABA A and GABA B. When
Answers: 824 333

coupled with GABA, presynaptic and postsy- receptor is an inhibitory channel that, when acti-
naptic GABA receptors cause a shift in mem- vated, decreases neuronal activity. Presynaptic
brane permeability to chloride primarily. This and postsynaptic GABA receptors cause a shift
change in chloride permeability results in in membrane permeability to chloride prima-
hyperpolarization of the receptive neurons in rily when coupled with GABA. This change in
case of postsynaptic inhibition or depolariza- chloride permeability results in hyperpolariza-
tion in case of presynaptic inhibition. The GABA tion of the receptive neurons in case of postsy-
A receptorassociated channels predominantly naptic inhibition or depolarization in case of
conduct chloride ions. Since the equilibrium presynaptic inhibition. Diazepam acts by
potential of chloride is close to the resting poten- increasing the frequency of opening of the chlo-
tial of most neurons, an increase in the perme- ride channels without altering either their con-
ability of chloride decreases the depolarization duction or their duration of opening. (Cooper,
effect of an excitatory input, resulting in the 105127)
depression of excitability. Picrotoxin is a non-
competitive antagonist at GABA A receptors and 20. (C) Diazepam is a benzodiazepine that acts by
thus a convulsant. It blocks the GABA-activated increasing the frequency of opening of the chlo-
chloride ionophore. Although it is most often ride channels without altering either their con-
used as a research tool, it has been used as a duction or their duration of opening; however,
central nervous system stimulant and an anti- phenobarbital, which is a barbiturate, prolongs
dote in poisoning by central nervous system the duration of chloride channel opening by
depressants, especially the barbiturates. (Kandell, slightly decreasing the opening frequency.
190) (Cooper, 105127)

16. (B) Muscimol is a direct postsynaptic GABA 21. (B) Flumazenil is a benzodiazepine agonist as it
agonist that passes the bloodbrain barrier and binds to the same site of action as benzodi-
is active after systemic administration. azepine in GABA A receptors. When adminis-
Muscimol interacts directly with GABA A recep- trated alone, flumazenil has no pharmacological
tors, causing their activation. (Cooper, 105127) effect. However, when administrated with a
benzodiazepine, it reverses its effect. (Cooper,
17. (A) Indirect GABA agonists act to facilitate 105127)
GABAergic transmission by increasing the
amount of GABA that reaches the receptors or 22. (D) GABA B receptors are not linked to a chlo-
by altering the interaction between the receptor ride channel. Presynaptic GABA B receptors are
and GABA. GABA transaminase inhibitors, such linked through GTP-sensitive proteins to a cal-
as gabaculine, are indirect GABA agonists, as cium or potassium channel. The inhibitory effect
the availability of the neurotransmitter to the of GABA B receptor activation is probably medi-
GABA receptor is increased. (Cooper, 105127) ated through either an increase in potassium
conductance or a decrease in calcium conduc-
18. (A) Allylglycine is a glycine derivative and an tance. GABA B receptor activation with baclofen
inhibitor of the enzyme glutamate decarboxy- decreases calcium conductance and GABA
lase. Inhibition of glutamate decarboxylase release. (Cooper, 105127)
blocks GABA biosynthesis, leading to lower lev-
els of the neurotransmitter. It is used to induce 23. (E) Postsynaptic GABA B receptors are indi-
convulsions in animals in scientific studies. rectly coupled to a potassium channel via G pro-
(Cooper, 105127) teins mediating late inhibitory postsynaptic
potentials. Phaclofen is an antagonist of GABA
19. (D) Diazepam is a benzodiazepine that binds to B receptors. (Cooper, 105127)
a specific subunit on the GABA A receptor at a
site that is distinct from the binding site of the 24. (C) Nipecotic acid is a GABA uptake inhibitor.
endogenous GABA molecule. The GABA A It acts as an indirect GABA agonist, as the
334 10: Neuropharmacology and Neurochemistry

availability of the neurotransmitter to the GABA features, hirsutism, and Dupuytren contracture.
receptor is increased. (Cooper, 105127) (Brodie, 168175)

25. (B) The dopamine reuptake transporter controls 28. (C) Valproic acid is effective in patients with all
the levels of dopamine in the synapse by rapidly types of seizures, especially in those with idio-
carrying the neurotransmitter back into nerve pathic generalized epilepsy. The drug may act
terminals after its release. Cocaine, which binds by limiting sustained repetitive neuronal firing
strongly to the dopamine reuptake transporter, through inhibition of frequency-dependent
blocks dopamine reuptake after normal neu- blockade of voltage-dependent sodium chan-
ronal activity and increases its level at the nels. It may also increase brain GABA concen-
synapses, producing the characteristic cocaine trations. Dose-related side effects to valproic acid
euphoria. Animal studies show that the effect of include tremor, weight gain, alopecia, periph-
dopamine is dependent on the type of dopamine eral edema, nausea, and vomiting. Idiosyncratic
synaptic receptor with which it interacts. D1- reactions to valproic acid include acute pancre-
receptor agonists suppress cocaine-seeking atitis, hepatotoxicity, encephalopathy, thrombo-
behavior and may diminish episodes of intense cytopenia, and teratogenicity. (Brodie, 168175)
craving for cocaine, whereas D2 receptor ago-
nists may increase the cocaine-seeking behavior. 29. (B) Primidone is metabolized to phenobarbital
Neutralization of the dopamine reuptake trans- and another active metabolite, phenylethyl-
porter inhibits the psychostimulatory effect of malonamide. The efficacy of primidone is simi-
cocaine. Methadone is an active medication lar to that of phenobarbital, but primidone is
against chronic addiction to heroin, whereas less well tolerated. Its dose-dependent side
naloxone is used in the treatment of heroin effects include fatigue, lethargy, depression, psy-
overdose. (Leshner, 128129) chosis, decreased libido, and impotence. Its idio-
syncratic side effects include rash, thrombocy-
26. (E) Antiepilepsy medication side effects can be topenia, lupus-like syndrome, agranulocytosis,
divided into two categories: dose-related and teratogenicity. (Brodie, 168175)
and idiosyncratic. Ethosuximide is the drug of
choice for the treatment of uncomplicated 30. (D) Phenobarbital is as effective as phenytoin
absence seizures. It acts by reducing low-thresh- in abolishing partial and generalized tonic
old, transient, voltage-dependent calcium con- clonic seizures. At the cellular level, it prolongs
ductance in thalamic neurons. Its dose-depend- inhibitory postsynaptic potentials by increasing
ent side effects include nausea, vomiting, the mean chloride-channel opening time and
abdominal pain, agitation, headaches, lethargy, hence the duration of GABA-induced bursts of
drowsiness, dizziness, and ataxia. Idiosyncratic neuronal activity. Its dose-related side effects
reactions to ethosuximide are rare and include include decreased cognition, fatigue, lethargy
rash, erythema multiforme, StevensJohnson and depression in adults and irritability, dis-
syndrome agranulocytosis, and aplastic anemia. tractability, hyperkinesia, and insomnia in chil-
(Brodie, 168175) dren. Idiosyncratic reactions include macu-
lopapular rash, toxic epidermal necrosis,
27. (B) Phenytoin is effective in the treatment of par- hepatotoxicity, arthritis, teratogenicity, and
tial and tonicclonic seizures. It appears to act by Dupuytren contracture. (Brodie, 168175)
inducing voltage and use-dependent blockade of
sodium channels. Its dose-dependent side effects 31. (A) Carbamazepine is effective for the treatment
include nausea, vomiting, ataxia, nystagmus, of partial and generalized tonicclonic seizures
depression, drowsiness, paradoxical increase of but is not effective and may even be deleterious
seizures, gum hypertrophy, and megaloblastic in patients with absence or myoclonic seizures.
anemia. Its idiosyncratic effects are hepatotoxic- The drug acts by preventing repetitive firing of
ity, teratogenicity, acne, StevensJohnson syn- action potentials in depolarized neurons
drome, lupus-like syndrome, coarsening of facial through voltage- and use-dependent blockade
Answers: 2539 335

of sodium channels. Its dose-dependent side in dopamine, norepinephrine, or epinephrine in


effects include hyponatremia owing to its antid- the brain. Tyrosine hydroxylase is the first
iuretic hormonelike effect, neutropenia, nausea, enzyme in the biosynthesis pathway of norepi-
drowsiness, headache, dizziness, and diplopia. nephrine, allowing the conversion of tyrosine
Idiosyncratic reactions include morbilliform to dopa. It requires molecular Fe2+, oxygen, and
rash in about 10% of cases, erythema multiforme tetrahydropteridine as cofactors. Because it is
and StevensJohnson syndrome, agranulocyto- the rate-limiting step in the synthesis of norep-
sis, aplastic anemia, hepatotoxicity, and terato- inephrine in the brain as well as in the periph-
genicity. (Brodie, 168175) eral nervous system, pharmacological blockade
at this stage would reduce norepinephrine syn-
32. (A) Carbamazepine induces hepatic enzymes thesis. Alpha-methyltyrosine is an amino acid
to accelerate the hepatic metabolism of other analog that competitively inhibits tyrosine
lipid-soluble drugs, such as oral contraceptives. hydroxylase. (Cooper, 184202, 216)
In addition to inducing its own metabolism, car-
bamazepine not only accelerates the metabo- 36. (E) Norepinephrine is stored in granules in the
lism of valproic acid, ethosuximide, corticos- sympathetic nerve endings as well as in the cen-
teroids, anticoagulants, antipsychotic drugs, and tral nervous system. Reserpine interferes with
cyclosporine but also decreases their serum lev- the storage of norepinephrine, causing irre-
els and the potency of their therapeutic effects. versible long-lasting depletion of the storage of
However, the metabolism of carbamazepine is norepinephrine. (Cooper, 184202, 216)
inhibited by the administration of phenytoin,
which paradoxically induces the metabolism of 37. (B) The release of norepinephrine from storage
carbamazepine. Thus, adding phenytoin decreases granules is calcium-dependent. Amphetamines
plasma carbamazepine concentrations by about may cause an increase in the release of norepi-
a third, whereas adding carbamazepine to nephrine: when norepinephrine is released into
phenytoin increases plasma phenytoin concen- the synaptic cleft, it binds to alpha or beta
trations by a similar amount. Cimetidine, adrenoceptors and elicits the expected physio-
propoxyphene, diltiazem, erythromycin, isoni- logical effect. The norepinephrine transporter
azid, and verapamil may inhibit the metabolism (NET) is the principal route by which nor-
of carbamazepine to the point where the drug pinephrine removal occurs. The NET, often sit-
reaches a toxic level. (Brodie, 168175) uated on the presynaptic neuronal membrane,
pumps the synaptic norepinephrine back into
33. (D) Phenytoin may increase the hepatic oxida- the neuron cell body. In the cell body, this nor-
tion of lipid-soluble drugs including carba- epinephrine may reenter the vesicles or undergo
mazepine, valproic acid, ethosuximide, antico- metabolism through monoamine oxidase to
agulants, corticosteroids, and cyclosporine. Drugs dihydroxy phenylglycol. Under normal circum-
that inhibit the metabolism of phenytoin include stances, presynaptic NET inactivates and recy-
allopurinol, amiodarone, cimetidine, imipramine, cles norepinephrine released by vesicular fusion.
and some sulfonamides. This inhibition of the Amphetamine acts as both a NET substrate and
metabolism of phenytoin may bring the drug a reuptake blocker, eliciting reverse transport
concentration to a toxic level. (Brodie, 168175) and blocking normal uptake, thereby increas-
ing norepinephrine levels in and beyond the
34. (B) Aspirin displaces valproic acid from its bind- synaptic cleft. (Cooper, 184202, 216)
ing sites on plasma proteins and inhibits its
metabolism. (Brodie, 168175; Goulden, 13921394) 38. (C) Clonidine is a potent stimulator of alpha-2
presynaptic receptors. (Cooper, 184202, 216)
35. (E) Catecholamines are synthesized from tyro-
sine in the brain, chromaffin cells, and sympa- 39. (C) Phentolamine is an effective adrenergic
thetic ganglia. Tyrosine is metabolized in nor- postsynaptic alpha-receptor blocking agent.
epinephrine in the peripheral nervous system or (Cooper, 184202, 216)
336 10: Neuropharmacology and Neurochemistry

40. (D) The action of norepinephrine is ended by its 51. (D)


reuptake into the presynaptic terminal. This
reuptake may be inhibited by desipramine. 52. (B)
(Cooper, 184202, 216)
53. (B)
41. (A) Norepinephrine is degraded by monoamine
oxidase (MAO) when it presents in a free state Explanations 46 through 53
within the presynaptic terminal. It is inhibited
Dopamine receptors are classified on the basis
by pargyline. (Cooper, 184202, 216)
of a positive coupling between the receptor and
adenylate cyclase activity, mainly into D1 and
42. (D) Outside the presynaptic neuron, norepi-
D2 receptors. When activated, D1 dopamine
nephrine is inactivated by catechol-O-methyl-
receptors increase adenylate cyclase activity,
transferase (COMT). Tropolone is an inhibitor of
whereas the activation of D2 receptors inhibits
COMT. (Cooper, 184202, 216)
adenylate cyclase activity, enhances potassium
conductance, and inhibits calcium entry through
43. (D) D1 dopamine receptors are densely
voltage-sensitive calcium channels. The devel-
expressed in the amygdala, whereas D2 dopamine
opment of molecular biology divides D2 recep-
receptors as well as D3, D4, and D5 dopamine
tor into four subtypes and D1 receptors into
receptors have low levels of expression. D1 and
two subtypes. D2 receptor subtypes are D2
D2 dopamine receptors are highly expressed in
short, D2 long, D3, and D4. D1 receptor sub-
the caudate nucleus, putamen, nucleus accum-
types include D1 and D5. D5 receptors have
bens, and olfactory tubercle. D1 dopamine
more affinity to dopamine than D1 receptors,
receptors are not expressed in substantia nigra,
which is the only difference between the two
ventral tegmental area, or zona inserta. (Cooper,
dopamine receptors. D4 receptors have the
241242)
highest affinity to clozapine, an atypical neu-
roleptic, whereas D3 receptors have the high-
44. (E) Among the drugs mentioned in this ques-
est affinity to the dopamine agonist quinpi-
tion, gabapentin is the only one that has no liver
role. Bromocriptine is a D2 receptor agonist,
metabolism, as it is entirely eliminated by the
whereas sulpiride is a D2 antagonist. The
kidney. Because of this pharmacological char-
expression of dopamine receptors has been
acteristic, gabapentin may be the drug of choice
observed to change in disease states. In schiz-
in treating patients with seizure and acute
ophrenia, postmortem studies showed a con-
intermittent porphyria. (Bourgeois, 11811183)
sistent elevation of D2 receptors of the brain,
whereas D1 receptors remain unchanged,
45. (C) Mesolimbic dopamine neurons are involved
even in tissue obtained from patients without
in the reinforcing properties of a variety of
neuroleptic treatment. In Parkinson disease,
abused drugs such as cocaine. It acts by blocking
there is an increase in the expression of both
dopamine reuptake and inducing dopamine
D1 and D2 dopamine receptors. The chronic
release. (Cooper, 232)
administration of dopamine antagonists, such
as neuroleptic drugs, may increase the expres-
46. (A)
sion of dopamine receptors in the striatum.
The development of tardive dyskinesia after a
47. (B)
chronic use of neuroleptic may be explained
by a supersensitivity of dopamine receptors
48. (C)
that have been chronically blocked. (Cooper,
227264)
49. (B)
54. (B) The major characteristic of Alzheimer dis-
50. (D)
ease is the deposition of A- protein in the
Answers: 4060 337

microvasculature. This protein is derived from the membrane and activate phospholipase C. 5-
amyloid precursor protein (APP), which is HT3 receptors are ligand-gated ion channel
encoded by a single gene on chromosome 21. APP receptors. They mediate fast excitation through
has the structure of a transmembrane receptor ligand-gated cationic ion channels that do not
with an N extracellular segment and a C intra- require coupling with G proteins or a second
cellular segment. The dominant isoform of APP messenger. (Cooper, 284287)
does not contain a protease inhibitor region. It
undergoes fast axonal transport to the synaptic 57. (D) Agranulocytosis is the most serious side
region to interact with the extracellular matrix. effect of clozapine use. It occurs in 0.25% to 1%
APP may undergo a nonamyloidogenic metab- of treated patients, with peak incidence in the
olism by cleavage of the A- region (which first 4 to 18 weeks of treatment. Other side
includes the first 28 extracellular and the fol- effects include increased risk of grand mal
lowing 11 to 15 transmembrane amino acids), by seizure, sedation, hypersalivation, and weight
an secretase or by cleavage of the A- sequence gain. Clozapine has a very low incidence of
by a secretase in the N-terminal sequence or a acute or chronic motor side effects. (Enna,
secretase in the C-terminal region. The pro- 3638)
duction of A- protein is thought to be a minor
part of the normal processing of APP. (Blennow, 58. (B) Clozapine modifies the action of a num-
7786) ber of neurotransmitter systems. It is an antag-
onist of both D1 and D2 dopamine receptors in
55. (B) The mesoprefrontal dopamine system is a the brain, with higher affinity for D1 than D2
part of the mesotelencephalic dopamine system, dopamine receptors. Its highest affinity is for
which also includes the mesocingulate dopamine the D4 receptor. Also, it has a serotonin recep-
system. The mesotelencephalic dopamine sys- tor antagonism, especially 5HT2a, an anti-
tem lacks neuron autoreceptors (in contrast to cholinergic and histaminergic action. Clozapine
other dopamine neurons possessing autorecep- induces depolarization blockade in A10
tors such as the mesopiriform, mesolimbic, and dopamine neurons and activates c-fos expres-
nigrostriatal dopamine systems), which may sion, a marker of cellular activity in the nucleus
explain some biochemical, physiological, and accubens, ventral striatum, anterior cingulate,
pharmacological characteristics of these mid- and medial prefrontal cortex. In contrast,
brain neurons. Compared to dopamine systems haloperidol activates c-fos expression in regions
possessing autoreceptors, the mesoprefrontal that receive projection from A9 dopamine neu-
dopamine cells have a higher rate of firing and rons. (Enna, 7475)
more bursting, a higher turnover rate and
metabolism of transmitter dopamine, a less- 59. (A) Buspirone is an effective treatment for gen-
ened response to dopamine agonists and antag- eralized anxiety. It belongs to the azapirone class
onists, and a lack of biochemical tolerance fol- of drug, which have a high affinity to 5HT1A
lowing chronic drug administration. (Cooper, receptors. Buspirone may act as a partial agonist
255261) at 5HT1A receptors at postsynaptic sites, poten-
tially in the hippocampus and prefrontal cor-
56. (C) Radioligand binding studies have identi- tex. (Enna, 7475)
fied numerous subtypes of serotonin receptors
in the brain. 5-HTA1 receptors have a high den- 60. (C) The coadministration of lithium with
sity in the raphe nuclei and the hippocampus. diuretics and angiotensin-converting enzymes
When activated, they hyperpolarize the cell may increase lithium serum levels by pro-
membrane via G protein by opening potassium moting sodium loss and consequently a
channels, inhibiting adenylate cyclase, or closing decrease in lithium excretion. Nonsteroidal
calcium channels. 5-HTA2 receptors are highly anti-inflamatory medications except for
concentrated in layer IV of the cortex and the aspirin may also increase lithium serum lev-
hippocampus. When activated, they depolarize els. (Enna, 126127)
338 10: Neuropharmacology and Neurochemistry

61. (A) Except for the neurons supplying the sweat has significantly greater activation than other
gland, where acetylcholine is used as a neuro- hypothalamic nuclei during sexual arousal.
transmitter, the postganglionic sympathetic neu- Through the connections of the suprachiasmatic
rons use norepinephrine as a primary neuro- nucleus with the retina and brain regions related
transmitter. (Haines, 474) to circadian rhythm, the hypothalamus plays an
important role as an internal clock, regulating
62. (A) The dorsomedial nucleus of the hypothala- cyclic variation of a number of bodily function
mus subserves a function related to emotional such as temperature, sleeping and waking, and
behavior. In laboratory animals, stimulation of hormonal changes. (Afifi, 274)
the dorsomedial nucleus results in unusually
aggressive behavior, which lasts only as long as 67. (D) Magnetic resonance imaging (MRI) uses a
the stimulation is present. (Haines, 490) powerful magnetic field to align the nuclear
magnetization of hydrogen atoms in water
63. (D) Corticotropin-releasing hormone is pro- within the body. Protons in different tissues and
duced by neuroendocrine cells in the paraven- fluid compartments, when placed in a strong
tricular nucleus and supraoptic nucleus of the magnetic field, have slightly different proper-
hypothalamus; it is released from neurosecre- ties. MRI takes advantage of such differences to
tory terminals of these neurons into the pri- construct an image of brain structure or even
mary capillary plexus of the hypothalamo- function. MRI relies on the simple property that
hypophyseal portal system. The portal system protons can be made to emit signals that reflect
carries the corticotrophin-releasing hormone to the local tissue environment. Hence, protons in
the anterior lobe of the pituitary, where it stim- different tissues or fluid emit different signals.
ulates corticotropes to secrete adrenocorti- This is achieved by exciting protons with low
cotropic hormone (ACTH) and other biologi- levels of energy, which are carried by electro-
cally active substances. (Haines, 494) magnetic waves emitted from a coil placed over
the tissue. Once excited, the protons emit a sig-
64. (C) Taurine is believed to be the neurotrans- nal with three components, or parameters, that
mitter of the superficial stellate cells of the cere- depend on tissue characteristics. The first
bellum. Taurine levels are high in the molecular parameter is related to proton density in the tis-
layer of the cerebellum and drop substantially sue. The second and third parameters are related
when the development of stellate cells is blocked to proton relaxation time. The two relaxation
by x-irradiation. (Afifi, 214215) time are termed T1 and T2. When an MRI scan
is generated, it can be made to be dominated by
65. (B) GABA is released from axons of the one of these parameters. T1-weighted images
Purkinje, basket and Golgi neurons of the cere- are dominated by T1 relaxation time, whereas
bellum; it exerts an inhibitory effect on target T2-weighted images are dominated by T2 relax-
neurons. (Afifi, 215) ation time. In diffusion-weighted MRI, the
images are generated after performing a major
66. (E) Although definite delineation of a sympa- modification of the basic MRI technique. This
thetic and parasympathetic center within the takes advantage of a component of the MRI sig-
hypothalamus is not feasible, it is generally held nal that depends on the direction of diffusion of
that the rostral and medial hypothalamus is con- water protons within the tissue, which is highly
cerned with parasympathetic control whereas restricted within white matter tracts. (Martin,
the caudal and lateral hypothalamus is con- 4647)
cerned with sympathetic control mechanisms.
Temperature regulation is under the control of 68. (A) Functional MRI provides images of brain
the anterior regions of the hypothalamus. function by measuring changes in the oxygena-
Drinking behavior is under the control of the tion state of hemoglobin within local brain
lateral and anterior regions of the hypothala- regions. Active neurons consume more oxygen
mus. The preoptic nucleus of the hypothalamus and more glucose and demand more blood flow
Answers: 6174 339

than silent ones. Typically, functional MRI scans important pathway that controls a broad spec-
reflect the difference in oxygenation state of trum of cell fates and has been shown to induce
hemoglobin between the resting condition and glial cells in the central nervous system.
that which obtains when the individual being Transient activation of NOTCH1 in the rat NSC
scanned is engaged in a particular task. (Martin, cell line induced commitment of these cells to
51) astrocytes. Leukemia inhibitor factor and fibrob-
last growth factor are growth factors used for the
69. (D) The cerebellar cortex consists of three layers. expansion of neural stem cells. Neural stem cells
From the cerebellar surface to the white matter transfection with the v-myc oncogene is used to
these are the molecular, Purkinje, and granular generate clonally derived immortalized human
layers. Five neuron classes are found in the cere- neural stem cell lines. (Kim, 193201)
bellar cortex: the Purkinje cells, which are pro-
jection neurons of the cerebellum and are 73. (B) In the vast majority of resting channels in
inhibitory; the granule cells, which are the only the membrane of the glial cell, which has a rest-
excitatory interneurons of the cerebellum; and ing potential of 75 mV, are permeable only to
the basket, stellate, and Golgi cells, which are potassium ions. As a result, the glial cell mem-
also inhibitory. Pyramidal cells are located in brane at rest is almost permeable almost only to
the cerebral cortex. (Martin, 324) potassium ions. (Kandel, 128)

70. (B) The amino acid tyrosine is the precursor of 74. (C) The action potential is followed by a brief
three different amine neurotransmitters that con- period of diminished excitablility, or refrac-
tain a chemical structure called a catechol. These toriness, which can be divided into two
neurotransmitter are collectively called cate- phases. The first phase, the absolute refractory
cholamines. The catecholamine neurotransmit- period, comes immediately after the action
ters are dopamine, norepinephrine, and epi- potential. During this period, it is impossible to
nephrine. Catecholaminergic neurons are found excite the nerve no matter how great the stim-
in regions of the nervous system involved in the ulating current applied. This phase is followed
regulation of movement, mood, attention, and directly by the relative refractory period, dur-
visceral function. (Bear, 143) ing which it is possible to trigger an action
potential but only by applying stimuli that are
71. (B) The existence of neural stem cells with the stronger than those normally required to reach
potential for multipotent differentiation has threshold. These periods of refractoriness are
also been reported in embryonic and adult caused by residual inactivation of sodium chan-
human brains. One study has demonstrated nels and increased opening of potassium chan-
that in a group of cancer patients who received nels. (Kandel, 157)
an infusion of bromodeoxyuridine (BrdU) for
diagnostic purposes and later died, BrdU-
labeled proliferating cells that colabeled with
neuronal markers were found in the granular REFERENCES
layer of the hippocampal dentate gyrus. It is
evident from this study that new neurons are Afifi AK, Bergman RA, eds. Functional Neuroanatomy: Text
continuously being generated in the adult and Atlas. New York: McGraw-Hill; 2005.
human central nervous system. (Reynolds, Bear MF, Connors BW, Paradiso MA. Neuroscience:
Exploring the Brain. 3rd ed. Baltimore: Lippincott
17071710)
Williams & Wilkins; 2007.
Blennow K, Cowburn RF. The neurochemistry of Alzheimers
72. (C) Differentiation of neural stem cells into neu- disease. Acta Neurol Scand Suppl. 1996;168:77-86.
rons can be increased by treatment with NT3, Bourgeois BF. New antiepileptic drugs. Arch Neurol. 1998;
NT4, and platelet-derived growth factor (PDGF). 55(9):1181-1183.
Signaling by Notch, a member of the basic helix- Brodie MJ, Dichter MA. Antiepileptic drugs. N Engl J Med.
loop-helix (bHLH) transcription factors, is an 1996;334(3):168-175.
340 10: Neuropharmacology and Neurochemistry

Cooper JR, Bloom FE, Roth RH. The Biochemical Basis of Kandell ER, Schwartz JH, Jessell TM. Principles of Neural
Neuropharmacology. New York: Oxford University Press; Science. 4th ed. New York: McGraw-Hill; 2000.
2003. Leshner AI. Molecular mechanisms of cocaine addiction. N
Enna SJ, Coyle JT. Pharmacological Management of Neurological Engl J Med. 1996;335:128-129.
and Psychiatric Disorders. New York: McGraw-Hill; 1998. Martin JH, ed. Neuroanatomy. Text and Atlas. New York:
Goulden KJ, Dooley JM, Camfield PR, Fraser AD. Clinical McGraw-Hill; 2003.
valproate toxicity induced by acetylsalicylic acid. Reynolds BA, Weiss S. Generation of neurons and astro-
Neurology. 1987;37:1392-1394. cytes from isolated cells of the adult mammalian central
Haines DE. Fundamental Neuroscience for Basic and Clinical nervous system. Science. 1992;255(5052):1707-1710.
Applications. 3rd ed. Philadelphia: Churchill Livingstone
Elsevier; 2006.
CHAPTER 11

Neurogenetics
Questions

1. Which of the following genes is involved in 5. Mutation in the gene coding for transthyretin
familial amyotrophic lateral sclerosis? causes
(A) Androgen receptor gene (A) familial amyloid polyneuropathy
(B) Dystrophin gene (B) hereditary neuropathy with liability to
(C) Calpain 3 gene pressure palsies
(D) Caveolin gene (C) CharcotMarieTooth type 4C disease
(E) Superoxide dismutase 1 gene (D) lipid storage myopathy
(E) hyperkalemic periodic paralysis
2. Which of the following disorders is consistently
observed in a subset of families with amy- 6. In CharcotMarieTooth type 1B disease, the
otrophic lateral sclerosis? genetic defect is located in the gene coding for
(A) Seizure (A) connexin 32
(B) Frontotemporal dementia (B) lamin-A/C
(C) Sensory neuropathy (C) peripheral myelin protein 22
(D) Bladder dysfunction (D) emerin
(E) Autonomic dysfunction (E) myelin protein 0

3. KearnsSayre syndrome has 7. Arylsulfatase A deficiency causes


(A) an autosomal dominant transmission (A) metachromatic leucodystrophy
(B) an autosomal recessive transmission (B) acute intermittent porphyria
(C) X-linked transmission (C) X- linked spinobulbar muscular
(D) mitochondrial transmission dystrophy
(E) sodium channel protein mutation (D) hypokalemic periodic paralysis
(E) mitochondrial myopathy
4. Mutation in the gene coding for calpain 3 causes
(A) oculopharyngeal muscular dystrophy
(B) paramyotonia congenita
(C) limb girdle muscular dystrophy 2A
(D) progressive external ophthalmoplegia
(E) CharcotMarieTooth type 1A disease

341
342 11: Neurogenetics

8. A 4-year-old boy was brought for a consulta- ments, and generalized weakness predomi-
tion because of fatigue on exertion. He had a nantly in the wrist, finger extensors, and cervi-
history of recurrent apneic episodes triggered cal muscles. EMG studies show a repetitive com-
by fever or vomiting. Neurological examination pound muscle action potentials in response
disclosed mild asymmetric ptosis. The test for to single nerve stimulation with decremental
acetylcholine receptor antibodies is negative. responses on 2-Hz stimulation, which is reversed
Electromyographic (EMG) studies show a decre- with neostigmine administration. A younger
mental response at 10-Hz stimulation but ab- brother and a maternal uncle have similar
sence such a response at 2 Hz in rested muscle. symptoms. The most likely diagnosis is
The most likely diagnosis is
(A) endplate acetylcholine esterase defi-
(A) endplate acetylcholine esterase defi- ciency
ciency (B) slow-channel congenital myasthenic
(B) slow-channel congenital myasthenic syndrome
syndrome (C) congenital myasthenic syndrome with
(C) congenital myasthenic syndrome with episodic apnea
episodic apnea (D) botulism
(D) botulism (E) LambertEaton myasthenic syndrome
(E) LambertEaton myasthenic syndrome
11. Which of the following disorders has an
9. A 3-year-old boy was brought in by his mother X-linked inheritance?
because of moderate generalized weakness. He
(A) LeschNyhan syndrome
had a history of weak fetal movements in utero
and was born after 38 weeks gestation. He was (B) Neurofibromatosis type I
hypotonic from birth with weak suck, lid ptosis, (C) SturgeWeber syndrome
and delayed motor milestones. Neurological (D) Tuberous sclerosis
examination demonstrates generalized hypoto- (E) Ataxiatelangiectasia
nia, weakness, severe limitation of ocular move-
ment, ptosis, and sluggish pupillary light reflexes. 12. Which of the following is a major criterion for
The test for acetylcholine receptor antibodies is the diagnosis of tuberous sclerosis?
negative and the child shows no response to the
edrophonium test. EMG studies show repetitive (A) Subependymal giant cell astrocytoma
compound muscle action potentials in response (B) Vestibular schwannoma
to single nerve stimulation. The most likely diag- (C) Meningioma
nosis is (D) Optic glioma
(A) endplate acetylcholine esterase defi- (E) Neurofibroma
ciency
13. Which of the following is not a criterion for neu-
(B) slow-channel congenital myasthenic
rofibromatosis type I?
syndrome
(C) congenital myasthenic syndrome with (A) Optic glioma
episodic apnea (B) Bilateral auditory nerve schwannoma
(D) botulism (C) Two or more Lisch nodules
(E) LambertEaton myasthenic syndrome (D) Six or more caf au lait lesions
(E) A first-degree relative with neurofibro-
10. A 16-year-old boy developed diplopia and matosis type I
weakness, exacerbated by effort. Neurological
examination demonstrates mild lid ptosis, lim-
itation of vertical and horizontal eye move-
Questions: 823 343

14. The primary mechanism leading to cerebral (D) cerebrotendinous xanthomatosis


infarction in patients with hereditary hemor- (E) globoid leukodystrophy
rhagic telangiectasia is
(A) cerebral abscess 19. Deficiency of the mitochondrial enzyme sterol
27-hydroxylase is observed in
(B) embolism from pulmonary arteriovenous
fistula (A) X-linked adrenoleukodystrophy
(C) cerebral aneurysm (B) Alexander disease
(D) cerebral telangiectasia (C) Canavan disease
(E) cerebral angioma (D) cerebrotendinous xanthomatosis
(E) globoid leukodystrophy
15. SturgeWeber disease is transmitted as a
(A) sporadic pattern 20. The accumulation of glial fibrillary acidic pro-
tein exclusively in astrocytes is a hallmark of
(B) autosomal dominant pattern
(C) chromosomal deletion (A) X-linked adrenoleukodystrophy
(D) X-linked transmission (B) Alexander disease
(E) mitochondrial transmission (C) Canavan disease
(D) cerebrotendinous xanthomatosis
16. Which of the following is true of neurofibro- (E) globoid leukodystrophy
matosis type I?
(A) Neurofibromin is the protein encoded by 21. Galactocerebroside deficit is a hallmark of
the neurofibromatosis gene. (A) X-linked adrenoleukodystrophy
(B) Merlin is the protein encoded by the neu- (B) Alexander disease
rofibromatosis gene. (C) Canavan disease
(C) It is linked to chromosome X. (D) cerebrotendinous xanthomatosis
(D) Bilateral vestibular schwannomas is one (E) globoid leukodystrophy
of its diagnostic criteria.
(E) It is associated with a high risk of severe 22. Aspartoacylase deficiency is observed in
mental retardation.
(A) X-linked adrenoleukodystrophy
17. Which of the following is true of neurofibro- (B) Alexander disease
matosis type I? (C) Canavan disease
(A) Neurofibromin is the protein encoded by (D) cerebrotendinous xanthomatosis
the neurofibromatosis gene. (E) globoid leukodystrophy
(B) Merlin is the protein encoded by the neu-
rofibromatosis gene. 23. Arylsulfatase deficiency is a hallmark of
(C) It is linked to chromosome X. (A) Canavan disease
(D) It may occur with moyamoya syndrome. (B) cerebrotendinous xanthomatosis
(E) It is associated with a high risk of severe (C) globoid leukodystrophy
mental retardation. (D) metachromatic leukodystrophy
(E) PelizaeusMerzbacher disease
18. Accumulation of very long chain fatty acids is
seen in
(A) X-linked adrenoleukodystrophy
(B) Alexander disease
(C) Canavan disease
344 11: Neurogenetics

24. Alteration of the proteolipid protein gene (PLP) 29. The gene defect in ataxiatelangiectasia is
is responsible for located on chromosome
(A) Canavan disease (A) 11
(B) cerebrotendinous xanthomatosis (B) 8
(C) globoid leukodystrophy (C) 17
(D) metachromatic leukodystrophy (D) 19
(E) PelizaeusMerzbacher disease (E) X

25. Which of the following trinucleotide repeat 30. Which of the following spinocerebellar atrophies
expansion diseases is a type I disorder (i.e., has a benign course and normal life span?
occurs in the frame within the coding region)?
(A) Spinocerebellar atrophy type 3
(A) Myotonic dystrophy (B) Spinocerebellar atrophy type 4
(B) Kennedy disease (C) Spinocerebellar atrophy type 1
(C) Fragile-X syndrome (D) Spinocerebellar atrophy type 6
(D) Friedreich ataxia (E) Spinocerebellar atrophy type 7
(E) Progressive myoclonic epilepsy type I
31. Which of the following progressive ataxias are
26. Which of the following types of frontotemporal related to CTG repeats?
dementia is linked to a tau gene mutation on
(A) Spinocerebellar ataxia type 8
chromosome 17?
(B) Spinocerebellar ataxia type 7
(A) Semantic dementia (C) Dentatorubral-pallidoluysian atrophy
(B) Corticobasal degeneration (DRLPA)
(C) Frontotemporal dementia with motor (D) Spinocerebellar ataxia type 6
neuron disease (E) Spinocerebellar ataxia type 3
(D) Pick disease
(E) Dementia with parkinsonism 32. The abnormal gene coding for Huntingtin is
located on
27. Abnormal expansion of the GAA trinucleotide
(A) chromosome 12
on chromosome 9 is a hallmark of
(B) chromosome X
(A) ataxia with selective vitamin E deficiency (C) chromosome 19
(B) ataxiatelangiectasia (D) chromosome 4
(C) Friedreich ataxia (E) chromosome 14
(D) myotonic dystrophy type 1
(E) Huntington disease 33. The abnormal gene coding for Myotonin is
located on
28. Mutation in the alpha-tocopherol transfer pro-
(A) chromosome 12
tein gene on chromosome 8 is observed in
(B) chromosome X
(A) ataxia with selective vitamin E deficiency (C) chromosome 19
(B) ataxiatelangiectasia (D) chromosome 4
(C) Friedreich ataxia (E) chromosome 14
(D) myotonic dystrophy type 1
(E) Huntington disease
Questions: 2443 345

34. The abnormal gene coding for Atrophin is (A) Metachromatic leukodystrophy
located on (B) Tuberous sclerosis
(A) chromosome 12 (C) Adrenoleukodystrophy
(B) chromosome X (D) Neurofibromatosis
(C) chromosome 19 (E) Wilson disease
(D) chromosome 4
40. Which of the following have similar genetic
(E) chromosome 14
transmission to Wilson disease?
35. The abnormal gene coding for Ataxin 3 is (A) Myotonic dystrophy
located on (B) Metachromatic leukodystrophy
(A) chromosome 12 (C) Adrenoleukodystrophy
(B) chromosome X (D) Neurofibromatosis
(C) chromosome 19 (E) Fabry lipid storage disease
(D) chromosome 4
41. Which of the following have similar genetic
(E) chromosome 14
transmission to tuberous sclerosis?
36. The abnormal gene coding for the androgen (A) Metachromatic leukodystrophy
receptor is located on (B) Adrenoleukodystrophy
(A) chromosome 12 (C) Fabry lipid storage disease
(B) chromosome X (D) Myoclonic epilepsy with ragged-red
(C) chromosome 19 fibers
(D) chromosome 4 (E) Neurofibromatosis
(E) chromosome 14
42. Which of the following is true about chronic
progressive external ophtalmoplegia (CPEO)?
37. The genetic phenomenon responsible for the clin-
ical heterogeneity of mitochondrial diseases is (A) Mitochondrial DNA deletion is the most
common mutation in CPEO.
(A) sporadic mutation
(B) The onset of the disorder is usually after
(B) chromosomal deletion
the age of 50 years.
(C) anticipation
(C) The transmission of the disorder is
(D) heteroplasmy usually X-linked.
(E) mitotic instability (D) Diplopia is a common manifestation of
CPEO.
38. Which of the following disorders has a mito-
(E) Muscle biopsy is normal in most patients
chondrial inheritance?
with CPEO.
(A) Myotonic dystrophy
(B) Wilson disease 43. Which of the following manifestations differen-
(C) Metachromatic leukodystrophy tiates KearnsSayre syndrome from chronic pro-
gressive external ophthalmoplegia?
(D) Myoclonic epilepsy with ragged-red
fibers (A) Proximal muscles weakness
(E) Adrenoleukodystrophy (B) Ptosis
(C) Ophthalmoplegia
39. Which of the following have similar genetic (D) Cardiomyopathy
transmission to the transmission of Fabry lipid
(E) Elevated cerebrospinal fluid protein
storage disease?
346 11: Neurogenetics

44. A 39-year-old man with a history of non-insulin- 48. Carbamazepine-induced StevensJohnson syn-
dependent diabetes suddenly developed right drome is more prevalent in patients carrying
hemisensory loss. A computed tomography (CT)
(A) mutations in the NOTCH3 gene
scan of the head showed bilateral calcification of
the basal ganglia. Magnetic resonance imaging (B) HLA-B 1502
(MRI) of the brain confirmed the presence of (C) HLA-B 5801
an acute stroke. Subsequently, genetic testing (D) HLA-B 0702
revealed an A3243G mitochondrial mutation. (E) mitochondrial DNA mutations
The patient may develop
(A) optic neuritis 49. Mutations in which of the following genes cause
autosomal dominant transmission of Parkinson
(B) ptosis
disease?
(C) lactic acidosis
(D) ophthalmoplegia (A) PARKIN
(E) cardiomyopathy (B) LRRK2
(C) PINK1
45. The NOTCH3 gene mutations causes (D) DJ-1
(A) mitochondrial encephalopathy, lactic (E) ATP13A2
acidosis, and stroke-like episodes
50. Mutations in which of the following genes cause
(B) myoclonic epilepsy with ragged-red fibers
Parkinson disease with early dementia, auto-
(C) Fabry disease nomic dysfunction, and premature death?
(D) cerebral autosomal dominant arteriopa-
thy with subcortical infarcts and leukoen- (A) PARKIN
cephalopathy (B) LRRK2
(E) familial intracranial aneurysms (C) PINK1
(D) DJ-1
46. Mutations in the procollagen type IV alpha 1 (E) Alpha-synuclein
gene COL4A1 have been associated with
(A) Fabry disease 51. Mutations in which of the following genes fre-
quently cause early onset of Parkinson disease
(B) mitochondrial encephalopathy, lactic
with slow progression and excellent response
acidosis, and stroke-like episodes
to levodopa?
(C) cerebral autosomal dominant arteriopa-
thy with subcortical infarcts and leukoen- (A) PARKIN A
cephalopathy (CADASIL) (B) LRRK2
(D) autosomal dominant small vessel disease (C) Alpha-synuclein
with hemorrhagic stroke (D) DJ-1
(E) myoclonic epilepsy and ragged-red (E) ATP13A2
fibers
52. Which of the following characteristics differen-
47. Which of the following is an inherited risk fac- tiate Parkinsons disease patients carrying PINK1
tor for cerebral venous thrombosis? mutations from those who are carrying PARKIN
(A) Mutation in the gene coding for amyloid mutations?
precursor protein (APP) (A) Autosomal dominant transmission
(B) Factor V Leiden (B) Autosomal recessive transmission
(C) Mutations in mitochondrial DNA (C) Early-onset Parkinson disease
(D) Homocystinuria (D) Higher rate of psychiatric symptoms
(E) NOTCH3 mutations (E) Higher rate of autonomic dysfunction
Questions: 4462 347

53. Mutations in the DJ-1 gene cause 58. Beside cerebellar ataxia, spinocerebellar ataxia
type 10,17 and DRPLA have in common the
(A) autosomal dominant Parkinson disease
occurrence of
(B) dementia
(C) severe autonomic dysfunction (A) ophthalmoaresis
(D) early-onset Parkinson disease (B) seizure
(E) intractable seizure (C) pigmentary retinopathy
(D) peripheral neuropathy
54. Mutations in which of the following genes fre- (E) parkinsonism
quently cause subacute juvenile-onset levodopa-
responsive Parkinson disease? 59. Myokymia can occur in
(A) PARKIN (A) DRPLA
(B) LRRK2 (B) spinocerebellar ataxia type 10
(C) Alpha-synuclein (C) spinocerebellar ataxia type 5
(D) DJ-1 (D) Friedreich ataxia
(E) ATP13A2 (E) ataxiatelangiectasia

55. Which of the following disorders causes ataxia 60. Linkage studies have mapped frontotemporal
with DNA repair defect? dementia with parkinsonism to chromosome
(A) Friedreich ataxia (A) 9
(B) Refsum disease (B) X
(C) Wilson disease (C) 3
(D) Metachromatic leukodystrophy (D) 17
(E) Ataxiatelangiectasia (E) 1

56. A 52-year-old man developed progressive ataxia 61. Which of the following types of episodic ataxia
and cognitive deficits. His neurological exami- (EA) causes the longest-lasting attacks?
nation showed a combination of kinetic tremor,
gait ataxia, autonomic dysfunction, and polyneu- (A) Episodic ataxia type 1
ropathy. The most likely diagnosis is (B) Episodic ataxia type 2
(A) CAG triplet repeat disorder (C) Episodic ataxia type 3
(B) CGG triplet repeat disorder (D) Episodic ataxia type 4
(C) pentanucleotide repeat expansion (E) Episodic ataxia type 7
(ATTCT) disorder
(D) Parkinson disease with -synuclein 62. Familial hemiplegic migraine type 1 is caused by
mutation mutations in which of the following genes?
(E) Parkinson disease with PARKIN mutation (A) CACN1A
(B) ATP1A2
57. The association of cerebellar ataxia and pig- (C) SCNA1
mentary retinopathy is suggestive of (D) CACNB4
(A) spinocerebellar ataxia type 7 (E) KCNA1
(B) spinocerebellar ataxia type 10
(C) spinocerebellar ataxia type 1
(D) herediary episodic ataxia
(E) Leigh syndrome
Answers and Explanations

1. (E) The greatest contribution toward an under- some noteworthy exceptions, mutations are pri-
standing of amyotrophic lateral sclerosis (ALS) marily dominant. (Amato, 53; Valdmanis, 144152)
thus far has come from the discovery of muta-
tions in the superoxide dismutase 1 (SOD1) gene 2. (B) Frontotemporal dementia (FTD) is the only
on chromosome 21q22.11, which account for reported disease which is consistently observed
10% to 20% of autosomal dominant ALS cases. in a subset of ALS families. Estimates of FTD
Tremendous efforts have been made to under- prevalence in ALS cases range between 5% and
stand not only the multiple likely pathogenic 15%. (Valdmanis, 144152)
mechanisms of SOD1 in eliciting disease but
also the genetic profile of the reported muta- 3. (D) Single large mitochondrial DNA deletions
tions in this gene. A number of conclusions can (ranging from 1.3 to 8.8 kb) can be demonstrated
be drawn from this research: (1) SOD1 muta- in most patients with KearnsSayre syndrome.
tions result in a toxic gain-of-function pathology. One is more likely to find mitochondrial DNA
Evidence for this arose largely from mouse stud- mutations in muscle tissue than in peripheral
ies in which the knockout of the SOD1 gene white blood cells, with the percentage of affected
failed to yield a phenotype; conversely, trans- mitochondrial genomes in muscle biopsies rang-
genic mice that overexpressed mutant SOD1 did ing from 20% to 90%. (Amato, 636637).
develop a motor neuron phenotype. (2) Loss of
the normal function of SOD1 is not the cause of 4. (C) Limb girdle muscular dystrophy 2A is
ALS. Mutations in the SOD1 gene have a broad caused by a mutation in the calpain-3 gene.
range of effects on the enzymatic activity of Calpain-3 is a muscle-specific, calcium-dependent,
SOD1; however, among the mutated SOD1 pro- nonlysosomal, proteolytic enzyme. The muta-
teins reported, some noticeably retained full tion leads to an absence or reduction in the activ-
enzymatic activity. The rest of the mutations ity of this enzyme, which can lead to accumula-
were shown to influence, among other things, tion of toxic substances in muscle cells. (Amato,
the stability of SOD1, its ability to dimerize, its 543)
hydrophobicity, and its ability to chelate cop-
per ions. Different SOD1 mutations may also 5. (A) Familial amyloid polyneuropathy is an auto-
influence various aspects of the disease, such as somal dominant disorder in which there is extra-
its onset or duration, but they nonetheless do not cellular deposition of amyloid in peripheral
cause ALS. (3) Mutations can occur at almost nerves and other organs. A painful sensory neu-
any position in the SOD1 gene. One of the most ropathy with early involvement of autonomic
remarkable aspects of this gene is that more than nerves and cardiomyopathy is typically present.
110 mutations have been reported in nearly 50% Age of onset can vary from 18 to 83 years. Small
of the 153 amino acids in the SOD1 protein. The fibers (for pain and temperature) are more
distribution of these mutations is quite uniform: affected than large fibers (for vibration and pro-
the largest interval without a reported mutation prioception); anhidrosis, gastrointestinal distur-
is only nine consecutive amino acids. (4) With bances (diarrhea alternating with constipation),

348
Answers: 19 349

impotence, orthostatic intolerance, visual changes, required for metabolizing galactosylsulfatide,


and arrhythmias are additional features. Muta- a glycolipid present in myelin membranes.
tions in transthyretin (FAP 1 and 2), apolipopro- (Amato, 193)
tein A1 (FAP 3), or gelosin (FAP 4) are responsi-
ble. Transthyretin is most often implicated in 8. (C) The case described in this vignette has the
peripheral neuropathy. Nearly 100 different characteristic features of congenital myasthenic
mutations have been identified in the TTR gene, syndrome with episodic apnea: recurrent apneic
the most common being the Val30Met mutation. episodes triggered by fever or vomiting, and
Liver transplantation halts disease progression. mild ptosis as well as absence of decremental
(Chaudhry, chapter 379) response on 2-Hz stimulation and presence of
this response on 10-Hz stimulation. Muscle
6. (E) Myelin protein 0 (MPZ) is a major periph- biopsy confirms the diagnosis by showing end-
eral nervous system protein responsible for plate acetylcholine receptor deficiency without
myelin compaction and the adherence of myelin postsynaptic structural abnormalities. The ab-
wraps to each other. It is also involved in normality involves a presynaptic defect. There is
the signal transduction cascade responsible for a marked decrease in the number of acetyl-
interaction between the Schwann cell and axon, choline quanta released by the nerve impulse
as well as regulation of myelin-specific gene due to a defect in the synthesis or axonal trans-
expression. Mutations in the MPZ gene are asso- port of vesicle precursors from the anterior horn
ciated with the autosomal dominant form of cell to the nerve terminal. (Conneally, 923)
CharcotMarieTooth disease type 1 (CMT1B),
which is characterized by progressive slowing of 9. (A) Endplate acetylcholine esterase deficiency
nerve conduction and hypertrophy of Schwann causes a congenital myasthenic syndrome char-
cells. Mutations in MPZ can also produce the acterized by delayed pupillary light reflexes,
more severe polyneuropathies, DejerineSottas refractoriness to cholinesterase inhibitors, and
syndrome (DSS) and congenital hypomyelinat- repetitive compound muscle action potentials.
ing neuropathy, as well as several types of The illness is caused by the absence of acetyl-
axonal CMT2. MPZ mutations cause hereditary choline esterase in the synaptic space. The neu-
neuropathy with phenotypic clustering into two romuscular transmission is compromised by the
major clinical, electrodiagnostic, and patholog- reduced size of the nerve terminals, with their
ical entities. The early-onset form causes severe encasement by Schwann cells reducing the num-
neuropathy in infancy with delayed motor mile- ber of releasable acetylcholine quanta. The cholin-
stones, slow conduction velocities in the demye- ergic overactivity may induce the Schwann cells
linating range, and predominant demyelination to encase the nerve terminals, thus protecting the
on nerve biopsy. It seems that mutations that endplate from overexposure to acetylcholine. The
significantly disturb the tertiary structure of absence of acetylcholine esterase in the synaptic
MPZ are responsible for this phenotype. The terminals results in an overexpression of acetyl-
late-onset form presents in adulthood with neu- choline, causing a prolongation of the synaptic
ropathy that is slowly progressive, with axonal potentials beyond the refractory period of the
features to a greater extent than demyelinating muscle fiber, which triggers repetitive compound
features on electrodiagnostic and nerve biopsy muscle fiber action potentials. The progression of
studies. Mutations that subtly affect the MPZ the disease is attributed to an endplate myopathy
structure may interfere with Schwann cellaxon from cholinergic overactivity. The molecular basis
interaction and cause this phenotype. (Souayah, of the disease involves a recessive mutation in
177179) COLQ, a triple stranded collagenic tail of the
acetylcholinesterase enzyme. Typical clinical
7. (A) Metachromatic leukodystrophy is an auto- manifestations of the disease are described in this
somal recessive disorder caused by a mutation vignette. The diagnosis is confirmed by muscle
in the arylsulfatase A or prosaposin gene. biopsy that demonstrates absence of acetyl-
Arylsulfatase and prosaposin are both enzymes choline esterase from endplates. (Conneally, 923)
350 11: Neurogenetics

10. (B) The case described in this vignette has fea- 13. (B) The gene for neurofibromatoses type I oc-
tures that point to slow-channel congenital curs as a spontaneous mutation in 1 per 10,000
myasthenic syndrome: selective involvement of individuals and can affect most organ systems.
wrist and finger extensors, possible dominant Initial signs and symptoms vary. In 1987, the
inheritance, and repetitive compound muscle National Institutes of Health issued a consensus
action potentials with decremental response statement enumerating the clinical diagnostic
on 2-Hz stimulation repaired by neostigmine criteria for neurofibromatosis type I. They in-
administration. An endplate myopathy is caused clude two of the following: (1) six or more cafe
by prolonged opening episodes of acetylcholine au lait macules greater than 5 mm in prepuber-
receptors during activity and spontaneous open- tal patients and greater than 15 mm in postpu-
ing of acetylcholine receptors at rest. (Conneally, bertal patients; (2) two or more neurofibromas of
923) any type or one plexiform neurofibroma; (3)
axillary or inguinal freckling; (4) optic nerve
11. (A) LeschNyhan disease has X-linked inheri- glioma; (5) two or more Lisch nodules (iris
tance and is characterized by self-mutilation of hamartomas); (6) sphenoid wing dysplasia or
digits and lips. Its neurological features include cortical thinning of long bones with or without
mental retardation and dystonia. Neurofibro- pseudarthrosis; and (7) a first-degree relative
matosis type I is an autosomal dominant disorder (parent, sibling, or child) with NF-1 based on
characterized by dermatological and neurological the preceding criteria. (Karnes, 10711076)
features. Cutaneous features include caf au lait
spots, axillary freckling, neurofibroma, Lisch nod- 14. (B) Hereditary hemorrhagic telangiectasia (HHT),
ules of the iris, and plexiform neurofibromas. also known as OslerWeberRendu disease, is a
Neurological features include learning disability, hereditary autosomal dominant syndrome char-
cognitive impairment, and neuraxis tumors. acterized by easy bleeding and vascular abnor-
SturgeWeber disease has sporadic inheritance. malities. The classic picture is that of a familial
The neurological features comprise epilepsy, men- pattern of telangiectasias and epistaxis. The char-
tal retardation, and focal deficits. Epilepsy, men- acteristic lesion is the telangiectasia: a lesion 1 to
tal retardation, autism, and giant cell astrocytoma 2 mm in diameter consisting of a dilated vessel
complicate tuberous sclerosis, a neurocutaneous directly connecting an artery and a vein.
disease with autosomal dominant transmission. Telangiectasias probably develop from dilated
Ataxiatelangiectasia is an autosomal recessive postcapillary venules. Telangiectasias usually
disease characterized by ataxia, intention tremor, appear on the skin and mucosal surfaces, espe-
abnormal saccades, and decreased deep tendon cially on the nose.
reflexes. (Roach, 591620) Larger arteriovenous malformations (AVMs),
consisting of thin-walled vascular spaces with
12. (A) Tuberous sclerosis complex (TSC) arises single or multiple feeding arteries occur mostly in
from abnormal cellular differentiation, prolifer- the lungs, liver, and brain. These may reach a
ation, and neuronal migration. It affects the brain diameter of several centimeters. Vascular mal-
(cortical and subcortical tubers, subependymal formations may appear in any organ, however.
nodules, and giant cell astrocytomas), kidney, Transforming growth factor beta is known to
skin (hypomelanotic macules, shagreen patches, have a regulatory role in tissue repair and angio-
facial angiofibromas, and periungual fibromas), genesis. Mutation on chromosome 9 seems to
eye (retinal hamartomas), heart, and to a lesser predispose to a high prevalence of pulmonary
extent other organs. The Tuberous Sclerosis AVMs, which are found in 15% to 33% of patients
Complex Consensus Conference divided the cri- with HHT and are usually fed by the pulmonary
teria for diagnosis into major and minor features. artery, draining through the pulmonary veins.
The major features include cortical tuber, Some 70% of pulmonary AVMs (PAVMs)
subependymal nodule, subependymal giant cell occur in the lower lung fields and may enlarge
astrocytoma, and skin changes, as mentioned with time or during pregnancy. They can result in
above. (Sparagana, 115119) a substantial right-to-left shunt, with significant
Answers: 1020 351

hypoxemia. Serious complications may occur: Dysplasia of the cerebral artery may occur, caus-
bleeding can result in potentially life-threatening ing moyamoya syndrome.
hemoptysis or hemothorax, and paradoxical The gene for NF-I is located on the long arm
emboli, bypassing the pulmonary capillary sys- of chromosome 17 at 17q11.2. Neurofibromin is
tem via the PAVM, may give rise to ischemic the protein encoded by the neurofibromatosis
cerebral events. This is the primary mechanism gene and may act as a tumor suppressor. (Karnes,
leading to cerebral infarction in patients with 10711076)
HHT (up to one third of patients with PAVMs suf-
fer from ischemic cerebral events). (Haitjema, 17. (B) The diagnostic criteria for NF-II are either
714719) (1) bilateral eighth-nerve masses or (2) a first-
degree relative with NF-II and either a unilateral
15. (A) SturgeWeber syndrome is a neurocuta- auditory nerve mass or two of the following:
neous syndrome characterized by port wine neurofibroma, meningioma, glioma, schwan-
facial nevi and associated leptomeningeal and noma, or juvenile posterior subcapsular lentic-
brain angiomas. The syndrome occurs sporadi- ular opacity. Schwannoma is the most common
cally but may result from a somatic mutation tumor in NF-II. Such tumors may involve cranial
disturbing the angiogenic process. (Huq, 780782) as well as peripheral nerves. Schwannomas of
spinal nerve sheath tumors are also common in
16. (A) Neurofibromatosis (NF) is a neurocutaneous patients with NF-II, and spinal cord ependy-
condition, of which two types exist. NF type I momas may occur. The gene for NF-II is located
(NF-I) occurs in about 1 in 3,000 persons and on chromosome 22 at 22q11. Merlin, the gene
accounts for 96% to 97% of all cases of NF. NF product of the chromosome of NF-II, is a tumor
type II (NF-II) accounts for about 3% of cases. suppressor. (Karnes, 10711076)
Both NF-I and NF-II have autosomal dominant
transmission. Almost every organ system can be 18. (A) X-linked adrenoleukodystrophy (X-ALD)
involved in NF-I; thus, initial signs and symp- encompasses widely differing clinical pheno-
toms vary. Lisch nodules and optic nerve gliomas types that reflect two distinct pathological mech-
are among the diagnostic criteria of NF-I. anisms: an inflammatory demyelinating process
The most common benign tumors in pa- that leads to a rapidly progressing fatal disorder
tients with NF-I are neurofibromas. They are and a slowly progressing, distal axonopathy that
composed of Schwann cells, fibroblasts, mast leads primarily to adrenomyeloneuropathy in
cells, and vascular elements. Plexiform neurofi- young adults. In all forms of X-ALD, very long
bromas are specific to NF-I. Schwannomas are chain fatty acids (VLCFAs) accumulate in tis-
uncommon in patients with NF-I but when they sues and body fluids, due to impaired activation
occur, they exist on spinal nerve sheaths. It has of these fatty acids. (Berger, 305312)
recently been suggested that when a single
vestibular schwannoma is detected on an imag- 19. (D) Cerebrotendinous xanthomatosis is a lipid
ing study of the head, it is unlikely that the storage disorder caused by deficiency of the
patient has NF-I. mitochondrial enzyme sterol 27-hydroxylase,
Central nervous system manifestations which leads to accumulation of cholesterol and
of NF-I include aqueductal stenosis, hydro- bile alcohols. (Berger, 305312)
cephalus, and seizures. Of patients with NF-I,
25% to 40% may have learning disabilities, and 20. (B) Alexander disease is a lethal leukodystrophy
5% to 10% may have mental retardation. with a variable clinical course. The most com-
Essential hypertension may occur in patients mon, infantile form is associated with megalen-
with NF-I; hypertension may also be due to cephaly, seizures, developmental retardation, and
pheochromocytoma, renal artery stenosis, neu- premature death. Juvenile and adult patients, on
rofibromas that compress the kidneys or renal the other hand, experience ataxia, spasticity, and
arteries, renal artery dysplasia, Wilms tumor, or bulbar signs, with relatively little loss of myelin.
coarctation of the abdominal or thoracic aorta. Neuroradiological and neuropathological studies
352 11: Neurogenetics

show extensive white matter involvement with Type I disorders are those in which the
frontotemporal predominance. The pathological expansion occurs in-frame (i.e., within the cod-
hallmark of Alexander disease is the accumula- ing region) and results in an expanded stretch of
tion of intracellular inclusions (Rosenthal fibers) amino acids generated by the abnormal gene.
exclusively in astrocytes. These consist of aggre- Huntington disease; Kennedy disease; spin-
gated glial fibrillary acidic proteins (GFAPs) and ocerebellar ataxia types 1, 2, 3, 6, 7, and 17;
small stress proteins. (Berger, 305312) DRLPA; and oculopharyngeal muscular dystro-
phy are among type I expansion disorders.
21. (E) Globoid cell leukodystrophy (GLD), also Type II disorders are those in which the
called Krabbe disease, is an inherited neurolog- expansion occurs outside the coding region:
ical disease caused by mutations in the GALC either upstream of the coding sequence, down-
gene, which encodes the lysosomal enzyme stream of the coding sequence, or within an
galactocerebrosidase, responsible for the degra- intron. Spinocerebellar ataxia type 8, 10, and 12;
dation of galactosylceramide and galactosyl- fragile-X syndrome; Jacobsen syndrome; pro-
sphingosine. (Berger, 305312) gressive myoclonic epilepsy type I; Friedreich
ataxia; and myotonic dystrophy are type II dis-
22. (C) Canavan disease is an inherited infantile eases.
leukodystrophy associated with spongy degen- In type I disorders, the mutant gene is tran-
eration of white matter, macrocephaly, severe scribed and translated normally but leads to the
psychomotor retardation, seizures, and prema- production of a protein harboring an expanded
ture death. It is characterized by aspartoacylase stretch of a particular amino acid. The trinu-
deficiency. The enzyme defect leads to the accu- cleotide expansions in type I disorders tend to be
mulation of N-acetyl aspartate (NAA) in the small, with a similar threshold for disease (36 to
brain and body fluids. (Berger, 305312) 40 trinucleotide repeats, with limited excep-
tions). To date, in each case of type I disease,
23. (D) A genetic deficiency in the lysosomal the mutant protein is endowed with a toxic
enzyme arylsulfatase A (ASA) causes the neu- gain of function. In general, all type I diseases
rometabolic disease metachromatic leukodys- except spinobulbar muscular atrophy are dom-
trophy (MLD). Three major clinical variants inantly inherited, tend to be of late onset, and
have been characterized: late infantile, juvenile, have manifestations that are limited to the nerv-
and adult MLD. ASA deficiency results in im- ous system.
paired degradation of the substrate galactosyl- Conversely, in type II disorders, the coding
sulfatide, which allows for the biochemical diag- sequence remains unchanged and the protein
nosis of MLD on the basis of ASA activity in product is normal, yet mutations in untrans-
leukocytes or fibroblasts and galactosylsulfatide lated regions of the gene lead to abnormal tran-
excretion in urine. (Berger, 305312) scription or RNA processing, resulting in altered
levels of gene expression. The trinucleotide
24. (E) PelizaeusMerzbacher disease (PMD) is an expansions leading to type II disorders tend to
X-linked dysmyelinating disorder caused by be large, with hundreds to over a thousand trin-
alterations in the proteolipid protein gene (PLP), ucleotides. These mutations often result in loss
which encodes two major proteins of central of function of the relevant gene. Most of type II
nervous system myelin: PLP and its spliced iso- disorders are multisystem disorders and tend
form DM20. (Berger, 305312) to have younger ages of onset than type I disor-
ders. (Taylor, 2425)
25. (B) Trinucleotide repeat expansion disorders may
be arbitrarily divided into two types, based on the 26. (E) Frontotemporal dementias occur in either
location of the mutations within their respective familial forms or, more commonly, as sporadic
genes. This classification is useful because the cases. Neuropathologically, they are character-
location of the mutation may have implications ized by a remarkably circumscribed atrophy of
regarding the mechanism of pathogenesis. the frontal and temporal lobes of the cerebral
Answers: 2134 353

cortex, often with additional subcortical changes. and limb ataxia, dysarthria, nystagmus, slowing
An autosomal dominantly inherited familial of saccades, signs of corticospinal tract disease,
form of frontotemporal dementia with parkin- hypotonia, and proprioceptive sensory loss.
sonism was linked to chromosome 17q21.2. Other less common features include ophthal-
A major neuropathological characteristic of moparesis, spasticity, rigidity, sphincter distur-
FTDP-17 is a filamentous pathology made of bances, pes cavus or hammertoes, dystonia, and
hyperphosphorylated tau protein. (Goedert, parkinsonism. The disease has a benign course
7483) and is associated with a normal life span.
(Evidente, 475490)
27. (C) Friedreich ataxia is the most common
genetic ataxia and is of autosomal recessive 31. (A) Spinocerebellar ataxias (SCAs) are a group
inheritance, with progressive gait and limb of neurodegenerative diseases characterized by
ataxia as the cardinal features. It is associated cerebellar dysfunction; other neurological ab-
with lower limb areflexia, dysarthria, pyramidal normalities may be associated. The expansions
weakness, and sensory loss manifesting later in of coded CAG trinucleotide repeats was found
the course of the disease. The abnormal gene of to cause dominantly inherited SCAs such as
Freidreich ataxia is located on chromosome 9. SCAs 1, 2, 3, 6, 7, and DRPLA. The abnormal
The abnormal gene contains an expansion of CAG triple repeat expansion gives rise to an
GAA trinucleotides that number between 200 elongated polyglutamine tract in the respective
and 900. (Normal chromosomes contain 10 to proteins, leading to a gain in function that is
21 GAA repeats). (Hammans, 327332) toxic to neurons. Spinocerebellar ataxia type 8 is
associated with an expansion of a CTG repeat.
28. (A) Secondary vitamin E deficiency (precipi- (Tan, 191195)
tated by a beta-lipoproteinaemia or other fat
malabsorptive syndromes) is associated with 32. (D) Huntington disease (HD) is an autosomal
ataxia. The onset of symptoms occurs between 4 dominant disorder with high penetrance. The
and 18 years of age, with progressive ataxia, are- characteristic findings of progressive chorea and
flexia, sensory loss, pyramidal signs, and some- dementia are caused by severe neuronal loss,
times cardiomyopathy. Ataxia with selective initially in the neostriatum and later in the cere-
vitamin E deficiency was linked to chromosome bral cortex. HD has been linked to chromosome
8q13 in 1993. Subsequently, the mutation was 4p16.3. The abnormal gene was found to contain
identified in the alpha-tocopherol transfer pro- an unstable CAG repeat in the open reading
tein on chromosome 8. (Hammans, 327332) frame of its first exon. Normal subjects have a
median of 19 CAG repeats (range 11 to 34),
29. (A) Ataxiatelangiectasia is an autosomal reces- whereas nearly all patients with HD have more
sive disease characterized by ataxia, diminished than 40. The increased number of CAG repeats
proprioception, areflexia, and dysarthria. The in the HD gene is expressed as an elongated
gene defect is located on chromosome 11q. The huntingtin protein with 40 to 150 glutamine
ataxiatelangiectasia mutated protein has residues. (Martin, 19701980; Price, 10791083)
sequence homologies to phosphatidylinositol-3
kinase and may be involved in a checkpoint 33. (C) Myotonic dystrophy is CTG repeat triplets
response protein to DNA damage. (Hammans, disease. The abnormal gene is located on chro-
327332) mosome 19 and the gene product is myotonin.
(Martin, 19701980; Price, 10791083)
30. (D) Spinocerebellar ataxia type 6 typically
occurs at a later age than other spinocerebellar 34. (A) DRPLA is caused by expanded polygluta-
ataxias, which occur between the ages of 24 and mine tracts in the coding region of huntingtin.
63 years. It accounts for 5.9% of autosomal dom- The gene defect is located on chromosome 12 and
inant cerebellar ataxia in Japan and 13% in the gene product is atrophin. (Martin, 19701980;
Germany. The disorder is characterized by gait Price et al., 10791083)
354 11: Neurogenetics

35. (E) Spinocerebellar ataxia 3 (SCA-3) or Machado both within and between families. Point muta-
Joseph disease is characterized by ataxia and tions in mitochondrial DNA tend to be inherited
lack of coordination. The gene defect is located through females, whereas deletions of mito-
on chromosome 14 and the gene product is chondrial DNA tend to be sporadic events in
ataxin-3. (Martin, 19701980; Price, 10791083) isolated individuals. Mitochondrial disorders
include the MERRF syndrome (myoclonic
36. (B) Spinal bulbar muscular atrophy is an X- epilepsy and ragged-red fibers), the MELAS
linked illness caused by expanded CAG repeats syndrome (mitochondrial encephalomyopathy,
in the coding region of the androgen receptor lactic acidosis, and stroke), Leber hereditary
gene. (Martin, 19701980; Price, 10791083) optic atrophy, and Leighs encephalopathy (Bird,
117)
37. (D) Mitochondrial disorders can affect virtually
every tissue. However, skeletal muscles and the 39. (C) In X-linked disorders, heterozygote female
brain are most often affected. Maternal trans- carriers are usually clinically normal, although
mission occurs, since the maternal ovum is the they may occasionally have mild manifestations
source of most of an individuals mitochondria. of the disease. In X-linked recessive disorders,
The clinical findings depend on the proportion each son of a carrier female is at 50% risk for the
of normal to abnormal mitochondria in a given disease. Each daughter of a carrier female is at
patient. This phenomenon is called hetero- 50% risk for also being a carrier. If an affected
plasmy. The rate of heteroplasmy differs, often male has children, his daughters are at 100%
drastically, among maternal family members, risk for being carriers (they must inherit his X
and the proportion of abnormal mitochondria chromosome) and his sons are at no risk to
may vary from one organ to another in the same inherit the mutation (because they must inherit
patient. Also, while one might assume that the only the Y chromosome from their father). Thus,
more mutant DNA a cell has the more abnor- X-linked recessive disease shows almost exclu-
malities it will exhibit, in practice the cell devel- sively affected males in multiple generations
ops the disease when the proportion of mutant with transmission through normal carrier females
mitochondrial DNA reaches a threshold. Below and never shows male-to-male transmission.
this threshold, the cell is normal. This threshold Examples of X-linked recessive neurological dis-
varies among different tissues (some are more orders include adrenoleukodystrophy, Pelizaeus
sensitive to energy deficiency than others) and Merzbacher disease, DuchenneBecker muscu-
different mutations. (Conneally, 120) lar dystrophy, Kennedy spinobulbar muscular
atrophy, fragile-X mental retardation, Emery
38. (D) Mitochondrial DNA codes for 13 proteins Dreifuss muscular dystrophy, and Fabry lipid
involved in oxidative phosphorylation, riboso- storage disease. (Bird, 117)
mal, and transfer RNAs. Mitochondrial DNA
is inherited in the cytoplasm surrounding a 40. (B) Autosomal recessive disorders are usually
mothers egg but not inherited from the sperm seen in only one generation, typically among
of the father. Therefore, mitochondrial disorders siblings; both males and females can be affected.
are transmitted only by mothers and never by Examples of autosomal recessive neurological
fathers. Male and female children can both be disorders are phenylketonuria, infantile spinal
affected. The disease has the potential to appear muscular atrophy, TaySachs disease, Wilson
in all children of an affected mother. Each child disease, metachromatic leukodystrophy, ataxia
of an affected mother may vary in the number of telangiectasia, Lafora body myoclonic epilepsy,
mitochondria containing the DNA mutation he Canavan disease, ceroid lipofuscinoses, Friedreich
or she has inherited. Furthermore, the propor- ataxia, and NiemannPick disease. (Bird, 117)
tion of mutant mitochondria may vary consid-
erably from cell to cell in any given affected indi- 41. (E) In autosomal dominant disorders, a muta-
vidual. Therefore, mitochondrial disorders often tion occurring in a single gene on any of the 22
show extreme variability in clinical expression autosomes can produce clinical symptoms or
Answers: 3545 355

signs. The carrier of a single mutation on one in mitochondrial encephalopathy, lactic acidosis,
chromosome is called a heterozygote. Each child and stroke-like episodes (MELAS). It is a mater-
of an affected person has a 50% risk of inheriting nally inherited syndrome caused by mutations
the mutation and potentially developing the dis- in mitochondrial DNA characterized by stroke
ease. Males and females are equally affected, before the age of 40 years, encephalopathy char-
the disease appears over multiple generations, acterized by seizures or dementia, and blood
and heterozygote mothers or fathers pass the lactic acidosis or ragged-red fibers in skeletal
gene on with equal risk to sons or daughters. muscle biopsy specimens. The mitochondrial
Examples of autosomal dominant neurological mutations that result in MELAS cause defects in
disorders include neurofibromatosis (NF-I and respiratory chain enzymes, particularly com-
II), myotonic dystrophy (1 and 2), tuberous scle- plex I. Substitution of an adenine for guanine at
rosis, juvenile myoclonic epilepsy, Huntington nucleotide position 3243 (A3243G) in the gene
disease, benign neonatal convulsions, and sev- encoding tRNALeu(UUR) accounts for 80% of the
eral forms of hereditary ataxias. With autoso- cases. Spontaneous A3243G mutations are rare.
mal recessive inheritance, the heterozygote car- Phenotypic expression of the A3243G mutation
riers of a single mutation are essentially always is variable. Although MELAS is the most com-
clinically normal. However, individuals who mon phenotype for mitochondrial A3243G, the
have inherited a mutation in the same gene from mutation can present as chronic progressive
both parents (homozygotes) will show clinical external ophthalmoplegia, KearnsSayre syn-
manifestations of the disease. If both parents are drome, or diabetes mellitus with or without
carriers of a mutation in the same gene, then deafness. Other genetic defects that can cause
each of their children has a 25% risk for being MELAS include a mutation at position 3260 and
homozygous and having the disease. (Bird, 117) the C3256T mutation in the tRNALeu(UUR) gene,
as well as large-scale mitochondrial DNA dele-
42. (A) Chronic progressive external ophthalmo- tions. (Meschia, 114132)
plegia (CPEO) is a mitochondrial disorder that
usually has its onset before the age of 30 years. 45. (D) Mutations in NOTCH3, a gene encoding a
Affected patients develop slowly progressive, large transmembrane receptor, cause cerebral
symmetric or asymmetric, usually nonfatigable autosomal dominant arteriopathy with subcorti-
ptosis in association with external ophthalmo- cal infarcts and leukoencephalopathy (CADASIL).
plegia. Diplopia occurs uncommonly. The most CADASIL mutations are highly stereotyped mis-
common mutation found in patients with CPEO sense mutations within epidermal growth factor
is a mitochondrial DNA deletion, which is found (EGF)-like repeats in the extracellular domain of
in 70% of patients. Definitive diagnosis is pro- NOTCH3. Mutations lead to loss or gain of a cys-
vided by muscle biopsy in most cases of CPEO. teine, thereby creating an odd number of cysteines
It is negative in only approximately 10% of within a given EGF domain). Most cases are inher-
patients with mitochondrial DNA mutations. ited in an autosomal dominant fashion, but a de
(Amato, 636637) novo symptomatic mutation (Arg182Cys) has
been reported. Stroke, dementia, psychiatric ill-
43. (D) Patients with chronic progressive external ness, and migraines are common features of
ophthalmoplegia may develop ptosis, ophthal- CADASIL. In a study of 102 patients from 29 fam-
moplegia with and without extremity weakness, ilies, 71% presented with recurrent transient
and elevated cerebrospinal fluid protein, but ischemic attacks (TIAs) or ischemic stroke (mean
they lack pigmentary retinopathy, cardiac con- age at onset, 46 years). Cognitive deficits were
duction defects, or other systemic manifesta- present in 48% of patients. More than 80% of those
tions observed in KearnsSayre syndrome. with dementia also had a gait disorder, urinary
(Amato, 636637) incontinence, or both. A total of 39% of patients
had a history of migraine, and 87% of these had
44. (C) The patient described in the vignette car- migraine with aura. The level of disability from
ries the A3243G mitochondrial mutation seen the disease varied considerably both within and
356 11: Neurogenetics

among pedigrees. Fewer than half the patients that ranges between 1 and 10 per 10,000 new
above 60 years of age could walk without assis- users. Observations in identical twins and in
tance. MRI of the brain is an essential screening families suggest a genetic association. A break-
test for CADASIL and presymptomatic carriers of through in the area of pharmacogenetics came in
a CADASIL mutation. Abnormalities in the white 2004 when Chung and colleagues reported a
matter can be observed on MRI long before 100% prevalence of the human leukocyte anti-
patients present with stroke or TIA. (Meschia, gen HLA-B*1502 allele among 44 Han Chinese
114132) in Taiwan with CBZ-induced SJS, compared
with a frequency of this allele of only 3% among
46. (D) Mutations in the COL4A1 gene, which 101 CBZ-tolerant patients (odds ratio [OR] 2504,
encodes procollagen type IV alpha 1, have been 95% confidence interval [CI] 12649,522). The
associated with autosomal dominant small ves- allele was present in 9% of healthy controls with-
sel disease and hemorrhagic stroke. The gene is out a history of CBZ use. (Chung, 486; Franciotta,
also associated with autosomal dominant poren- 144149)
cephaly and infantile spasms. Tortuosity of reti-
nal vessels is commonly seen on funduscopic 49. (B) Six of the known genes associated with
examination. Brain MRI shows lacunar infarcts, Parkinson disease (PD) were localized after link-
microbleeds on gradient imaging, leukoaraiosis, age analysis in large families, and mutations
and dilated perivascular spaces. Head trauma have been identified in the -synuclein (SNCA),
due to parturition, sporting injuries, and anti- PARKIN, PINK1, DJ-1, LRRK2, and ATP13A2
coagulation are risk factors for intracranial hem- genes. While mutations in the SNCA and DJ-1
orrhage in patients who harbor a COL4A1 gene genes are extremely rare, mutations in the
mutation. (Meschia, 114132) PARKIN, PINK1, and LRRK2 genes combined
account for about 3% of all patients and are thus
47. (B) The factor V Leiden mutation is a single- more likely to be encountered in clinical practice.
base substitution (G1691A) in the factor V gene Mode of inheritance is considered autosomal
that leads to the sequence change of Arg506Gln. dominant for -synuclein and LRRK2 and auto-
The mutation destroys a cleavage site for the somal recessive for PARKIN, PINK1, DJ-1, and
inactivating enzyme known as activated pro- ATP13A2. This is in agreement with functional
tein C. Factor V Leiden is the most common findings suggesting a gain-of-function mecha-
inherited risk factor for deep venous thrombosis. nism for dominant and a loss-of-function mech-
The factor V Leiden mutation is also a risk fac- anism for recessive forms. (Lohmann, 9113)
tor for cerebral venous thrombosis. The odds
for cerebral venous thrombosis rise nearly eight- 50. (E) The SNCA (-synuclein) gene was the first
fold with the factor V G1691A mutation. (Meschia, to be unequivocally associated with familial
114132) Parkinson disease (PD). SNCA-linked cases of
PD often result in a complex phenotype with
48. (B) Immune-mediated cutaneous hypersensi- additional features, such as early dementia,
tivity reactions are the most common idiosyn- autonomic dysfunction, and premature death.
cratic reactions to antiepileptic drugs (AEDs) Interestingly, the severity of the phenotype
and affect 5% to 15% of patients started on treat- appears to depend on gene dosage, and patients
ment with carbamazepine (CBZ), phenytoin with SNCA duplications clinically present with
(PHT), phenobarbital, or lamotrigine (LTG). These classic PD more often than those with triplica-
reactions usually consist of mild erythematous tions. Although the phenotypic spectrum can
or maculopapular rashes. However, the same be remarkably broad, those with triplications
AEDs are also associated with a risk of poten- are often characterized by fast disease progres-
tially life-threatening StevensJohnson syn- sion, marked dementia, and a reduced life span.
drome (SJS), toxic epidermal necrolysis (TEN), SNCA is abundantly expressed in neurons,
and drug-related rash with eosinophilia and where it is believed to participate in the matu-
systemic symptoms (DRESS), with a frequency ration of presynaptic vesicles and to function as
Answers: 4656 357

a negative coregulator of neurotransmitter release. tion of increased DJ-1 levels in the cerebrospinal
SNCA localizes to the nucleus and presynaptic fluid of patients with sporadic PD: this was most
nerve terminals. Outside the nucleus, SNCA is pronounced in the early stages of the disease.
peripherally attached to vesicles or is freely (Bonifati, 159160; Lohmann, 9113)
diffusible in the cytoplasm, with mutants exhibit-
ing increased nuclear targeting in cell culture. 54. (E) Homozygous and compound heterozygous
(Lohmann, 9113) mutations in a predominantly neuronal P-type
ATPase gene (ATP13A2) have recently been
51. (A) Homozygous or compound heterozygous demonstrated in the two identified families with
mutations in the recessive Parkinson disease the rare KuforRakeb syndrome (KRS), a form of
(PD) gene PARKIN are unequivocally associated recessively inherited atypical parkinsonism. KRS
with heritable PD and represent a common cause is clinically characterized by subacute, juvenile-
(10% to 20%) of early-onset PD. Alterations in onset, levodopa-responsive PD, pyramidal signs,
PARKIN are spread over the entire gene and dementia, a supranuclear gaze palsy, along with
include deletions and duplications of one or globus pallidus atrophy and later generalized
more exons in more than 50% of the reported brain atrophy. Another homozygous mutation
cases. Overall, carriers of the PARKIN mutation was detected in a patient with juvenile parkin-
present with classic PD but with an earlier age sonism (onset at the age of 12 years), levodopa-
of onset, a slower disease progression, and an responsive severe akinetic-rigid parkinsonism,
excellent and sustained response to levodopa. levodopa-induced motor fluctuations and dysk-
(Lohmann, 9113) inesias, severe visual hallucinations, supranu-
clear vertical gaze palsy, and diffuse brain atro-
52. (D) Carriers of the PINK1 mutation are clini- phy but no pyramidal deficit or dementia.
cally indistinguishable from carriers of the (Lohmann, 9113)
PARKIN mutation with the possible exception of
a higher rate of psychiatric symptoms. (Lohmann, 55. (E) Autosomal recessive cerebellar ataxias
9113) (ARCAs) are a heterogeneous group of neuro-
logical disorders involving both the central and
53. (D) Four chromosomal loci (PARK2, PARK6, peripheral nervous system and in some case other
PARK7, and PARK9) associated with autosomal systems and organs. Onset is usually before the
recessive, early-onset parkinsonism are known. age of 20 years. ARCAs are divided into four main
The PARK7 locus was mapped to chromosome groups: degenerative ataxias (Friedreich ataxia),
1p36 in a large family from a genetically iso- congenital ataxias, metabolic ataxias (Wilson dis-
lated population in the Netherlands; this linkage ease, metachromatic leukodystrophy), and atax-
was confirmed in an Italian family. By positional ias with DNA repair defects. Ataxia telangiectasia
cloning within the refined PARK7 critical region, is the most common recessive ataxia in children
mutations were recently identified in the DJ-1 under 5 years of age. It is characterized by ataxia
gene in the two PARK7-linked families. The and oculocutaneous telangiectases appearing
function of DJ-1 remains largely unknown. DJ-1 between the age of 2 and 8 years and is associated
is ubiquitously expressed and was initially with immunodeficiency leading to recurrent
described in association with oncogenesis and infections, endocrine and skin abnormalities, radi-
male rat infertility. However, the protein has ation sensitivity, and a predisposition for malig-
also been shown to confer chaperone-like activ- nancies. Ataxia telangiectasia is due to mutations
ity and to function as an intracellular sensor of in the ataxia telangiectasia mutated gene (ATM)
oxidative stress. The oxidation of DJ-1 seems to involved in DNA repair. (Manto, 419429)
play a critical role through translocation of the
protein to mitochondria in response to oxidative 56. (B) The patient described in this vignette has
stress, as demonstrated in mouse and Drosophila sign and symptoms suggestive of X-linked cere-
models. The neuroprotective role of DJ-1 against bellar ataxia. Fragile-X tremor ataxia syndrome
oxidative stress was also supported by the detec- (FXTAS) is representative of this group. The
358 11: Neurogenetics

disorder usually starts after age 50 years. Pa- associated tinnitus. Genome scan mapped the
tients exhibit combinations of kinetic tremor, locus of EA7 to chromosome 19q13. The respon-
ataxia of gait, parkinsonism, autonomic dys- sible gene has not been identified.
function, polyneuropathy, and cognitive deficits. Episodic ataxia type 1 (EA1) is an autosomal
The age of onset of tremor or ataxia inversely dominant condition characterized by brief
correlates with the repeat size (CGG triplet). episodes of ataxia with interictal myokymia.
Brain MRI in FXTAS shows, in particular, hyper- Triggered by exertion, stress, or startle, and last-
intense lesions in the middle cerebellar pedun- ing from seconds to minutes, these attacks of
cles on T2-weighted sequence. (Manto, 419429) ataxia usually diminish with age and may spon-
taneously resolve in the teens. EA1 is caused by
57. (A) Some clinical features have specific value for mutations in KCNA1, located on chromosome
predicting a gene defect in spinocerebellar ataxia 12, which encodes Kv1.1, a human homolog of
(SCA). Slowing of saccades is seen in SCA2; oph- the Shaker voltage-gated potassium channel in
thalmoplegia is seen in SCA1, SCA2,and SCA3; Drosophila.
pigmentary retinopathy is seen in SCA7; spas- EA2 is a dominantly inherited neurological
ticity is seen in SCA3; dyskinesias associated with disorder characterized by bouts of vertigo and
a mutation in the fibroblast growth factor 14 ataxia with interictal nystagmus and progressive
(FGF14) gene, cognitive impairment, and behav- ataxia. Episodes are typically triggered by exer-
ioral symptoms are seen in SCA17 and DRPLA; cise or stress, and they are often dramatically
seizure is seen in SCA10, SCA17, and DRPLA; relieved by treatment with the carbonic anhy-
and peripheral neuropathy is seen in SCA1, drase inhibitor acetazolamide. Flunarizine and
SCA2, SCA3, SCA4, SCA8, SCA18, and SCA25. 4-aminopyridine have also been reported to be
(Manto, 419429) effective in EA2. Migraine headaches occur in
about 50% of patients with EA2, and there is over-
58. (B) Seizures may occur in SCA10, SCA17, and lap in the clinical features of EA2, familial hemi-
DRPLA. (Manto, 419429) plegic migraine type 1 (FHM1), basilar migraine,
and progressive ataxia. EA2 and FHM1 are
59. (C) Myokemia can occur in spinocerebellar ataxia caused by mutations in the gene CACNA1A,
type 5. (Manto, 419429) which codes for the alpha-1A subunit of the P/Q-
type of voltage-gated calcium channel.
60. (D) Linkage studies originally mapped some EA3 was described in a single large Canadian
families with frontotemporal dementia with family with episodic vertigo, nausea, tinnitus,
parkinsonism (FTDP-17) to chromosome 17q21- ataxia, and migraine. Interictal myokymia was
22. Subsequent analysis of this region identified observed in some; none had nystagmus or base-
mutations in the gene encoding microtubule- line ataxia. There is much clinical overlap between
associated protein tau (MAPT). FTDP-17 patients EA3 and MAV. The disease locus for EA3 is dis-
exhibited frontotemporal atrophy with neuronal tinct from EA1 and EA2 and was recently mapped
loss, gray and white matter gliosis, and superfi- to chromosome 1q42. The responsible gene has
cial cortical microvacuolation. Reported cases of not been identified.
FTDP-17 all have neuropathological features of EA4, also called familial periodic vestibu-
intraneuronal hyperphosphorylated tau inclu- locerebellar ataxia, is an autosomal dominant
sions, with glial tau inclusions observed in some disorder characterized by episodes of vertigo
families. (Sikkink, 693698) and ataxia beginning in the third to sixth
decades of life. Patients may have interictal nys-
61. (E) Episodic ataxia 7 (EA7) was described in a tagmus and mild ataxia similar to EA2, or they
single family with episodic vertigo without tin- may be completely normal in between attacks.
nitus, weakness, dysarthria, and ataxia lasting The attacks typically last hours and are not
from hours to days typically triggered by exer- relieved by acetazolamide. The most consistent
tion or excitement, with onset before age 20. symptom is the inability to suppress the
There is no interictal finding. Neither is there vestibuloocular reflex when objects move in the
Answers: 5762 359

periphery. The responsible gene has not been standing the neurogenerative process. Ann N Y Acad
identified. (Jen, 37) Sci. 2000;920:74-83.
Haitjema T, Westermann CJ, Overtoom TT, Timmer R,
62. (A) Familial hemiplegic migraine (FHM) diag- Disch F, Mauser H, et al. Hereditary hemorrhagic telang-
nosis is based on the presence of one-sided iectasia (OslerWeberRendu disease): new insights in
pathogenesis, complications, and treatment. Arch Intern
motor weakness during the aura phase and sim-
Med. 1996;156:714-719.
ilar attacks in at least one first-degree or sec-
Hammans SR. The inherited ataxias and the new genetics.
ond-degree family member. FHM is genetically J Neurol Neurosurg Psychiatr. 1996;61:327-332.
heterogenous with mutations in CACNA1A, a Huq AH, Chugani DC, Hukku B, Serajee FJ. Evidence of
gene coding for the P/Q-type calcium channel somatic mosaicism in SturgeWeber syndrome. Neurology.
alpha subunit (FHM type1), ATP1A2, a gene 2000;59:780-782.
coding for Na/K-adenosine triphosphatase Jen JC. Recent advances in the genetics of recurrent vertigo
(FHM type 2), and SCNA1, a gene coding for and vestibulopathy. Curr Opin Neurol. 2008;21:37.
sodium channel alpha subunit (FHM type 3). Karnes PS. Neurofibromatosis: a common neurocutaneous
(Ropper, 288293) disorder. Mayo Clin Proc. 1998;73:1071-1076.
Lohmann, Katja; Klein, Christine. Genetics of Parkinson
disease. Continuum: Lifelong Learning in Neurology. 2008;
14(2):Neurogenetics)90-113.
REFERENCES Manto M, Marmolino D. Cerebellar ataxias. Curr Opin
Neurol. 2009;22(4):419-429.
Amato AA, Russell JA. Neuromuscular disorders. New York: Martin JB. Molecular basis of the neurodegenerative dis-
McGraw-Hill Medical; 2008. orders. N Engl J Med. 1999;340:1970-1980.
Berger J, Moser HW, Forss-Petter S. Leukodystrophies: recent Meschia JF. Genetics of stroke. Continuum: Lifelong Learning
developments in genetics, molecular biology, pathogene- in Neurology. 2008;14(2):(Neurogenetics)114-132.
sis and treatment. Curr Opin Neurol. 2001;14:305-312. Price DL, Sisodia SS, Borchelt DR. Genetic neurodegener-
Bird TD. The language and basic concepts of medical ative diseases: the human illness and transgenic models.
genetics for neurologists. From Genetics in Neurology. Science. 1998;282:1079-1083.
AAN Courses 2002. Roach ES. Neurocutaneous syndromes. Neurol Clin North
Bonifati V, Rizzu P, Squitieri F, Krieger E, Vanacore N, van Am. 1992;39:591-620.
Swieten JC, et al. DJ-1 (PARK7), a novel gene for auto- Ropper AH, Samuels MA. Headache and other craniofacial
somal recessive, early onset parkinsonism. Neurol Sci. pains. In: Ropper AH, Samuels MA, eds. Adams and
2003;24:159-160. Victors Principles of Neurology. 9th ed. Available at http://
Chaudhry V. Peripheral neuropathy. In: Fauci AS, www.accessmedicine.com/content.aspx?aID=3630946
Braunwald E, Kasper DL, Hauser SL, Longo DL, Jameson Sikkink S, Rollinson S, Pickering-Brown SM. The genetics
JL, et al. Harrisons Principles of Internal Medicine. 17th ed. of frontotemporal lobar degeneration. Curr Opin Neurol.
Chapter 379. Available at http://www.accessmedi- 2007;20:693-698.
cine.com/content.aspx?aID=2907120 Souayah N, Seltzer WK, Brannagan TH, Chin RL, Sander
Chung WH, Hung SI, Hong HS, Hsih MS, Yang LC, Ho HC, HW. Rare myelin protein zero sequence variant in late
et al. Medical genetics: a marker for StevensJohnson onset CMT1B. J Neurol Sci. 2007;263(12):177-179.
syndrome. Nature. 2004;428:486. Sparagana SP. Roach ES. Tuberous sclerosis complex. Curr
Conneally M. Congenital myasthenic syndromes. Opin Neurol. 2000;13:115-119.
Continuum: Lifelong Learning in Neurology. Neurogenetics. Stam AH, van den Maagdenberg AM, Haan J, Terwindt
2000;6(6):9-34. GM, Ferrari MD. Genetics of migraine: an update with
Evidente VG, Gwinn-Hardy KA, Caviness JN, Gilman S. special attention to genetic comorbidity. Curr Opin
Hereditary ataxias. Mayo Clin Proc. 2000;75:475-490. Neurol. 2008;21:288-293.
Franciotta D, Kwan P, Perucca E. Genetic basis for idio- Tan EK, Ashizawa T. Genetic testing in spinocerebellar atax-
syncratic reactions to antiepileptic drugs. Curr Opin ias: defining a clinical role. Arch Neurol. 2001;58:191-195.
Neurol. 2009;22:144-149. Taylor JP et al. Repeat expansion and neurological dis-
Goedert M, Ghetti B, Spillantini MG. Tau gene mutations eases. From Genetics in Neurology. AAN Courses 2002.
in frontotemporal dementia and parkinsonism linked Valdmanis PN, Rouleau GA. Genetics of familial amy-
to chromosome 17 (FTDP-17). Their relevance for under- otrophic lateral sclerosis. Neurology. 2008;70(2):144-152.
This page intentionally left blank
CHAPTER 12

Neuroophthalmology
Questions

1. Which of the following is true of Balint syndrome? 5. Which of the following conditions may cause
upbeat nystagmus?
(A) It results from bilateral extensive frontal
damage. (A) Spasmus nutans
(B) It is characterized by a difficulty initiating (B) ArnoldChiari malformation
slow eye movement. (C) Phenytoin use
(C) It is characterized by the presence of (D) Posterior fossa tumor
simultagnosia. (E) Suprasellar mass lesions
(D) It is usually associated with preservation
of visual field. 6. Seesaw nystagmus is frequently associated with
(E) Visual guidance pointing is preserved.
(A) spasmus nutans
2. What type of nystagmus is frequently associ- (B) ArnoldChiari malformation
ated with vertigo and tinnitus? (C) phenytoin use
(D) posterior fossa tumor
(A) Downbeat nystagmus
(E) suprasellar mass lesions
(B) Seesaw nystagmus
(C) Spasmus nutans 7. The eye movement that brings objects of inter-
(D) Upbeat nystagmus est onto the fovea is called
(E) Vestibular nystagmus
(A) a saccade
3. What type of nystagmus is frequently associ- (B) smooth pursuit
ated with ArnoldChiari malformation? (C) vergence eye movement
(D) nystagmus quick phase
(A) Downbeat nystagmus
(E) optokinetic movement
(B) Seesaw nystagmus
(C) Spasmus nutans 8. The optokinetic system is responsible for
(D) Upbeat nystagmus
(A) bringing objects of interest to the fovea
(E) Vestibular nystagmus
(B) holding images of the seen world steady
4. Head turning and head nodding, is usually asso- on the retina during sustained head
ciated with rotation
(C) holding the image of a small moving
(A) downbeat nystagmus
target on the fovea
(B) seesaw nystagmus
(D) bringing an object of interest to the fovea
(C) spasmus nutans
(E) holding images of the seen world steady
(D) upbeat nystagmus on the retina during brief head rotations
(E) vestibular nystagmus

361
362 12: Neuroophthalmology

9. Seesaw nystagmus is caused by 14. ArgyllRobertson pupils may be caused by


(A) frontal stroke (A) chronic ethanol abuse
(B) cerebellar tumor (B) Parkinson disease
(C) ArnoldChiari malformation (C) hydrocephalus
(D) B12 deficiency (D) hypertension
(E) congenital aqueductal stenosis (E) Huntington disease

10. The most likely cause of convergenceretraction 15. Which of the following vitamin deficiencies does
nystagmus in a 10-year-old boy is not cause optic atrophy?
(A) congenital aqueductal stenosis (A) Vitamin C
(B) pinealoma (B) Vitamin B12
(C) brainstem vascular malformation (C) Pyridoxine
(D) multiple sclerosis (D) Riboflavin
(E) head trauma (E) Folic acid

11. Opsoclonus in an infant is most likely seen in a 16. Which of the following is not true of ocular
case of myasthenia?
(A) hyperosmolar coma (A) Ocular involvement occurs in 90% of
(B) brainstem encephalitis myasthenic patients in the course of the
(C) neuroblastoma disease.
(D) lung cancer (B) Ocular symptoms account for 75% of ini-
tial complaints.
(E) toxic encephalopathy
(C) Approximately 20% of patients with ocu-
12. Ocular myoclonus caused by a lesion in the tri- lar onset of myasthenia progress to
angle of Guillain and Mollaret involves which of involve other muscle groups within 2
the following anatomical sites? years.
(D) Only one third of patients with ocular
(A) Interstitial nucleus of Cajal myasthenia have positive acetylcholine
(B) Red nucleus receptors antibodies.
(C) Ipsilateral dentate nucleus (E) The major ophthalmological complaints
(D) Contralateral inferior olive of ocular myasthenia are ptosis and
(E) Midbrain diplopia.

13. The most common cause of nontraumatic oculo- 17. Which of the following muscles is the most
motor nerve palsy with pupillary involvement is affected in Graves disease?

(A) diabetes (A) Superior oblique


(B)basilar artery aneurysm (B) Inferior rectus
(C) schwannoma (C) Medial rectus
(D) aneurysm at the junction of the posterior (D) Lateral rectus
communicating artery and the internal (E) Superior rectus
carotid artery
(E) cavernous sinus thrombosis
Questions: 928 363

18. Which of the following is true of ophthalmo- 23. Posterior subcapsular cataract is associated with
plegic migraine?
(A) neurofibromatosis type I
(A) The onset is in the fourth decade of life. (B) neurofibromatosis type II
(B) The abducens nerve is more often (C) tuberous sclerosis
affected than the oculomotor nerve. (D) Von HippelLindau disease
(C) The ophthalmoplegia resolves when the (E) SturgeWeber syndrome
headache clears.
(D) Pupils and accommodation are frequently 24. Retinal hemangioblastoma is associated with
involved.
(A) neurofibromatosis type I
(E) The ophthalmoplegia is contralateral to
the headache. (B) neurofibromatosis type II
(C) tuberous sclerosis
19. Retinal artery obstruction by a platelet fibrin (D) Von HippelLindau disease
embolus is associated with (E) SturgeWeber syndrome
(A) bacterial endocarditis
25. Retinal astrocytic hamartomas are associated with
(B) bright orangeyellow refractile emboli
(C) a white intra-arterial plug lodged at the (A) neurofibromatosis type I
bifurcation of the arter (B) neurofibromatosis type II
(D) a friable mass (C) tuberous sclerosis
(E) graywhite nonrefractible emboli (D) Von HippelLindau disease
(E) SturgeWeber syndrome
20. Retinal artery obstruction by a fat emboli is asso-
ciated with 26. Racemose angioma is associated with
(A) a long bone (A) tuberous sclerosis
(B) bright orangeyellow refractile emboli (B) Von Hippel-Lindau disease
(C) a white intra-arterial plug lodged at the (C) SturgeWeber syndrome
bifurcation of the artery (D) ataxiatelangiectasia
(D) a friable mass (E) WyburnMason syndrome
(E) graywhite nonrefractible emboli
27. Lisch nodules are associated with
21. Choroidal hemangioma is associated with
(A) neurofibromatosis type I
(A) neurofibromatosis type I (B) neurofibromatosis type II
(B) neurofibromatosis type II (C) tuberous sclerosis
(C) tuberous sclerosis (D) Von HippelLindau disease
(D) Von HippelLindau disease (E) SturgeWeber syndrome
(E) SturgeWeber syndrome
28. What is the most likely finding in the funduscopic
22. Bilateral bulbar and conjunctival telangiectasia examination of the early stage of Leber disease?
is associated with
(A) Optic nerve atrophy
(A) neurofibromatosis type II (B) Papilledema
(B) tuberous sclerosis (C) Hyperemic optic nerve with telangiectatic
(C) Von HippelLindau disease capillaries
(D) SturgeWeber syndrome (D) Optic disk vasculitis
(E) ataxiatelangiectasia (E) Optic nerve drusen
364 12: Neuroophthalmology

29. A 25-year-old man with a 3-year history of dia-


betes underwent a routine ophthalmological
examination. His visual acuity was normal. On
funduscopic examination, the ophthalmologist
noted the following in both eyes: glistening hya-
line bodies and the absence of disk hyperemia,
exudates, or hemorrhage. The disk borders were
irregular and the cup was absent. The retinal
vessels had a central origin and were trifurcated.
Spontaneous venous pulsations were present.
This funduscopic report is consistent with
(A) papilledema from increased intracranial
pressure
(B) drusen FIG. 12-1. See color insert. (Reproduced with permission from Savino PJ,
Danesh-Meyer HV. Neuroopthalmology. New York: McGraw-Hill; 2003.)
(C) optic neuritis
(D) early diabetic retinopathy
(E) anterior ischemic optic neuropathy
(C) central retinal artery occlusion
30. Downbeat nystagmus is a feature of (D) retinal vein detachment
(A) episodic ataxia type II (E) central retinal vein occlusion
(B) pituitary tumors
(C) progressive supranuclear palsy 34. Figure 12-2 illustrates the fundoscopic findings
in a 34-year-old man who developed right visual
(D) healthy subjects
loss of sudden onset. The most likely cause of his
(E) optic neuritis visual loss is
31. Pendular seesaw nystagmus is a feature of (A) platelet fibrin emboli
(B) cholesterol emboli
(A) episodic ataxia type II
(C) calcium emboli
(B) pituitary tumors
(D) septic emboli
(C) progressive supranuclear palsy
(E) myxomatous emboli
(D) healthy subjects
(E) optic neuritis

32. Square wave jerks are a feature of


(A) episodic ataxia type II
(B) pituitary tumors
(C) optic neuritis
(D) healthy subjects
(E) drug intoxication

33. A 57-year-old woman developed a sudden pain-


less loss of vision in the right eye. Her fundus-
copic examination is illustrated in Figure 12-1.
The most likely diagnosis is
(A) diabetic neuropathy FIG. 12-2. See color insert. (Reproduced with permission from Savino PJ,
Danesh-Meyer HV. Neuroopthalmology. New York: McGraw-Hill; 2003.)
(B) acute optic neuritis
Questions: 2940 365

35. Which of the following is true about optic neu- 38. Which of the following is inconsistent with the
ritis? diagnosis of cortical blindness?
(A) Typically, it occurs after the age of 60 (A) Loss of vision in both eyes
years. (B) Preservation of extraocular movements
(B) Visual acuity is preserved until late in (C) Retinal integrity
disease progression. (D) Absence of pupillary constriction to light
(C) It reduces color vision. (E) Preserved pupillary constriction to con-
(D) Magnetic resonance imaging (MRI) of the vergence
head is usually normal.
(E) Oral corticosteroids are effective in has- 39. A 50-year-old man developed a subacute mild
tening recovery from the acute phase and headache. Neurological examination demon-
improving the long-term prognosis. strated right lid ptosis, right miosis, and right
anhydrosis. Cocaine and hydroxyamphetamine
36. Which of the following is false about anterior drops failed to dilate the affected pupil. The
ischemic neuropathy? right pupil dilated on 2% epinephrine drops and
became larger than the left. Among the follow-
(A) It is characterized by bilateral painful
ing diseases, which is consistent with the above
visual loss.
findings?
(B) Migraine is a risk factor in young
patients. (A) Right hypothalamic infarction
(C) Spontaneous improvement of visual loss (B) Right lateral medullary infarction
may occur. (C) Malignant mass in the apex of the right
(D) Funduscopic examination may show lung
flame-shaped hemorrhages near the optic (D) Right brachial plexus trauma
disk margin. (E) Right internal carotid artery dissection
(E) A small optic disk is a predisposing factor
for developing the nonarteritic form of 40. Which of the following supports the diagnosis of
ischemic optic neuropathy. a dilated pupil from Adie syndrome rather than
pharmacologically induced mydriasis?
37. Disturbance of depth perception is produced by
(A) Oculomotor nerve palsy
a lesion located in the
(B) Ptosis
(A) optic chiasm (C) Segmental contraction of the pupils on
(B) optic tract slit-lamp examination
(C) optic radiation (D) Ophthalmoplegia
(D) optic nerve (E) Diplopia
(E) lateral geniculate body
Answers and Explanations

1. (C) Balint syndrome is an acquired oculomotor low in amplitude, and of high frequency. (Kline,
apraxia caused by an extensive bilateral pari- 8285)
etooccipital lesion. There is a difficulty in initi-
ating reflexive visually guided saccades and 5. (D) Upbeat nystagmus has an up phase while
pursuit in all directions with intact vestibular the eyes are in primary position of gaze. Reported
eye movements. Other signs of Balint syndrome causes include cerebellar degeneration, multiple
include simultagnosia (inability to perceive sclerosis, brainstem stroke, posterior fossa tumor,
more than one object at a time), optic ataxia and Wernicke encephalopathy. (Kline, 8285)
(inaccurate visual guided pointing), and ocular
motor apraxia (difficulty in initiating voluntary 6. (E) Seesaw nystagmus is a nystagmus with one
saccades). These symptoms are frequently asso- eye elevated and intorted and the other de-
ciated with dementia and visual field defects. pressed and extorted, frequently associated with
(Kline, 68) suprasellar mass lesions. (Kline, 8285)

2. (E) Vestibular nystagmus is characterized by a 7. (A) Saccades are fast eye movements responsi-
mixed direction, horizontaltorsional primary ble for bringing objects of interest onto the fovea.
position nystagmus. It is of maximal amplitude Saccades are stimulated by voluntary changes in
when the gaze is directed toward the fast com- direction. They are also stimulated by sudden
ponent. The nystagmus is suppressed by visual peripheral visual, auditory, or sensory stimuli.
fixation and increased when fixation is removed. (Kline, 4748)
The fast phase usually beats away from the dam-
aged end organ. The nystagmus is usually asso- 8. (B) The optokinetic system is stimulated by sus-
ciated with tinnitus, vertigo, and deafness. tained head rotation. It is responsible for hold-
(Kline, 8285) ing images of the seen world steady on the retina
during sustained head rotation. The vestibular
3. (A) Downbeat nystagmus is characterized by system responds only to acceleration. With sus-
the occurrence of a fast phase down while the tained head rotation at a constant velocity, the
eyes are in primary position. It is usually asso- vestibular response fades and the optokinetic
ciated with lesions at the craniocervical junc- system supplements visually driven compensa-
tion, such as ArnoldChiari malformation, and tory slow-phase eye movements. (Kline, 4748)
is often accentuated during lateral downgaze.
(Kline, 8285) 9. (C) Seesaw nystagmus has been associated with
suprasellar mass lesions, midbrain stroke, mul-
4. (C) The triad of head turning, head nodding, and tiple sclerosis, head trauma, ArnoldChiari mal-
nystagmus is highly suggestive of spasmus formation, and congenital causes. (Kline, 8384)
nutans. Symptoms begin in the first 18 months of
life and resolve within the first decade of life. The 10. (B) Convergenceretraction nystagmus is a
nystagmus is horizontal or vertical, pendular, jerk convergenceretraction movement due to

366
Answers: 118 367

contraction of the extraocular muscles, especially and supranuclear inhibitory fibers as they
on attempted convergence or upward gaze. Its approach the EdingerWestphal nuclei. More
etiology may depend on age. Congenital aque- ventrally located fibers for near response are
ductal stenosis is the most likely cause in a new- spared. The classic cause of ArgyllRobertson
born. At the age of 10 years, pinealoma is the pupils is neurosyphilis. Other reported causes
most likely cause. Head trauma and brainstem include diabetes mellitus, multiple sclerosis, sar-
vascular malformations may cause conver- coidosis, and chronic alcoholism. (Kline, 141142)
genceretraction nystagmus in the 20- and 30-
year age groups, respectively. Multiple sclerosis 15. (A) Deficiency optic neuropathy is character-
(at the age of 40) and basilar artery stroke (at the ized by a progressive bilateral visual loss with
age of 50) are the most likely causes of conver- central or centrocecal scotoma and optic atrophy.
genceretraction nystagmus. (Kline, 84) Deficiencies in the following vitamins may be
responsible for optic atrophy: vitamin B12 or
11. (C) Opsoclonus is a rapid, involuntary, multi- cobalamin, vitamin B6 or pyridoxine, vitamin
vectorial, and unpredictable conjugate fast eye B1 or thiamine, niacin, vitamin B2 or riboflavin,
movement that stops during sleep. Neuroblastoma and folic acid. (Kline, 166)
is the most likely cause of opsoclonus in infants,
as a paraneoplastic phenomenon. Autoimmune 16. (C) Myasthenia involves skeletal but not visceral
brainstem encephalitis responsive to adrenocorti- neuromuscular transmission. Therefore, the major
cotropic hormone (ACTH) is also seen in infants. ophthalmological complaints are ptosis and
In adults, opsoclonus may occur as a paraneo- diplopia. Ocular involvement occurs in 90% of
plastic syndrome caused by lung, breast, or ovar- individuals with myasthenia and accounts for the
ian cancer. (Kline, 8788) initial complaint in 75% of cases. Approximately
36% of patients with ocular onset progress to
12. (B) The triangle of Guillain and Mollaret is involve other muscle groups within 2 years.
formed by the red nucleus, ipsilateral inferior Acetylcholine receptor antibodies, if present, are
olive, and contralateral dentate nucleus. (Kline, 88) diagnostic of myasthenia, but they are present in
only one third of patients with ocular myasthenia.
13. (D) In its course toward the cavernous sinus, (Kline, 473; Kupersmith, 243248)
the oculomotor nerve travels lateral to the pos-
terior communicating artery. The pupillomotor 17. (B) A restrictive myopathy of ocular muscles
fibers are situated in the periphery of the nerve may occur in Graves disease, leading to oph-
and are affected early in case of compression of thalmoparesis and diplopia. The inferior rectus is
the nerve by an aneurysm at the junction of the the most frequently involved muscle. Its fibrotic
posterior communicating and internal carotid shortening leads to elevator palsy. Abduction
arteries. This is the most common cause of iso- weakness may occur in case of involvement of
lated third nerve palsy with pupillary involve- the medial rectus, mimicking an abducens nerve
ment. In cases of ischemic lesions, as in diabetes, palsy. Superior and lateral rectus muscles and
the pathology is confined to the core of the nerve superior obliques are less frequently involved.
and spares the peripheral pupillomotor fibers. (Kline, 176; Kupersmith, 243248)
(Brazis, 168; Kline, 108)
18. (D) The onset of ophthalmoplegic migraine is
14. (A) ArgyllRobertson pupils are miotic and usually before the age of 10 years. There is always
irregular; they are characterized by absence of a history of typical migraine. The ophthalmople-
the pupillary light response and brisk pupillary gia is ipsilateral to the headache. The oculomotor
constriction to near stimuli, normal function of nerve is affected 10 to 1 over the abducens nerve.
the anterior visual pathway, and poor dilatation The pupils and accommodation are frequently
in the dark. The lesion is most likely located in involved. The ophthalmoplegia occurs at the
the region of the sylvian aqueduct in the rostral height of the headache, persisting after the head-
midbrain, interfering with the light reflex fibers ache clears. It may last days to weeks. (Kline, 207)
368 12: Neuroophthalmology

19. (C) Platelet fibrin emboli are white intra-arterial 24. (D) Von HippelLindau disease occurs in 1 of
plugs that lodge at bifurcations; they arise from 36,000 persons. It is inherited as an autosomal
ulcerative atheromas that may be located in the dominant disease with incomplete penetrance.
internal carotid artery. Cholesterol emboli are The gene is located on chromosome 3 (3q26) and
bright orangeyellow and refractile. The source has a tumor-suppressing function. Ocular man-
may be carotid or aortic atheroma. Calcium ifestations include retinal hemangioblastomas,
emboli are graywhite and nonrefractile. They which are found in both eyes in 50% of patients;
originate from the cardiac valves or the aortic 60% of patients have multiple lesions in one eye.
wall and are usually lodged in retinal arterioles (Kline, 249255)
near or on the optic disk. Septic emboli may
originate from infected cardiac valves, especially 25. (C) Tuberous sclerosis (TSC) is an autosomal
aortic or mitral valves. The heart is the source of dominant disease with an incidence ranging from
myxomatous emboli. (Kline, 212214) 1 of 6,000 to 1 of 10,000 persons. Spontaneous
mutations may occur in up to 66% of cases. The
20. (A) Long bones are the sources of fat emboli. condition is caused by defects, or mutations, on
(Kline, 212214) two genes, TSC1 and TSC2; both are believed to be
tumor suppressor genes. Only one of the genes
21. (E) Ocular manifestations of SturgeWeber syn- needs to be affected for TSC to be present. The
drome include glaucoma, which is found in 60% TSC1 gene is on chromosome 9 and produces a
of patients before the age of 2, and choroidal protein called hamartin. The TSC2 gene is on chro-
hemangioma, which is seen in 40% of patients mosome 16 and produces the protein tuberin.
with SturgeWeber syndrome. It is located ipsi- Some 75% of TSC patients have ocular lesions
laterally to the facial angioma (also referred to as including retinal astrocytic hamartomas. Multiple
port-wine stain), which is one of the criteria for lesions are found in one eye; 25% of patients have
the diagnosis of the disease. (Kline, 249255) bilateral lesions. (Kline, 249255)

22. (E) Ataxiatelangiectasia is inherited as an 26. (E) WyburnMason syndrome is also known as
autosomal recessive trait. Its gene is located on retinocephalic vascular malformation. Ocular
chromosome 11 (11q2223). The gene encodes a manifestations include arteriovenous malfor-
protein called ATM, which is important for cell mation of the retinal, orbital, and optic nerves.
cycle control and DNA repair. Ocular manifes- Arteriovenous malformation of the retina, also
tations of the disease include bilateral bulbar known as racemose angioma, is usually unilat-
conjunctivitis, telangiectasia, and ocular motil- eral and most often located in the posterior pole.
ity disturbances. These include, at the begin- Arteriovenous malformations are also found in
ning, ocular apraxia, which may progress to the central nervous system and are symptomatic
impairment of smooth pursuit and eventually in 50% of patients. (Kline, 249255)
to complete supranuclear ophthalmoplegia.
(Kline, 249255) 27. (A) Neurofibromatosis type I occurs in approx-
imately 1 of 3,000 persons. It may be inherited as
23. (B) Neurofibromatosis type II is characterized an autosomal dominant trait (the gene is located
by bilateral acoustic neurons. It occurs in 1 of on chromosome 17q12-22) and produces neu-
50,000 persons and is inherited as an autosomal rofibromin, a protein that regulates a tumor
dominant trait. The gene is located on chromo- suppressor gene named oncoprotein ras. The
some 22 (22q12), a tumor suppressor gene that mutated gene leaves the ras unopposed to stim-
suppresses a protein called schawannomin. ulate cell growth. Clinical manifestations may
Cutaneous lesions and peripheral neurofibro- involve cutaneous, ocular, neurological, and vis-
mas are rare. Ocular lesions include posterior ceral organs. Lisch nodules are ocular melano-
subcapsular cataracts, epiretinal membranes, cytic hamartomas, brown or yellow in color.
and retinal hamartomas. (Kline, 249255) Dome-shaped lesions protrude from the iris
Answers: 1931 369

surface. They are uncommon prior to age 6 but may be normal with retrobulbar involvement.
increase in number with age. Other ocular lesions The patient in this vignette is asymptomatic and
include neurofibromas of the eyelids and orbits has normal visual acuity. Optic neuritis is unlikely
and optic nerve gliomas. (Kline, 249255) to be the diagnosis.
Ischemic optic neuropathy is the most com-
28. (C) Leber hereditary optic neuropathy is a mon cause of acute painless monocular visual
maternally inherited disease linked to abnor- loss in the elderly population. It may be seen
malities in mitochondrial DNA. In the early in patients after the age of 50 years. Diabetes
stage of the disease, funduscopic examination and hypertension are predisposing factors.
may show hyperemia of the optic disk, dilata- Funduscopic examination may demonstrate
tion, and tortuosity of vessels. A classic triad is segmental disk edema. Ischemic optic neuropa-
seen in many cases of Leber hereditary optic thy is unlikely to be the diagnosis in this patient,
neuropathy, including circumpapillary telang- as he has normal visual acuity. In diabetic
iectatic microangiopathy, swelling of the nerve retinopathy, funduscopic examination may show
fiber layer around the disk (pseudoedema), and microaneurysms, hemorrhages that may occur
absence of leakage from the disk or papillary within the compact middle layers of the retina,
region on fluorescein angiography. This last dis- hard exudates, and retinal edema. Diabetic
tinguishes Leber hereditary optic neuropathy retinopathy is due to microangiopathy affecting
from a truly swollen disk. (Miller, 304308) the retinal precapillary arterioles, capillaries, and
venules. The funduscopic findings in this patient
29. (B) This vignette raises the differential diagnosis are not suggestive of diabetic retinopathy. The
of an abnormal funduscopic examination in a 25- patient in this vignette is asymptomatic, has nor-
year-old diabetic male. Although papilledema mal visual acuity and bilaterally irregular disk
from increased intracranial pressure is a medical borders with absent cup on funduscopic exami-
emergency that should not be missed, other less nation. These findings are highly suggestive of
ominous causes of abnormal disk appearance drusen. This is a congenital elevation of the optic
should be considered, such as congenital anom- disk not associated with cotton-wool spots, peri-
alies, inflammatory processes, ischemia, and dia- papillary swelling, or hemorrhage. Retinal venous
betic retinopathy. Increased intracranial pressure pulsations are preserved. Retinal vessels may
is characterized clinically by nausea, morning appear to originate from the center of the disk.
headache, transitory visual obscuration, and There are no exudates, neovascularization, or
ataxia. Visual examination may show an enlarged hyperemia. (Laskowitz, 323353)
blind spot and visual field constriction. Color
vision and visual acuity are preserved early in 30. (A) Downbeat nystagmus is a vertical jerk nys-
the disease. Funduscopy may show the absence of tagmus. It is exacerbated by looking down and
retinal venous pulsations, disk hyperemia, pre- laterally. It is poorly suppressed by visual fixa-
served cup, cotton-wool spots, exudates, and blur- tion. Downbeat nystagmus is encountered when
ring of vessels in the peripapillary area. In this lesions affect the vestibular pathways or in cases
patient, the absence of any physical sign suggest- of drug intoxication. Downbeat nystagmus is
ing increased intracranial pressure, the preserva- also a feature of episodic ataxia type II, a calcium
tion of the retinal venous pulsations, and the channelopathy. (Serra, 615618)
absence of hyperemia argue against the diagnosis
of papilledema. Optic neuritis may complicate 31. (B) Pendular seesaw nystagmus consists of ele-
the course of multiple sclerosis rather than dia- vation and intorsion of one eye and synchronous
betes. The patient may report retroorbital pain on depression and extorsion of the other eye in the
eye movement. Visual examination may demon- first half cycle, followed by change in direction
strate loss of central acuity and color discrimina- during the next half cycle. It is encountered in dis-
tion, while funduscopic examination may show eases affecting the crossing axons of the optic chi-
unilateral disk swelling. The disks appearance asm, such as pituitary tumors. (Serra, 615618)
370 12: Neuroophthalmology

32. (D) Square-wave jerks are small conjugate sac- adults below 46 years of age, where it is the most
cades that briefly take the eye away from the fix- common cause of acute optic neuropathy. Most
ation position and then return it there. It is a cases are idiopathic or associated with multiple
prominent finding in progressive supranuclear sclerosis. Its clinical features may include perioc-
palsy and also occurs in healthy subjects. (Serra, ular pain, particularly with eye movement, and
615618) progressive visual loss over several days. Visual
acuity, color vision, and visual fields are reduced
33. (C) Figure 12-1 illustrates central retinal artery early in disease onset. Funduscopy may show a
occlusion with a cherry-red spot. Nerve fiber normal optic nerve or optic disk edema. In 50% to
layer hemorrhages are seen superior to the disk. 70% of cases, MRI of the head may show white
In patients 55 years of age or older with cen- matter abnormalities identical to those seen in
tral retinal artery occlusion, giant cell arteritis multiple sclerosis. Low doses of oral prednisone
must be considered. Carotid and cardiac sources have no demonstrable efficacy in the recovery of
of emboli must be sought in central and in visual function in acute monosymptomatic optic
branch retinal artery occlusion in particular, even neuritis. Although high-dose oral or parenteral
if no retinal emboli are identified, so that appro- methylprednisolone has been shown to have an
priate treatment can be given to reduce the risk effect in hastening recovery from the acute phase
of stroke. Migraine, oral contraceptives, systemic of optic neuritis, it does not confer long-term ben-
vasculitis, congenital or acquired thrombophilia, efit on visual function. (Kaufman, 20392044)
and hyperhomocysteinemia should be consid-
ered in young patients. Internal carotid artery 36. (A) Anterior ischemic optic neuropathy (AION)
dissection should be considered when there is results from an ischemic lesion of the laminar
neck pain or a recent history of neck trauma. and prelaminar portions of the optic nerve. The
Diabetes, hyperlipidemia, and systemic hyper- disease occurs in patients over 50 years of age.
tension should be considered in all patients. Diabetes mellitus, hypertension, and giant cell
Central retinal artery occlusion presents as sudden arteritis are predisposing factors. Migraine and
profound monocular visual loss. Visual acuity is systemic vasculitis particularly increase the risk
usually reduced to counting fingers or worse, and of AION in young patients. Congenital small
the visual field is restricted to an island of vision optic disk with absent or small central cup is
in the temporal field. Ophthalmoscopy reveals thought to be the major predisposing factor for
pallid swelling of the retina, most obvious in developing nonarteritic AION.
the posterior segment, with a cherry-red spot at Clinically, AION is characterized by a sud-
the fovea. The retinal arteries are attenuated, den monocular and painless loss of vision. In
and box-car segmentation of blood in the about 40% of cases of the nonarteritic form, the
veins may be seen. Occasionally, emboli are seen loss of vision may improve spontaneously over
in the central retinal artery or its branches. The weeks or months. Funduscopic examination may
retinal swelling subsides over a period of 4 to 6 show either hyperemic or pallid disk swelling
weeks, leaving a relatively normal retinal with flame-shaped hemorrhages near the mar-
appearance but a pale optic disk and attenuated gins of the disk. (Miller, 138140)
arterioles. (Savino, 29; Riordan-Eva, chapter 7)
37. (A) In addition to the bitemporal field defects,
34. (B) Figure 12-2 shows a bright plaque that ap- patients with lesions of the optic chiasm may
pears larger than the artery in which it resides. develop a disturbance of depth perception.
The plaque is seen at a retinal arteriole bifurca- Clinically, the patient complains of difficulties
tion. This glistening appearance suggests a with near tasks such as using precise tools. In
cholesterol embolus of carotid artery origin. such tasks the required convergence causes
(Savino, 30) crossing of the two blind temporal hemifields.
This produces a completely blind triangular area
35. (C) Optic neuritis is an inflammatory disorder of of field with its apex at fixation. The image of an
the optic nerve. Its incidence is higher among object beyond fixation falls on blind nasal retinas
Answers: 3240 371

and thus disappears; binocular vision, however, preganglionic (second-order) neuron lesion.
is preserved. (Miller, 323327) (Miller, 434774)

38. (D) Pupillary constriction to light and to con- 40. (C) HolmesAdie tonic pupil syndrome is caused
vergence is preserved in cortical blindness. by a generalized peripheral or autonomic neu-
Retinal structures are preserved except if the ropathy that also affects the ciliary ganglion or
blindness is caused by prenatal or perinatal in- the short ciliary nerves. Clinically, the patient has
jury. (Miller, 358362) a unilaterally dilated pupil or bilaterally dilated
pupils, which may be confused with pharmaco-
39. (E) The patient described in this vignette has logically induced mydriasis.
Horner syndrome. This syndrome can be con- The distinction between these two entities
firmed and further characterized by pharmaco- can be made by slit-lamp examination. Patients
logical tests. The cocaine test is most commonly with HolmesAdie tonic pupils have segmental
used to confirm the diagnosis. Cocaine blocks contraction of the iris sphincter. In pharmaco-
the reuptake of norepinephrine into the sympa- logical anticholinergic blockade, the sphincter
thetic nerve endings. In the normal eye, it causes is entirely paralyzed and there is no segmental
dilatation of the pupil. This dilatation occurs if contraction with light stimulation. (Miller, 450455)
only there is continuous release of norepineph-
rine from the sympathetic nerves. In case of
sympathetic denervation, cocaine fails to dilate
the affected pupil. REFERENCES
Hydroxyamphetamine can be used to differ-
entiate between postganglionic and pregan- Brazis PW. Localization in Clinical Neurology. 3rd ed. Boston:
glionic Horner syndrome. Hydroxyamphetamine Little, Brown; 1996.
acts by releasing norepinephrine from adrenergic Kaufman DI, Trobe JD, Eggenberger ER, Whitaker JN.
stores in nerve endings. In case of damage to the Practice parameter: the role of corticosteroids in the man-
postsganglionic neuron (third-order neuron), nor- agement of acute monosymptomatic optic neuritis.
Report of the Quality Standards Subcommittee of the
epinephrine stores are depleted; thus hydrox-
American Academy of Neurology. Neurology. 2000;54:
yamphetamine fails to dilate the denervated
2039-2044.
pupil. Kline LB, Bajandas, FJ, eds. Neuroophthalmology Review
In this vignette, the affected pupil failed to Manual. 6th ed. Thorofare, NJ: Slack; 2001.
dilate after exposure to cocaine and hydroxyam- Kupersmith MJ, Latkany R, Homel P. Development of gen-
phetamine. Postganglionic sympathetic neuron eralized disease at 2 years in patients with ocular myas-
dysfunction is the most likely cause. Evidence of thenia gravis. Arch Neurol. 2003;60(2):243-248.
denervation supersensitivity to adrenergic sub- Laskowitz D, Liu GT, Galetta SL. Acute visual loss and
stances of the postganglionic neuron is supported other disorders of the eyes. Neurol Clin. 1998;16:323-353.
by full dilation of the right pupil after exposure Miller NR, Newman NJ, eds. Walsh & Hoyts Clinical Neuro-
to a 2% solution of epinephrine, a weak direct- Ophthalmology: The Essentials. 5th ed. Baltimore: Lippincott,
acting topical adrenergic drug. Postganglionic Williams & Wilkins;1999.
Riordan-Eva P. Disorders of the eyes & lids. In: McPhee SJ,
Horner syndrome may occur in diseases of the
Papadakis MA, Tierney LM Jr, eds. Current Medical
internal carotid artery, such as internal carotid
Diagnosis & Treatment 2009. Chapter 7. Available at http://
artery dissection or atherosclerosis, and in cav- www.accessmedicine.com/content.aspx?aID=2002
ernous sinus infection or tumors. Savino PJ, Danesh-Meyer HV. Neuro-ophthalmology. New
Lesions of the hypothalamus and lateral York: McGraw-Hill; 2003.
medulla cause damage to the first-order neuron Serra A, Leigh RJ. Diagnostic value of nystagmus: sponta-
of the sympathetic pathway. Tumors of the lung neous and induced ocular oscillations. J Neurol Neurosurg
apex and brachial plexus damage may cause a Psychiatry 2003;73:615-618.
This page intentionally left blank
CHAPTER 13

Neurooncology
Questions

1. What is the most likely diagnosis of a well- (C) the suprasellar region
circumscribed lobulated mass displacing the (D) the cerebellopontine angle
brain that, on microscopic examination, exhibits (E) the ventricles, with multifocal involvement
perivascular pseudorosettes?
(A) Oligodendroglioma 5. The most common location of germ cell tumors
in the central nervous system is the
(B) Pilocytic astrocytoma
(C) Fibrillary astrocytoma (A) basal ganglia
(D) Germ cell tumor (B) thalamus
(E) Ependymoma (C) pineal region
(D) cerebellum
2. Ependymomas have (E) brainstem
(A) a greater incidence among males than
among females 6. Hormonal assays indicate that most menin-
giomas express receptors to
(B) a predominant supratentorial location in
the adult population (A) androgen
(C) thoracic predominance when located in (B) glucocorticoid
the spinal cord (C) estrogen
(D) an association with neurofibromatosis (D) progesterone
type II when there is multifocal spinal (E) somatostatin
cord involvement
(E) a peak incidence at the age of 23 years 7. The most frequent location of intracranial
meningiomas is the
3. Myxopapillary ependymomas occur most fre-
(A) olfactory groove
quently in the
(B) parasagittal/falcine area
(A) fourth ventricle (C) sphenoidal ridge
(B) third ventricle (D) optic sheath
(C) cervical spinal cord (E) choroid plexus
(D) conuscaudafilum terminal
(E) lumbar spinal cord 8. Psammoma bodies are seen in
(A) glioblastoma
4. The most frequent location of choroid plexus
papillomas in children is (B) pituitary adenoma
(C) ependymoma
(A) the lateral ventricle
(D) medulloblastoma
(B) the third ventricle
(E) meningioma

373
374 13: Neurooncology

9. What is the most common location of intracra- activity but without necrosis or endothelial pro-
nial schwannomas? liferation is best classified as a
(A) Facial nerve (A) pilocytic astrocytoma (WHO grade I)
(B) Abducens nerve (B) fibrillary astrocytoma (WHO grade II)
(C) Trigeminal nerve (C) anaplastic astrocytoma (WHO grade III)
(D) Oculomotor nerve (D) glioblastoma multiforme (WHO grade IV)
(E) Optic nerve (E) pleomorphic xanthoastrocytoma (WHO
grade II)
10. The presence of a biphasic architectural pattern
composed of Antoni A and B areas is a hallmark 15. Which of the following immunohistochemical
of reactions helps to differentiate glioblastoma
from metastatic melanoma?
(A) meningiomas
(B) gliomas (A) Glial fibrillary acidic protein
(C) ependymomas (B) S-100 protein
(D) oligodendrogliomas (C) HMB 45
(E) schwannomas (D) Vimentin
(E) Keratin
11. Neurofibromas are composed of
16. Which of the following often does not appear as
(A) Schwann cells
a cystic lesion with an enhancing mural nodule
(B) astrocytes on gadolinium-enhanced magnetic resonance
(C) oligodendrocytes imaging (MRI)?
(D) melanocytes
(A) Oligodendroglioma
(E) neuronal cells
(B) Ependymoma
12. The most frequent pituitary secreting adenoma (C) Pilocytic astrocytoma
is the (D) Hemangioblastoma
(E) Ganglion cell tumor
(A) prolactinoma
(B) growth hormone adenoma 17. Which of the following neoplasms is typically
(C) TSH-secreting adenoma located near the foramen of Monro as an intra-
(D) ACTH-secreting adenoma ventricular mass?
(E) FSH/LH-secreting adenoma (A) Lymphoma
(B) Medulloblastoma
13. What is the dominant type of hormone-secreting
cell seen in the normal anterior lobe of the pitu- (C) Oligodendroglioma
itary gland? (D) Dysembrioplastic neuroepithelial tumor
(E) Central neurocytoma
(A) Prolactin-secreting cell
(B) Adrenocorticotrophin-secreting cell 18. Tuberous sclerosis is associated with molecular
(C) Thyrotrophin-secreting cell abnormalities involving the
(D) Growth hormonesecreting cell
(A) p53 suppressor on 17p 13.1
(E) Luteinizing stimulating hormonesecreting
(B) CDKN2 suppressor
cells.
(C) N-myc oncogene
14. An astrocytoma of intermediate differentiation (D) neurofibromin
with the presence of nuclear atypia and mitotic (E) tubulin
Questions: 928 375

19. Anaplastic astrocytoma is associated with molec- 24. The most consistent chromosomal abnormality
ular abnormalities involving in glioblastoma multiforme is a
(A) the p53 suppressor on 17p 13.1 (A) gain of chromosome 7
(B) the CDKN2 suppressor (B) gain of chromosome 17p
(C) the N-myc oncogene (C) loss of chromosome 1p
(D) neurofibromin (D) loss of chromosome 11p
(E) tubulin (E) loss of chromosome 9q

20. Neuroblastoma is associated with molecular 25. Mutations in the PTEN gene (phosphatase and
abnormalities involving tensin homologue gene located on chromosome
10) occur most commonly in cases of
(A) the p53 suppressor on 17p 13.1
(B) the CDKN2 suppressor (A) oligodendroglioma
(C) the N-myc oncogene (B) medulloblastoma
(D) neurofibromin (C) pilocytic astrocytoma
(E) tubulin (D) anaplastic astrocytoma
(E) de novo glioblastoma
21. WHO grade II astrocytoma is associated with
molecular abnormalities involving 26. Which of the following molecular features is
shared between well-differentiated oligoden-
(A) the p53 suppressor on 17p 13.1
drogliomas and anaplastic oligodendrogliomas?
(B) the CDKN2 suppressor
(C) the N-myc oncogene (A) Loss of heterozygosity on chromosome 15
(D) neurofibromin (B) Deletion of the CDKN2A gene
(E) tubulin (C) Loss of heterozygosity on chromosome 4
(D) Loss of heterozygosity on chromosome 1p
22. von Recklinghausen neurofibromatosis is asso- (E) Mutation of the PTEN gene
ciated with molecular abnormalities involving
27. What is the most common endocrine abnormal-
(A) the p53 suppressor on 17p 13.1
ity seen in suprasellar germ cell tumors?
(B) the CDKN2 suppressor
(C) the N-myc oncogene (A) Precocious puberty
(D) neurofibromin (B) Diabetes insipidus
(E) tubulin (C) Impotence
(D) Acromegaly
23. The presence of cellular pleomorphism, nuclear (E) Amenorrhea
atypia, and marked mitotic activity with the
absence of necrosis and endovascular prolifera- 28. The most efficacious single agent in the chemo-
tion is highly suggestive of therapeutic treatment of malignant gliomas is
(A) WHO grade I astrocytoma (A) a nitrosourea derivative (BCNU, CCNU)
(B) WHO grade II astrocytoma (B) procarbazine
(C) WHO grade III astrocytoma (C) temozolomide
(D) glioblastoma multiforme (D) vincristine
(E) gemistocytic astrocytoma (E) carboplatin
376 13: Neurooncology

29. Which of the following statements is true about 33. Cerebrospinal fluid alpha fetoprotein may be
primary central nervous system lymphoma in elevated in
immunocompetent patients?
(A) choroid plexus tumors
(A) It is more common before age 40. (B) ependymomas
(B) Glucocorticoids should be administrated (C) medulloblastomas
before stereotactic biopsy. (D) germ cell tumors
(C) The survival rate significantly improves (E) craniopharyngiomas
with tumor resection.
(D) The B lymphocyte phenotype is found in 34. The most frequent origin of brain metastasis is
more than 80% of cases.
(A) breast cancer
(E) It is resistant to radiation therapy.
(B) lung cancer
30. The most frequent brain tumor in the pediatric (C) skin cancer
population is a (D) kidney cancer
(A) craniopharyngioma (E) an unknown primary site
(B) brainstem glioma
35. The most frequent origin of metastasis causing
(C) medulloblastoma epidural spinal cord compression is
(D) germ cell tumor
(A) the prostate
(E) ependymoma
(B) the lung
31. Which of the following statements is true of (C) lymphoma
medulloblastoma? (D) the GI tract
(A) It typically arises from the vermis and the (E) the breast
roof of the fourth ventricle.
36. Which of the following structures of the brain is
(B) Its peak incidence is at the age of 20
the most sensitive to radiation damage in a 4-
years.
year-old girl who is undergoing a whole-brain
(C) Hydrocephalus is usually seen late in the radiation for leukemia treatment?
course of the disease.
(D) High tyrosine protein kinase C receptors (A) Hippocampus
expression may be an indicator of poor (B) Thalamus
prognosis. (C) Frontal cortex
(E) Radiotherapy has a modest benefit in the (D) Occipital cortex
management of medulloblastoma. (E) Vermis

32. Which of the following criteria suggests a higher 37. The most frequent malignancy associated with
risk of disease recurrence in the case of medul- paraneoplastic sensory neuronopathy is
loblastoma?
(A) breast cancer
(A) Posterior fossa location of the tumor (B) small cell lung cancer
(B) Patients age above 4 and below 21 years (C) ovarian cancer
at the time of the diagnosis
(D) lymphoma
(C) Decreased tyrosine kinase C receptor
(E) colon cancer
activity
(D) Total resection of nondisseminated tumor
(E) None of the above
Answers and Explanations

1. (E) Ependymomas arise throughout the neuraxis, most arising in the filum terminale (as myx-
often in an intraventricular location. In the adult opapillary variants) and occurring primarily
population, 64.1% of ependymomas are in the in adults. Multifocal spinal cord ependymomas
spinal cord, 11.8% are supratentorial, and 24.1% are associated with neurofibromatosis type II.
are infratentorial. Among the pediatric popula- (Parisi, 68; McGuire, 725729)
tion, ependymomas are the third most common
intracranial tumors after pilocytic astrocytomas 3. (D) Myxopapillary ependymomas appear as
and primitive neurectodermal tumors. About 30% well-defined, sausage-shaped lesions located in
of them appear before the age of 3 years and about the cauda equina and tending to distend it.
50% before the age of 5 years. Nearly 90% of pedi- Hemorrhagic rupture of these tumors may result
atric ependymomas are intracranial and only 10% in seeding of the subarachnoid space. (Parisi, 68)
are intraspinal. Approximately, two thirds of in-
tracranial ependymomas in children occur in the 4. (A) The site of predilection for the development
infratentorial compartment. Classic ependymo- of choroid plexus papillomas is the lateral ven-
mas grow as demarcated soft, gray masses that tricle in children and the fourth ventricle in
arise in the ventricular system. In the posterior adults. (Parisi, 910)
fossa, they may fill the fourth ventricle and pass
through its exit foramina. On microscopic exam- 5. (C) Central nervous system germ cell tumors
ination, ependymomas are generally composed of are most common in children and adolescents,
uniform cells with indistinct cytoplasmic borders with a peak incidence between the ages of 10
and round or oval nuclei. The nuclear/cytoplas- and 12 years. They are more common in males
mic ratio varies; it is usually high, but infrequent than females, with an overall male-to-female
nodules of densely packed cells may be scattered ratio of 2.5 to 1. The most common locations of
throughout paucicellular areas. The pseudorosette germ cell tumors are the pineal and suprasellar
is a perivascular anuclear zone of radial fibrillary regions. Occasionally, these tumors may occur in
processes that taper toward a vessel and is a hall- the basal ganglia, thalamus, and other sites in
mark of ependymomas. Less commonly, ependy- the central nervous system. (Jones, 2)
momas show the characteristic epithelial features
of true ependymal pseudorosettes. (Parisi, 68; 6. (D) Meningiomas may become clinically evi-
McGuire, 725729) dent during pregnancy or the luteal phase of
the menstrual cycle, suggesting that their
2. (D) Most ependymomas occur in childhood, growth may be hormonally related. Hormone
with a peak incidence between 1 and 5 years. assays indicate that most meningiomas express
Males and females are nearly equally affected. In receptors to progesterone rather than to estrogen
the adult population, infratentorial ependymo- or both hormones. (Parisi, 1)
mas are more common that supratentorial
ependymomas. Ependymomas represent 60% 7. (B) Approximately 90% of meningiomas are
of intramedullary gliomas of the spinal cord, intracranial, while 9% are intraspinal. The most

377
378 13: Neurooncology

frequent intracranial location is the skull base encountered in neurosurgical practice, with a
(planum spheroidale, sphenoid wing, petrous reported annual incidence ranging from 1 to
ridge, etc.), and at sites of dural reflection (falx, 14.7 per 100,000 persons. They may account for
tentorium, etc.). Other less frequent locations approximately 10% to 15% of primary brain
include the tuberculum sella, olfactory groove, tumors. Prolactinomas have the highest inci-
foramen magnum, optic nerve sheath, and dence among pituitary-secreting tumors. They
choroid plexus. (Parisi, 11; Whittle, 15351543) account for 40% to 60% of functioning adeno-
mas and are the most common subtype of pitu-
8. (E) Psammoma bodies are concentrically lami- itary tumor diagnosed in adolescents. Men are
nated calcifications seen in meningiomas. (Parisi, 2) generally diagnosed in their fourth and fifth
decades, whereas women are generally diag-
9. (C) Schwannomas account for approximately 8% nosed earlier. GH-secreting adenomas repre-
of all primary intracranial neoplasms. The most sent nearly 30% of all functioning tumors, fol-
common intracranial schwannomas develop from lowed by ACTH adenomas, which account for
the vestibular nerve and occupy the posterior cra- 15% to 25% of all functioning adenomas. TSH-
nial fossa. The second most common develop and LH/FSH-secreting tumors are the least fre-
from the trigeminal nerve and account for less quent functioning pituitary adenomas. An
than 8% of intracranial schwannomas. Trigeminal immunocytochemical study of 100 subclinical
schwannomas are encapsulated masses that are pituitary adenomas discovered at autopsy
predominantly solitary. They are more common in found that 50% were null cell adenomas
females between the ages of 35 and 60 years. (non-hormone-secreting adenomas) and 45%
Facial nerve schwannomas are less frequent than were prolactinomas. (Freda, 38593866; Lafferty,
trigeminal schwannomas, followed by glos- 43174323; McComb, 488)
sopharyngeal, vagus, and spinal accessory nerve
schwannomas. Schwannomas involving the ocu- 13. (D) The normal adenohypophysis pituitary
lomotor, trochlear, abducens, and hypoglossal gland comprises growth hormonesecreting cells
nerves are rare. Intraparenchymal schwannomas (somatotroph cells), prolactin-secreting cells (lac-
are very rare. Since the olfactory and optic nerves totroph cells), ACTH-secreting cells (corticotroph
do not have a Schwann cell layer, they do not cells), LH/FSH-secreting cells (gonadotroph
develop schwanommas. (Parisi, 67) cells), and TSH-secreting cells. The total compo-
sition of secreting cells in the pituitary gland is as
10. (E) The microscopic hallmark of schwannomas follows:
is a biphasic histological pattern composed of
50% growth hormonesecreting cells
distinct compact (Antoni A) areas intermixed
10% to 30% prolactin-secreting cells
with loose microcytic (Antoni B) areas. The
10% to 20% ACTH-secreting cells
Antoni A regions are cellular but lack the mitotic
10% LH/FSH-secreting cells
figures of a malignant nerve-sheath tumor. The
5% TSH-secreting cells (Scheithauer, 2)
identification of an encapsulated Schwann cell
tumor implies a benign nature. The Antoni B
14. (C) The World Health Organization has assigned
areas are hypocellular and lack a patterned
four grades to the spectrum of astrocytic tumors:
arrangement. Cells are loosely arranged in a
grade I (pilocytic astrocytoma), grade II (diffuse
myxoid matrix accompanied by thin strands of
astrocytoma), grade III (anaplastic astrocytoma),
collagen. Occasional mast cells may be identified
and grade IV (glioblastoma multiforme). The
in the Antoni B area. (Parisi, 7)
pilocytic astrocytoma is one of the most benign
forms of astrocytic tumor. The 10-year survival
11. (A) Schwann cells, perineural cells, and fibrob-
in supratentorial cases is generally over 90% in
lasts are all present in neurofibromas. (Parisi, 9)
most studies after gross total resection and 74%
after subtotal resection. Most but not all pilo-
12. (C) Pituitary adenomas represent the third
cytic astrocytomas occur in children or young
most common primary intracranial neoplasm
Answers: 819 379

adults. They are most abundant in the cerebel- by cells strongly positive for glial fibrillary acidic
lum, where they represent the majority of child- protein (GFAP), often with histiocytic features.
hood astrocytomas. They are also found in the (Scheithauer, 12)
region of the third ventricle, thalamus, hypo-
thalamus, and neurohypophysis, where they 15. (C) HMB 45 staining is useful because it is
can be difficult to treat owing to their location specific for melanoma. Both melanoma and
near clinically sensitive brain structures. glioblastoma can react with S-100 proteins and
Pilocytic means hair cell, referring to vimentin. Glioblastomas may or may not react
one of the major microscopic features of this with GFAP and keratin. Glioblastomas lack reac-
tumor, namely, parallel bundles of elongated, tivity to HMB 45, whereas metastatic melanoma
fibrillar cytoplasmic processes resembling mats reacts specifically with this immunohisto-
of hair. These hair-like processes contain large chemical stain, which is helpful in differenti-
amounts of glial fibrils that stain well with either ating between these two tumors. (Scheithauer,
Mallory PTAH (phosphotungstic acid and hema- 15)
toxylin stain) or immunoperoxidase for GFAP
(glial fibrillary acidic protein). 16. (A) The following tumors may appear cystic
Diffuse (low-grade) grade II astrocytomas with an enhancing mural nodule: pleomorphic
are well-differentiated, diffusely infiltrative neo- xanthoastrocytoma, pilocytic astrocytoma, gan-
plasms composed of fibrillary astrocytes with glion cell tumors, hemangioblastoma, and
nuclear atypia but no mitoses. Anaplastic astro- ependymoma. (Burger, 295)
cytomas are grade III astrocytomas. They are
characterized by an intermediate differentiation, 17. (E) Central neurocytomas are rare large intra-
increased cellular density, increased nuclear ventricular globular masses, commonly strad-
pleomorphism, and moderately increased nuclear dling the midline in the region of the septum
hyperchromatism plus mitoses. The lack of pellucidum. They can obstruct the flow of cere-
endothelial proliferation and foci of coagulation brospinal fluid, resulting in increased intracra-
necrosis in anaplastic astrocytic gliomas distin- nial pressure and hydrocephalus. Other intra-
guishes them from glioblastomas, but individual ventricular masses arising near the foramen of
cells with pyknotic nuclei may be interspersed in Monro include colloid cysts, subependymomas,
anaplastic astrocytomas. and subependymal giant cell astrocytomas.
Glioblastoma multiforme (glioblastoma) is (Burger, 296)
classified as a grade IV astrocytoma. It is a glioma
that may be uniformly undifferentiated or may 18. (E) Mutations of two different genes (TSC1 at
contain focal areas of differentiation, including 9q34 and TSC2 at 16p13.3) result in the tuberous
oligodendroglioma, and rarely ependymoma- sclerosis complex (TSC). It is inherited as an
like elements. Endothelial proliferation (increased autosomal dominant trait with a high rate of
density of cells in vascular walls), necrosis, spontaneous mutations in the TSC genes (65% to
nuclear atypia, and mitotic activity are the most 75% of cases arise from new mutations). Tubulin
important characteristic of grade IV astrocy- is the gene product of TSC2. Although the phe-
toma. The pleomorphic xanthoastrocytoma is a notypic expression of TSC is highly variable, it
bizarre supratentorial astrocytoma of young indi- is not determined by the specific gene mutation.
viduals that often involves both leptomeninges In fact, even affected members of the same fam-
and cerebral cortex. It is occasionally hemor- ily often develop very different manifestations.
rhagic. Its fibrillary, pleomorphic, hyaline, and (Davis, 846847; Rasheed, 162167; Sparagana, 115119;
lipid-laden multinucleated giant cells are clues Tai, 255262)
to its diagnosis. Protein granular degeneration
may be prominent, similar to that seen in pilo- 19. (B) The most common genetic abnormality ob-
cytic astrocytomas. Intracellular lipid content served in anaplastic astrocytomas and glioblas-
varies from abundant to absent between indi- tomas is a defect in the cell-cycle regulatory
vidual tumors. Astrocytomas are characterized pathway, such as inactivation of the CDKN2A
380 13: Neurooncology

and B genes. (Davis, 846847; Rasheed, 162167; variant of astrocytoma with a tendency toward
Sparagana, 115119; Tai, 255262) rapid progression to glioblastoma. Pilocytic
astrocytomas (WHO grade I) are typically cir-
20. (C) The N-myc gene plays an essential role in cumscribed, slow-growing, cystic neoplasms.
organogenesis. Gene overexpression due to Histologically, they are characterized by a bipha-
genomic amplification has been observed in sic pattern of compact bipolar cells and loose-
many human tumors such as the common child- textured multipolar cells, the presence of
hood tumor neuroblastoma. (Davis, 846847; Rosenthal fibers, microcysts, and eosinophilic
Rasheed, 162167; Sparagana, 115119; Tai, 255262) granular bodies. (Hildebrand, 11)

21. (A) Mutation in the p53 tumor-suppressor gene 24. (A) The most consistent chromosomal changes
may be seen in WHO grade II astrocytoma. The in glioblastoma multiforme are gains of chro-
gene is located on chromosome 17p13.1. It en- mosome 7; losses of chromosomes 9p, 10, and
codes nuclear phosphoprotein, a transcription 17p; and genetic amplification represented by
factor that enables passage through the cell the presence of double-minute chromosomes
cycle. (small extrachromosomal segments of amplified
DNA sequences containing oncogenic alleles).
22. (D) Neurofibromin belongs to the family of (Hildebrand, 12)
GTPase-activating proteins (GAPs), which turn
off the growth-promoting function of the Ras 25. (E) The PTEN gene (phosphatase and tensin
family of proteins by stimulating the hydrolysis homologue deleted from chromosome 10)
of GTP bound to Ras. Without enough neurofi- encodes a protein that dephosphorylates phos-
bromin, Ras remains unchecked, resulting in cel- patidylinositol-3,4,5-triphosphate (PIP3), thus
lular overgrowth and tumors. Mutation of the inactivating a cellular growth pathway. Growth
neurofibromin gene causes neurofibromatosis 1 factor receptors activate phospatidylinositide-
(NF-1), also known as von Recklinghausen dis- 3-kinase (PI3K), which phosphorylates phos-
ease. This is an autosomal dominant condition patidylinositol-4,5-diphosphate (PIP2) to PIP3,
caused by mutations of the NF-1 gene, which is thus activating protein kinase B (PKB). Activated
located on chromosome 17q11.2. (Davis, 846847; PKB stimulates cell growth and blocks apopto-
Rasheed, 162167; Sparagana, 115119; Tai, 255262) sis. PTEN shuts off this pathway, suppressing
oncogenesis. Analyses of LOH (loss of het-
23. (C) WHO grade III astrocytoma (also called erozygosity) have consistently shown losses of
anaplastic astrocytoma) diffusely infiltrates the all or part of chromosome 10 in more than 80%
surrounding brain parenchyma and has an of glioblastoma cases as well as a common dele-
intrinsic tendency for malignant progression to tion region in distal 10q. PTEN mutations and
glioblastoma. Histological examination shows overexpression of (epidermal growth factor
greater cellular and nuclear atypia than seen in receptors (EGFRs) are more common in primary
grade II astrocytoma, but there is absence of the de novo tumors rather than secondary progress-
necrosis and microvascular glomeruli or fes- ing glioblastoma multiforme tumors. (Rasheed,
toons seen in glioblastoma. Glioblastoma mul- 162167; Weiss, 543548)
tiforme is the most common primary malignant
tumor in adults and is characterized by the pres- 26. (D) Anaplastic oligodendrogliomas share many
ence of endothelial proliferation (increased den- molecular features with well-differentiated
sity of cells in vascular walls), necrosis, nuclear oligodendrogliomas, including loss of heterozy-
atypia, and mitotic activity. Grade II astrocy- gosity for 19q, 1p, or both. In contrast to well-
tomas are slow-growing, diffusely infiltrating, differentiated oligodendrogliomas, anaplastic
well-differentiated astrocytomas. Histologically, oligodendrogliomas can exhibit allelic loss of 9p,
they are composed of well-differentiated astro- homozygous deletion of the CDKN2A gene, and
cytes exhibiting moderate cellular density and losses involving chromosomes 4, 14, 15, and 18.
nuclear atypia. Gemistocytic astrocytoma is a Patients with anaplastic oligodendrogliomas
Answers: 2032 381

with 1p deletions, especially if coupled with 19q age. However, tumor resection does not signifi-
loss, have a better response to chemotherapy and cantly improve survival. (DeAngelis, 687691;
an improved prognosis. (Hildebrand, 15) Hildebrand, 5456)

27. (B) Diabetes insipidus is the most likely 30. (C) Medulloblastoma is the most common malig-
endocrine manifestation of germ cell tumors in nant brain tumor of childhood, with a peak inci-
the suprasellar areas. Precocious puberty is the dence around the first decade of life and a male-
most frequent endocrine disorder seen in hamar- to-female preponderance of about 2 to 1. It is a
toma and hypothalamic glioma. Impotence, member of the primitive neuroectodermal tumor
amenorrhea, galactorrhea, and acromegaly are family of CNS neoplasms and is considered a
seen in pituitary tumors. (Hildebrand, 31) grade IV lesion by the World Health Organization.
(Reddy, 681685)
28. (A) Nitrosourea derivatives (BCNU, CCNU)
remain for many neurooncologists the most 31. (A) Medulloblastomas are invasive embryonal
effective single agents, with a 20% to 30% partial tumors of the cerebellum with a tendency to
or complete response rate and 20% to 30% rate metastasize in the CNS. They represent 10% to
of stabilization. (Hildebrand, 48) 20% of brain tumors and 30% of tumors local-
ized in the posterior fossa. The peak incidence
29. (D) For unknown reasons, the incidence of pri- of medulloblastoma is in the first decade of life,
mary central nervous system lymphoma (PCNSL) with an annual incidence of 0.5 per 100,000 chil-
in immunocompetent patients has increased dren. In the brain, medulloblastoma typically
fivefold during the last decades. Most CNS lym- arises in the vermis of the cerebellum or roof of
phomas occur in patients over the age of 50 the fourth ventricle, causing ataxia and signs
years, with a median onset of 58 years. There is of hydrocephalus early on. At the time of diag-
a slight preponderance of men among immuno- nosis, over 80% of children have hydrocephalus.
competent patients. PCNSL has a poor progno- Tyrosine protein kinase C receptor (TrkC) is
sis if untreated, with a median survival of 3 to 4 expressed on mature granular cells. It is also
months. The predominant locations of PCNSL the receptor for neurotropin-3, which is one of
are the corpus callosum, frontal lobes, and deep the regulators of cerebellar granular cell devel-
periventricular structures of the brain. Most opment. TrkC expression in medulloblastoma
PCNSLs are non-Hodgkin lymphomas, with correlates with a favorable clinical outcome.
approximately 80% being non-Hodgkin B-cell The management of patients with medul-
lymphoma. The clinical features of the disease loblastoma includes surgical resection followed
may include deterioration of cognitive function, by craniospinal radiation. With this treatment,
headache, and seizures. When lymphoma is sus- patients with average-risk disease (patients who
pected, a biopsy should be performed before have localized tumor totally or nearly totally
starting treatment, particularly with glucocorti- resected by surgery) have approximately a 60%
costeroids. In about 50% of patients, the tumor 5-year progression-free survival. The addition
responds to steroid administration and occa- of chemotherapy to the management of high-
sionally may transiently disappear completely, risk patients with medulloblastoma (patients
compromising the pathological diagnosis and with disseminated disease or partially resected
delaying treatment. PCNSLs can initially be tumor) may improve the outcome. (Reddy, 681
highly sensitive to the combination of radiation 685; Hildebrand, 61)
therapy and glucocorticosteroids, which in-
creases the median of survival from a few to 12 32. (C) Factors associated with poor outcome in
to 18 months. High-dose of methotrexate can medulloblastoma include nonposterior fossa
extend 5-year survival rates to about 20%, but location, disseminated tumor, nondisseminated
when combined with radiation therapy, it is incompletely resected tumor with residual tumor
associated with significantly delayed neurotox- greater than 1.5 cm in its greatest dimension,
icity, especially in individuals over 60 years of decreased tyrosine protein kinase C receptor
382 13: Neurooncology

activity, and age less than 3 years at the time of asymmetric, associated with decreased or abol-
diagnosis. (Hildebrand, 61) ished reflexes and relative preservation of
strength. All types of sensation can be affected,
33. (D) Alpha fetoprotein and -HcG are elevated but loss of proprioception is often predominant.
in the cerebrospinal fluid of the majority of As a result, patients develop sensory ataxia and
patients with mixed germ cell tumors, while pseudoathetoid movements of the extremities
only -HcG is elevated in patients with chorio- (predominantly the hands). PSN results from
carcinomas. (Hildebrand, 72) inflammatory involvement of the dorsal root
ganglia, usually accompanied by dorsal nerve
34. (B) Brain metastases are seen in approximately root inflammation. PSN is frequently associated
15% to 20% of cancers. The most frequent pri- with paraneoplastic encephalomyelitis, particu-
mary tumor associated with brain metastasis is larly in patients with small cell lung cancer.
nonsmall cell lung cancer, which represents (Bataller, 6992)
about 50% of brain metastases. Breast cancer is
the second most frequent cause of brain metas-
tases and represents about 19% of all brain
REFERENCES
metastases. Skin/melanoma brain metastases
represent about 10.5% of total brain metastasis,
Bataller L, Dalmau J. Paraneoplastic disorders of the nerv-
whereas GI metastases account for about 10% of ous system. Continuum: Lifelong Learning In Neurology.
all brain metastases. Unknown primary site 2005;11(5):69-92.
metastases represent 11% of brain metastases. Burger PC, Scheithauer BW, Vogel FS. Surgical pathology of
(Kleihues, 252; Hildebrand, 76) the nervous system and its coverings. 4th ed. New York:
Churchill Livingstone, xii, 2002:657.
35. (E) The most frequent origin of metastasis caus- Davis RL. Neurofibromin progress on the fly. Nature.
ing epidural spinal cord compression is the 2000;403:846-847.
breast (it represents 22% of epidural metasta- DeAngelis LM. Primary central nervous system lym-
sis). Other origins include lung, prostate, and phoma. Curr Opin Neurol. 1999;12:687-691.
malignant lymphoma with about 15%, 10%, and Freda PU, Wardlaw SL. Diagnosis and treatment of pitu-
itary tumors. J Clin Endocrinol Metab. 1999;84:3859-3866.
10% of epidural spinal cord compression cases,
Hildebrand J, ed. Clinical relevance of advances in molec-
respectively. (Kleihues, 252)
ular biology. From tumors of the brain and spinal cord.
Continuum: Lifelong Learning in Neurology. 2001;7:7-141.
36. (A) The radiation tolerance of normal brain and Jones RV. Germ cell tumors of the central nervous system.
spinal cord depends mostly on the rate of Neuropathology Review. AFIP course 2002.
turnover of mature functioning cells. Neurons, Kleihues P et al. Pathology & Genetics of Tumors of the
because they do not replicate, are resistant to Nervous System. World Health Organization Classification of
radiotherapy but dependent for their function- Tumors. Lyons, France: IARC Press; 2000.
ing on the slow renewal of glial cells and the Lafferty AR, Chrousos GP. Pituitary tumors in children and
endothelial cells of blood vessels, which con- adolescents. J Clin Endocrinol Metab. 1999;84:4317-4323.
tinue to proliferate throughout life. In addition, McComb DJ, Ryan N, Horvath E, Kovacs K. Subclinical
the hippocampus, where neurogenesis contin- adenomas of the human pituitary. Arch Path Lab Med.
1983;107:488-491.
ues to occur postnatally, has been shown to be
McGuire CS, Sainani KL, Fisher PG. Incidence patterns for
especially sensitive to radiation damage. (Monje,
ependymoma: a surveillance, epidemiology, and end
2003; Taphoorn, 93115)
results study. J Neurosurg. 2009;110:725-729.
Monje ML, Palmer T. Radiation injury and neurogenesis.
37. (B) Paraneoplastic sensory neuronopathy (PSN) Curr Opin Neurol. 2003;16:129-134.
is characterized by progressive numbness and Parisi JE. Other glial tumors. Neuropathology Review. AFIP
often painful dysesthesias involving the limbs, course 2002.
trunk, and, less frequently, the cranial nerves, Rasheed BK. Wiltshire RN. Bigner SH. Bigner DD. Molecular
causing facial numbness or sensorineural hear- pathogenesis of malignant gliomas. Curr Opin Oncol. 1999;
ing loss. The symptom presentation is frequently 11:162-167.
Answers: 3337 383

Reddy AT. Packer RJ. Medulloblastoma. Curr Opin Neurol. Taphoorn MJB, Bromberg JEC. Neurological effects of ther-
1999;12:681-685. apeutic irradiation. Continuum: Lifelong Learning in
Scheithauer BW. Pituitary tumors. Neuropathology Review. Neurology. 2005;11(5):93-115.
AFIP course 2002. Weiss WA. Genetics of brain tumors. Curr Opin Pediatr.
Sparagana SP, Roach ES. Tuberous sclerosis complex. Curr 2000;12:54354-54358.
Opin Neurol. 2000;13:115-119. Whittle IR, Smith C, Navoo P, Collie D. Meningiomas.
Tai KF, Rogers SW, Pont-Kingdon G, Carroll WL. Definition Lancet. 2004;363:1535-1543.
of the human N-myc promoter region during development
in a transgenic mouse model. Pediatr Res. 1999;46:255-262.
This page intentionally left blank
CHAPTER 14

Movement Disorders
Questions

1. The combination of generalized seizures with (C) reduced executive function


ataxia and dementia is not seen in which one of (D) apathy
the following conditions? (E) disinhibition
(A) Huntington disease
5. Lesion of the anterior cingulate circuit results in
(B) Neuronal ceroid lipofuscinosis
(C) Lafora disease (A) apathy
(D) GM2 gangliosidosis (B) euphoria
(E) Mitochondrial encephalomyelopathy (C) agitation
(D) hallucination
2. Alien limb occurs significantly in case of (E) loss of executive function
(A) Huntington disease
6. Which of the following movement disorders car-
(B) corticobasal degeneration
ries the highest risk of depression with suicide?
(C) Parkinson disease (PD)
(D) Wilson disease (A) Huntington disease
(E) carbon monoxide intoxication (B) PD
(C) Progressive supranuclear palsy
3. Which of the following is true of the dorsal pre- (D) Wilson disease
frontal circuit? (E) Gille de la Tourette syndrome
(A) It originates in the frontal convexity and
7. The main movement disorder associated with
projects to the nucleus accumbens.
apraxia is
(B) It involves the ventral posterolateral
nucleus of the thalamus. (A) PD
(C) Lesion of the circuit results in deficits (B) Huntington disease
in executive function and motor (C) CreutzfeldtJacob disease
programming. (D) corticobasal degeneration
(D) The Mini-Mental State Examination is (E) Wilson disease
typically impaired when there is a lesion
of the prefrontal cortex. 8. Which of the following is characteristic of
(E) The prefrontal circuit is particularly dementia in PD?
spared in Huntington disease.
(A) Aphasia
(B) Agnosia
4. Lesion of the lateral orbitofrontal circuit results in
(C) Psychomotor slowing
(A) contralateral hemiplegia (D) Amnesia
(B) depression (E) Apraxia

385
386 14: Movement Disorders

9. Cerebral blood flow studies showed that when (D) cingulate motor area
untreated PD patients are asked to perform a (E) parietal cortex
paced movement with a joystick, there is a
decrease of blood flow in the 14. The motor component of the basal gangliothal-
(A) sensorimotor cortex amocortical circuits is processed by the
(B) lateral premotor cortex (A) caudate
(C) lateral parietal cortex (B) globus pallidus internal segment
(D) contralateral anterior cingulated (C) nucleus accumbens
(E) ipsilateral lentiform nucleus (D) globus pallidus external segment
(E) putamen
10. Age-related mitochondrial deletion is most fre-
quently seen in the 15. Which of the following is true of cognitive
(A) putamen impairment and Parkinson disease?
(B) globus pallidus (A) Dementia is observed in less than 10% of
(C) hippocampus patients with Parkinson disease, usually
(D) cerebellum at an advanced stage of the disease.
(E) cerebral cortex (B) Mild cognitive impairment is rarely
observed in the early stage of Parkinson
11. Inherited dystonia is caused by a defect of the disease. and is usually reported in severe
oxidative phosphorylation complex involving cases.
(C) The risk of developing dementia in
(A) complex I (NADH) Parkinson disease patients is similar to
(B) complex II (succinate) the risk in the general population.
(C) complex III (ubiquinone) (D) Attentional impairment is a major cogni-
(D) complex IV (cytochrome oxidase) tive feature of dementia in Parkinson dis-
(E) complex V (adenosine triphosphate) ease.
(E) Impairment of ability to copy a figure
12. The neurotoxin 1 methyl-4-phenyl-1,2,3,6 tetra- showing intersecting pentagons may be
hydropyridine (MPTP) may cause PD. It acts by observed in Parkinson disease dementia;
however, it does not predict a rapid
(A) inhibiting monoamine oxidase B in basal
decline in cognitive function.
ganglia neurons
(B) increasing ATP generation 16. An inverse relationship has been observed
(C) blocking complex I of the oxidative phos- between the occurrence of Parkinson disease
phorylation complex and
(D) blocking complex II of the oxidative
(A) cigarette smoking
phosphorylation complex
(B) exposure to pesticides
(E) blocking complex V of the oxidative
phosphorylation complex (C) exposure to lead
(D) exposure to manganese
13. Proximal limb kinetic apraxia is caused by a (E) hypertension
lesion in the
17. Mutations in which of the following genes cause
(A) primary motor cortex
autosomal recessive forms of parkinsonism?
(B) supplementary motor cortex
(C) lateral premotor cortex (A) PARK1
(B) PARK2
(C) PARK3
Questions: 925 387

(D) PARK4 (D) PD


(E) PARK5 (E) corticobasal degeneration

18. Which of the following is true about parkinson- 22. Which of the following is associated with an
ism caused by mutations in the gene for parkin? increased risk of PD?
(A) It causes an autosomal dominant parkin- (A) Vitamin E
sonism. (B) Manganese
(B) The mutation is located on chromosome (C) Caffeine
4q21. (D) Cigarette smoking
(C) The age of onset of parkinsonism is usu- (E) Alcohol
ally around 60 years.
(D) Point mutations of the parkin gene lead 23. Which of the following is true about 1-methyl-
to a milder form of parkinsonism than 4-phenyl-1,2,3,6 tetrahydropyridine (MPTP)?
gene deletion.
(A) MPTP-induced parkinsonism has more
(E) Rapid disease progression.
resting tremor than idiopathic PD.
19. Which of the following neurotransmitters is (B) MPTP is oxidized inside the neuron into
common to the direct and indirect pathways of MPP.
the striatum? (C) MPTP inhibits mitochondrial respiration
in astrocytes.
(A) GABA
(D) MPP inhibits mitochondrial complex I,
(B) Substance P which results in failure of ATP synthesis.
(C) Enkephalin (E) MPTP has a cocaine-like mechanism of
(D) Glutamate action.
(E) Glycine
24. Which of the following is most disabling in PD?
20. The subthalamic nucleus projects to the internal
(A) Tremor
part of the globus pallidus and substantia nigra
using which of the following neurotransmitters? (B) Akinesia
(C) Rigidity
(A) Glutamate A
(D) Postural instability
(B) Glycine
(E) Depression
(C) GABA
(D) Acetylcholine 25. Which of the following dopamine receptors
(E) Substance P stimulate adenylate cyclase?
(A) D1 receptors
21. A 65-year-old man died of progressive demen-
tia complicating a parkinsonian syndrome that (B) D2 receptors
was poorly responsive to levodopa. Pathological (C) D3 receptors
examination showed a shrinking globus pal- (D) D4 receptors
lidus associated with atrophy of the subthalamic (E) None of the above
nucleus and pallor of the substantia nigra with
enlargement of the aqueduct of Sylvius. The
most likely diagnosis is
(A) Huntington disease
(B) progressive supranuclear palsy
(C) multisystem atrophy
388 14: Movement Disorders

26. Which of the following dopamine receptors are (D) a dopamine agonist
D2-like receptors? (E) a COMT inhibitor
(A) D2 and D3 receptors
32. Selegiline is
(B) D1 and D2 receptors
(C) D2 and D5 receptors (A) an anticholinergic drug
(D) D1, D2, and D5 receptors (B) an MAOB inhibitor
(E) D1, D2, D3, and D5 receptors (C) an anti-NMDA receptor
(D) a dopamine agonist
27. Clozapine is a selective (E) a COMT inhibitor
(A) D4 receptor agonist
33. Pramipexole is
(B) D4 receptor antagonist
(C) D2 receptor agonist (A) an anticholinergic drug
(D) D2 receptor antagonist (B) an MAOB inhibitor
(E) D5 receptor agonist (C) an anti-NMDA receptor
(D) a dopamine agonist
28. The medulla contains (E) a COMT inhibitor
(A) no dopamine receptors
34. Remacemide is
(B) D1 receptors
(C) D2 receptors (A) an anticholinergic drug
(D) D3 receptors (B) an MAOB inhibitor
(E) D4 receptors (C) an anti-NMDA receptor
(D) a dopamine agonist
29. A 73-year-old man diagnosed with early-stage (E) a COMT inhibitor
PD who is given L-dopa three times a day may
show a sustained motor response because of 35. Riluzole is
(A) hypersensitivity to L-dopa (A) an anticholinergic drug
(B) presynaptic storage of exogenous (B) an MAOB inhibitor drug
dopamine (C) an anti-NMDA receptor
(C) half-life of L-dopa (8 hours) (D) a dopamine agonist
(D) postsynaptic storage of L-dopa (E) an inhibitor of glutamic acid release
(E) none of the above
36. The most common neurobehavioral abnormality
30. Entacapone is observed in PD is
(A) an anticholinergic drug (A) depression
(B) an MAOB inhibitor (B) personality change
(C) an anti-NMDA receptor (C) panic attacks
(D) a dopamine agonist (D) illusion
(E) a catechol-O-methyl transferase (COMT) (E) hallucination
inhibitor
37. Which of the following differentiate supranu-
31. Pergolide is clear palsy from PD?
(A) an anticholinergic drug (A) Staring gaze
(B) an MAOB inhibitor (B) Absence of tremor
(C) an anti-NMDA receptor
Questions: 2646 389

(C) Flexed posture 42. MPTP may cause parkinsonism by causing dam-
(D) Hypersensitivity to dopamine age to the
(E) Gait disturbance late in the progression of (A) caudate
the disease (B) putamen
(C) globus pallidus
38. The earliest sign of progressive supranuclear
palsy is (D) substantia nigra
(E) none of the above
(A) dysarthria
(B) visual symptoms 43. Manganese may cause parkinsonism by causing
(C) gait difficulties damage to the
(D) dysphagia (A) caudate
(E) dementia (B) putamen
(C) globus pallidus
39. Which of the following is not in favor of the
diagnosis of supranuclear palsy? (D) substantia nigra
(E) none of the above
(A) Absence of tremor
(B) Vertical gaze palsy 44. Cyanide may cause parkinsonism by causing
(C) Preserved horizontal oculocephalic reflex damage to the
(D) Early cerebellar sign (A) caudate
(E) Neck rigidity greater than limb rigidity (B) putamen
(C) globus pallidus
40. Which of the following is observed in Parkinson
disease but not in progressive supranuclear (D) substantia nigra
palsy? (E) none of the above

(A) Cerebral cortical and midbrain atrophy 45. Methanol may cause parkinsonism by causing
on magnetic resonance imaging (MRI) of damage to the
the head
(B) Neurofibrillary tangles (A) caudate
(C) Symmetric axial rigidity with postural (B) putamen
instability (C) globus pallidus
(D) Positive L-dopa response in early stage of (D) substantia nigra
the disease (E) none of the above
(E) Down-gaze palsy
46. Carbon monoxide may cause parkinsonism by
41. Which of the following cerebral cortical layers causing damage to the
are the most affected in case of progressive (A) caudate
supranuclear palsy?
(B) putamen
(A) Layers I and II (C) globus pallidus
(B) Layers III and V (D) substantia nigra
(C) Layers IV and VI (E) none of the above
(D) Layers III and IV
(E) Layers V and VI
390 14: Movement Disorders

47. On MRI of the head, a bilateral decrease of signal (A) dystonia


intensity on T-2 weighted images of the globus (B) akathisia
pallidus in a 20-year-old man with a history of (C) choreic movement
Parkinson syndrome is highly suggestive of
(D) tics
(A) HallervordenSpatz disease (E) myoclonus
(B) neuroacanthocytosis
(C) Rett syndrome 52. What is the initial treatment of choice for a 15-
(D) MPTP intoxication year-old teenager who is complaining of motor
tics related to Tourette syndrome?
(E) diffuse Lewy body disease
(A) Clonidine
48. Tremor is most commonly caused by which of (B) Haloperidol
the following drugs? (C) Fluoxetine
(A) Phenytoin (D) Botulinum toxin
(B) Phenobarbital (E) Clonazepam
(C) Valproic acid
(D) Carbamazepine 53. Which of the following is true of the disturbance
of eye movements in cases of cerebellar lesion?
(E) Lamotrigine
(A) Saccadic dysmetria results from a lesion
49. Which of the following is true of the huntingtin of the dorsal vermis.
protein? (B) Gaze-evoked nystagmus is seen in case of
(A) It is formed by consecutive proline a lesion of the fastigial nucleus.
residues encoded by a CCG repeat. (C) A lesion of the nodulus results in
(B) Its gene is located on the long arm of impaired smooth tracking.
chromosome 4. (D) A lesion of the flocculus results in impair-
(C) It is expressed in neuronal and nonneu- ment of the duration of the vestibular
ronal cells. response.
(D) The pattern of huntingtin expression is (E) A parafloccular lesion causes saccadic
parallel to areas of Huntington pathology. dysmetria.
(E) Huntingtin is a nuclear protein; its location
54. Which of the following molecular abnormali-
is altered in cases of Huntington disease.
ties is related to stiff-man syndrome?
50. Which of the following is true of the clinical fea- (A) Abnormal CAG repeat
tures of Huntington disease? (B) Abnormal CCG repeat
(A) The severity of chorea correlates with dis- (C) Glutamic acid decarboxylase antibodies
ease progression. (D) Abnormal Cooper metabolism
(B) Chorea is the most disabling symptom. (E) Alpha-synuclein abnormality
(C) Executive function is selectively lost.
(D) Apraxia is the earliest sign of cognitive 55. Which of the following gene mutations is seen in
impairment. PD?
(E) Optokinetic nystagmus is typically con- (A) Huntingtin gene
served. (B) Alpha-synuclein
(C) Adhalin gene
51. The unpleasant sensation of internal restless-
(D) Dystrophin gene
ness that is partially relieved by volitional move-
ment in a patient on chronic neuroleptic treat- (E) Synaptophysin gene
ment is called
Answers and Explanations

1. (A) Progressive myoclonic epilepsy (PME) is a substantia nigra. These structures project to ven-
slowly progressive autosomal recessive disor- tral anterior and medial dorsal thalamic nuclei,
der occurring in late childhood or early adult- which connect back to the dorsolateral prefrontal
hood. Generalized seizures, ataxia, and demen- cortex. Lesion of this circuit results in deficits in
tia are prominent features. Linkage analysis has executive function and motor programming. The
shown that the gene responsible is located on the patient exhibits difficulties in maintaining or shift-
long arm of chromosome 21q22.3. The common ing set, generating organizational strategies, and
causes of PME are neuronal ceroid lipofusci- retrieving memory. The dorsolateral prefrontal
nosis, mitochondrial encephalomyelopathy, siali- circuit is assessed by the Wisconsin Card Sort test.
dosis, Lafora disease, Baltic myoclonus, GM2 The circuit is disturbed in Huntington disease, as
gangliosidosis, and dentatorubropallidoluysian the degenerative process involves the caudate
atrophy. (Evidente, 475490) nucleus. (Watts, 15)

2. (B) Alien limb is defined as the lack of recogni- 4. (E) The lateral orbitofrontal circuit of the frontal
tion of movement in the affected limb, or a feel- subcortical pathways originates in the inferolat-
ing that one limb is foreign associated with eral prefrontal cortex and projects to the ven-
observable involuntary motor activity. The upper tromedial caudate nucleus, which then projects
extremity is the most frequently affected limb. to the dorsomedial globus pallidus and sub-
Signs include the failure to perceive ownership of stantia nigra. The return pathway is via the ven-
ones limb in the absence of visual cues, an tral anterior and dorsothalamic nuclei, which
impression that the seen limb is foreign, person- project back to the orbitofrontal cortex. Lesion of
ification of the affected limb, and autonomous the orbitofrontal circuit causes personality
motor activity deemed by the patient as beyond changes with irritability and disinhibition (similar
voluntary control. Alien limb syndrome is a well- to personality changes seen in idiopathic calci-
established part of corticobasal degeneration. It is fication of the basal ganglia and neuroacantho-
reported to complicate the course of the disease cytosis). (Watts, 16)
in nearly 50% of cases. Vascular etiology is most
commonly reported in ischemic or hemorrhagic 5. (A) The anterior cingulate circuit of the subcor-
lesions of the anterior cerebral artery. Surgical tical frontal lobe pathway originates in the ante-
lesions such as corpus callosotomy and thalamo- rior cingulate gyrus and projects to the ventral
tomy have been associated with alien limb. Other striatum, which includes the nucleus accum-
causes of alien limb include Alzheimer disease bens, olfactory tubercles, and parts of the cau-
and CreutzfeldtJacob disease. (Hanna, 135145) date and putamen. The ventral striatum then
sends afferents to the globus pallidus and sub-
3. (C) The dorsal lateral prefrontal circuit originates stantia nigra, which in turn project to the para-
in the frontal lobe convexity and projects to the median part of the medial dorsal thalamus. The
dorsolateral head of the caudate and subsequently thalamus projects back to the cingulate gyrus.
to the dorsomedial globus pallidus and rostral Lesion of the anterior cingulate circuit causes

391
392 14: Movement Disorders

apathy, reduced drive and initiative, and de- increase of the blood flow of the anterior cingu-
creased motivation. Akinetic mutism and pro- late, supplementary motor cortex, lateral premo-
found apathy may result from bilateral lesions of tor cortex, and dorsolateral prefrontal cortex.
the anterior cingulate circuit. (Watts, 16) When an untreated PD patient performs the same
task, there is normal activation of sensorimotor
6. (A) Depression may complicate the course of a cortex, lateral premotor cortex, and lateral pari-
number of movement disorders. Among all of etal association areas. There is, however, de-
these, Huntington disease carries the highest risk creased activation of the contralateral lentiform
of depression and suicide. Depression may affect nucleus, anterior cingulate, supplementary motor
half of the patients, with 30% meeting the criteria area, and dorsolateral prefrontal area. (Watts, 33)
of major depression. Suicide is four to six times
more common among Huntington disease pa- 10. (A) The susceptibility of the central nervous sys-
tients than other depressed patients. In one study tem to somatic mitochondrial mutations is high
including Huntington disease patients and their and occurs with age in specific brain regions.
relatives, the rate of death caused by suicide The caudate and putamen are the locations of the
reached 7.3%. Depression may complicate the highest accumulation of mitochondrial DNA
course of PD patients in approximately 40% of mutations, whereas the cerebellum and myeli-
cases. Depression is seen in 20% to 30% of pa- nated axons have the lowest level of mitochon-
tients with the diagnosis of Wilson disease. De- drial DNA mutations. (Watts, 54)
pression is less frequently seen in progressive
supranuclear palsy and Gilles de la Tourette dis- 11. (A) Complex Ispecific activity as determined
orders. (Di Maio, 293295; Poewe, S2S6; Watts, 17) by the nicotinamide adenine dinucleotide ubi-
quinone assay is reduced in brains of patients
7. (D) Apraxia is the inability to perform motor with inherited dystonia compared with controls.
acts despite intact comprehension, cooperation, (Watts, 58)
and motor and sensory skills. Corticobasal degen-
eration is the main movement disorder associ- 12. (C) 1-methyl-4-phenyl-1,2,3,6-tetrahydropyridine
ated with apraxia (reported in 71% of cases in (MPTP) induces parkinsonism by causing selec-
one series). The disorder is attributed to neuronal tive degeneration of nigral neurons. 1-Methyl-
loss and achromasia in the frontoparietal cortex. 4-phenylpyridinium ion (MPP), the oxidative
(Watts, 23) metabolite of MPTP, is actively taken up into
dopaminergic neurons through the dopamine
8. (C) Studies of the frequency of dementia in transporters and concentrated within dopaminer-
patients with Parkinson disease (PD) have found gic neurons. In nigral neurons, MPP inhibits
rates ranging from 8% to 81%. The addition of complex I of the mitochondrial electron trans-
mental impairment to the motor symptoms of PD port chain and the A-ketoglutarate dehydroge-
increases functional impairment and the need for nase complex (KGDHC) of the Krebs cycle. This
health care in patients with PD. The dementia in has been considered the major mechanism of
PD is of the subcortical type. It is characterized by neuronal death in MPTP-induced PD, although
a psychomotor slowing (also called bradyphre- other mechanisms such as apoptosis have also
nia), memory retrieval deficits, abnormal cognition been postulated. (Mizuno, 893902)
with impaired ability to manipulate knowledge,
and disturbed executive function. Aphasia, aprax- 13. (C) The control of proximal limb musculature
ia, agnosia, and amnesia are absent. (Watts, 23) and interlimb coordination is related to the lat-
eral premotor area of the cortex. This is sug-
9. (D) When normal subjects perform paced move- gested by the heavy projections from this region
ments with a joystick in freely selected directions to the medial pontomedullary reticular forma-
with the right hand, there is an increase of the tion, where spinal projections constitute the bulk
cerebral blood flow in the contralateral lentiform of the ventromedial descending brainstem sys-
nucleus and sensorimotor cortex and bilateral tem associated with control of the proximal
Answers: 618 393

musculature. Lesion of the lateral premotor cor- cognitive impairment has been demonstrated
tex causes a proximal limb kinetic apraxia that with semantic fluency and psychomotor skills as
contrasts sharply with the bimanual distal well as attention being related to functional out-
apraxia of the supplementary motor cortex. come. (Aarsland, 676682)
Such patients may have difficulties making coor-
dinated rotary movements of both shoulders 16. (A) Although an inverse association between cig-
whereas independent movement of either shoul- arette smoking and PD has been reported in many
der is performed with ease within the limits of studies, the origin of this association remains
any associated weakness. (Watts, 76) debated. Two studies explored this relation in
detail. As part of the Cancer Prevention Study II
14. (E) Motor circuitry is initiated in cortical regions Nutrition Cohort (n 143,325, with 9 years of
that transmit parallel glutaminergic projections follow-up and 413 incident cases), one study
to the striatum. The motor component of the investigated the temporal relationship between
basal gangliothalamocortical circuits is processed cigarette smoking and PD and showed that par-
by the putamen, whereas the caudate and ticipants with older age at quitting smoking and
nucleus accumbens mediate cognitive, emotive, fewer years since quitting smoking had lower PD
and limbic processes. (Hallett, 177183; Watts, 100) risk. However, a 30% to 60% decreased risk of PD
was apparent for smoking as early as 15 to 24
15. (D) Point-prevalence of dementia is approxi- years before symptom onset, but not for smoking
mately 30%, with a sixfold increased risk for 25 years or more before onset. This finding sug-
developing dementia in PD compared with non- gests that risk reduction is unlikely to result from
PD. Two recent longitudinal studies of 12 and 20 changes in smoking behavior in the preclinical
years duration have highlighted the inevitability, phase of the disease. A similar pattern was ob-
over time, of dementia in patients with PD, with served in a pooled analysis of eight case-control
over 80% developing dementia. The time from and three cohort studies that also confirmed an
onset of PD to dementia varies considerably, how- inverse association between PD and cigarette
ever; it is therefore of key importance to identify smoking, with stronger relations in current com-
the factors associated with time to dementia so as pared with former smokers, in cohort than in case-
to optimize management and planning of care. control studies, and in younger- than older-onset
Currently, only general features such as the sever- cases. An inverse association was also found for
ity of parkinsonismin particular gait and pos- smoking cigars or pipes and for chewing tobacco
tural disturbances, mild cognitive impairment in male participants. The inverse association
(MCI), and agehave been found to be associated between PD and cigarette smoking was confirmed
with a shorter time to dementia. In addition, the in a cohort study from Singapore. (Elbaz, 454460;
concept of MCIPD disease has recently attracted Thacker 764768; Ritz, A990A997)
increasing interest. Already at disease onset, 25%
to 30% of patients have some cognitive impair- 17. (B) Recessive forms of parkinsonism are known
ment. Although consensus criteria for MCIPD to be caused by mutations in parkin (PARK2),
are not yet available, evidence in support of the PINK1 (PARK6), and DJ-1 (PARK7), and repre-
concept has come both from longitudinal studies sent an important cause of early-onset PD (onset
showing shorter time to dementia and more cor- usually before 40 years of age). In addition to the
tical atrophy and metabolic changes in those with typical triad of akinesia, rigidity, and resting
MCI compared with those with cognitively intact tremor, the clinical phenotype of early-onset
PD and healthy individuals. A rapid decline in recessive parkinsonism often includes dystonia
cognitive function in PD patients has been asso- at onset, hyperreflexia, and relatively slow dis-
ciated with impairment in neuropsychological ease progression. (Gasser, 363369)
tasks with a more posterior cortical basis, includ-
ing semantic fluency and ability to copy an inter- 18. (D) Mutations in the PARKIN gene are by far the
secting pentagons figure, as well as a gait and most common among the recessive forms of
postural motor changes, The functional impact of parkinsonism. More than 40 mutations have been
394 14: Movement Disorders

identified, and a recent review suggests both nesia and axial more than limb rigidity), resist-
mutational hot spots and founder effects as a ance to levodopa therapy, pseudobulbar palsy,
source of recurrent mutations in PARKIN regard- and subcortical dementia. Subsequent to the
less of the mutation type. There is only a weak onset of postural instability, dysarthria and
correlation between clinical manifestation and bradykinesia are the most common problems.
type of mutation, but the relatively large case An absent, poor, or waning response to levodopa
series available now indicate that patients with is a characteristic feature defining the atypical
point mutations that presumably lead only to a parkinsonian disorders. Often although not
partial loss of PARKIN function tend to be more always, patients with these disorders may exhibit
mildly affected than patients with deletions. The axial more than limb muscle involvement.
age of onset of parkinsonism is between 20 and 40 Several features should make us suspect that
years. The gene mutations are located on chro- a patient may suffer from PSP. Early instability
mosome 6q25. The clinical phenotype of early- and falls, particularly during the first year of
onset recessive parkinsonism seen with PARKIN symptom onset, should suggest the diagnosis
gene mutations often includes dystonia at onset, of PSP. However, these features may also rarely
hyperreflexia, and relatively slow disease pro- develop in patients with corticobasal degenera-
gression. (Gasser, 363369) tion (CBD), when asymmetric symptoms develop
in the lower extremities. Instability and falls may
19. (A) The spiny neurons of the putamen are the also develop early in multiple system atrophy
principal inputoutput cells of the striatum. (MSA), although these symptoms are usually pre-
These cells project to the internal segment of the sent when patients already exhibit autonomic
globus pallidus via two separate pathways: disturbances.
direct and indirect. The direct pathway involves Marked slowing of vertical saccades usu-
a subpopulation of spiny cells using GABA and ally precedes the development of vertical supra-
substance P as neurotransmitters and project- nuclear gaze palsy and should readily point
ing directly to the internal segment of the globus toward the diagnosis of PSP. The saccades in
pallidus and the substantia nigra pars reticu- CBD may have increased latency but normal
lata, which are the output nuclei of the basal speed and are similarly affected in the vertical
ganglia. The indirect pathway involves another and horizontal planes, whereas in MSA the sac-
subpopulation of spiny cells that use GABA and cades have normal speed and latency. Patients
enkephalin as neurotransmitters and project to with PSP may present prominent early or severe
the external segment of the globus pallidus, speech and swallowing difficulties and may
which sends GABAergic projections to the sub- exhibit oversized mouthfuls or overstuffing the
thalamus. (Watts, 99100) mouth when eating, but these features may also
be present in CBD.
20. (A) The subthalamic nucleus projects to the Neuropathologically, PSP is characterized by
internal part of the globus pallidus and substan- abundant neurofibrillary tangles and/or neuropil
tia nigra, using glutamate as a neurotransmitter. threads in particular areas of the basal ganglia
This model of neurotransmission suggests that and brainstem; neuronal loss and gliosis are vari-
there is an equilibrium between the direct path- able. Neurofibrillary tangles, neuronal loss, and
way, tending to reduce the basal ganglia output, gliosis in PSP affect the striatum, pallidum, sub-
and the indirect pathway, tending to increase the thalamic nucleus, substantia nigra, oculomotor
output from the internal segment of the globus complex, periaqueductal gray, superior colliculi,
pallidus and substantia nigra. (Watts, 99100) basis pontis, dentate nucleus, and prefrontal cor-
tex. (Litvan, 4148)
21. (B) The pathological features mentioned in this
question are highly suggestive of progressive 22. (B) The only definite risk factor for PD is age.
supranuclear palsy (PSP). Typical PSP patients However, epidemiological studies have suggested
present with early postural instability, supranu- an increased risk of PD in males, those with a
clear vertical gaze palsy, parkinsonism (bradyki- family history of the disease, exposure to iron,
Answers: 1932 395

farming, rural residence, steel alloy industries, encoded by a gene located on chromosome 4.
and herbicide and pesticide exposure. Manganese (Watts, 202)
was found to increase the incidence PD. Smoking
cigarettes is consistently associated with a de- 27. (B) D2 receptors have bromocriptine as an ago-
creased risk for PD. However, whether this asso- nist and haloperidol as an antagonist. Clozapine
ciation is truly caused by cigarette smoking or is a selective D4 receptor antagonist. (Watts, 202)
instead reflects a personality characteristic or
another behavior associated with smoking is not 28. (E) D1 and D2 receptors are located mainly in the
known. Caffeine was recently reported to decrease striatum and substantia nigra in the postsynaptic
the risk of PD. Other factors associated with de- areas, although D2 and D3 receptors also have
creased risk of PD include vitamin E, tocopherol, presynaptic locations. The olfactory tubercle con-
and alcohol consumption. (Ross, 26742679; Tanner, tains both D1 and D2 receptors; the medulla con-
427430) tains D4 receptors. (Watts, 202)

23. (D) MPTP-induced parkinsonism is very much 29. (B) Despite the short half-life of L-dopa (60
like idiopathic PD except for the resting tremor, minutes), patients in early stages of the disease
which is less frequent with MPTP intoxication. may experience a sustained motor response with
In the brain, MPTP is taken up into astrocytes, the administration of L-dopa three or four times
where it is oxidized to MPP by monoamine per day. This sustained response is speculated to
oxidase B. MPP is then actively taken up into be caused by storage of exogenous dopamine in
nigrostriatal neurons through dopamine trans- presynaptic terminals in survival dopaminergic
porters and concentrated in dopaminergic neu- striatal cells. (Watts, 204)
rons. MPP inhibits mitochondrial respiration
in the dopaminergic neurons, which causes the 30. (E) Tolcapone and entacapone are catechol-O-
selective death of that type of cell. MPP methyl transferase (COMT) inhibitors that may
inhibits mitochondrial complex I and NADH- be used as an adjuvant therapy to dopamine for
linked state 3 respiration, causing a loss of oxida- the management of PD. Both drugs are used for
tive phosphorylation mechanism and fall in the treatment of motor fluctuations in patients
ATP level. (Watts, 162) treated with levodopa for long periods. COMT
is an enzyme that convert levodopa into an inac-
24. (D) Among the cardinal signs of PD, postural tive metabolite, 3-O-methyldopa. Coadministra-
instability is usually the last sign to appear, the tion of entacapone delays clearance of levodopa
most disabling, and the least treatable. Postural from plasma and prolongs the action of indi-
instability results from the combination of changes vidual doses of levodopa. (Jankovic, 785790)
in postural adjustment, loss of postural reflexes,
rigidity, and akinesia. (Watts, 187) 31. (D) Dopamine agonists exert their pharmaco-
logical effect by directly activating dopamine
25. (A) D1receptors are able to stimulate adenylate receptors, bypassing the presynaptic synthesis
cyclase. (Watts, 202) of dopamine. Several new dopamine agonists
cabergoline, pramipexole, and ropinirolehave
26. (A) Dopamine receptors are divided into five been added to the previously known potent anti-
subtypes based on their action on adenylate parkinsonian drugs bromocriptine and pergolide.
cyclase: D1-like receptors include D1 and D5 and (Jankovic, 785790)
are able to stimulate adenylate cyclase. D2-like
receptors include D2, D3, and D4 and are able 32. (B) Neuroprotective therapies can be defined as
to inhibit adenylate cyclase. D1 receptors are en- medical or surgical interventions that favorably
coded by a gene located on chromosome 5; D2 alter the underlying etiology or pathogenesis
and D4 receptor genes are located on chromo- and thus delay the onset or slow or even halt
some 11; D3 receptors are coded by a gene the progression of the neurodegenerative process
located on chromosome 3; and D5 receptors are such as PD. Selegiline was found to prevent
396 14: Movement Disorders

parkinsonism induced by the oxidated form of Other neurobehavioral abnormalities seen in PD


MPTP, which has stimulated interest in an include sleep alterations, personality change, anx-
antioxidative therapy to retard the progression of iety, and panic attacks. (Watts, 257259)
PD. Selegiline acts as a suicide substrate for
monoamine oxidase (MAO) type B, irreversibly 37. (B) Progressive supranuclear palsy is character-
inhibiting this enzyme. Selegiline has a lev- ized by gait disturbance with multiple falls, erect
odopa-sparing effect, and it smoothes out lev- posture with retrocollis, contracted facial mus-
odopa-related motor fluctuations, possibly by cles, bradykinesia, predominantly proximal rigid-
prolonging dopamine-induced responses in mid- ity, vertical supranuclear gaze abnormalities, and
brain dopaminergic neurons. (Jankovic, 785790) spastic dysarthria. Progressive supranuclear palsy
can be differentiated from PD by the presence of
33. (D) Pramipexole is a nonergot dopamine agonist contracted rather than flaccid face, undirected
with specificity of D2 dopamine receptor but rather than staring gaze, erect rather than flexed
also has been shown to bind to D3 and D4 recep- posture, spastic dysarthria, and absence of rest
tors and may stimulate dopamine activity on tremor. (Watts, 279)
nerves of striatum and substantia nigra. (Jankovic,
785790) 38. (C) Early instability and falls, supranuclear ver-
tical gaze palsy, and poor response to levodopa
34. (C) Remacemide, an anticonvulsant with anti- are features highly suggestive of the diagnosis of
NMDA effects, has been shown to enhance the PSP. Other features of PSP include subcortical
effects of levodopa in parkinsonian rats and mon- dementia and pseudobulbar palsy. Marked slow-
keys. It may have a neuroprotective effect in PD ing of vertical saccades is seen earlier than the
and Huntington disease. The rationale behind development of vertical supranuclear palsy in
using remacemide as a neuroprotective agent is PSP. Instability and falls may also develop in cor-
that in PD, there is an increased activity in the ticobasal degeneration as well as in early multiple
subthalamic nucleus and internal segment of the system atrophy (MSA), although these symptoms
globus pallidus. The subthalamic nucleus pro- are usually present when patients already exhibit
vides an excitatory glutaminergic input to the autonomic disturbances. The saccades in CBD
internal part of the globus pallidus. Glutamate may have increased latency but normal speed,
inhibition would be expected to improve parkin- and are similarly affected in the vertical and hor-
sonism. (Jankovic, 785790) izontal planes, whereas in MSA the saccades have
normal speed and latency. (Litvan, 4148)
35. (E) Riluzole, a drug approved for the treatment
of amyotrophic lateral sclerosis, acts primarily by 39. (D) The presence of early or prominent cere-
inhibiting glutamic acid release and noncom- bellar signs, unexplained polyneuropathy, and
petitively blocking NMDA receptors, and as such prominent noniatrogenic dysautonomia other
it may exert antiexcitotoxic effects similar to than isolated postural instability contradict the
those of NMDA antagonists. (Jankovic, 785790) diagnosis of PSP. (Watts, 280)

36. (A) Depression is the most frequent neurobehav- 40. (D) MRI of the head does not show specific
ioral abnormality seen in PD, with a prevalence abnormalities in most patients with the diagnosis
of 25% to 40%. Depression may appear before of PSP. However, it may show more prominent
the emergence of the first motor signs and does cerebral cortical atrophy than in PD. In moderate
not correlate with the severity of the disease. to advanced stages, there may be a thinning of the
Psychotic signs such as illusions or hallucinations anteroposterior diameter of the midbrain tectum
are among the most disabling complications of L- and tegmentum with atrophy of the colliculi and
dopa treatment in PD. They are seen in 8% to disproportionate enlargement of the sylvian fis-
15% of patients with PD, especially elderly sures and posterior third ventricle. Cerebellar
patients who show signs of impaired cognition atrophy is seen in multisystem atrophy. The pres-
and have longer duration on L-dopa treatment. ence of neurofibrillary tangles is necessary for the
Answers: 3347 397

neuropathological diagnosis of PSP. Most of these white matter and globus pallidus. Carbon mon-
neurofibrillary tangles are rounded in shape, oxide causes tissue anoxia. The globus pallidus is
whereas in Alzheimer disease most of them have vulnerable to anoxic injury, probably from intrin-
a flame-shaped form. Clinically, patients with PSP sic metabolic susceptibility. (Watts, 315332)
have a symmetric neurological deficit, prominent
axial rigidity, postural instability, and severe ver- 47. (A) HallervordenSpatz syndrome (HSS) is a
tical gaze restriction. In PD, the neurological rare autosomal recessive disease that has been
symptoms are not symmetric, axial rigidity and mapped to chromosome 20p12.3p13. The symp-
postural instability are less prominent than in PSP, toms usually start in childhood and involve the
and there is a good response to L-dopa early in the cognitive, speech, and motor domains. Children
course of the disease. (Watts, 281283) demonstrate signs of cognitive and motor regres-
sion, the speech becomes dysarthric, and extra-
41. (E) The cerebral cortex is one of the major areas pyramidal symptoms appear.
of primary involvement in PSP. Motor strip 4 and Typical symptoms at onset involve difficulty
the oculomotor association areas are the most walking or postural abnormalities. Personality
important sites of pathology. Area 17, the pri- changes and cognitive changes infrequently are
mary visual cortex, is the least affected. The large the presenting symptoms. Rigidity gradually pro-
pyramidal and small neurons of layers V and VI gresses. Spasticity associated with hyperreflexia is
are the most affected layers of the cerebral cortex seen in over half of the cases. Dysarthria becomes
in PSP; layers III and V are the most affected in evident in all cases. Dystonia, chorea, and tremor
Alzheimer disease. (Watts, 283) are also seen. Ophthalmic abnormalities are seen
in HSS patients including pigmentary retinopathy
42. (D) MPTP induces parkinsonism by selective and optic atrophy. Movement disorders associ-
damage to the substantia nigra. MPTP is con- ated with HSS include rigidity, which can involve
verted by glial cells to MPDP, which is then half of the body, axial structures, arms, legs, or
converted to MPP and enters dopaminergic be generalized. Dystonia is also seen, typically
neurons through a dopamine uptake system. involving the facial musculature and the feet.
MPP induces mitochondrial damage, causing Parkinsonism as an initial manifestation is
cell death. (Watts, 315332) rare, and usually occurs only in adult-onset
cases. In rare adult-onset cases, presenting symp-
43. (C) Overexposure to manganese may cause toms can be indistinguishable from PD. The
Parkinson syndrome by inducing selective neu- cognitive abnormalities are common in HSS.
ronal loss in the globus pallidus, probably by They may precede the motor symptoms of the
increasing autooxidation of dopamine by a disease. Seizures can also be seen in this disor-
higher valence ion, causing an increase in the der. The disease usually starts between the ages
generation of free radicals. (Watts, 315332) of 7 and 12 years. The disorder typically pro-
gresses and leads to death within 20 years.
44. (C) Pathological changes after acute cyanide in- Definitive diagnosis of HSS can only be made
toxication have demonstrated selective destruc- histologically. Presumptive clinical diagnosis is
tion of the basal ganglia, especially the globus based on the constellation of the clinical signs,
pallidus. Cyanide radicals inactivate cytochrome supported by the neuroimaging data. Computed
oxidase and other oxidative systems, leading to tomography (CT) may reveal cerebral atrophy
cell death. (Watts, 315332) with increased ventricular size. Mineralization of
the globus pallidus is also seen. Hyperlucency of
45. (B) Methanol is metabolized to formic acid, which the putamen and globus pallidus is seen on CT.
achieves high concentration in the putamen and MRI is more sensitive. There is decreased T-2
causes selective damage there. (Watts, 315332) weighted and proton density signal in the globus
pallidus, which is caused by iron deposition. In
46. (C) Carbon monoxide intoxication may cause some patients there is a hyperintense area within
Parkinson syndrome, with more damage in the the area of hypointensity, the eye of the tiger
398 14: Movement Disorders

sign. Pathologically, the hallmark of the disease, is mal and secretory pathways, and for preventing
rust-brown discoloration of the pars reticulata of cells from undergoing apoptosis. Mutant hunt-
the substantia nigra and the internal segment of ingtin is proteolytically processed, and the result-
the globus pallidus. The pigmentation is caused ing amino-terminal fragments containing the glu-
not only by the abnormal iron deposition but also tamine expansions form aggregates that are
by high concentration of the organic pigments deposited in nuclear and cytoplasmic inclusions
lipofuscin and neuromelanin. (Colcher, 629649) in the brains of HD patients. (Trottier, 445446; Watts,
484485)
48. (C) Valproic acid is the most common cause of
tremor among antiepileptic medications. Chronic 50. (C) Chorea, although of cosmetic concern, is not
treatment with valproic acid may cause a tremor disabling per se and does not correlate with the
in up to 25% of patients. Occasionally, phenytoin severity of disease. Patients may be able to ambu-
and carbamazepine have been reported to cause late and accomplish activities of daily living
tremor. (Watts, 350) despite suffering from severe chorea. Brady-
kinesia, rigidity, dystonia, and postural instabil-
49. (C) An aberrant expansion of glutamines in the ity are more disabling. Dementia in HD is of the
protein huntingtin causes Huntington disease subcortical type. There is prominence of slowed
(HD), a neurodegenerative disorder that strikes thinking, impairment of sequencing, with the
in middle age. The HD gene is located on the absence of cortical deficit, such as aphasia,
short arm of chromosome 4. It is a CAG repeat agnosia, and apraxia. Diminished executive func-
located in exon1 of a 67-exon gene, which is tran- tion includes loss of the ability to plan, sequence,
scribed into huntingtin. Huntingtin is expressed and carry out complex tasks. Eye movement
in neuronal and nonneuronal tissues, suggesting abnormalities occur early in the course of PD.
that its normal function is not confined to cells in Optokinetic nystagmus is impaired in both ver-
the areas of HD. Huntingtin is a cytoplasmic pro- tical and horizontal directions as well as volun-
tein that conserves its location in HD. tary initiation of ocular saccades. (Watts, 492493)
The pattern of huntingtin expression does
not parallel the region of HD neuropathology. 51. (B) Akathisia is an unpleasant sensation of
Only a small subset of neurons that express internal restlessness that is partially relieved by
huntingtin in neuronal population succumb to volitional movements occurring in a patient who
the effect of HD. It has been presumed that has received chronic neuroleptics. (Watts, 319)
mutant huntingtin with its extra glutamines is
toxic to neurons, possibly because it has a ten- 52. (A) Clonidine, an alpha-adrenergic agonist orig-
dency to form aggregates. In HD, there is selec- inally approved for treatment of hypertension, is
tive destruction of the medium-sized spiny neu- actually the treatment of choice for children with
rons in the striatum of the brain, which has been mild or moderate tics. Clonidine has the poten-
attributed either to the accumulation of mutant tial for decreasing impulsiveness and improving
huntingtin aggregates or to the continued expan- attention span along with decreasing tics. The
sion of glutamine repeats. Mutant huntingtin most common side effects are sedation and
affects cortical neurons, producing brain-derived orthostatic hypotension. Dizziness can occur at
neurotrophic factor (BDNF), which is necessary higher doses. Guanfacine is similar to clonidine
for the survival of striatal neurons. and is also marketed as an antihypertensive
One proposal suggests that partial loss of the agent. It has shown some promise as a medica-
beneficial effects of wild-type huntingtin com- tion capable of both decreasing tics and improv-
bined with the toxicity of the mutant huntingtin ing behavior. Clonidine is often the drug of first
conspire to selectively destroy the striatum of the choice for children with tics or Tourette syn-
brain. Huntingtin is a widely expressed protein drome who require symptomatic help. This is
that resides in the cell cytoplasm and may be because the tic problem is often accompanied by
important for transport of vesicles in the endoso- attentional or other behavioral difficulties.
Answers: 4855 399

The dopamine-blocking agent haloperidol has to 60% of these patients have autoantibodies in the
been found to be effective in treating tics. Al- serum and CSF directed against glutamic acid
though newer neuroleptic agents are available, decarboxylase (GAD), an enzyme present in
haloperidol has always been the gold standard GABAergic neurons and pancreatic beta-cells, and
by which all new medications have been judged. a high proportion of them have other autoim-
Experience indicates that haloperidol can reduce mune diseases including diabetes mellitus. (Folli,
tics in approximately 70% of treated persons, but 618)
over 50% of treated patients will complain of side
effects. Only 25% of patients report significant 55. (B) Two distinctive mutation have been identi-
improvement without side effects. The potential fied in the -synuclein gene (SNCA) located on
side effects of all neuroleptics are similar to those chromosome 4q have been linked to a familial
seen with haloperidol, most commonly fatigue form of PD. Alpha-synuclein is a highly con-
and increased appetite. Others include depres- served, abundant 140amino acid protein of
sion and school or work phobia. A third type of unknown function that is expressed mainly in
medication used for the control of tics has been presynaptic nerve terminals in the brain. (Lang,
clonazepam. Traditionally used as a medication 10441053)
either for control of seizures or anxiety, this has
helped individual patients. The dosage is titrated
in a weekly basis until a clinical effect is observed REFERENCES
or until there are side effects. Other medications
that have been reported to improve tics include Aarsland D, Beyer MK, Kurz MW. Dementia in Parkinsons
tetrabenzamine, local injection of botulinum toxin, disease. Curr Opin Neurol. 2008;21:676-682.
and calcium channel antagonists. (Watts, 573) Colcher A, Simuni T. Other Parkinson syndromes. Neurol
Clin. 2001;19:629-649.
53. (A) Cerebellar control of extraocular movements Di Maio L, Squitieri F, Napolitano G, Campanella G, Trofatter
is performed mainly by the following structures: JA, Conneally PM. Suicide risk in Huntington disease.
the dorsal vermis and underlying fastigial J Med Genet. 1993;30:293-295.
nucleus, the flocculus and paraflocculus, and the Elbaz A, Moisan F. Update in the epidemiology of Parkin-
nodulus. Lesions of the dorsal vermis and fastigial sons disease. Curr Opin Neurol. 2008;21:454-460.
Evidente VG, Gwinn-Hardy KA, Caviness JN, Gilman S.
nucleus result in saccadic dysmetria, typically
Hereditary ataxias. Mayo Clin Proc. 2000;75:475-490.
with hypermetric movements and at time with
Folli F. Stiff man syndrome, 40 years later. J Neurol,
macrosaccadic oscillations. Lesions of the floccu- Neurosurg Psychiatry. 1998;65:618.
lus and paraflocculus cause gaze-evoked nystag- Gasser T. Genetics of Parkinsons disease. Curr Opin Neurol.
mus, rebound nystagmus, downbeat nystagmus, 2005;18:363-369.
impaired smooth tracking, glissadic postsaccadic Hallett M. Physiology of basal ganglia disorders: an
drift, and disturbance in adjusting the gain of the overview. Can J Neurol Sci. 1993;20:177-183.
vestibuloocular reflex. Lesions of the nodulus lead Hanna PA. Doody RS. Alien limb sign. Adv Neurol. 2000;82:
to an increase in the duration of the vestibular 135-145.
response. (Watts, 580) Jankovic J. New and emerging therapies for Parkinson dis-
ease. Arch Neurol. 1999;56:785-790.
54. (C) Stiff-man syndrome is a rare disorder of the Lang AE, Lozano AM. Parkinson disease. First of two
parts. N Engl J Med. 1998;339:1044-1053.
CNS, which is characterized clinically by fluctua-
Litvan I. Diagnosis and management of progressive
tion and progressive muscle rigidity and spasms.
supranuclear palsy. Semin Neurol. 2001;21:41-48.
The diagnosis relies also on the presence of con- Mizuno Y, Hattori N, Matsumine H. Neurochemical and
tinuous motor unit activity, without evidence of neurogenetic correlates of Parkinson disease. J Neurochem.
neuromyotonia, extrapyramidal or pyramidal 1998;71:893-902.
dysfunction, or focal lesions of the spinal cord. Poewe W. Luginger E. Depression in Parkinson disease:
Rigidity and spasms may dominate in the axial impediments to recognition and treatment options.
muscles, or in one or more distal limbs. Some 50% Neurology. 1999;52(Suppl 3):S2-S6.
400 14: Movement Disorders

Ritz B, Ascherio A, Checkoway H, et al. Pooled analysis of Thacker EL, OReilly EJ, Weisskopf MG, et al. Temporal
tobacco use and risk of Parkinson disease. Arch Neurol. relationship between cigarette smoking and risk of
2007;64:990-997. Parkinson disease. Neurology. 2007;68:764-768.
Ross GW, Abbott RD, Petrovitch H, et al. Association of cof- Trottier Y, Mandel JL. Biomedicine. Huntingtinprofit and
fee and caffeine intake with the risk of Parkinson disease. loss. Science. 2001;293:445-446.
JAMA. 2000;283:2674-2679. Watts RL, Koller WC. Movement Disorders. Neurological
Tanner CM, Aston DA. Epidemiology of Parkinson disease Principle and Practice. New York: McGraw-Hill;
and akinetic syndromes. Curr Opin Neurol. 2000;13:427-430. 1997.
CHAPTER 15

Neuropathology
Questions

1. Figure 15-1 shows 2. Figure 15-2 is a myofibrillary adenosine triphos-


phatase (ATPase)-stained slide. The dark fibers
(A) polymyositis
are characterized by
(B) dermatomyositis
(C) mitochondrial myopathy (A) a slow twitch speed
(D) inclusion body myositis (B) an intermediate resistant to fatigue
(E) a cluster of regenerating muscle fibers (C) strong staining with modified Gomori
trichome
(D) high myoglobin content
(E) high glycogen content

FIG. 15-1. See color insert. (Reproduced with permission from Amato AA,
Russell JA. Neuromuscular Disorders. New York: McGraw-Hill; 2008.)

FIG. 15-2. See color insert. (Reproduced with permission from Amato AA,
Russell JA. Neuromuscular Disorders. New York: McGraw-Hill; 2008.)

401
402 15: Neuropathology

3. Figures 15-3 shows 4. Figure 15-4 shows


(A) ragged-red fibers (A) target fibers
(B) inclusion body myositis (B) amyloid deposition
(C) polymyositis (C) dermatomyositis
(D) amyloid deposition (D) ragged-red fibers
(E) glycogen deposition (E) metabolic myopathy

A A

B B
FIG. 15-3. See color insert. (Reproduced with permission from Amato AA, FIG. 15-4. See color insert. (Reproduced with permission from Amato AA,
Russell JA. Neuromuscular Disorders. New York: McGraw-Hill; 2008.) Russell JA. Neuromuscular Disorders. New York: McGraw-Hill; 2008.)
Questions: 37 403

5. Figure 15-5 shows 7. Figure 15-7 is suggestive of


(A) nemaline myopathy (A) central core myopathy
(B) Duchenne muscular dystrophy (B) inclusion body myositis
(C) central core myopathy (C) target fibers
(D) ragged-red fibers (D) amyloid deposition
(E) inclusion body myositis (E) mitochondrial myopathy

FIG. 15-5. See color insert. (Reproduced with permission from Amato AA,
Russell JA. Neuromuscular Disorders. New York: McGraw-Hill; 2008.)

6. Figure 15-6 is suggestive of


(A) centronuclear myopathy
(B) hyaline body myopathy
(C) central core myopathy
B
(D) Becker muscular dystrophy FIG. 15-7. (Reproduced with permission from Amato AA, Russell JA. Neuro-
muscular Disorders. New York: McGraw-Hill, 2008.)
(E) mitochondrial myopathy

A B
FIG. 15-6. See color insert. (Reproduced with permission from Amato AA, Russell JA. Neuromuscular Disorders. New York: McGraw-Hill; 2008.)
404 15: Neuropathology

8. Figure 15-8 is suggestive of 10. Figure 15-10 is suggestive of


(A) nemaline myopathy (A) astrocytoma grade II
(B) inclusion body myositis (B) glioblastoma multiforme (GBM)
(C) target fibers (C) ependymoma
(D) amyloid deposition (D) medulloblastoma
(E) mitochondrial myopathy (E) oligodendroglioma

FIG. 15-10. See color insert. (Reproduced with permission from Schiff D,
ONeill BP, eds. Principles of Neurooncology. New York: McGraw-Hill; 2005.)

11. Figure 15-11 is suggestive of


FIG. 15-8. (Reproduced with permission from Amato AA, Russell JA.
Neuromuscular Disorders. New York: McGraw-Hill, 2008.) (A) anaplastic astrocytoma grade III
(B) medulloblastoma
9. Figure 15-9 is suggestive of (C) ependymoma
(D) meningioma
(A) mitochondrial myopathy
(E) lymphoma
(B) steroid myopathy
(C) statin myopathy
(D) nemaline myopathy
(E) dermatomyositis

FIG. 15-11. See color insert. (Reproduced with permission from Schiff D,
FIG. 15-9. See color insert. (Reproduced with permission from Amato AA,
ONeill BP, eds. Principles of Neurooncology. New York: McGraw-Hill; 2005.)
Russell JA. Neuromuscular Disorders. New York: McGraw-Hill; 2008.)
Questions: 815 405

12. Figure 15-12 is suggestive of 14. Figure 15-14 is suggestive of


(A) astrocytoma grade II (A) GBM
(B) GBM (B) pilocytic astrocytoma
(C) ependymoma (C) ependymoma
(D) choroid plexus papilloma (D) oligodendroglioma
(E) meningioma (E) pleomorphic xanthoastrocytoma

FIG. 15-14. See color insert. (Reproduced with permission from Schiff D,
ONeill BP, eds. Principles of Neurooncology. New York: McGraw-Hill; 2005.)

15. Figure 15-15 is suggestive of


(A) central neuroblastoma
B (B) pilocytic astrocytoma
FIG. 15-12. See color insert. (Reproduced with permission from Schiff D,
ONeill BP, eds. Principles of Neurooncology. New York: McGraw-Hill; 2005.) (C) gangliocytoma
(D) oligodendroglioma
13. Figure 15-13 is suggestive of (E) pleomorphic xanthoastrocytoma
(A) astrocytoma grade IV
(B) pilocytic astrocytoma
(C) ependymoma
(D) oligodendroglioma
(E) meningioma

FIG. 15-15. See color insert. (Reproduced with permission from Schiff D,
ONeill BP, eds. Principles of Neurooncology. New York: McGraw-Hill; 2005.)

FIG. 15-13. See color insert. (Reproduced with permission from Schiff D,
ONeill BP, eds. Principles of Neurooncology. New York: McGraw-Hill; 2005.)
406 15: Neuropathology

16. Figure 15-16 is suggestive of 17. Figure 15-17 is suggestive of


(A) central neuroblastoma (A) GBM
(B) pilocytic astrocytoma (B) pilocytic astrocytoma
(C) gangliocytoma (C) ependymoma
(D) oligodendroglioma (D) oligodendroglioma
(E) pleomorphic xanthoastrocytoma (E) pleomorphic xanthoastrocytoma

FIG. 15-17. See color insert. (Reproduced with permission from Schiff D,
ONeill BP, eds. Principles of Neurooncology. New York: McGraw-Hill; 2005.)

FIG. 15-16. See color insert. (Reproduced with permission from Schiff D, 18. Figure 15-18 is suggestive of
ONeill BP, eds. Principles of Neurooncology. New York: McGraw-Hill; 2005.)

(A) GBM
(B) pilocytic astrocytoma
(C) meningioma
(D) central neurocytoma
(E) pleomorphic xanthoastrocytoma

A B
FIG. 15-18. (Reproduced with permission from Schiff D, ONeill BP, eds. Principles of Neurooncology. New York: McGraw-Hill; 2005.)
Questions: 1622 407

19. Figure 15-19 is suggestive of


(A) GBM
(B) ependymoma
(C) meningioma
(D) medulloblastoma
(E) pleomorphic xanthoastrocytoma

FIG. 15-20. See color insert. (Reproduced with permission from Schiff D,
ONeill BP, eds. Principles of Neurooncology. New York: McGraw-Hill; 2005.)

21. Figure 15-21 is suggestive of


(A) lymphoma
(B) ependymoma
(C) meningioma
(D) medulloblastoma
(E) shwannomas

FIG. 15-21. See color insert. (Reproduced with permission from Schiff D,
ONeill BP, eds. Principles of Neurooncology. New York: McGraw-Hill; 2005.)

22. Figure 15-22 is suggestive of


(A) lymphoma
(B) ependymoma
(C) meningioma
(D) neurofibroma
(E) shwannomas

B
FIG. 15-19. (Reproduced with permission from Schiff D, ONeill BP, eds.
Principles of Neurooncology. New York: McGraw-Hill; 2005.)

20. Figure 15-20 is suggestive of


(A) GBM
(B) ependymoma
(C) meningioma
(D) medulloblastoma
FIG. 15-22. See color insert. (Reproduced with permission from Schiff D,
(E) pilocytic astrocytoma ONeill BP, eds. Principles of Neurooncology. New York: McGraw-Hill; 2005.)
408 15: Neuropathology

23. Figure 15-23 is suggestive of 27. Alzheimer type II glia are seen in
(A) lymphoma (A) Canavan disease
(B) ependymoma (B) Alzheimer disease
(C) meningioma (C) Parkinson disease
(D) neurofibroma (D) Huntington disease
(E) medulloblastoma (E) supranuclear palsy

28. Brain herniation through a skull defect is called


(A) fungating herniation
(B) subfalcine herniation
(C) tonsillar herniation
(D) central herniation
(E) unclear herniation

29. Cowdry A inclusions are pathological hallmarks


of
FIG. 15-23. See color insert. (Reproduced with permission from Schiff D, (A) CreutzfeldtJacob disease
ONeill BP, eds. Principles of Neurooncology. New York: McGraw-Hill; 2005.)
(B) rabies encephalitis
(C) herpes encephalitis
24. Which of the following is a pathological nuclear
inclusion? (D) progressive multifocal leukodystrophy
(E) cytomegalovirus (CMV) encephalitis
(A) Marinesco body
(B) Lipofuscin 30. Owls-eye cells are pathological hallmarks of
(C) Lewy bodies
(A) CreutzfeldtJacob disease
(D) Cowdry type A
(B) rabies encephalitis
(E) Hirano body
(C) herpes encephalitis
25. Neurofibrillary tangles are not found in (D) progressive multifocal leukodystrophy
(E) CMV encephalitis
(A) normal aging
(B) Alzheimer disease 31. Negri bodies are hallmarks of
(C) Huntington disease
(A) CreutzfeldtJacob disease
(D) progressive supranuclear palsy
(B) rabies encephalitis
(E) postencephalitic Parkinson disease
(C) herpes encephalitis
26. Bunina bodies are found in (D) progressive multifocal leukodystrophy
(E) CMV encephalitis
(A) Pick disease
(B) amyotrophic lateral sclerosis (ALS) 32. Oligodendrocytes with inclusion bodies are hall-
(C) multiple system atrophy marks of
(D) Alzheimer disease (A) CreutzfeldtJacob disease
(E) normal aging (B) rabies encephalitis
(C) herpes encephalitis
(D) progressive multifocal leukodystrophy
(E) CMV encephalitis
Questions: 2342 409

33. Spongiform changes in the cortex are hallmarks 38. Alpha fetoprotein immunohistochemical stain
of is useful to identify
(A) CreutzfeldtJacob disease (A) meningioma
(B) rabies encephalitis (B) Choroid plexus tumor
(C) herpes encephalitis (C) neurofibroma
(D) progressive multifocal leukodystrophy (D) endodermal sinus tumor
(E) CMV encephalitis (E) medullomyoblastoma

34. Mumps virus has an affinity for which of the 39. Desmin immunohistochemical stain is useful to
following central nervous system (CNS) cells? identify
(A) Neurons (A) meningioma
(B) Astrocytes (B) Choroid plexus tumor
(C) Ependymal cells (C) neurofibroma
(D) Oligodendrocytes (D) endodermal sinus tumor
(E) Microglia (E) medullomyoblastoma

35. HIV encephalitis is characterized by 40. Cytokeratin immunohistochemical stain is


useful to identify
(A) microglial nodules with perivascular or
parenchymal multinucleated cells (A) Choroid plexus tumor
(B) periventricular mixed large and small (B) neurofibroma
B cells (C) endodermal sinus tumor
(C) hemorrhagic necrotizing lesions in the (D) medullomyoblastoma
cortex, basal ganglia, and brainstem with (E) chordoma
Cowdry type A nuclear inclusions and
small cytoplasmic basophilic inclusions 41. Epithelial membrane antigen immunohisto-
(D) Cowdry type B nuclear inclusions chemical stain is useful to identify
(E) meningoencephalitis with ventriculitis
(A) meningioma
36. The most common histological characteristic of (B) choroid plexus tumor
lacunar strokes is (C) neurofibroma
(D) endodermal sinus tumor
(A) atherosclerosis
(E) medullomyoblastoma
(B) mycotic aneurysm
(C) amyloid deposition 42. L26 immunohistochemical stain is useful to
(D) lipohyalinosis identify
(E) coagulation necrosis
(A) endodermal sinus tumor
37. Cytokeratin is positive in which of the following (B) medullomyoblastoma
neoplasms? (C) chordoma
(D) melanoma
(A) Pituitary adenoma
(E) B-cell lymphoma
(B) Meningioma
(C) Melanoma
(D) Glioma
(E) Craniopharyngioma
410 15: Neuropathology

43. Transthyretin immunohistochemical stain is 48. The transverse section of the brain in Figure 15-
useful to identify 24 shows
(A) meningioma (A) Nocardia abscess
(B) choroid plexus tumor (B) mucormycosis
(C) neurofibroma (C) Plasmodium falciparum malaria
(D) endodermal sinus tumor (D) brain metastasis
(E) medullomyoblastoma (E) cysticercosis

44. HMB-45 immunohistochemical stain is useful


to identify
(A) endodermal sinus tumor
(B) medullomyoblastoma
(C) chordoma
(D) melanoma
(E) B-cell lymphoma

45. Neurofilament immunohistochemical stain is


useful to identify
(A) meningioma
(B) choroid plexus tumor FIG. 15-24

(C) neurofibroma
(D) endodermal sinus tumor 49. Figure 15-25 shows
(E) medullomyoblastoma
(A) cavernous hemangioma
(B) temporal arteritis
46. Deletion of chromosome 19q occurs in
(C) arteriovenous malformation
(A) ependymoma (D) polymyositis
(B) glioblastoma (E) cerebral amyloid angiopathy
(C) oligodendroglioma
(D) schwannoma
(E) meningioma

47. The presence of cellular monotony, uniform cell


density, and nuclei surrounded by a rim of clear
cytoplasm giving a fried egg appearance is
most suggestive of
(A) oligodendroglioma
(B) ependymoma
(C) meningioma
(D) dysembryoplastic neuroepithelial tumor
(E) pilocytic astrocytoma
FIG. 15-25. See color insert.
Questions: 4353 411

50. Figure 15-26 shows 52. The lesion in Figure 15-28 is an example of
(A) acute inflammatory demyelinating (A) poliomyelitis
neuropathy (B) BrownSequard syndrome
(B) vasculitic neuropathy (C) Krabbe disease
(C) CharcotMarieTooth neuropathy (D) vacuolar myelopathy
(D) leprosy neuropathy (E) multiple sclerosis
(E) diabetic neuropathy

FIG. 15-28. See color insert.


FIG. 15-26

53. The lesion in Figure 15-29 is characteristic of


51. Figure 15-27 shows
(A) central pontine myelinolysis
(A) acute inflammatory demyelinating (B) multiple sclerosis
neuropathy
(C) progressive multifocal leukoencephalo-
(B) vasculitic neuropathy pathy
(C) CharcotMarieTooth neuropathy (D) ischemic stroke
(D) leprosy neuropathy (E) Krabbe disease
(E) diabetic neuropathy

FIG. 15-29. See color insert.


FIG. 15-27
412 15: Neuropathology

54. Figure 15-30 shows a 56. Figure 15-32 shows


(A) neurofibrillary tangle (A) Cowdry A inclusions
(B) neuritic plaque (B) Marinesco bodies
(C) Lewy body (C) Bunina bodies
(D) Bunina body (D) Lafora bodies
(E) Hirano body (E) Lewy bodies

FIG. 15-30. See color insert. FIG. 15-32. See color insert.

55. Figure 15-31 shows 57. The section of the brain in Figure 15-33 shows a
lesion characteristic of
(A) Lewy bodies
(B) Bunina bodies (A) Wernicke encephalopathy
(C) neurofibrillary tangles (B) carbon monoxide intoxication
(D) Lafora bodies (C) central pontine myelinolysis
(E) Cowdry A inclusions (D) Parkinson disease
(E) Huntington disease

FIG. 15-31. See color insert.


FIG. 15-33. See color insert.
Questions: 5460 413

58. The lesion in the brain section shown in Figure 59. Figure 15-35 illustrates
15-34 is characteristic of
(A) heterotopia
(A) chronic ethanol intoxication (B) agyria
(B) carbon monoxide intoxication (C) polymicrogyria
(C) chronic phenytoin toxicity (D) porencephaly
(D) methanol intoxication (E) holoprosencephaly
(E) lead intoxication

FIG. 15-35

60. Figure 15-36 shows


(A) polymicrogyria
(B) pachygyria
(C) heterotopia
(D) porencephaly
(E) agenesis of the corpus callosum

FIG. 15-34

FIG. 15-36
414 15: Neuropathology

61. Figure 15-37 shows 63. Figure 15-39 shows an


(A) porencephaly (A) ependymoma
(B) holoprosencephaly (B) medulloblastoma
(C) agenesis of the corpus callosum (C) fibrillary astrocytoma
(D) heterotopia (D) subependymoma
(E) cortical dysplasia (E) colloid cyst of the third ventricle

FIG. 15-39. See color insert.


FIG. 15-37. See color insert.

62. Figure 15-38 is a CT scan of the head and biopsy 64. Figure 15-40 illustrates
slide of a 40-year-old man who developed a (A) psammoma bodies
new onset of seizure. What is the most likely (B) Lewy bodies
diagnosis?
(C) neuritic plaques
(A) GBM (D) Cowdry type A inclusions
(B) Clear cell ependymoma (E) Hirano bodies
(C) Fibrillary astrocytoma
(D) Oligodendroglioma
(E) Dysembrylastic neuroepithelial tumor

FIG. 15-38. See color insert.


FIG. 15-40. See color insert.
Questions: 6168 415

65. Figure 15-41 illustrates 67. Figure 15-43 illustrates


(A) normal muscle (A) normal muscle
(B) central core disease (B) central core disease
(C) nemaline myopathy (C) nemaline myopathy
(D) ragged-red fibers (D) ragged-red fibers
(E) dermatomyositis (E) dermatomyositis

FIG. 15-41 FIG. 15-43. See color insert.

66. Figure 15-42 illustrates 68. Figure 15-44 illustrates

(A) normal muscle (A) ragged-red fibers


(B) central core disease (B) dermatomyositis
(C) nemaline myopathy (C) target fibers
(D) ragged-red fibers (D) infantile spinal muscular atrophy
(E) dermatomyositis (E) polymyositis

FIG. 15-42. See color insert. FIG. 15-44. See color insert.
416 15: Neuropathology

69. Figure 15-45 illustrates 71. Figure 15-47 illustrates


(A) ragged-red fibers (A) ragged-red fibers
(B) dermatomyositis (B) dermatomyositis
(C) target fibers (C) target fibers
(D) infantile spinal muscular atrophy (D) infantile spinal muscular atrophy
(E) polymyositis (E) polymyositi

FIG. 15-47. See color insert.


FIG. 15-45. See color insert.

70. Figure 15-46 illustrates 72. Figure 15-48 illustrates


(A) normal muscle (A) normal muscle
(B) central core disease (B) central core disease
(C) nemaline myopathy (C) nemaline myopathy
(D) ragged-red fibers (D) ragged-red fibers
(E) dermatomyositis (E) dermatomyositis

FIG. 15-46. See color insert. FIG. 15-48. See color insert.
Answers and Explanations

1. (C) Figure 15-1 illustrates a modified Gomori cytoarchitecture within denervated muscle cells
trichome stain of a muscle biopsy. It reveals a results in a rounded central zone of disorganized
ragged-red fiber in a patient with mitochondrial filaments that contain fewer mitochondria and
myopathy. (Amato, 73) glycogen. Target fibers have three zones that are
circumferentially oriented; this is best seen on
2. (B) The myofibrillar adenosine triphosphatase NADH-TR staining, as illustrated in Figure 15-
(ATPase) is typically performed at three pHs: 4A. The innermost zone is devoid of mitochon-
4.3, 46, and 9.4 in order to assess the size and the dria, glycogen, phosporylase, and ATPase enzy-
distribution of different muscle fiber types. matic activity. The second zone has increased
Individual muscle fibers can be classified into enzymatic activity, while the third zone exhibits
four different fiber types based on their staining intermediate enzymatic activity. Target fibers
characteristics and physiological properties: can also be appreciated on Gomoritrichome
type I (slow-twitch, fatigue-resistant, with oxida- stain, as illustrated in Figure 15-4B, where they
tive metabolism), 2A (fast-twitch, intermediate stain dark and are surround by a pale-staining
fatigue resistance, with oxidative and glycolytic zone. (Amato, 81)
metabolism), 2B (fast-twitch, poor fatigue resist-
ance, with glycolytic metabolism), and 2C (undif- 5. (A) Figure 15-5 is an illustration of modified
ferentiated and embryonic). The specific muscle Gomoritrichome staining of a muscle biopsy
fiber type is determined by the innervating cross section. It reveals a subsarcolemmal clus-
motor neuron. The different muscle fibers type ter of rods stained reddish-purple, suggestive
are distributed randomly. Figure 15-2 shows a of nemaline myopathy. (Amato, 583)
muscle biopsy myofibrillary adenosine triphos-
phatase (ATPase) staining at a pH 9.4. Type 1 6. (B) Myosin storage or hyaline body myopathy
fibers are lightly stained while type 2 fibers are is a rare congenital myopathy characterized by
dark. (Amato, 7475) large areas devoid of sarcomeres in type I (slow)
fibers on skeletal muscle biopsy. The term hyaline
3. (D) Congo red stain was used in Figures 15-3A body myopathy was introduced because of the
and B. It demonstrates amyloid deposition sur- glassy appearance of the inclusions on
rounding muscle fibers and blood vessels. trichrome stain. Myosin storage or hyaline body
Under routine light microscopy, the amyloid myopathy is a rare congenital myopathy char-
deposition stains pinkish red, as illustrated in acterized by large areas devoid of sarcomeres in
Figure 5-3A; it is apple-green under polarized type I (slow) fibers on skeletal muscle biopsy.
light but is most easily appreciated as bright The characteristic histological feature of myosin
red, using rhodamine optics, as illustrated in storage myopathy is the presence of subsar-
Figure 15-3B. (Amato, 78) colemmal hyalinized bodies in several muscle
fibers that stain pale pink with hematoxylin and
4. (A) A feature of muscle fiber denervation is the eosin (H&E), as illustrated in Figure 15-6A, and
presence of target fibers. Reorganization of the pale green with the modified Gomoritrichrome

417
418 15: Neuropathology

stain, as illustrated in Figure 15-6B; they lack histological features are the presence of ragged-
reactivity for glycogen and oxidative enzyme red fibers on modified Gomoritrichome stain.
stains. On electron microscopy, the hyaline bod- Oxidative enzyme stains with nicotinamide ade-
ies are subsarcolemmal in location and not nine dinucleotide dehydrogenase (NADH), suc-
membrane-bound. They consist of an amor- cinate dehydrogenase (SDH), and cytochrome c
phous, granular substance. Disorganized deli- oxidase (COX). The ragged-red fibers and small
cate profiles of filaments visible at higher mag- arteries intensely react with NADH and SDH
nification have been described. (Amato, 589) stains, forming ragged-blue fibers. Some patients
with mitochondrial myopathies may have no
7. (A) Central core myopathy is a rare disorder ragged-red fibers and normal NADH and SDH
characterized by the occurrence of weakness staining. COX stain (directed against one of the
and hypotonia soon after birth and a general subunits encoded by mitochondrial DNA)
delay in motor development, particularly in appears to be the most sensitive stain and can
walking, which is not achieved until the age of demonstrate scattered muscle fibers with
4 to 5 years. The weakness is greater in proximal reduced or absent stain, as illustrated in Figure
than in distal muscles. Facial, bulbar, and ocular 15-8. (Amato, 632)
muscles are spared. The disease has another
remarkable attribute in that every patient is a 9. (C) Muscle biopsies in statin myopathy reveal
potential candidate for the development of muscle fiber necrosis with phagocytosis and
malignant hyperthermia. Pathologically, the small regenerating fibers, as illustrated in Figure
majority of the muscle fibers appear normal in 15-9, which is a muscle biopsy with modified
size or enlarged; no focal destruction or loss of Gomoritrichome staining. (Amato, 739)
fibers can be found. The unique feature of the
disease is the presence in the central portion of 10. (A) Astrocytomas (WHO grade II) are low-
each muscle fiber of a dense, amorphous con- grade, diffusely infiltrative neoplasms composed
densation of myofibrils or myofibrillar material. of astrocytes. These tumors may occur in patients
This altered zone characteristically lacks mito- of all ages and may arise at any location of the
chondria and other organelles and gives a neuraxis. However, site predilection varies
reduced positive periodic acidSchiff reaction according to the patients age; in adults, they are
and a dark blue coloration with the Gomori more common in the cerebral hemispheres; in
trichrome stain, contrasting with the normal children, these tumors usually arise in the brain-
blue-green color of the peripheral myofibrils. stem. Both grossly and microscopically, astrocy-
Within the core, there is a lack of phosphorylase tomas are ill-defined masses that diffusely infil-
and oxidative enzymes. Most of the cores are in trate the surrounding brain. Histologically, low
type 1 fibers, which predominate in muscle biop- cellularity and minimal cellularnuclear atypia
sies. These cores run the length of the muscle characterize low-grade astrocytomas. Special
fiber, thus differing from the multiple cores or stains are helpful in characterizing the astrocytic
minicores seen in oculopharyngeal and multi- nature of low-grade astrocytomas. These tumors
minicore myopathy. Figure 15-7A and B shows a are conspicuously immunoreactive for GFAP, as
muscle biopsy stained with nicotinamide ade- illustrated in Figure 15-10, and vimentin in both
nine dinucleotide tetrazolium reductase (NADH- cytoplasm and cellular processes. (Schiff, 86)
TR). It demonstrates areas devoid of oxidative
enzyme activity in the center of the fibers or 11. (A) Anaplastic astrocytomas (WHO grade III)
sometimes in eccentric regions (Figure 15-7A) constitute a group of astrocytic neoplasms that,
that extend the length of the fiber longitudinally although exhibiting more prominent histological
(Figure 15-7B). (Amato, 579) features of anaplasia than low-grade astrocy-
toma, do not exhibit the degree of anaplasia
8. (E) The histopathological abnormalities in mus- seen in GBM. Similar to astrocytomas grade II,
cle biopsies of the various mitochondrial anaplastic astrocytomas arise most frequently
myopathies are nonspecific. The characteristic in the cerebral hemispheres; however, they tend
Answers: 717 419

to occur in patients a decade older than those cells arranged in dense fascicles alternating with
with astrocytomas (median age 41 years). a loose microcytic tissue formed by stellate astro-
Microscopically, anaplastic astrocytomas cytic cells. (Schiff, 9091)
(Figure 15-11) are hypercellular compared with
astrocytomas and exhibit a variable degree 15. (D) The oligodendroglioma is derived from
of cytoplasmic and nuclear pleomorphism. oligodendrocytes or their precursor cells and
Gemistocytic elements are more commonly seen. may occur at any age, most often in the third
(Gemistocytic cells are characterized by abun- or fourth decade, with an earlier peak at 6 to
dant eosinophilia, round to slightly angulated 12 years. It is relatively infrequent, constituting
cytoplasm, and eccentric nuclei.) Anaplastic approximately 5% to 7% of all intracranial
astrocytomas typically display increased mitotic gliomas. Males outnumber females 2:1. In some
activity and higher levels of Ki-67 than do low- cases the tumor may be recognized at surgery by
grade astrocyotmas. (Schiff, 8586) its pinkgray color and multilobular form, rela-
tive avascularity, and firmness (slightly tougher
12. (B) GBM is the most common and most aggres- than surrounding brain); it has a tendency to
sive type of primary brain tumor in humans, encapsulate and form calcium and small cysts.
accounting for 52% of all primary brain tumors The neoplastic oligodendrocyte has a small
and 20% of all intracranial tumors. Despite being round nucleus and a halo of unstained cyto-
the most prevalent form of primary brain tumor, plasm, giving the appearance of a fried egg, as
GBM occurs in only 2 to 3 cases per 100,000 illustrated in Figure 15-5. The cell processes are
people in Europe and North America. few and stubby, visualized only with silver car-
Microscopically, glioblastomas manifest a variety bonate stains. Some of the oligodendrocytes
of histological aspects. The tumors are hypercel- have intense immunoreactivity to GFAP, similar
lular and exhibit a high degree of cytoplasmic to normal myelin-forming oligodendrocytes.
and nuclear polymorphism. Figure 15-12A illus- Microscopic calcifications are observed fre-
trates increased mitotic activity and Figure 15- quently, both within the tumor and in immedi-
12B shows areas of necrosis characteristic of ately adjacent brain tissue. The most common
glioblastoma. (Schiff, 87) sites of this tumor are the frontal and temporal
lobes (40% to 70%), often deep in the white
13. (A) Glioblastoma multiforme (astrocytoma matter, with one or more streaks of calcium but
grade IV) is characterized by the presence of little or no surrounding edema. (Ropper, chapter
small areas of necrotizing tissue surrounded 31; Schiff, 9495)
by anaplastic cells forming pseudopalisading
necrosis. This characteristic, seen in Figure 15-3, 16. (B) Figure 15-16 illustrates dense areas of
as well as the typical hyperplastic blood ves- fibrillary cells with the formation of Rosenthal
sels, differentiates the tumor from grade III fibers in a pilocytic astrocytoma. (Schiff, 91)
astrocytoma, which does not have these fea-
tures. (Schiff, 87) 17. (C) Ependymomas are derived from ependy-
mal cellsthat is, the cells lining the ventricles
14. (B) Figure 15-14 shows a pilocytic astrocytoma. of the brain and the central canal of the spinal
These well-circumscribed tumors occur most cord; they are the most common gliomas of the
frequently in children and young adults, with a spinal cord. The cells have both glial and epithe-
peak incidence in the second decade of life. lial characteristics. As one might expect, the
Pilocytic astrocytomas are typically located in tumors grow either into the ventricle or adjacent
midline structures, including the cerebellum, otic brain tissue. The most common cerebral site is
pathways, hypothalamus, third ventricle regions, the fourth ventricle; less often, they occur in the
and brainstem. Most are cystic lesions with a solid lateral or third ventricles. Grossly, those in the
component or mural nodule. Microscopically, fourth ventricle are grayish pink, firm, cauli-
pilocytic astrocytomas typically demonstrate a flower-like growths; those in the cerebrum, aris-
biphasic pattern of growth with bipolar piloid ing from the wall of the lateral ventricle, may be
420 15: Neuropathology

large (several centimeters in diameter), reddish ependymal pseudorosettes are more orderly and
gray, and softer and more clearly demarcated fibrillary. (Schiff, 100102)
from adjacent tissue than astrocytomas, but they
are not encapsulated. The tumor cells tend to 19. (D) Medulloblastoma is the most common
form rosettes with central lumens or, more often, malignant tumor of the CNS arising in child-
circular arrangements around blood vessels hood. This is an invasive and rapidly growing
(pseudorosettes), as illustrated in Figure 15-17. tumor that arises in the posterior part of the
(Ropper, chapter 31; Schiff, 9899) cerebellar vermis and neuroepithelial roof of the
fourth ventricle, accounting for 20% of child-
18. (D) Figure 15-18A and B illustrates a central hood brain tumors. Rarely, it presents elsewhere
neurocytoma. This is a rare tumor of the CNS in the cerebellum or other parts of the brain in
with neurocytic differentiation and a favorable adults. Medulloblastomas are hypercellular neo-
prognosis. Although neurocytoma comprises plasms composed of relatively small cells with
only 0.25% to 0.5% of all intracranial tumors, scant cytoplasm and hyperchromatic nuclei, as
they are the most frequent intraventricular illustrated in Figure 15-19A and B. The cells are
tumor in adults. They occur predominantly in usually arranged n sheets, although focal for-
the third and fourth decades but can be seen in mation of rhythmic nuclear palisades can be
other age group. About three quarter of the cases seen. The tumor cells diffusely infiltrate the cere-
occur in the lateral ventricles or the foramina of bellar cortex with obliteration of the granular
Monro; the remaining one quarter occur in the and molecular layers. (Schiff, 105)
third ventricle. Rarely, they can occur in the cere-
bral hemispheres. Their ventricular location 20. (C) Meningiomas represent approximately 15%
makes symptoms and signs of hydrocephalus of all primary intracranial tumors; they are more
the most common manifestations. The acuteness common in women than in men (2:1) and have
of clinical manifestations is variable. Central neu- their highest incidence in the sixth and seventh
rocytoma is distinguished histologically by a decades of life. Some are familial. The histolog-
homogenous population of cells with round, lob- ical hallmark of meningiomas remains the cel-
ulated nuclei and scant cytoplasm in a conspic- lular meningothelial whorl. The meningothe-
uously fibrillated matrix, as seen in Figure 15- liomatous and transitional meningiomas display
18A and B. Histologically, the tumor is composed the most typical meningothelial appearance.
of solid sheets of small, round to polygonal iso- These are characterized by groups of cells with
morphic tumor cells with a distinct cell mem- poorly defined cell borders, forming the typical
brane. Within the neoplastic cells is a delicate whorls around individual cells, blood vessels,
vascular network. Microcalcifications, when and stromal elements. Psammoma bodies, lamel-
present, are distributed throughout the tumor. lated, calcified structures normally seen inter-
The cytoplasm is fine and amphophilic. A per- mixed with meningioepithelial cells in the arach-
inuclear halo is usually present and prominent, noid membrane, are commonly seen in these
leading to an impression of a clear cell tumor. tumors, but when psammoma bodies are the
The summation of these features generates a predominant feature, as seen in Figure 15-20,
honeycomb pattern similar to that of oligo- the tumor is designed a psammomatous menin-
dendroglioma and clear cell ependymoma. The gioma. (Schiff, 108109)
nuclei of neurocytomas are round, small, and
contain fine speckled chromatin. Nucleoli are 21. (E) Figure 15-21 showed alternation of hyper-
indistinct. Mitotic figures are not readily seen or cellular areas (Antoni A) and loose areas (Antoni
infrequent. Irregular small islands of neuropils B) of the spindle-shaped cells typically seen in
are present. The perivascular arrangement of schwannomas. (Schiff, 111)
neuropils in some areas may closely mimic the
coronary perivascular fibrillary hypocellular 22. (A) Figure 15-22 illustrates a primary CNS lym-
mantles (perivascular pseudorosettes) that are phoma, Histologically, these lymphomas are
considered evidence of ependymoma. The real characteristically angiocentric neoplasms with
Answers: 1829 421

tumor cells arranged in vascular concentric cuff- Hirano bodies are cytoplasmic inclusions.
ings with invasion of the vascular walls and They are eosinophilic refractile inclusions found
increased reticulin deposition. The tumor cells within the cytoplasm and adjacent to neurons.
also diffusely infiltrate the brain parenchyma. They stain positively for actin, tau protein, and
(Schiff, 113) vinculin. They are seen most frequently in Pick
and Alzheimer diseases and are most com-
23. (D) Figure 15-23 illustrates a neurofibroma, a monly located in the hippocampus. Lewy bod-
complex mixture of Schwann cells, fibroblasts, ies are eosinophilic cytoplasmic inclusions that
and perineurial cells that expands a diffuse seg- consist of a dense core surrounded by a halo of
ment of nerve. Neurofibromas display a uni- wide radiating fibrils. (Davis, 35; Sandberg, 78)
form array of delicately waving Schwann cells
with comma-shaped nuclei intermixed with per- 25. (C) Neurofibrillary tangles are abnormal, coarse,
ineurial-like cells and fibroblasts, often within a fibrillary cytoplasmic inclusions, some of which
background myxoid matrix. (Schiff, 111112) are composed of hyperphosphorylated tau pro-
teins that may be ubiquitinated or glycated. They
24. (D) Nuclear inclusions are divided into patho- are flame-shaped in pyramidal neurons and take
logical and nonpathological types. Nonpatho- on a globose appearance in neurons of the locus
logical nuclear inclusions include: ceruleus. Commonly located in the hippocampus
and the temporal lobe, they are seen in normal
The nucleolus: it is the site of processing and aging, progressive supranuclear palsy, posten-
partial assembly of ribosomes, which are cephalitic Parkinson disease, Alzheimer disease,
required for cytoplasmic protein synthesis. and ALSparkinsonismdementia complex of
It is prominent in large neurons, including Guam. (Davis, 2830; Sandberg, 9)
motor neurons and Purkinje cells.
The Marinesco body: a small round eosino- 26. (B) Bunina bodies are abnormal cytoplasmic
philic paranucleolar inclusion seen in normal inclusions. They are small bead-like eosinophilic
aging. It stains with ubiquitin. Its ultrastruc- inclusions most commonly found in motor
ture is composed of intermediate filaments. It neurons of patients with ALS. (Davis, 4244;
is commonly located in pigmented neurons of Sandberg, 910)
the substantia nigra, pyramidal cells of the
hippocampus, and tegmentum of the brain- 27. (A) Alzheimer type II cells are astrocytic cells
stem. with enlarged nuclei and marginated chromatin.
They are commonly located in the globus pal-
Pathological nuclear inclusions include viral lidus, cerebellar dentate nucleus, and cerebral
inclusions: cortex. They are seen in hepatic encephalopathy,
Cowdry type A is an amorphous, large, spher- Wilson disease, and Canavan disease. (Davis,
ical eosinophilic inclusion that displaces the 470471; Sandberg, 13)
nucleus and chromatin to the periphery and
may be surrounded by a halo. It is usually 28. (A) Fungating herniation is a herniation of the
indicative of viral infection. brain through a defect in the skull secondary to
Cowdry type B is a small eosinophilic inclu- trauma or surgery. It is caused by increased
sion that does not displace the nucleus. Its intracranial pressure and often has a fatal
origin may be viral or nonviral. outcome. (Sandberg, 5)

Lipofuscin is a normal cytoplasmic inclusion. 29. (C) The pathological hallmarks of herpes
It is composed of lipid, protein, and carbohydrate. encephalitis are necrotizing lesions of the limbic
It is produced by lysosomes by oxidation of lipid areas and Cowdry type A intranuclear inclusion
and lipoproteins. Hippocampal pyramidal neu- bodies, which are amorphous, large, spherical
rons, thalamus, and motor neurons of the brain- eosinophilic inclusion that displace the nucleo-
stem and spinal cord are among the common loca- lus and chromatin to the periphery. (Davis,
tions of lipofuscin cytoplasmic inclusions. 977981, 10021010, 10181022, 10321034)
422 15: Neuropathology

30. (E) Microglial nodules and necrotizing lesions some 20 and converted into an abnormal one.
in the region of the conus medullaris, cauda Partial breakdown of prion protein may pro-
equina, and the periventricular areas are char- duce protein products that spontaneously poly-
acteristic of central nervous CMV infection. merize into amyloid fibers. (Davis, 977981,
They are associated with the presence of cytome- 10021010, 10181022, 10321034)
galic cells, also known as owls-eye cells. (Davis,
977981, 10021010, 10181022, 10321034) 34. (C) Mumps virus infects ependymal cells. Herpes
and polioviruses infect neuronal cells, whereas
31. (B) In most cases, rabies encephalitis is trans- JC virus attacks both astrocytes and oligoden-
mitted to humans through infected saliva drocytes. Microglial cells are preferentially
injected into soft tissue at the site of a bite from infected by herpesvirus. (Takano, 22152221)
an animal such as a fox or a dog. The etiological
agent of rabies is a rhabdovirus that contains a 35. (A) The combination of multinucleated giant
single-stranded RNA. Histopathological fea- cells, microglial nodules, and perivenular
tures of rabies encephalitis are cytoplasmic negri inflammation has been termed HIV encephalitis
bodies, best seen in large neurons of the and has been identified in 30% to 90% of patients
hippocampus, brainstem, and Purkinje cells. dying with AIDS. There is frequently a diffuse
(Davis, 977981, 10021010, 10181022, 10321034) pallor of the myelin, particularly in deep areas of
the centrum semiovale, with microscopic evi-
32. (D) Progressive multifocal leukoencephalopa- dence of macrophage activation, astrocytosis,
thy (PML) is an infection caused by the JC virus, and productive HIV infection. Cerebral atrophy
a papova (papillomapolyomavacuolating) is common in patients with HIV dementia (HIV-
virus. The hallmark pathological features of PML D), often occurring in a frontotemporal distribu-
are oligodendrocytes with inclusion bodies and tion. The pathology of HIV-D is that of a chronic
multiple frequently coalescing large and small encephalitis with marked macrophage activa-
foci of demyelination. JC virus causes lytic infec- tion. Multiple small nodules containing macro-
tion of oligodendrocytes, leading to demyelina- phages, lymphocytes, and microglia are scat-
tion and to the development of corresponding tered throughout gray and white matter of the
signs. The cut surface of the fixed brain affected brain, appearing more commonly in white mat-
by PML appears asymmetrically pitted by small ter and subcortical gray matter of the thalamus,
foci of gray discoloration mixed with larger con- basal ganglia, and brainstem. These inflamma-
fluent areas of abnormal parenchyma, which tory nodules are not specific to HIV-1 infection
may be centrally necrotic. The lesions tend to be and occur in other infections, including toxo-
most numerous in the cerebral white matter but plasmosis and CMV encephalitis.
also involve the cerebral cortex and deep gray Multinucleated giant cells are also charac-
matter. On microscopic examination, there are teristically seen; their presence correlates with
multiple foci of demyelination. Some are small the degree of dementia and the detection of HIV-
and rounded, others confluent and irregular and 1 DNA. These giant cells are thought to reflect
occasionally centrally necrotic. The homoge- HIV-1 replication because giant multinucleated
neous amphophilic inclusions, seen in oligoden- cells form in HIV-infected macrophage cultures.
drocytes, largely fill the nuclei and consist of Gross pathological examination of primary
closely packed polyomavirus particles, which CNS lymphoma (PCNSL) reveals a bulky tumor
can be identified on electron microscopy. (Davis, with indistinct borders, often contiguous with
977981, 10021010, 10181022, 10321034) meningeal or ventricular surfaces. Most lesions
are supratentorial. Whereas solitary lesions occur
33. (A) The neuropathological characteristics of in one third of cases, multiple lesions are evident
CreutzfeldtJacob disease are spongiform in most cases. These lesions are histologically
changes in the cortex, subcortical astrogliosis, and diffuse, with perivascular involvement, high
deposition of prion proteins. The normal prion mitotic rates, and variable degrees of necrosis
protein is encoded by a gene located on chromo- and microglial reaction. Immunohistochemical
Answers: 3046 423

studies of PCNSL identify these tumors as B cells result from miniature atheromas (microathero-
in origin. mas) that form at the origin of penetrating arter-
Cytomegalovirus has been identified in ies, as well as by plaques within the parent arter-
astrocytes, neurons, oligodendroglia, and capil- ies that obstruct or extend into the branches
lary endothelia. Four pathological lesions are (junctional plaques). Rarely, they are occluded
associated with CMV encephalitis in patients by microemboli. (Goetz, 913914)
with AIDS:
37. (E) Cytokeratin immunohistochemical stains
1. Isolated cytomegalic cells: cytomegalic cells
are useful in the diagnosis of craniopharyn-
without associated microglial nodules or
gioma, carcinoma, chordoma, and epithelial
inflammation.
cyst. (Kubo, 131134)
2. Microglial nodules: dense cellular aggregates
of macrophages, rod cells, or both; typically
38. (D) Immunohistochemistry is the most common
well demarcated from the adjacent parenchy-
method used to identify cell types and tumor
ma and more common in gray matter than in
phenotypes and to classify tumors. Endodermal
white matter. Few microglial nodules (only
sinus tumor stains positively for alpha fetopro-
7% to 12%) contain cytomegalic inclusions.
tein. (McKeeveer, 1921)
3. Focal parenchymal necrosis: discrete foci of
parenchymal necrosis with cytomegalic cells
39. (E) Desmin is used to identify rhabdosarcoma
and macrophages.
and medullomyoblastoma. (McKeeveer, 1921)
4. CMV ventriculoencephalitis: focal or diffuse
destruction of the ependymal lining and necro-
40. (E) Cytokeratin is used to identify cranio-
sis of periventricular parenchymal tissue asso-
pharyngioma as well as chordoma and epithelial
ciated with dense accumulation of cytomegalic
cysts. (McKeeveer, 1921)
cells in the ependymal and periependymal
areas. Ventriculomegaly, necrosis, and hemor-
41. (A) Meningioma, carcinoma, and epithelial
rhage or fibrinous exudates covering the ven-
cysts stain positively for epithelial membrane
tricular system may be evident on gross
antigen. (McKeeveer, 1921)
inspection.
Varicella zoster virus causes meningoen- 42. (E) L 26 identifies B-cell lymphoma. (McKeeveer,
cephalitis with ventriculitis. CNS toxoplasmosis 1921)
is the most common cause of focal brain lesions
in patients with AIDS. The pathology of CNS 43. (B) Transthyretin stains choroid plexus tumors.
toxoplasmosis may include necrosis and hem- (McKeeveer, 1921)
orrhage of choroid plexus. (Arribas, 577587;
Ciacci, 213221; McArthur, 129150) 44. (D) HMB 45 identifies melanoma. (McKeeveer,
1921)
36. (D) Lipohyalinosis, a destructive vasculopathy
linked to severe hypertension, affects arteries 40 45. (C) Neurofilament stains neurofibroma, gan-
to 200 m in diameter. The arterial lumen is glion cell tumors, and pineocytoma. (McKeeveer,
compromised not by an intimal process but by 1921)
thickening of the vessel wall itself. Subintimal
lipid-laden foam cells and pink-staining fibri- 46. (C) Several genetic alterations are known to
noid material thicken the arterial walls, some- exist in human gliomas. In brief, these include
times compressing the lumen. In places the alterations of chromosomes 9p, 10p, 10q, 11p,
arteries are replaced by tangles and wisps of 13q, 17p, 19q, and 22p in diffuse fibrillary astro-
connective tissue that obliterate the usual vas- cytomas and the loss of 1p and 19q combined or
cular layers. The small, deep infarcts that result in isolation in both pure and mixed oligoden-
from occlusion of these arteries are usually drogliomas. The loss of heterozygosity (LOH) of
called lacunes. Small, deep infarcts can also 1p and 19q appears to be specific to tumors of
424 15: Neuropathology

oligodendroglial origin, and this change is displace a rim of Schwann cell cytoplasm, and
shared in both the astrocytic and the oligoden- strip away otherwise normal-appearing myelin.
droglial portion of mixed oligoastrocytomas, (Schmidt, 56)
suggesting that these are clonal. The high fre-
quency of 1p and 19q loss in oligodendrogliomas 51. (C) This slide shows axonal demyelination and
suggests that these regions also harbor tumor remyelination with the development of concentric
suppressor genes. (Perry, 705710) periaxonal Schwann cell processes in the form of
an onion bulb. This is highly suggestive of hyper-
47. (A) Cellular monotony is the main microscopic trophic neuropathy such as CharcotMarieTooth
feature of oligodendroglioma. This is formed by neuropathy. (Schmidt, 910)
monomorphous cells, characterized by uniformly
round more often than oval nuclei with open 52. (D) Vacuolar myelopathy is the most common
chromatin. Nuclei are surrounded by either a rim disease of the spinal cord in AIDS patients. It is
of clear cytoplasm, resulting in the classic fried- characterized by a spongy vacuolation of myelin
egg appearance or a scant, slightly eccentric rim sheets in the posterior and lateral columns, as
of pink cytoplasm and few processes. (Parisi, 12) illustrated in this slide. There is vacuolation of
the spinal white matter in the posterior columns
48. (E) This slide illustrates a brain with dissemi- and lateral corticospinal tracts. Breakdown of
nated cysticercosis. It is the commonest para- myelin, and later axons, is accompanied by an
sitic infection of the CNS in Mexico as well as in accumulation of macrophages containing debris.
other parts of the world, such as South America, (Davis, 996997)
India, and some European countries. The num-
ber of cysts within the CNS varies from one to 53. (A) The two slides illustrate pontine sections;
several hundred. They occur in the parenchyma the left image uses a LFB/PAS stain, demon-
(especially the gray matter), meninges, or ven- strating myelin loss, while the image on the right
tricles. The viable intraparenchymal cysticerci uses a Bielschowsky stain, showing relative
are usually 1 to 2 cm in diameter and contain a preservation of axons. These slides are highly
single invaginated scolex. After degeneration, suggestive of central pontine myelinolysis
they become fibrotic and are represented by a (CPM), a monophasic demyelinating disease that
firm white nodule. (Davis, 896898) predominantly involves the basis pontis. It usu-
ally occurs as a complication of rapid correction
49. (B) This slide illustrates temporal arteritis (giant of hyponatremia. The mechanism of the
cell arteritis), an autoimmune disease involving demyelination is poorly understood. On macro-
large and medium-sized arteries, including the scopic examination, the basis pontis typically
carotid and vertebral arteries and their major includes a fusiform region of gray discoloration,
branches. On pathological examination, temporal which is abnormally soft and appears granular.
arteritis is characterized by a widespread granu- On sectioning, the extent of the lesion is variable.
lomatous inflammation of the arterial walls; it Its cross-sectional area is usually greatest in the
can cause cerebral infarction. Multinucleated upper part of the pons, where only a narrow
giant cells are usually evident, and their cyto- rim of subpial tissue may be spared. It may
plasm may contain fragments of elastic lamina. involve the middle cerebral peduncles but rarely
(Davis, 800801) extends rostrocaudally beyond the confines of
the pons and lower midbrain. The lesion may be
50. (A) This slide shows various stages of demyeli- asymmetric, being largely or completely con-
nation and remyelination with myelin loss, fined to one side of the pons. On microscopic
which may result in naked axons. This is highly examination, CPM appears as an active demyeli-
suggestive of acute demyelinating inflamma- nation. The lesions contain reactive astrocytes and
tory polyneuropathy. The pathological exami- large numbers of foamy lipid-laden macrophages
nation is characterized also by macrophages, but only very scanty lymphocytes. (Davis,
which penetrate the Schwann cell basal lamina, 518520)
Answers: 4759 425

54. (B) This slide shows a neuritic plaque seen in and the cerebral cortex. Typically, the involved
Alzheimer disease (AD). Amyloid plaques are regions are slightly shrunken and show brown
one of the pathological characteristics of AD. discoloration due to hemosiderin deposition,
They are formed by extracellular proteinaceous and there may be petechial hemorrhages. The
deposits, either as amyloid filaments or in non- periventricular and periaqueductal lesions often
filamentous form, with variable associated spare a slender strip of subependymal tissue. In
abnormalities involving neuronal processes that some patients, particularly those with previ-
traverse the abnormal region. The abnormal neu- ously treated disease, the mammillary bodies
ronal processes are termed dystrophic neurites. may be only mildly discolored and other lesions
Plaques associated with abnormal neurites are may be inconspicuous. On microscopic exami-
termed neuritic plaques. Plaques are widely dis- nation, acute lesions are edematous, with rela-
tributed in the brain of patients with AD. The tive preservation of neurons, variable necrosis of
neocortex and hippocampus are always involved. intervening tissue, and loss of myelinated fibers.
Plaques may also be present in the basal ganglia, Capillaries may appear strikingly prominent
the hypothalamus, the tegmentum of the mid- owing to endothelial hyperplasia and cuffing
brain and pons, and the subcortical white matter. by macrophages. (Davis, 536538; Rushing, 9)
(Davis, 10711073; Hart, 13)
58. (D) This patients FLAIR MRI of the head shows
55. (C) This slide illustrates neurofibrillary tangles, a bright signal in the putamen area that correlates
intraneuronal abnormalities seen in AD. with the ischemic necrosis seen in the brain coro-
Neurofibrillary tangles (NFTs) are neuronal inclu- nal section, which also shows a left putamen hem-
sions composed largely of filamentous aggre- orrhagic lesion. Selective bilateral lesions of the
gates of hyperphosphorylated tau proteins that putamen are highly suggestive of methanol intox-
are variably ubiquitinated and glycated. In silver ication. Acute methanol intoxication causes gen-
preparation, several morphological forms of eralized edema of the brain, which usually shows
NFTs can be identified. The shape of the NFT is features of global hypoxic injury. There may be
probably determined by that of the neuron con- scattered petechial hemorrhages and larger, sym-
taining it. Ultrastructural investigation reveals metric foci of hemorrhagic infarction in the puta-
that NFTs are composed of paired helical neu- men and claustrum. Some patients develop exten-
rofilaments with a maximum diameter of 20 nm sive white matter necrosis. Degeneration of retinal
and a periodic narrowing to 10 nm every 80 nm. ganglion cells results in optic nerve atrophy and
(Davis, 10731076; Hart, 13) gliosis. (Davis, 520521; Rushing, 9)

56. (E) This slide shows eosinophilic cytoplasmic 59. (B) The slide in this picture illustrates agyria. It
neuronal inclusions corresponding to a Lewy results from injury of the germinal cells as they
body. Lewy bodies are seen in idiopathic reach the cortex and leads to abnormalities of
Parkinson disease as well as in diffuse Lewy gyral development. The thick cortical ribbon is
body disease. (Davis, 3841; Hart, 5) disproportionately represented compared with
the relative paucity of the centrum semiovale.
57. (A) This is a coronal brain section showing con- The characteristic histological appearance is a
fluent petechial hemorrhages in the mammillary four-layer cortex instead of the six layers of the
bodies consistent with Wernicke encephalopa- normal neocortical pattern:
thy. It is caused by deficiency of thiamine. A mMolecular layer
Lesions are usually discernible in the mammil- A thin external neuronal layer
lary bodies, but may also involve parts of the A sparsely cellular layer with a tangential
hypothalamus, the medial thalamic nuclei, the myelin fiber plexus
floor of the third ventricle, the periaqueductal A thick inner neuronal layer, which splits in
gray, the colliculi, the nuclei in the pon- its deeper zone into columns of cells
tomedullary tegmentum (particularly the dorsal
motor nuclei of the vagus), the inferior olives, (Davis, 294296; Henry, 1314)
426 15: Neuropathology

60. (A) The slide in this question illustrates polymi- typify meningiomas. Psammoma bodies are con-
crogyria, which is characterized by a hypercon- centrically laminated calcifications and are
voluted cortical ribbon of miniature, individu- crowded with psammoma bodies. They are
ally thin gyri, often fused together or piled on often spinal in location. This benign variant is
top of one another. The macrogyric cerebral sur- recognized as meningioma by finding syncytial
face is irregular and has been likened to cobble- cells between the conspicuous, concentrically
stones. Sections of the cerebrum reveal heaped laminated psammoma bodies. (Davis, 212)
up or submerged gyri that widen the cortical
ribbon. Polymicrogyria may be 65. (A) The slide in this question demonstrates a
normal muscle. Normal frozen muscle stains for
Widespread in one or both hemispheres
ATPase at pH 9.4, and this differentiates type I
Bilateral and symmetric in a particular arterial
from type II fibers. At an alkaline pH, the type II
territory (usually the middle cerebral artery)
fibers are dark and the type I fibers are light.
Confined to the opercular region or depths of
There is a normal mosaic pattern of both types
the insula
of fibers with approximately two thirds being
Around porencephalic or hydranencephalic
type II and one third type I. (Heffner, 113)
defects
Focal in almost any neocortical area except 66. (B) The slide in this question illustrates central
the cingulate or striate cortex core disease. This is an autosomal dominant dis-
On microscopic examination, the cortical gray order and the gene defect is located on chromo-
matter is abnormally thin and excessively folded; some 19. Mutation on the ryanodine receptor con-
there is fusion of adjacent gyri and abnormal tains a Ca release channel. Type I fibers have a
cortical lamination. (Davis, 294296; Henry, 14) central pale area representing the lack of oxidative
enzymes. The core tends to be single and central
61. (C) This picture illustrates an agenesis of the within the affected fiber. Structured cores have
corpus callosum. (Henry, 16) preserved cross banding, whereas unstructured
cores have lost their cross binding. (Heffner, 113)
62. (D) The head CT of the patient shows a tumor
with a gyriform pattern of calcification involving 67. (C) The slide in this question shows nemaline
the left frontal lobe. The microscopic features of rod myopathy, a congenital myopathy of auto-
the tumor include cellular monotony and cells somal or recessive inheritance. The diagnosis is
with round nuclei and perinuclear halos, result- suggested by the variable number of rods in the
ing in the classic fried-egg appearance. These trichome stains. (Heffner, 113)
findings are highly suggestive of oligodendri-
oglioma. (Parisi, 13) 68. (C) The slide in this question shows target fibers
in a denervating disease. Target fibers are seen in
63. (E) The slide in this question shows a colloid atrophic fibers and consist of central nonstaining
cyst. Its location in the third ventricle, usually areas surrounded by a darkened rim of oxida-
near the choroid plexus and foramen of Monro, tive enzymes. (Heffner, 113)
helps to distinguish the colloid cyst from other
cysts that superficially resemble it (enteroge- 69. (D) The slide in this question shows panfascic-
nous cysts, ependymal cysts, and Rathkes cleft ular atrophy, suggestive of infantile muscular
cysts), but these occur in different locations. atrophy. (Heffner, 113)
The simple columnar and cuboidal epithelium,
which may be flattened to simple squamous 70. (E) The slide in this question shows the perifas-
epithelium, often contains a mixture of ciliated cicular atrophy of dermatomyositis. (Heffner, 113)
and nonciliated cells. (Parisi, 1011)
71. (E) The slide in this question shows lympho-
64. (A) The slide in this question shows psammoma cytes infiltrating the endomysium between
bodies from a psammomatous meningioma. fibers, suggesting the diagnosis of polymyositis.
Meningothelial whorls and psammoma bodies (Heffner, 113)
Answers: 6072 427

72. (D) The slide in this question shows a modified Kubo O, Tajika Y, Uchimuno H, Muragaki Y, Shimoda M,
Gomori trichome stain of mitochondrial myopa- Hiyama H, et al. Immunohistochemical study of cran-
thy, which stains the mitochondria red, hence iopharyngiomas. Noshuyo Byori. 1993;10:131-134.
the term ragged-red fibers. (Heffner, 113) McArthur JC, Sacktor N, Selnes O. Human immunodefi-
ciency virus-associated dementia. Semin Neurol. 1999;19:
129-150.
McKeever PE. New methods of brain tumor analysis.
Neuropathology Review. AFIP Course 2002.
REFERENCES Parisi JE. Other glial tumors. Neuropathology Review. AFIP
Course 2002.
Amato AA, Russell JA. Neuromuscular Disorders. New York: Perry JR. Oligodendrogliomas: clinical and genetic corre-
McGraw-Hill; 2008. lations. Curr Opin Neurol. 2001;14:705-710.
Arribas JR, Storch GA, Clifford DB, Tselis AC. cyto- Ropper AH, Samuels MA. Intracranial neoplasms and
megalovirus encephalitis. Ann Intern Med. 1996;125:577- paraneoplastic disorders. In: Ropper AH, Samuels MA,
587. eds. Adams and Victors Principles of Neurology. 9th ed.
Ciacci JD, Tellez C, Von Roenn J, Levy RM. Lymphoma of Chapter 31. Available at http://www.accessmedicine.
the central nervous system in AIDS. Semin Neurol. com/content.aspx?aID3637579
1999;19:213-21. Rushing EJ. Toxic metabolic disorders and nutritional defi-
Davis RL, Robertson DM. Textbook of Neuropathology. 3rd ciencies. Neuropathology Review. AFIP Course 2002.
ed. 1997, Baltimore: Williams & Wilkins. xvii, 1409. Sandberg. GD. Introduction to neuropathology. Neuro-
Goetz CG, Pappert EJ, eds. Textbook of Clinical Neurology. pathology Review. AFIP Course 2002.
Philadelphia: Saunders; 1999. Schiff D, ONeill BP. Principles of Neuro-oncology. New York:
Hart MN. Degenerative disease of the CNS. Neuropathology McGraw-Hill; 2005.
Review. AFIP Course 2002. Schmidt RE. Disease of the peripheral nervous system.
Heffner RR. Neuromuscular diseases. Neuropathology Neuropathology Review. AFIP Course 2002.
Review. AFIP Course 2002. Takano T, Takikita S, Shimada M. Experimental mumps
Henry JM. Pediatric neuropathology. Neuropathology virusinduced hydrocephalus: viral neurotropism and
Review. AFIP Course 2002. neuronal maturity. NeuroReport. 1999;10:2215-2221.
This page intentionally left blank
CHAPTER 16

Neuroradiology
Questions

1. These studies show


(A) infarction of the right middle cerebral
artery
(B) bilateral infarction of the anterior cerebral
artery
(C) infarction of the right anterior cerebral
artery
(D) bilateral infarction of the middle cerebral
artery
(E) subdural hematoma

FIG. 1A FIG. 1B

429
430 16: Neuroradiology

2. These are magnetic resonance images (MRIs) of


the head of a 43-year-old man with a history of
progressive ataxia. The most likely diagnosis is
(A) pilocytic astrocytoma
(B) hemangioblastoma
(C) medulloblastoma
(D) metastatic tumor
(E) meningioma

FIG. 2A FIG. 2C

FIG. 2B FIG. 2D
Questions: 23 431

3. These images show the head MRI of a 65-year-


old man who became acutely confused. The
most likely diagnosis is
(A) amyloid angiopathy
(B) metastatic lung cancer
(C) metastatic melanoma
(D) glioblastoma multiforme
(E) hemorrhagic stroke

FIG. 3A FIG. 3C

FIG. 3B FIG. 3D
432 16: Neuroradiology

4. These are T1-weighted images of a gadolinium-


enhanced head MRI of a 50-year-old asympto-
matic man. The most likely diagnosis is
(A) anterior cerebral artery aneurysm
(B) arachnoid cyst
(C) ependymoma
(D) metastasis
(E) meningioma

FIG. 4A FIG. 4B
Questions: 45 433

5. Which of the following clinical manifestations


would one expect to see in a 56-year-old man
with the following head MRI?
(A) Left-sided weakness
(B) Left oculomotor palsy
(C) Anosmia
(D) Left facial palsy
(E) Vertical gaze palsy

FIG. 5A FIG. 5C

FIG. 5B FIG. 5D
434 16: Neuroradiology

6. The most likely diagnosis suggested by this


head MRI is
(A) craniopharyngioma
(B) pituitary adenoma
(C) thrombosed aneurysm
(D) brain metastasis
(E) primary central nervous system (CNS)
lymphoma

FIG. 6A FIG. 6C

FIG. 6B FIG. 6D
Questions: 67 435

7. This head MRI of a newborn baby girl at 38


weeks gestation shows
(A) cystic encephalomalacia
(B) lissencephaly
(C) polymicrogyria
(D) schizencephaly
(E) focal cortical dysplasia

FIG. 7A FIG. 7B
436 16: Neuroradiology

8. These are the imaging studies of an 80-year-old


man with an acute onset of left-sided weakness
and slurred speech. The most likely diagnosis is
(A) metastatic melanoma
(B) metastatic choriocarcinoma
(C) cavernous hemangiomas
(D) arteriovenous malformation
(E) aneurysm

FIG. 8A FIG. 8C

FIG. 8B FIG. 8D
Questions: 88 437

FIG. 8E FIG. 8G

FIG. 8F FIG. 8H
438 16: Neuroradiology

9. These are computed tomography (CT) images


and MRIs of a 39-year-old woman with a history
of AIDS who developed progressive left-sided
weakness. The most likely diagnosis is
(A) progressive multifocal leukodystrophy
(PLM)
(B) CNS lymphoma
(C) brain metastasis
(D) glioblastoma multiforme
(E) bacterial abscess

FIG. 9A FIG. 9C

FIG. 9B FIG. 9D
Questions: 910 439

10. This is a head CT and MRI of a 62-year-old


woman with no past medical history who was
hospitalized with a chief complaint of change in
her mental status. The most likely diagnosis is
(A) brain metastasis
(B) meningioma
(C) aneurysm
(D) oligodendroglioma
(E) pilocytic astrocytoma

FIG. 10A FIG. 10C

FIG. 10B FIG. 10D


440 16: Neuroradiology

FIG. 10E FIG. 10G

FIG. 10F FIG. 10H


Questions: 1111 441

11. This is the imaging study of a 20-year-old


woman with a new onset of acute headache,
subacute fever, and new-onset seizures. The
most likely diagnosis is
(A) normal head MRI
(B) viral meningitis
(C) left anterior cerebral artery stroke
(D) superior sagittal sinus thrombosis
(E) cortical dyplasia

FIG. 11A FIG. 11C

FIG. 11B FIG. 11D


Answers and Explanations

1. (B) This is an unenhanced CT scan of the head. focus on tumors that are sensitive to systemic
It shows a large area of hypodensity within the treatment. MRI is superior to (CT for the detec-
left medial frontal lobe. There is a similar region tion of metastatic disease. (Van den Bent, 717723)
of hypodensity within the right medial frontal
region, which is smaller in size. These findings 3. (C) MRI examination of the brain parenchyma
are typical of evolving infarcts within the terri- demonstrates a right cerebellar lesion with a
tory of the left and right anterior cerebral arter- fluid level, surrounding edema, and minimal
ies (ACAs). There is no mass or midline shift. gadolinium enhancement. There is a lesion with
The basal cisterns are intact. The ventricles are a left posterior temporal fluid level and sur-
normal in size and shape. rounding edema as well as mass effect on the
Infarction in the ACA distribution is uncom- posterior horn of the left lateral ventricle. There
mon and bilateral infarction is rare. ACA infarc- are multiple intracranial lesions demonstrating
tions are cortical or subcortical and are caused fluid levels and hemorrhagic components con-
by embolism. The clinical spectrum of unilat- sistent with hemorrhagic metastasis.
eral infarction in the ACA territory is broad and Cerebral metastases from melanoma, renal
may include disinhibition and hemiparesis pre- cell carcinoma, thyroid carcinoma, and chorio-
dominant in the leg. Bilateral infarction in the carcinoma have a propensity to bleed. Although
ACA territories causes a profound neurological metastases from breast and lung cancer are less
syndrome, highlighted by akinetic mutism and prone to bleed, a cerebral hemorrhagic metasta-
poor recovery. (Minagar, 886) sis is still more likely to originate from lung or
breast cancer because these are the most fre-
2. (D) These are multiple axial images of the brain quently occurring malignancies. A single cere-
MRI, with and without contrast. There is a left bral hemorrhagic mass should lead one to sus-
cerebellar mass with low signal intensity on pect a primary brain tumor such as glioblastoma
T1-weighted images, demonstrating minimal multiforme. (Grossman, 7982)
enhancement. This mass causes compression of
the fourth ventricle. There is also increased sig- 4. (B) This image shows a 2-cm nonenhancing
nal intensity in the subependymal region, con- suprasellar mass. It is rounded, with regular
sistent with transependymal resorption of cere- borders, and has signal characteristics similar
brospinal fluid (CSF). The lateral and third to those of CSF. These characteristics are consis-
ventricles are dilated. These findings are con- tent with an arachnoid cyst.
sistent with obstructive hydrocephalus. Arachnoid cysts are usually congenital mal-
This profile is suggestive of a metastatic formations derived from the meninx primitiva.
tumor. Most brain metastases that present with Their most common location is the midcranial
an unknown primary tumor stem from lung fossa. Less commonly, they may be seen in the
cancer. A more extensive search for the primary cerebral convexities, the basal cisterns, and the
tumor does not appear to improve overall sur- retrocerebellar region. They result from a splitting
vival. The search for a primary tumor should of normal arachnoid membranes by congenital

442
Answers: 18 443

aberrations of increased CSF pulsatile flow. They globin. This lesion is most consistent with pitu-
constitute 1% of all intracranial space-occupying itary macroadenoma. MRI of the sellar and
lesions. On MRI, an arachnoid cyst follows the parasellar regions is the imaging study of choice
intensity of the CSF with low signal intensity for the diagnosis of pituitary adenoma. The size
on T1-weighted images and high signal intensity of the pituitary gland is variable and depends on
on T2-weighted images. The major differential the age and sex of the patient. The maximum
diagnosis with arachnoid cyst is epidermoid height allowed for the pituitary gland is 12 mm
cyst. The latter may show an internal matrix on in women in late pregnancy and postpartum,
T1-weighted nonenhanced MRI and hyperin- 10 mm in other women of childbearing age,
tense signal compared with CSF on FLAIR 8 mm in men and postmenopausal women, and
images. (Grossman, 247249; Pollice, 764765) 6 mm in children. Microadenomas are more
common than macroadenomas; their MRI detec-
5. (D) This is an MRI study with T1-weighted, T2- tion rate varies from 65% to 90%. MRI signs of
weighted, FLAIR-, and diffusion-weighted microadenomas may include hypointensity on
images. There are areas of abnormal signal pres- T1-weighted sequences, focal enlargement of
ent on the FLAIR-weighted images in the pons; the gland, and sella floor thinning or erosion. In
the left inferior pons is particularly bright on case of macroadenomas, head MRI may show
the diffusion-weighted images. The abnormal sellar enlargement in 94% to 100% of cases. It
signals on the FLAIR- and T2-weighted images may also show a lobulated pituitary margin.
in the periventricular deep white matter, which MRI of the pituitary gland is a useful tool in
are not seen on diffusion, correspond to small defining the extent of a pituitary mass to the
vessel ischemic changes. The ventricular system cavernous sinus and in following the effect of
is normal in size and is midline. The subarach- medical treatment on the size of the macroade-
noid spaces are unremarkable. These findings noma. (Anderson, 703721)
are compatible with the diagnosis of acute left
pontine ischemic stroke. 7. (B) MRI of the brain shows a decrease in sulca-
Clinical symptoms of pontine stroke are tion over the temporal, frontal, and right occip-
variable and depend on the location and extent ital lobes consistent with the diagnosis of
of the ischemic damage. In a ventral pontine lissencephaly. The ventricular system is in the
infarct, motor symptoms vary from mild hemi- midline. There is dilatation of both lateral ven-
paresis in a small ventrolateral pontine infarc- tricles; the right lateral ventricle is significantly
tion to contralateral hemiplegia with dysarthria more dilated than the left. Especially dilated are
and ataxia in a ventromedial infarction. Teg- the occipital horns of both lateral ventricles.
mental pontine lesions may cause sensory dis- Lissencephaly is divided into two types based
turbances and ipsilateral cranial nerve dysfunc- on histological criteria, extent of damage, and
tion, such as facial palsy. Basilar artery branch clinical features. In both types, the brain appears
disease is more commonly associated with pon- smooth on gross inspection. In lissencephaly
tine stroke than cardioembolism or large artery type 1, the neocortex has 4 poorly organized
stenosis. Large ventral infarcts carry the worst layers instead of 6 well-organized layers. In type
prognosis. (Bassetti, 165175) 2 lissencephaly, the cortex is unlayered, with a
cobblestone surface and thickened meninges.
6. (B) This is a head MRI study with T1- and T2- The clinical spectrum of lissencephaly is het-
weighted images, including sagittal and coronal erogeneous and may include seizures, motor
sections. It shows a large (4.0 by 2.5 cm) pituitary delay, hypotonia, and mental retardation. (Porter,
sellar mass, with suprasellar extension and bow- 361365)
ing into the sphenoid sinus that has intermedi-
ate signal on T1 and T2. The mass protrudes 8. (C) Figure 16-8A is a head CT upper left image
ventrally into the sphenoid sinus and superi- without contrast. It shows a lesion approxi-
orly compresses the optic chiasm. There is no mately 1.2 cm in diameter with hyperdensity
evidence of chronic hemorrhage or methemo- along the region of the genu of the right internal
444 16: Neuroradiology

capsule. This represents hemorrhage. There is no CT. Positron emission tomography (PET) or sin-
subarachnoid hemorrhage and no associated gle photon emission CT (SPECT) scanning can
mass lesion. The rest of the imaging study com- help confirm the diagnosis. Biopsy is now rarely
prises head MRI images that include T1, proton performed. (Ciacci, 213221)
density, gradient-echo, and T2-weighted images.
Gradient-echo images demonstrate multiple foci 10. (A) The head CT scan of this patient shows a
of markedly decreased signal. These are consistent large region of hypoattenuation within the ante-
with hemosiderin staining, likely related to mul- rior left frontal region, suggesting the presence
tiple cavernous hemangiomas. Cavernous heman- of edema with mass effect on the lateral ventri-
gioma is a benign vascular lesion that may occur cle and a shift of the midline. MRI examination
at any site within the CNS. It is formed by ectatic of the brain shows, within the superior left
endothelium-lined channels without mural mus- frontal lobe, in homogeneously enhancing
cular or elastic fibers and without any neuronal lesions with edema and mass effect as well as
elements. It does not have a direct high-pressure minimal midline shift. These findings are sug-
arterial supply or distinct venous drainage, which gestive of brain metastasis.
distinguishes cavernous hemangioma from arte- Brain metastasis is the most common cause
riovenous malformation. It is characterized, on of an intracranial mass in adults. It is an impor-
T2-weighted MRI, by a reticulated core of mixed tant cause of morbidity and mortality in cancer
signal representing blood various state of degra- patients. These lesions are found at autopsy in
dation surrounded by a hypointense halo due to approximately 25% of patients who die of can-
hemosiderin. (Moran, 561568) cer. Metastases are often multiple; however, in
30% to 50% of cases they may be solitary lesions
9. (B) This imaging study shows a contrast- on brain imaging. Cancers of the lung, breast,
enhancing lesion within the right basal ganglion skin, kidney, and thyroid frequently metasta-
with significant vasogenic edema and mass effect size to the brain. Most patients who present with
on the surrounding structures; there is compres- a brain metastasis of unknown origin suffer
sion of the anterior horn of the right lateral ven- from lung cancer. The typical location of brain
tricle and midline shift. These findings are sug- metastasis is the graywhite matter junction
gestive of either toxoplasmosis or lymphoma and because tumor cells lodge in the small-caliber
less likely PLM. These three diagnoses are par- vessels in this location. Head MRI with contrast
ticularly increased in frequency in AIDS, whereas is the imaging technique of choice in detecting
bacterial abscess, glioblastoma multiforme, and brain tumors. It is more sensitive than head CT
bacterial metastasis are not. Primary CNS lym- with contrast in detecting brain metastasis. A
phoma (PCNSL) accounts for 1% to 1.5% of all metastatic brain lesion typically appears as an
primary brain tumors. It is a common complica- enhanced, rounded, and circumscribed mass
tion in HIV patients and occurs in as many as surrounded by a disproportionate vasogenic
20%. The most common presenting signs of edema, which shows up as an increased signal
PCNSL are altered level of consciousness, focal on T2-weighted images. (Van den Bent, 717723)
neurological deficits, and seizures. Radiological
signs of PCNSL may include a homogeneous 11. (D) The FLAIR head MRI of this patient shows a
enhancing lesion on CT in the central gray mat- serpiginous right frontal hyperintensity signal.
ter or corpus callosum or a ring-enhancing lesion. MR venography reveals cutoff of the anterior half
Head MRI is more sensitive than CT for detecting of the superior sagittal sinus. These findings sug-
CNS lymphoma. Most lesions are located close to gest a right frontal lobe infarction secondary to
the ependymal or meningeal areas. The main dif- superior sagittal venous thrombosis. Cerebral
ferential diagnosis of PCNSL in HIV patient is venous thrombosis (CVT) is commonly under-
CNS toxoplasmosis and progressive multifocal diagnosed, causing a delay in recognizing this
leukoencephalopathy. Lesions in CNS toxoplas- entity, which may lead to devastating disability,
mosis may tend to include more multiple en- and even death. The clinical spectrum of the
hancing lesions of smaller size than PCNSL on disease includes seizures, focal neurological
Answers: 911 445

signs, headache, and papilledema related to Bassetti C, Bogousslavsky J, Barth, A, Regli F. Isolated
increased intracranial pressure. CT (especially infarcts of the pons. Neurology. 1996;46:165-175.
without contrast) as well as conventional T1- and Ciacci JD, Tellez C, VonRoenn J, Levy RM. Lymphoma of
T2-weighted MRI images have a low sensitivity the central nervous system in AIDS. Semin Neurol. 1999;
for diagnosing CVT. Alteration in blood flow and 19:213-221.
Grossman RI, Yousem DM. Neuroradiology: The Requisites.
hemoglobin degradation in cerebral venous
St. Louis: Mosby; 2002.
thrombosis may produce signal changes in con-
Minagar A, David NJ. Bilateral infarction in the territory of
ventional MRI imaging that may suggest the the anterior cerebral arteries. Neurology. 1999;52(4): 886-
diagnosis; however, these changes are often sub- 888.
tle. MR venography (MRV) and echo-planar T2- Moran NF, Fish DR, Kitchen N, Shorvon S, Kendall BE,
weighted imaging are becoming the techniques of Stevens JM. Supratentorial cavernous haemangiomas
choice to establish the diagnosis of CVT and and epilepsy. J Neurol Neurosurg Psychiatry. 1999;66:561-
should be performed when there is a high index 568.
of suspicion of the disease. (Selim, 10211026) Pollice PA, Bhatti NI, Niparko JK. Imaging quiz case 1.
Posterior fossa arachnoid cyst. Arch Otolaryngol Head
Neck Surg. 1997;123:762, 764-765.
Porter BE, Brooks-Kayal, A, Golden JA. Disorders of cor-
REFERENCES tical development and epilepsy. Arch Neurol. 2002;
59:361-365.
Anderson JR, Antoun N, Burnet N, Chatterjee K, Edwards Selim M, Fink J, Linfante I, Kumar S, Schlaug G, Caplan LR.
O, Pickard JD, et al. Neurology of the pituitary gland. J Diagnosis of cerebral venous thrombosis with echo-planar
Neurol Neurosurg Psychiatry. 1999;66:703-721. T2*-weighted magnetic resonance imaging. Arch Neurol.
Atlas SW, Grossman RI, Gomori M, et al. Hemorrhagic 2002;59:1021-1026.
intracranial malignant neoplasms: spin-echo MR imag- Van den Bent MJ. The diagnosis and management of brain
ing. Radiology. 1987;164:71-77. metastases. Curr Opin Neurol. 2001;14:717-723.
This page intentionally left blank
CHAPTER 17

Psychiatry
Questions

1. The highest rate of completed suicide among 5. A 6-year-old boy was brought to the outpatient
adult males is at the age of clinic by his mother because of easy distractibil-
ity and poor school performance. The mother
(A) 20 years
states that for the past 12 months, her son expe-
(B) 35 years rienced difficulty engaging in quiet leisure activ-
(C) 55 years ities, talked excessively, and interrupted others
(D) 65 years frequently. In school, he was reported to avoid
(E) 75 years activities that require mental effort, to have poor
concentration, and to be easily distracted. What
2. The peak of a panic attack is reached within is the most likely diagnosis?
(A) 6 hours (A) Oppositional defiant disorder
(B) 3 hours (B) Bipolar disorder
(C) 1 hour (C) Obsessivecompulsive disorder
(D) 20 seconds (D) Conduct disorder
(E) 10 minutes (E) Attention deficithyperactivity disorder
(ADHD)
3. At what age does an infant drink from a cup,
walk, and say dada/mama nonspecifically? 6. Which of the following medications is con-
traindicated in the management of acute agita-
(A) Four months tion in an 84-year-old man with a history of
(B) Six months normal-pressure hydrocephalus and coronary
(C) Eight months artery disease with arrhythmia?
(D) Ten months
(A) Midazolam
(E) Twelve months
(B) Haloperidol
(C) Droperidol
4. Autonomy versus shame and the doubt stage of
Eriksons epigenetic model of development cor- (D) Morphine sulfate
respond to which stage of Freuds psychosexual (E) Lorazepam
model of development?
(A) Oral phase
(B) Anal phase
(C) Phallic phase
(D) Latency phase
(E) Adolescence phase

447
448 17: Psychiatry

7. The positive reinforcement of alcohol is medi- 11. The craving for alcohol is reduced by
ated by
(A) lorazepam
(A) activation of glutamate receptors (B) naltrexone
(B) activation of gamma aminobutyric acid A (C) disulfiram
(GABA A) receptors (D) diazepam
(C) decreased norepinephrine activity in the (E) amitriptyline
brain
(D) inhibition of dopamine release 12. In the central nervous system (CNS), cocaine
(E) opioid release inhibition acts by
(A) blocking D1 dopamine receptors
8. A 30-year-old alcoholic man was admitted to
the emergency room because he was found by (B) inhibiting acetylcholine esterase in the
his neighbors to be acting agitated and confused. CNS
Which of the following is more suggestive of (C) mediating its rewarding effect through
delirium tremens rather than acute alcoholic hal- dopamine cells in the ventral tegmentum
lucination? area that projects to the basal ganglia
(D) increasing the reuptake of norepineph-
(A) Mild tremor
rine
(B) Auditory hallucination
(E) activating GABA receptors
(C) 20 days duration
(D) Dilated pupils with slow reaction to light 13. Epidemiological studies have shown that in
(E) Clear sensorium schizophrenia,
(A) women tend to have earlier age of onset
9. What is the mechanism of action of disulfiram?
of the disease and poorer outcome than
(A) Inhibition of intestinal absorption of men
alcohol (B) the urban poor population has a lower
(B) Inhibition of liver transport of alcohol incidence of the disease
(C) Inhibition of alcohol dehydrogenase (C) the majority of cases occurring after the
(D) Inhibition of aldehyde dehydrogenase age of 40 are men
(E) Increased renal excretion of alcohol (D) at risk children have a normal scholastic
test
10. Which of the following questions is not a part of (E) children who have been abused have an
the CAGE questionnaire? earlier age of onset and a poorer outcome
(A) Have you gotten into physical fights
14. Major depression is characterized by loss of
when drinking?
interest or pleasure for more than
(B) Have you ever felt you should cut down
on your drinking? (A) 2 years
(C) Have people annoyed you by criticizing (B) 6 months
your drinking? (C) 3 months
(D) Have you ever felt bad or guilty about (D) 6 weeks
your drinking? (E) 2 weeks
(E) Have you ever had a drink first thing in
the morning to steady your nerves or to 15. Which of the following statements about major
get rid of a hangover? depression is true?
(A) The lifetime prevalence rates for adult
men range from 3% to 9%.
Questions: 724 449

(B) Relapse after a single episode is about 50%. (C) major depression
(C) Thirty percent of individuals with a sin- (D) delusion disorder
gle episode of major depression develop (E) conversion disorder
bipolar disorder.
(D) The average age of onset of unipolar 21. Factitious disorder is differentiated from malin-
major depression is 50 years. gering by which of the following characteris-
(E) Full recovery from major depression tics?
occurs in 25% of patients by 6 months. (A) The production of physical signs is under
voluntary control.
16. A good correlation between the blood level and
(B) The absence of secondary gain.
the clinical effect of an antidepressant is seen
with (C) The presence of a serious organic
disorder as a comorbid factor.
(A) imipramine (D) The primary motivation of the patient is
(B) fluoxetine to assume the sick role.
(C) paroxetine (E) The patient may intentionally produce
(D) trazodone symptoms of another person who is
(E) phenelzine under his or her care.

17. Which of the following is not a cardinal feature 22. Which of the following anatomical structures is
of mania? responsible for the genesis of rapid-eye-move-
ment (REM) sleep?
(A) Insomnia
(B) Distractibility (A) Ascending reticular activating system
(C) Low self-esteem (B) Thalamus
(D) Flight of ideas (C) Red nucleus
(E) Thoughtlessness (D) Putamen
(E) Nucleus ceruleus (locus ceruleus)
18. The treatment of choice of rapid-cycling bipolar
disorder is 23. Restless leg syndrome may be exacerbated by
using
(A) lithium
(B) valproic acid (A) fluoxetine
(C) carbamazepine (B) L-dopa
(D) clonazepam (C) bromocriptine
(E) haloperidol (D) clonazepam
(E) pergolide
19. Posttraumatic stress disorder spares the
24. Neuroleptic malignant syndrome results from
(A) thalamus
(B) prefrontal cortex (A) anaphylactic reaction to a neuroleptic
medication
(C) red nucleus
(B) depletion of synaptic dopamine stores in
(D) hippocampus
the CNS
(E) amygdale
(C) blockade of central dopamine receptors
20. La belle indifference is an associated feature of (D) central dopamine receptor hypersensitiv-
ity to neuroleptics
(A) somatization disorder (E) inhibition of serotonin reuptake in the
(B) hypochondriasis CNS
450 17: Psychiatry

25. Which of the followings is a marker of Tourette (D) risperidone


syndrome? (E) olanzapine
(A) Short arm of chromosome 4
31. Prolongation of the QT interval and risk of
(B) X chromosome
arrhythmia is a major side effect of
(C) 5 hydroxy indoleacetic acid
(D) A1 allele of D2 receptors (A) thioridazine
(E) Y chromosome (B) haloperidol
(C) clozapine
26. Which of the followings is a marker of infantile (D) risperidone
autism? (E) olanzapine
(A) Short arm of chromosome 4
32. Thyroid dysfunction is a side effect observed
(B) X chromosome
with
(C) 5 hydroxy indoleacetic acid
(D) A1 allele of D2 receptors (A) thioridazine
(E) Y chromosome (B) haloperidol
(C) clozapine
27. Which of the followings is a marker of (D) risperidone
Huntington disease? (E) quetiapine
(A) Short arm of chromosome 4
33. Orthostatic hypotension is a side effect observed
(B) X chromosome
with
(C) 5 hydroxy indoleacetic acid
(D) A1 allele of D2 receptors (A) thioridazine
(E) Y chromosome (B) haloperidol
(C) clozapine
28. Which of the following is a common marker of (D) risperidone
suicidal behavior? (E) olanzapine
(A) Short arm of chromosome 4
34. Elevation of hepatic transaminase is a side effect
(B) X chromosome
observed with
(C) 5 hydroxy indoleacetic acid
(D) A1 allele of D2 receptors (A) thioridazine
(E) Y chromosome (B) haloperidol
(C) clozapine
29. Agranulocytosis is a major side effect of (D) risperidone
(A) thioridazine (E) olanzapine
(B) haloperidol
35. Grand mal seizure is the most prominent side
(C) clozapine effect of
(D) risperidone
(A) bupropion
(E) olanzapine
(B) phenelzine
30. Akathisia is a major side effect of (C) fluoxetine
(D) amitriptyline
(A) thioridazine
(E) venlafaxine
(B) haloperidol
(C) clozapine
Questions: 2544 451

36. Delirium with abnormal electroencephalographic (A) Somatization disorder


(EEG) and renal function is seen at a minimal (B) Dysmorphic disorder
lithium level in the range of (C) Factitious disorder
(A) 1.2 to 1.5 mEq/L (D) Malingering
(B) 1.6 to 1.9 mEq/L (E) Conversion disorder
(C) 2.0 to 2.5 mEq/L
(D) 2.6 to 3 mEq/L 41. Weight gain is least likely to be caused by the long-
term use of
(E) above 3 mEq/L
(A) clozapine
37. Stimulant drugs appear to be more effective in (B) piperidine
the treatment of which of the following symp- (C) haloperidol
toms of narcolepsy?
(D) molindone
(A) Sleep paralysis (E) risperidone
(B) Sleep attacks
(C) Cataplexy 42. A 10-year-old boy was brought to a psychiatric
(D) Hypnagogic hallucination clinic by his mother because of marked impair-
ment in his social interaction. He has no friends
(E) Restless nighttime sleep
and does not make eye contact. He is unable to
identify objects of interest to others and exhibits
38. The lithium level increases with the coadminis-
stereotyped and repetitive hand and finger flap-
tration of
ping. He has normal language and cognitive
(A) theophylline development. Neurological examination is nor-
(B) mannitol mal. What is the most likely diagnosis?
(C) sodium chloride (A) Asperger syndrome
(D) acetazolamide (B) Autism
(E) captopril (C) Pervasive developmental disorder not
otherwise specified
39. Drug-induced hypertension may be seen with (D) Schizoid personality disorder
the use of
(E) Rett syndrome
(A) venlafaxine
(B) imipramine 43. With kleptomaniacs,
(C) clozapine (A) objects are stolen for their financial value
(D) nortriptyline (B)thievery is pleasurable
(E) risperidone (C) there is antisocial behavior
(D) there is a decreasing sense of tension
40. A 30-year-old woman was evaluated over a 5- immediately before the theft
year period for various symptoms including
(E) after the theft, there is anger and
headaches, arthralgia, and pain in the abdomen,
vengeance
chest, and pelvis. An extensive outpatient workup
in different subspecialty clinics (neurology, car-
44. Priapism is a serious side effect of
diology, gastroenterology, rheumatology, pul-
monary diseases, gynecology, and urology) has (A) haloperidol
been negative. The patient reports a chaotic (B) lorazepam
lifestyle because of her condition despite the (C) trazodone
absence of an organic abnormality. What is the
(D) risperidone
most likely diagnosis?
(E) valproic acid
452 17: Psychiatry

45. The next step in the treatment of a child who has (D) Coarse tremor
ADHD and who fails to respond to methylpheni- (E) Acute renal failure
date is to use
(A) bupropion 50. Which of the following is not a sign of lithium
toxicity?
(B) clonidine
(C) magnesium pemoline (A) Dry mouth
(D) dextroamphetamine (B) Seizure
(E) guaifenesin (C) Constipation
(D) Delirium
46. A 20-year-old woman with no past medical his- (E) Polyuria
tory consults a physician because of recurrent
abdominal pain, nausea, vomiting, and weak- 51. Neurotoxicity of lithium may increase with the
ness in the lower extremities. She also reports coadministration of which of the following
paranoid ideation and auditory hallucinations. drugs?
What is the most likely diagnosis?
(A) Sodium bicarbonate
(A) Hepatic encephalopathy (B) Caffeine
(B) Acute intermittent porphyria (C) Mannitol
(C) Niacin deficiency (D) Acetazolamide
(D) Thiamine deficiency (E) Amlodipine
(E) Cobalamin deficiency
52. Which of the followings is a common side effect
47. What is the substance most likely to provoke an of lithium?
acute panic attack in a patient suffering from a
panic disorder? (A) Agranulocytosis
(B) Hepatoxicity
(A) Carbon dioxide inhalation
(C) Cardiac conduction disturbance
(B) Dopamine
(D) Hypothyroidism
(C) Lactate
(E) Acute pancreatitis
(D) Caffeine
(E) Yohimbine 53. In which stage of pregnancy do major pharma-
cokinetic changes of lithium metabolism occur?
48. Which of the following medications is most
appropriate for an 80-year-old man with major (A) First trimester
depression and history of glaucoma, orthostatic (B) Second trimester
hypotension, and urinary hesitation? (C) Third trimester
(A) Nortriptyline (D) At delivery
(B) Amitriptyline (E) Postpartum and during breast-feeding
(C) Trimipramine
54. Functional neuroimaging of patients with ADHD
(D) Doxepin shows decreased activity in which region of the
(E) Imipramine brain?

49. Which of the following symptoms is the earliest (A) The prefrontal cortex
indication of lithium intoxication? (B) The temporal lobe
(C) The parietal lobe
(A) Impaired consciousness
(D) The thalamus
(B) Myoclonus
(E) The amygdala
(C) Seizures
Questions: 4561 453

55. Irreversible pigmentation of the retina is seen as 59. Heroin overdose may cause
a side effect of the chronic use of
(A) impairment in attention or memory,
(A) chlorpromazine slurred speech, euphoria, and pupillary
(B) thioridazine vasoconstriction
(C) lithium (B) anxiety, depression, paranoid ideation,
(D) risperidone subjective intensification of perceptions,
hallucinations, tachycardia, and sweating
(E) valproic acid
(C) dysarthria, ataxia, muscle rigidity,
56. Which of the following is true about the effect of decreased response to pain, vertical nys-
chronic alcohol consumption on sleep? tagmus, and labile affect
(D) impaired judgment, slurred speech, inco-
(A) Decreased REM sleep ordination, unsteady gait, and nystagmus
(B) Increased stage IV sleep (E) euphoria, tachycardia, elevated blood
(C) Decreased sleep fragmentation pressure, pupillary dilatation, and seizure
(D) Increased sleep latency
(E) None of the above 60. Cannabis overdose may cause
(A) dysarthria, ataxia, muscle rigidity,
57. Which of the following is a sign of acute alcohol decreased response to pain, vertical nys-
intoxication? tagmus, and labile affect
(A) Impairment in attention or memory, (B) impaired judgment, slurred speech, inco-
slurred speech, euphoria, and pupillary ordination, unsteady gait, and nystagmus
vasoconstriction (C) euphoria, tachycardia, elevated blood
(B) Anxiety, depression, paranoid ideation, pressure, pupillary dilatation, and seizure
subjective intensification of perceptions, (D) excitement, restlessness, tachycardia,
hallucinations, tachycardia, and sweating muscle twitching, insomnia, and diuresis
(C) Dysarthria, ataxia, muscle rigidity, (E) euphoria, conjunctival injection,
decreased response to pain, vertical nys- increased appetite, tachycardia, and
tagmus, and labile affect impaired judgment
(D) Impaired judgment, slurred speech, inco-
ordination, unsteady gait, and nystagmus 61. Acute phencyclidine intoxication may cause
(E) Euphoria, tachycardia, elevated blood (A) dysarthria, ataxia, muscle rigidity,
pressure, pupillary dilatation, and seizure decreased response to pain, vertical
nystagmus, and labile affect
58. Acute cocaine intoxication causes
(B) impaired judgment, slurred speech, inco-
(A) impairment in attention or memory, ordination, unsteady gait, and nystagmus
slurred speech, euphoria, and pupillary (C) euphoria, tachycardia, elevated blood
vasoconstriction pressure, pupillary dilatation, and
(B) anxiety, depression, paranoid ideation, seizure
subjective intensification of perceptions, (D) excitement, restlessness, tachycardia,
hallucinations, tachycardia, and sweating muscle twitching, insomnia, and diuresis
(C) dysarthria, ataxia, muscle rigidity, (E) euphoria, conjunctival injection,
decreased response to pain, vertical nys- increased appetite, tachycardia, and
tagmus, and labile affect impaired judgment
(D) impaired judgment, slurred speech, inco-
ordination, unsteady gait, and nystagmus
(E) euphoria, tachycardia, elevated blood
pressure, pupillary dilatation, and seizure
454 17: Psychiatry

62. Acute intoxication with lysergic acid diethy- 65. Signs of cocaine withdrawal are
lamide (LSD) may cause
(A) insomnia, transient visual, tactile, or
(A) impairment in attention or memory, auditory hallucination, and autonomic
slurred speech, euphoria, and pupillary hyperactivity
vasoconstriction (B) dysphoric mood, fatigue, vivid, unpleas-
(B) anxiety, depression, paranoid ideation, ant dreams, and increased appetite
subjective intensification of perceptions, (C) headache, nausea, vomiting, marked
hallucinations, tachycardia, and sweating fatigue and depression
(C) dysarthria, ataxia, muscle rigidity, (D) dysphoric mood, nausea, vomiting, mus-
decreased response to pain, vertical cle aches, pupillary dilation, piloerection,
nystagmus, and labile affect and sweating
(D) impaired judgment, slurred speech, inco- (E) depressed mood, insomnia, decreased
ordination, unsteady gait, and nystagmus heart rate, and increased appetite
(E) euphoria, tachycardia, elevated blood
pressure, pupillary dilatation, and 66. Signs of heroin withdrawal are
seizure
(A) insomnia, transient visual, tactile or
auditory hallucination, and autonomic
63. Acute caffeine intoxication may cause
hyperactivity
(A) dysarthria, ataxia, muscle rigidity, (B) dysphoric mood, fatigue, vivid, unpleas-
decreased response to pain, vertical nys- ant dreams, and increased appetite
tagmus, and labile affect (C) headache, nausea, vomiting, marked
(B) impaired judgment, slurred speech, inco- fatigue and depression
ordination, unsteady gait, and nystagmus (D) dysphoric mood, nausea, vomiting, mus-
(C) euphoria, tachycardia, elevated blood cle aches, pupillary dilation, piloerection,
pressure, pupillary dilatation, and seizure and sweating
(D) excitement, restlessness, tachycardia, (E) depressed mood, insomnia, decreased
muscle twitching, insomnia, and diuresis heart rate, and increased appetite
(E) euphoria, conjunctival injection, increased
appetite, tachycardia, and impaired 67. Signs of nicotine withdrawal are
judgment
(A) insomnia, transient visual, tactile, or
auditory hallucinations, and autonomic
64. Signs of alcohol withdrawal are
hyperactivity
(A) insomnia, transient visual, tactile or audi- (B) dysphoric mood, fatigue, vivid, unpleas-
tory hallucination, and autonomic hyper- ant dreams, and increased appetite
activity (C) headache, nausea, vomiting, marked
(B) dysphoric mood, fatigue, vivid, unpleas- fatigue and depression
ant dreams, and increased appetite (D) dysphoric mood, nausea, vomiting, mus-
(C) headache, nausea, vomiting, marked cle aches, pupillary dilation, piloerection,
fatigue and depression and sweating
(D) dysphoric mood, nausea, vomiting, mus- (E) depressed mood, insomnia, decreased
cle aches, pupillary dilation, piloerection, heart rate, and increased appetite
and sweating
(E) depressed mood, insomnia, decreased
heart rate, and increased appetite
Questions: 6271 455

68. Signs of caffeine withdrawal are (D) Chronic schizophrenia


(A) insomnia, transient visual, tactile. or (E) Obsessivecompulsive disorder
auditory hallucinations, and autonomic
hyperactivity 70. The CAGE questionnaire is used in case of
(B) dysphoric mood, fatigue, vivid, unpleas- (A) catatonic schizophrenia
ant dreams, and increased appetite (B) Pick disease
(C) headache, nausea, vomiting, marked (C) mental retardation
fatigue and depression (D) bipolar disorder
(D) dysphoric mood, nausea, vomiting, mus- (E) alcohol abuse
cle aches, pupillary dilation, piloerection,
and sweating 71. Alprazolams half-life increases with the coad-
(E) depressed mood, insomnia, decreased ministration of
heart rate, and increased appetite
(A) fluoxetine
69. Electroconvulsive therapy is least likely to be (B) fluvoxamine
successful in which of the following diseases? (C) paroxetine
(A) Major depression (D) sertraline
(B) Acute schizophrenia (E) clozaril
(C) Acute manic episodes
Answers and Explanations

1. (E) Suicide, the eighth leading cause of death in sequential, clearly defined stages, and that each
the United States, accounts for more than 30,000 stage must be satisfactorily resolved for develop-
deaths per year. The suicide rate in men (18.7 ment to proceed smoothly. If successful resolution
suicides per 100,000 men in 1998) is more than of a particular stage does not occur, all subsequent
four times that in women (4.4 suicides per stages reflect the failure in the form of physical,
100,000 women in 1998). In females, suicide rates cognitive, social, or emotional maladjustment.
remain relatively constant beginning in the mid- Erikson described eight stages of the life cycle:
teens. In males, suicide rates are stable from the
Stage 1 corresponds to trust versus mistrust.
late teenage years until the late 70s, when the
Stage 2 corresponds to autonomy versus shame
rate increases substantially to 41 suicides per
and doubt.
100,000 persons annually in men 75 to 84 years
Stage 3 corresponds to initiative versus guilt.
of age. (Mann, 302311)
Stage 4 corresponds to industry versus inferiori-
ty.
2. (E) Panic disorder is a syndrome characterized
Stage 5 corresponds to ego identity versus role
by unexpected and unprovoked attacks of anxi-
confusion.
ety that produce both cognitive and physical
Stage 6 corresponds to intimacy versus isolation.
symptoms. The lifetime prevalence of panic dis-
Stage 7 corresponds to generativity versus
order in the general population is 1.6%. The dis-
stagnation.
order has a unimodal distribution, peaking in the
Stage 8 corresponds to ego integrity versus
third decade of life. Panic disorder affects more
despair.
females than males. The major distinguishing fea-
ture of panic disorder is the combination of cog- Stage 2, autonomy versus shame and doubt
nitive and physical symptoms. Onset is rapid, (about 1 to 3 years), corresponds to the anal
peaking within 10 minutes, and the attack lasts phase of Freuds psychosexual model of devel-
about 60 minutes. The typical patient has 2 to 4 opment. Autonomy concerns childrens sense of
attacks per week, often accompanied by antici- mastery over themselves and over their drives
patory anxiety. A patient who sustains 4 panic and impulses. For Erikson, it is the time for chil-
attacks in 4 weeks or 1 or more attacks followed dren either to retain feces (holding in) or to elim-
by 4 weeks of continuous anticipatory anxiety inate feces (letting go); both behaviors have an
may be said to have panic disorder. (Zun, 9296) effect on the mother. Children in the second and
third years of life learn to walk alone, feed them-
3. (E) At the age of 12 months, an infant with nor- selves, control the anal sphincter, and talk.
mal psychomotor development can drink from Muscular maturation sets the tone for this stage
a cup, walk, and say dada/mama nonspecifi- of development. When parents permit children
cally. (Stern, 25) to function with some autonomy and are sup-
portive without being overprotective, toddlers
4. (B) Eriksons formulations were based on the con- gain self-confidence and feel that they can control
cept that epigenetic development occurs in themselves and their world. But if toddlers are

456
Answers: 17 457

punished for being autonomous or are overcon- Diagnosis of ADHD requires the presence of
trolled, they feel angry and ashamed. If parents at least six manifestations from the inattentive-
show approval when children show self-control, ness cluster, the hyperactivity/impulsivity clus-
childrens self-esteem is enhanced and a sense of ter, or both. Children whose symptoms are
pride develops. (Kaplan, 214215, 233239) predominantly from one cluster are said to be
primarily inattentive or hyperactive/impulsive.
5. (E) The patient in this vignette has symptoms of Clinical diagnosis requires that the symptoms be
inattention (he was reported to avoid activities evident before age 7 and be constant for at least
that require mental effort, to have poor concen- 6 months. (Behrman, 107110)
tration, and to be easily distractible), hyperac-
tivity (had difficulty engaging in leisure activity 6. (B) High doses of haloperidol have been asso-
quietly), and impulsivity (he grabbed things and ciated with prolongation of cardiac conduction.
interrupted others frequently). These symptoms Patients with a previous history of dilated ven-
are highly suggestive of ADHD. tricles, arrhythmia, or alcohol abuse have an
ADHD is characterized by poor ability to increased risk of developing torsades de pointes.
attend to a task, motor overactivity, and impul- (Stern, 392)
sivity. Oppositional and aggressive behaviors are
often seen in conjunction with ADHD. The cause 7. (B) Addictive behavior associated with alco-
of ADHD is unknown. Genetic factors as well as holism is characterized by compulsive preoccu-
other factors affecting brain development during pation with obtaining alcohol, loss of control
prenatal and early postnatal life are most likely over consumption, and development of toler-
responsible. An association of the dopamine ance and dependence as well as impaired social
receptor D4 gene with a refined phenotype of and occupational functioning. Like other addic-
ADHD has been demonstrated. Growing evi- tive disorders, alcoholism is characterized by
dence shows that children with ADHD differ from chronic vulnerability to relapse after cessation of
normal children on neuroimaging measures of drinking. To understand the factors that compel
brain structure and function. In particular, a pre- some individuals to drink excessively, alcohol
frontalstriatothalamocortical circuit has been research has focused on the identification of
implicated. ADHD-afflicted children display var- brain mechanisms that support the reinforcing
ious behaviors indicative of problems with atten- actions of alcohol and the progression of changes
tion, hyperactivity, and impulsivity. in neural function induced by chronic ethanol
According to the fourth edition of the consumption that lead to the development of
Diagnostic and Statistical Manual of Mental dependence. More recently, increasing attention
Disorders (DSM-IV), inattentiveness is manifest has been directed toward the understanding of
when a child often or constantly (1) makes care- neurobiological and environmental factors in
less mistakes, failing to give close attention, susceptibility to relapse.
(2) has difficulty sustaining attention, (3) does Ethanol interacts with dopamine function
not seem to listen, (4) does not follow through on in the mesolimbic reward pathway by acti-
tasks, (5) has difficulty getting organized, vating the dopaminergic neurons of the ventral
(6) dislikes or avoids sustained mental effort, tegmental area (VTA). Ethanol increases the fir-
(7) loses things, (8) is easily distracted, and (9) is ing of VTA DA neurons through direct excita-
forgetful. Hyperactivity is evidenced when a tion. The activation of GABA A receptors opens
child often or constantly (1) fidgets, (2) is out of chloride channels, inducing a primary CNS
his or her seat, (3) runs and climbs excessively, depressant effect as well as inhibiting glutamate
(4) has difficulty playing quietly, (5) is always on NMDA receptors; these effects comprise the pos-
the go as though driven by a motor, and (6) talks itive reinforcement produced by the ingestion of
excessively. Impulsivity is reflected in a child alcohol. Other factors involved in the positive
who often or constantly (1) blurts out answers, reinforcement of alcohol include interaction
(2) has difficulty awaiting his or her turn, and with serotonin systems and the release of opioid
(3) interrupts or intrudes on others. peptides. (Stern, 7374; Weiss, 33323337)
458 17: Psychiatry

8. (D) Both delirium tremens and acute alcoholic G Have you ever felt bad or guilty about
hallucinosis occur during the withdrawal period your drinking?
in an alcohol-dependent patient. Acute alcoholic E Have you ever had a drink first thing in the
hallucinosis may start without a drop in blood morning to steady your nerves or to get rid
alcohol concentration and without delirium, of a hangover?
tremor, or autonomic hyperactivity. Halluci- A yes answer is scored 1 and a no answer is
nations are usually auditory and paranoid and scored 0. A score of 2 or more is considered
may last more than 10 days. In delirium tremens, clinically significant.
the patient is confused, with prominent tremor
(Mayfield, 11211123)
and psychomotor activity, disturbed vital signs,
autonomic dysfunction with dilated pupils, and
11. (B) Naltrexone is a pure competitive antagonist
a slow reaction to light. Hallucinations are usually
principally of m but also of k and d opioid recep-
of the visual type. There is difficulty sustaining
tors in the CNS. The effect of naltrexone on alco-
attention, disorganized thinking, and perceptual
hol craving is not well understood but presum-
disturbance. The duration of symptoms is
ably involves antagonism of endogenous opioid
between 3 to 10 days, whereas in acute alcoholic
agonists, which may be released on alcohol
hallucinosis symptom duration is between 5 to
ingestion and may contribute to the subjective
30 days. (Stern, 75)
high. It aids in achieving the goal of abstinence
by preventing relapse and decreasing alcohol
9. (D) Disulfiram is an alcohol-sensitizing agent
consumption. (Kaplan, 10641067)
that alters the response of the body to alcohol,
making its ingestion unpleasant or toxic. It
12. (C) Cocaine acts by blocking reuptake of neuro-
inhibits aldehyde dehydrogenase, the enzyme
transmitters (norepinephrine, dopamine, and
that catalyzes the oxidation of acetaldehyde,
serotonin) at the synaptic junctions, resulting
causing blood acetaldehyde levels to increase.
in increased neurotransmitter concentrations.
The disulfiramethanol reaction (DER) varies
Because norepinephrine is the primary neuro-
inversely with the dose of disulfiram and the
transmitter of the sympathetic nervous system,
volume of alcohol consumed. The most com-
sympathetic stimulation results and leads to vaso-
mon symptoms of the DER are warmness and
constriction, tachycardia, mydriasis, and hyper-
flushed skin, especially in the upper chest and
thermia. CNS stimulation may appear as increased
face; tachycardia; palpitations; and decreased
alertness, energy, talkativeness, repetitive beha-
blood pressure. Nausea, vomiting, shortness of
vior, diminished appetite, and increased libido.
breath, sweating, dizziness, blurred vision, and
Psychological stimulation by cocaine produces an
confusion may also occur. In addition to its
intense euphoria that is often compared with
effects on aldehyde dehydrogenase, disulfiram
orgasm. Pleasure and reward sensations in the
inhibits a variety of other enzymes, including
brain have been correlated with increased neuro-
dopamine beta-hydroxylase. Thus, in addition
transmission in the mesolimbic or mesocortical
to the toxicity of the DER caused by the accu-
dopaminergic tracts (or both). Cocaine increases
mulation of acetaldehyde, adverse effects of
the functional release of dopamine, which acti-
disulfiram or its metabolites can occur as a result
vates the ventral tegmentalnucleus accumbens
of multiple drug interactions. (Kranzler, 401423)
pathway, which seems to be a major component of
the brain reward system. Activation of this path-
10. (A) The CAGE test is a quick and reliable tool
way is essential for the reinforcing actions of psy-
with which to assess alcohol abuse. It comprises
chomotor stimulants. (Warner, 226235; Withers,
four simple questions:
6378)
C Have you ever felt you should cut down
on your drinking? 13. (E) The prevalence of schizophrenia varies by
A Have people annoyed you by criticizing region in the United States. Its incidence appears
your drinking? to be higher among the urban poor. Males
Answers: 819 459

manifest the illness between 18 and 25 years of task force concluded that plasma level measure-
age whereas females show their first symptoms ments of imipramine, desmethylimipramine,
between 26 and 45 years. Twenty percent of and nortriptyline are unequivocally clinically
cases of schizophrenia occur after age 40; most useful in certain situations. For imipramine, the
are women. Children at risk of schizophrenia percentage of favorable responses correlates with
have a lower scholastic test score, abnormal plasma levels in a linear manner between 200
affect, and thought disorder early in infancy. and 250 ng/mL, but some patients may respond
Those who are abused have an earlier onset and at a lower level. At levels that exceed 250 ng/mL,
a worse course. (Stern, 99) there is no improved favorable response, and
side effects increase. (Task Force, 155162)
14. (E) Major depression is a cluster of psycholog-
ical and physical symptoms that persist for 2 or 17. (C) The diagnosis of manic episodes is estab-
more weeks and interfere with a persons ability lished by the presence of irritability or euphoria
to function or enjoy life. (Kaplan, 534) associated with 3 (euphoria) or 4 (irritability) of
the 7 cardinal symptoms of mania. The cardinal
15. (B) The National Comorbidity Survey carried symptoms of mania are distractibility, insom-
out a structured psychiatric interview of a rep- nia, grandiosity, flight of ideas, increased activ-
resentative sample of the general population and ities, pressured speech, and thoughtlessness.
reported a lifetime rate of major depression of (Stern, 116)
21.3% in women and 12.7% in men, producing a
female-to-male ratio of 1.0 to 1.7. A gender dif- 18. (B) Divalproex sodium (valproic acid with
ference was found beginning in early adoles- sodium valproate) was approved by the U.S.
cence and persisting through the mid-50s. Food and Drug Administration (FDA) for the
Although this increased tendency for depression treatment of acute mania in 1995. A therapeutic
in women reflects a long-term trend, an increase blood-level window of 45 to 125 g/mL has
has also been seen in the rate of depression been demonstrated to correlate with antimanic
among young women over the short term. The response. Valproate may have better efficacy
highest rate occurs in adult women above 44 than lithium in the treatment of mixed manic
years of age. Major depression is a recurrent ill- states, rapid cycling mania or other complex,
ness; the risk of relapse after one episode is about comorbid forms of bipolar disorder and thus
50%, whereas it is greater than 80% after three may synergize with lithium to prevent relapses.
episodes. The average lifetime number is four. Valproate can also be useful in the treatment of
The average age of onset of unipolar depression AIDS-related mania. (Goldberg, 211231)
is 29 years. Some 5% to 10% of individuals with
a single episode of major depression will even- 19. (C) The neurobiology of posttraumatic stress
tually develop bipolar disease, whereas 50% of disorder (PTSD) involves the thalamus as a relay
those experiencing major depression will recover of information about a threat to the prefrontal
fully by 6 months. (Brown, 241268; Stern, 104105) cortex and amygdala. The hippocampus was
found to be affected in adults with PTSD and
16. (A) Blood levels can be obtained for all antide- appears to be related to increased exposure to
pressant drugs, but not all of them have shown excitatory amino acids and glucocorticoids. The
a correlation between therapeutic effect and amygdala plays a key role in consolidating the
blood level. In 1985, a task force examined the emotional significance of events. In fact,
present status of studies investigating the rela- Vietnam combat veterans with PTSD showed
tionship between blood plasma concentrations of left amygdala activation on single photon emis-
tricyclic antidepressants and clinical outcome. It sion computed tomography (SPECT) study in
discussed some of the discrepancies that have response to exposure to combat sound, whereas
developed among various antidepressant drugs combat veterans without PTSD and noncom-
and evaluated the clinical implications of the batant controls did not exibit amygdala activa-
current status of blood level monitoring. The tion. (Newport, 211218)
460 17: Psychiatry

20. (E) Somatization disorder is characterized by leg syndrome, whereas medications such as ben-
the recurrence of multiple somatic complaints zodiazepines, levodopa, quinine, opioids, pro-
not accounted for by medical findings. It is a pranolol, and carbamazepine (Tegretol) have
chronic condition with female predominance. some benefit. (Stern, 178)
Hypochondriasis is also a chronic condition
characterized by a fear or the belief that one has 24. (C) Neuroleptic malignant syndrome is an
a serious illness despite adequate medical eval- uncommon but potentially fatal idiosyncratic
uation. Its prevalence is 4% to 9% of medical reaction characterized by the development of
outpatients, with equal incidence between men altered consciousness, hyperthermia, autonomic
and women. Major depression is a comorbid dysfunction, and muscular rigidity on exposure
condition of both somatization disorder and to neuroleptic (and probably other psychotropic)
hypochondriasis. A family history of somatiza- medications. The pathophysiology of neurolep-
tion disorder, antisocial disorder, and substance tic malignant syndrome (NMS) is poorly under-
abuse is reported in somatization disorder, stood. The postulated mechanism involves
whereas a history of illness in family members blockade of central dopamine receptors in the
is reported in hypochondriasis. La belle indif- basal ganglia and hypothalamus and peripher-
ference is an associated feature of conversion ally in postganglionic sympathetic neurons and
disorder, where symptoms do not conform to smooth muscle. The known plasticity of the
anatomical pathways. Delusion is not a com- mesostriatalmesolimbic dopaminergic system
mon feature of either somatization disorder or is important in protecting the brain against
hypochondriasis. Delusional disorder may be a severe biopsychosocial stressors by means of an
comorbid condition in body dysmorphic disor- appropriately timed receptor downregulation.
der. (Stern, 144146) In most people, this homeostatic mechanism
is sufficient to protect against psychosis; neu-
21. (B) Absence of secondary gain is the main fea- roleptics may help further decrease dopamine
ture that differentiates factitious disorder from receptor sensitivity when the native mechanisms
malingering. In factitious disorder, the patient are insufficient. For some patients, however, this
intentionally produces physical or psychological further reduction in general dopaminergic tone
signs or symptoms that are under voluntary con- will result in NMS. As a corollary, it was
trol and not explained by any other underlying suggested that the primary mesolimbic hyper-
physical or mental disorder. The primary moti- dopaminergia might induce a homeostatic
vation of the behavior is to assume the sick role. response, via GABAergic feedback from the
There is no secondary gain such as economic nucleus accumbens, consisting of downregula-
benefit or avoidance of legal responsibilities. In tion of dopamine receptors in the mesostriatum
malingering, the patient has an obvious recog- and hypothalamus. Such a response would then
nizable secondary gain in producing signs and result in a reduction in local dopaminergic tone
symptoms such as avoiding work or prosecu- sufficient to produce lethal catatonia, despite
tion, or obtaining financial gain. (Stern, 147150) the fact that such mesolimbic hyperdopaminer-
gia would simultaneously cause psychosis.
22. (E) Hobson proposed the most currently accept- (Longhurst, 537538)
able neuroanatomic model for wakefulness and
sleep where REM is proposed to arise from the 25. (D) The prevalence of the D2A1 allele in a range
activation of the nucleus ceruleus and the gigan- of impulsive, compulsive, addictive disorders
tocellular tegmental field, whereas wakefulness ranged from 42.3% to 54.5%. An indication of
is maintained by the ascending reticular activat- the importance of the dopamine D2 receptor in
ing system. (Hobson, 1990, 371382, 1975, 369403; Tourette syndrome comes from SPECT studies of
Stern, 76) monozygotic twins discordant for tic severity.
Differences in the D2 receptor density in the head
23. (A) Serotonin reuptake inhibitors such as flu- of the caudate nucleus predicted differences
oxetine may exacerbate symptoms of restless in phenotypic severity with a high correlation
Answers: 2030 461

coefficient, suggesting that striatal dopamine D2 a low CSF 5-HIAA level is the medical severity
receptor density accounted for 98% of the vari- of the attempt. CSF 5-HIAA is low in serious
ance of tic severity. Dopamine is a stress-respon- suicide attempters, even when the presence of a
sive neurotransmitter. Some studies, using psychiatric illness (e.g., major depression) is con-
SPECT or positron emission tomography (PET), trolled for and the patients are studied in a drug-
show increased density of the presynaptic dopa- free, controlled environment. (Comings, 5083;
mine transporter and the postsynaptic D2 dopa- Margolis, 10191031; Oquendo, 1125; Weeks, 401408)
mine receptor in Tourette syndrome. These stud-
ies suggest that there is abnormal regulation of 29. (C) Clozapine is classified as an atypical
dopamine release and uptake in this disease. antipsychotic drug because its profile of binding
(Comings, 5083; Margolis et al., 10191031; Oquendo, to dopamine receptors and its effects on various
1125; Weeks, 401408) dopamine-mediated behaviors differ from those
exhibited by more typical antipsychotic drug
26. (B) Fragile-X syndrome is primarily a disorder products. In particular, although clozapine does
of neurodevelopment, although other organ sys- interfere with the binding of dopamine at D1, D2,
tems are also involved. In addition to mental D3, and D5 receptors and has a high affinity for
retardation, the mutation also predisposes af- the D4 receptor, it does not induce catalepsy. This
fected individuals to a variety of psychiatric syn- evidence, consistent with the view that clozapine
dromes. A substantial number of males with the is preferentially more active at limbic than at stri-
fragile-X mutation have autism. In one study, atal dopamine receptors, may explain the relative
nearly 100% had one or more behaviors com- lack of clozapine-induced extrapyramidal side
monly observed in autism, such as hand flap- effects. Clozapine also acts as an antagonist at
ping and biting, poor eye contact, or tactile defen- adrenergic, cholinergic, histaminergic, and sero-
siveness. (Comings, 5083; Margolis et al., 10191031; tonergic receptors. The incidence of clozapine-
Oquendo, 1125; Weeks, 401408) induced agranulocytosis is about 1.3% at 1 year,
based on the occurrence of 15 U.S. cases out of
27. (A) Huntington disease has an autosomal dom- 1743 patients in the premarketing period. This
inant transmission. Its gene is located on the reaction could prove fatal if not detected early
short arm of chromosome 4. Huntington disease and therapy interrupted. The incidence rates of
is caused by a mutation in the IT15 gene, result- agranulocytosis based upon a weekly monitor-
ing in abnormal polyglutamine expansion in the ing schedule rose steeply during the first 2 months
N-terminal region of huntingtin protein (Htt). of therapy, peaking in the third month. Among
The expanded polyglutamine repeat alters the clozapine patients who continued the drug
normal functions of Htt. In addition, the mutated beyond the third month, the weekly incidence of
protein, expanded Htt, is itself toxic. Htt interacts agranulocytosis fell to a substantial degree, so that
with an array of proteins in neuronal cells. One by the sixth month the weekly incidence of agran-
important characteristic of Huntington disease is ulocytosis was reduced to 3 per 1000 person-years.
the particular vulnerability of a particular brain After 6 months, the weekly incidence of agranu-
region, the caudateputamen, despite similar locytosis declined still further; however, it never
expression of the mutated protein in other parts reached zero. (Coyle, 3442)
of the brain. (Roze, 497503)
30. (B) Haloperidol is a neuoleptic of high potency.
28. (C) Platelet 5-HT2A receptors have been found Extrapyramidal syndrome (EPS) during its
to be increased in proportion to the lethality of administration has been reported frequently, often
the suicide attempt in depressed subjects. during the first few days of treatment. EPS can be
Approximately two thirds of studies compar- categorized generally as having Parkinson-like
ing subjects who have attempted suicide ver- symptoms, akathisia, or dystonia (including
sus nonattempters show that those who have opisthotonos and oculogyric crisis). While all can
attempted suicide have low levels of CSF 5- occur at relatively low doses, they occur more fre-
HIAA. One of the factors that is correlated with quently and with greater severity at higher doses.
462 17: Psychiatry

The symptoms may be controlled with dose combination of dopamine type 2 (D2) and sero-
reductions or administration of antiparkinson tonin type 2 (5HT2) antagonism. Antagonism at
drugs such as benztropine. (Coyle, 3442) receptors other than D2 and 5HT2 may explain
some of the other effects of risperidone. It may
31. (A) Thioridazine is a D2 dopamine antagonist of induce orthostatic hypotension associated with
low potency. CNS side effects include occasional dizziness, tachycardia, and in some patients,
drowsiness, especially where large doses are given syncope, especially during the initial dose-titra-
early in treatment. Generally, this effect tends tion period, probably reflecting its alpha-adren-
to subside with continued therapy or a reduction ergic antagonistic properties. Syncope was re-
in dosage. Pseudoparkinsonism and other ported in 0.2% of risperidone-treated patients
extrapyramidal symptoms may occur but are in phase 2 and phase 3 studies. (Coyle, 3442)
infrequent. Nocturnal confusion, hyperactivity,
lethargy, psychotic reactions, restlessness, and 34. (E) Olanzapine is an antipsychotropic agent that
headache have been reported but are extremely belongs to the thienobenzodiazepine class.
rare. Adverse cardiovascular reactions are the Olanzapine is a selective monoaminergic antag-
most serious side effects and include a dose- onist with high affinity binding to the following
related prolongation of the QTc interval, which is receptors: serotonin 5HT2(A)2C, dopamine D1-D4,
associated with the ability to cause torsades de muscarinic M1-5, histamine H1, and adrenergic 1
pointestype arrhythmias, a potentially fatal poly- receptors. In placebo-controlled studies, clinically
morphic ventricular tachycardia, and sudden significant ALT (SGPT) elevations (equal to or
death. Autonomic nervous system side effects of greater than three times the upper limit of the
thioridazine include dryness of mouth, blurred normal range) were observed in 2% of patients
vision, constipation, nausea, vomiting, diarrhea, exposed to olanzapine compared to none of the
and urinary retention. (Coyle, 3442) placebo patients. Periodic assessment of transam-
inases is recommended in patients with signifi-
32. (E) Quetiapine fumarate is an antagonist at mul- cant hepatic disease. (Coyle, 3442)
tiple neurotransmitter receptors in the brain: sero-
tonin 5HT1A and 5HT2, dopamine D1 and D2, 35. (A) Bupropion is associated with grand mal
histamine H1 and adrenergic 1 and 2 recep- seizures in approximately 0.4% (4 of 1000) of
tors. Clinical trials with quetiapine fumarate patients treated at doses up to 450 mg/day. This
demonstrated a dose-related decrease in total and incidence of seizures may exceed that of other
free thyroxin (T4) of approximately 20% at the marketed antidepressants by as much as four-
higher end of the therapeutic dose range, which fold. This relative risk is only an approximate
was maximal in the first 2 to 4 weeks of treatment estimate because of the lack of direct compara-
and maintained without adaptation or progres- tive studies. The estimated seizure incidence for
sion during more chronic therapy. Generally, Bupropion increases almost 10-fold between 450
these changes were of no clinical significance and and 600 mg/day, which is twice the usually
TSH was unchanged in most patients, and levels required daily dose (300 mg). (Coyle, 107)
of TBG were unchanged. In nearly all cases, ces-
sation of quetiapine fumarate treatment was asso- 36. (C) Lithium adverse reactions may be encoun-
ciated with a reversal of the effects on total and tered at serum levels below 1.5 mEq/L. Mild to
free T4 irrespective of the duration of treatment. moderate adverse reactions may occur at levels
(Coyle, 3442) from 1.5 to 2.5 mEq/L and moderate to severe
reactions may be seen at levels of 2.0 mEq/L and
33. (D) Risperidone is an antipsychotic agent above. Fine hand tremor, polyuria, and mild
belonging to the benzisoxazole derivatives. The thirst may occur during initial therapy for the
mechanism of action of risperidone, as with acute manic phase and may persist throughout
other antipsychotic drugs, is unknown. How- treatment. Transient and mild nausea and gen-
ever, it has been proposed that this drugs eral discomfort may also appear during the first
antipsychotic activity is mediated through a few days of lithium administration. Diarrhea,
Answers: 3143 463

vomiting, drowsiness, muscular weakness, and 41. (D) A common adverse effect of treatment with
lack of coordination may be early signs of lithium dopamine receptor antagonists is weight gain,
intoxication and can occur at lithium levels below which can be significant in some cases. Molindone
2.0 mEq/L. At a level between 2 and 2.5 mEq/L, and, perhaps, loxapine are not associated with
moderate to severe signs of toxicity may appear, the symptom and may be indicated in patients
such as delirium, abnormal EEG, abnormal renal for whom weight gain is a serious health hazard
function cardiac arrhythmia, and risk of coma. At or a reason for noncompliance. (Kaplan and
a level above 2.5 mEq/L, signs of severe intoxi- Sadock, 1030)
cation may appear that include acute renal fail-
ure seizure and death. Treatment is by dialysis. 42. (A) Asperger syndrome is characterized by a
(Coyle, 125128; Kaplan, 1050) severe, sustained impairment in social interaction
and restricted, repetitive patterns of behavior,
37. (B) Stimulants appear most effective against interests, and activities. Unlike autistic disorder,
daytime somnolence and sleep attacks associ- in Asperger syndrome there are no significant
ated with narcolepsy; they are less beneficial for delays in language, cognitive development, or
catalepsy. (Stern, 177178) age-appropriate self-help skills. The cause is
unknown, but family studies suggest a possi-
38. (E) The lithium level decreases with the coad- ble relation to autism. The similarity of Asperger
ministration of mannitol, urea, theophylline and syndrome to autism leads to genetic, metabolic,
aminophyline, sodium chloride, acetazolamide, infectious, and perinatal hypotheses. The clini-
and sodium bicarbonate. It increases with the cal features include at least two of the following
coadministration of angiotensin converting indications of qualitative social impairment:
enzyme (ACE) inhibitors, thiazide diuretics, and markedly abnormal nonverbal communicative
nonsteroidal anti-inflammatory drugs. (Stern, gestures, the failure to develop peer relation-
385) ships, the lack of social or emotional reciprocity,
and an impaired ability to express pleasure in
39. (A) Venlafaxine is an effective antidepressant other peoples happiness. (Kaplan, 11901191)
drug that acts by nonselective inhibition of the
reuptake of three biogenic amines: serotonin, 43. (B) The essential feature of kleptomania is a re-
norepinephrine, and dopamine. An increase in current failure to resist impulses to steal objects
blood pressure was seen in clinical trials in not needed for personal use or for monetary
patients treated with venlafaxine. This occurred value. The objects taken are often given away,
most often with doses above 200 mg per day returned surreptitiously, or kept and hidden.
and seems to be dose-dependent. (Stern, 356) People with kleptomania usually have the
money to pay for the objects they impulsively
40. (A) The patient in this vignette has recurrent steal. Like other impulse-control disorders, klep-
multiple chronic somatic complaints that started tomania is characterized by mounting tension
at the age of 25 years. This objective pathology before the act, followed by gratification and less-
is found despite evaluations; she also has a ening of tension with or without guilt, remorse,
chaotic lifestyle. This is most suggestive of som- or depression during the act. The stealing is not
atization disorder; the patient has a chronic con- planned and does not involve others but is the
dition without identifiable secondary gain. goal of the patient with kleptomania. Although
Dysmorphic disorder is unlikely, since the the thefts do not occur when immediate arrest is
patient did not report an imagined ugliness. probable, people with kleptomania do not
Conversion disorder is unlikely, because her always consider their chances of being appre-
condition is not self-limited. The absence of sec- hended, even though repeated arrests lead to
ondary gain makes the diagnosis of malingering pain and humiliation. These people may feel
unlikely. The absence of voluntary control of the guilt and anxiety after the theft, but they do not
symptoms makes the diagnosis of factitious dis- feel anger or vengeance. Furthermore, when the
order unlikely. (Stern, 139140) stolen object is itself the goal, the diagnosis is not
464 17: Psychiatry

kleptomania; in kleptomania only the act of 47. (C) Up to 72% of patients with panic disorder
stealing is the goal. (Kaplan, 763764) have a panic attack when administered an intra-
venous injection of sodium lactate. Therefore
44. (C) Trazodone is associated with the rare occur- lactate provocation is used to confirm a diagno-
rence of priapism, the symptom of prolonged sis of panic disorder. Hyperventilation, another
erection in the absence of sexual stimuli. That known trigger of panic attacks in predisposed
symptom appears to result from the alpha2-adren- persons, is not as sensitive as lactate provocation
ergic antagonism of trazodone. (Kaplan, 10981100) in inducing panic attacks. Carbon dioxide (CO2)
inhalation also precipitates panic attacks in those
45. (D) Dextroamphetamine is usually the second so predisposed. (Kaplan, 262)
line of pharmacological treatment when methy-
lphenidate is not effective. Pemoline has the 48. (A) Amitriptyline, imipramine, trimipramine,
advantage of a longer half-life and thereby and doxepin have the most anticholinergic side
allows less frequent dosing and round-the-clock effects of the tricyclic antidepressants. Amoxapine,
effects, but there have been some recent reports nortriptyline, and maprotiline are less anticholin-
of serious liver failure in patients being treated ergic; and desipramine may be the least anti-
with pemoline. (Kaplan, 1198) cholinergic. Anticholinergic effects include dry
mouth, constipation, blurred vision, and urinary
46. (B) The most likely diagnosis here is acute inter- retention. Narrow-angle glaucoma can also be
mittent porphyria. It is a disorder of heme biosyn- aggravated by anticholinergic drugs, and the pre-
thesis, which results in excessive accumulation of cipitation of glaucoma requires emergency treat-
porphyrin. It is characterized by the triad of symp- ment with a miotic agent. Severe anticholinergic
toms: acute abdominal pain, motor polyneuropa- effects can lead to a CNS anti-cholinergic syn-
thy, and psychosis. Acute intermittent porphyria drome with confusion and delirium, especially if
is an autosomal dominant disorder that affects tricyclic and tetracyclic drugs are administered
more women than men; its onset is between ages with antipsychotics or anticholinergic drugs. The
20 and 50. The psychiatric symptoms include anx- most common autonomic effect of tricyclic anti-
iety, insomnia, lability of mood, depression, and depressant medications, partly because of alpha1-
psychosis. Some studies have found that between adrenergic blockade, is orthostatic hypotension,
0.2% and 0.5% of chronic psychiatric patients may which can result in falls and injuries in affected
have undiagnosed porphyria. Barbiturates pre- patients. Nortriptyline may be the drug least likely
cipitate or aggravate the attacks of acute por- to cause the problem, and some patients respond
phyria, and the use of barbiturates for any reason to fludrocortisone (Florinef), 0.05 mg twice a day.
is absolutely contraindicated in a person with Other possible autonomic effects are profuse
acute intermittent porphyria and in anyone who sweating, palpitations, and increased blood pres-
has a relative with the disease. Niacin deficiency sure. (Kaplan, 1104)
is unlikely to be the diagnosis. The neuropsychi-
atric symptoms of pellagra include apathy, irri- 49. (D) The early signs and symptoms of lithium
tability, insomnia, depression, and delirium; the toxicity include coarse tremor, dysarthria, and
medical symptoms include dermatitis, peripheral ataxia; the later signs and symptoms include
neuropathy, and diarrhea. impaired consciousness, muscular fascicula-
Thiamine deficiency is unlikely, because tions, myoclonus, seizures, and coma. The
there is no history of ethanol abuse and no signs higher the lithium levels (and the longer they
of or psychiatric symptoms such as apathy, have been elevated), the worse the symptoms of
depression, irritability, nervousness, and poor lithium toxicity. (Kaplan and Sadock, 1050)
concentration. The presence of a clear senso-
rium and the associated clinical and psychiatric 50. (C) Diarrhea, vomiting, drowsiness, muscular
symptoms argue against the diagnoses of weakness and lack of coordination may be early
hepatic encephalopathy and cobalamin defi- signs of lithium intoxication, and can occur at
ciency, respectively. (Kaplan, 362363) lithium levels below 2.0 mEq/L. At higher levels,
Answers: 4456 465

ataxia, giddiness, tinnitus, blurred vision and a the third month. Lithium use has been associ-
large output of dilute urine may be seen. Serum ated with the formation of euthyroid goiter,
lithium levels above 3.0 mEq/L may produce a hypothyroidism accompanied by lower T3 and
complex clinical picture, involving multiple T4. Imipramine use may be associated with car-
organs and organ systems. Serum lithium levels diovascular side effects such as arrhythmias,
should not be permitted to exceed 2.0 mEq/L heart block, ECG changes, orthostatic hypoten-
during the acute treatment phase. (Coyle, 125127) sion, hypertension, tachycardia, palpitation, and
myocardial infarction. (Coyle, 38, 126; Kaplan,
51. (E) Concurrent use of calcium channel block- 566569, 953)
ing agents with lithium may increase the risk of
neurotoxicity in the form of ataxia, tremors, nau- 53. (D) The maternal lithium level must be moni-
sea, vomiting, diarrhea, and tinnitus. Caution tored closely during pregnancy and especially
should be used when lithium and diuretics are after delivery because of the significant change
used concomitantly because diuretic-induced in renal function with massive fluid shift that
sodium loss may reduce the renal clearance of occurs over that time period. Lithium should be
lithium and increase serum lithium levels with discontinued shortly before delivery, and the
risk of lithium toxicity. Patients receiving such drug should be restarted after an assessment of
combined therapy should have serum lithium the usually high risk of postpartum mood dis-
levels monitored closely and the lithium dosage order and the mothers desire to breast-feed her
adjusted if necessary. Lithium levels should be infant. (Kaplan, 1051)
closely monitored when patients initiate or dis-
continue the use of nonsteroidal anti-inflamma- 54. (A) Functional neuroimaging studies of people
tory drugs (NSAIDs). Concurrent use of metron- with ADHD have either been normal or have
idazole with lithium may provoke lithium shown decreased volume of the right prefrontal
toxicity due to reduced renal clearance. There is cortex and the right globus pallidus. In addition,
evidence that ACE inhibitors, such as enalapril whereas normally the right caudate nucleus is
and captopril, may substantially increase steady- larger than the left caudate nucleus, people with
state plasma lithium levels, sometimes result- ADHD may have caudate nuclei of equal size.
ing in lithium toxicity. When such combinations These findings suggest dysfunction of the right
are used, lithium dosage may need to be prefrontalstriatal pathway for control of atten-
decreased and plasma lithium levels should be tion. (Kaplan, 1194)
measured more frequently. The concomitant
administration of lithium with selective sero- 55. (B) Thioridazine is associated with irreversible
tonin reuptake inhibitors should be undertaken pigmentation of the retina when given in dosages
with caution, as this combination has been of more than 800 mg/day. An early symptom of
reported to result in symptoms such as diar- this effect can sometimes be nocturnal confu-
rhea, confusion, tremor, dizziness, and agita- sion related to difficulty with night vision. The
tion. The following drugs can lower serum pigmentation is similar to that seen in retinitis
lithium concentrations by increasing urinary pigmentosa; it can progress even after the thior-
lithium excretion: acetazolamide, urea, xanthine idazine is stopped and can finally result in
preparations, and alkalinizing agents such as blindness. In contrast, chlorpromazine is asso-
sodium bicarbonate. (Coyle, 127; Stern, 366367) ciated with benign pigmentation of the eyes.
Most patients who show the deposits are those
52. (D) Hepatotoxicity has been reported with the who have ingested 1 to 3 kg of chlorpromazine
use of pemoline. Agranulocytosis has been esti- throughout their lives. (Kaplan, 1030)
mated to occur in association with clozapine use
at a cumulative incidence at 1 year of approxi- 56. (A) Although alcohol consumed in the evening
mately 1.3% of patients. Its incidence rate based usually results in an increased ease of falling
upon a weekly monitoring schedule rose steeply asleep (decreased sleep latency), alcohol also has
during the first 2 months of therapy, peaking in adverse effects on sleep architecture. Specifically,
466 17: Psychiatry

alcohol use is associated with decreased REM or marked anxiety or depression, ideas of refer-
dream sleep, decreased deep sleep (stage 4), and ence, fear of losing ones mind, paranoid
increased sleep fragmentation, with more and ideation, and impaired judgment. Perceptual
longer episodes of awakening. (Kaplan, 396) changes may occur in a state of full wakeful-
ness and alertness: the patient may express a
57. (D) The CNS effects of acute alcohol intoxica- subjective intensification of perceptions,
tion depend on the blood level of alcohol. Signs depersonalization, derealization, illusions,
of intoxication include loss of inhibition, slurred and hallucination. Other physical signs
speech, staggering gait, euphoria, nystagmus, of LSD intoxication include pupillary dila-
motor incoordination, confusion, stupor, and tion, tachycardia, blurred vision, incoordina-
coma with high levels of alcohol in the blood. tion, and sweating. (Kaplan, 379383; Stern, 73,
(Kaplan, 379383; Stern, 73, 85, 89) 85, 89)

58. (E) Signs of cocaine intoxication include vaso- 63. (D) Caffeine overdose results in restlessness,
constriction, increased heart rate and blood nervousness, excitement, insomnia, flushed
pressure, chest pain, pupillary dilation, muscle face, diuresis, gastrointestinal disturbance, and
weakness, respiratory depression, euphoria, muscle twitching. (Kaplan, 379383; Stern, 73,
increased energy, anxiety, and increased risk of 85, 89)
psychosis. (Kaplan, 379383; Stern, 73, 85, 89)
64. (A) Signs of alcohol withdrawal are divided into
59. (A) Acute heroin intoxication results in an initial minor and major groups. Minor symptoms start
euphoria followed by apathy, dysphoria, psy- 8 to 9 hours after the last drink. The patient may
chomotor agitation, or retardation. Other signs have insomnia, sweating, hallucinations, and
include pupillary constriction, slurred speech, seizures. Major symptoms occur 48 to 96 hours
impaired attention or memory, and drowsiness after the last drink. The patient may have in-
or coma depending on the severity of the heroin creased psychomotor activity, tremor, hallucina-
overdose. (Kaplan, 379383; Stern, 73, 85, 89) tions, profound disorientation, and increased
autonomic activity. (Kaplan, 379383; Stern, 75, 85,
60. (E) Acute cannabis intoxication results in clini- 89)
cally significant maladaptive behavioral or psy-
chological changes such as impaired motor coor- 65. (B) Cocaine withdrawal involves a dysphoric
dination, euphoria, anxiety, sensation of slowed mood, fatigue, vivid and unpleasant dreams,
time, and impaired judgment. Other symptoms insomnia or hypersomnia, increased appetite,
of cannabis intoxication include conjunctival and agitation or psychomotor retardation.
injection, increased appetite, dry mouth, and (Kaplan, 379383; Stern, 75, 85, 89)
tachycardia. (Kaplan, 379383; Stern, 73, 85, 89)
66. (D) Heroin withdrawal is characterized by a
61. (A) Symptoms of phencyclidine intoxication dysphoric mood, nausea or vomiting, muscle
include clinically significant maladaptive behav- aches, lacrimation, pupillary dilation, fever diar-
ioral changes such as belligerence, assaultive- rhea, piloerection, and sweating. (Kaplan, 379383;
ness, impulsiveness, unpredictability, vertical or Stern, 75, 85, 89)
horizontal nystagmus, hypertension or tachy-
cardia, numbness or diminished responsiveness 67. (E) Signs of nicotine withdrawal include dys-
to pain, ataxia, dysarthria and muscle rigidity, phoric or depressed mood, insomnia, irritability,
hyperacusis. Seizure and coma may occur in case frustration, or anger, anxiety, difficulty concen-
of severe intoxication. (Kaplan, 379383; Stern, 73, trating, restlessness, decreased heart rate, and
85, 89) increased appetite or weight gain. These symp-
toms are associated with significant distress or
62. (B) LSD intoxication is characterized by impairment in social functioning. (Kaplan, 379383;
marked behavioral abnormalities such as Stern, 75, 85, 89)
Questions: 5771 467

68. (C) Abrupt cessation of caffeine use or reduc-


REFERENCES
tion in the amount of caffeine used causes
headache associated with marked fatigue and
American Psychiatric Association. Diagnostic and Statistical
anxiety, possible depression, and nausea or
Manual of Mental Disorders, 4th ed. Washington, DC:
vomiting, as well as clinically significant dis- American Psychiatric Association; 1994.
tress and impaired social functioning. (Kaplan, Behrman RE. Nelson Textbook of Pediatrics. 16th ed.
379383; Stern, 75, 85, 89) Philadelphia: Saunders; 2000.
Brown CS. Depression and anxiety disorders. Obstet
69. (D) Catatonia, mania, major depression, and acute Gynecol Clin North Am. 2001;28:241-268.
exacerbation of schizophrenia are well-established Comings DE. Clinical and molecular genetics of ADHD
indications for electroconvulsive therapy (ECT). and Tourette syndrome. Two related polygenic disor-
Other indications for ECT with less evidence of its ders. Ann NY Acad Sci. 2001;931:50-83.
effectiveness include Parkinson disease, obses- Coyle JT, Enna SJ, eds. Pharmacological Management
sivecompulsive disorder, neuroleptic malignant of Neurological and Psychiatric Disorders. New York:
syndrome, and intractable epilepsy. ECT is effec- McGraw-Hill; 1998.
tive only in the treatment of the acute symptoms Ewing JA. Detecting alcoholismthe CAGE questionnaire.
of schizophrenia, not those of chronic schizo- JAMA. 1984;252:1905-1907
Goldberg JF. New drugs in psychiatry. Emerg Med Clin
phrenia. (Kaplan, 11161118; Stern, 360)
North Am. 2000;18:211-231.
70. (E) Four clinical interview questions, the CAGE Hobson JA. Sleep and dreaming. J Neurosci. 1990;10:371-
382.
questions, have proved useful in helping to make
Hobson JA. The sleep-dream cycle: a neurobiological
a diagnosis of alcoholism. The questions focus on
rhythm. Pathobiol Annu. 1975:5:369-403.
Cutting Down, Annoyance by Criticism, Guilty Kaplan BJ, Sadocks VA. Synopsis of Psychiatry. 8th ed.
Feeling, and Eye-Openers. The acronym CAGE Baltimore: Lippincott Williams & Wilkins; 1997.
helps the physician recall the questions, which Kranzler HR, Amin H, Modesto-Lowe V, Oncken C. Pharma-
are as follows: cologic treatments for drug and alcohol dependence.
Psychiatr Clin North Am. 1999;22:401-423.
C: Have you ever felt you should cut down
Longhurst JG. Neuroleptic malignant syndrome. J Psychiatry.
on your drinking?
1995;166:537-538.
A Have people annoyed you by criticizing Margolis RL, McInnis MG, Rosenblatt A, Ross CA.
your drinking? Trinucleotide repeat expansion and neuropsychiatric
G Have you ever felt bad or guilty about disease. Arch Gen Psychiatry. 1999:56:1019-1031.
your drinking? Mann JJ. A current perspective of suicide and attempted
E Have you ever had a drink first thing in the suicide. Ann Intern Med. 2002;136:302-311.
morning to steady your nerves or to get rid Mayfield D, McLeod G, Hall P. The CAGE questionnaire:
of a hangover? validation of a new alcoholism screening instrument.
Am J Psychiatry. 1974;131:1121-1123.
(Ewing, 19051907) Newport DJ, Nemeroff CB. Neurobiology of posttraumatic
stress disorder. Curr Opin Neurobiol. 2000;10:211-218.
71. (B) Among the selective serotonin reuptake Oquendo MA. The biology of impulsivity and suicidality.
inhibitors (SSRIs), fluvoxamine appears to pres- Psychiatr Clin North Am. 2000;23:11-25.
ent the greatest risk of drugdrug interactions. Roze E, Saudou F, Caboche J. Pathophysiology of
Fluvoxamine is metabolized by CYP 3A4. Huntingtons disease: from huntingtin functions to poten-
Fluvoxamine may increase the half-lives of tial treatments. Curr Opin Neurol. 2008;21:497-503.
alprazolam and diazepam and should not be Stern TA, Herman JB. Psychiatry Update and Board
coadministered with these agents. Fluvoxamine Preparation. New York: McGraw-Hill; 2000.
may increase theophylline concentrations three- Task Force on the Use of Laboratory Tests in Psychiatry.
fold and warfarin concentrations twofold, with Tricyclic antidepressantsblood level measurements
and clinical outcome: an APA Task Force report. Am J
important clinical consequences. Fluvoxamine
Psychiatry. 1985;142:155-162.
raises concentrations and may increase the activ-
Warner EA. Cocaine abuse. Ann Intern Med. 1993;119:226-
ity of clozapine, carbamazepine, methadone, 235.
propranolol, and diltiazem. (Kaplan, 1090)
468 17: Psychiatry

Weeks RA, Turjanski N, Brooks DJ. Tourettes syndrome: a Withers NW, Pulvirenti L, Koob GF, Gillin JC. Cocaine
disorder of cingulate and orbitofrontal function? QJM. abuse and dependence. J Clin Psychopharmacol. 1995;
1996;89:401-408. 15:63-78.
Weiss F. Porrino LJ. Behavioral neurobiology of alcohol Zun LS. Panic disorder: diagnosis and treatment in emer-
addiction: recent advances and challenges. J Neurosci. gency medicine. Ann Emerg Med. 1997;30:92-96.
2002;22:3332-3337.
Index

Page numbers followed by italic f indicate figures.

A current therapies for, 253 Atelencephaly, 115


AAN. See American Academy of Neurology deficits in recent memory, 257 Athetosis, 132
(AAN) disease process in, 257 Attention deficithyperactivity disorder (ADHD)
AASM. See American Academy of Sleep Medicine features, 193 characteristic, 457
(AASM) mild cognitive impairment (MCI), 253255 functional MRI imaging, 122
Abdominal migraine, 135 neural plaque, 425 neuroimaging studies, 465
Abducens nerve, 19f, 25, 42, 292 nucleus basalis of Meynert, 259260 structural MRI studies, 122
Abducens nucleus, 25, 82 pathological characteristics of, 259 Auditory pathway, 22
Abnormal docility, 69 symptoms, 255 Auditory thalamus, 39
Abnormal gene, 146 Alzheimer type II cells, 421 Autism spectrum disorder (ASD), 125
Absence epilepsy, 124 Amaurosis fugax, 274 Autoimmune encephalitis, 314
Absence seizures, 186, 188 American Academy of Neurology (AAN), 121 Autonomic impairment, DLB, 254
Acanthocytes, 147 American Academy of Sleep Medicine (AASM), Autoreactive T cells, 309310
Acetylcholine receptor (AChR), 240 124, 181 Autosomal dominant disorders, 354355
AChR. See Acetylcholine receptor (AChR) Amino acids, 186 Autosomal dominant nocturnal frontal lobe
Acquired immunodeficiency syndrome (AIDS), 215 tyrosine, 339 epilepsy (ADNFLE), 124
associated vacuolar myelopathy, 291 Ammonia intoxication, 117, 118 Autosomal recessive disorders, 354
CMV encephalitis in, 423 Amnestic mild cognitive impairment (aMCI), 255 Autosomal recessive inheritance, 147
retinopathy, 292 AMP. See Adenosine monophosphate (AMP) AVMs. See Arteriovenous malformations (AVMs)
Acute alcoholic hallucinosis, 458 Amphetamines, 123, 335 Axillary nerve, 23, 221
Acute axonal damage, 224 Amyloid deposition stains, 402f, 417 Axonal transport, 193194
Acute Chagas disease, 296 Amyloid precursor protein (APP), 252, 253, 311, Axon hillock, 196
Acute heroin intoxication, 466 337 Axons, 26, 31
Acute hypothalamic lesion, 91 Amyotrophic lateral sclerosis (ALS), 218, 348
Acute inflammatory demyelinating Anaplastic astrocytomas, 404f, 418 419. See also B
polyneuropathy (AIDP), 215, 318 WHO grade III astrocytoma Babinski sign, 85
Acute intermittent porphyria, 464 Anderson syndrome, 227 Bacterial meningitis, 294295
Acute ischemic stroke, 271272 Angelman syndromes, 125 Balint syndrome, 69, 93, 366
Acute motor axonal neuropathy, 220 Angiocentric neoplasms, 407f, 420421 Basal plates, 36
Acute pancreatitis, 183 Anoxic encephalopathy, 179 Basilar artery stenosis, 273
Acute vestibular lesions, 28 Anterior cerebral arteries (ACAs), 442 Basilar-type migraine, 121
AD. See Alzheimer disease (AD) Anterior choroidal artery, 33 Basket stellate cells, 40
Adductor magnus, 75 Anterior ischemic optic neuropathy (AION), 370 B-cell lymphoma, 423
Adenosine triphosphate, 195 Anthrax meningitis, 296 BDNF. See Brain-derived neurotrophic factor (BDNF)
ADNFLE. See Autosomal dominant nocturnal Antiepilepsy medication, side effects, 334 Becker muscular dystrophy (BMD), 222
frontal lobe epilepsy (ADNFLE) Antiepileptic drugs (AEDs), 356 Becker myotonia congenita, 227
Adrenoleukodystrophy, 147, 190 Antigen-presenting cells (APCs), 309310 Benedikt syndrome, 72, 90
Adsons test, 96 Antiglial nuclear antibody (AGNA), 319 Benign occipital epilepsy, 189
Adenosine monophosphate (AMP), 223 Anti-Hu antibodies, 317 Benign rolandic epilepsy, 116, 189
Age-dependent epilepsy, 184 Anti-Huassociated syndrome, 317 Benzodiazepines, 180, 186, 188
Agranulocytosis, 337 Antineuronal antibodies, 148 Beta rhythm, 176, 177
Aicardi syndrome, 127, 128 Anti-N-methyl-D-aspartate (NMDA), 319 Biceps, 24
AIDP. See Acute inflammatory demyelinating Aplastic anemia, 183 Bilateral strabismus, 190
polyneuropathy (AIDP) Apneic spells, 122 Bilirubin, 117, 128, 133
AIDS. See Acquired immunodeficiency syndrome Apneustic respiration, 70 Binswanger syndrome, 254
(AIDS) APP. See Amyloid precursor protein (APP) Bjerrum arcuate scotoma, 81
Akinetic mutism, 70 Arachnoid cysts, 442443 Bloodbrain barrier (BBB), 17f, 41, 309
Alcoholism, 457, 467 ARCAs. See Autosomal recessive cerebellar ataxias Blood plasma concentrations, 459
sleep architecture, 465466 (ARCAs) BM. See Basilar-type migraine
withdrawal sign, 466 ArgyllRobertson sign, 83 BMD. See Becker muscular dystrophy (BMD)
Alexander disease, 351352 Arteriosclerotic carotid artery, 77 Botulinum toxin, 237
Alien hand syndromes, 9394 Arteriovenous malformations (AVMs), 350 Botulism, 293, 296297
Alien limb sign, 69, 391 ASD. See Autism spectrum disorder (ASD) Brachial plexus, 24, 96, 131
Alien limb syndrome, 391 Asperger syndrome, 463 Brachioradialis muscle, 79
Alpers disease, 233234 Aspirin, 335 Bradyphrenia, 392
Alpha fetoprotein, 382 Astrocytes, 194 Brain-derived neurotrophic factor (BDNF), 398
Alzheimer disease (AD), 255, 258, 336337, 391 Ataxia Breach rhythm, 183
aging, 252 Friedreich, 353 Bromocriptine, 395
amnestic-type of memory, 253 telangiectasia, 147 Bromodeoxyuridine, 339
amyloid precursor protein (APP) gene, 252253 Ataxic breathing, 70 Buspirone, 337

469
470 Index

C Chronic motor syndrome, 85 Diabetes insipidus, 381


CABG. See Coronary artery bypass grafting Chronic plaques, 309 Diabetic cranial mononeuropathies, 225226
(CABG) Chronic polyneuropathy, 146 Dialysis dementia, 180
CADASIL, 355356 Chronic progressive external ophthalmoplegia Diazepam, 333
CAE. See Childhood absence epilepsy (CAE) (CPEO), 233, 355 Diencephalon, 36, 71
Caffeine CIDP. See Chronic inflammatory demyelinating Diffusion-weighted imaging, 278
abrupt cessation of, 467 polyneuropathy (CIDP) DiGeorge syndrome, 128
overdose, 466 Cigarette smoking, 279 Diphtheria
CAGE test, 458 Ciliospinal reflex, 71 neurological complications, 225
Calcium channel blocking agents, 465 Circumferential arteries, 34 neuropathy, 239
Calcium channel gene (CACNA1A), 120 Clarkes nucleus, 4142 Distal arm weakness, 227
Callosal alien hand syndrome, 94 Claude syndrome, 72 Distal sensory polyneuropathy (DSP), 215
Canavan disease, 191, 352 Climbing fibers, 31, 39 Distal symmetric polyneuropathy, 291
Carbamazepine, 116, 183, 184, 185, 192, 193, 334335 Clostridium botulinum, 237 Divalproex sodium, 459
Carbamazepine intoxication, 184 Clostridium tetani, 294 DLB. See Dementia with Lewy bodies (DLB)
Carbamyl phosphate synthetase (CPS), 118 Clozapine, 337, 461 Dopamine receptors, 395
Carbon monoxide intoxication, 397 CMS. See Congenital myasthenic syndromes (CMS) Dopamine reuptake transporter, 334
Carcinoma, 423 CMV meningoencephalitis, 294295 Dorellos canal, 82
Cardiac arrhythmia, 462463 CMV myelopathy, 290 Dorsal midbrain syndrome. See Moritz-Benedikt
Cardiac conduction defects, 235 CNS anti-cholinergic syndrome, 464 syndrome
Cardiac disease, 132 CNS dysfunction, 308 Dorsal motor nucleus, 26
Cardiomyopathy, 220 Cocaine, 458 Dorsal scapular nerve, 24
Carnitine palmityl transferase deficiency, 147 intoxication, 466 Dorsal spinocerebellar tract, 23
Carpal tunnel syndrome, 25, 74 Cognitive deficits, 355 Dorsal vermis, 399
defined, 80 COL4A1 gene, 356 Dorsolateral midbrain syndrome, 277
feature, 80 Compound muscle action potential (CMAP), 196, Dorsolateral parvicellular division, 3637
Catecholamines, 335, 339 216, 218 Dorsomedial nucleus, 37
Catechol-O-methyl transferase (COMT) inhibitors, Congenital myasthenic syndromes (CMS), 238, 349, Downs syndrome, 126, 258
395 350 Dravet syndrome, 126
Caudal pontine syndrome, 48f, 71 Congenital myotonic dystrophy, 225, 232 DRESS. See Drug-related rash with eosinophilia
CBD. See Corticobasal degeneration (CBD) Contralateral brainstem, 3334 and systemic symptoms (DRESS)
Cocaine test, 371 Contralateral hemiplegia, 90 Drug ethambutol, 77
CD8 T lymphocytes, 310 Coracobrachialis muscle, 2324 Drug-related rash with eosinophilia and systemic
Cellular monotony, 424 Cord damage, 85 symptoms (DRESS), 356
Centers classification, 238239 Coronary artery bypass grafting (CABG), 274 DSP. See Distal sensory polyneuropathy
Central apnea, 119 Corpus callosum, 69, 426 DSS. See Dejerine-Sottas syndrome (DSS)
Central core myopathy, 418 Cortical dystrophin, 144 Duchenne and Becker muscular dystrophies, 144
Central herniation, 71 Corticobasal degeneration (CBD), 250251, 394 Duchenne muscular dystrophy, 144
Central nervous lymphoma, 292 Corticospinal tract lesions, 28 Duchenne-type muscular dystrophy, 232
Central neurocytoma, 379, 406f, 420 Corticotropin-releasing hormone, 338 DVT. See Deep venous thrombosis (DVT)
Central retinal artery, 364f, 370 CPEO. See Chronic progressive external Dyschromatopsia, 77
Central scotoma, 81 ophthalmoplegia (CPEO) Dyslexia, 119
Central spinal cord syndrome, 85 CPS. See Carbamyl phosphate synthetase (CPS) Dysmorphic disorder, 463
Centrocecal scotoma, 81 Cranial nerve nuclei, 276 Dystonia, 397
Cerebellar atrophy, 184 Creutzfeldt-Jakob disease, 258, 391, 422
Cerebellar cortex, 30, 39, 338, 339 Cri-du-chat syndrome, 138 E
Cerebellar hematomas, 280 CSF, 250, 318 EA4, 358. See also Familial periodic
Cerebellar nuclei, 30 abnormalities, 292 vestibulocerebellar ataxia
Cerebellar syndrome, 318 drain pathway, 136 Early infantile epileptic encephalopathy (EIEE), 127
Cerebellar vermis hypoplasia (CVH), 125 protein, 230 Echinocytes, 147
Cerebral abnormalities, 139 CVA. See Cerebrovascular accident (CVA) EdingerWestphal nuclei, 27, 39
Cerebral atrophy, 290 CVH. See Cerebellar vermis hypoplasia (CVH) EDS. See Excessive daytime sleepiness (EDS)
Cerebral cortex, 3233, 397 CVT. See Cerebral venous thrombosis (CVT) EEG discontinuity, 178
Cerebral hemisphere, 16f, 40 Cyanide intoxication, 397 EEG montage, 121122
Cerebral hemorrhagic metastasis, 443 Cyanide radicals, 397 EhlersDanlos syndrome, 275
Cerebral metabolic rate for oxygen (CMRO2), 280 Cyclooxygenase enzymes 1, 257 EIEE. See Early infantile epileptic encephalopathy
Cerebral palsy, 116, 133 Cytokeratin immunohistochemical stains, 423 (EIEE)
Cerebral venous thrombosis (CVT), 127, 444 Cytomegalovirus, 423 Electrocerebral inactivity, 180
Cerebrospinal fluid (CSF), 17f, 41 Electrocerebral silence, 180
Cerebrotendinous xanthomatosis, 351 D Electrophysiology, 143
Cervical bruit, 273 DandyWalker malformation (DWM), 118 markers, 223
Cervical plexus, 95 Dantrolene, 313 Emboliform nucleus, 3031
CharcotMarieTooth (CMT) disease, 145, 218, 226 Dawsons fingers, 309 EME. See Encephalopathy (EME)
CharcotMarieTooth neuropathy, 424 Deep peroneal nerve, 28 EmeryDreifuss muscular dystrophy type 1, 145146
type 1 (CMT1), 219220 Deep tendon reflexes, 297 Encephalitis, rabies, 422
type 2 (CMT2), 226 Deep venous thrombosis (DVT), 271 Encephalopathy, 127
type 3 (CMT3), 226 Deficiency optic neuropathy, 367 Endodermal sinus tumor stains, 423
type 4 (CMT4), 226227 DejerineKlumpke palsy, 97 Endothelium properties, 279280
Charles Bonnet syndrome, 93 DejerineSottas syndrome (DSS), 145, 349 Endplate noise, 223
ChediakHigashi syndrome, 141 Dejerine syndrome, 275276 Endplate spikes, 223
CheyneStokes breathing, 140 Dementia, 393 Enteroviruses, 295, 299
Childhood absence epilepsy (CAE), 124 Dementia with Lewy bodies (DLB), 256, 260261 Environmental dependency syndrome, 69, 94
Choline acetyltransferase (ChAT), 332 cognitive symptom, 254 Eosinophilic cytoplasmic neuronal inclusions, 425
Cholinergic receptors, 331 consensus criteria for antemortem, 261 Ependymomas, 377, 419420
Cholinergic synapse, 331 Demyelination, 202f, 218, 424 Epidemic encephalitis lethargica, 67
Choreoathetotic movements, 132 Depressed gag reflex, 34 Epilepsy, 187, 192
Choroid plexus, 17f, 41, 87-88, 377 Depression, 92, 258, 392 Epileptic encephalopathy, 127
Chronic inflammatory demyelinating Dermatomyositis, 240, 426 Epileptogenic right temporal lesion, 92
polyneuropathy (CIDP), 215, 229 Dextroamphetamine, 464 Episodic apnea, 349
Index 471

Episodic ataxia, 358 GBS. See Guillain-Barr syndrome (GBS) Horner syndrome, 6768, 82, 91, 131, 276
Epithelial cysts stain, 423 GDHC. See A-ketoglutarate dehydrogenase Human gliomas, 423424
EPS. See Extrapyramidal syndrome (EPS) complex (KGDHC) Huntington disease (HD), 353, 398, 461
ErbDuchenne palsy, 97, 130 Geniculocalcarine fibers, 2223 Hutchinson triad, 138
Eriksons formulations, 456457 Geniculotemporal fibers, 34 Hyaline body myopathy, 406f, 417418
Ethosuximide (Zarontin), 182 Genitourinary symptoms, 218 Hydroxyamphetamine, 371
Euphoria, 459 Germinomas, 136 Hyperammonemia, 118, 183, 185
Excessive daytime sleepiness (EDS), 123 Gerstmann syndrome, 69 Hypermetamorphosis, 92
Extensor carpi radialis brevis, 79 Glatiramer acetate, 309 Hypertension, 279
Extensor carpi radialis longus, 79 GLD. See Globoid cell leukodystrophy (GLD) Hyperventilation, 117118, 119, 186, 188, 280
Extensor indicis proprius, 24 Glioblastoma. See Glioblastoma multiforme Hypervitaminosis A, 133134
Extensor pollicis longus, 79 Glioblastoma multiforme, 379, 380, 419, 442 Hypothalamus, 36, 42, 371
External ophthalmoplegia, 233 Globoid cell leukodystrophy (GLD), 190191, 352. subserves, 338
Extraocular ophthalmoplegia, 233 See also Krabbe disease Hypothenar muscles, 7374
Extrapyramidal syndrome (EPS), 461 Globus pallidus, 17f, 41, 122, 394, 397 Hypothermia, 136
Globus pallidus externa (GPe), 17f, 41 Hypotonia, 137
F Globus pallidus interna (GPi), 17f, 41 Hypoxia/ischemia, 121
Facial nerve, 26, 8889 Glomus jugulare tumors, 34 Hypsarrhythmia, 122, 128, 193
Facioscapulohumeral muscular dystrophy (FSHD), Glossopharyngeal nerves, 26, 34, 37
223, 231 Glucosylceramidase, 140 I
Factitious disorder, 460 Glucosylceramide, 141 Ideomotor apraxia, 263
Familial amyloid polyneuropathy, 348349 Glutamate-driven excitotoxic mechanisms, 311 Idiopathic generalized epilepsy, 189
Familial hemiplegic migraine (FHM), 120, 134135, Glutamate receptor gene (GRK1), 124 IgM serum antibodies, 220
359 Glutamic acid decarboxylase (GAD), 316, 399 Imipramine, 464
Familial periodic vestibulocerebellar ataxia, 358 Gluteus medius, 28 Immune system, 310
Fascioscapulohumeral syndrome, 132 Glycine encephalopathy, 118, 128 IMP. See Inosine monophosphate (IMP)
Fatal insomnia, 297 Glycogenosis type II, 217 Inclusion body myositis (IBM), 319
Febrile seizures, 132, 135 Glycogen storage disease type II, 144145 Infantile spasms, 128, 189
Felbamate, 185 Golgi cells, 31 Inferior cerebellar peduncle, 21f, 43, 91
Femoral nerve, 7677 Gomori trichome stain, 427 Inferior colliculus level, 5f, 27
FHM. See Familial hemiplegic migraine (FHM) Gottron papules, 221 Inferior olivary nuclear complex, 42
Fibrillation potentials, 223 Gradenigo syndrome, 82 Inferior quadrantic hemianopia, 1f, 2223
Flame-shaped carotid occlusion, 272 Gradient-echo images, 444 Inferior salivary nucleus, 41
Flexor carpi radialis, 24, 80 Graves disease, 367 Inflammatory demyelinating
Flexor carpi ulnaris, 73, 81 Guillain-Barr syndrome (GBS), 28, 129130, polyradiculoneuropathy (CIDP), 229230
Flexor digitorum profundus, 80 143144, 239 Influenza pandemic, 215216
Flexor digitorum superficialis, 80 Gustatory fibers, 41, 88 Inosine monophosphate (IMP), 223
Flumazenil, 333 Guyons canal, 25 Insulin-mediated inhibition, 236
Fluvoxamine, 467 Gyrus, 40 Intermittent photic stimulation, 178
Focal brain ischemia, 134 Internal auditory canal, 29
Focal seizures, 9293 H Internal carotid arteries, 40
Food and Drug Administration (FDA), 123 Haemophilus influenzae, 134, 294 Internal globus pallidus, 29, 30
Foramina of Monro, 16f, 40 HallervordenSpatz syndrome (HSS), 147, 397 Internal pyramidal layer, 32
Foville syndrome, 82, 89, 276 definitive diagnosis of, 397398 Interosseous muscles, 81
Fragile-X syndrome (FRAX), 125, 136137, 352, 461 Haloperidol, 457, 461 Intracerebral hemorrhage, 280
Fragile-X tremor ataxia syndrome (FXTAS), Hamartin, 368 Intracranial aneurysms, 274
357358 Harp syndrome, 147 Intracranial pressure (ICP) forces blood, 274
Friedreich ataxia, 353 Hartnup disease, 147 Ipsilateral corticospinal tract, 86
Frontotemporal dementia (FTD), 250, 251, 256, 258, Hematoma, 280 Ipsilateral Horner syndrome, 70
348, 352353 Hemiballismus, 69, 91 Ipsilateral inferotemporal fibers, 82
Frontotemporal lobar degeneration (FTLD), 250 Hemicranial headaches, 192 Ischemic optic neuropathy, 369
Fructose-1,6-diphosphatase (FDPase) deficiency, Hemorrhagic stroke, 279 Ischemic penumbra, 271
117118 Heparin prophylaxis, 271
FSHD. See Facioscapulohumeral muscular Hepatotoxicity, 188, 465 J
dystrophy (FSHD) Hereditary hemorrhagic telangiectasia (HHT), 350 Jacobsen syndrome, 352
FTD. See Frontotemporal dementia (FTD) Hereditary neuropathy, 219 Janz syndrome, 130
FTD syndrome, 251 Hereditary spastic paraplegias (HSPs), 218219 Japanese encephalitis (JE) virus, 297298
Full-term neonate, 178 Herpes encephalitis, 135, 298, 421 JC virus, 422
F wave response, 174f, 196 Heteroplasmy, 354 JME. See Juvenile myoclonic epilepsy (JME)
Heterotopia, 37 Joubert syndrome, 119, 125
G Heterozygote, 354355 Joubert syndrome and related cerebellar disorders
GABA. See Gamma-aminobutyric acid (GABA) Hexosaminidase, 138 (JSRD), 125
systems Hind limb perception, 2, 23 JSRD. See Joubert syndrome and related cerebellar
GABA B receptors, 333 Hip flexion, weakness, 97 disorders (JSRD)
GABAergic transmission, 333 Hippocampal pathways, 262 Jugular foramen, 37
Gabapentin, 185, 336 HIV Juvenile myoclonic epilepsy (JME), 125, 189
GABA receptor, 394 -associated distal sensory polyneuropathy, 299
GAD, 318. See Glutamic acid decarboxylase (GAD) -associated neuropathies, 215 K
Galactocerebroside, 191 cryptococcal meningitis, 292 KayserFleischer ring, 192
Galactosemia, 142 dementia, 260, 290 K complexes, 177
Galactosylceramide beta-galactosidase, 191 differential diagnosis, 290291 Kearns-Sayre syndrome (KSS), 146, 227, 234, 235,
Gamma-aminobutyric acid (GABA) systems, 123, encephalitis, 422 348, 355
332333, 338 myelitis, 291 Kennedy disease, 219
Gap junction channels, 194 neuropsychological testing, 290291 Keratitis, 297
Gasserian ganglion, 26 other causes, 291 Kinaseencoding gene (DMK), 225
Gaucher cells, 141, 190 peripheral neuropathy, 215 KluverBucy syndrome, 69, 94
Gaucher disease, 140141, 142143, 190 severity of, 290 Krabbe disease, 139, 142, 190191, 352
Gaze-evoked nystagmus, 399 toxoplasmosis encephalitis, 291292 KRS. See Kufor-Rakeb syndrome (KRS)
Gaze-holding neural integrator, 25 HolmesAdie tonic syndrome, 67, 371 KuforRakeb syndrome (KRS), 357
472 Index

L Menkes disease, 141, 191192 Myelin sheaths, 34


Lacrimal nerves, 89 Menkes syndrome, 146147 Myelomeningocele, 133
Lacrimation, 89 Meralgia paresthetica, 76 Myeloschisis, 36
LaCrosse virus, 135 Merkel cells, 194 Myoadenylate deaminase deficiency, 146
Lactic acidosis, 234 MERRF syndrome (myoclonic epilepsy and Myoclonic epilepsies, 189
Lacunar strokes, 278 ragged-red fibers), 354 Myofibrillar adenosine triphosphatase (ATPase),
Lafora disease, 129, 189, 190 Mesolimbic dopamine neurons, 336 407f, 417
Lambda waves, 181 Mesoprefrontal dopamine system, 337 Myokymia, 224, 358
LambertEaton myasthenic syndrome (LEMS), Metacarpophalangeal joint, 79 Myopathies, 224
217218, 236237, 238, 315, 316, 319 Metachromatic leukodystrophy, 139, 190, 349 Myosin storage, 417418
LandauKleffner syndrome, 129 Methadone, 334 Myotonia, 133, 225
Laryngeal stridor, 190 Methanol, 397 congenita, 222, 227
Lateral cord, 96 Methylphenidate, 123 dystrophy, 144, 353
Lateral geniculate body, 37 Meyers loop, 82 myotonic discharges, 217
Lateral medullary syndrome, 70 Microadenomas, 444 Myotonic dystrophy (DM), 144
Lateral occipital cortex, 69 Microcalcifications, 406f, 420 Myotonic muscle disorders, 232
Leber hereditary optic neuropathy, 369 Microglial cells, 194 Myxopapillary ependymomas, 377
Left posterior femoral cutaneous, 76 Microglial nodules, 422
Legato cells, 30 Microscopic hallmark of schwannomas, 378 N
Leigh disease, 140 Midbrain, 276277 N-acetyl aspartate (NAA), 255, 281
Leighs syndrome, 233, 235 Middle cerebral artery (MCA), 273 as nural marker, 312
LennoxGastaut syndrome (LGS), 126, 128 Migraine, 120, 370 NADH-TR staining, 417
Leprosy, 293 headache, 121 Naltrexone, 458
LeschNyhan disease, 350 without aura, 120 Narcolepsy, 123, 128, 193, 463
Lewy bodies, 425 Mild cognitive impairment (MCI), 253, 393 treatment of, 123
LGS. See Lennox-Gastaut syndrome (LGS) MillardGubler syndrome, 48f, 71, 82, 83, 89, 276 Narrow-angle glaucoma, 464
Limb girdle muscular dystrophy 2A, 348 MillerFisher syndrome, 239 Natalizumab (Tysabri), 321
Limbic or brainstem encephalitis, 316 Mitochondrial disorder, 140, 354 Neisseria meningitidis, 294
Limbic encephalitis, 321 Mitochondrial encephalomyopathy, 234 Nemaline rod myopathy, 227, 426
Lincomycin, 238 Mitoxantrone, 320321 Neonatal adrenoleukodystrophy, 117
Lipohyalinosis, 423 Mitral stenosis, medical stenosis, 68 Neonatal hypoglycemia, 118
Lisch nodules of the iris, 122123 Miyoshi myopathy, 232 Neoplastic oligodendrocyte, 403f, 419
Lissencephaly, 444 MMN. See Multifocal motor neuropathy (MMN) Nerve fascicles, 5f, 27
Listeria monocytogenes, 294 MMPSI. See Malignant migrating partial seizure in Nerve temperature reduction, 196
Lithium intoxication, 464465 infancy (MMPSI) Nervus intermedius, 29, 33
Lithium toxicity, 464 Molar tooth sign (MTS), 125 NET. See Norepinephrine transporter (NET)
Locus ceruleus, 27, 30 Monoradiculopathy, 73 Neural integrator, 25
Long-term memory, 261262 MoritzBenedikt syndrome, 277 Neurapraxia, 217
Loss of heterozygosity (LOH), 423424 Mossy fibers rosettes, 39 Neuroacanthocytosis, 148
Lowe syndrome, 144 Motor circuitry, 393 Neuroblasts, 36
LSD intoxication, 466 Motor median nerve conduction study, 175f, 197 Neurocutaneous disorder, 141
Lumbar plexus, 74 Motor neuron disease (MND), 250 Neurocysticercosis infection, 295296
Lyme disease, 226 Motor neuron syndrome, 321 Neurofibrillary tangles, 421, 425
Lysosomal enzyme arylsulfatase A (ASA), 352 Moyamoya syndrome, 351 Neurofibroma, 408f, 421
MPZ mutations, 216 Neurofibromatosis (NF), 351
M MRI-based anatomic studies, 122 type 1, 122123, 141, 368
Macrocephaly, 143 MR venography (MRV), 445 type II, 368
Magnetic resonance imaging (MRI), 115, 338 MSA. See Multiple system atrophy (MSA) Neurofibromin, 380
Magnetic resonance spectroscopy (MRS), 281 MS brains Neurofilament stains neurofibroma, 423
Malignant hyperthermia, 221222 factors associated with, 313 Neurogenic ulcers, 91
Malignant migrating partial seizure in infancy histopathologic studies, 311 Neuroleptic malignant syndrome (NMS), 460, 467
(MMPSI), 126 spasticity, 313 Neuromuscular and gastrointestinal symptoms, 233
Marcus Gunn pupil, 2728, 67 MS plaques, 309 Neuromuscular transmission, 239
Marfan syndrome, 275 Mucopolysaccharidoses, 143 Neuromyotonia, 315
MarieFoix syndrome, 90 Multifocal motor neuropathy (MMN), 220, Neuromyotonic discharges, 224
Masseter muscles, 133 228229, 230 Neuronal systems, 252
MBP. See Myelin basic protein (MBP) Multinucleated giant cells, 260, 422 Neuropathic toxicity of pyridoxine, 231
McArdle disease, 236 Multiple system atrophy (MSA), 193, 394, 396 Neuroprotective therapies, 395396
Meckels cave, 29 Multisystemic disorder, 233 Neurosyphilis, 67
Medial cord, 96 Mumps virus, 422 Neurotransmitter, 262263
Medial dorsal nucleus, 91 Muscle biopsy, 146 Neurulation, 35
Medial dorsal thalamus, 391392 Gomori-trichome stain, 401f, 403f, 417 NFTsneurofibrillary pathology, 256
Medial frontal lobe syndrome, 92 hematoxylin-and-eosin staining, 221 Nicotinic receptors, 331
Medial geniculate body, 37 histopathological abnormalities, 418 NiemannPick disease, 138, 189190
Medial geniculate nucleus, 39 in statin myopathy reveal muscle fiber necrosis, NiemannPick syndrome, 141
Medial hypothalamus, 40 418 Nitrosourea derivatives, 381
Medial lemniscus, 38 Muscle dystrophin, 144 NMDA receptor, 253, 457
Medial medullary syndrome, 47f, 7071, 275276 Muscle fiber denervation, 417 NMO. See Neuromyelitis optica (NMO)
Medial pontomedullary reticular formation, 392 Musculocutaneous dysfunction, 73 Nondeclarative memory, 262
Medial tegmental syndrome, 49f, 72 Musculocutaneous nerve, 23, 73 Nondominant temporal lobe, 94
Median nerve somatosensory evoked potentials, Myasthenia, 144, 367 Nonfluent aphasia (NFA), 250
182183 Myasthenia gravis (MG), 236, 315 Nonketotic hyperglycinemia, 192
Medulloblastomas, 381382, 420 Myasthenic syndrome, 236 Non-REM sleep, 129
MELAS syndrome (mitochondrial Myelin-associated glycoprotein (MAG), 34, 35, 314 Nonsteroidal anti-inflammatory drugs (NSAIDs),
encephalomyopathy, lactic acidosis, and Myelination, 3435 121, 463, 465
stroke), 354 Myelin basic protein (MBP), 34, 35, 314 Nontraumatic subarachnoid hemorrhage, 274
Mendelian transmission, 124 Myelinoligodendrocyte basic protein (MOBP), 35 Norepinephrine transporter (NET), 335
Meningiomas, 377, 378, 420, 423 Myelin-oligodendrocyte glycoprotein (MOG), 35 Normal adenohypophysis pituitary gland, 378
Meningomyelocele, 36 Myelin protein zero (MPZ) gene, 219 Normal frozen muscle stains, 426
Index 473

Normal-pressure hydrocephalus, 254255 PCNSL. See Primary central nervous system Progressive multifocal leukoencephalopathy
Nortriptyline, 464 lymphoma (PCNSL) (PML), 291, 422
NSAIDs. See Nonsteroidal anti-inflammatory drugs PDGF. See Platelet-derived growth factor (PDGF) Progressive myoclonic epilepsy (PME), 188189,
(NSAIDs) Pectectal midbrain lesions, 71 391
Nucleus ambiguus, 26, 275 Peduncular hallucinosis syndrome, 72 Progressive necrotizing encephalopathy, 233
Nucleus solitarius, 26 PelizaeusMerzbacher disease (PMD), 352 Progressive nonfluent aphasia, 260
PEM. See Paraneoplastic encephalomyelitis (PEM) Progressive supranuclear palsy (PSP), 250, 258259,
O Pendular Seesaw nystagmus, 369 370, 394, 396397
Occipital epileptic seizures, 93 Perceptive visual agnosia, 262 Proinflammatory cytokines, 314
Oculocerebrorenal syndrome. See Lowe syndrome Perihypoglossal nuclei, 26 Prolactinomas, 378
Oculomotor apraxia, 190 Periodic lateralizing epileptiform discharges Prosopagnosia, 262
Oculomotor nerve, 42, 50f, 72 (PLEDs), 186 Proteolipid protein (PLP), 34
compression, 71 Peripheral mechanoreceptors, 194 Protuberant eyes, 139
palsy, 83 Peripheral myelin protein (PMP22), 314 Psammoma bodies, 378
visceral motor component of, 39 Peripheral neuropathy, 230 PSN. See Paraneoplastic sensory neuronopathy
Ohtahara syndrome, 127 Peripheral vision, 77 (PSN)
Olanzapine, 462 Perirolandic cortex, 94 Psoas muscles weakness, 81
Olfactory bulb, 15f, 40 Periventricular leukomalacia (PVL), 121 Psychiatric symptoms, 464
Olfactory pathway, 22 Petrous bone fracture, 82 Psychiatric syndromes, 461
Oligodendrocyte-specific protein (OSP), 34 Phencyclidine intoxication, 466 Psychomotor retardation, 92
Oligodendroglioma, 419 Phenobarbital, 334 PTEN gene, 380
Olivocerebellar climbing fiber system, 31 Phentolamine, 335 p53 tumor-suppressor gene, 380
Onufs nucleus, 85 Phenytoin, 182, 183, 184, 334, 335 Pudendal nerve, 77
Oogenesis, 136137 levels, 186 Pupillary dilation, 71
Ophthalmic artery, 40 Phenytoin toxicity, 182 Pupillomotor fibers, 83, 367
Ophthalmoplegic migraine, 367 Phosphatidylinositide-3-kinase (PI3K), 380 Purkinje cells, 3031, 39
Ophthalmoscopy, 370 Photic stimulation, 188, 189 PVL. See Periventricular leukomalacia (PVL)
Optic neuritis, 369, 370 Pick cell, 251 Pyramidal cells, 30, 32
Optic neuropathy, 7778 Pick disease, 251 Pyramidal dysfunction, 399
Optic tract syndrome, 82 Pilocytic astrocytoma, 378380, 405f, 419 Pyridoxine dependency, 116, 117
Optokinetic system, 366 Pinch sign weakness, 74
Orbicularis oculi, 33 Pineal gland, 20f, 42 Q
Orbitofrontal cortex, 69, 92 Pinpoint pupils, 70 Quetiapine fumarate, 462
Organophosphorus compounds, 238 Piriformis muscle, 75 Quiet sleep, 179
Ornithine transcarbamylase (OTC), 118 Platelet-derived growth factor (PDGF), 339 Quinidine, 238
Oxidative enzyme stains, 418 Platelet fibrin emboli, 368
Oxidative phosphorylation system (OXPHOS), 234 PLEDs. See Periodic lateralizing epileptiform R
Oxotremorine, 332 discharges (PLEDs) Radial nerve arises, 24
Poliovirus, 294 Ramsay Hunt syndrome, 89, 147, 297
P Polyarteritis nodosa, 239 Rankin Scale score, 272
PA administration, 272 Polyglucosans, 129 Rasmussen syndrome, 129
Pallidopontonigral degeneration, 251 Polymyalgia rheumatica, 231 Rathkes cleft, 426
Palmar interosseous muscles, 81 Polyneuropathy, 396 RaymondCestan syndrome, 83
Palmaris brevis muscle, 81 Pompe disease, 144145 Recurrent artery of Heubner, 40
Palpable purpura, 215 Pontine hemorrhage, 70 Reflex seizures, 187
Pancoast syndrome, 97 Pontine myelinolysis (CPM), 424 Refsum disease, 220
Panfascicular atrophy, 426 Pontine raphe nuclei, 30 REM behavior disorder (RBD)
Panic disorder, 456, 464 Pontine reticular nucleus, 26 characteristic, 192193
Papez circuit, 37 Pontine stroke, 443, 444 REM parasomnias, 186
Papillomacular bundle, 81 Pontine tegmentum, 89 REM sleep, 123, 128
Paramedian vessels, 34 Porphyria, neuropathy in, 226 Remyelination, 202f, 218, 424
Paraneoplastic antigens, 314 Positron emission tomography (PET), 257258 Restless leg syndrome, 460
Paraneoplastic cerebellar degeneration (PCD), 314, Posterior cord, 96 Retinal arteriole bifurcation, 364f, 370
316, 318 Posterior fossa, 118 Retinal nerve fibers, 81
Paraneoplastic encephalomyelitis (PEM), 316317 Posterior interosseous nerve, 28 Retrograde transport, 193, 194
Paraneoplasticlimbic encephalitis, 317318 Posterolateral choroidal arteries, 37 Rett syndrome, 125, 142
Paraneoplastic opsoclonusmyoclonus syndrome, Postganglionic Horner syndrome, 68 Reye syndrome, 136
319 Postganglionic sympathetic neurons, 338 Rhomboid muscles rotation, 24
Paraneoplastic sensory neuronopathy (PSN), 382 Postinfectious encephalomyelitis, 321322 Right cerebellar syndrome, 90
Paraneoplastic stiff-man syndrome, 321 Posttraumatic stress disorder (PTSD), 459 Right pronator quadratus, 29
Parasympathetic nerves, 38 Potent antiretroviral therapy, 296 Right pyramidal system, 90
Parasympathetic neural transmission, 71 PraderWilli phenotype, 137 Risperidone, 462
Parasympathetic preganglionic neurons, 41 PraderWilli syndromes, 125 Rituximab, 320
Parasympathetic systems, 27 Pramipexole, 396 Robertson pupil, 83
Parinaud syndrome, 67, 72, 136 Preganglionic parasympathetic fibers, 33, 39 Rolandic epilepsy, 188
PARKIN gene, 393394 Pregnancy Romberg sign, 85
Parkinson disease (PD), 356, 392 lithium level, 465 Rosenthal fibers, 406f, 419
cardinal signs of, 395 obturator nerve, 75 Rostral basal pontine syndrome, 49f, 71
definite risk factor for, 394395 Presynaptic disorder, 237 Rostral interstitial nucleus, 25
depression, 396 Presynaptic neuron, 336 Ryanodine receptor mutation, 426
Parkinsonism, recessive forms, 393 Primary central nervous system lymphoma
Parkinsonism linked to chromosome 17 (FTLD-17), (PCNSL), 381 S
251 Primary lateral sclerosis (PLS), 218 Saccades, 366
Parkinson syndrome, 397 Primary neurulation, 37 Sacral plexus, 9596
ParksBielschowsky test, 83 Progressive ataxia, 147 Sail waves, 179
Paroxysmal attacks, 193 Progressive calcification, 148 Sarcoidosis, 226
Paroxysmal torticollis, 135 Progressive cerebellar ataxia, 147 Sartorius muscle, 25
PCD. See Paraneoplastic cerebellar degeneration Progressive cerebellar syndrome, 234, 297 Schizophrenia varies, 458459
(PCD) Progressive disorder, 133 Schwann cells, 378
474 Index

Schwannomas, 407f, 420 Superior gluteal nerve, 28, 95 U


Schawannomin, 368 Superior gluteal vessels, 95 Uhthoffs phenomenon, 314
SchwartzJampel syndrome, 146 Superoxide dismutase 1 (SOD1) gene, 348 Ulnar nerve, 81
Sciatic nerve, 2425, 75, 76 Suppression bursts, 127 Unconjugated free bilirubin, 117
SD. See Semantic dementia Supranuclear palsy, 259 Underlying cerebral peduncle, 50f, 72
Secondary progressive MS (SP-MS), 311 Supranuclear vertical gaze palsy, 396 Unimodal afferents, 23
Segmental demyelination, 218 Suprascapular nerve, 23 UnverrichtLundborg disease, 189
Seidel scotoma, 81 Suprasellar extension, 444 Upbeat nystagmus, 366
Seizure disorder, 129 Sydenham chorea, 117, 130, 132, 148 Upper midbrain, 93
Semantic dementia (SD), 251, 260 Sylvian aqueduct syndrome, 91 Upper motor neuron syndrome, 308
Sensory aprosodia, 92 Sylvian fissure, 42 Urea cycle disturbance, 118
Sensory axons, 35 Sympathetic system starts, 67 Urinary incontinence, 132
Sensory symptoms, 79 Synchronized sleep, 181
Septum pellucidum, 91 Syringomyelia, 85 V
Septum verum, 9192 Vacuolar myelopathy, 424
Serotonin reuptake, 460 T Vagus nerves, 26, 37
Serratus anterior muscle, 78 Tau proteins, 258 Valproate, 128
Sialidosis, 190 TaySachs disease, 138, 143 Valproic acid, 183, 185, 188, 334, 398
Simple visual hallucinations, 93 Teased nerve fibers, 201f, 218 Valsalva, 120
Single myelinated fiber, 196 Tectal midbrain lesions, 71 Vascular dementia, 256
Single photon emission computed tomography Telencephalon, 36 Vasculitic neuropathy, 215
(SPECT), 444, 459 Temporal arteritis, 424 VEE. See Venezuelan equine encephalitis (VEE)
Sjgren syndrome, 239 Temporal lobe epilepsy, 189 Venezuelan equine encephalitis (VEE), 297
Sleep Temporal lobe lesions, 33 Venlafaxine, 463
Benign epileptiform transients of, 181 Temporal transients, 179 Venom poisoning, 238
Positive occipital sharp transients of, 181 Tentorium cerebelli, 20f, 42 Ventral midbrain syndrome, 277
spindles, 178 Terson syndrome, 274 Ventral pontine syndrome, 276
stages, 181 Tethered cord syndrome, 36 Ventral posterolateral nucleus, 38
Slow-channel congenital myasthenic syndrome, 350 Thalamic neurons project, 29 Ventral posteromedian nuclei, 38
Slow-channel syndrome, 238239 Thalamocortical projections, 29 Ventral tegmental area (VTA), 457
Small cell lung cancer (SCLC), 218, 223, 314 Thalamomesencephalic arteries, 37 Ventricular system, 36
paraneoplastic encephalomyelitis (PEM), 314315 Thalamus, 36 Ventrolateral pons, 88
subacute sensory neuronopathy (SSN), 315 dorsomedial nucleus of, 39 Vertebrobasilar migraine, 121
SNCA (-synuclein) gene, 356357 preoptic nucleus of, 38 Vertical saccades, 25
Sodium oxybate, 123 Th2 cytokine production, 318 Vestibular nystagmus, 366
Somatic mitochondrial mutations, 392 Theophylline, 186 Vignette patient, 29
Somatization disorder, 460 Thermoanesthesia, 86 Viral meningitis, 299
Source amnesia, 262 Thoracic nerve, 23 Visual allesthesia, 69
Spina bifida aperta, 36 Thoracic outlet syndrome, 96 Voltage-gated calcium channels (VGCCs), 217
Spinal accessory nerve, 37 Thoracodorsal nerve lesion, 221 Voltage-gated potassium channel antibody
Spinal bulbar muscular atrophy, 354 Thoracolumbar paraspinal muscles, 146 (VGKC), 315
Spinal cord, 16f, 40 Thrombolytic therapy, 272 Von HippelLindau disease, 368
cytoarchitectonic map of, 29 Thymectomy, 240 VTA. See Ventral tegmental area (VTA)
Spinal muscular atrophy, 143, 219 Tizanidine, 313 VZV, 297
Spinal nucleus, 28 TolosaHunt syndrome, 78
Spinocerebellar ataxia (SCA), 353, 358 Tonicclonic seizures, 313 W
Spinothalamic fibers, 38 Tourette syndrome, 131132, 398, 460461 WalkerWarburg syndrome, 120
Spinothalamic tract, 84 Toxic optic neuropathy, 77 Wallenberg syndrome, 90, 276
Square-wave jerks, 370 Toxoplasmosis, 292, 444 WARSS, 279
SSPE. See Subacute sclerosing panencephalitis Transcortical sensory aphasia, 6869, 94 Weber syndrome, 72, 90
(SSPE) Transient ischemic attacks (TIAs), 355 WerdnigHoffman disease, 133
SSRIs. See Selective serotonin reuptake inhibitors Transient paresis, 134 Wernicke aphasia, 68
(SSRIs) Transient peduncular hallucinations, 72 Wernicke disease, 140
Staphylococcus aureus, 294, 295 Transthyretin stains choroid plexus tumors, 423 Wernicke encephalopathy, 425
StevensJohnson syndrome (SJS), 334, 335, 356 Transverse myelitis, 322 WernickeKorsakoff syndrome, 91
Stiff-man syndrome, 316, 317 Transverse temporal gyrus of Heschl, 37 Wernicke pupil, 83
Stiffperson syndrome, 146 Trapezius muscle weakness, 79 West syndrome, 122, 126, 187188
Streptococcus pneumoniae, 294 Trazodone, 464 WHO grade II astrocytoma, 380
Striatum, 29 Triangle of Guillain, 367 WHO grade III astrocytoma, 380
SturgeWeber syndrome, 351, 368 Trigeminal nerve, 5f, 27, 28 Wicket spikes, 181
Subacute necrotizing encephalopathy, 233 Trigeminal primary afferent neurons, 38 Wilson disease, 192, 392
Subacute sclerosing panencephalitis (SSPE), 293 Trigeminal semilunar (gasserian), 38 WyburnMason syndrome, 368
Subarachnoid hemorrhage, 127, 278 Trimipramine, 464
Subclavian muscle, 24 Triphasic waves, 179 X
Subdural hemorrhage, 127 Trochlear nerve, 5f, 20f, 2526, 27, 42 X-linked adrenoleukodystrophy (X-ALD), 351
Subhyaloid hemorrhage, 274 Trochlear nucleus, 26 X-linked disorders, 354
Substantia nigra, 394, 395 TTHs. See Tension-type headaches (TTHs) X-linked doublecortin (DCX), gene 119120
Sudden bilateral paramedian thalamic lesions, 70 Tuberculosis meningitis, 299
Superior cerebellar arteries, 34, 91 Tuberoinfundibular tract, 31 Z
Superior cerebellar peduncle, 30 Tuberous sclerosis complex (TSC), 125, 350, 368, 379 Zellweger syndrome, 143
Superior colliculus, 5f, 27 Typical aura, 187 Zonisamide, 183, 185
A B
Figure 5-4: Chapter 5, question 15

Figure 5-6: Chapter 5, question 28 Figure 5-9: Chapter 5, question 31

A B
Figure 5-8: Chapter 5, question 30
Figure 12-1: Chapter 12, question 33 Figure 12-2: Chapter 12, question 34

Figure 15-1: Chapter 15, question 1 Figure 15-2: Chapter 15, question 2

A B
Figure 15-3: Chapter 15, question 3
A B
Figure 15-4: Chapter 15, question 4

Figure 15-5: Chapter 15, question 5 Figure 15-9: Chapter 15, question 9

A B
Figure 15-6: Chapter 15, question 6
Figure 15-10: Chapter 15, question 10 Figure 15-11: Chapter 15, question 11

A B
Figure 15-12: Chapter 15, question 12

Figure 15-13: Chapter 15, question 13 Figure 15-14: Chapter 15, question 14
Figure 15-15: Chapter 15, question 15 Figure 15-16: Chapter 15, question 16

Figure 15-17: Chapter 15, question 17 Figure 15-20: Chapter 15, question 20

Figure 15-21: Chapter 15, question 21 Figure 15-22: Chapter 15, question 22
Figure 15-23: Chapter 15, question 23 Figure 15-25: Chapter 15, question 49

Figure 15-28: Chapter 15, question 52 Figure 15-29: Chapter 15, question 53

Figure 15-30: Chapter 15, question 54 Figure 15-31: Chapter 15, question 55
Figure 15-32: Chapter 15, question 56 Figure 15-33: Chapter 15, question 57

Figure 15-38: Chapter 15, question 62

Figure 15-37: Chapter 15, question 61

Figure 15-40: Chapter 15, question 64

Figure 15-39: Chapter 15, question 63

Figure 15-42: Chapter 15, question 66


Figure 15-43: Chapter 15, question 67 Figure 15-44: Chapter 15, question 68

Figure 15-45: Chapter 15, question 69 Figure 15-46: Chapter 15, question 70

Figure 15-47: Chapter 15, question 71 Figure 15-48: Chapter 15, question 72

Potrebbero piacerti anche